СОДЕРЖАНИЕ
Предисловие
Часть I. Элементарные функции
1.2 Элементарные функции: графики и преобразования
1.3 Линейные функции и прямые линии
1.4 Квадратичные функции
Глава 2. Другие элементарные функции
2.2 Показательная функция
2.3 Логарифмическая функция
Часть II. Конечная математика
3.2 Сложные проценты
3.3 Будущая стоимость аннуитета: амортизационные фонды
3.4 Текущая стоимость аннуитета: погашение долга
Глава 4. Системы линейных уравнений и матрицы
4.2 Системы линейных уравнений и расширенные матрицы
4.3 Метод исключения Гаусса-Жордана
4.4 Матрицы: основные операции
4.5 Нахождение обратной квадратной матрицы
4.6 Матричные уравнения и системы линейных уравнений
4.7 Анализ межотраслевых связей по Леонтьеву
Глава 5. Линейные неравенства и линейное программирование
5.2 Линейное программирование в двух измерениях: геометрический подход
5.3 Геометрическое введение в симплекс-метод
5.4 Симплекс-метод: максимизация с ограничениями вида “меньше или равно”
5.5 Двойственная задача: минимизация с ограничениями вида “больше или равно”
5.6 Задачи максимизации и минимизации со смешанными ограничениями
Глава 6. Теория вероятностей
6.2 Перестановки и сочетания
6.3 Пространство элементарных исходов, события и вероятность
6.4 Объединение, пересечение и дополнение событий; шансы
6.5 Условная вероятность, пересечение и независимость событий
6.6 Формула Байеса
6.7 Случайная величина, распределение вероятностей и математическое ожидание
Глава 7. Цепи Маркова
7.2 Регулярные цепи Маркова
7.3 Поглощающие цепи Маркова
Ответы к упражнениям
Предметный указатель
Текст
                    ПРИКЛАДНАЯ МАТЕМАТИКА
Восьмое издание
Том 1
Основы и линейная алгебра



APPLIED MATHEMATICS for business, economics, life sciences, and social sciences Eight Edition Raymond A. Barnett Merritte College Michael R. Ziegler Marquette University Karl E. Byleen Marquette University Prentice Hall Prentice Hall Pearson Education, Inc. Upper Saddle River, NJ 07458
ПРИКЛАДНАЯ МАТЕМАТИКА Восьмое издание Том 1 Основы и линейная алгебра Реймонд А. Барнетт Колледж Мерритта Майкл К. Циглер Колледж Маркетта КарлЭ. Байлин Колледж Маркетта В|лцАлаапикА Москва • Санкт-Петербург 2020
ББК 22.143 Б25 УДК 510.2 ООО “Диалектика” Зав. редакцией С. Н. Тригуб Перевод с английского докт. физ.-мат. наук Д. А. Илюшина, В. В. Марченко, А. В. Романенко, О. М. Ядренко Под редакцией докт. физ.-мат. наук Д. А. Илюшина По общим вопросам обращайтесь в издательство “Диалектика” по адресу: info@dialektika.com, http://www.dialektika.com Барнетт, Реймонд А., Циглер, Майкл P., Байлин, Карл Э. Б25 Прикладная математика, 8-е изд. Том 1. Основы и линейная алгебра.: Пер. с англ. — СПб. : ООО “Диалектика”, 2020 — 864 с. : ил. — Парал. тит. англ. ISBN 978-5-907114-36-4 (рус., том 1) ISBN 978-5-907114-35-7 (рус., многотом.) ББК 22.143 Все названия программных продуктов являются зарегистрированными торговыми марками соответствующих фирм. Никакая часть настоящего издания ни в каких целях не может быть воспроизведена в какой бы то ни было форме и какими бы то ни было средствами, будь то электронные или механические, включая фотокопирование и запись на магнитный носитель, если на это нет письменного разрешения издательства Prentice Hall, Inc. Copyright © 2020 by Dialektika Computer Publishing. Authorized translation from the English language edition published by Prentice Hall, Copyright © 2003, 2000, 1997, 1994, 1991, 1988, 1985, 1983 by Pearson Education, Inc., Upper Saddle River, New Jersey 07458. All rights reserved. No part of this book may be reproduced or transmitted in any form or by any means, electronic or mechanical, including photocopying, recording or by any information storage retrieval system, without permission from the Publisher. Научно-популярное издание Реймонд А. Барнетт, Майкл Р. Циглер, Карл Э. Байлин Прикладная математика, 8-е издание, том 1. Основы и линейная алгебра ООО "Диалектика", 195027, Санкт-Петербург, Магнитогорская ул., д. 30, лит. А, пом. 848 ISBN 978-5-907114-36-4 (рус., том 1) ISBN 978-5-907114-35-7 (рус., многотом) ISBN 0-13-065580-5 (англ.) © ООО “Диалектика”, 2019 © 2003 by Pearson Education, Inc.
Оглавление Предисловие 8 Часть I. Элементарные функции 16 Глава 1. Базовые элементарные функции 17 Глава 2. Другие элементарные функции 130 Часть II. Конечная математика 212 Глава 3. Финансовая математика 213 Глава 4. Системы линейных уравнений и матрицы 282 Глава 5. Линейные неравенства и линейное программирование 401 Глава 6. Теория вероятностей 536 Глава 7. Цепи Маркова 669 Ответы к упражнениям 736 Предметный указатель 848
Содержание Предисловие 8 Часть I. Элементарные функции 16 Глава 1. Базовые элементарные функции 17 1.1 Функции 18 1.2 Элементарные функции: графики и преобразования 44 1.3 Линейные функции и прямые линии 62 1.4 Квадратичные функции 90 Глава 2. Другие элементарные функции 130 2.1 Полиномы и рациональные функции 131 2.2 Показательная функция 157 2.3 Логарифмическая функция 181 Часть П. Конечная математика 212 Глава 3. Финансовая математика 213 3.1 Простые проценты 214 3.2 Сложные проценты 224 3.3 Будущая стоимость аннуитета: амортизационные фонды 244 3.4 Текущая стоимость аннуитета: погашение долга 255
Содержание 7 Глава 4. Системы линейных уравнений и матрицы 282 4.1 Системы линейных уравнений с двумя переменными 283 4.2 Системы линейных уравнений и расширенные матрицы 301 4.3 Метод исключения Гаусса-Жордана 314 4.4 Матрицы: основные операции 333 4.5 Нахождение обратной квадратной матрицы 353 4.6 Матричные уравнения и системы линейных уравнений 367 4.7 Анализ межотраслевых связей по Леонтьеву 380 Глава 5. Линейные неравенства и линейное программирование 401 5.1 Системы линейных неравенств с двумя переменными 402 5.2 Линейное программирование в двух измерениях: геометрический подход 420 5.3 Геометрическое введение в симплекс-метод 442 5.4 Симплекс-метод: максимизация с ограничениями вида “меньше или равно” 451 5.5 Двойственная задача: минимизация с ограничениями вида “больше или равно” 476 5.6 Задачи максимизации и минимизации со смешанными ограничениями 499 Глава 6. Теория вероятностей 536 6.1 Основные принципы счета 537 6.2 Перестановки и сочетания 551 6.3 Пространство элементарных исходов, события и вероятность 567 6.4 Объединение, пересечение и дополнение событий; шансы 586 6.5 Условная вероятность, пересечение и независимость событий 607 6.6 Формула Байеса 628 6.7 Случайная величина, распределение вероятностей и математическое ожидание 639 Глава 7. Цепи Маркова 669 7.1 Свойства цепей Маркова 670 7.2 Регулярные цепи Маркова 688 7.3 Поглощающие цепи Маркова 705 Ответы к упражнениям 736 Предметный указатель 848
Предисловие В восьмом издании книги Прикладная математика изложен курс дискретной математики и математического анализа, рассчитанный на студентов, прослушавших курс алгебры в высшей школе. Полнота и относительная независимость рассматриваемый в книге тем позволяют легко адаптировать ее для любого курса. В данном издании учтены замечания и пожелания многочисленных читателей предыдущих выпусков, преподавателей и студентов высших учебных заведений. Главным достоинством книги является ее ориентация на активную классную работу и обязательную обратную связь преподавателей со студентами. Стиль изложения и целевая аудитория Текст написан на доступном для студентов уровне, при этом большое внимание уделялось математической строгости и точности изложения. Основной акцент сделан на методах решения задач, а не на математической теории. Авторы избегали пространных доказательств, за исключением случаев, когда это было необходимо для объяснения математической идеи. Как правило, общие результаты излагаются после обсуждения конкретных примеров. Примеры и упражнения Книга содержит более 470 полностью решенных примеров, иллюстрирующих математические понятия и демонстрирующие методы решения практических задач. Многие примеры состоят из нескольких частей. Каждый пример сопровождается аналогичным упражнением, решение которого позволяет закрепить пройденный материал. Ответы к упражнениям приведены в конце книги.
Предисловие 9 Задания Каждый раздел книги включает задания, иллюстрирующие применимость изученных концепций в реальной жизни. Они призваны развить у студентов навыки глубокого анализа и словесного описания изучаемых вопросов. Каждая глава содержит также два домашних задания, выполнение которых требует знаний сразу нескольких математических концепций и понятий, описанных в главе. Практикум Книга содержит более 6300 задач, ориентированных на студентов, имеющих разный уровень подготовки. Задачи разделены на три категории: А (простые, стандартные), Б (более сложные) и В (повышенной сложности). Приложения Основная цель книги — выработать у студентов навыки моделирования и решения реальных задач. Большое количество приложений призвано убедить даже скептически настроенного студента в практической ценности математики. Описание реальных ситуаций сопровождаются пиктограммой gW. Почти все упражнения заключаются в решении действительных экономических, биологических или социальных задач. В приложениях, приведенных в конце разделов книги, рассматриваются упрощенные версии действительных задач, описанных в научных журналах и книгах. Заметьте, что для их решения вам не нужно обладать глубокими познаниями указанных дисциплин. Для получения решения достаточно навыков, получаемых в процессе изучения материала главы. Поддержка книги в Интернете В Интернете можно найти огромное количество ресурсов, содержащих вспомогательные материалы к рассматриваемым в книге вопросам. К ним относятся как находящиеся в свободном доступе источники данных, так и сборники интерактивных задач, призванных повысить уровень математической подготовки студентов. Каждый раздел книги содержит пиктограмму www, отсылающую читателя к определенному интернет-ресурсу. Ссылки на соответствующие сайты размещены на странице http: //wps .prenhall. com/esm_barnett_appliedmath_8/. Программное обеспечение Под графической утилитой понимается любой из существующих графических калькуляторов или программ визуализации данных, снабженных функциями построения графиков функций и нахождения их математических характеристик. Несмотря на необязательность использования, они заметно упростят вам жизнь, автоматически выполняя сложные математические расчеты. Для тех из вас, кто
10 Предисловие планирует освоить такое программное обеспечение, в книге сформулированы отдельные примеры, упражнения и домашние задания. Уже начиная с главы 1, мы рекомендуем применять специальные графические утилиты для математического моделирования регрессионных зависимостей, основанных на реальных данных. В тексте книги решения математических задач, основанные на использовании таких программ, обозначаются пиктограммой или % Графики Все графики, приведенные в книге, построены с помощью компьютерных программ. Иллюстрации, изображающие окна графических утилит, являются реальными снимками экрана. Специальные обозначения Критические замечания в решении многих примеров и задач сопровождаются комментариями, набранными специальным шрифтом (см., например, разделы 1.1 и 4.2). Блоки вычислений, которые следует выполнять мысленно, выделены пунктиром (см., например, разделы 1.1 и 1.4). Важные определения, утверждения и процедуры помещены во врезках (см., например, разделы 4.3 и 4.5). Новые понятия и важные пояснения выделены полужирным шрифтом. В заключении каждой главы перечислены основные понятия и наиболее важные обозначения. В конце книги приведены ответы на большинство задач. Они сопровождаются номерами разделов, в которых обсуждались соответствующие темы. В конце книги приведены также ответы ко всем задачам с нечетными номерами. Ответы на задачи из области линейного программирования состоят из математической модели и числового ответа. Структура книги Книга состоит из двух томов. В первом изложены элементарные функции и основные концепции дискретной математики. В нем вы найдете перекрестные ссылки на материал приложений А и Б, которые включены во второй том издания. В главах 1 и 2 описаны элементарные функции, перечислены их свойства и способы применения. Мы рекомендуем студентам внимательно изучить математические идеи, лежащие в основе этих функций, проиллюстрировав их графически, численно и алгебраически. Эти сведения понадобятся не только при изучении книги, но и при освоении других курсов. Преподаватели могут самостоятельно распределять материал, изложенный в первых двух главах по разным курсам, ориентируясь на уровень подготовки своих студентов.
Предисловие 11 Материал, составляющий вторую часть книги (дискретная математика), можно разделить на четыре категории: финансовая математика (глава 3), линейная алгебра, включая матричные вычисления, линейные системы и линейное программирование (главы 4 и 5), теория вероятностей (глава 6), а также применение линейной алгебры и теории вероятностей для исследования цепей Маркова (глава 7). Первые три темы рассматриваются независимо от остальных, а в изложении главы 7 используются понятия и концепции, описанные в главах 4 и 6. В главе 3 описаны правила начисления простых и сложных процентов, а также подсчета текущей и будущей стоимости обычного аннуитета. Сведения об арифметических и геометрических прогрессиях, применяемых при решении таких задач, рассмотрены в приложении Б. В главе 4 приведены основные сведения о линейных системах и матрицах в контексте рассмотрения операций над строками и метода исключения переменных Гаусса-Жордана, применяемого при решении систем линейных алгебраических уравнений и вычислении обратных матриц. В главе также перечислены многочисленные примеры использования систем линейных уравнений и матриц при решении практических задач. Чтобы облегчить понимание принципов решения подобных задач, в конце книги приведены ответы, содержащие не только готовые решения, но и описание математических моделей, на которых они основаны. Операции над строками обсуждаются в разделах 4.2 и 4.3. Впоследствии они используются при описании симплекс-метода, рассматриваемого в главе 5. Знание принципов умножения матриц, нахождения обратных матриц и решения систем линейных уравнений пригодится при детальном изучении цепей Маркова, изложенных в главе 7. В главе 5 содержится подробное описание методов линейного программирования. Первые два раздела этой главы посвящены графическим методам решения задач линейного программирования с двумя переменными. Преподаватели, делающие упор на методологии обучения могут сначала изложить материал разделов 5.3 и 5.4, посвященных симплекс-методу, и только затем перейти к рассмотрению либо двойственных задач линейного программирования (раздел 5.5) и 7И-методу их решения (раздел 5.6), либо двухэтапного симплекс-метода (домашнее задание 5.1). Преподаватели, отдающие предпочтение построению математических моделей, могут сразу акцентировать внимание на примерах, изложенных в разделах 5.4-5.6. Описание пакетов прикладных программ, предназначенных для решения этих задач, является факультативным. Чтобы облегчить работу студентам, все ответы к упражнениям, приведенным в разделах 5.4-5.6, включают описание математических моделей, на которых они построены. Глава 6 рассматривает основные положения классической теории вероятностей, включая формулу Байеса и определение случайных величин. Глава 7 оперирует понятиями, введенными ранее, и способами их применения при построении и анализе цепей Маркова. Эта глава как нельзя нагляднее
12 Предисловие иллюстрирует высокую эффективность самых разных концепций дискретной математики при решении практических задач. Второй том книги начинается с третьей части и полностью посвящен математическому анализу. Материал, изложенный во втором томе в третьей части книги (математический анализ), посвящен дифференциальному исчислению (главы 8-10), интегральному исчислению (главы 11 и 12) и некоторым другим темам математического анализа (глава 13-15). Главы 8-11 следует изучать полностью и в строго заданной последовательности. Остальные главы можно рассматривать в произвольном порядке, поскольку они мало связаны друг с другом. В главе 8 определение производной. В первых двух разделах вы познакомитесь с понятиями предела, непрерывности и их свойствами, изучение которых является первым шагом на пути к овладению концепцией производной (раздел 8.3). В остальных разделах этой главы изложены правила дифференцирования (включая правило дифференцирования степенных функций), а также приведены примеры применения производной в приложениях бизнес-анализа. Предложенные в этой главе решения предполагают комбинирование числовых, алгебраических и графических методов математического анализа. Глава 9 посвящена методам графического решения задач и оптимизации данных. В первых двух разделах вы узнаете о геометрическом смысле первой и второй производных, а также познакомитесь со способами построения графиков полиномов. В разделе 9.3 изложены основные принципы построения графиков рациональной функции. Если эта тема в вашем курсе не рассматривается, то можете легко исключить данный раздел из учебного плана. Методы оптимизации изложены в разделе 9.4. В нем вы найдете также примеры решения задач, требующих анализа конечных результатов. Первые три раздела главы 10 расширяют понятие производной, рассмотренное в главах 8 и 9, на показательные и логарифмические функции, описывая правила вычисления сложных функций такого вида. Приведенные в них сведения используются при рассмотрении концепций, изложенных в последующих главах книги. В разделе 10.4 описаны принципы неявного дифференцирования, а в разделе 10.5 рассматривается важная экономическая задача вычисления связанных скоростей роста. Последние два раздела не обязательны для изучения и могут быть пропущены без ущерба для понимания материала следующих глав. В главе 11 вы познакомитесь с операцией интегрирования. Первые два раздела этой главы посвящены вычислению первообразной и необычайно важны для понимания дальнейшего материала. В разделе 11.3 приведены примеры решения практических задач и использованием дифференциальных уравнений. Приведенные в нем сведения не обязательны для изучения и его можно смело пропустить. В разделах 11.4 и 11.5 вводится понятие определенного интеграла, описываются методы их приближенного вычисления и оценки погреш-
Предисловие 13 ностей. Как и ранее, изложение нового материала сопровождается описанием графических, вычислительных и алгебраических методов решения практических задач. Эти два раздела также важны для всей остальной части книги. В главе 12 изучены дополнительные задачи, решаемые методами интегрального исчисления. В разделе 12.1 вы узнаете о способах вычисления площадей фигур (глава 11), ограниченных двумя кривыми. В разделе 12.2 описаны примеры решения других практических задач интегрального исчисления. Разделы 12.3 и 12.4 посвящены некоторым специальным способам вычисления интегралов. Все они являются необязательными для изучения и могут быть пропущены. Первые пять разделов главы 13 содержат описание свойств и области применения функций нескольких переменных. К изучению этих разделов можно приступать сразу после знакомства с разделом 10.3. Для понимания концепций, приведенных в разделе 13.6, достаточно быть знакомым с понятием интеграла, введенным в главе 11. В разделе 14.1 изложены основные принципы решения дифференциальных уравнений. В последующих двух разделах описан метод разделения переменных, рассмотрены способы решения дифференциальных уравнений первого порядка, а также приведены примеры их практического применения. После описания несобственных интегралов в разделе 15.1 основное внимание остальных разделов главы 15 уделено понятию плотности вероятности, а также функциям равномерного, экспоненциального и нормального распределений вероятностей. В приложении А содержатся задачи для самопроверки и приведен краткий обзор основных алгебраических методов, широко применяемых при решении практических задач. Приложение Б включает полезные сведения из некоторых других разделов математики, которые помогут в полной мере освоить весь приведенный в книге материал. Благодарности Мы хотели бы поблагодарить многих людей, причастных к изданию книги. В частности, мы признательны нашим коллегам, сделавшим ценные замечания по поводу предыдущих изданий: Крису Болдту (Chris Boldt), Бобу Брадшо (Bob Bradshow), Селесте Чаффи (Celeste Chaffee), Роберту Чейни (Robert Chaney), Диане Кларк (Dianne Clark), Чарльзу Э. Клеверу (Charles Е. Cleaver), Барбаре Коэн (Barbara Cohen), Ричарду Л. Конлону (Richard L. Conlon), Кэтрин Крон (Catherine Cron), Лоу Д’Алотто (Lou D’Alotto), Мадху Дешпанде (Madhu Deshpande), Джону Дикерсону (Jonh Dickerson), Кеннету А. Дорадо (Kenneth A. Dorado), Майклу У. Экеру (Michael W. Ecker), Джерри Р. Эхману (Jerry R. Ehman), Люсине Галлахер (Lucina Gallagher), Жоэлю Хааку (Joel Haack), Марте М. Харвей (Martha М. Harvey), Сью Хендерсон (Sue Hen-
14 Предисловие derson), Ллойду Р. Хиксу (Lloyd R. Hicks), Льюису Ф. Хетцле (Louis F. Hoetzle), Паулю Хатчинсу (Paul Hutchins), К. Уэйну Джеймсу (К. Wayne Janies), Роберту X. Джонстону (Robert Н. Johnston), Роберту Кристоку (Robert Krystock), Инессе Леви (Inessa Levi), Джеймсу Т. Лоатсу (James Т. Loats), Фрэнку Лопесу (Frank Lopez), Уэйну Миллеру (Wayne Miller), Мел Митчелл (Mel Mitchell), Майклу Монтано (Michael Montano), Рональду Перски (Ronald Persky), Шале Петерман (Shala Peterman), Кеннету А. Петерсу-мл. (Kenneth A. Peters, Jr.), Диксу Петти (Dix Petty), Тому Плавчаку (Tom Plavchak), Бобу Приелиппу (Bob Prielipp), Стивену Роди (Stephen Rodi), Артуру Розенталю (Arthur Rosental), Шелдону Ротману (Sheldon Rothman), Элен Рассел (Elaine Russel), Джону Райану (John Rayan), Дэниэлю Э. Сканлону (Daniel Е. Scanlon), Джорджу Р. Шриро (George R. Schriro), Арнольду Л. Шредеру (Arnold L. Schroeder), Хари Шанкеру (Hari Shanker), Ларри Смоллу (Larri Small), Джоан Смит (Joan Smith), Стивену Терри (Steven Terry), Делорес А. Уильямс (Delores A. Williams), Каролин Вудс (Caroline Woods), Чарльзу У. Циммерману (Charles W. Zimmerman) и Пэту Жролке (Pat Zrolka). Мы также выражаем благодарность Хуссейну Хамедани (Hossein Hamedani), Каролин Мейтлер (Caroline Meitler), Грегу Росику (Greg Rosik) и Каролин Вудс за тщательную и строгую проверку всех математических вычислений, выполненных в книге, Джоанн Уоллес (Jeanne Wallace) за аккуратную и эффективную подготовку рукописи, Джорджу Моррису (George Morris) и штату сотрудников его компании Scientific Illustrator за превосходную подготовку рисунков и графиков. Мы признательны всем сотрудникам издательства Prentice Hall, внесшим свой вклад в подготовку книги, особенно нашим редакторам: Куинси Мак-Дональд (Quincy McDonald) и Линн Савино Уэндел (Lynn Savino Wendel). Сотрудничество с этими чрезвычайно компетентными профессионалами доставило нам истинное удовольствие. Р. А. Барнетт М. Р. Циглер К. Е. Байлин
Часть I Элементарные функции 30
1 Базовые элементарные функции ■ 1.1. Функции ■ 1.2. Элементарные функции: графики и преобразования ■ 1.3. Линейные функции и прямые линии ■ 1.4. Квадратичные функции ■ Ключевые слова, основные обозначения и формулы ■ Упражнения для повторения ■ Домашнее задание 1.1. Введение в регрессионный анализ ■ Домашнее задание 1.2. Математическое моделирование в бизнес-анализе Введение Функция является одним из наиболее важных понятий в математике. Для успешного изучения математики требуется твердое усвоение базового набора элементарных функций, их свойств и графиков. В приложении Г приведен список формул, который образует своего рода “библиотеку элементарных функций”. Большая их часть будет представлена в конце главы 2. Кроме того, они окажутся полезными при изучении новых курсов или при решении задач, где требуется применение математики. В этих дисциплинах иногда используются и другие элементарные функции, однако приведенных в списке более чем достаточно для решения задач, перечисленных в этой книге.
18 Часть I. Элементарные функции 1.1. Функции ■ Декартова система координат ■ Построение графиков по точкам ■ Определение функции ■ Функции, определяемые уравнениями ■ Обозначения функций ■ Решение практических задач После краткого обзора свойств декартовой (прямоугольной) системы координат на плоскости и построения графиков по точкам в разделе обсуждается понятие функции, одно из важнейших в математике. Декартова система координат Декартова, или прямоугольная, система координат образуется двумя действительными числовыми прямыми — горизонтальной и вертикальной, пересекающимися, как показано на рис. 1.1. Обычно их направляют вверх и вправо. Эти числовые прямые называются горизонтальной и вертикальной осью соответственно, а вместе — координатными осями. Горизонтальная ось часто называется осью х, а вертикальная — осью у. В некоторых случаях используются и другие обозначения. (Иногда горизонтальную ось называют осью абсцисс, а вертикальную — осью ординат. — Прим, ред.} Координатные оси разбивают плоскость на четыре части, которые называются квадрантами и нумеруются римскими цифрами от I до IV против часовой стрелки (см. рис. 1.1). Рис. 1.1. Декартова (прямоугольная) система координат
Глава 1. Базовые элементарные функции 19 Сопоставим каждой точке плоскости ее координаты. Выберем на плоскости произвольную точку Р и проведем через нее горизонтальную и вертикальную прямые (см. рис. 1.1). Вертикальная прямая пересекает горизонтальную ось в точке с координатой а, а горизонтальная прямая пересекает вертикальную ось в точке с координатой Ь. Эти два числа записываются как упорядоченная пара (а, Ь) и называются координатами точки Р. Первая координата, а, называется абсциссой точки Р, а вторая координата, Ь, — ординатой точки Р. Так, на рис. 1.1 точка Q имеет абсциссу —5 и ординату 5. О координатах точки можно также говорить, используя названия осей. Например, на рис. 1.1 координата х точки R равна 10, а координата у равна -10. Точка с координатами (0, 0) называется началом координат. Процедура, описанная выше, позволяет однозначно сопоставить любой точке Р, лежащей на плоскости, единственную пару действительных чисел (а, Ь). И наоборот, по заданной упорядоченной паре действительных чисел можно определить единственную точку плоскости Р. Таким образом, справедливо следующее утверждение. Между точками плоскости и элементами множества упорядоченных пар действительных чисел существует однозначное соответствие. Это утверждение часто называется основной теоремой аналитической геометрии. Построение графиков по точкам Основная теорема аналитической геометрии позволяет описать каждое алгебраическое уравнение с помощью графика и, наоборот, каждому графику сопоставить некоторое уравнение. Начнем с рассмотрения алгебраического уравнения для двух переменных: у = 9 —х2. (1.1) Решением уравнения (1.1) является упорядоченная пара действительных чисел (а, Ь) такая, что: Ь = 9 - а2. Множество решений уравнения (1.1) состоит из набора таких упорядоченных пар. Для поиска решения уравнения (1.1) заменим х некоторым числом и найдем значение у. Например, если х = 2, то у = 9—22 = 5, т.е. упорядоченная пара (2, 5) будет решением уравнения (1.1). Аналогично, если х = —3, то у = 9 — (—З)2, и (—3, 0) — также решение. Поскольку все решения уравнения (1.1) получаются подстановкой произвольного действительного числа вместо х, то множество решений уравнения (1.1) содержит бесконечно большое количество элементов.
20 Часть I. Элементарные функции В прямоугольной системе координат мы можем изобразить это множество геометрически. Графиком уравнения называется кривая, на которой представлены все упорядоченные пары, составляющие множество решений. Для построения графика уравнения нужно построить достаточное количество точек (такое, чтобы вся кривая была видна в общих чертах) и соединить их гладкой линией. Эта процедура называется построением по точкам. Пример 1.1 (Построение графика по точкам). Постройте график уравнения у = 9 — х2. Решение. Построим таблицу решений, т.е. набор упорядоченных пар действительных чисел, которые удовлетворяют заданному уравнению. Для простоты вычислений возьмем целые значения переменной х. X -4 -3 —2 -1 0 1 2 3 4 У —7 0 5 8 9 8 5 0 —7 Если после изображения этих точек на плоскости какая-то часть графика осталась невыясненной, следует добавлять новые точки до тех пор, пока общий вид кривой не станет отчетливым. Затем соединим точки гладкой линией, как показано на рис. 1.2. Стрелочка здесь использована для того, чтобы подчеркнуть, что поведение графика вне изображенной области качественно не меняется. ■ Упражнение 1.1.1 Постройте график уравнения у = х2 — 4. ■ Задание 1.1. Постройте график уравнения у = ~х3 + Зх по точкам. 1. Правильный ли этот график? Обоснуйте свой ответ. 2. Добавьте точки с координатами х = —2; —1,5; —0,5; 0,5; 1,5 и 2 на график. 3. Теперь, по вашему мнению, на что стал похож график? Постройте график полностью, при необходимости добавляя новые точки. Ч 4. Постройте график с помощью графической утилиты и сравните его с графиком, полученным в п. 3. ■ www Эта пиктограмма указывает на то, что для иллюстрации примера можно использовать графическую утилиту, размещенную на веб-сайте издательства (см. предисловие). Например, чтобы построить график уравнения, упомянутый в задании 1.1, в большинстве графических утилит нужно ввести уравнение (рис. 1.3, а) и переменные окна (рис. 1.3, б). 1 Ответы к упражнениям можно найти в конце каждого раздела, перед списком задач.
Глава 1. Базовые элементарные функции 21 Рис. 1.2. Построение графика по точкам PMl Plotz tuts \ViS3X-XA3 xVz= \Vs= \7ч= \Ys= sVe= \V?= a) WINDOW Xnin= -5 Xmax=5 Xscl=l Vnin="5 Ynax=5 Yscl=l Xres=l 6) Рис. 1.3. Построение графика с помощью графической утилиты Из задания 1.1 можно сделать вывод, что далеко не всегда график бывает ясным, поскольку это зависит от выбора точек. Одна из целей этого раздела состоит в том, чтобы составить некоторый набор базовых уравнений, которые будут полезны при построении графиков. Например, кривая на рис. 1.2 называется параболой. Обратите внимание на то, что если сложить рисунок пополам вдоль оси у, то его левая часть будет полностью накладываться на правую. В таком случае говорят, что график симметричен относительно оси у, которая называется осью параболы. Позже мы убедимся, что графики всех парабол имеют общие свойства, позволяющие значительно упростить процедуру их построения. Определение функции Главным аспектом в понятии функции является соответствие. Каждый из нас ежедневно сталкивается с разными видами соответствий в повседневной жизни. Вот лишь несколько примеров. У каждого человека есть соответствующий годовой доход. У каждой вещи в магазине есть соответствующая цена. У каждого студента есть соответствующий средний балл. Каждый день температура поднимается до соответствующей наивысшей отметки. Производство х изделий имеет соответствующую стоимость. У каждого квадрата есть соответствующая площадь. Каждому числу соответствует его куб. Важной особенностью любой науки является возможность устанавливать связи или соответствия между разными явлениями. Используя эти соответствия, можно
22 Часть I. Элементарные функции делать предсказания. Например, аналитик может предсказывать цены на различные производственные товары, врач может устанавливать связь между сердечным заболеванием и избыточным весом пациента, психолог может выявлять зависимость качества выполнения определенного действия от количества повторений этого действия и т.п. Что общего между всеми этими примерами? Для каждого элемента одного множества по некоторому правилу подбирается подходящий элемент другого. Рассмотрим кубы, квадраты и квадратные корни чисел, приведенные в табл. 1.1-1.3 (см. иллюстрацию). Таблица 1.1 Таблица 1.2 Таблица 1.3 Область Область Область Область определения значений определения значений Число Куб Число Квадрат -2 > -8 -2 . -1 .-1 -iZZZz4 0 ► 0 0 '-ZZb* 1 1 ► 1 1 2 > 8 2 / Область Область определения значений Число Корень квадратный 0 0 1 > 1 2 4 >-2 3 9 --3 В табл. 1.1 и 1.2 определяются функции, а в табл. 1.3 — нет. Почему? Это будет ясно после определения того, что является функцией. Определение функции Функцией называется правило (способ или метод), устанавливающее соответствие между двумя множествами элементов так, что каждому элементу одного множества соответствует один и только один элемент другого. Первое множество называется областью определения, а второе — областью значений функции. В табл. 1.1 и 1.2 функции задаются потому, что каждому элементу из области определения соответствует только один элемент в области значений (например, кубом числа —2 будет число —8 и никакое другое). С другой стороны, табл. 1.3 не определяет функцию, поскольку одной величине из области определения соответствует несколько значений (например, квадратными корнями числа 9 являются два числа —3 и 3). Задание 1.2. Рассмотрите множество студентов и множество преподавателей учебного заведения. Установите соответствие между ними, считая, что студенту соответствует преподаватель, если студент посещает его лекции. Является ли такое сопоставление функцией? Аргументируйте ответ. ■
Глава 1. Базовые элементарные функции 23 Функции, определяемые уравнениями Области определений и значений функций, рассмотренные в книге, как правило, представляют собой (бесконечные) множества действительных чисел. Правила, связывающие элементы областей определения и значений, являются уравнениями относительно двух переменных. Рассмотрим, например, уравнение для площади прямоугольника, ширина которого на 1 дюйм меньше длины (рис. 1.4). Если длина прямоугольника равна х, то площадь задается выражением у = х(х — 1), х 1. х-1 X Рис. 1.4. Прямоугольник, ширина которого на единицу меньше длины Для каждого входного значения х (длины) с помощью этой формулы можно вычислить результат у (площадь). Например: Если х = 5, то у = 5(5 — 1) = 5 • 4 = 20. Если х = 1, то у = 1(1 — 1) = 1 • 0 = 0. Если х = \/5, то у = а/5(-\/5 — 1) = 5 — ч/б ~ 2,76. Входные данные принадлежат области определения функции, а результаты — ее области значений. Уравнение — это правило, которое заданной величине х из области определения ставит в соответствие значение у. Переменная х называется независимой переменной (потому, что ее значение выбирается из области определения совершенно произвольно), а переменная у — зависимой (поскольку ее значение зависит от выбора величины х). Вообще говоря, независимой переменной называется любая переменная, принимающая свои значения в области определения функции, а зависимой переменной — переменная, изменяющаяся в области значений функции. Что понимается под функцией, определяемой уравнением? Функции, определяемые уравнениями Уравнение с двумя переменными определяет функцию, если по заданному значению одной переменной (независимой) оно позволяет вычислить одно и только одно значение другой переменной (зависимой). Иначе говоря, одному значению на входе процедуры соответствует один результат на выходе. Если на выходе получается более одного значения, уравнение не определяет функцию.
24 Часть I. Элементарные функции Пример 1.2 (Функции и уравнения). Определить, какое из нижеследующих уравнений определяет функцию: 1. 4у — Зх = 8, х — действительное число, 2. у2 — х2 = 9, х — действительное число. Решение. 1. Решая уравнение относительно зависимой переменной, получим следующий результат. 4у — Зх — 8, 4у = 8 + 3ж, 0 2) п 3 у = 2 + -х. Каждому значению х соответствует только одно значение у (а именно, у = = 2 + |z), поэтому данное уравнение определяет функцию. 2. Решая уравнение относительно зависимой переменной, получаем следующие формулы. у2 -х2 = 9, ^^9 + ж2, (1.3) у = ±\/9 + х2. Для произвольного действительного числа х сумма 9 + х2 является положительной. Однако любое положительное число имеет два квадратных корня2, каждому значению х соответствуют два значения у, т.е. \/9 + х2 и —л/9 + я2. Например, если х = 4, то уравнение (1.3) выполняется для значений у = 5 и у = —5. Следовательно, уравнение (1.3) не определяет функцию. ■ Упражнение 1.2. Определите, какое из приведенных ниже уравнений задает функцию, зависящую от независимой переменной х. у2 — х2 = 9, где х — действительное число; Зх — 2у = 3, где х — действительное число. а Поскольку график уравнения состоит из всех упорядоченных пар, которые ему удовлетворяют, проверить, задает ли он функцию, довольно просто. Графики уравнений, рассмотренных в примере 1.2, показаны на рис. 1.5. 2Напомним, что любое положительное число N имеет два квадратных корня: y/N (главный квадратный корень), и — y/N (отрицательный квадратный корень). Подробности см. в приложении А.7, т. 2.
Глава 1. Базовые элементарные функции 25 Рис. 1.5. Примеры графиков На рис. 1.5, а любая вертикальная прямая пересекает график уравнения 4у — — Зж = 8 только один раз. Это значит, что каждому значению х соответствует только одно значение у. Следовательно, данное уравнение определяет функцию. А вот на рис. 1.5, б вертикальная прямая пересекает график уравнения у2 — х2 = 9 в двух точках, т.е. каждому значению х соответствует два различных значения у. Поэтому данное уравнение не определяет функцию. Обобщим эти выводы. Теорема 1.1 (Графическая проверка свойств функции). Уравнение определяет функцию, если любая вертикальная прямая пересекает его график только один раз. Если вертикальная прямая пересекает график более чем один раз, то уравнение не определяет функцию. ■ Задание 1.3. Функция сопоставляет любому числу из области определения одно и только одно число в области значений. 1. Приведите пример функции, каждому значению которой соответствуют два числа из области определения. 2. Приведите пример функции, каждому значению которой соответствует только одно число из области определения. ■ В примере 1.2 области определения были найдены прямо из уравнений. Однако во многих случаях этого сделать нельзя. Относительно функций, задаваемых уравнениями, будем придерживаться следующего соглашения.
26 Часть I. Элементарные функции Соглашение об областях определения и значений Если функция задана уравнением и область определения не указана, то будем считать, что область определения — вся действительная ось, а множество значений получается непосредственными подстановками значений независимой переменной в уравнение так, чтобы после вычисления получались действительные значения функции. Во многих практических приложениях математики область определения функции задается контекстом задачи (см. пример 1.7). Пример 1.3 (Поиск области определения). Найдите область определения функции, заданной уравнением у = у/4 — х, где х — независимая переменная. Решение. Для того чтобы значения у были действительными, выражение 4 - х должно быть неотрицательным. 4 — х О, —х —4, После деления обеих частей на —1 знак х 4 неравенства меняется на обратный. Следовательно, область определения задается неравенством х < 4 (запись с помощью неравенства), т.е. представляет собой интервал (—сю, 4] (запись в виде интервала). ■ Упражнение 1.3. Найдите область определения функции, заданной уравнением у = \/х — 2, где х — независимая переменная. ■ Обозначение функций Функция подразумевает существование двух множеств (областей определения и значений), а также правило сопоставления каждому элементу области определения одного элемента в области значений. Для обозначения имен функций так же, как и для чисел, используются различные символы. Например, символы f и g можно использовать для обозначения имен функций, которые задаются уравнениями у = 2х + 1 и у = х2 + 2х — 3: f : у = 2х + 1, 2 О-4) g : у = х2 + 2х — 3. Если независимая переменная обозначается символом х, то вместо буквы у для обозначения функции часто используется запись /(х), чтобы указать, какое значение принимает функция при заданном значении независимой переменной х (рис. 1.6). Это обозначение не является произведением f их. Запись /(х) читается
Глава 1. Базовые элементарные функции 27 Рис. 1.6. Функция как “/ от х" или “значение / в точке х”. Когда пишут /(ж), то х считается независимой переменной, а обе величины у и /(ж) — зависимыми. Используя обозначения функций, мы можем записать выражения (1.4) в следующем виде. /(ж) = 2ж 4-1 и д(х) = ж2 + 2ж — 3. Найдем значения /(3) и д(—5). Чтобы вычислить значение /(3), нужно заменить символ ж числом 3 везде, где он фигурирует в формуле /(ж) = 2ж 4- 1, и вычислить получившуюся правую часть: /(ж) = 2ж -I-1, /(3) = 2-34-1 = = 6-Ь1 = 7. Дано: 3, результат: 7. Следовательно, /(3) = 7. Числу 3 соответствует число 7. Аналогично для вычисления д{—5) заменим символ ж на —5 везде, где он фигурирует в формуле </(ж) = ж2 4- 2ж — 3, и вычислим получившуюся правую часть. <?(ж) = ж2 4- 2ж — 3, 5(-5) = (-5)2 + 2 • (-5) - 3 = = 25 — 10 — 3 = 12. Дано: —5, результат: 12. Итак, приходим к следующему выводу. д(—5) = 12. Числу—5 соответствует число 12. Очень важно понимать и помнить определение символа /(ж). Символ f(x) Для каждого значения ж из области определения функции / символ /(ж) задает элемент в области значений функции /, который соответствует ж. Если число ж не принадлежит области определения функции /, то говорят, что функция f не определена в точке ж и значение /(ж) не существует.
28 Часть I. Элементарные функции Пример 1.4 (Вычисление функций). Если 12 f(x) = 9&) = 1 - х2, /г(х) = Vx- 1, х — 2 то 1- w=^=7 = 3-3 2. Р(-2)[=^Ц2)> 1 - 4 = -3. 3. /1(-2):=7^2 ^’1’;= 5/=3. Однако число не является действительным. Поскольку мы договорились, что функции в точках области определения принимают только действительные значения, то число —2 не принадлежит области определения функции h, и h(—2) не существует. , - ; 4. ДО) + 5(1) - h(10);= о-? + (1 - I2) - УТО^Т;= = + 0 - V9 = -6 - 3 = -9. и Упражнение 1.4. Используя функции примера 1.4, вычислите следующие значения. 1. /(-2). 2. р(-1). 3. h(—8) 4. ® Пример 1.5 (Поиск области определения). Найдите области определения функций /, gnh: 12 f(x) = 5(х) = 1 - х2, 7i(x) = Vx - 1. х — 2 Решение. Область определения функции f выясняется, исходя из следующих рассуждений. Значение выражения является действительным числом во всех случаях, кроме х = 2 (деление на нуль запрещено). Следовательно, значение /(2) не существует, и область определения функции f состоит из всех действительных чисел, кроме числа 2. Это часто записывают следующим образом. 12 Областью определения функции д(х) является все множество действительных чисел R, поскольку при любом значении х значение выражение 1 — х2 будет действительным числом. 3В формулах рамкой выделяются математические выражения, вычисляемые в уме.
Глава 1. Базовые элементарные функции 29 Область определения функции h(x) состоит из таких чисел х, для которых выражение у/х — 1 является действительным числом. х — 1 > О, ГС > 1, или [1,оо). в Упражнение 1.5. Найдите области определения функций F, G и Н. F(x)=x2-Зх + 1, = Я(ж) = у/2^ж. и Кроме поиска значения функции в данной точке, нужно также уметь вычислять выражения, которые зависят от нескольких переменных. Например, довольно часто в математическом анализе вычисляется разностное отношение /(ж 4- /г) - У(ж) h где числа х и х + h принадлежат области определения функции /, а число h / 0. Задание 1.4. Пусть х и h — действительные числа. 1. Если /(г) = 4х 4- 3, то какая из приведенных ниже формул верна? 1) f(x + h) = 4х 4-3 4- h. 2) f(x 4- h) = 4x + 4h + 3. 3) f(x + h) = 4x 4- 4h 4- 6. 2. Если g(x) = x2, то какая из приведенных ниже формул верна? 1) д(х 4- h) = х2 4- h. 2) д(х 4- h) = х2 4- h2. 3) д(х + h) = х2 4- 2hx 4- h2. 3. Пусть M(x) = x2 4- 4x 4- 3. Опишите последовательность вычисления выражения М (х 4- h). ■ Пример 1.6 (Использование обозначений функций). Вычислите следующие значения функции /(ж) = х2 — 2а; 4- 7. 1. /(а). 2. f(a 4- h). 3 /(а 4- h) - /(а)
30 Часть I. Элементарные функции Решение. 1. /(а) = а2 — 2а + 7. 2. f (cl + /i) — (а ~h /i)2 — 2(а + /i) 4- 7 = а2 -I- 2a/i -|- /i2 — 2a — 2/i + 7. f(a + h) — /(a) _ (a2 + 2ah + h2 — 2a — 2h + 7) — (a2 — 2a + 7) ’ h = h = 2ah + h2 — 2h ; h(2a + h — 2); — ;= ;= 2a + h — 2. h ; h ; ■ Упражнение 1.6. Повторите эти же вычисления для функции /(х) = х2 — 4х + 9. ■ Решение практических задач П Оценим уровни безубыточности и прибыльности производства, описанного в задаче, исходные данные которой использованы во многих других примерах книги. Выпуская некую продукцию, предприятие несет затраты С и получает доходы R. Предприятие считается убыточным, если R < С, безубыточным, если R = С, и прибыльным, если R> С. Производственные затраты разделяются на постоянные — накладные расходы, стоимость проектирования, стоимость наладки оборудования, затраты на рекламу и тому подобное, а также переменные издержки, зависящие от объема производства. При расчете затрат нужно обязательно учитывать влияние спроса на цену производимого товара. Уравнение зависимости цены от спроса обычно определяется финансовым отделом на основе исторических или статистических данных. Пусть переменная х обозначает объем (количество) производимой предприятием продукции. Определим функции затрат и дохода, а также уравнение зависимости цены от спроса через константы а, Ь, т, п, значения которых определяются контекстом задачи. Функция затрат: С(х) = (постоянные затраты) + (переменные издержки) = а + Ьх. Уравнение зависимости цены от спроса: р = т — ПХ, где ос — объем реализуемой продукции по цене р долларов за единицу. Функция дохода: R(x) = (объем реализуемой продукции) • (цена единицы) = — хр(х) = х(т — пх).
Глава 1. Базовые элементарные функции 31 Функция прибыли: Р(х) = R — С ~ х(т — пх) — (а 4- Ьх). В примере 1.7 и упражнении 1.7 показана связь между алгебраическим определением функции, ее значениями и графиком. Чтобы в полной мере овладеть понятием функции, необходимо рассмотреть взаимосвязь между указанными математическими понятиями. В примере 1.7 будет продемонстрировано, что функции как нельзя лучше подходят для описания данных в практических задачах. Этот процесс обычно называется математическим моделированием. К рассмотренному выше примеру мы будем неоднократно возвращаться при решении многих других задач, имеющих несколько отличные условия. Пример 1.7 (Моделирование зависимости цены от спроса и прогнозиро- вание дохода). Производитель популярной модели фотоаппарата реализует свою продукцию через магазины по всей территории Соединенных Штатов. Используя статистические методы, финансовый отдел компании спрогнозировал цены, приведенные в табл. 1.4. Здесь р — оптовая цена фотоаппарата в долларах, х — общее количество реализуемых фотоаппаратов в миллионах штук. Заметьте, что при увеличении объема продаж цена за единицу падает. Приведенные в табл. 1.4 данные моделирует следующая формула, полученная методом регрессионного анализа. р(х) = 94,8 — 5х, 1 х 15. (1.5) Таблица 1.4. Моделирование зависимости цены от спроса Ху млн. шт. Ру ДОЛЛ. 2 87 5 68 8 53 12 37 Таблица 1.5. Доход Ху МЛН. шт. R(x] 1, млн. долл. 1 90 3 6 9 12 15 1. Изобразите данные, приведенные в табл. 1.4. Затем в той же системе координат постройте график уравнения (1.5). 2. Найдите функцию дохода компании и выясните ее область определения. 3. Заполните второй столбец табл. 1.5, вычислив доходы компании при заданных объемах производства в миллионах долларов. 4. Изобразите данные, приведенные в табл. 1.5, графически. Затем в той же системе координат постройте график функции дохода. 5. Постройте график функции дохода с помощью графической утилиты.
32 Часть I. Элементарные функции Решение. 1. На рис. 1.7 приведен график функции р(х), полученной методом регрессионного анализа. Обратите внимание на то, что эта функция достаточно хорошо согласуется с данными в интервале значений переменной х от 1 до 15 миллионов. 2. Функция дохода определяется следующей формулой. R(x) = хр(х} = я(94,8 — 5х) (млн. долл.) Ее область определения задается неравенствами 1 < х < 15 (так же, как и функции (1.5)). 3. Результаты вычисления приведены в следующей таблице. Рис. 1.7. Зависимость цены от спроса Таблица 1.5. Доход (полный вариант) X, млн. шт. А(х), млн. долл. 1 90 3 239 6 389 9 448 12 418 15 297 4. График функции дохода показан на рисунке.
Глава 1. Базовые элементарные функции 33 5. Результат построения графика функции дохода в окне графической утилиты приведен на следующей иллюстрации. Упражнение 1.7. Используя статистические методы, финансовый отдел компании (см. пример 1.7) спрогнозировал затраты на производство и продажу х миллионов фотоаппаратов. Данные прогноза приведены в табл. 1.6. Для описания полученных данных применяется следующая функция затрат, полученная методом регрессионного анализа С(ж) = 156 + 19,7т, 1 х 15. (1.6) 1. Изобразите данные, приведенные в таблице. Затем в той же системе координат постройте график уравнения (1.6). 2. Найдите функцию прибыли компании и очертите ее область определения. 3. Заполните табл. 1.7, вычислив прибыль (млн. долл.). Таблица 1.6. Затраты Таблица 1.7. Прибыль х, млн. шт. С(а?), млн. долл. ху млн. шт. Р(х)у млн. долл. 1 175 1 -86 5 260 3 6 305 6 12 395 9 12 15 4. Изобразите данные, приведенные в табл. 1.7. Затем в той же системе координат постройте график уравнения (1.6). 5. Постройте график функции прибыли с помощью графической утилиты. ■
34 Часть I. Элементарные функции Ответы к упражнениям 1.1. График приведен на рисунке. 1.2. 1) Уравнение не определяет функцию. 2) Уравнение определяет функцию. 1.3. х 2 (запись с помощью неравенства), [2, оо) (запись в виде интервала). 1.4. 1) -3. 2) 0. 3) Не существует. 4) 6. 1.5. Область определения функции F: R. Область определения функции G: все действительные числа, кроме —3. Область определения функции Н: х < 2 (запись с помощью неравенства) или (—оо, 2] (запись в виде интервала). 1.6. 1) а2 - 4а + 9. 2) а2 + 2a/i + h2 — 4а — 4h 4- 9. 3) 2а h — 4. 2) Р(х) = R(x)-C(x) = х(94,8 —5ж)-(156 + 19,7ж). Область определения: 1 х 15. 3) Ответ содержится в следующей таблице.
Глава 1. Базовые элементарные функции 35 Таблица 1.7. Прибыль (полный вариант) х, млн. шт. Р(ж), млн. долл. 1 -86 3 24 6 115 9 115 12 25 15 -155 Практикум 1.1 А Укажите, какие таблицы в задачах 1-6 задают функции. Область определения Область значений • Область определения Область значений 3 —> 0 -1 —- 5 5 —- 1 -2 —► 7 7 > 2 -3 —> 9 3- Область Область определения значений 3 - 5 6 4 7 5 8 Область Область определения значений 8 > 0 9 * 1 2 10 > 3 5* Область Область определения значений 6* Область Область определения значений
36 Часть I. Элементарные функции Укажите, какие из графиков в задачах 7-12 определяют функцию. В задачах 13-30 необходимо вычислить указанные выражения, если f(x) = х — 3, а = я2 + 13. 7(2). 14. /(1). 15. /(-1). 16. /(-2). 17. 3(3). 18. з(1). 19- /(0). 20. /(1/3). 21. з(-3). 22. з(—2). 23. /(1)4-3(2). 24. з(1) 4- /(2). 25. з(2) - /(2). 26. /(3) - з(3). 27. з(3) • /(0). 28. 5(0)-Л-2). 29 #(~2) /(-2) 30 « /(2) ’
Глава 1. Базовые элементарные функции 37 В задачах 31-38 с помощью приведенного графика функции определите значения х или у, округляя результат до ближайшего целого числа. В некоторых задачах возможно несколько ответов. 31. 2/= /(-5). 32.2/ =/(4). 33. у = /(5). 34. у = /(-2). 35. О = /(х). 36. 3 = /(х), х < 0. 37. —4 = /(х). 38. 4 = /(х). Б В задачах 39-48 необходимо найти области определения указанных функций. 39. F(x) = 2х3 - х2 + 3. 40. Я(х) = 7 - 2х2 - х4. 4Х F(X) = х27~3х~4- 44‘ G(X) = 45. = \/7 — х. 46. /(х) = \/Ъ + х. 41. G(x) = 48. Г(х) = -7=3=. v 7 — х V 5 + х * 49. Два математика обсуждают свойства функции х2 - 4 х2 — 9 Один говорит другому: “/(2) существует, а /(3) — нет”. В чем смысл этой фразы? *50. Возвращаясь к функции в задаче 49, выясните, существуют ли значения /(2) и /(3)? Объясните ответ. Высказывание “функция f получается умножением квадрата элемента области определения на 3 и вычитанием 7 из полученного результата ” и формула f(x) = Зх2 — 7 определяют одно и то же. В задачах 51-54 переведите словесные формулировки на язык формул. * 51. Функция g получается вычитанием числа 5 из удвоенного куба элемента области определения. * 52. Функция f получается умножением элемента области определения на число —3 и добавлением числа 4.
38 Часть I. Элементарнее функции * 53. Функция G получается умножением квадратного корня элемента области опре¬ деления на число 2 и вычитанием квадрата элемента области определения из полученного результата. * 54. Функция F получается умножением куба элемента области определения на чис¬ ло —8 и сложением полученного результата с утроенным квадратным корнем числа 3. В задачах 55-58 сформулируйте на словах определение функции, заданной алгебраически. * 55. f(x) = 2х - 3. * 56. д(х) = -2х + 7. * 57. F(x) = Зя3 - 2у/х. * 58. G(x) = kyjx - х2. В задачах 59-68 укажите, какое из уравнений определяет функцию. Если уравнение определяет функцию, то найдите ее область определения. Если нет, то укажите значение х, которому соответствует более одного значения у. 59. 4х — 5т/ = 20. 60. Зу — 7х = 15. 61. х2 — у = 1. 62. х — у2 = 1. 63. х 4- у2 = 10. 64. х2 4- у = 10. 65. ху — 4т/ = 1. 66. ху 4- у — х = 5. 67. х2 4- у2 = 25. 68. 69. Пусть F(t) = 4t 4- 7. Вычислите величину 70. Пусть G(h) = 3 — 5г. Вычислите величину 71. Пусть Q(x) = х2 — Зх 4-1. Вычислите величину 72. Пусть Р(ж) = 2х2 — Зх — 7. Вычислите величину В В задачах 73-80 необходимо вычислить и упростить выражение 74. /(ж) = —Зх 4- 9. 76. /(я) = Зх2 4- Зх - 8. 78. f(x) = х3 — х. 80- /(*) = 1. х В задачах 81-84 идет речь о площади S и периметре Р прямоугольника с длиной I и шириной w (см. рисунок). х2 - у2 = 16. F(3+fc)-F(3) h G(2-Ri)-G(2) h Q(2+h)-Q(2) h P(3+fr)-P(3) h /(а+Ь)-/(а) h 73. fix') = 4а? - 3. 75. /(а?) = 4а?2 - 7а? + 6. 77. /(х) = а?3. P=2/+2w / 81. Известно, что площадь прямоугольника составляет 25 кв. дюймов. Выразите периметр P(w) как функцию ширины w и найдите область определения этой функции.
Глава 1. Базовые элементарные функции 39 82. Известно, что площадь прямоугольника равна 81 кв. дюйму. Выразите периметр P(Z) как функцию длины I и найдите область определения этой функции. 83. Известно, что периметр прямоугольника равен 100 м. Выразите площадь S(V) как функцию длины I и найдите область определения этой функции. 84. Известно, что периметр прямоугольника равен 160 м. Выразите площадь S(w) как функцию ширины w и найдите область определения этой функции. Применение математики Экономика и бизнес 85. Зависимость цены от спроса. Компания производит микросхемы памяти для персональных компьютеров. Используя статистические методы, отделом маркетинга были собраны данные, приведенные в табл. 1.8, где р — оптовая цена одной микросхемы в долларах, а х — количество реализуемых микросхем в миллионах штук. Методом регрессионного анализа была получена следующая функция, описывающая зависимость цены микросхемы от спроса на нее р(я) = 75 — Зя, 1 х 20. Изобразите данные этой таблицы и постройте график функции в общей системе координат. Вычислите приблизительную цену микросхемы при спросе 7 млн. шт. Какова будет цена микросхемы памяти при спросе 11 млн. шт.? Таблица 1.8. Зависимость цены от спроса я, млн. шт. р, долл. 1 72 4 63 9 48 14 33 20 15 86. Зависимость цены от спроса. Компания производит ноутбуки. Используя статистические методы, отдел маркетинга получил данные, приведенные в табл. 1.9, где р — оптовая цена ноутбука в долларах, а х — общее количество реализуемых Таблица 1.9. Зависимость цены от спроса X, тыс. шт. р, долл. 1 1940 8 1520 16 1040 21 740 25 500
40 Часть I. Элементарные функции ноутбуков в тысячах штук. Зависимость цены ноутбука от спроса описывает следующая функция, полученная методом регрессионного анализа. р(х) = 2000 — 60я, 1 х 25. Изобразите данные этой таблицы и постройте график функции в одной и той же системе координат. Определите приблизительную цену ноутбука при спросе 11 тыс. шт. Вычислите ту же величину при объеме спроса, равном 18 тыс. шт. 87. Доход. а) Используя функцию зависимости цены от спроса р(х) = 75 — Зя, 1 х 20, полученную при решении задачи 85, постройте функцию дохода компании от спроса на продукцию, и найдите ее область определения. б) Заполните табл. 1.10, вычисляя доход компании для приведенных значений х (с точностью до млн. долл.). Таблица 1.10. Доход х, млн. шт. R(x), млн. долл. 1 72 4 8 12 16 20 в) Нанесите точки, координаты которых приведены в заполненной табл. 1.10, на координатную плоскость и постройте по ним график функции дохода. Значения на осях координат указывайте в миллионах штук и долларов. 88. Доход. а) Используя функцию зависимости цены от спроса р(х) = 2000 — 60я, 1 х < 25, полученную при решении задачи 86, постройте функцию дохода компании от спроса на продукцию, и найдите ее область определения. б) Заполните табл. 1.11, вычисляя доход компании для приведенных значений х (с точностью до тыс. долл.). в) Нанесите точки, координаты которых приведены в заполненной табл. 1.11, на координатную плоскость и постройте по ним график функции дохода. Значения на осях координат указывайте в тысячах штук и долларов. 89. Прибыль. Финансовым отделом компании (см. задачи 85 и 87) найдена следующая функция, описывающая зависимость производственных затрат в миллионах долларов от количества х выпущенных микросхем памяти в миллионах штук. С(х) = 125 + 16х (млн. долл.)
Глава 1. Базовые элементарные функции 41 Таблица 1.11. Доход X, ТЫС. ШТ. R(x), тыс. долл. I 1940 5 10 15 20 25 а) Запишите функцию прибыли компании, зависящую от количества произведенных микросхем памяти, и найдите ее область определения; б) Заполните табл. 1.12, вычисляя прибыль компании для приведенных значений х (с точностью до миллиона долларов). Таблица 1.12. Прибыль х, млн. шт. Р(х), млн. долл. I =69 4 8 12 16 20 в) Нанесите точки, координаты которых приведены в заполненной табл. 1.12, на координатную плоскость и постройте по ним график функции прибыли. 90. Прибыль. Финансовым отделом компании (см. задачи 86 и 88) найдена следующая зависимость производственных затрат в тысячах долларов от количества ноутбуков х в тысячах штук. С(ж) = 4000 + 500х а) Постройте функцию прибыли компании и найдите ее область определения. б) Заполните табл. 1.13, вычисляя прибыль компании для приведенных значений х (с точностью до тысячи долларов). Таблица 1.13. Прибыль х, тыс. шт. Р(х), тыс. долл. I =2650 5 10 15 20 25
42 Часть I. Элементарные функции в) Нанесите точки, координаты которых приведены в заполненной табл. 1.13, на координатную плоскость и постройте по ним график функции прибыли. 91. Упаковка. Коробка конфет изготовлена из прямоугольного куска картона размерами 8 на 12 дюймов. От каждого уголка отрезаются одинаковые квадратики со стороной х так, чтобы лист можно было согнуть в прямоугольную коробку. а) Выразите объем коробки V(х) как функцию, зависящую от переменной х. б) Найдите область определения функции V(ж) (принимая во внимание физические ограничения на размер коробки). в) Заполните табл. 1.14. Таблица 1.14. Объем х3 дюймы V(ж), куб. дюймы 1 2 3 г) Нанесите точки, координаты которых приведены в заполненной табл. 1.14, на координатную плоскость и постройте по ним график функции объема. 92. Упаковка (см. задачу 91). а) В таблице 1.15 приведены значения объема коробки для некоторых значений х в диапазоне от 1 до 2 дюймов. Используя эти значения, оцените с точностью до одного десятичного знака значение х, необходимое для изготовления коробки объемом 65 куб. дюймов. Таблица 1.15. Объем X, дюймы У(х), куб. дюймы 1,1 1,2 1,3 1,4 1,5 1,6 1,7 62,524 64,512 65,988 66,976 67,500 67,584 67,582 б) Как добиться оценки значения х с точностью до двух десятичных знаков? в) Выполните задачу, обозначенную в п. б).
Глава 1. Базовые элементарные функции 43 *93. Упаковка (см. задачи 91 и 92). а) Изучите график V (х), построенный при решении задачи 91г; при каких еще значениях х, объем коробки будет равен 65 кубическим дюймам? Составьте таблицу наподобие табл. 1.15, в которой будут указаны все такие значения с точностью до одного десятичного знака. б) Обновите таблицу, построенную в п. а так, чтобы она включала значения, указанные с точностью до двух десятичных знаков. 94. Упаковка. Служба доставки разносит посылки только в том случае, если размер коробки (длина плюс обхват) не превышает 108 дюймов. Упаковочная коробка имеет квадратное сечение со стороной х. а) Выразите объем коробки V(x) как функцию переменной х, если коробка имеет максимально допустимый размер (108 дюймов). б) Какова область определения функции V(х) (с учетом физических ограничений на размер коробки)? в) Заполните табл. 1.16. Таблица 1.16. Объем дюймы V(ж), куб. дюймы 5 10 15 20 25 г) Нанесите точки, координаты которых приведены в полностью заполненной табл. 1.16, на координатную плоскость и постройте по ним график функции объема. Биологические науки 95. WWW Сокращения мышц. Изучая скорость сокращений мышц у лягушек при различных нагрузках, известный британский биолог и нобелевский лауреат
44 Часть I. Элементарные функции А. У. Хилл (A. W. Hill) открыл приблизительную зависимость скорости их сокращения v (см/с) от прилагаемого усилия w (г). (w -I- а) (у + Ь) = с. Здесь а, b и с — некоторые константы. Для обычной мышцы можно считать а = 15, b = 1 и с = 90. Выразите w как функцию переменной v. Найдите скорость сокращения мышцы, если к ней прилагается усилие 16 г. Социальные науки 96. Политика. Зависимость процента мест 8, получаемых демократами в Палате представителей от процента v отданных за них голосов описывается следующим уравнением (процентные значения представлены десятичными дробями). 5v — 2s = 1,4; 0 < s < 1; 0,28 < v < 0,68. а) Выразите величину v как функцию переменной з. Вычислите процент голосов, который должны получить демократы, чтобы занять 51% мест в Палате представителей. б) Выразите величину s как функцию переменной v. Определите, сколько процентов мест получат демократы в Палате представителей, если за них отдан 51% голосов. 1.2. Элементарные функции: графики и преобразования ■ Библиотека элементарных функций ■ Вертикальные и горизонтальные переносы ■ Отражения, растяжения и сжатия ■ Кусочно-определенные функции Функции д(х) = х2 — 4; h(x) = (х — 4)2; к(х) = —4х2 можно выразить через функцию /(ж) = х2 как д(х) = /(х) - 4; h(x) = f(x - 4); fc(z) = -4/(ж). В этом разделе вы увидите, что графики функций д, h и к тесно связаны с графиком функции /. Получив представление о связи между графиками простых элементарных функциями, вы сможете легко проанализировать и построить графики любых других, в т.ч. более сложных функций.
Глава 1. Базовые элементарные функции 45 Библиотека элементарных функций По мере изучения новых разделов математики вы будете постоянно возвращаться к рассмотрению одних и тех же элементарных функций. В этом разделе мы дадим определения таким элементарным функциям, познакомимся с их основными свойствами и занесем их в библиотеку базовых математических инструментов, применяемых при решении самого широкого круга задач. На рис. 1.8 изображены шесть базовых элементарных функций, с которыми нам придется встречаться чаще всего, их области определения и значений, а также графики. Напомним, что абсолютное значение числа, т.е. его модуль вычисляется следующим образом (см. рис. 1.8, б): если х < О, если х > 0. а) тождественная функция Лх)=х Область определения: R Область значений: R б) модуль числа sW = l*l Область определения: R Область значений: [0, «>) в) квадрат А(х)=х2 Область определения: R Область значений: [0, «>) Область определения: R Область значений: R Область определения: [0, «>) Область значений: [0, <») Область определения: R Область значений: R Рис. 1.8. Базовые элементарные функции и их графики
46 Часть I. Элементарные функции Замечание. Символы, используемые в записи определения функции, зависят от контекста рассматриваемой задачи. Во всех приведенных выше примерах R обозначает множество действительных чисел. Вертикальные и горизонтальные переносы Если функция получена в результате изменения некой исходной функции, то график такой функции называется преобразованием графика исходной функции. Например, графики функций у = /(х) + к и у = f{x + К) можно легко получить из графика функции у = f(x). Задание 1.5. Пусть f(x) = х2. 1. Постройте графики функций у = f(x) + к для значений к = —4, 0 и 2 в одной и той же системе координат. Опишите связь между графиками у = = f(x) и у = f(x) + к при любом действительном значении к. 2. Постройте графики функций у = f(x + h) для значений h — —4, 0 и 2 в одной и той же системе координат. Опишите связь между графиком у = = f(x) и у = f(x + h) при любом действительном значении h. ■ Пример 1.8 (Вертикальные и горизонтальные переносы). 1. Как связаны графики функций у = | х | + 4 и у = | гг | — 5 с графиком функции у = |ж|? Проверьте ответ, построив все три графика в одной и той же системе координат. 2. Как связаны графики функций у = |х + 4| и у = |х — 51 с графиком функции у = |ж|? Проверьте ответ, построив все три графика в одной и той же системе координат. Решение. 1. График у = |ж| + 4 похож на график у = |гг|, только поднят на 4 единицы вверх, а график у = |ж| — 5 опущен на 5 единиц вниз (рис. 1.9). Иначе говоря, график у = f(x) + к получается из графика у = f(x) сдвигом на к единиц вверх, если к — положительная величина, и вниз, если отрицательная. 2. График у = |х + 4| похож на график у = |х|, только сдвинут на 4 еди¬ ницы влево, а график у = |ж — 5| смещен на 5 единиц вправо (рис. 1.10). Иначе говоря, график у = f(x + h) получается из графика у = f(x) сдвигом на h единиц влево, если h — положительная величина, и вправо, если отрицательная. ■ Упражнение 1.8. 1. Как графики функций у = у/х + 5 и у = у/х — 4 связаны с графиком функции у = у/х? Проверьте вывод, построив все три графика в одной и той же системе координат.
Глава 1. Базовые элементарные функции 47 Рис. 1.10. Горизонтальные переносы 2. Как графики функций у = у/х + 5 и у = у/х — 4 связаны с графиком функции у = у/х! Проверьте вывод, построив все три графика в одной и той же системе координат. ■ Сравнивая график у = /(х) + к с графиком у = /(х), можно увидеть, что первый график получается из второго с помощью вертикального переноса на к единиц вверх, если к — положительная величина, и на |А: единиц вниз, если к — отрицательное число. Аналогично сравнение графика у = /(х + /i) с графиком у = /(х) позволяет сделать вывод, что первый график получается из второго с помощью горизонтального переноса на h единиц влево, если h — положительная величина, и на |/г| единиц вправо, если h — отрицательное число. (В русскоязычной литературе оба эти преобразования называются параллельным переносом. — Примеч. ред.) Пример 1.9 (Вертикальный и горизонтальный перенос). Графики, изображенные на рис. 1.11, построены с помощью горизонтальных и вертикальных переносов графика /(х) = х2. Напишите подходящие уравнения для функций Н, G, М и N, выражая их через функцию /. Решение. Графики функций G и Н получаются с помощью вертикального переноса графика /(х) = х2 и описываются формулами: .Н(х) = х2 4- 2, G(x) = х2 — 4. Графики функций М и N получаются с помощью горизонтального переноса графика /(х) = х2 и описываются формулами: Af (х) = (х + 2)2, ^х) = (х-3)2. а Упражнение 1.9. Графики, изображенные на рис. 1.12, получаются с помощью горизонтального и вертикального переносов графика функции /(х) = у/х. Напишите подходящие уравнения для функций Н, G, М и N, выражая их через функцию f. ■
48 Часть I. Элементарные функции Рис. 1.11. Вертикальные и горизонтальные переносы Рис. 1.12. Вертикальные и горизонтальные переносы Отражения, растяжения и сжатия Теперь рассмотрим, как между собой связаны графики функций у = Af(x') и у = /(ж) для различных действительных значений числа А. Задание 1.6. 1. Постройте на одном и том же рисунке графики функции у = Af(x), если А = 1; 4 и 1/4. 2. Постройте на одном и том же рисунке графики функции у = Af(x), если А = -1, -4 и -1/4. 3. Опишите связи между графиками функций у = Л/(х) и у = f(x) при любом действительном А. ■
Глава 1. Базовые элементарные функции 49 Сравнивая графики функций у = Af{x) и у = /(т), мы видим, что график функции у = А/(т) получается из графика у = /(ж) умножением каждой ординаты на число А. Результатом будет вертикальное растяжение графика функции у = Цх), если А > 1, вертикальное сжатие, если А < 1, и отражение относительно оси х, если А = — 1. Если величина А отрицательна и меньше —1, то преобразование будет состоять из одновременного растяжения по вертикальной оси и отражения относительно горизонтальной. Пример 1.10 (Отражения, растяжения и сжатия). 1. Как графики функций у = 2 |х| и у = 0,5 |т| связаны с графиком функции у = |т|? Подтвердите свой вывод, построив все три графика в одной и той же системе координат. 2. Как график функции у = — 2 |т| связан с графиком функции у = |т|? Подтвердите свой вывод, построив все три графика в одной и той же системе координат. Решение. 1. График у = 2 |т| является результатом вертикального растяжения графика у — |ж| в 2 раза, а у = 0,5 |т| — вертикального сжатия (тоже в два раза). Этот вывод подтверждается рис. 1.13. 2. График у = — 2 |т| является результатом вертикального растяжения графика у = |т| в 2 раза и отражения относительно оси х. Этот вывод подтверждается рис. 1.14. ■ Рис. 1.13. Вертикальное растя- Рис. 1.14. Вертикальное рас- жение и сжатие тяжение и отражение Упражнение 1.10. 1. Как графики функций у = 2х и у = 0,5т связаны с графиком функции у = х? Подтвердите свой вывод, построив все три графика в одной и той же системе координат. 2. Как график у = —0,5т связан с графиком у = х? Подтвердите свой вывод, построив графики в одной и той же системе координат. ■
50 Часть I. Элементарные функции Обозначим рассмотренные ранее свойства преобразований вместе. Преобразования графиков (обзор) Вертикальные переносы ч , к > 0 — сдвиг графика у = f(x) на к единиц вверх, у = f(x) + к : < к < 0 — сдвиг графика у = f(x) на \к\ единиц вниз. Горизонтальные переносы _|_ д) h > 0 — сдвиг графика у = f(x) на h единиц влево, h < 0 — сдвиг графика у = f(x) на \h\ единиц вправо. Отражение у = —f(x) : отражение графика у = f(x) относительно оси х. Вертикальные растяжения и сжатия А > 0 — растяжение графика у = /(ж) по вертикали < путем умножения каждой ординаты на А, 0 < А < 1 — сжатие графика у = f(x) по вертикали путем умножения каждой ординаты на А, Задание 1.7. Постройте график функции у = А(х + Н)2 + к для различных значений А, h и к с помощью графической утилиты. Какая связь между графиками функций у = А(х + h)2 + к и у = х2? ■ Пример 1.11 (Комбинированные графические преобразования). Определите связь между графиками функций у = — |ж — 3| + 1 и у = |ж|. Подтвердите свой вывод, построив все три графика в одной и той же системе координат. Решение. График функции у = — |х — 3| + 1 получается из графика у = |я| отражением относительно оси х, сдвигом на три единицы вправо и сдвигом на одну единицу вверх. Этот вывод подтверждается рис. 1.15. ■ Упражнение 1.11. На рис. 1.16 изображен график функции у = G(rr), который получается в результате отражения и переноса графика у = х3. Объясните, как связаны между собой графики функций у = G(x) и у = х3 и найдите уравнение для функции У = G(x). в
Глава 1. Базовые элементарные функции 51 Рис. 1.15. Комбинированные преобразования Рис. 1.16. Комбинированные преобразования Кусочно-определенные функции Выше упоминалось, что модуль (абсолютная величина) числа определяется следующей формулой. если х < О, если х 0. Эта функция задана с помощью различных формул на разных частях области определения. Функции, определяемые с помощью нескольких формул, называются кусочно-определенными. При построении графика такой функции на каждом участке области определения используются разные формулы (см. рис. 1.17). На рис. 1.17, в изображены два типа точек: “выколотая” точка (0, —2), которая не является частью графика, и жирная точка (0, 2), принадлежащая графику. а) у = х2 — 2 б) у = 2 — х2 х2 — 2 если х < О 2 — х2 если х > О Рис. 1.17. График кусочно-определенной функции Рассмотрим практический пример.
52 Часть I. Элементарные функции И Пример 1.12 (Тарифы на природный газ). Компания Easton Utilities использует тарифы на природный газ, приведенные в табл. 1.17, чтобы определить месячную стоимость природного газа для конечных потребителей. Напишите формулы для вычисления кусочно-определенной функции стоимости потребления х (СКФ — сотен куб. футов) природного раза и постройте график этой функции. WWW Таблица 1.17. Месячная стоимость природного газа 0,7866 долл, за каждую СКФ для первых 5 СКФ 0,4601 долл, за каждую СКФ для следующих 35 СКФ 0,2508 долл, за каждую СКФ, если общий объем превышает 40 СКФ Решение. Пусть С(х) — месячная стоимость х СКФ газа в долларах. Для первой строки табл. 1.17 получаем следующую формулу. С'(я) = 0,7866х, если 0 х 5. Обратите внимание на то, что 5 СКФ будут стоить С(5) = 3,933. Если 5 < х < 40, то объем х — 5 СКФ газа рассчитывается по тарифу 0,4601 долл, за СКФ, а его стоимость равна 0,4601 (ж — 5). В результате общая стоимость выражается следующей формулой. С(ж) = 3,933х + 0,46010 - 5). Аналогично для х > 40 получим такую формулу. С(х) = 20,0365х + 0,2508(х - 40), где 20,0365 = С(40), стоимость первых 40 СКФ газа. Объединяя эти уравнения, получим следующее определение функции С(х). {0,7866а:, 3,933а: + 0,4601 (о: — 5), 20,0365а: + 0,2508(а: — 40), если 0 < х < 5, если 5 < х 40, если 40 < х. При построении графика С (о:) будем считать что все формулы в определении функции С получены в результате преобразования функции /(х) = х. Последовательно выполняя все необходимые преобразования, получим график, изображенный на рис. 1.18. ■ И Упражнение 1.12 (Тарифы на природный газ). Компания Trussville Utilities использует тарифы на природный газ, приведенные в табл. 1.18, чтобы вычислить месячную стоимость природного газа для частных потребителей. Напишите формулы определения кусочно-заданной функции стоимости потребления природного газа и постройте график этой функции. ■
Глава 1. Базовые элементарные функции 53 Рис. 1.18. Розничная стоимость х СКФ природного газа WWW Таблица 1.18. Месячная стоимость природного газа 0,7675 долл, за каждую СКФ для первых 50 СКФ 0,6400 долл, за каждую СКФ для следующих 150 СКФ 0,6130 долл, за каждую СКФ, если общий объем превышает 200 СКФ Ответы к упражнениям 1.8. 1) График функции у = у/х + 5 получается сдвигом графика у = у/х вверх на 5 единиц, а у = у/х — 4 — сдвигом вниз на 4 единицы. Иллюстрация подтверждает эти выводы. у 2) График функции у = у/х + 5 получается сдвигом графика у = у/х влево на 5 единиц, а у = у/х — 4 — сдвигом вправо на 4 единицы. Иллюстрация подтверждает эти выводы.
54 Часть I. Элементарные функции 1.9. Я(ж) = + 3, G(x) = tyx — 2, М(х) = \^х + 2, ./V(a;) = tyx — 3. 1.10. 1) График функции у = 2х получается вертикальным растяжением графика у = х, а график функции у = 0,5а? — вертикальным сжатием графика у = х. Иллюстрация подтверждает эти выводы. 2) График у = —0,5а? получается вертикальным сжатием графика у = х и отражением относительно оси х. Иллюстрация подтверждает эти выводы. 1.11. График функции G получается из графика у = х3 отражением относительно оси х и сдвигом на 2 единицы влево. Уравнение функции G имеет вид ад = - (х + 2)3.
Глава 1. Базовые элементарные функции {0,7675, 38,375 + 0,64(ж — 50), 134,375 + 0,613(^ — 200), 55 если 0 С х 50, если 50 < х 200, если 200 < х. Практикум 1.2 А Не заглядывая в ответы, найдите области определения следующих функций. 1. /(х) = 2х. 2. д(х) = — 0,3гг. 3. h(x) = —0,6\/S- 4. k(x) = 4у/х. 5. m(x) = 3|х|. 6. n(x) = — 0,lx2. 7. r(x) = —x3. 8. s(x) = 5tfx. Постройте графики функций в задачах 9-20, если /(х) и д(х) определены так. 9.у = /(ж) + 2. Ю- У = 9(.х) ~ 1- И. у = /(х + 2). 12. у = д(х - 1). 13. у = д(х — 3). 14. у = /(х + 3). 15. у = д(х) - 3. 16. у = f (х) + 3. 17. у = -/(х). 18. у = —д(х). 19. у = 0,5<?(х). 20. у = 2f(x).
56 Часть I. Элементарные функции Б Определите, как графики функций в задачах 21-28 связаны с графиками шести базовых элементарных функций, упомянутых в начале раздела (см. рис. 1.8). 21. g(x) = — |х -I- 3|. 22. h(x) = — |х — 5|. 23. /(ж) = (х - 4)2 - 3. 24. /(х) = (х + З)2 + 4. 25. /(х) = 7 — /х. 26. д(х) = — 6 4- fa 27. h(x) = —3|яг|. 28. т(х) = —0,4ят2. jcl Проверьте полученные результаты и графики с помощью графической утилиты. Каждый из графиков в задачах 29-36 получен в результате преобразования графика одной из шести базовых элементарных функций (см. рис. 1.8 в начале этого раздела). Для каждого из следующих графиков определите, какая из базовых функций была преобразована и как. Запишите уравнение, соответствующее графику.
Глава 1. Базовые элементарные функции 57 zi. Проверьте полученные результаты и графики с помощью графической утилиты. В задачах 37-42 график функции д(х) получается в результате указанной последовательности преобразований над заданной функцией f(x). Найдите уравнение функции д(х) и постройте ее график в диапазонах — 5 х 5 и — 5 < у < 5. 37. График функции /(ж) = \/х сдвигается на 2 единицы вправо и на 3 единицы вниз. 38. График функции /(ж) = tyx сдвигается на 3 единицы влево и на 2 единицы вверх. 39. График функции /(ж) = |я| отражается относительно оси х и сдвинут на 3 единицы влево. 40. График функции /(ж) = |ж| отражается относительно оси х и сдвинут на единицу вправо. 41. График функции /(ж) = х3 отражается относительно оси х и сдвинут на 2 единицы вправо и на единицу вниз. 42. График функции /(ж) = х2 отражается относительно оси х и сдвинут на 2 единицы влево и на 4 единицы вверх. В задачах 43-48 постройте графики каждой функции. если х < 2, z I 2 — 2ж, 43. / х = { ’ I х - 2, 45. h(x) = х -2, х +1, если х < О, если ж > 0. 44. д(х) = 46. к(х) = х + 1, если х 1 2 -J- 2т, если х : •1 f х +1, если х < :0, 2 + я, к если х ■ 0. -0,5я - 1, 47. т(х) = < 4 — я2, 0,5ж - 1, 2я;-ь 7, I 48. п(ж) = < х2 — 1, ( -2х + 7, I если х < —2, если - 2 х < 2, если х > 2. если х < —2, если х > 2. Проверьте полученные результаты и графики с помощью графической утилиты.
58 Часть I. Элементарные функции В В каждой из задач 49-54 график получается в результате отражения относительно оси х и/или вертикального сжатия или растяжения графика одной из шести базовых элементарных функций (см. рис. 1.8 в начале этого раздела). В каждом случае определите, какая из базовых функций была преобразована и опишите это преобразование словами. Запишите уравнение, соответствующее каждому графику. £1. Проверьте полученные результаты и графики с помощью графической утилиты. Изменение порядка преобразований может влиять на окончательный результат. В задачах 55-60 исследуйте каждую пару преобразований; меняется ли результат, если преобразования провести в обратном порядке? Проиллюстрируйте выводы на конкретном примере и/или используйте формальные математические аргументы. * 55. Вертикальный перенос; горизонтальный перенос. * 56. Вертикальный перенос; отражение относительно оси у. * 57. Вертикальный перенос; отражение относительно оси х. * 58. Вертикальный перенос; вертикальное растяжение.
Глава 1. Базовые элементарные функции 59 * 59. Горизонтальный перенос; отражение относительно оси у. * 60. Горизонтальный перенос; вертикальное растяжение. Применение математики Экономика и бизнес 61. Цены и спрос. В сеть розничных магазинов поступили CD-плейеры. Розничная цена р(х) из моделей (в долларах) и недельный спрос х на нее связаны следующим отношением. р(х) = 115 - 4v^, 9 х 289. *а) Объясните, как построить график функции р, преобразуя одну из базовых элементарных функций (см. рис. 1.8 в начале этого раздела); б) Постройте график функции р, используя выводы, полученные при решении п. а. 62. Цены и предложение. Производители CD-плейеров (см. задачу 61) готовы предложить х плейеров по цене р(я), причем эти величины связаны следующим отношением. р(х) = 4у/х, 9 х 289. *а) Объясните, как построить график функции р, преобразуя одну из базовых элементарных функций (см. рис. 1.8 в начале этого раздела); б) Постройте график функции р, используя выводы, полученные при решении п. а. 63. Расходы на лечение. С помощью статистических методов финансовое управление больницы получило следующее уравнение вычисления затрат. С(я) = 0,00048(я - 500)3 + 60 000, 100 х 1000, где С(я) — затраты (в долларах) на лечение х пациентов в месяц. ♦а) Объясните, как построить график функции р с помощью, преобразуя одну из базовых элементарных функций (см. рис. 1.8 в начале этого раздела); Ч б) Постройте график функции С, используя выводы, полученные при решении п. а, а также графическую утилиту. 64. Цены и спрос. Компания спортивного оборудования производит и продает коньки. Финансовым отделом получено следующее уравнение, описывающее зависимость цен на роликовые коньки от спроса на них. р(х) = 190 — 0,013(х — 10)2, 10 х 100, где р(х) — цена при реализации х тысяч роликовых коньков.
60 Часть I. Элементарные функции *а) Объясните, как построить график функции р, преобразуя одну из базовых элементарных функций (см. рис. 1.8 в начале этого раздела); б) Постройте график функции р, используя выводы, полученные при решении п. а. 65. Тарифы на электроэнергию. В табл. 1.19 приведены летние тарифы на электроэнергию поставляемую компанией Monroe Utilities. Под базовым тарифом понимают цену киловатта (кВт) электроэнергии, не зависящую от объема месячного потребления. а) Найдите кусочно-определенную функцию расчета месячной стоимости электроэнергии S(x), зависящей от количества киловатт ж, потребляемых в летний период. б) Постройте график функции S(x). WWW Таблица 1.19. Летние (июль—октябрь) тарифы на электроэнергию Базовый тариф: 8,50 долл, за кВт Первые 700 кВт (или меньше): 0,0650 долл, за кВт Более 700 кВт: 0,0900 долл, за кВт 66. Тарифы на электроэнергию. В табл. 1.20 приведены зимние тарифы на электроэнергию, поставляемую компанией Monroe Utilities. а) Найдите кусочно-определенную функцию расчета месячной стоимости электроэнергии W(ж), зависящей от количества киловатт х, потребляемых в зимний период. б) Постройте график функции ТУ (я). WWW Таблица 1.20. Зимние (ноябрь—июнь) тарифы на электроэнергию Базовый тариф: 8,50 долл. Первые 700 кВт (или меньше): 0,0650 долл, за кВт Более 700 кВт: 0,0530 долл, за кВт 67. Подоходный налог. В табл. 1.21 приведены последние утвержденные ставки налогообложения граждан штата Канзас, вступивших в брак в этом же штате. а) Дайте определение кусочно-заданной функции подоходного налога Т(х), где х — заработанная сумма (в долларах). б) Постройте график Т(х). в) Вычислите налог на доход 40 тыс. долл. Его же для дохода 70 тыс. долл. 68. Подоходный налог. В табл. 1.22 приведены последние утвержденные ставки налогообложения одиноких граждан штата Канзас. а) Дайте определение кусочно-определенной функции подоходного налога Т(х), где х — заработанная сумма (в долл.). б) Постройте график функции Т(х). в) Вычислите налог на доходы, равные 20 000 и 35 000 долл.
61 Глава 1. Базовые элементарные функции WWW Таблица 1.21. Ставка подоходного налога в штате Канзас Программа I (для совместно проживающих семейных пар) Доход больше, Доход не более, Т¥ Налоговая ставка долл. долл. 0 30 000 3,5% на доход, облагаемый налогом 30 000 60 000 1050 плюс 6,25% на часть суммы, превышающей 30 000 60 000 2925 плюс 6,45% на часть суммы, превышающей 60 000 *г) Как выгоднее платить налоги, проживая в Канзасе, — будучи одиноким или женатым человеком? WWW Таблица 1.22. Подоходные налоги в штате Канзас Программа II (для одиноких лиц или раздельно проживающих семейных пар) Доход больше, Доход не более, тт Налоговая ставка долл. долл. 0 15 000 3,50% на доход, облагаемый налогом 15 000 30 000 525 плюс 6,25% на часть суммы, превышающей 15 000 30 000 1462,50 плюс 6,45% на часть суммы, превышающую 30 000 Биологические науки 69. Физиология. Как правило, вес нормального человека ростом от 60 до 80 дюймов можно определить по формуле w(x) = 5,5# — 220, где х — рост человека в дюймах, a w(x) — вес в фунтах. *а) Объясните, как построить график функции w, преобразуя одну из базовых элементарных функций (см. рис. 1.8 в начале этого раздела). б) Постройте график функции w, используя выводы, полученные при решении п. а.
62 Часть I. Элементарные функции 70. Физиология, Средний вес змей некоторых видов можно определить по формуле ы(х) = 463ж3, 0,2 < х < 0,8, где х — длина змеи в метрах, a w(x) — вес в граммах. ♦а) Объясните, как построить график функции w, преобразуя одну из базовых элементарных функций (см. рис. 1.9 в начале этого раздела). б) Постройте график функции w, используя выводы п. а. Социальные науки 71. Безопасность на дорогах. В идеальных условиях скорость автомобиля г>(х) (в милях в час) непосредственно перед аварийным торможением определяется по формуле v(x) = С у/х, где х — длина тормозного пути (в футах), С — константа, которая зависит от состояния дорожного покрытия и веса автомобиля. Для наиболее распространенных моделей автомобилей v(x) = 7,08^/5 и 4 < х 144. *а) Объясните, как построить график функции v, преобразуя одну из базовых элементарных функций (см. рис. 1.9 в начале этого раздела). б) Постройте график функции v, используя решение п. а. 72. Обучение персонала. В аналитическом отчете сказано, что новый рабочий тратит в среднем Т'(х) минут на операцию сборки некоторой детали, где х — количество уже произведенных операций, причем Т(х) = 10 — у/х, 1 < х С 125. *а) Объясните, как построить график функции Т, преобразуя одну из базовых элементарных функций (см. рис. 1.8 в начале этого раздела). б) Постройте график функции Т, используя выводы, полученные при решении п. а. 1.3. Линейные функции и прямые линии ■ Сдвиг функции ■ Линейные функции, уравнения и неравенства ■ Графики функций Ах + By = С ■ Наклон прямой ■ Формы записи уравнения прямой ■ Решение практических задач В этом разделе наша библиотека пополнится новым классом элементарных функций. Это класс линейных функций включающий частный случай — тождественную функцию: /(ж) = х. Мы рассмотрим связь между линейными функциями и решениями линейных уравнений и неравенств (алгебраические методы решения линейных уравнений и неравенств см. в приложении А.8). И, наконец, мы обсудим понятие наклона прямой и ее формы для некоторых стандартных уравнений. Полученные знания будут использованы для решения важных практических задач, в частности, при нахождении зависимости цены от спроса.
Глава 1. Базовые элементарные функции 63 Сдвиги функции На рис. 1.19 показаны графики трех функций f,gnh. а)Дх) = —2х+1 б) g(x) = 3 + 2x—х2 в)й(х)=х3—2х Рис. 1.19. Графики некоторых функций Если график функции f пересекает ось х в точке с абсциссой, равной а, то число а называется сдвигом функции f по оси х. Аналогично, если график функции f пересекает ось у в точке с ординатой, равной Ь, то число b называется сдвигом функции f по оси у. Обычно точки пересечения обозначаются либо числами а и Ь, либо упорядоченными парами значений (а, 0) и (0, Ь). График функции f пересекается с осью у, если число 0 попадает в ее область определения. В этом случае сдвиг функции по оси у равен /(0). Заметьте, график функции может пересекать ось у только один раз. Точки пересечений графика с осью х являются действительными решениями уравнения /(ж) = 0. На рис. 1.19 функция f пересекает оси х и у по одному разу, функция g пересекает ось у один раз и ось х два раза, а функция h пересекает ось у один раз и ось х три раза. Линейные функции, уравнения и неравенства График функции /(ж) = — 2х + 1 на рис. 1.19 представлен прямой линией. Именно поэтому эта функция называется линейной. Поскольку выражение тх + Ь при т / 0 является полиномом первого порядка, то линейные функции часто называются функциями первого порядка. Графики линейных функций f и д, а также постоянной функции h приведены на рис. 1.20.
64 Часть I. Элементарные функции Линейные и постоянные функции Функция f называется линейной, если: /(х) = тх + Ь, т 7^ О, где т и b — некоторые действительные числа. Область определения и область значений линейной функции состоит из всех действительных чисел. Если т = 0, то функция называется постоянной: /(*) = Ъ, ее область определения состоит из всех действительных чисел, а область значений — из постоянной Ь. в) h(x) = 3 Рис. 1.20. Две линейные функции и одна постоянная В разделе 1.2 было показано, что график тождественной функции /(ж) = х также представлен прямой линией. График линейной функции общего вида д(х) = = тх + b при т 0 получается из графика f(x) = х отражением относительно оси х, если т — отрицательное число, вертикальным растяжением или сжатием на множитель |т| и сдвигом вверх или вниз на величину \Ь\. График линейной функции является прямой линией (не горизонтальной и не вертикальной). График постоянной функции является горизонтальной линией. А как же вертикальные линии? В разделе 1.1 говорилось о том, что на графике функции не может включать сразу несколько точек с одинаковыми значениями координат х. Поскольку у всех точек вертикальной прямой координаты х одинаковы, она не является графиком функции. Далее в этом разделе будут обсуждаться уравнения вертикальных прямых линий, но с оговоркой, что они не определяют функций.
Глава 1. Базовые элементарные функции 65 Задание 1.8. 1. Может ли линейная функция дважды пересекать ось ж? Может ли она ни разу не пересекать ось ж? Если ответ утвердительный, приведите пример. 2. Может ли линейная функция дважды пересекать ось у? Может ли она ни разу не пересекать ось у? Если ответ утвердительный, приведите пример. 3. Сколько всего раз график постоянной функции пересекает координатные оси? ■ Пример 1.13 (Сдвиги, уравнения и неравенства). 1. Постройте график функции /(ж) = |ж — 4 в декартовой системе координат. 2. Найдите точки пересечения графика этой функции с осями ж и у с точностью до двух десятичных знаков. 3. Постройте график функции /(ж) = |ж — 4, используя графическую утилиту. 4. Найдите точки пересечения графика этой функции с осями ж и у с точностью до двух десятичных знаков, меняя масштаб изображения. 5. Решите неравенство |ж—4 0 графически с точностью до двух десятичных знаков, используя решения пп. 1 и 2 или 3 и 4. Решение. 1. График функции /(ж) = |ж — 4 в декартовой системе координат выглядит следующим образом. Пересечение с осью у. пересечение с осью х: 2. Сдвиги можно определить, проведя следующие алгебраические вычисления. /(0) = |(0)-4 = -4, /(ж) = 0. |ж - 4 = 0, |* = 4’ х=|»2,67. О
66 Часть I. Элементарные функции %. 3. График, построенный с помощью графической утилиты, выглядит таким образом. 4. Результаты поиска пересечений с помощью графической утилиты проде¬ монстрированы на следующих рисунках. Точка пересечения с осью х: 2,67 Точка пересечения с осью у: -4 5. Графическое решение неравенства — 4 < Ос использованием решений задач 1 и 2 или 3 и 4 основано на следующих рассуждениях. Линейное неравенство |х — 4 < 0 выполняется для таких значений х, ддя которых график функции /(ж) = — 4, построенный при решении задач 1 или 2, расположен ниже оси х. В таком случае значение х должно быть не больше, чем значение точки пересечения с осью х, которое было найдено при решении задач 3 и 4. Следовательно, решением неравенства будет х 2,67 или (—оо, 2,67]. ■ Упражнение 1.13. 1. Постройте график функции f(x) = 5 в декартовой системе координат. 2. Найдите точки пересечения графика этой функции с осями х и у с точностью до двух десятичных знаков. 3. Постройте график этой функции f(x) = — + 5, используя графическую утилиту. 4. Найдите точки пересечения графика этой функции с осями х и у с точностью до двух десятичных знаков, меняя масштаб изображения. 5. Решите неравенство — + 5 0 графически с точностью до двух десятичных знаков, используя решения пп. 1 и 2 или 3 и 4. ■
Глава 1. Базовые элементарные функции 67 Графики функций Ах + By = С Теперь рассмотрим линейное уравнение первого порядка двух переменных Ах + By = С, (1.7) где оба коэффициента А и В одновременно не равны нулю. В зависимости от значений А и В это уравнение может задавать линейную функцию, постоянную функцию или вообще не определять функцию. Если А / 0 и В 0, то уравнение (1.7) можно представить следующим образом. А С у = + — Линейная функция (наклонная прямая) (1.8) В 13 Это уравнение имеет вид f(x) = тх + п, где т / 0, поэтому оно определяет линейную функцию. Если А = 0, а В 0, то уравнение (1.7) можно записать следующим образом. (j Orr + By = С =Ф> у = — Постоянная функция (горизонтальная прямая) (1.9) В Это уравнение имеет вид д(х) = Ь и определяет постоянную функцию. Если же А / 0, а В = 0, то уравнение (1.7) можно записать иначе. Ах + Оу = С => X = —. Вертикальная прямая (не является функцией) (1-Ю) А Графиком уравнения (1.10) является вертикальная прямая. Это значит, что уравнение (1.10) удовлетворяется при любом значении у, в то время как число х остается постоянной величиной, равной С) А, Поэтому такое уравнение не определяет функцию. Обобщим результаты нашего обсуждения в следующей теореме. Теорема 1.2 (График линейного уравнения двух переменных). В декартовой системе координат графиком любого уравнения вида Стандартная форма линейного уравнения Ах + By = С, (1.11) где А, В и С — некоторые действительные постоянные (А и В одновременно не равны нулю) является прямая линия. Любая прямая линия в декартовых координатах задается уравнением вида (1.11). Вертикальные и горизонтальные прямые являются простейшими частными случаями уравнения (1.11), а именно: вертикальная прямая, пересекающая ось у в точке С/В = Ь: у = Ъ\ горизонтальная прямая, пересекающая ось х в точке С/А = а: х = а. ■
68 Часть I. Элементарные функции Задание 1.9. Постройте графики трех уравнений вида Ах + By = С в одной и той же системе координат. 1. Зх + 2у — 6. 2. Ох — Зу = 12. 3. 2х + Оу = 10. В каких случаях эти уравнения определяют функции? Аргументируйте свой ответ. Постройте графики в одном окне вывода, используя графическую утилиту (см. инструкции к программе). ■ Построение графиков вида Ах + By = С или у = тх + Ь не представляет сложности, поскольку график каждого уравнения — прямая линия. Для этого достаточно изобразить на плоскости две точки, удовлетворяющие уравнению, и соединить их прямой линией. В качестве таких точек удобнее всего использовать точки пересечения линии с осями х и у. Пример 1.14 (Построение прямых). 1. Постройте графики уравнений х = — 4 и у = 6 в одной и той же прямоугольной системе координат. Ч Для сравнения постройте график с помощью графической утилиты. 2. Запишите уравнения вертикальной и горизонтальной прямых, которые проходят через точку (7, —5). 3. Постройте график уравнения 2х — Зу = 12. Для сравнения постройте график с помощью графической утилиты. Решение. 1. Графики уравнений х = — 4 и у = 6 выглядят следующим образом. утилите
Глава 1. Базовые элементарные функции 69 2. Горизонтальная линия, проходящая через (7, —5): у = —5. Вертикальная линия, проходящая через (7, —5): х — 7. 3. Чтобы построить график 2х — Зу = 12 вручную, найдем точки его пересечения с координатными осями, подставляя значения х = 0 или у = О в уравнение и вычисляя значение оставшейся переменной. Проведем прямую через найденные точки пересечения. ч Чтобы построить график с помощью графической утилиты, выразим у через х и введем полученное уравнение в программу. Построен в графической утилите Упражнение 1.14. 1. Постройте графики уравнений х = 5 и у = — Зв одной и той же прямоугольной системе координат. Для сравнения постройте график с помощью графической утилиты. 2. Запишите уравнения вертикальной и горизонтальной прямых, которые проходят через точку (—8, 2). 3. Постройте вручную график уравнения Зя + 4у = 12. Для сравнения постройте график с помощью графической утилиты. ■ Наклон прямой Выберем точки Pi(xi, yi) и Рг(^2> Уг), лежащие на прямой. Наклоном прямой называется отношение изменения переменной у к изменению переменной х, когда точка на прямой перемещается от Pi до Рг. Наклон прямой определяет “крутизну” прямой по отношению к оси х. Изменения переменных х и у называются приращениями.
70 Часть I. Элементарные функции Наклон прямой Если точка на прямой пробегает расстояние между точками и Рг(^2, У2), то наклон прямой определяется с помощью формулы У2 — У1 изменение по вертикали . т = = , Xi Ф Х2- Х2 — х± изменение по горизонтали На горизонтальной прямой переменная у не изменяется, поэтому наклон такой прямой равен нулю. На вертикальной линии не изменяется переменная х9 поэтому xi = Х2, и наклон не определен. Все возможные варианты наклона описаны в табл. 1.23. Таблица 1.23. Геометрическая интерпретация наклона прямой Прямая Наклон Пример Возрастающая при увеличении х Положительный Убывающая при увеличении х Горизонтальная Вертикальная Не определен
Глава 1. Базовые элементарные функции 71 Порядок указания координат точек Pi и Рг в формуле вычисления наклона прямой не имеет особого значения. Если координаты точек Pi и Рг поменять местами, знаки операций изменятся на противоположные одновременно как в числителе, так и в знаменателе, и наклон прямой останется неизменным. Более того, величина наклона прямой не зависит от положения точек Pi и Рг на прямой, главное, чтобы они имели разные координаты. Последнее утверждение подтверждается тем фактом, что отношения длин соответствующих сторон в подобных треугольниках — одинаковы. Пример 1.15 (Вычисление наклона прямой). Постройте графики прямых, соединяющих две заданные точки, и вычислите наклон каждой из них. 1. (-3, -2), (3, 4). 2. (-1, 3), (2, -3). 3. (-2, -3), (3, -3). 4. (-2, 4), (-2, -2). Решение. 1. График прямой, проходящей через точки (—3, —2) и (3, 4), выглядит следующим образом. 2. График прямой, проходящей через точки (—1, 3) и (2, —3), выглядит следующим образом. -3-3 -6 Ш‘2-(-1)- 3 - 2-
72 Часть I. Элементарные функции 3. График прямой, проходящей через точки (—2, —3) и (3, —3), выглядит следующим образом. т = -3- (-3) _ О 3 - (-2) ” 5 4. График прямой, проходящей через точки (—2, 4) и (—2, —2), выглядит следующим образом. -2-4 -6 m ~ -2 - (-2) " "(Г’ наклон не определен. Упражнение 1.15. Вычислите наклоны прямых, соединяющих две заданные точки. 1. (-2, 4), (3, 4). 2. (-2, 4), (0, -4). 3. (-1, 5), (-1, -2). 4. (-1, -2), (2, 1). и Формы записи уравнения прямой Выясним, почему зависимость вида у = тх + b часто называется уравнением прямой с угловым коэффициентом.
Глава 1. Базовые элементарные функции 73 Задание 1.10. 1. Постройте график уравнения у = х + b для значений b = —5, —3, 0, 3 и 5 в одной и той же системе координат. Объяснить на словах геометрический смысл постоянной Ь. 2. Постройте график уравнения у = тх — 1 для значений т = —2, —1, 0,1 и 2 в одной и той же системе координат. Объяснить на словах геометрический смысл постоянной т. Ч 3. Построить график у = тх + Ь для различных значений т и Ь с помощью графической утилиты. ■ Выполняя предыдущее задание, мы выяснили, что величины т и b в уравнении у = тх + Ь имеют особый геометрический смысл, к обсуждению которого мы сейчас перейдем. Если х = 0, то у = Ь, т.е. график у = тх + Ь пересекает ось у в точке (0, Ь). Константа Ь является сдвигом графика по оси у. Например, график прямой у = —4а; — 1 пересекает ось у в точке —1. Для выяснения геометрического смысла константы т поступим так. Если у = = тх + Ь, то, полагая х = 0 и х = 1, мы найдем, что точки (0, Ь) и (1, т + 6) принадлежат прямой. Наклон такой прямой выражается следующей формулой. „ 1/2 - У1 (т + Ь)-Ь Наклон = -—-—-— = т. Х2 — Х\ 1 — 0 Таким образом, наклоном прямой у = тх + b равен т. Уравнение прямой с угловым коэффициентом Уравнение у = тх + Ь, т = наклон, b = пересечение с осью у (1-12) называется уравнением прямой с угловым коэффициентом и пересечением. П Пример 1.16 (Использование уравнения прямой с угловым коэффициентом) 1. Найдите наклон и сдвиг по оси у, а также постройте график прямой у = = — |х — 3. 2. Найдите уравнение прямой с наклоном | и сдвигом —2. Решение. 1. Наклон = т = — |, сдвиг по оси у = b = —3.
74 Часть I. Элементарные функции 2. т = j и b = —2, поэтому у = |х — 2. ■ Упражнение 1.16. Найдите уравнение прямой, наклон которой равен |, а сдвиг по оси у равен — 1. Постройте график этой прямой. ■ Пусть прямая имеет наклон т и проходит через заданную точку (xi, у\). Если (х, у) — любая другая точка на прямой (рис. 1.21), то = т или у-У1 = m(x-xi). (1.13) Рис. 1.21. Прямая, проходящая через указанную точку с заданным наклоном Очевидно, точка (xi, у±) удовлетворяет уравнению (1.13). Это уравнение задает прямую, которая имеет наклон т и проходит через заданную точку (xi, у\). Уравнение прямой, проходящей через указанную точку, с заданным наклоном Уравнение прямой, которая имеет наклон т и проходит через указанную точку (#1, 3/1), имеет следующий вид. У - У1 = т(х - Х1).
Глава 1. Базовые элементарные функции 75 Эта форма уравнения прямой также часто применяется в математических расчетах. Она позволяет записать уравнение прямой, если известны координаты какой-либо ее точки и наклон (или координаты еще одной точки, тогда наклон можно вычислить). В Пример 1.17 (Уравнение прямой, проходящей через указанную точку, с заданным наклоном). 1. Найдите уравнение прямой с наклоном которая проходит через точку (—4, 3). Окончательный ответ запишите в форме Ах + By = С. 2. Найдите уравнение прямой, которая проходит через точки (—3, 2) и (—4, 5). Окончательный ответ запишите в форме у = тх -I- Ь. Решение. 1. Используем уравнение у—у\ = т(х—х\). Пусть т = | и (a?i, yi) = (—4, 3). Выполним следующие вычисления. 1/~3 = |[®-(-4)], у — 3 = - (я + 4), Умножаем на 2 2у - 6 = X + 4, —х + 2у = 10, или х — 2у = —10. 2. Для начала найдем наклон прямой по следующей формуле. _ У2 — г/i _ 5-2 _ 3 _ яг — Xi —4 — (—3) —1 Теперь используем уравнение у — yi = т(х — a?i), в которое подставим параметры т = —3 и (яц, yi) = (—3, 2). Выполним следующие вычисления. у-2 = -3[х-(-3)], У - 2 = -3[х 4- 3], у — 2 = — Зх — 9, у = — Зх — 7. я Упражнение 1.17. 1. Найдите уравнение прямой с наклоном которая проходит через точку (6, —2). Окончательный ответ представьте в форме Ах + By = С, А > 0. 2. Найдите уравнение прямой, которая проходит через две точки (2, —3)и(4, 3). Окончательный ответ представьте в форме у = тх + Ь. ■
76 Часть I. Элементарные функции Обозначим все рассмотренные формы уравнения прямой в таблице. Таблица 1.24. Уравнения прямой Стандартная форма Уравнение прямой с угловым коэффициентом Уравнение прямой, проходящей через заданную точку, с указанным наклоном Горизонтальная прямая Ах + By = С у = тх + b У-У1 =т(х-Х1) У = b х = а А и В одновременно не равны нулю Наклон: т, сдвиг по оси у: b Наклон: т, точка: (xi,j/i) Наклон: О Наклон: не определен Вертикальная прямая Решение практических задач Рассмотрим практические задачи, в решении которых применяется уравнение прямой. И Пример 1.18 (Уравнение затрат). Ежедневные постоянные затраты — расходы в отсутствии выпуска товара — компании, производящей роликовые коньки, равны 300 долл., а общие производственные затраты — 4300 долл, при выпуске 100 пар роликовых коньков в день. Предположим, что функция затрат С линейно зависит от объема х выпускаемой продукции. 1. Найдите наклон прямой, проходящей через точки с абсциссами 0 и 100, т.е. (0, 300) и (100, 4300). 2. Найдите уравнение функции затрат. Ответ представьте в форме у = тх + Ъ. 3. Постройте график функции затрат в диапазоне 0 < х < 200. Решение. . У2-У1 4300-300 4000 AfX 1.772 = = = = 40. Х2 — х\ 100 — 0 100 2. Необходимо найти уравнение прямой, которая проходит через точку (0, 300) с наклоном, равным 40. Используем соответствующую форму уравнения прямой. С = тх + Ь, С = 40ж + 300.
Глава 1. Базовые элементарные функции 77 В примере 1.18 постоянные затраты подразумевают расходы на содержание завода, страхование и т.п. Компания несет эти затраты независимо от того, производит она товар или нет. Переменные издержки равны 40т. Их величина определяется объемом производимой за день продукции. Заметим, что величина наклона прямой определяет издержки на выпуск одной пары роликовых коньков, т.е. указывает затраты на производство единицы продукции. В Упражнение 1.18. Ответьте на вопросы 1 и 2 примера 1.18, если постоянные затраты составляют 250 долл, в день, а общие затраты на производство 80 пар роликовых коньков — 3450 долл, в день. ■ И www Пример 1.19 (Зависимость цены от спроса).В начале 21-го века мировой спрос на сырую нефть был равен 75 млн. баррелей в день, а цена барреля колебалась между 20 и 40 долл. Допустим, что при цене 25,52 долл, за баррель дневной спрос на нефть равен 76,1 млн. баррелей. Предположим также, что при цене 33,68 долл, за баррель спрос на нефть падает до 74,9 млн. баррелей. Найдите функцию вида р = ах + Ь, которая описывает связь между ценой р и спросом х на сырую нефть, считая, что она определяется линейной зависимостью. Примените полученное уравнение для предсказания спроса на нефть при цене 39,12 долл, за баррель. Решение. Найдем уравнение прямой, соединяющей точки (76,1; 25,52) и (74,9; 33,68). Сначала вычислим наклон прямой. 33,68 - 25,52 8,16 т= 74,9-76,1 = .ПГ~(^
78 Часть I. Элементарные функции Используя общий вид уравнения прямой, проходящей через указанную точку с заданным наклоном, получим следующие выражения. р — pi = т(х — а?1); р - 25,52 = -6,8(а; - 76,1); р - 25,52 = —6,8а; + 517,48; р = — 6,8а? -I- 543. Для того чтобы найти спрос при цене 39,12 долл, за баррель, решим уравнение р = 39,12 относительно х. р = 39,12; - 6,8а; + 543 = 39,12; —6,8а: = —503,88 : -503,88 х = —— = 74,1 млн. барелей в день. Упражнение 1.19. Дневное предложение сырой нефти также зависит от цены. Допустим, что дневное предложение нефти было равно 73,4 млн. баррелей при цене 23,84 долл, за баррель. Пусть предложение выросло до 77,4 млн. баррелей при цене 34,24 долл, за баррель. Найдите функцию вида р = ах + Ь, которая описывает связь между предложением р и ценой аг, считая, что она определяется линейной зависимостью. Примените эту модель для предсказания предложения нефти при падении цены до 20,98 долл, за баррель. ■ На рис. 1.22 приведены графики функций спроса и предложения в зависимости от цены (пример 1.19 и упражнение 1.19). На конкурентном рынке цена товара определяется балансом спроса и предложения. Стоимость товара стремится к стабилизации в точке пересечения графиков спроса и предложения. Эта точка называется точкой равновесия, соответствующая ей цена — равновесной ценой, а общий объем спроса и предложения — равновесным количеством. Точку равновесия для графиков зависимости цены от предложения и спроса, представленных на рис. 1.22, можно найти так. 2,6а; — 167 = —6,8а; + 543; 9,4а; = 710; 710 „ гпп х = = 75,532 млн. баррелей; р = 2,6(75,532) - 167 = 29,36 долл. р = —6,8(75,532) 4- 543 = 29,36 долл. Равновесное количество Равновесная цена Проверка
Глава 1. Базовые элементарные функции 79 Млн. баррелей Рис. 1.22. Точка равновесия Ответы к упражнениям 2) Пересечение с х: 3,75; пересечение с у: 5; 4) Пересечение с х: 3,75; пересечение с у: 5; 5) х 3,75, или (—оо; 3,75].
80 Часть I. Элементарные функции 1.14. 1) График, построенный вручную. График, построенный с помощью графической утилиты 2) Горизонтальная прямая: у = 2; вертикальная прямая: х = —8. 3) График, построенный вручную. График, построенный с помощью графической утилиты -10
Глава 1. Базовые элементарные функции 81 1.15. 1) 0. 3) Не определен. 1.16. у = ^-х — 1. * 2 1.17. 1) 2х - Зу = 18. 1.18. 1) т = 40. 2) у = За: — 9. 2) С = 40а: + 250. 1.19. p(z) — 2,6а; — 167; 72,3 млн. баррелей. Практикум 1.3 А Задачи 1-4 относятся к графикам а)-г)
82 Часть I. Элементарные функции 1. Укажите график линейной функции с отрицательным наклоном. 2. Укажите график линейной функции с положительным наклоном. 3. Укажите график постоянной функции. Какой наклон у этого графика? 4. Укажите рисунок, который не является графиком функции. Что можно сказать о его наклоне? В задачах 5-8 постройте график каждого уравнения в декартовых координатах. 5. у = 2х-3. 6. у = + 1. 7. 2х + Зу = 12. 8. 8х - Зу = 24. В задачах 9-12 найдите наклон и пересечение каждого графика с осью у. 9. у = Зх + 1. 10. у = f - 2. 5 3 11. у = —~х — 6. 12. у = 0,7х -F 5. В задачах 13-16 найдите уравнение прямой по заданному наклону и пересечению с осью у. 13. Наклон равен —2; координата пересечения с осью у равна 3. 14. Наклон равен координата пересечения с осью у равна —5. 15. Наклон равен координата пересечения с осью у равна —4. 16. Наклон равен —5; координата пересечения с осью у равна 9. Б В задачах 17-22 постройте график каждого уравнения (или пары уравнений) в декартовых координатах. 2 3 17. у = --х - 2. 18. у = --х + 1. 19. Зх - 2у = 10. 20. 5х - бу = 15. 21. х = 3, у = —2. 22. х = —3, у = 2. jcl Проверьте графики, построенные в задачах 17-22, с помощью графической утили- ° ты. В задачах 23-26 найдите наклон графика каждого уравнения. 23. 4ж 4- у = 3. 24. 5х — у — —2. 25. Зх 4- бу = 15. 26. 2х — Зу = 18. 27. Выполните следующие задания. а) Постройте график функции /(ж) = 1,2а; — 4,2. б) Найдите пересечения графика с осями х и у с точностью до одного десятичного знака. в) Постройте график функции /(ж) = 1,2х — 4,2 с помощью графической утилиты. г) С помощью графической утилиты найдите пересечения графика с осями х нус точностью до одного десятичного знака, меняя масштаб изображения. д) Используя результаты, полученные в пп. а-б или в-г, решите линейное неравенство 1,2ж — 4,2 > 0.
Глава 1. Базовые элементарные функции 83 28. Выполните следующие задания. а) Постройте график функции /(х) = -0,8х + 5,2. б) Найдите пересечения графика с осями х и у с точностью до одного десятичного знака. в) Постройте график функции f(x) = — 0,&г + 5,2 с помощью графической утилиты. г) С помощью графической утилиты найдите пересечения графика с осями х и у с точностью до одного десятичного знака, меняя масштаб изображения. д) Используя результаты, полученные в пп. а-б или в-г, решите линейное неравенство -0,8# + 5,2 < 0. В задачах 29-32 напишите уравнения вертикальных и горизонтальных прямых, проходящих через заданную точку. 29. (4, -3). 30. (-5, 6). 31. (-1,5; -3,5). 32. (2,6; 3,8). В задачах 33-38 найдите уравнения прямых, проходящих через указанную точку с заданным наклоном. Окончательный ответ представьте в виде у = тх + Ь. 33. т = -4; (2, -3). 34. т = -6; (-4, 1). 35. m = |; (—4, —5). 36. т = (—6, 2). 37. т = 0; (-1,5; 4,6). 38. т = 0; (3,1; -2,7). В задачах 39-46 найдите наклон каждой прямой, проходящей через две заданные точки. 39. 41. 43. 45. 40. (1, 2) 42. (2, 3) 44. (1, 4) 46. (2, 0) и и и и (3, 5). (-3, 7). (0, 4). (2, -3). 48. (1, 2) 50. (2, 3) 52. (1, 4) 54. (2, 0) (3, 5). (-3, 7). (0, 4). (2, -3). (2, 5) и (5, 7). (“2, -1) и (2, —6). (5, 3) и (5, -3). (-2, 5) и (3, 5). В задачах 47-54 найдите уравнения прямых, проходящих через две заданные точки. Окончательный ответ представьте в виде Ах -I- By = С. В каких случаях уравнение будет задавать линейную либо постоянную функцию или вообще не определять функцию? 47. (2, 5) и (5, 7). 49. (-2, -1) и (2, -6). 51. (5, 3) и (5, -3). 53. (-2, 5) и (3, 5). * 55. Какая связь между графиками, описываемых уравнением у = тх + 2, где т — произвольное действительное число? и и и и * 56. Какая связь между графиками, описываемых уравнением у = — 0,5я 4- Ь, где Ь — произвольное действительное число?
84 Часть I. Элементарные функции В 57. Выполните следующие задания. а) Постройте графики следующих уравнений в одной и той же системе координат. Зх + 2у = 6, Зх + 2у = 3, Зх 4- 2у = —6, Зх 4- 2у = —3. *б) Руководствуясь выводом, сделанным при решении п. а, опишите семейство кривых, порождаемых уравнением Ах -I- By = С при фиксированных значениях А и В и переменном значении С. 58. Выполните следующие задания. а) Постройте графики следующих уравнений в одной и той же системе координат. Зх 4- 4г/ = 12, 4т — Зу = 12. б) Постройте графики следующих уравнений в одной системе координат. 2х -I- Зу = 12, Зх — 2у = 12. *в) Руководствуясь выводами, полученными при решении пп. а-б, опишите связь между графиками Ах + By = С и Вх — Ау = С. * 59. Найдите связь между графиками f(x) = тх + Ь и д(х) = \тх + 0. Вы¬ вод проиллюстрируйте примером. Всегда ли д(х) является линейной функцией (иногда, никогда)? * 60. Найдите связь между графиками f(x) = тх 4- b и д(х) = т\х\ + Ь,т 0. Вы¬ вод проиллюстрируйте примером. Всегда ли д(х) является линейной функцией (иногда, никогда)? Применение математики Экономика и бизнес 61. Простые проценты. Предположим, что вы положили сумму Р (долл.) на депозитный счет с процентной ставкой г. Через t лет на депозитном счету будет находиться сумма А, рассчитываемая по формуле А = Prt 4- Р. При инвестировании 100 долл, под 6% (т.е. г = 0,06) расчетная формула принимает вид А = 6t 4-100, при 0. а) Какая будет сумма будет накоплена на счете через 5 лет, если исходно на нем находилось 100 долл.? А через 20 лет? б) Постройте график функции А = 6t 4-100 для 0 < t < 20. *в) Найдите наклон графика и опишите его словесно.
Глава 1. Базовые элементарные функции 85 62. Простые проценты. Рассмотрим формулу из задачи 61. При инвестировании 1000 долл, под 7,5% (т.е. г = 0,075) расчетная формула принимает вид А = = 75t + 1000, при t > 0. а) Какая сумма будет накоплена на счете через 5 лет, если исходно на нем находилось 1000 долл.? А через 20 лет? б) Постройте график функции А = 75t + 1000 для 0 t < 1000. *в) Найдите наклон графика и опишите его словесно. 63. Функция затрат. Постоянные затраты (в отсутствие выпуска продукции) компании, производящей доски для серфинга, составляют 200 долл, в день, а ежедневные общие затраты равны 3800 долл, при изготовлении 20 досок в день. а) Найдите уравнение функции затрат С, считая, что она линейно зависит от объема произведенной продукции х. б) Чему равны общие затраты при выпуске 12 досок в день? в) Постройте график функции затрат в пределах 0 х 20. 64. Функция затрат. Выполните задачу 63 при условии, что постоянные затраты составляют 300 долл, в день, а общие затраты — 5100 долл, при выпуске 20 досок в день. 65. Зависимость цен от спроса. Компания собирается выпустить на рынок новую мощную газонокосилку. Аналитическим отделом получен следующий прогноз зависимости цен от спроса (табл. 1.25). Таблица 1.25. Цены и спрос Спрос Ж, шт. Оптовая цена р(х), долл. 0 200 2400 160 4800 120 7800 70 а) Изобразите точки, приведенные в таблице, на графике, считая р(х) ценой (вертикальная ось), а х — количеством реализуемых газонокосилок (горизонтальная ось). б) Очевидно, что точки, рассматриваемые в п. а, принадлежат общей прямой. Найдите ее уравнение.
86 Часть I. Элементарные функции в) Какая цена соответствует спросу, равному 3000 единиц? *г) Дайте словесную интерпретацию наклона прямой, полученной при решении п. б. 66. Обесценивание. Оборудование офиса стоимостью 20000 долларов через 10 лет будет стоить 2000 долларов. Считается, что цена падает линейно от 20 000 до 2000 долларов. а) Найдите линейное уравнение, описывающее зависимость цены оборудования (У) от его возраста (t). б) Какова будет стоимость оборудования офиса после 6 лет эксплуатации? в) Постройте график функции затрат на отрезке 0 < t < 10. *г) Дайте словесную интерпретацию наклона прямой, полученной при решении задачи 1. 67. WWW Точка равновесия. Годовое предложение и спрос на зерно в Соединенных Штатах составляет 8,5 и 9,8 млн. бушелей соответственно при цене 2,50 долл, за бушель. При увеличении цены до 3,30 долл, за бушель предложение выросло до 10,5 млн. бушелей, а спрос упал до 7,8 млн. бушелей. а) Найдите уравнения зависимости предложения и спроса от цены, считая, что они — линейны. б) Найдите точку равновесия для американского рынка зерна. 68. WWW Точка равновесия. Годовое предложение и спрос на соевые бобы в Соединенных Штатах составляет 2,4 и 2,9 млн. бушелей соответственно при цене 5,50 долл, за бушель. При увеличении цены до 7,30 долл, за бушель предложение выросло до 2,8 млн. бушелей, а спрос упал до 2,4 млн. бушелей. а) Найдите уравнения зависимости предложения и спроса от цены, считая, что они — линейны. б) Найдите точку равновесия для американского рынка соевых бобов. WWW Корпорация Merck & Со — одна из крупнейших на фармацевтическом рынке. В задачах 69 и 70 используются данные табл. 1.26, основанные на годовых отчетах компании. Таблица 1.26. Финансовые показатели компании Merck & Со (млрд, долл.) Год 1995 1996 1997 1998 1999 Объем продаж 16,7 19,8 23,6 26,9 32,7 Доход 3,3 3,8 4,6 5,2 5,9 69. Анализ продаж. Объем продаж компании Merck & Со рассчитывается по следующей формуле. /(х) = 16,12^-1-3,91; где значение х = 0 соответствует 1995 году.
Глава 1. Базовые элементарные функции 87 а) Заполните следующую таблицу (значения /(ж) округляйте до одного десятичного знака). X Объем продаж, млрд. долл, /(ж) 0 16,7 1 19,8 2 23,6 3 26,9 4 32,7 б) Изобразите данные о продажах и график функции /(х) на одном и том же рисунке. в) Используя исходное уравнение, оцените объем продаж в 2005 и 2010 гг. *г) Дайте словесное описание объема продаж компании за период с 1995 по 1999 год. 70. Анализ доходов. Доход компании Merck & Со рассчитывается по следующей формуле. f(x) = 3,24z + 0,66; где значение х = 0 соответствует 1995 году. а) Заполните следующую таблицу (значения f(x) округляйте до одного десятичного знака). х Доход, млрд. ДОЛЛ. “О зз 1 3,8 2 4,6 3 5,2 4 5,9 б) Изобразите данные о доходах и график функции f(x) на одном рисунке; в) Используя исходное уравнение, оцените доходы компании в 2005 и 2010 гг *г) Дайте словесное описание доходов компании за период с 1995 по 1999 год. 71. WWW Потребление угля. Анализируя данные по Соединенным Штатам за период с 1920 по 1960 год, министерство энергетики обнаружило, что доля угля в общей структуре традиционных источников производства электроэнергии (дрова, уголь, нефть, природный газ, ГЭС и АЭС) неуклонно снижается почти по линейному закону. Данные за указанный период приведены в следующей таблице. Год Производство электроэнергии из угля, % 1920 72 1930 60 1940 50 1950 37 1960 22
88 Часть I. Элементарные функции а) Пусть х обозначает год (начиная с 1900). Найдите уравнение линейной функции, для которой /(20) = 72 и /(60) = 22. Постройте график функции / и изобразите данные, приведенные в таблице, на одном и том же рисунке. Насколько хорошо функция / описывает эти данные? б) Используя функцию /(я), оцените с точностью до 1% объем производства электроэнергии из угля в 1927 и 1953 г. *в) Если считать, что функция /(ж) достаточно хорошо описывает данные по генерации электроэнергии из угля и после 1960 года, то в какой момент времени она выйдет на нулевой уровень? Может ли такое на самом деле произойти? (Если вы не уверены в ответе, посмотрите в какой-либо справочник.) Если нет, то какова возможная причина увеличения объемов генерации электроэнергии из угля после 1960 года? 72. WWW Потребление нефти. Анализируя данные по Соединенным Штатам с 1920 по 1960 год, министерство энергетики обнаружило, доля нефти в общей структуре традиционных источников производства электроэнергии (дрова, уголь, нефть, природный газ, ГЭС и АЭС) неуклонно растет почти по линейному закону. Данные за этот период приведены в следующей таблице. Год Производство электроэнергии из нефти, % 1920 1930 1940 1950 1960 11 22 29 37 44 а) Пусть х обозначает год (начиная с 1900). Найдите уравнение линейной функции, для которой /(20) = 11 и /(60) = 44. Постройте график функции / и изобразите данные, приведенные в таблице, на одном и том же рисунке. Насколько хорошо функция / описывает данные таблицы? б) Используя функцию /(х), оцените с точностью до 1% объем производства электроэнергии из нефти в 1932 и 1956 гг. *в) Если считать, что функция /(ж) достаточно хорошо описывает данные по генерации электроэнергии из нефти и после 1960 года, то в какой момент времени она достигнет значения 100%? Может ли такое на самом деле произойти? Аргументируйте свой ответ. Биологические науки 73. Рацион питания. Проводя научные исследования, биолог содержит подопытных животных на специальной диете. В его распоряжении находятся две пищевые смеси, А и В. Смесь А содержит 20% протеинов, а смесь В — 10%. В какой комбинации обеих смесей будет содержаться 20 г протеина? Пусть х обозначает количество смеси А, а у — смеси В. Напишите линейное уравнение, которое связывает величины А, В и число 20. Постройте график этого уравнения для х 0 и у 0.
Глава 1. Базовые элементарные функции 89 74. Экология. При погружении в океан давление, действующее на водолаза, возрастает по линейному закону. Если на поверхности океана давление составляет 15 фунтов на кв. дюйм, то на 33 фута ниже этого уровня оно будет равно 30 фунтам на кв. дюйм. а) Пусть р — давление в фунтах, d — глубина (в футах). Напишите уравнение, выражающее р через d. [Подсказка: напишите уравнение прямой, которая проходит через точки (0, 15) и (33, 30).] б) Какое будет давление на глубине 12 540 футов (средняя глубина океана)? в) Постройте график уравнения в пределах 0 < d 12 540. *г) Кратко опишите геометрический смысл наклона прямой, найденной при решении п. а. Социальные науки 75. Поведение животных. В своих исследованиях мотивационного поведения Дж. С. Браун (J. S. Brown) дрессировал крыс. В одном из опытов для того, чтобы добраться до пищи, животные должны были пробраться через узкий проход в клетке. Подопытные крысы снабжались ошейниками, соединяемыми проволокой с пружинными весами. Суть опыта заключалась в измерении (вот для чего нужны весы) усилия р (г), которое приходится преодолевать крысам, находясь на неком расстоянии d от кормушки с пищей. Браун нашел простое линейное отношение между величинами pud, которое имеет следующий вид. р = — 4- 70, где 30 d < 175. 5 (Источник: J. S. Brown, Journal of Comparative and Physiological Psychology, 1948,41,450-465). а) Какое усилие прилагает крыса, двигаясь к кормушке и находясь от нее на расстоянии d = 30? А на расстоянии d = 70? б) Постройте график уравнения. в) Какой наклон у этой прямой?
90 Часть I. Элементарные функции 1.4. Квадратичные функции ■ Квадратичные функции, квадратные уравнения и неравенства ■ Свойства квадратичных функций и их графиков ■ Решение практических задач Если степень линейной функции увеличить на единицу, то мы получим функцию второго порядка, которая обычно называется квадратичной функцией. Это следующая функция, которую нужно включить в нашу библиотеку элементарных функций. В этом разделе будет рассмотрена связь между свойствами квадратичных функций и решениями квадратных уравнений и неравенств. (Подробное изложение свойств алгебраических решений квадратных уравнений приведено в приложении А.9, т. 2.) Мы также рассмотрим многие другие важные свойства квадратичных функций, в том числе научимся определять их максимумы и минимумы. Все они часто применяются при решении многих практических задач, в частности определении объема производства, обеспечивающего получение максимального дохода или прибыли. Квадратичные функции, квадратные уравнения и неравенства На рис. 1.23 приведен график квадратичной функции h(x) = х2. Отметим, что график симметричен относительно оси у и точка (0, 0) является самой нижней точкой графика. Рассмотрим базовые преобразования этого графика. Рис. 1.23. Квадратичная функция h(x) = х2 Задание 1.11. Объясните, как графики приведенных ниже функций связаны с графиком h(x) = = х2. Найдите низшую или высшую точки (если они существуют) на каждом из графиков. 1. f(x) = (х — З)2 — 7 = х2 — 6х + 2. 2. д(х) = 0,5(ж + 2)2 + 3 = 0,5а;2 + 2х + 5.
Глава 1. Базовые элементарные функции 91 3. т(х) = —(я — 4)2 + 8 = —х2 + 8я — 8. 4. п(я) = —3(я + I)2 — 1 = —Зя2 — 6я — 4. ■ Графики функций из предыдущего задания по своей форме очень напоминают график квадратичной функции, изображенный на рис. 1.23. Такие кривые называются параболами. Функции, порождающие такие графики, составляют важный класс квадратичных функций. Дадим определение этого класса функций. Квадратичные функции Если а,Ь, с — действительные числа и а / 0, то функция /(ж) = ах2 + Ьх + с называется квадратичной функцией, а ее график — параболой. Поскольку выражение ах2 + Ьх + с является действительной величиной при всех действительных х, то справедливо следующее утверждение. Областью определения квадратичной функции является все множество действительных чисел. Методы определения области значений квадратичных функций мы рассмотрим позже. Типичные графики квадратичных функций приведены на рис. 1.24. Рис. 1.24. Графики квадратичных функций Пример 1.20 (Пересечения, уравнения и неравенства). 1. Постройте график функции /(я) = —х2 + 5я + 3 в декартовой системе координат. 2. Найдите пересечения этого графика с осями декартовой системы координат (с точностью до двух десятичных знаков). *4 3. Постройте график функции /(я) = —я2 + 5я + 3 с помощью графической утилиты.
92 Часть I. Элементарные функции 4. Найдите пересечения этого графика с осями декартовой системы координат с помощью графической утилиты, меняя масштаб изображения (с точностью до двух десятичных знаков). 5. Решите неравенство —х2 + 5т + 3 > 0 графически с точностью до двух десятичных знаков, используя результаты решения задач 1 и 2 или 3 и 4. М 6. Решите уравнение —х2 + 5х + 3 = 4 графически с точностью до двух десятичных знаков, используя команду поиска корней уравнения графической утилиты. ■ Решение. 1. Построение графика вручную. 2. Алгебраический поиск пересечений. Пересечение с осью у: /(0) = —(О)2 + 5(0) + 3 = 3, пересечение с осью х: f(x) = 0. — х2 + 5х + 3 = 0 —b ± Vb2 — 4ас х = 2а Квадратное уравнение Решение квадратного уравнения Таким образом, х = Ч5)±^2;4(-1)(3) = ~5±У^ = 0,54 „ 5,54 2( 1) —2 3. Построение графика с помощью графической утилиты.
Глава 1. Базовые элементарные функции 93 10 -10 4. Поиск пересечений с осями х и у с помощью графической утилиты. Пересечение с х: -0.54 Пересечение с х: 5.54 Пересечение с у: 3 5. Графическое решение неравенства —х2 + 5т + 3 0. Квадратичное неравенство —х2 + 5т + 3 0 выполняется для тех значе¬ ний х, при которых функция /(ж) = —х2 + 5т + 3 (см. задачи 1 или 4) лежит выше оси х. Это условие выполняется, если значение х находится между двумя пересечениями с осью х (они найдены при решении задач 2 и 3). Поэтому множество решений квадратного неравенства определяется как -0,54 х 5,54 или [—0,54; 5,54]. 6. Поиск решения уравнения —т2 + 5х + 3 = 4 с помощью утилиты. ю -10 —х2 + 5х + 3 = 4 при х = 4,79 —х2 + 5т + 3 = 4 при х = 0,21 Упражнение 1.20. 1. Постройте график функции д (х) = 2х2 — 5х — 5 в декартовой системе координат. 2. Найдите пересечения этого графика с осями декартовой системы координат (с точностью до двух десятичных знаков).
94 Часть I. Элементарные функции 3. Постройте график функции д(х) — 2х2 — 5х — 5 с помощью графической утилиты. Ч 4. Найдите пересечения этого графика с осями декартовой системы координат с помощью графической утилиты, меняя масштаб изображения (с точностью до двух десятичных знаков). 5. Решите неравенство 2х2 — 5х — 5 > 0 графически с точностью до двух десятичных знаков, используя результаты решения задач 1 и 2 или 3 и 4. 6. Решите уравнение 2х2 — 5х — 5 = — 3 графически с точностью до двух де¬ сятичных знаков, используя команду поиска корней уравнения графической утилиты. ■ Задание 1.12. Сколько раз график квадратичной функции может пересекать ось х? Какое количество раз график квадратичной функции может пересекать ось у? Объясните свой ответ. ■ Свойства квадратичных функций и их графиков Для изучения большинства свойств квадратичной функции общее выражение вида /(ж) = ах2 + Ьх + с, а О нужно привести к стандартной форме /(х) = а(х — h)2 + к. Как правило такое преобразование выполняется методом выделения полного квадрата (см. приложение А.9, т. 2). Давайте сначала рассмотрим принципы ее выполнения на конкретном примере, а затем обобщим полученные результаты. Рассмотрим квадратичную функцию f(x) = —2х2 + 16ж - 24. (1-14) Выделим полный квадрат (т.е. перепишем функцию в стандартной форме). /(ж) = —2х2 + 16ж - 24 = = —2(х2 - 8х) - 24 = = —2(х2 - 8х + ?) - 24 = Вынесем за скобки множитель при а?2 в первых двух слагаемых
Глава 1. Базовые элементарные функции 95 = —2(х2 - 8ж + 16) -24 + 32 = -2(ж - 4)2 + 8. Прибавим число 16 к выражению в скобках, чтобы дополнить его до полного квадрата. Поскольку за круглыми скобками есть множитель —2, то на самом деле мы добавили —32. Чтобы полное выражение для функции не изменило своего значения, нужно прибавить 32 за скобками. На этом преобразование завершено и может быть проверено подстановкой какого-либо числа. В результате /(х) =-2(я - 4)2 + 8. (1.15) Если х = 4, то — 2(х — 4)2 = 0 и /(4) = 8. Для других значений х к числу 8 добавляется отрицательное число —2(х — 4)2, и результат вычисления становится меньше. (Подумайте над этим утверждением.) Итак, число /(4) = 8 является максимальным значением функции /(х) при всех возможных значениях х. Это очень важный результат! Кроме того, если мы выберем какие-либо значения переменной х на одинаковых расстояниях от числа 4, то получим одинаковые значения функции. Например, подставим в /(х) значения х = 3 и х = 5, которые находятся на единичном расстоянии от х = 4. Соответствующие значения функции равны. /(3) = -2(3 - 4)2 + 8 = 6, /(5) = -2(5 - 4)2 + 8 = 6. Это значит, что данная функция симметрична относительно вертикальной прямой х = 4. Другими словами, если перевернуть график уравнения (1.14) относительно прямой х = 4, то кривая перейдет сама в себя. Для иллюстрации этих результатов, на рис. 1.25 построены графики уравнений (1.14) и (1.15) и прямая х = 4 в одной системе координат. В предыдущем обсуждении показано, что при изменении х в направлении слева направо функция f(x) возрастает в области (оо, 4] и убывает в области [4, оо), достигая при этом максимального значения 8. Это позволяет сделать следующий вывод. Область значений f(x) — множество у 8 или (-оо, 8]. В общем случае график квадратичной функции является параболой, симметричной относительно прямой, параллельной оси у. Самая высокая или самая низкая точка параболы (какая имеется) называется вершиной. Максимальное или
96 Часть I. Элементарные функции Рис. 1.25. График квадратичной функции минимальное значение квадратичной функции всегда достигается на вершине параболы. Линия симметрии проходит через вершину и называется осью параболы. В примере, который мы только что рассматривали, осью параболы является прямая х = 4, а вершиной — точка (4, 8). Рассмотрим преобразования (см. раздел 1.2), позволяющие получить уравнение, записанное в стандартной форме, из базовой квадратичной функции /(х) = —2х2 + 16х - 24 = —2(х - 4)2 4- 8. Легко видеть, что график /(х) = — 2х2 + 16х — 24 получается из графика функции д(х) = х2 в результате вертикального растяжения в 2 раза, отражения относи- тельно оси х, сдвига вправо на 4 единицы и вверх на 8 единиц, как показано на рис. 1.26. Перечислим характеристики, которые мы определили, анализируя функцию /. ■ Вершина параболы. ■ Ось параболы. ■ Максимальное значение /(х). ■ Область значений функции /. ■ Связь между графиками д(х) = х2 и /(т) = — 2х2 + 1бх — 24. Давайте посмотрим, что произойдет с графиком функции /(ж) = а(х — К)2 + к при изменении констант a, h и к. Задание 1.13. 1. Пусть а = 1 и h = 5. Постройте графики функции /(х) = а(х — К)2 + к для значений к = —4, 0 и 3 в одной и той же системе координат. Объясните, как меняется график f при изменении константы к.
Глава 1. Базовые элементарные функции 97 Рис. 1.26. График функции /, полученный в результате преобразования графика функции д 2. Пусть а = 1 и к = 2. Постройте графики функции /(ж) = а(х — h)2 + к для значений h = —4, 0 и 5 в одной и той же системе координат. Объясните, как меняется график функции f при изменении константы h. 3. Пусть h = 5 и к = —2. Постройте графики функции f(x) = а(х — /г)2 -I- к ддя а = 0,25; 1 и 3 в одной и той же системе координат. Постройте график функции f для а = 1, —1 и —0,25 в одной и той же системе координат. Объясните, как меняется график функции f при изменении константы а. *4 4. Обсудите результаты решения задач 1-3, выполнив их с помощью графической утилиты в стандартном окне вывода. ■ Резюмируем наше обсуждение общих свойств квадратичных функций. Свойства квадратичной функции и ее графика Квадратичная функция приводится к стандартной форме методом выделения полного квадрата /(ж) = ах2 + Ьх + с = = а(х — К)2 + к, а / 0, и обладает следующими свойствами.
98 Часть I. Элементарные функции Ветви направлены вверх Ветви направлены вниз 2. Вершина находится в точке (h, к) (парабола убывает по одну сторону от вершины и возрастает по другую). 3. Ось (симметрии): х = h (параллельна оси у) 4. Значение f(Ji) = k при а > 0 является минимумом, а при а < 0 — максимумом. 5. Область определения: все действительные числа. Область значений: (—оо, к] при а < 0 и [к, оо) при а > 0. 6. График /(ж) получается из графика д(х) = ах2 сдвигом по горизонтали на h единиц и по вертикали на к единиц. Пример 1.21 (Анализ квадратичной функции). Задана квадратичная функция /(ж) = 0,5х2 - 6х + 21. 1. Запишите /(х) в стандартной форме. 2. Найдите вершину У(х), а также ее максимум или минимум и область значений. 3. Укажите последовательность графических преобразований (см. раздел 1.2), с помощью которых график функции /(ж) может быть получен из графика функции д(х) = х2. 4. Постройте график функции f в декартовых координатах. ** 5. Постройте график f с помощью графической утилиты, используя подходящее окно вывода. *5 6. Графически найдите вершину, максимум или минимум функции /, используя команду вычисления минимума. Укажите область значений функции /.
Глава 1. Базовые элементарные функции 99 Решение. 1. Выделение полного квадрата и приведение к стандартной форме. /(ж) = 0,5ж2 — бж 4- 21 = = 0,5(ж2 - 12ж 4- ?) 4- 21 = = 0,5(ж2 - 12ж + 36) + 21 - 18 = = 0,5(я - б)2 4-3. 2. Из записи функции в стандартной форме видно, что h = 6, к = 3. Поэтому вершина: (6, 3); минимум: /(6) = 3: область значений: у 3 или [3, оо). 3. График функции /(ж) = 0,5(ж — б)2 4- 3 получается из графика функции д(х) = х2 сжатием по вертикали в 2 раза, сдвигом вправо на 6 единиц и вверх на 3 единицы. 4. Построение графика в декартовой системе координат. Л») 5. Построение графика с помощью графической утилиты. 6. Поиск вершины, минимума и области значений с помощью графической утилиты. Вершина: (6, 3); минимум: /(6) = 3: область значений: у 3 или [3, оо).в
100 Часть I. Элементарные функции Упражнение 1.21. Задана квадратичная функция /(ж) = — 0,25л;2 — 2х + 2 1. Запишите функцию /(ж) в стандартной форме. 2. Найдите вершину /(х), а также ее максимум или минимум и область значений. 3. Укажите последовательность графических преобразований (см. раздел 1.2), с помощью которых график функции /(ж) можно получить из графика функции д(х) = х2. 4. Постройте график f в декартовых координатах. 5. Постройте график f с помощью графической утилиты, используя подходящее окно вывода. 6. Графически найдите вершину, максимум или минимум функции /, используя программу вычисления минимума. Укажите область ее значений. ■ Решение практических задач Е Пример 1.22 (Максимальный доход). Вернемся к примеру 1.7 раздела 1.1. Напомним, что финансовый отдел компании, производящей фотоаппараты, получил следующую функцию зависимости цены от спроса и функцию дохода. р(х) = 94,8 - 5х; R(x) = хр(х) = х(94,8 — 5х); Функция зависимости цены от спроса Функция дохода где р(х) — оптовая цена фотоаппарата, х — объем реализуемой продукции (млн. шт.) и R(x) — jxqxqr (млн. долл.). Обе функции определены на интервале 1 х < 15. 1. Найдите условие максимального дохода с точностью до тысячи штук. Чему равен максимальный доход? Решите задачу методом выделения полного квадрата. 2. Чему равна оптовая цена фотоаппарата при условии максимального дохода? Ответ округлите до ближайшего целого числа. Ч 3. Постройте график функции дохода с помощью графической утилиты в под¬ ходящем окне вывода.
Глава 1. Базовые элементарные функции 101 4. Найдите условие максимального дохода с помощью графической утилиты (с точностью до тысячи штук). Чему равен максимальный доход? Решите задачу графически, используя программу поиска максимума. Решение. 1. Алгебраическое решение. Л(х) = ж(94,8 — 5а;) = = —5г2 + 94,8а; = = —5(а;2 - 18,96а; + ?) = = —5(х2 - 18,96а; + 89,8704) + 449,352 = = —5(ж - 9,48)2 + 449,352. Доход достигает максимума 449,352 млн. долл. (449 352 000 долл.) при х = = 9,480 млн. фотоаппаратов (9 480 000 фотоаппаратов). 2. Найдем оптовую цену фотоаппарата. Подставим значение 9,480 в функцию зависимости цены от спроса. р(х) = 94,8 — 5а:, р(9,480) = 94,8 - 5 • 9,480 = 47 долл. “ 3. Построение графика функции дохода с помощью графической утилиты. 4. Графическое решение с помощью утилиты. Выпуск 9,480 миллионов фотоаппаратов (9480000 фотоаппаратов) приносит максимальный доход 449,352 млн. долл. (449 352 000 долл.). ■
102 Часть I. Элементарные функции Финансовым отделом компании (см. пример 1.22) с помощью статистических методов (см. дополнительную задачу 1.7 раздела 1.1) получена следующая функция затрат. С(х) = 156 + 19,7х\ Функция затрат где С(х) — затраты (млн. долл.) на производство и продажу х фотоаппаратов (млн. шт.). 1. Используя эту функцию затрат и функцию дохода из примера 1.22, найдите уравнение функции прибыли. 2. Найдите условие максимальной прибыли с точностью до тысячи штук. Чему равна максимальная прибыль? Решите задачу методом выделения полного квадрата. 3. Чему равна оптовая цена при условии максимальной прибыли? Ответ округлите до ближайшего целого числа. Ч 4. Постройте график функции прибыли с помощью графической утилиты в подходящем окне вывода. Ч 5. Найдите условие максимальной прибыли с помощью графической утилиты (с точностью до тысячи штук). Чему равна максимальная прибыль? Решите задачу графически, используя команду поиска максимума. ■ И Пример 1.23 (Анализ безубыточности производства). Используем функцию дохода из примера 1.22 и функцию затрат из упражнения 1.22. R{x) = хр(х) = х(94,8 — 5ж); Функция дохода С(х) = 156 + 19,7т. Функция затрат Обе функции определены на интервале 1 < х < 15. 1. Постройте графики обеих функций в одной и той же системе координат. 2. Точки безубыточности производства определяются условием R(x) = С(х). Найдите эти точки с точностью до тысячи штук. Ч 3. Постройте графики обеих функций с помощью графической утилиты в одном окне вывода. Ч 4. Найдите точки безубыточности графически, используя команду поиска корней уравнения (ответ округлить до тысячи штук). 5. Напомним, что производство убыточно, если R(x) < С(ж), и прибыльно, если R(x) > С(х). При каком количестве фотоаппаратов производство будет убыточным? Прибыльным?
Глава 1. Базовые элементарные функции 103 Решение. 1. Построение графиков функций: 2. Найдем значение х, при котором R(x) = С(х): ж (94,8 — 5а;) = 156 + 19,7х, —5а;2 + 75,1а; - 156 = 0. -75,1 ± л/75,12 — 4(—5)(—156) X = —-—-г = Решение квадратного уравнения 2(-5) -75,1 ± х/2520,01 -10 х = 2,490; или 12,530. Компания будет безубыточной при выпуске от 2,490 и до 12,530 млн. фотоаппаратов. 3. Построение с помощью графической утилиты.
104 Часть I. Элементарные функции 4. Графическое решение. Компания будет безубыточной при выпуске от 2,490 и до 12,530 млн. фотоаппаратов. 5. Используя результаты п. 1 и 2 или 3 и 4, получаем следующее. Убыточное производство: 1 < х < 2,490 или 12,530 < х С 15; Прибыльное производство: 2,490 < х < 12,530. ■ И Упражнение 1.23. Запишем функцию прибыли через функции дохода и затрат (см. пример 1.23). Р(х) = R(x) — С(х) = —5х2 + 75,1х — 156. Функция прибыли Область определения: 1 х 15. 1. Постройте график функции прибыли в декартовой системе координат. 2. Точки безубыточности производства определяются условием Р(ж) = 0. Найдите эти точки для первых нескольких тысяч фотоаппаратов. 41 3. Постройте график функции прибыли с помощью графической утилиты в подходящем окне вывода. 4. Найдите точки безубыточности производства графически (с точностью до тысячи штук), используя подходящую команду. 5. Производство убыточно, если Р(ж) < 0, и прибыльно, если Р(т) > 0. При каком объеме выпуска фотоаппаратов производство будет убыточным? При каком объеме выпуска фотоаппаратов производство будет прибыльным? ■ Ответы к упражнениям 1.20. 1) -ю
Глава 1. Базовые элементарные функции 105 2) Точки пересечения с осью х: —0,77 и 3,27; пересечение с осью у: —5. 3) “i 4) Пересечения с осью х: —0,77 и 3,27; пересечение с осью у: —5. 5) х —0,77 или х 3,27; или (—оо; —0,77] или [3,27; оо). 6) х = -0,35; 2,85. 1.21. 1) /(ж) = —0,25(я? + 4)2 + 6. 2) Вершина: (—4, 6); максимум: /(—4) = 6: область значений: у < 6 или (—оо, 6]. 3) График функции/(ж) = —0,25(ж+4)2+6 получается из графика функции <?(ж) = х2 вертикальным сжатием в 4 раза, отражением относительно оси х, сдвигом влево на 4 единицы и вверх на 6 единиц. 4) у ю-- “ 6) Вершина: (—4, 6); максимум: /(—4) = 6: область значений: у < 6 или (—оо, 6]. 1.22. 1) Р(ж) = Я(ж) - С(ж) = -5ж2 + 75,1х - 156 2) Р(ж) = Я(ж) - С(ж) = -5(ж - 7,51)2 + 126,0005. Максимальная прибыль 126,001 млн. долл, достигается при выпуске 7,510 млн. фотоаппаратов.
106 Часть I. Элементарные функции 3) р(7,51) = 57 долл. Максимальная прибыль 126,001 млн. долл, достигается при выпуске 7,510 миллионов фотоаппаратов. (Заметим, что максимум дохода и максимум прибыли не совпадают.) 2) х = 2,490 или 12,530 млн. фотоаппаратов. Ъ 4) х = 2,490 или 12,530 миллионов фотоаппаратов. 5) Убыточное производство: 1 < х < 2,490 или 12,530 < х < 15. Прибыльное производство: 2,490 < х < 12,530.
Глава 1. Базовые элементарные функции 107 Практикум 1.4 А В задачах 1-4 выделите полный квадрат и приведите к стандартной форме квадратичные функции, 1. f(x) = х2 — 4х + 3. 2. д(х) = х2 — 2х — 5. 3. т(х) = -х2 + 6я - 4. 4. п(х) = —х2 + &г — 9. В задачах 5-8 коротко объясните (на словах), какие преобразования нужно применить к функции д(х) = х2, чтобы построить графики из пп. 1-4. *5. f(x) = х2 - 4х + 3. *6. д(х) = х2 - 2х — 5. *7. т(х) = —х2 + 6я — 4. *8. п(х) = —х2 + 8х — 9. 9. Определите, какие из приведенных ниже уравнений соответствуют графикам функций /, д, т и п на рисунке 1.27. а) 7/ = -(я + 2)2 + 1. б) 2/ = (х-2)2-1. в) у = (х + 2)2 - 1. г) у = -(х - 2)2 + 1. Рис. 1.27. Иллюстрация к задаче 9 10. Определите, какие из приведенных ниже уравнений соответствуют графикам функций /, д9 т и п на рисунке 1.28. Рис. 1.28. Иллюстрация к задаче 10 а) у = (х - З)2 - 4. б) з/=-(я + З)2 + 4. в) У = “ З)2 + 4. г) у = (х + З)2 - 4.
108 Часть I. Элементарные функции Для функций, указанных в задачах 11—14 (см. задачи 9 и 10), найти указанные характеристики, округляя результат до ближайшего целого числа: 1) точки пересечения; 2) вершина; 3) максимум или минимум; 4) область значений; 5) область возрастания; 6) область убывания. 11. Функция п на рисунке к задаче 9. 12. Функция т на рисунке к задаче 10. 13. Функция f на рисунке к задаче 9. 14. Функция g на рисунке к задаче 10. В задачах 15-18 для каждой функции найти указанные характеристики: 1) точки пересечения; 2) вершина; 3) максимум или минимум; 4) область значений. 15. /(х) = -(х - З)2 + 2. 16. д(х) = -(х + 2)2 + 3. 17. т(х) = (х + I)2 — 2. 18. п(х) = (х — 4)2 — 3. Б В задачах 19-22 найдите уравнение вида у = а(х—h)2+k, соответствующее каждому из графиков (здесь константа а равна 1 или —1, h и к — целые числа).
Глава 1. Базовые элементарные функции 109 В задачах 23-28 представьте каждое из уравнений в стандартной форме. После этого найдите следующие характеристики: 1) точки пересечения; 2) вершина; 3) максимум или минимум; 4) область значений. 23. f(x) = х2 —8х + 12. 24. д(х) = о?2 — бо: + 5. 25. г(х) = —4х2 + 16# — 15. 26. s(x) = —4а:2 — 8х — 3. 27. и(х) = 0,5а:2 — 2х + 5. 28. v(x) = 0,5а:2 + 4х + 10. 29. Пусть /(ж) = 0,3а:2 —х—8, Решите каждое из уравнений с помощью графической утилиты с точностью до двух десятичных знаков. а) /(ж) = 4. б) /(ж) = -1. в) /(ж) = -9. * 30. Пусть р(ж) = —0,6ж2 4- Зх + 4. Решите каждое из уравнений с помощью графической утилиты с точностью до двух десятичных знаков. а) 9(.х) = “2. б) д(х) = 5. в) д(х) = 8. *31. Объясните, при каких условиях квадратичная функция имеет в точности один действительный нуль. * 32. Объясните, при каких условиях квадратичная функция не имеет действительных нулей. В В задачах 33-36 представьте каждое из уравнений в стандартной форме. После этого найдите следующие характеристики: 1) точки пересечения; 2) вершина; 3) максимум или минимум; 4) область значений. 33. д(х} = 0,25а:2 - 1,5а: - 7. 34. т(х) = 0,2а:2 - 1,6а: - 1. 35. Да;) = —0,12а:2 + 0,96а: 4-1,2. 36. п(а?) = -0,15а:2 - 0,9а: + 3,3. В задачах 3 7-42 решите уравнения и неравенства с помощью графической утилиты с точностью до двух десятичных знаков. 37. 2 - 5а: - х2 = 0. 38. 7 + За? - 2х2 = 0. 39. 1,9а:2 - 1,5а: - 5,6 < 0. 40. 3,4 + 2,9а: - 1,1а:2 > 0. 41. 2,8 + 3,1а: - 0,9а:2 С 0. 42. 1,8а:2 - 3,1а: - 4,9 > 0. 43. Известно, что f — квадратичная функция с минимумом f(x) = /(2) = 4. Исходя из этих данных, найдите ось, вершину, область значений и пересечения этой функции с осью х, 44. Известно, что f — квадратичная функция с максимумом f(x) = f (—3) = —5. Исходя из этих данных, найдите ось, вершину, область значений и пересечения этой функции с осью х.
110 Часть I. Элементарные функции В задачах 45-48 выполните следующие задания. 1) Постройте графики функций f и g в одной системе координат. 2) Решите уравнение f(x) = g(x) алгебраически с точностью до двух десятичных знаков. 3) Решите неравенство f(x) > g(x), используя результаты решения задач 1 и 2. 4) Решите неравенство f(x) < g(x), используя результаты решения задач 1 и 2. 45. /(я) = -0,4я(я — 10); g(x) = 0,Згг + 5; 0 х 10. 46. f(x) = — 0,7я(я — 7); д(х) = 0,5я -I- 3,5; 0 х 7. 47. /(ж) = —0,9я2 + 7,2я; д(х) = 1,2я + 5,5; 0 х 8. 48. f(x) = —0,7я2 4- 6,Зя; д(х) = 1,1я + 4,8; 0 х 9. * 49. Приведите пример квадратичной функции, у которой нет действительных нулей. Объясните, как должен быть расположен график такой функции относительно оси х. * 50. Приведите пример квадратичной функции, у которой есть в точности один дей¬ ствительный нуль. Объясните, как должен быть расположен график такой функции относительно оси х. Применение математики Каждую из задач решите с помощью графической утилиты, используя команды поиска нулевых значений, максимумов и минимумов. Экономика и бизнес 51. Пробег автомобильных шин. Производителем автомобильных шин были собраны данные, описывающие зависимость давления в шине х (фунтов на кв. дюйм) от пройденного ею расстояния (тыс. миль), приведенные в следующей таблице. ж, фунт/кв. дюйм Пробег, тыс. миль 28 45 30 52 32 55 34 51 36 47 Взаимосвязь между собранными производителем статистическими данными описывается следующей функцией. /(х) = —0,518х2 + 33,3х - 481. а) Заполните следующую таблицу, округляя вычисляемые значения до первого десятичного знака.
Глава 1. Базовые элементарные функции 111 х, фунт/кв. дюйм Пробег, тыс. миль f(x) 28 45 30 52 32 55 34 51 36 47 б) Изобразите график функции /(ж) и данные таблицы в одной системе координат. в) Методом прямой подстановки в исходную функцию найдите (с точностью до двух десятичных знаков) значение расстояния, при прохождении которого давление будет равно 31 фунту на кв. дюйм. Какое расстояние соответствует давлению 35 фунтов на кв. дюйм. *г) Дайте словесное описание зависимости давления от расстояния. 52. Производство автомобилей. В следующей таблице приведены данные о доле (в процентах) компании Ford Motors Company на розничном рынке легковых автомобилей в Соединенных Штатах. WWW Год Доля на рынке, % 1975 23,6 1980 17,2 1985 18,8 1990 20,0 1995 20,7 Взаимосвязь между указанными данными описывается следующей функцией, /(аг) = 0,04а:2 - 0,8а: + 22, где значение х = 0 соответствует 1975 году. а) Заполните следующую таблицу: х Доля на рынке, % /(х) 0 23,6 5 17,2 10 18,8 15 20,0 20 20,7 б) Изобразите график функции /(х) и данные таблицы в одной системе координат. в) Оцените долю компании Ford на американском рынке легковых автомобилей в 2000 году, вычислив его с помощью исходной функции. Сделайте прогноз на 2005 год. *г) Кратко опишите положение компании Ford на американском рынке легковых автомобилей с 1975 по 1995 гг..
112 Часть I. Элементарные функции 53. Доход. Вернемся рассмотрению задач 85 и 87 (см. практикум 1.1). Отделом маркетинга компании, специализирующийся на производстве микросхем памяти для персональных компьютеров, получены следующие функции дохода и зависимости цены от спроса: р(х) = 75 — Зх, Функция зависимости цены от спроса R(x) = хр(х) = х(75 — Зх). Функция дохода Здесь р(х) — оптовая цена в долларах, х — количество микросхем памяти в миллионах штук, Я(х) — доход в миллионах долларов. Обе функции определены в области 1 х 20. а) Постройте график функции дохода в декартовой системе координат; б) Найдите объеме производства с максимальным доходом. Чему равен максимальный доход? в) Какой будет цена микросхемы памяти при максимальном доходе? 54. Доход. Вернемся к рассмотрению задач 86 и 88 (см. практикум 1.1). Отделом маркетинга компании, специализирующийся на производстве ноутбуков, получены следующие функции дохода и зависимости цены от спроса: р(х) = 2000 — 60х Функция зависимости цены от спроса R(x) = хр(х) = х(2000 — 60х) Функция дохода Здесь р(х) — оптовая цена (долл.), х — количество ноутбуков в тысячах штук, Я(х) — доход (тыс. долл.). Обе функции определены в области 1 х 25. а) Постройте график функции дохода в декартовой системе координат; б) Найдите объем производства (округляя с точностью до 100 единиц) с максимальным доходом. Чему равен максимальный доход? в) Какова цена ноутбука (округляя с точностью до доллара) при максимальном доходе? 55. Анализ безубыточности. Используем функцию дохода из задачи 53 и функцию затрат из задачи 89 (см. практикум 1.1). Я(х) = х(75 — Зх) С(х) = 125 + 1бх Функция дохода Функция затрат
Глава 1. Базовые элементарные функции 113 Здесь х — количество микросхем памяти (млн. шт.), R(x), С(х) — доходы и затраты (млн. долл.). Обе функции определены в области 1 < х < 20. а) Постройте графики обеих функций в одной и той же декартовой системе координат. б) Найдите точки безубыточности производства (округляя с точностью до 1000 единиц). в) Чему равен объем производства в этом случае? Каков максимальный доход? 56. Анализ безубыточности. Используем функцию дохода из задачи 54 и функцию затрат из задачи 90 (см. практикум 1.1). R(x) = #(2000 — 60х) Функция дохода С(х) = 4000 + 500х Функция затрат Здесь х — количество ноутбуков (тыс. шт.), R(x), С(х) — доходы и затраты (тыс. долл.). Обе функции определены в области 1 х < 25. а) Постройте графики обеих функций в одной и той же декартовой системе координат. б) Найдите точки безубыточности производства. в) При каком объеме выпуска ноутбуков производство будет убыточным? При каком объеме выпуска ноутбуков производство будет прибыльным? 57. Анализ прибыльности. Используем функцию дохода из задачи 53 и функцию затрат из задачи 89 (см. практикум 1.1). R(x) = х(75 — Зх) Функция дохода С(х) = 125 + 16х Функция затрат Здесь х — количество микросхем памяти (млн. шт.), Я(х), С(х) — доходы и затраты (млн. долл.). Обе функции определены в области 1 х < 20. а) Найдите функцию прибыли Р(х). Постройте графики функций R, С и Р в одной и той же декартовой системе координат. *б) Проанализируйте взаимосвязь между точками пересечения графиков R и С и точками пересечения графика Р с осью х. в) Найдите точки пересечения графика Р с осью х (с точностью до тысячи штук). Найдите точки безубыточности производства с точностью до тысячи долларов. *г) Будет ли максимальный доход достигаться при том же объеме производства, что и максимальная прибыль (см. графики, построенные при решении задачи а. Могут ли быть одинаковыми максимальный доход и максимальная прибыль? Аргументируйте ответ. д) Проверьте вывод, полученный при решении задачи г, вычислив объем производства (с точностью до тысячи штук), обеспечивающий получение максимальной прибыли. Найдите максимальную прибыль (с точностью до тысячи долларов) и сравните результат с решением задачи 53.6.
114 Часть I. Элементарные функции 58. Анализ прибыльности. Используем функцию дохода из задачи 54 и функцию затрат из задачи 90 (см. практикум 1.1). Я(х) = х(2000 - 60х) С(х) = 4000 4- 500х Функция дохода Функция затрат Здесь х — количество компьютеров (тыс. шт.), Я(х), С(х) — доходы и затраты (тыс. долл.). Обе функции определены в области 1 < х < 25. а) Найдите функцию прибыли Р(х) Постройте графики функций R, С и Р в одной декартовой системе координат. *б) Проанализируйте взаимосвязь между точками пересечения графиков R и С и точками пересечениями графика Р с осью х. в) Найдите точки пересечения графика Р с осью х (с точностью до тысячи штук). Найдите точки безубыточности производства. *г) Будет ли максимальный доход достигаться при том же объеме производства, что и максимальная прибыль (см. графики, построенные при решении задачи а). Могут ли быть одинаковыми максимальный доход и максимальная прибыль? Аргументируйте ответ. д) Проверьте вывод, полученный при решении задачи г, вычислив объем производства обеспечивающий получение максимальной прибыли. Найдите максимальную прибыль (с точностью до тысячи долларов) и сравните результат с решеним задачи 54.6. Биологические науки 59. Медицина. Французский врач Пуазейль (Poiseulle) первым открыл, что по центру артерии кровь течет быстрее, чем вдоль стенок. Экспериментальным путем было установлено, что скорость крови v (см/с) в точке, удаленной на х сантиметров от центра артерии (см. рисунок), можно вычислить, используя уравнение v = /(x) = 1000(0,04-X2), 0 < х 0,2. Найдите расстоянии от центра артерии, где скорость течения крови равна 20 см/с. Ответ округлите до двух десятичных знаков. Артерия Рис. 1.29. Иллюстрация к задачам 59 и 60 60. Медицина. Вернемся к рассмотрению задачи 59. Найдите расстояние от центра артерии, где скорость течения крови равна 30 см/с. Ответ округлите до двух десятичных знаков.
Глава 1. Базовые элементарные функции 115 Ключевые слова, основные обозначения и формулы 1.1. Функции. Декартовы и прямоугольные координаты; координатные оси; квадранты; упорядоченные пары; координаты; абсцисса; ордината; основная теорема аналитической геометрии; решение и множество решений уравнения с двумя переменными; график уравнения с двумя переменными; построение графика по точкам; функция; область определения; область значений; функции, определяемые уравнениями; входящие значения; результат; независимая переменная; зависимая переменная; определение функции; разностное отношение; функция затрат; функция зависимости цен от спроса; функция дохода; функция прибыли; У (ж); С(х) = а + Ьх] р = т — пх\ R = хр\ P = R-С. 1.2. Элементарные функции: графики и преобразования. Шесть базовых элементарных функций: тождественная функция, абсолютное значение, квадрат, куб, квадратный корень, кубический корень; горизонтальный перенос; вертикальный перенос; параллельный перенос; отражение относительно оси х\ растяжение по вертикали; растяжение по горизонтали; кусочно-определенные функции; разрывы. /(®) = х; д(х) = |х|; h(x) = х2; т{х) = а:3; п(я) = р(х) = \/х. 1.3. Линейные функции и прямые линии. Пересечение с осью х; пересечение с осью у; линейная функция; постоянная функция; функция первого порядка; график линейной функции; график линейного уравнения двух переменных; стандартная форма уравнения прямой; уравнения вертикальных и горизонтальных прямых; наклон; уравнение прямой с угловым коэффициентом; уравнение прямой, проходящей через указанную точку, с заданным наклоном; точка равновесия; равновесная цена; равновесное количество. /(ж) = тх + Ь, т 0; f(x) = b, х = 6; Ах + By = С; у = тх + Ь; у-У1 = т(х-х1); ™ _ Уз ~У1 „ _L„ т = , Х2 Т2 ху. Х2 Xi 1.4. Квадратичные функции. Квадратичная функция; парабола; поиск пересечений; стандартная форма; вершина; ось; максимум или минимум; область значений; точки безубыточности. f(x) = ах2 + bx + с = а(х — h)2 + fc, а 0.
116 Часть I. Элементарные функции Упражнения для повторения Выполните все упражнения этого обзорного раздела и сравните результаты с ответами, помещенными в конце книги. Ответы ко многим упражнениям на повторение приводятся вместе с номером соответствующего раздела (курсивом). Если у вас возникают затруднения при решении какой-либо задачи, повторите материал соответствующего раздела. А 1. Постройте по точкам график функции у = 5 — х2. Используйте целочисленные значения х от —3 до 3. 2. Укажите, какой из графиков определяет функцию: 3. Для /(х) = 2х — 1 и р(х) = х2 — 2х вычислите следующие значения. а) Л-2) + 5(-1). б) /(0)^(4). ’ /(3)' ’ 9(2)' 4. Для приведенного на рис. 1.30 графика функции f определите величину х или у, округляя результат до ближайшего целого числа. а) 2/ = /(0). б) 4 = /(х). в) у = /(3). г) 3 = f(x). Д) У = f(-6). е) -1 = f(x). 5. Постройте график каждой функции, используя график функции /(ж), приведенный ниже на рис. 1.31. а) у = в) y = f(x-2). б) у = /(ж) + 4. г) У = -/(х + 3) - 3.
Глава 1. Базовые элементарные функции 117 Рис. 1.30. Иллюстрация к задаче 4 Рис. 1.31. Иллюстрация к задаче 5 6. Рассмотрим рисунок 1.32. а) Укажите графики линейных функций с положительным наклоном. б) Укажите графики линейных функций с отрицательным наклоном. в) Укажите графики постоянных функций. Какой у них наклон? г) Укажите графики, которые не представляют функции. Что можно сказать об их наклоне? 7. Напишите уравнение вида у = тх + b для прямой с наклоном —|, которая пересекается с осью у в точке (0; 6). 8. Напишите уравнения вертикальной и горизонтальной прямых, которые проходят через точку (—6, 5). 9. Постройте график функции 2х — Зу = 18. Найдите его наклон и точки пересечения с координатными осями. *10. Выделите полный квадрат и представьте в стандартной форме квадратичную функцию f(x) = ~х2 -|- 4а?. Объясните связь между графиком функции f и графиком у = я2. 11. Определите, какое из приведенных ниже уравнений соответствует графикам функций /, д, т или п на рис. 1.33. а) у = (х - 2)2 - 4. б) у = -(х + 2)2 + 4. в) у = -{х - 2)2 -1-4. г) у= (я + 2)2-4. Рис. 1.32. Иллюстрация к задаче 6 Рис. 1.33. Иллюстрация к задаче 11
118 Часть I. Элементарные функции б) вершина. г) область значений. е) область убывания. 12. Для функции /, изображенной на рис. 1.33 в задаче 11, найдите все основные характеристики квадратичной функции (округляя результаты до ближайших целых чисел): а) точки пересечения. в) максимум или минимум. д) область возрастания. Б х = -2. У = -5. н- б) г) е) х 2 13. Вычислите, какие из приведенных ниже графиков определяют линейную функцию, а какие постоянную, а) 2х — Зу = 5. в) у = 4 - Зх. jf) х = Зу + 5. 14. Найдите области определения следующих функций. \ г / \ 2х б / \ Зх а) Л®) = R- б) 9^ = л х£ — х — о у 5 — х * 15. Функция д(х) определена формулой д(х) = 2х — 3\/х. Дайте словесное определение этой функции. 16. Запишите функцию f(x) = 4х2 + 4х — 3 в стандартной форме. Найдите точки ее пересечения с осями, вершину, максимум или минимум, область значений. *17. Опишите свойства графиков х = — 3 и у = 2. Постройте оба графика на одном и том же рисунке в декартовой системе координат. 18. Пусть /(х) = 0,4х(х 4- 4)(2 - х). а) Постройте график этой функции по точкам, используя нечетные значения переменной х от —3 до 3. Затем дополните график с помощью графической утилиты. Определите количество решений следующих уравнений. *б) Л*) = 3, /(х) = 2, /(ж) = 1. в) Найдите приблизительные решения уравнений п. б с помощью графической утилиты (с точностью до двух десятичных знаков). 19. Вычислите для /(х) = 3 — 2х. 20. Вычислите для /(#) = х2 - Зх 4-1. *21. Объясните, как получить график т(х) = — |х — 4| преобразованием графика У= И- *22. Объясните, как получить график д(х) = 0,Зх3 4- 3 преобразованием графика У = х3. * 23. График, приведенный на рис. 1.34, получен в результате преобразования графика у = х2. Опишите словесно эти преобразования и найдите уравнение для графика на рис. 1.34.
Глава 1. Базовые элементарные функции 119 24. График функции f был получен в результате вертикального растяжения графика у = у/х в два раза и сдвигов — влево на три единицы и вниз на одну единицу. Постройте график функции f в области — 5 х 5 и — 5 у С 5. 25. Постройте график функции /(*) = { —х — 2; 0,2х2; если х < О, если х > 0. 26. Напишите уравнение прямой, проходящей через заданную точку с данным наклоном. Окончательный ответ представьте в виде у = тх + Ь. 2 а) т = --;(-3, 2). б) т = 0;(3, 3). о 27. Напишите уравнение прямой, проходящей через две заданные точки. Окончательный ответ представьте в виде Ах + By = С. а) (-3,5), (1,-1). б) (-1,5), (4, 5). в) (—2, 7), (—2,—2). 28. Для приведенного ниже графика (рис. 1.35) напишите уравнение в форме у = = а(х — h)2 + fc, где константа а равна —1 или 1, h и к — целые числа. Рис. 1.34. Иллюстрация к задаче 23 Рис. 1.35. Иллюстрация к задаче 28 29. Задана функция f(x) = — 0,4я2 + 3,2# + 1,2. Не используя графика, найдите алгебраически следующие характеристики с точностью до одного десятичного знака. а) точки пересечения. б) вершина. в) максимум или минимум. г) область значений. 30. Постройте график функции /(z) = —0,4я2 + 3,2# +1,2 с помощью графической утилиты и найдите следующие характеристики с точностью до одного десятичного знака (подбирая подходящий масштаб изображения). а) точки пересечения. б) вершина. в) максимум или минимум. г) область значений. В *31. График, приведенный на рис. 1.36, получен в результате преобразования графика у = Опишите словесно эти преобразования и найдите уравнение для графика на рис. 1.36.
120 Часть I. Элементарные функции Рис. 1.36. Иллюстрация к задаче 31 * 32. Постройте графики функций , 1 у = тх -F о и у =—х 4- о т в общей системе координат при неизвестном значении Ь и нескольких отличных значениях m, m 0. Как связаны графики функций между собой? 33. Найдите и упростите выражение для следующих функций: а) /(х) = >/х, б) /(х) = X 34. Задана функция G(x) = 0,3а:2 +1,2а; — 6,9. Не используя график, найдите алгеб¬ раически следующие характеристики с точностью до одного десятичного знака, а) точки пересечения. б) вершина. в) максимум или минимум. г) область значений. д) Области убывания и возрастания. 35. Постройте график функции G(x) = 0,Зх2 4- 1,2ж — 6,9 с помощью графической утилиты и найдите следующие характеристики с точностью до одного десятичного знака (подбирая подходящий масштаб изображения). а) точки пересечения. б) вершина. в) максимум или минимум. г) область значений. д) Области убывания и возрастания. Применение математики Экономика и бизнес 36. Линейная амортизация. Предприятие приобретает офисное оборудование на сумму 12 000 долл. Через 8 лет остаточная стоимость этого оборудования составляет всего 2000 долл. Считается, что в течение срока срока эксплуатации стоимость оборудования изменяется линейно от 12000 до 2000 долл. а) Найдите линейное уравнение, которое выражает стоимость V как функцию времени t. Постройте график этого уравнения в декартовой системе координат. б) Какова остаточная стоимость оборудования после 5 лет эксплуатации?
Глава 1. Базовые элементарные функции 121 37. Сложный процент. Если сумма в 1000 долл, инвестируется под 100г процентов ежегодно, то по истечении 3 лет она вырастет до А = 1000(1 + г)3. а) Постройте график этого уравнения с помощью графической утилиты в интервале 0 г С 0,25 (т.е. для годовой процентной ставки в диапазоне 0 до 25%). б) При какой процентной ставке вклад в размере 1000 долларов вырастет до 1500 долларов за 3 года? Найдите значение г с помощью графической утилиты с точностью до одного десятичного знака, меняя масштаб изображения. 38. Наценка, В магазине спортивных товаров теннисные ракетки, которые стоят 130 долларов, продаются по 208 долларов, а теннисные туфли стоимостью 50 долларов — 80 долл. а) Считая, что величина наценки на товары стоимостью выше 10 долл, рассчитывается согласно линейной функцией, найдите уравнение, которое выражает розничную цену R как функцию затрат С. б) Найдите розничную цену коньков стоимостью 120 долл. в) Какова стоимость лыж, если они продаются по розничной цене 176 долл.? г) Чему равен наклон графика, построенного при решении п. а? Дайте интерпретацию наклона в контексте условий задачи. 39. Спрос. В таблице приведены данные о ежегодном потреблении куриных яиц на душу населения в Соединенных Штатах. WWW Год Потребление куриных яиц, шт. 1980 271 1985 255 1990 233 1995 236 2000 256 Математическая модель для указанных данных выражается функцией /(ж) = 0,28а:2 - 6,5а: + 274, где значение х = 0 соответствует 1980 году. а) Заполните следующую таблицу: Год Потребление куриных яиц, шт f(x) 1980 ?71 1985 255 1990 233 1995 236 2000 256
122 Часть I. Элементарные функции б) Постройте график функции /(х) и отобразите данные таблицы в одной и той же системе координат. в) Используя предложенную функцию, оцените годичное потребление яиц на душу населения в 2005 и 2010 годах. *г) Дайте короткое описание уровня потребления яиц с 1980 по 2000 год, руководствуясь данными таблицы. 40. WWW Тарифы на электроэнергию. В таблице приведены летние тарифы на электроэнергию компании Easton Utilites. а) Постройте кусочно-заданную функцию месячной стоимости электроэнергии S(x) (долл.) при потреблении х кВт в летний период. б) Постройте график функции S(x). Тарифы на электроэнергию (июнь-сентябрь) 3 доллара за первые 20 кВт или меньше 5,70 центов за 1 кВт для следующих 180 кВт 3,46 центов за 1 кВт для следующих 800 кВт 2,17 центов за 1 кВт потребления более 1000 кВт 41. WWW Точка равновесия. Годичное предложение и спрос на ячмень в Соединенных Штатах составляет 410 и 455 млн. бушелей при цене 1,90 долл, за бушель. При увеличении цены до 2,70 долл, за бушель предложение выросло до 430, а спрос упал до 415 млн. бушелей. а) Найдите уравнения функций зависимости спроса и предложения от цены, считая, что они линейны. б) Найдите точку равновесия для рынка ячменя в Соединенных Штатах. 42. Анализ безубыточности. Компания, специализирующаяся на выпуске видеопродукции, планирует снять образовательный видеофильм. Продюсер оценил, что производство фильма обойдется в 84000 долл., а копирование и распространение пленки будет стоить по 15 долл, за копию. Оптовая цена кассеты с фильмом оценена в 50 долл. а) Найдите функции затрат и дохода и постройте их в общей системе координат. б) Определите условие, при котором R = С. Используя результат п. а, найдите условия, при которых R > С и R < С. в) Определите условия, при которых Я = С, Я>СиЯ<С, с помощью графической утилиты. 43. Анализ безубыточности. Исследовательским отделом компании, которая производит средневолновые радиоприемники с часами, получены следующие зависимости спроса, затрат и дохода от цены. р(х) = 50 — 1,25ж Функция зависимости спроса от цены С(х) = 160 + Юж Функция затрат Л(х) = хр(х) — ж(50 — 1,25а;) Функция дохода Здесь х — объем выпускаемой продукции (тыс. шт.), С(х) и R(x) — затраты и доходы (тыс. долл.). Все три функции определены на интервале 1 < х < 40. а) Постройте графики функций затрат и дохода в общей системе координат.
Глава 1. Базовые элементарные функции 123 б) Определите условие, при котором R = С. Далее, используя результаты, полученные при решении п. а, найдите условия, при которых R < С и R > С (с точностью до единицы). в) Найдите максимальный доход (с точностью до тысячи долларов) и объем производства (с точностью до единицы), при котором он реализуется. Какова при этом оптовая цена радиоприемника? 44. Анализ безубыточности. Используем функции затрат и дохода, приведенные в задаче 43. а) Найдите функцию прибыли и постройте ее график с помощью графической утилиты. б) Используя график, определите условия, при которых Р = 0, Р<0иР>0 с точностью до единицы. в) Используя график, найдите максимальную прибыль (с точностью до тысячи долларов) и обеспечивающий ее объем производства (с точностью до единицы). Какова при этом оптовая цена радиоприемника? (Сравните ответ с результатом задачи 43.в). 45. Строительство. Строительная компания планирует оградить секционным забором склад оборудования и материалов, имея при этом 840 футов ограждений. Планируется установить два идентичных прямоугольных забора (см. рис. 1.37). Будем считать, что принято именно такое решение. а) Выразите общую площадь огражденной территории S(x) как функцию, зависящую от переменной х. Рис. 1.37. Иллюстрация к задаче 45 б) Укажите область определения функции S(x) (принимая во внимание очевидные физические ограничения). в) Постройте график функции S в прямоугольной системе координат. *г) Используя график, определите количество и приближенные значения переменной х, при котором огражденная площадь будет равна 25 000 кв. футов. д) С помощью графической утилиты определите количество и приближенные значения переменной х (с точностью до фута), при котором огражденная площадь будет равна 25 000 кв. футов. е) Найдите размеры максимальной огражденной площади. Чему будет равна площадь в этом случае?
124 Часть I. Элементарные функции Биологические науки 46. Загрязнение воздуха. В обычный летний день в крупном городе индекс загрязненности воздуха в 8 часов утра достигает значения 20 частей на миллион, а к 3 часам дня линейно уменьшается до значения 15 частей на миллион. Пусть Р(х) обозначает долю загрязняющих веществ в воздухе через х часов после 8 утра. а) Найдите уравнение для функции Р(х). б) Чему будет равен индекс загрязненности в час дня? в) Постройте график функции Р в прямоугольных координатах. г) Чему равен наклон графика? [Наклон задает значение, на которое уменьшается уровень загрязненности за один час.] Социальные науки 47. Психосенсорное восприятие. Следующая задача является одной из старейших в физиологии. Она ставится для определенного фактора воздействия (светового, звукового, силового, электрического и т. п.). В задаче спрашивается: на сколько нужно увеличить силу воздействия, чтобы она стала ощутимой для человеческого восприятия? В середине девятнадцатого века немецкий физиолог Эрнст Генрих Вебер (Е. Н. Weber) открыл закон, носящий его имя: если As — минимальная добавка к силе воздействия $, которую замечает человек, то отношение As к s — постоянная величина. s Таким образом, минимальные заметные добавки As к силе воздействия зависит от самой силы воздействия s линейным образом. Чем больше воздействующий фактор, тем заметнее его добавка. В эксперименте с поднятием веса константа к оказалась равной а) Найдите значение As (минимальное ощутимое изменение веса) для веса 30 и 90 фунтов. б) Постройте график функции As = s/ЗО в интервале 0 s 120. в) Чему равен наклон этого графика? Домашнее задание 1.1. Введение в регрессионный анализ Решение многих практических задач требует построения математической модели, описывающей исходные данные, представленных в табличной форме. Зачастую взаимосвязь между такими данными описывается функцией, уравнение которой можно найти только с помощью специального математического метода — регрессионного анализа. Познакомиться с функциями, полученными методом регрессионного анализа, можно в многочисленных примерах и упражнениях раздела 1. Регрессионный анализ — это метод построения подходящей функции по набору точек данных. Эта операция также часто называется подгонкой кривой.
Глава 1. Базовые элементарные функции 125 В настоящем дополнении основное внимание уделяется линейной регрессии — подбору линейной функции по заданным точкам данных. Другие типы регрессионного анализа будут рассмотрены в следующем разделе. Сначала давайте посмотрим, как метод регрессионного анализа применяется для обработки данных в графической утилите. На данном этапе мы не будем вникать в детальное описание регрессионных формул. При решении примера 1.7 раздела 1.1 мы использовали следующую функцию зависимости цены от спроса р{х) = 94,8 - 5х (1.16) для описания данных табл. 1.27. Здесь р(х) — оптовая цена фотоаппарата в миллионах долларов, а х — спрос в миллионах штук. Таблица 1.27. Зависимость цены от спроса X, млн. шт. р, долл. 2 87 5 68 8 53 12 37 Рассмотрим, как можно получить функцию р(ж) методом линейной регрессией с помощью графической утилиты. В большинстве графических утилит задача нахождения функции, описывающей исходные данные, разбивается на три этапа. 1. Ввод данных. 2. Вычисление уравнения регрессии. 3. Отображение входных данных и графика регрессионного уравнения в одном окне вывода. Последовательность выполняемых при этом команд и операций зависит от программы. Детально они описываются в справочном руководстве к программному обеспечению. Если вы пользуетесь графическим калькулятором Texas Instruments или процессором электронных таблиц, то всю необходимую информацию можете почерпнуть в приложениях к этой книге. На рис. 1.38 приведены результаты обработки данных табл. 1.27 на графическом калькуляторе, а на рис. 1.39 — в процессоре электронных таблиц. Отметим, насколько хорошо прямые, изображенные на рис. 1.38 и 1.39, описывают данные, приведенные в табл. 1.27. Сейчас мы не будем вдаваться в математические подробности вычисления уравнений этих прямых. Достаточно знать, что прямая, получена методом линейной регрессии, хорошо (а в некоторых случаях даже весьма точно) описывает исходные данные.
126 Часть I. Элементарные функции и L2 L3 2 2 5 В 12 07 ев 53 L2C5) = LinReg y=ax+b а= -4.96347032 b=94.75342466 Рис. 1.38. Линейная регрессия на графическом калькуляторе Большинство графических утилит вычисляет коэффициенты уравнения регрессии с большей точностью, чем требуют рассматриваемые нами задачи. Обычно после получения регрессионного уравнения в графической утилите коэффициенты нужно округлить. Например, округляя коэффициенты уравнения, приведенного на рис. 1.38 или 1.39, мы получим точное уравнение функции /, описанной в примере 1.7 раздела 1.1 (см. уравнение (1.16)). 1. Выполните пп. 1 и 3 для данных табл. 1.27, если вы еще этого не сделали. Чтобы упростить дальнейшую работу в графической утилите, составьте короткую инструкцию по использованию в ней команд и функций регрессионного анализа. (В следующих разделах вам придется выполнять упражнения, требующие обязательного нахождения уравнений регрессии.) 2. В каждом из приведенных далее упражнений главы 1 наряду с исходными данными указывается описывающее их уравнение. Вам придется использовать графическую утилиту для вычисления линейного уравнения регрессии в каждом из случаев отдельно. Постройте данные и график полученного уравнения в одном окне вывода. Обсудите различия между функцией, описывающей исходные данные, и полученным вами уравнением регрессии.
Глава 1. Базовые элементарные функции 127 ■ Раздел 1.1, упражнение 1.7. ■ Практикум 1.1, задачи 85 и 86. ■ Практикум 1.3, задачи 69 и 70. Домашнее задание 1.2. Математическое моделирование в бизнес-анализе Компания производит и продает горные велосипеды. Оценка безубыточности производства требует выполнения расчетов с использованием функции затрат и зависимости цен от спроса. Для нахождения таких функций нужно сначала собрать данные об объеме производства при разных уровнях затрат и цены на продукцию, а затем подобрать функцию (из библиотеки элементарных функций), максимально точно описывающей зависимости, обнаруживаемые в собранных данных. Предположим, что используя статистические методы, финансовый отдел компании получил данные о ценах и затратах, приведенные в табл. 1.28 и 1.29. Таблица 1.28. Зависимость цены от спроса х, сотни шт. р, долл. б 525 64 370 125 270 185 130 Таблица 1.29. Затраты х, сотни шт. S, сотни долл. 0 8470 30 13510 120 19140 180 22 580 220 28490 1. Получение уравнения зависимости цены от спроса. Отобразив данные табл. 1.28 на графике, легко заметить, что р зависит от х линейно. 1) В домашнем задании 1 было показано, как обрабатывать табличные данные в графической утилите. Найдите уравнение регрессионной и постройте ее график в одном окне с исходными данными. 2) Уравнение линейной регрессии, полученное в п. 1, опиисывает зависимость цены от спроса и имеет следующий вид. р(гс) = — 2,09а? + 519 Функция зависимости цены от спроса
128 Часть I. Элементарные функции Отобразите данные табл. 1.27 и уравнение функции зависимости цен от спроса на одном графике. 3) График уравнения линейной регрессии представлен прямой линией. Определите наклон этой прямой, области определения и значений. С помощью приведенной в п. 2 функции найдите цену велосипеда при спросе 10000 и 20000. 2. Получения уравнения затрат. Отобразите данные табл. 1.29 на графике в прямоугольной системе координат. Какая функция лучше всего подходит для описания этих данных? Ч 1) В домашнем задании 1 было показано, как обрабатывать табличные данные в графической утилите. Найдите уравнение линейной регресии для данных табл. 1.29 и постройте ее график в одном окне с исходными данными. 2) Уравнение линейной регрессии, полученное в п. 1, описывает зависимость затрат от объема производства и имеет следующий вид. С(х) = 81х + 9498 Функция затрат Постройте данные табл. 1.29 и уравнение функции затрат на одном графике. 3) Определите наклон этой прямой, области определения и значений. С помощью приведенной в п. 2 функции найдите затраты на выпуск 10000 и 20 000 велосипедов. 3. Анализ безубыточности. Обратимся к уравнениям затрат (см. задачу 2) и зависимости цен от спроса (см. задачу 1). Найдите функции дохода и ее область определения. Найдите функции прибыли и ее область определения. 1) Постройте графики функций дохода и затрат в общей декартовой системе координат. Определите (с точностью до единицы) безубыточный объем производства. Найдите также, при каком объеме производства уровень затрат выше уровня доходов и наоборот. 2) Постройте графики функций дохода и затрат с помощью графической утилиты в одном окне вывода. Определите (с точностью до единицы), безубыточный объем производства, а также объем производства, при котором уровень затрат выше уровня доходов и наоборот. 3) Постройте график функции прибыли в декартовой системе координат. Определите (с точностью до единицы) точки безубыточности производства. Определите условия, при которых производство можно считать прибыльным и убыточным. Какой будет цена при условии максимальной
Глава 1. Базовые элементарные функции 129 прибыли? Можно ли получить максимальную прибыль и максимальный доход при одном и том же объеме производства? Аргументируйте ответ. Ч 4) Постройте график функции прибыли с помощью графической утилиты. Определите (с точностью до единицы), точки безубыточности производства и условия прибыльности и убыточности производства. Чему равна цена при условии максимальной прибыли? Можно ли получить максимальную прибыль и максимальный доход при одном и том же объеме производства? Аргументируйте ответ.
2 Другие элементарные функции ■ 2.1. Полиномы и рациональные функции ■ 2.2. Показательная функция ■ 2.3. Логарифмическая функция ■ Ключевые слова, основные обозначения и термины ■ Упражнения для повторения ■ Домашнее задание 2.1. Сравнение скоростей возрастания показательных функций и полиномов, логарифмических функций и функций извлечения корня ■ Домашнее задание 2.2. Сравнение регрессионных моделей Введение В главе рассматриваются четыре класса функций: полиномы, рациональные, показательные и логарифмические функции. Они дополняют библиотеку элементарных функций, описанную в главе 1. Функций такой расширенной библиотеки достаточно для решения большинства задач, рассматриваемых в книге. Линейные и квадратичные функции, рассмотренные в главе 1, являются подмножеством более широкого класса функций, называемых полиномами.
Глава 2. Другие элементарные функции 131 2.1. Полиномы и рациональные функции ■ Полиномы ■ Нахождение приближенных корней полиномов ■ Полиномиальные регрессионные уравнения ■ Рациональные функции ■ Решение практических задач Полиномы В главе 1 были описаны следующие элементарные функции: f (х) = Ь Постоянная функция /(#) = ах + Ь, а / 0 Линейная функция f (х) = ах2 + Ьх + С, а 0 Квадратичная функция Кроме этих функций, также были рассмотрены некоторые частные случаи кубической функции. f (х) = ах3 + Ьх2 + сх + d, а 0 Кубическая функция Эти функции использовались при решении большинства рассмотренных ранее прикладных задач, например, при расчете производственных затрат, определении дохода и чистой прибыли, анализе убыточности производства, а также при вычислении размера упаковки для выпускаемой продукции. Обратите внимание на то, что уравнения, определяющие эти функции, начиная с постоянной и заканчивая кубической, состоят из однотипных слагаемых вида ахп9 где а — любое действительное число, ап- неотрицательное целое число. Все рассмотренные выше функции являются представителями более широкого класса функций, называемых полиномами. Полином Полиномом (polynom) называют функцию, заданную уравнением вида: /(ж) = апхп + an-ixn~r Н h а^х + а0> где п — неотрицательное целое число, которое называют степенью полинома. Коэффициенты ао, ai,..., ап — действительные числа, причем ап 0. Область определения полинома — множество всех действительных чисел. Форма графика полинома связана с его степенью. Следует подчеркнуть, что графики полиномов нечетной степени точно так же, как и графики полиномов четной степени, ведут себя, в общих чертах, сходным образом. В схожести свойств
132 Часть I. Элементарные функции графиков полиномов можно убедиться, изучив рис. 2.1 на котором изображены графики полиномов с 1-й по 6-ю степень. а)Дх) = х-2 в) А(х)=х5 —5х3 + 4х+1 г) Г(х)=х2 — 2х +2 д) ед = 2/ -4х2+х-1 е) Я(х)=х6-7х4+14х2-х-5 Рис. 2.1. Графики полиномиальных функций Обратите внимание на то, что график полинома нечетной степени начинается в полуплоскости, лежащей ниже оси х, затем как минимум один раз пересекает ось х и заканчивается в полуплоскости, лежащей выше оси абсцисс. Графики полиномов четной степени начинаются и заканчиваются в полуплоскости, лежащей выше оси х, и могут не пересекать ось абсцисс вовсе. Во всех уравнениях полиномов, графики которых изображены на рис. 2.1, коэффициент при наивысшей степени независимой переменной был выбран положительным. Если изменить знак коэффициента в слагаемом с наибольшей степенью ж, то графики таких полиномов станут отражением первоначальных относительно оси х. Графиком полинома является непрерывная линия, т.е. он не имеет разрывов, а также точек, в которых функция не определена. Это значит, что график полинома можно нарисовать, не отрывая карандаш от бумаги. Кроме того, графики полиномов не содержат точек излома. На рис. 2.2 приведены графики двух функций. График первой функции не обладает свойством непрерывности. График второй функции является непрерывным, но имеет точку излома. Ни один из представленных на рисунке графиков не является графиком полинома.
Глава 2. Другие элементарные функции 133 Рис. 2.2. Графики функций, один из которых имеет разрыв, а второй — излом Графики полиномов, приведенные на рис. 2.1, содержат максимально возможное для полинома указанной степени количество точек экстремума. На графике непрерывной функции точка экстремума разделяет область возрастания и область убывания функции, или наоборот. В общем случае справедливо следующее утверждение. График полинома положительной степени п может иметь максимум (п — 1) точку экстремума и пересекать ось х максимум п раз. Задание 2.1. 1. Какое минимальное количество точек экстремума могут иметь полиномы нечетной и четной степеней? 2. Какое максимальное количество точек пересечения с осью х может иметь график полинома n-й степени? 3. Какое максимальное количество действительных решений может иметь уравнение полинома n-й степени? 4. Какое минимальное количество точек пересечения с осью х могут иметь графики полиномов нечетной и четной степеней? 5. Какое минимальное количество действительных решений могут иметь полиномы нечетной и четной степеней? ■ Сравним графики двух полиномов в окрестности начала координат. Затем уменьшим масштаб изображения и снова сравним графики. Графики полиномов, о которых идет речь, изображены на рис. 2.3. Анализируя рис. 2.3, можно сделать вывод, что член полинома, содержащий наивысшую степень независимой переменной, доминирует над остальными. Так, при постепенном уменьшении масштаба изображения график функции у = х5 — — 5я3 + 4ж + 1 становится все больше и больше похожим на график функции
134 Часть I. Элементарные функции б)у=/ в) у = х5 — 5*3 4- 4х + 1 УМЕНЬШЕНИЕ МАСШТАБА Рис. 2.3. Графики полиномов при уменьшении масштаба у = х5. Такое поведение характерно для полиномов любой степени и является их общим свойством. Задание 2.2. Сравните графики полиномов у = х6 и у = х6—7х4+14х2—х—5, построив их в указанном масштабе. 1. — 5 х < 5, — 5 у 5. 2. — 5 х 5, —500 у 500. Нахождение приближенных корней полиномов Jfi Точка пересечения графика функции f с осью х называется нулем1 функции или корнем уравнения /(х) = 0. Приближенно вычислить нули функции до- 1 Координаты точек пересечения графика с осью х обязательно являются действительными числами. Уравнения соответствующих функций могут иметь корни, не принадлежащие множеству действительных чисел. Однако такие корни не являются точками пересечения графика функции с осью х. Поскольку мы условились рассматривать лишь те функции, область определения которых ограничена множеством действительных чисел, то комплексные корни уравнений соответствующих функций, которые не принадлежат множеству действительных чисел, здесь рассматриваться не будут.
Глава 2. Другие элементарные функции 135 статочно легко, если воспользоваться возможностями графической утилиты. Однако при этом должен быть известен интервал значений, в котором расположены координаты точек касания или пересечения графика с осью х. Но можно ли быть уверенным, что в найденном интервале значений находятся все нули функции? Найти интервал значений, в котором расположены все нули полинома, позволяет теорема 2.1, доказанная французским математиком О. Л. Коши (A. L. Cauchy) (1789-1857). Кроме этой теоремы, с именем Коши связан еще целый ряд теорем и математических понятий. Теорема 2.1 показывает, что классические математические инструменты могут эффективно применяться для решения современных прикладных задач. Теорема 2.1 (Локализация нулей полинома). Если число г является нулем многочлена Р(х) = хп + an_ixn-1 + an_2xn-2 Н h aix + а0, 2 ТО2 |rI < 1 -Ь max{|an-i|, |ап_2|, • ■ •, |«i|, |а0|}. а В полиноме коэффициент члена с наивысшей степенью х часто называют старшим коэффициентом. Обратите внимание на то, что теорема 2.1 относится к полиномам, у которых старший коэффициент равен 1. Пример 2.1 (Приближенное вычисление нулей полинома). Найдите приближенно (с точностью до двух значащих цифр) действительные нули полинома Р(х) = 2ж4 — 5х3 — 4т2 + Зх + 6. Решение. Поскольку старший коэффициент полинома Р(х) равен 2, то теорему 2.1 непосредственно применить к полиному Р(х) нельзя. Однако ее можно применить к следующему полиному. Q(x) = = х (2х4 — 5х3 — 4х2 + Зх + б) = = I4 — |я3 — 2х2 + ^-х + 3. Л л При умножении функции на положительную константу ее график растягивается или сжимается вдоль вертикальной оси (см. раздел 1.2), а координаты точек пересечения графика с осью х при этом не меняются, поэтому полиномы Р(х) и Q(x) Примечание: запись max{|an_i|, |ап-2|, •. •, |ai|, |«о|} обозначает максимальное из чисел |an-i|, |ап—21,.. •, |ai|, |а0|.
136 Часть I. Элементарные функции имеют одинаковые нули. Таким образом, из теоремы 2.1 следует, что любой нуль г полинома Р(х) должен удовлетворять условию |r| < 1 + max Рис. 2.4. Приближенное вычисление нулей полинома с помощью графической утилиты Иначе говоря, все нули полинома расположены в интервале значений от —4 до 4. Из рис. 2.4, а, на котором изображен график полинома Р(ж), видно, что полином Р(х) имеет два действительных нуля. При этом можно быть уверенным, что в указанном интервале значений лежат все нули полинома. Пользуясь возможностями программного пакета, можно приближенно (с точностью до двух десятичных знаков) найти действительные нули полинома, которые равны 1,15 (рис. 2.4, б) и 2,89 (рис. 2.4, в). Обратите внимание на то, что в указанном интервале значений независимой переменной расположены три точки экстремума полинома Р(х). Это максимально возможное количество точек экстремума для полинома четвертой степени. Иными словами, все точки экстремума также расположены в пределах указанного интервала. ■ Упражнение 2.1. Вычислите приближенно (с точностью до двух значащих цифр) нули полинома Р(т) = Зх3 + 12х2 + 9х + 4. в Полиномиальное регрессионные уравнения В домашнем задании, помещенном в конце главы 1, показано, что для поиска прямой линии, наиболее адекватно описывающей какой-либо набор данных, могут использоваться методы регрессионного анализа. Кроме линейных для этих целей используют функции других классов. Большинство графических утилит позволяют описать имеющийся набор данных разными функциональными зависимостями. В этом разделе рассматриваются полиномиальные регрессионные уравнения. Регрессионные уравнения других типов описаны в следующих главах.
Глава 2. Другие элементарные функции 137 ■к Пример 2.2 (Оценка веса рыбы). Задача оценки веса рыбы по известной ее длине представляет интерес как для ученых, так и для любителей спортивной рыбалки. В табл. 2.1 приведены данные о длине и среднем весе озерной форели. Исходя из табличных данных и используя методы регрессионного анализа, найдите полиномиальную зависимость, позволяющую оценить вес озерной форели по ее размеру. Определите (с точностью до унции) вес озерной форели длиной 39, 40, 41, 42 и 43 дюйма соответственно. Таблица 2.1. Размеры озерной форели WWW Длина, дюймы Вес, унции Длина, дюймы Вес, унции X У X У 10 5 30 152 14 12 34 226 18 26 38 326 22 56 44 536 26 96 Решение. На рис. 2.5, а на координатной плоскости изображены точки, которые соответствуют данным из табл. 2.1. По оси абсцисс отложена длина речной форели, а по оси ординат — вес рыбы. Из рисунка видно, что модель линейной регрессии в данном случае оказывается несостоятельной. И в самом деле, вряд ли можно ожидать, что между весом форели и ее длиной существует линейная зависимость. Наоборот, более вероятно, что вес рыбы связан с кубом ее длины. По этой причине кривую, описывающую табличные данные, мы будем искать среди полиномов третьей степени (рис. 2.5, б). (Процедура поиска полиномиального уравнения регрессии в графической утилите, детально описана в справочном руководстве.) На рис. 2.5, в изображен график найденного программой полинома (сплошная линия), описывающего данные (обозначены точками). Из рис. 2.5, в видно, что найденный кубический полином в самом деле достаточно хорошо аппроксимирует табличные данные. (Более детально критерии выбора аппроксимирующих функций, а также оценка точности выбранной регрессионной модели Рис. 2.5. Оценка веса рыбы по ее длине
138 Часть I. Элементарные функции будут рассмотрены позже.) Вес форели указанной длины, оцененный с помощью найденной зависимости, приведен в таблице, изображенной на рис. 2.5, г. ■ __ Упражнение 2.2. Ви| В табл. 2.2 приведен средний вес щуки разной длины. Аппроксимируйте табличные данные с помощью кубического полинома. Оцените (с точностью до унции) вес щуки длиной 39, 40, 41, 42, и 43 дюйма соответственно. ■ Таблица 2.2. Размеры щуки WWW Длина, дюймы Вес, унции Длина, дюймы Вес, унции X У X У 10 5 30 108 14 12 34 154 18 26 38 210 22 44 44 326 26 72 52 522 Рациональные функции Аналогично рациональным числам, которые определяются как частное двух целых чисел, рациональные функции определяются как частное двух полиномов. Уравнения, которые приведены ниже, определяют рациональные функции. /(*) = X р(х) = Зх2 — 5х, ж —2 X2 — х — б ’ = 9(ж) = 7, Рациональная функция Рациональной функцией называется любая функция вида Л®) = $7’ °> а\Х) где п(х) и d(x) — полиномы. Область определения рациональной функции — множество всех действительных чисел, при которых d(x) 0. Предполагается, что дробь n(x)/d(x) приведена к несократимому виду. Пример 2.3 (Область определения функции и точки пересечения графика с осями координат). Найдите область определения и точки пересечения с осями координат графика рациональной функции л/ х х — 2
Глава 2. Другие элементарные функции 139 Решение. Область определения. Знаменатель равен 0 при х = — 1. Следовательно, область определения данной функции — множество всех действительных чисел за исключением числа —1. Отсюда следует, что график функции / не может пересекать вертикальную прямую х = — 1. Точки пересечения графика с осью х. Найдем значения независимой переменной х, при которых выполняется равенство /(х) = 0. Указанное равенство имеет место лишь в том случае, если х — 2 = 0, т.е. если х = 2. Таким образом, число 2 — абсцисса единственной точки пересечения графика с осью х. Точки пересечения с осью у. Ордината точки пересечения графика данной функции с осью у равна Упражнение 2.3. Найдите область определения и точки пересечения с осями координат графика рациональной функции: д(х) = ■ В следующем примере исследуется поведение графика функции /(т) = в окрестности точки разрыва х = — 1, а также поведение функции при неограниченном увеличении или уменьшении независимой переменной х. Используя полученную информацию, а также некоторые дополнительные сведения, мы построим эскиз графика функции /. В результате проведенного исследования мы ознакомимся с некоторыми характерными для графиков рациональных функций свойствами. Пример 2.4 (График рациональной функции). Рассмотрим функцию f из примера 2.3 х — 2 х + 1 ’ 1. Исследуйте график функции f в окрестности точки разрыва х = — 1. 2. Исследуйте поведение графика функции f при неограниченном увеличении или уменьшении независимой переменной х. 3. Схематически изобразите график функции /. Решение. 1. Пусть х стремится к — 1 слева. X /(®) —2 4 -1,1 31 -1,01 301 -1,001 3001 -1,0001 30001 -1,00001 300001
140 Часть I. Элементарные функции Из таблицы видно, что функция /(ж) неограниченно возрастает, если х стремится к числу —1 слева. Это утверждение можно записать в символьной форме. /(ж) —> оо при X —» — I- Пусть х стремится к — 1 справа: /(g) 0 -0,9 -0,99 -0,999 -0,9999 -0,99999 —2 -29 -299 -2999 -29999 -299999 Из таблицы видно, что функция /(ж) безгранично возрастает, если х стре- мится к —1 справа. Это утверждение можно записать в символьной форме. /(х) —> —сю при х —> —1+ Вертикальная прямая линия х = — 1 называется вертикальной асимптотой. График функции / приближается к этой линии, если х приближается к — 1. Вертикальные асимптоты, нанесенные на чертеж, облегчают построение графика функции f вблизи его асимптот (рис. 2.6). Л*) W Ы t I 1 I I t -10 -5 J N I I I N I I I—kX 5 10 Рис. 2.6. Поведение графика вблизи вертикальной асимптоты х = — 1 2. Разделим каждый член в числителе и знаменателе функции /(ж) на переменную х, т.е. на наивысшую степень независимой переменной, с которой она входит в числитель и знаменатель.
Глава 2. Другие элементарные функции 141 При неограниченном увеличении или уменьшении переменной х величины 2/х и 1/х стремятся к нулю, и, следовательно, функция /(ж) стремится к единице. Горизонтальная прямая линия у = 1 называется горизонтальной асимптотой. График функции у = /(ж) приближается к этой линии при уменьшении или увеличении независимой переменной х. Однако как именно график функции у = /(ж) приближается к горизонтальной прямой у = 1? Сверху? Или снизу? Или может быть одновременно с обеих сторон? Чтобы ответить на эти вопросы, проанализируем приведенные в таблице значения функции для некоторых значений независимой переменной. Пусть х стремится к оо. X /(ж) 10 0,72727 100 0,97030 1000 0,99700 10000 0,99970 100000 0,99997 При неограниченном возрастании переменной х график функции у = /(ж) приближается к прямой линии у = 1 Пусть х стремится к —оо. снизу. X -10 1,33333 -100 1,03030 -1000 1,00300 -10000 1,00030 -100000 1,00003 При неограниченном уменьшении переменной х график функции у = f(x) приближается к прямой линии у = 1 сверху. Чтобы облегчить построение графика функции /(ж) в интервале значений независимой переменной х, достаточно далеко отстоящих от начала координат, на рисунке сначала нужно изобразить горизонтальную асимптоту (рис. 2.7). 3. Теперь можно достаточно легко завершить построение эскиза графика функции /, нанеся на чертеж найденные в примере 2.3 точки пересечения графика с осями координат, а также некоторые другие вспомогательные точки, принадлежащие графику функции (рис. 2.8). х f(x) -4 2 -2 4 О —2 2 О
142 Часть I. Элементарные функции функции у — f(x) вблизи горизонтальной асимптоты у = 1 Рис. 2.8. Эскиз графика рациональной функции f Нахождение вертикальных и горизонтальных асимптот графиков рациональных функций Пусть дана рациональная функция где п(х) и d(x) полиномы, которые не имеют общего делителя. 1. Если а действительное число, такое что d(a) = 0, то прямая линия х = а является вертикальной асимптотой графика функции у = f(x). 2. Горизонтальные асимптоты, если они существуют, можно найти, разделив каждый член числителя п(я) и знаменателя d(x) на наивысшую степень х в числителе и знаменателе, а затем выполнив последовательность действий, описанную в примере 2.4. Упражнение 2.4. Дана функция g из упражнения 2.3: д(х) = . х — 2 1. Исследуйте поведение графика функции д в окрестности точки разрыва х = 2, 2. Исследуйте поведение графика функции д при неограниченном увеличении и уменьшении независимой переменной х. 3. Схематически изобразите график функции д, ■ Вертикальные и горизонтальные асимптоты, если они, конечно, существуют, значительно облегчают построение графиков рациональных функций. Ниже опи-
Глава 2. Другие элементарные функции 143 сана процедура поиска асимптот графиков рациональных функций, которая уже была использована в примере 2.4. Пример 2.5 (Построение графиков рациональных функций). Рассмотрим следующую рациональную функцию. ж2-4' 1. Найдите точки пересечения графика функции с осями координат, а также составьте уравнения вертикальных и горизонтальных асимптот графика. 2. Используя информацию, полученную при решении задачи 1, а также при необходимости вспомогательные точки, постройте график функции f в масштабе —7 х 7 и “7 у 7. Решение. 1. Точки пересечения с осью х. Равенство f(x) — 0 выполняется лишь в том случае, если Зх = 0, т.е. если х = 0. Следовательно, график пересекает ось х в единственной точке, абсцисса которой равна 0. Точки пересечения с осью у. Вычислим значение /(0). _ 3-0 0 /(О) = 62Т4 —=»• Следовательно, график пересекает ось у в точке, ордината которой равна 0. Вертикальные асимптоты. Разложим знаменатель на множители. _ Зх _ Зх № “ х2_4 - (ж —2)(ж + 2)’ Знаменатель равен 0 при х = — 2 и х = 2. Следовательно, вертикальными асимптотами графика являются прямые х = — 2 и х = 2. Горизонтальные асимптоты. Разделим каждый член числителя и знаменателя на ж2, наивысшую степень х в числителе и знаменателе. Зх 3 ,, . Зх Х^ X = ^ = d/ JU X2 X2 При неограниченном уменьшении или увеличении переменной х числитель стремится к нулю, а знаменатель — к единице. Иначе говоря, функция /(х) стремится к нулю. Следовательно, горизонтальной асимптотой графика является прямая у = 0.
144 Часть I. Элементарные функции 2. Используя информацию, полученную при решении задачи 1, а также нанеся на чертеж дополнительные точки, приведенные в таблице, закончим построение графика. График функции /(х) изображен на рис. 2.9. X -4 -1 -2,3 -5,3 —1,7 4,6 0 0 1,7 -4,6 2,3 5,3 4 1 Рис. 2.9. График рациональной функции Упражнение 2.5. Дана рациональная функция: д(х) = 1. Найдите все точки пересечения графика функции с осями координат, а также составьте уравнения вертикальных и горизонтальных асимптот графика. 2. Используя информацию, полученную при решении задачи 1, а также при необходимости вспомогательные точки, схематически изобразите график функции д в масштабе: —10 С х < 10 и —10 < у < 10. ■ Решение практических задач В Рациональные функции естественным образом возникают при решении разного рода прикладных задач. Пример 2.6 (Обучение сотрудников). В компании, выпускающей компьютерные комплектующие, было замечено, что пройдя обучение в течение t дней, новый сотрудник производит за рабочую смену в среднем N(t) компонентов. Обозначенная зависимость описывается следующей функцией. W) = 50t £4-4 при t 0. Схематически изобразите график функции N на отрезке 0 t 100. Постройте вертикальные и горизонтальные асимптоты графика. К какому значению стремится функция N(t) при неограниченном возрастании переменной £?
Глава 2. Другие элементарные функции 145 Решение. Вертикальные асимптоты. В интервале t 0 график асимптот не имеет. Горизонтальные асимптоты. Функция JV(t) стремится к 50 при неограниченном увеличении переменной t. Таким образом, горизонтальной асимптотой графика является прямая у = 50. Эскиз графика. График функции схематически изображен на следующем рисунке. МО Функция N(t) стремится к 50 при неограниченном увеличении переменной t. Оказывается, что опытный сотрудник может изготовить на протяжении одного дня 50 компонентов. ■ П Упражнение 2.6. Решите пример 2.6 при условии, что зависимость имеет следующий вид. N(t) = ТД5 при t 0. L т О Ответы к упражнениям 2.1. -3,19. 39 40 41 42 229 246 264 293 303 CubicReg y=ax*+bx*+cx+d а=.0031108574 b=.0405684119 с=-.5340734768 d=3.341615319 CubicReg
146 Часть I. Элементарные функции 2.3. Область определения: все действительные числа за исключением 2; абсцисса точки пересечения графика с осью х: 0; ордината точки пересечения графика с осью у: 0. 2.4. 1) Вертикальная асимптота: х = 2. 2) Горизонтальная асимптота: у = 2. 2.5. 1) Абсцисса точки пересечения графика с осью х: —1; ордината точки пересечения графика с осью у: — Вертикальные асимптоты: х = — 3 и х = 3. Горизонтальная асимптота: у = 0.
Глава 2. Другие элементарные функции 147 2.6. В интервале t > 0 график вертикальных асимптот не имеет; горизонтальной асимптотой является прямая у = 25. Функция 7V(t) стремится к 25 при неограниченном увеличении переменной t. Оказывается, что опытный сотрудник может изготовить на протяжении одного дня 25 компонентов. МО Практикум 2.1 А В задачах 1-6 для каждого полинома найдите следующие характеристики, 1) Степень полинома. 2) Максимально возможное для полинома данной степени количество точек экстремума. 3) Максимально возможное для полинома данной степени количество точек пересечения графика с осью х. 4) Минимальное количество точек пересечения графика с осью х, которое может иметь полином данной степени. 5) Максимально возможное количество точек пересечения графика с осью у, которое может иметь полином данной степени. 6) Минимальное количество точек пересечения графика с осью у, которое может иметь полином данной степени. 1. /(я) = ах2 + Ьх + с, а / 0. 2. f(x) = ах + Ь, а / 0. 3. f(x) = ахБ + bx4 + ex3 -I- dx2 + ex + f, а / 0. 4. f(x) = ах4 4- bx3 + ex2 + dx + e, a Ф 0. 5. /(ж) = ахБ + Ьхъ + ex4 + dx3 4- ex2 + fx + g, a / 0. 6. f(x) = ax3 4- bx2 + or + d, a 0.
148 Часть I. Элементарные функции Каждый график в задачах 7-14 представляет полиномиальную функцию. Ответьте на следующие вопросы. 1) Сколько точек экстремума имеет график полинома? 2) Полиному какой наименьшей степени может соответствовать приведенный график? 3) Какой знак имеет старший коэффициент полинома? 7. Дх) 8. Дх) 5-- 5--
Глава 2. Другие элементарные функции 149 Б В задачах 15-20 для каждой рациональной функции выполните следующие задания. 1) Найдите точки пересечения графика с осями координат. 2) Определите область существования функции. 3) Составьте уравнения вертикальных и горизонтальных асимптот графика функции. 4) Схематически изобразите асимптоты пунктирными линиями. Затем нарисуйте график функции у = f(x) в масштабе —10 х 10 и —10 у < 10. 5) Используя графическую утилиту, постройте график функции у = f(x) в масштабе, предложенном по умолчанию. 15, = F4' 16- = iri’ х — 2 х 4- 3 17‘ = TZ? 18’ = х 4- 2 х — 3 \ 4 — 2х 3 — Зх 19- Л®) = 20./(х) = — X — 4 X — 2 21. Уменьшая масштаб изображения, сравните между собой графики функций/(х) = = 2х4 — 5х2 4- х + 2 и у = 2х4 (см. рис. 2.3). Какие при этом можно сделать выводы? 22. Уменьшая масштаб изображения, сравните между собой графики функций /(х) = = х3 — 2х 4- 2 и у = х3. Какие при этом можно сделать выводы? 23. Уменьшая масштаб изображения, сравните между собой графики функций /(х) = = —х5 4- 4х3 — 4х 4-1 и у = —х5. Какие при этом можно сделать выводы? 24. Уменьшая масштаб изображения, сравните между собой графики функций/(х) = = —х5 4- 5х3 4- 4х — 1 и у = —х5. Какие при этом можно сделать выводы? *5 25. Сравните графики функций у = 2х4 и у = 2х4 — 5х2 4- х 4- 2, построенные в следующем масштабе. а) -5 х 5, -б у С 5. б) -5 х 5, —500 < у < 500. 26. Сравните графики функций у ~ х3 и у = х3 — 2х 4- 2, построенные в следующем масштабе. а) -5 х 5, -5 у 5. б) -5 х 5, -500 у 500. 27. Сравните графики функций у = —х5 и у = —х5 4- 4х3 — 4х 4- 1, построенные в следующем масштабе.
150 Часть I. Элементарные функции а) — 5 х 5, — 5 у 5. б) — 5 х 5, —500 С у С 500. 28. Сравните графики функций у = —х5 и у = —х5 + 5ж3 — 5х + 2, построенные в следующем масштабе. а) — 5 х 5, — 5 у < 5. б) — 5 х 5, —500 у < 500. их В задачах 29-34 воспользуйтесь теоремой 2.1 и определите для каждого полинома на оси х интервал значений, в котором расположены все его нули. Приближенно, с точностью до двух десятичных знаков, вычислите все действительные нули каждого полинома. 29. 2х3 - х2 - 7х + 3. 31. х4 + 2х3 - Зх2 - 4х + 1. 33. х5 - 12х4 + 7х3 + 15. 30. За:3 + 10а:2 + 6а: — 2. 32. а:4 - Зх3 - 4а;2 + За: + 1 34. х5 + 14а:4 - 10а:2 - 15. 35. Постройте на координатной плоскости прямую линию у = 0,5ж + 3. Укажите на этой прямой две разные точки. Постройте модель линейной регрессии для множества данных, которое образовано координатами этих двух точек. Предложите свои уравнения прямых и проделайте с ними описанную выше процедуру. Объясните, какая существует взаимосвязь между найденной моделью линейной регрессии и прямой, которая проходит через две точки, по которым построена модель. 36. Постройте на координатной плоскости график параболы у = х2 — 5х. Укажите на параболе три разные точки. Постройте модель квадратичной регрессии для множества данных, которое образовано координатами этих трех точек. Предложите свои уравнения парабол и проделайте с ними описанную выше процедуру. Объясните, какая существует взаимосвязь между найденной моделью квадратичной регрессии и параболой, которая проходит через три точки, по которым построена модель. В Для каждой рациональной функции в задачах 3 7-42 выполните следующие задания. 1) Найдите все точки пересечения ее графика с осями координат. 2) Найдите все вертикальные и горизонтальные асимптоты графика. 3) Изобразите схематически асимптоты пунктирными линиями. Затем постройте график функции f в масштабе —10 х 10 и —10 у 10. 4) Используя графическую утилиту, постройте график функции у = f(x) в масшта- бе, предложенном по умолчанию. 2х^ »• /W = Х2 _ 9 ■ 4,-'W = l!+t<r г/ \ Зх2 38. f(x) = . х2 4- х — 6 43. Напишите уравнение полинома наименьшей степени, график и точки пересечения которого с осями координат изображены на рис. 2.10.
Глава 2. Другие элементарные функции 151 44. Напишите уравнение полинома наименьшей степени, график и точки пересечения которого с осями координат изображены на рис. 2.11. 45. Напишите уравнение полинома наименьшей степени, график и точки пересечения которого с осями координат изображены на рис. 2.12. 46. Напишите уравнение полинома наименьшей степени, график и точки пересечения которого с осями координат изображены на рис. 2.13. Рис. 2.11. Иллюстрация к задаче 44 Рис. 2.12. Иллюстрация к задаче 45 Рис. 2.13. Иллюстрация к задаче 46 Применение математики Экономика и бизнес 47. Средние затраты. Постоянные затраты компании, которая производит сноуборды, составляют 200 долл, в день. При выпуске 20 сноубордов за день общие затраты составляют 3800 долл. а) Составьте уравнение функции затрат при условии, что величина общих затрат за день С (ж) линейно связана с количеством сноубордов х, произведенных в течение дня. б) Средние затраты на производство сноуборда при выпуске х сноубордов за день выражаются формулой С(х) = С(х)/х. Найдите уравнение функции средних затрат.
152 Часть I. Элементарные функции в) Постройте эскиз графика функции средних затрат на отрезке 1 < х < 30. Схематически изобразите асимптоты графика. г) К какому значению стремится величина средних затрат на производство сноуборда при растущем объеме производства. 48. Средние затраты. Постоянные затраты компании, производящей доски для серфинга, составляют 300 долл, в день. При выпуске 20 досок для серфинга за день общие затраты составляют 5100 долл. а) Составьте уравнение функции затрат, предполагая, что величина общих затрат за день С(х) линейно связана с количеством х продукции, произведенной в течение дня. б) Средние затраты на производство доски для серфинга при выпуске х rocqk в день выражается формулой С(х) = С(х)/х. Найдите уравнение функции средних затрат. в) Постройте эскиз графика функции средних затрат на отрезке 1 < х 30. Схематически изобразите асимптоты графика. г) К какому значению стремится величина средних затрат на производство доски для серфинга при растущем объеме производства. 49. Техническое обслуживание. Первоначальная стоимость офисного копировального аппарата составляет 2500 долл. В соответствии с договором техническое обслуживание копировального аппарата в течение первого года эксплуатации обходится в 200 долл. За каждый последующий год стоимость технического обслуживания аппарата увеличивается на 50 долл. Таким образом суммарная стоимость копировального аппарата через п лет его эксплуатации рассчитывается согласно следующей формуле. С(п) = 2500 + 175п + 25п2. Среднегодовая стоимость копировального аппарата через п лет после его покупки выражается формулой С(п) = С(п)/п. а) Найдите рациональную функцию С. б) Схематически изобразите график функции С на отрезке 2 С п < 20. в) Через какой интервал времени функция среднегодовой стоимости достигнет минимального значения? Чему равно это минимальное значение? (Подсказка. Используя эскиз графика, построенный при решении п. б, определите интервал значений независимой переменной, в котором функция принимает минимальное значение. Вычислите значения функции С(п) для целых значений независимой переменной, лежащих в этом интервале, и найдите среди них минимальное.) Интервал времени, через который функция среднегодовой стоимости оборудования достигает минимального значения, часто называют интервалом между техническим обслуживанием оборудования. г) Используя графическую утилиту, постройте график функции среднегодовой стоимости оборудования С. Попеременно переходя в режимы изменения масштаба изображения и определения координат указанных точек, определите, через какой интервал времени функция среднегодовой стоимости достигает минимального значения.
Глава 2. Другие элементарные функции 153 50. Минимальные средние затраты. Финансовые аналитики компании, производящей проигрыватели музыкальных компакт-дисков, определили, что величина затрат на производство х проигрывателей в течение дня выражается следующим уравнением. С(х) = х2 + 2х + 2000. Величина средних затрат на производство проигрывателя при выпуске х проигрывателей в день выражается формулой С(х) = С(х)/х. а) Найдите рациональную функцию С. б) Схематически изобразите график функции С на отрезке 5 < х < 150. в) При каком уровне дневного производства проигрывателей (с точностью до ближайшего целого значения) величина средних затрат на единицу продукции достигает минимального значения? Чему равно это минимальное значение (с точностью до цента)? (Подсказка. Используя эскиз графика, построенный при решении п. б, определите интервал значений независимой переменной, в котором функция принимает минимальное значение. Вычислите значения функции С(х) для целых значений независимой переменной, лежащих в этом интервале, и найдите среди них минимальное.) Н г) Используя графическую утилиту, постройте график функции средних затрат С. Попеременно переходя в режимы изменения масштаба изображения и определения координат указанных точек, определите, при каком дневном объеме производства (ответ округлите до ближайшего целого значения) величина средних затрат на выпуск проигрывателя достигает минимального значения. Чему равно минимальное значение средних затрат с точностью до цента? 51. Минимальные средние затраты. По просьбе ветеринарной клиники консалтинговая фирма, используя статистические методы, составила следующее уравнение затрат. С(ш) = 0,00048(1 - 500)3 + 60000 при 100 х < 1000 Здесь С(х) — величина месячных затрат (долл.) на услуги, оказанные в х случаях. Средние затраты на оказание услуг определяются следующей формулой. С(х) = С(х)/х. а) Напишите уравнение функции средних затрат С.
154 Часть I. Элементарные функции б) Используя графическую утилиту, постройте график функции С. в) Изменяя масштаб изображения и определения координат указанных точек графической утилиты определите, при каком количестве обращений в ветеринарную клинику месячная величина средних затрат на оказанные услуги достигает минимального значения. Каково это минимальное значение? 52. Минимальные средние затраты. Финансовый отдел больницы, используя статистические методы, вывел следующее уравнение затрат на оказание медицинской помощи. С(х) = 20х3 - 360а:2 + 2300а: - 1000 при 1 < х 12. Здесь С(х) — месячные затраты (тыс. долл.) на помощь, оказанную в х случаях (тыс.). Средние затраты на оказание помощи выражаются формулой С(ж) = = С(х)/х. а) Напишите уравнение функции средних затрат С. б) Используя графическую утилиту, постройте график функции С. в) Переключаясь между режимами изменения масштаба изображения и определения координат указанных точек графической утилиты определите, при каком количестве обращений за помощью в больницу месячная величина средних затрат на оказание помощи достигает минимального значения. Чему равно это минимальное значение с точностью до доллара? 53. Точка равновесия. Музыкальные компакт-диски реализуются через сеть розничных магазинов. Проанализировав рынок, маркетинговая компания составила таблицы зависимости спроса и предложения от цены (табл. 2.3 и табл. 2.4 соответственно). В первой таблице через переменной х обозначено количество компакт-дисков, приобретаемых ежедневно по цене р долларов за штуку. Во второй таблице через х обозначено количество компакт-дисков, предлагаемых к ежедневной продаже, по цене р долларов. а) Найдите уравнение линейной регрессии, наиболее точно описывающее данные табл. 2.3, а также уравнение квадратичной регрессии, наиболее полно описывающее данные табл. 2.4. б) Определите координаты точки пересечения графиков уравнений, найденных при решении п. а. Определите равновесную цену с точностью до цента и равновесное количество (т.е. величину спроса и предложения при равновесной цене на конкурентном рынке) с точностью до целого значения. Таблица 2.3. Цена и спрос Таблица 2.4. Цена и предложение X р = D(x)3 долл. X р = S(x), долл. 25 19,50 25 2,10 100 14,25 100 3,80 175 10,00 175 8,50 250 8,25 250 15,70
Глава 2. Другие элементарные функции 155 54. Точка равновесия. Прочитайте условие задачи 53. Обобщите их на данные приведенных ниже таблиц. В этот раз для описания данных табл. 2.5 используйте уравнение квадратичной регрессии, а данных табл. 2.5 — уравнение линейной регрессии. Таблица 2.5. Цена и спрос Таблица 2.6. Цена и предложение X р = D(x), долл. X р = 5(ж), долл. 0 24 0 5 40 23 40 10 65 20 65 12 115 И 115 16 Биологические науки 55. Здравоохранение. В табл. 2.7 приведена общая сумма расходов правительства Соединенных Штатов Америки на здравоохранение (млрд, долл.), а также размер расходов в расчете на душу населения (долл.) в указанные годы, начиная с 1970 г. а) Обозначьте через х количество лет, которое прошло с 1970 г., и найдите кубическое регрессионное уравнение, описывающее рост общих расходов правительства США на здравоохранение. б) Используя полином, найденный при решении п. а, оцените общие расходы на здравоохранение в 2010 г. Ответ округлите до ближайших десяти миллиардов долларов. Таблица 2.7. Общие расходы на здравоохранение WWW Год Общие расходы, млрд. долл. Расходы на душу населения, долл. 1970 73,2 341 1975 132,9 592 1980 247,3 1052 1985 428,2 1666 1990 699,4 2689 1995 993,7 3638 56. Здравоохранение. Обратитесь к данным табл. 2.7. а) Обозначьте через х количество лет, которое прошло с 1970 г., и найдите кубический полином регрессии, описывающий расходы на здравоохранение в расчете на душу населения. б) Используя полином, найденный в пункте 1, оцените величину расходов на здравоохранение в расчете на душу населения в 2010 г. Ответ округлите до ближайших десяти миллиардов долларов. 57. Физиология. Изучая зависимость скорости сокращения мышц лягушки от величины воздействующей на них нагрузки, исследователи Б. Фемз (В. О. Fems) и Дж. Марш (J. Marsh) установили, что она снижается при увеличении нагрузки.
156 Часть I. Элементарные функции В частности, ими была найдена следующая зависимость скорости сокращения мышц v (см/с) от нагрузки х (г): , ч 26 + 0,06# v[x) = при # > 5. а) Определите, к какому значению стремится функция v(x) при увеличении переменной х. б) Нарисуйте эскиз графика функции v. Социальные науки 58. Теория обучения. В 1917 г. Луис Леон Тёрстоун (L. L. Thurstone), основоположник психометрии, предложил описывать зависимость между числом успешно выполняемых за единицу времени действий и количеством занятий по обучению таким действия х следующей рациональной функцией. а(х + (# + с) + Ь' Предположим, что процесс приобретения навыка набора текста описывается следующей функцией. г/ ч 55(# + 1) = ~г . о при х > 0, х + о где /(#) — количество корректно набираемых слов спустя х недель практических занятий. а) Определите, к какому значению стремится функция /(#) при увеличении переменной х. б) Нарисуйте эскиз графика функции /, а также схематически изобразите его вертикальные и горизонтальные асимптоты. 59. Супружество. В табл. 2.8 приведены данные о количестве заключенных и расторгнутых браков на 1000 человек за указанные годы, начиная с 1960 г. а) Пусть х — количество лет, которое прошло с 1960 г. Найдите уравнение полинома кубической регрессии, описывающее количество заключаемых браков. б) Используя полином, найденный при решении п. а, оцените количество браков (с точностью до первого знака после десятичной запятой), которые будут заключены в 2005 и 2010 гг. 60. Расторжение брака. Обратитесь к данным табл. 2.8. а) Пусть х — количество лет, которое прошло с 1950 г. Найдите кубическое регрессионное уравнение, описывающее количество расторгнутых браков.
Глава 2. Другие элементарные функции 157 Таблица 2.8. Количество заключенных и расторгнутых браков (в расчете на 1000 человек) WWW Год Количество заключенных браков Количество расторгнутых браков 1960 8,5 2,2 1965 9,3 2,5 1970 10,6 3,5 1975 10,0 4,8 1980 10,6 5,2 1985 10,1 5,0 1990 9,8 4,7 1995 8,9 4,4 б) Используя полином, найденный при решении п. а, оцените количество браков (с точностью до первого знака после десятичной запятой), которые будут расторгнуты в 2005 г. 2.2. Показательная функция ■ Показательная функция ■ Показательная функция с основанием е ■ Решение задач, описывающих процессы роста и распада, с помощью показательных функций ■ Сложные проценты ■ Непрерывное начисление сложных процентов В разделе рассматривается важный класс функций, называемых показательными. Эти функции служат решением многих прикладных задач, а также широко используются при моделировании различного рода процессов. Так, показательными функциями описываются увеличение суммы на депозитном счете при начислении сложных процентов, рост численности населения планеты, темпы размножения животных и бактерий, процесс радиоактивного распада химических элементов, скорость развития технологий, в частности, компьютерных, а также приобретения профессиональных навыков. Показательная функция Прежде всего, обратим внимание на то, что функции /(ж) = 2х и д(х) = х2 не являются тождественными. Более того, это совершенно разные функции. Функция д — это уже знакомая нам квадратичная функция. В ней показатель степени —
158 Часть I. Элементарные функции константа, а независимая переменная служит основанием степени. В уравнении, определяющем функцию /, константой является основание степени, а независимая переменная входит в показатель степени. Функция / является представителем нового класса функций, называемых показательными. В общем случае показательная функция может быть определена следующим образом. Показательная функция Функция, заданная уравнением /(ж) = Ьх, где b > О, b 7^ 1, называется показательной функцией с основанием Ь. Область определения функции f — множество всех действительных чисел; область значений функции f — множество всех положительных действительных чисел. Поскольку комплексные числа, например, (—2)1/2 = = i\/2, выходят за рамки материала, рассматриваемого в данном учебном пособии, в дальнейшем будем считать, что основание показательной функции Ь является положительным. Показательная функция с основанием Ь, равным единице, рассматриваться не будет, поскольку она эквивалентна постоянной функции, которая была описана в предыдущей главе: /(х) = Iх = 1. Многие учащиеся без малейших колебаний нарисуют от руки эскиз графика показательной функции, заданной, например, уравнением у = 2х или у = 2~х. (Примечание: 2~х = 1/2х = (1/2)х.) Очевидно, что сначала нужно составить таблицу значений функции для нескольких целых значений независимой переменной х. Полученные пары значений представляют координаты точек, отображаемых в декартовой системе координат и соединяемые плавной линией, как показано на рис. 2.14. Приведенный способ построения графика показательной функции имеет всего один недостаток — значения функции 2х определены не для всех действительных значений независимой переменной. Детально выражения 25, 2-3, 22/3, 2~3/5, 21’4 и 2~3,14 (т.е. выражения вида 2Р, где р — рациональное число) описаны в приложении А.7 (т. 2). Но какой математический смысл выражения 2х/2? Ответить на этот вопрос не так-то легко, обладая лишь знаниями, полученными до сих пор. Строгое определение смысла, который вкладывается в запись 2^, приводится в более специализированных изданиях. В них показано, что выражение вида 2х обозначает положительное действительное число при любых действительных значениях переменной х, а график функции у = 2х на самом деле имеет вид, показанный на рис. 2.14. С познавательной точки зрения полезно сравнить графики функций у — 2х и у = 2~х. Для этого построим их в одной системе координат, как показано на
Глава 2. Другие элементарные функции 159 Рис. 2.14. График функции у = 2х рис. 2.15, а. График функции /(ж) = Ьх, где b > 1 (рис. 2.15, б), очень напоминает график функции у = 2®, а график функции /(ж) = Ъх при 0 < b < 1 (рис. 2.15, б) очень похож на график функции у = 2 х. Обратите внимание на то, что для графиков рассмотренных функций горизонтальной асимптотой является ось х. определения = (—«>, *>) значений = (0, °°) б) Рис. 2.15. Показательные функции Графики, изображенные на рис. 2.15, обладают рядом важных свойств, общими для всех показательных функций. Эти свойства приведены ниже без доказательства.
160 Часть I. Элементарные функции Основные свойства графиков показательных функций, f(x) = Ьх, где Ь > 0 и Ь 0 1 1. График любой показательной функции проходит через точку с координатами (0, 1) (причем 6° = 1 при любых разрешенных значениях основания Ъ). 2. Графиком любой показательной функции является непрерывная кривая. Иначе говоря, график показательной функции не имеет точек разрыва или точек, в которых значение функции изменяется скачком. 3. Ось х является горизонтальной асимптотой графиков всех показательных функций. 4. Если b > 1, то при возрастании х функция f(x) = Ьх возрастает. 5. Если 0 < b < 1, то при возрастании х функция /(ж) = Ьх убывает. Построение графика показательной функции не представляет особых трудностей, если под рукой есть калькулятор, на панели которого имеется клавиша, обозначенная ух , или ее эквивалент. Процесс построения графика показательной функции с помощью калькулятора описанан в примере 2.7. Пример 2.7 (Построение графиков показательных функций). Изобразите схематически график функции у — (|) 4х на отрезке — 2 х 2. Решение. Используя калькулятор, составим таблицу значений функции при некоторых целых значениях независимой переменной. Полученные точки графика нанесем на координатную плоскость и соединим их плавной кривой, как показано на рис. 2.16. х у -2 0,031 -1 0,125 0 0,50 1 2,00 2 8,00 и Упражнение 2.7. Изобразите график функции у = (|) 4-ж на отрезке — 2 х 2. ■ Задание 2.3. Постройте в одной той же системе координат графики функций /(х) = 2х и д(х) = 3х. При каких значениях независимой переменной х графики этих функций пересекаются? При каких значениях независимой переменной х график функции f лежит выше графика функции д, а при каких — ниже? Как ведут себя графики функций при неограниченном увеличении переменной х: приближаются или удаляются друг от друга? А как они ведут себя при неограниченном уменьшении переменной х? Обсудите ответы. ■
Глава 2. Другие элементарные функции 161 Показательные функции, область которых включает иррациональные числа, обладают такими же свойствами, как и показательные функции с рациональными значениями степени, подробно описанными в приложении А.7 (т. 2). Ниже приведено описание этих и еще двух, не менее важных, свойств показательных функций, область определения которых представлена множеством действительных чисел. Свойства показательных функций Для положительных а и Ь, где о 1 и b / 1, а также любых действительных чисел х и у справедливы следующие соотношения. 1. Свойства степеней. ахау = ах+у (ах')У = аху — = ах~у аУ (аЬУ = ахЪх Д?У ! = 42у-5у! _ 4~3у 451/'- ач1 _ ах Ь/ Ьх 2. ах = ау тогда и только тогда, когда х = у. (Например, если 76*+1 = 73t-3, то 5t + 1 = 3t — 3, т.е. t = —2.) 3. Например, если х 0, то ах = Ьу тогда и только тогда, когда а = Ь. (Если а5 = 2б, то а = 2.) Показательная функция с основанием е Какое из значений основания b показательной функции у — Ьх используется чаще остальных? Если вы посмотрите на панель калькулятора, то, скорее всего, обнаружите клавиши, обозначенные Ю1 и е11. Причина, по которой основание
162 Часть I. Элементарные функции 10 имеет столь большое значение, понятна — мы пользуемся десятичной системой счисления. А какому значению соответствует символ е и почему основание е стоит в одном ряду с основанием 10? Оказывается, что показательная функция с основанием е используется чаще, чем остальные показательные функции вместе взятые. Причина заключается в том, что при использовании показательной функции с основанием е некоторые формулы, численные решения некоторых математических моделей реальных процессов, а также многие сложные математические проблемы принимают наиболее простую форму. Именно поэтому показательная функция с основанием е встречается чрезвычайно часто в выражениях и формулах, описывающих прикладные задачи. Фактически показательная функция с основанием е, у = ех, настолько широко распространена, что для нее даже придумали специальное название — экспонента. Число в является иррациональным, т.е. его, как и число л, нельзя точно представить в виде конечной десятичной дроби. При достаточно больших значениях х приближенное значение числа е можно вычислить с любой заданной точностью при помощи следующего выражения. (2.1) К какой величине стремится значение выражения (2.1) при неограниченном увеличении ж? Прежде чем прочесть ответ, попытайтесь найти его самостоятельно. Быть может, вы считаете, что значения выражения (2.1) стремятся к единице, поскольку к единице стремятся значения, которые принимает выражение 1 + Проверим это предположение. Для этого непосредственно вычислим значения выражения (2.1) для некоторых увеличивающихся значений х. Результаты вычислений приведены в табл. 2.9. Таблица 2.9. Результаты вычислений числа е X (1 + ^Г 1 2 10 2,59374... 100 2,70481... 1000 2,71692... 10000 2,71814... 100000 2,71827... 1000000 2,71828... Интересно отметить, что выражение (2.1) ни при каких значениях х не принимает значений, близких к единице. Наоборот, оказывается, что значения, которые принимает выражение (2.1), стремятся к числу, близкому к 2,7183. Фактически при неограниченном возрастании величины х значения выражения (2.1) стремятся к иррациональному числу, называемому числом е. Значение иррационального
Глава 2. Другие элементарные функции 163 числа е с точностью до 12 десятичных знаков приведено ниже. е = 2,718281828459. Сравните это значение со значением выражения е1, вычисленным с помощью калькулятора. Единого мнения относительно того, кто открыл константу е, до сих пор не существует. Названо оно в честь великого швейцарского математика Леонарда Эйлера (Leonhard Euler) (1707-1783). Показательная функция с основанием е Показательные функции с основанием е и 1/е определяются уравнениями у = ех и у = е~х соответственно. у=е~х | У 10 1 ! У = 1 1 1 1 1 1 Область определения функций: (—оо, оо). Область значений функций: (0, оо). \ 5 1 1 1 1 1 1 1 1 1 1 1 1 ! t ! Г t -5 —* ► X 5 Задание 2.4. Постройте в одной и той же системе координат графики функций /(ж) = ех, д(х) = 2х и /г(х) = 3х. При каких значениях х графики функций пересекаются? Графики каких функций лежат выше и ниже графиков двух остальных функций соответственно при условии, что переменная х принимает положительные значения? Как изменится ответ, если переменная х принимает отрицательные значения? ■ Решение задач, описывающих процессы роста и распада, с помощью показательных функций В Решение большинства задач, в которых смоделированы процессы роста или распада, требует использования показательной функции с основанием е. В данном разделе приведены два примера решения подобных задач. Намного больше задач такого типа предлагается для самостоятельного решения в практикуме 2.2. Е Пример 2.8 (Экспоненциальный рост). Холера — кишечное заболевание, которое вызывается бактериями холеры. Бактерии размножаются путем деления клеток. Рост количества бактерий в зараженном организме приближенно можно описать следующим показательным уравнением. N = Mjc1’386*,
164 Часть I. Элементарные функции где N — количество бактерий в организме через t часов после заражения, а No — количество бактерий в организме в момент заражения (начало отсчета времени t = 0). При условии, что в начальный момент времени в организме присутствует 25 бактерий холеры, определите (с точностью до одной бактерии), сколько их будет насчитываться в организме через указанное время. 1. 0,6 ч. 2. 3,5 ч. Решение. Подставляя в приведенное выше уравнение Nq = 25, получаем следу¬ ющую формулу. N = 25e1,386t График этого уравнения изображен на рис. 2.17 N Рис. 2.17. Количество бактерий холеры через t часов после заражения 1. Находим значение N при t = 0,6. N = 25ех’386(0’6) = 57 бактерий Используйте калькулятор 2. Находим значение N при t = 3,5. N = 25е1>386(3’5) = 3 197 бактерий Используйте калькулятор ■ Упражнение 2.8. Пусть количество бактерий холеры увеличивается по экспоненциальному закону, рассмотренному в примере 2.8. При условии, что в момент заражения в организме присутствует 55 бактерий, определите (с точностью до одной бактерии), сколько их будет присутствовать в организме через указанное время. 1. 0,85 ч. 2. 7,25 ч.
Глава 2. Другие элементарные функции 165 В Пример 2.9 (Радиоактивный распад). Под воздействием космических лучей в земной атмосфере образуются нейтроны. В результате столкновения нейтронов с атомами азота образуется радиоактивный изотоп углерода 14С. Этот изотоп проникает в ткани живых организмов (в первую очередь растений) вместе с углекислым газом. На протяжении всей жизни растения или животного содержание изотопа углерода 14С в его тканях поддерживается на постоянном уровне. Сразу же после смерти организма содержание изотопа 14С в его тканях начинает уменьшаться по следующему закону. А = A0e-0’000124t, где А — содержание углерода в организме через t лет после его смерти, Ао — содержание углерода в организме в момент смерти t = 0. Предположим, что в пробе, взятой из черепной кости непосредственно после смерти человека, содержалось 500 мг изотопа 14С, определите, сколько миллиграммов этого изотопа останется в этой пробе через указанное время. 1. 15 000 лет. 2. 45 000 лет. Ответ вычислите с точностью до двух десятичных знаков. Решение. Подставляя значение Aq = 500 в уравнение экспоненциального распада, получаем следующий результат. А = бООе-0,0001244 График уравнения изображен на рис. 2.18 1. Находим значение А при t = 15 000. А = 500е~0’000124(15 000) _ 77,§4 мг Используем калькулятор 2. Находим значение А при t = 45 000. А = 500е“0»000124(45 000) _ мг Используем калькулятор ■
166 Часть I. Элементарные функции Рис. 2.18. Содержание изотопа 14С в черепной кости через t лет после смерти человека В Упражнение 2.9. Вернемся к рассмотрению модели экспоненциального распада, описанной в примере 2.9. Определите, сколько миллиграммов изотопа 14С содержалось в пробе, взятой из черепной кости непосредственно после смерти человека, если через 18 000 лет в пробе содержится 25 мг изотопа. Ответ округлите до миллиграмма. ■ Задание 2.5. 1. Постройте в одной и той же системе координат графики трех уравнений экспоненциального распада А = Age-0,35*, t 0 при Aq = 10, 20, и 30 соответственно. 2. Найдите асимптоты графиков, построенных при решении п. а. 3. Опишите, как ведут себя функции, заданные уравнениями из п. а, при больших временных значениях. ■ Пример 2.10 (Амортизация). В табл. 2.10 приведены данные о ры- ! ночной стоимости микроавтобуса (долл.) через х лет после его покупки. Определите коэффициенты показательного регрессионного уравнения вида у = abx, наиболее адекватно описывающей табличные данные. Используя это уравнение, вычислите текущую цену микроавтобуса. Оцените его рыночную стоимость через 10 лет после покупки. Ответы округлите до ближайшего доллара. Решение. Введем данные, приведенные в табл. 2.10, в графическую утилиту (рис. 2.19, а), а затем, используя соответствующие команды, найдем коэффициенты искомого уравнения регрессии (рис. 2.19, б), С помощью найденного уравнения, находим текущую цену микроавтобуса: у\(0) = 14910 долл. Исходные данные и график уравнения регрессии, отображенные в окне графической утилиты, показаны на рис. 2.19, в. Пользуясь соответствующими командами графической
Глава 2. Другие элементарные функции Таблица 2.10. Рыночная стоимость микроавтобуса х Стоимость, долл. И 12 575 2 9455 3 8115 4 6845 5 5225 6 4485 L1 L2 1 2 3 Ч 5 б 12575 9455 В115 6045 5225 ЧЧВ5 L3(D= а) ExpRe9 у«а*Ьлх а«14910.20311 Ь«.8162940177 167 Рис. 2.19. Вычисление стоимости микроавтобуса утилиты, определим ординату точки графика, абсцисса которой равна 10, и тем самым найдем стоимость микроавтобуса через 10 лет после его покупки. Она равна 1959 долл. ■ Упражнение 2.10. В табл. 2.11 приведены данные о роночной стоимости автомобиля класса ‘люкс’ (долл.) через х лет после его покупки. Определите коэффициенты показательного регрессионного уравнения вида у = = abx, наиболее адекватно описывающей табличные данные. Пользуясь найденным уравнением, вычислите текущую цену автомобиля. Оцените его рыночную стоимость через 10 лет после приобретения. Ответы округлите до целых чисел.и Таблица 2.11. Стоимость автомобиля класса “люкс” х Стоимость, долл. Л 23125 2 19050 3 15625 4 11875 5 9450 6 7125
168 Часть I. Элементарные функции Сложные проценты В этом разделе рассматривается рост денежной суммы в результате начисления на нее сложных процентов. Процентом называется плата за пользование отданными в ссуду деньгами. Обычно процент вычисляется как сотая доля от основной суммы (называется также процентной ставкой) за определенный период времени. Если при наступлении срока выплаты процентов причитающиеся начисления повторно инвестируются по той же ставке, то в будущем периоде процент начисляется как на основную сумму, так и на уже начисленные проценты. Проценты, выплачиваемые на повторно инвестированные суммы, называются сложными. Сложные проценты вычисляются по следующей формуле. Сложные проценты Если основная сумма Р (текущая стоимость) инвестируется под сложные проценты по годовой процентной ставке г (выраженной в виде десятичного числа), начисляемой т раз в год, то сумма А (будущая стоимость), которая будет лежать на счете через t лет, выражается формулой (г \ mt 1 + -) . т/ [Примечание. Принятые выше обозначения не позволяют заменить символ Р символом Ад.] И Пример 2.11 (Начисление денег по сложным процентам). На банковский счет положена денежная сумма размером 1000 долл. Сложные проценты выплачиваются ежемесячно из расчета 10% годовых. Какая сумма денег будет на счете через 10 лет? Ответ округлите до цента. Решение. Воспользуемся формулой начисления сложных процентов. (г \ mt 1 + -) = т/ = 1000 1210 Используйте калькулятор = 2707,04 долл. График уравнения А = 1000 (1 + j на отрезке 0 t 20 изображен на рис. 2.20. ■ В Упражнение 2.11. Определите, какая сумма будет на счете через 5 лет, если на сумму размером 5000 долл, ежедневно начисляются сложные проценты по процентной ставке 9% годовых. Ответ округлите до цента. ■
Глава 2. Другие элементарные функции 169 Рис. 2.20. Результаты начисления сложных процентов Задание 2.6. Предположим, что на депозитный счет в банке положена сумма 1000 долл, под 5% годовых. Оцените в уме сумму денег, которая будет на счете к концу первого года, при условии, что сложные проценты начисляются 1) ежеквартально, 2) ежемесячно, 3) ежедневно и 4) ежечасно. Затем воспользуйтесь формулой начисления сложных процентов и для каждого варианта определите размер суммы на счете к концу первого года с точностью до цента. Оцените точность вычислений, произведенных в уме. ■ Непрерывное начисление сложных процентов | Вернемся к рассмотрению формулы начисления сложных процентов. / г \ mt А = Р(1 + -) . \ т/ Предложим, что основная сумма Р, годовая процентная ставка г и срок вклада t остаются неизменными, а количество периодов времени в году, по истечении которых начисляются сложные проценты, т, неограниченно растет. Как при этом будет изменяться накопленная сумма А: неограниченно возрастать или стремиться к некоторому предельному значению? Для начала положим Р равным 100 долл., г = 0,08 и t = 2 года. При помощи калькулятора составим таблицу значений накопленной суммы для некоторых значений величины т (табл. 2.12). Из табл. 2.12 следует, что накопленная сумма имеет наибольший прирост, если сложные проценты начисляются не раз в год, а раз в полгода. При дальнейшем увеличении т рост накопленной суммы замедляется. Оказывается, что при увеличении т накопленная сумма А приближается к величине 117,35 долл.
170 Часть I. Элементарные функции Таблица 2.12. Начисление сложных процентов Периодичность начисления сложных процентов т А = 100 (1 + (^*)2ГПдолл. Ежегодно 1 116,6400 Раз в полгода 2 116,9859 Ежеквартально 4 117,1659 Еженедельно 52 117,3367 Ежедневно 365 117,3490 Ежечасно 8760 117,3510 Можно показать, что при увеличении количества периодов времени т, по истечении которых начисляются сложные проценты, накопленная сумма, выражающаяся формулой (г \ mt 1 + -) > т/ стремится к величине Pert. Последнее выражение известно как формула непрерывного начисления сложных процентов. Эта формула широко используется в бизнесе, банковской сфере и экономике в целом. Формула непрерывного начисления сложных процентов Если на основную стоимость Р, инвестированную по годовой процентной ставке г (выраженной в виде десятичного числа), непрерывно начисляются сложные проценты, величина накопленной суммы А по истечении t лет определяется формулой А = Pert. И Пример 2.12 (Ежедневное и непрерывное начисление сложных процентов). Какая сумма денег будет накоплена на счете через два года, если текущая стоимость вложенных средств составляет 5000 долл., ежегодная процентная ставка равна 8%, а сложные проценты начисляются: 1. ежедневно; 2. непрерывно. Ответ округлите до цента. Решение. 1. Воспользуемся формулой начисления сложных процентов. (г \ mt 1 + -) • ТП/ Подставим в нее следующие значения: Р = 5000; г = 0,08; т = 365 и t = 2. 365-2 = 5867,45 ДОЛЛ. Используйте калькулятор А = 5000 (1 + ^ \ ООО
Глава 2. Другие элементарные функции 171 2. Воспользуемся формулой непрерывного начисления сложных процентов А = Pert. Подставим в нее следующие значения: Р = 5000; г = 0,08; t = 2. А = 5ОООе0’08'2 = 5867,55 ДОЛЛ. Используйте калькулятор g И Упражнение 2.12. Определите размер денежной суммы, которая будет накоплена за 1,5 года при условии, что текущая стоимость вклада равна 8000 долл., процентная ставка составляет 9% годовых, а сложные проценты начисляются по следующим правилам. 1. ежедневно; 2. непрерывно. Ответы вычислите с точностью до цента. ■ Для удобства формулы расчета накопленной стоимости при начислении на основную сумму простых процентов, сложных процентов, а также непрерывном начислении сложных процентов, обобщены ниже. Формулы начисления процентов Простые проценты А = Р(1 + rt) Сложные проценты / г \ А = Р(1 + — ] \ тп/ Непрерывное начисление сложных процентов А = Pert 2.8. 1) 179 бактерий; 2.9. 233 мг. 2) 1271 659 бактерий.
172 Часть I. Элементарные функции 2.10. Покупная цена: 30363 долл.; стоимость через 10 лет: 2864 долл. ExpRee у=а*Ьлх а=3@363.17638 Ь=.7896877851 2.11. 7841,13 долл. 2.12. 1) 9155,23 долл. 2) 9156,29 долл. Практикум 2.2 А 1. С каждым приведенным ниже уравнением сопоставьте функции f,g,hn к, графики которых изображены на рисунке. а) у = 2х. в) у = 4®. 2. Сопоставьте с каждым приведенным ниже уравнением одну из функций /, д, h и к, графики которых изображены на рисунке. . /IV а) У= • в) у - 51.
Глава 2. Другие элементарные функции 173 В задачах 3-14 постройте графики данных функций в указанных интервалах значений независимой переменной. З.у = 5®; [-2; 2]. 5. У = Q) = 5"®; [-2; 2]. 7. /(х) = -5®; [—2; 2]. 9. у = -е~®; [-3; 3]. 11. у = ЮОе0-1®; [-5; 5]. 13. g(i) = 10e-°’2t; [-5; 5]. 4. у = 3®; [-3; 3]. 6- У = Q) = 3-®; [-3; 3]. 8. р(х) = -3®; [-3; 3]. 10. у = —е®; [-3; 3]. 12. у = Юе0’2®; [-10; 10]. 14. /(t) = 100e~o,lt; [-5; 5]. В задачах 15-20 упростите указанные выражения. 15. (43®)2". 16. IO3®-1™4"®. «z-З „х 17. j. 18. п. е®-4 е1-® 19. (ге1’2*)3. 20. (Зе"1-4®)2. Б В задачах 21-28 опишите, путем каких преобразований из графика функции f можно получить график функции g (см, раздел 1.2). 21. д(х) = -2х; /(ж) = 2х. 22. д(х) = 2х"2; /(ж) = 2х. 23. д(х) = 3x+1; f(x} = 3х. 24. д(х) = -3х; /(ж) = 3х. 25. д(х) = ех + 1; /(ж) = ех. 26. д(х) = ех — 2; f(x) = ех. 27. д(х) = 2е"(х+2); /(ж) = е“х 28. д(х) = f(x) = е~х. Zl Проверьте правильность своих ответов к задачам 21-28. Для этого постройте графики каждой пары функций в одном и том же окне используемой вами графической утилиты. 29. График функции / известен и приведен на рисунке ниже. Изобразите схематически графики следующих функций. а) У = - 1. 6) у = У(х 4- 2). в) у = 3/(х)-2. г) !/ = 2-/(х-3). Рисунок к задачам 29 и 30 30. График функции / известен и приведен на рисунке выше. Изобразите схематически графики следующих функций.
174 Часть I. Элементарные функции 32. G(t) = з*/100; [-200; 200]. 34. у = 2 + е1"2; [-1; 5]. 36. у = [-3; 3]. 38. М(х) = е*/2 + е"*/2; [-5; 5]. 40. у = 2-*2; [-3; 3]. а) у = /(х) + 2. б) |/ = /(х-3). в) у = 2/(х) - 4. г) у = 4 - /(х + 2). В задачах 31-40 постройте графики данных функций в указанном интервале значений независимой переменной. 31. /(f) = 2‘/10; [-30; 30]. 33. у = -3 + е1+х; [-4; 2]. 35. у = е^; [-3; 3]. ех -к е~х 37. С(х) = —; [-5; 39. у = е"*2; [-3; 3]. * 41. Найдите все действительные числа а, для которых справедливо равенство а2 = = а~2. Объясните, почему приведенное равенство не противоречит второму свойству показательных функций. * 42. Найдите все действительные числа а и Ь такие, что а Ь, ддя которых справед¬ ливо равенство а4 = Ь4. Объясните, почему приведенное равенство не противоречит третьему свойству показательных функций. В задачах 43-48 решите указанные уравнения относительно х. 43. 102-3х = 105х"6. 44. 53х = 54х~2. 45. 45х-х2 = 4-6. 46. 7х* = ?2я+з 47. 53 = (х + 2)3. 48. (1 - х)5 = (2х - I)5. В В задачах 49-52 решите указанные уравнения относительно переменной х. (Помните: ех/0и е~х 0 при любых значениях х.) 49. (х — 3)еж = 0. 50. 2хе~х = 0. 51. Зхе~х 4- х2е~х = 0. 52. х2ех — 5хех = 0. В задачах 53-56 постройте графики указанных функций в заданном интервале значений независимой переменной. 53. h(x) = х(2х); [-5; 0]. 54. т(х) = х (3-*); [0; 3]. s6'w = fT^;l°;5l- 55. W = 58. /(х) = 5 - 3-*. 60. /(х) = 7 - 2х2 + 2-*. Jeu В задачах 57-60 вычислите с точностью до двух десятичных знаков действительные нули указанных функций. 57. f(x) = 4® - 7. 59. f(x) = 2 + 3x4-10*.
Глава 2. Другие элементарные функции 175 Применение математики Экономика и бизнес 61. Финансы. Предположим, что сумма размером 2500 долл, инвестирована под 7% годовых. При условии, что сложные проценты наращиваются ежеквартально, определите, какая сумма будет накоплена на счете счете через указанный строк. 3 а) - года. б) 15 лет. Ответ округлите до цента. 62. Финансы. Предположим, что сумма размером 4000 долл, инвестирована под 6% годовых. При условии, что сложные проценты наращиваются еженедельно, определите какая сумма будет накоплена на счете через указанный срок. а) | года. б) 10 лет. Ответ округлите до цента. 63. Начисление процентов. На депозитный счет с годовой ставкой 8,35% внесена сумма размером 7500 долл. При условии, что сложные проценты начисляются непрерывно, определите, какая сумма будет накоплена на счете через указанный срок. а) 5,5 лет. б) 12 лет. 64. Начисление процентов. На депозитный счет с годовой ставкой 7,45% внесена сумма размером 5250 долл. При условии, что сложные проценты насчитываются непрерывно, определите сумму на счете через: а) 6,25 года; б) 17 лет. 65. Финансы. Для покупки нового автомобиля вкладчику за 5 лет нужно накопить на счете 15 000 долларов. Какую сумму ему следует положить на счет с непрерывно начисляемой годовой ставкой 6,75%, чтобы успеть в срок. Ответ укажите с точностью до доллара. 66. Финансы. У супружеской пары родился ребенок. Какую сумму родители должны положить в банк сейчас, чтобы через 17 лет иметь возможность потретить на образование ребенка 40 000 долл. На основную сумму ежедневно насчитываются сложные проценты с годовой ставкой 5,5%. Ответ укажите с точностью до доллара. 67. WWW Начисление процентов. Служба BanxQuote представляет собой сеть вебсайтов, на которых в режиме реального времени отображаются предложения по предоставлению услуг крупнейшими игроками мирового финансового рынка. Ниже приведены размеры процентных ставок на годовые депозитные сертификаты некоторых американских банков. Информация взята из их официальных веб-сайтов. а) Банк Stonebridge*. сложная годовая ставка 6,93%, начисляемая ежемесячно. б) Банк DeepGreen*. сложная годовая ставка 6,96%, начисляемая ежедневно. в) Банк Provident*, сложная годовая ставка 6,80%, начисляемая непрерывно. Определите, какая сумма будет накоплена на счете в каждом банке к концу первого года, если первоначальная сумма вклада составляет 10 000 долл.
176 Часть I. Элементарные функции 68. WWW Начисление процентов. Прочтите условие задачи 67. Службой BanxQuote предоставлена следующая информация о размере процентных ставок, выплачиваемых различными банками по 30-месячных депозитных сертификатах. а) Банк Oriental Bank & Trust: сложная годовая ставка 6,50%, начисляемая ежеквартально. б) Банк BMW Bank of North America: сложная годовая ставка 6,36%, начисляемая ежемесячно. в) Банк BankFirst Corporation: сложная годовая ставка 6,30%, начисляемая ежедневно. Определите, какая сумма будет накоплена на счете в каждом банке через 2,5 года, если первоначальная сумма вклада составляет 10000 долл. 69. Текущая стоимость. По истечении срока погашения, который составляет 5| года, по простому векселю выплате подлежит сумма размером 50 000 долл. Определите первоначальную стоимость векселя, при условии, что годовая процентная ставка по векселю равна 8%, а сложные проценты начисляются непрерывно. 70. Текущая стоимость. По истечению срока погашения, который составляет 10 лет, по простому векселю выплате подлежит сумма размером 30 000 долл. Определите первоначальную стоимость векселя, при условии что годовая процентная ставка по векселю равна 7%, а сложные проценты начисляются непрерывно. 71. Реклама. Чтобы ознакомить с новым продуктом максимальное количество потенциальных покупателей, производитель проводит телевизионную рекламную компанию, которая охватывает население некого города и его пригородов. Целевая телевизионная аудитория составляет два миллиона зрителей. Уравнение, описывающее зависимость количества людей ЛГ, которые узнали о новом продукте, от продолжительности (в днях) проведения рекламной кампании, t имеет вид. N = 2 (1 - е”0’037*) Постройте график найденной функции на отрезке 0 t 50. Определите, к какому значению стремится N при безграничном возрастании t? 72. Кривая обучаемости. Компания производит компьютеры. Сотрудники компании, занятые на производстве монтажных плат, проходят предварительное обучение на рабочем месте. Используя методы статистического анализа, была построена так называемая кривая обучаемости, которая характеризует процесс постепенного приобретения опыта среднестатистическим сотрудником компании. Зависимость имеет следующий вид: N = 40 (1 - е~°’12‘), где N — количество монтажных плат, которое сотрудник компании в состоянии произвести в течение дня после t дней обучения. Постройте график функции на отрезке 0 < t < 30. Используя найденное уравнение, определите максимально возможное количество монтажных плат, которое в состоянии произвести среднестатистический сотрудник компании после 1 дня обучения?
Глава 2. Другие элементарные функции 177 73. Заработная плата в спорте. В табл. 2.13 приведена средняя заработная плата (в тысячах долларов) игроков Национальной хоккейной лиги (National Hockey League — NHL) и Национальной баскетбольной ассоциации (National Basketball Association — NBA) в указанные годы, начиная с 1990 г. а) Обозначим через х количество лет, прошедшее с 1990 года. Найдите коэффициенты показательного регрессионного уравнения вида у = аЬху наиболее точно описывающее динамику роста средней заработной платы игроков NHL. Оцените среднюю заработную плату, игроков в 1998 и 2010 году. Ответ округлите до тысячи долларов. б) Средняя заработная плата игроков NHL в 1998 году составила 1167 000 долл. Как данная величина соотносится с величиной средней заработной платы, оцененной в п. а)? Какие изменения следует внести в прогноз средней заработной платы игроков NHL в 2010 году? Аргументируйте свой ответ. Таблица 2.13. Средняя заработная плата, тыс. долл. WWW Год NHL NBA 1990 211 750 1991 271 900 1992 368 1100 1993 467 1300 1994 562 1700 1995 733 1900 1996 892 2000 74. Обратитесь к данным табл. 2.13. а) Пусть х — количество лет, прошедшее с 1990 года. Найдите коэффициенты показательного регрессионного уравнения вида у = abx, наиболее точно описывающей динамику увеличения средней заработной платы игроков NBA. Оцените средние заработные платы баскетболистов (с точностью до тысячи долларов) в 1997 и 2010 году. *б) Средняя заработная плата игроков NBA в 1997 году составила 2 200 000 долл. Как данная величина соотносится с величиной, найденной в п. а? Какие изменения следует внести в прогноз относительно величины средней заработной платы игроков NBA в 2010 году? Аргументируйте свой ответ. Биологические науки 75. Биология морей. Жизнь в морях возможна благодаря микроскопическим растениям, которые обитают в так называемой световой зоне, или, другими словами, приповерхностном слое воды, в котором поглощается 99% интенсивности света, падающего на поверхность воды. Воды некоторых морей содержат большое количество осадка, и толщина световой зоны в них составляет всего лишь
178 Часть I. Элементарные функции 15-20 футов. В некоторых темных гаванях поглощение света слоем воды толщиной d футов приближенно можно описать уравнением вида: I = /0е~°’23</. Определите, какой процент интенсивности падающего на поверхность воды света достигнет указанной глубины. а) 10 футов. б) 20 футов. 76. Биология морей. Прочтите условие задачи 75. Считается, что вода в Саргассовом море, расположенном в Вест-Индии, одна из самых прозрачных в мире. Интенсивность света I на глубине d футов от поверхности в этом море приближенно можно оценить при помощи формулы: I = I0e-0’00942d, где 10 — интенсивность света, падающего на поверхность воды. Определите, сколько процентов от интенсивности падающего на поверхность воды света, достигнет глубины: а) 50 футов; б) 100 футов. 77. WWW Эпидемия ВИЧ/СПИД. В Объединенной программе Организации Объединенных Наций по борьбе с эпидемией ВИЧ/СПИД утверждается, что к 1999 году вирусом иммунодефицита человека (Human Immunodeficiency Virus — HIV) во всем мире будет инфицировано 47,3 миллиона человек. Согласно проведенным исследованиям в ближайшие годы заболевание будет продолжать непрерывно распространяться на 11,6% в год. В качестве начальной точки отсчета выберем 1998 год. (Предполагается, что темпы распространения заболевания в ближайшей перспективе не изменятся.) а) Составьте уравнение, описывающее рост численности ВИЧ-инфицированных по сравнению с 1998 годом. б) Используя найденное уравнение, оцените (с точностью до ближайшей тысячи) количество людей, которое будет инфицировано к концу 2005 года или к концу 2010 года. в) Изобразите схематически график найденного уравнения на временном отрезке с 1998 по 2010 год. 78. WWW Эпидемия ВИЧ/СПИД. В Объединенной программе Организации Объединенных Наций по борьбе с эпидемией ВИЧ/СПИД утверждается, что к 1999 году количество умерших от синдрома приобретенного иммунодефицита (Acquired Immune Deficiency Syndrome — AIDS) во всем мире составит 13,9 миллионов
Глава 2. Другие элементарные функции 179 человек. Согласно сделанным прогнозам, смертность, вызванная заболеванием, в ближайшие годы будет непрерывно расти на 16,5% в год. В качестве точки начала отсчета выберем 1998 год. (Предполагается, что уровень смертности в ближайшей перспективе не изменится.) а) Составьте уравнение, которое описывает уровень смертности в мире, вызванной синдромом приобретенного иммунодефицита, начиная с 1998 года. б) Используя найденное уравнение, определите, сколько всего человек в мире умрет от заболевания к концу 2005 года или к концу 2010 года? (Ответ укажите с точностью до ста тысяч человек.) в) Изобразите график найденного уравнения на временном отрезке с 1998 по 2010 год. Социальные науки 79. Рост численности начеления. С момента возникновения человечества и до 1830 года численность населения планеты впервые достигла миллиарда человек. Население Земли увеличилось до двух миллиардов всего лишь за 100 лет (к 1930 году). За следующие 60 лет (к 1990 году) население планеты увеличилось еще на 3 миллиарда. В 1995 году численность населения земного шара оценивалось в 5,7 миллиарда человек. Согласно прогнозам Международного банка (World Bank), которые были сделаны в 1994 году, до 2030 года ежегодный непрерывный прирост населения планеты будет составлять 1,14%. а) Постройте модель, описывающую динамику роста численности населения планеты, приняв в качестве начальной точки отсчета 1995 год. б) Исходя из полученной модели, спрогнозируйте численность населения планеты (с точностью до ста миллионов) в 2010 году и в 2030 году. в) Изобразите график уравнения, найденного в п. а, на временном отрезке с 1995 по 2030 год. 80. Рост численности населения в Эфиопии. В 1995 году население Эфиопии оценивалось в 88 миллионов человек. По прогнозам банка World Bank, сделанным в 1994 году, до 2030 года ежегодно численность населения этой страны будет непрерывно возрастать на 1,67%. а) Постройте модель, описывающую динамику роста численности населения Эфиопии, приняв в качестве начальной точки отсчета 1995 год. б) Исходя из полученной модели, спрогнозируйте численность населения страны (с точностью до миллиона) в 2010 году. В 2030 году. в) Изобразите схематически график уравнения, найденного в п. а на временном отрезке начиная с 1995 по 2030 год. 81. Рост количества пользователей Интернета. С 1994 по 2000 год количество хостов в Интернете стремительно увеличилось (см. табл. 2.14). а) Обозначим через х количество лет, которое прошло с 1994 года. Найдите коэффициенты показательного регрессионного уравнения вида у = abx, наиболее точно описывающего табличные данные. С помощью найденного уравнения оцените количество хостов, которое ожидается в Интернет к 2010 году. (Ответ округлите до миллиона.)
180 Часть I. Элементарные функции Таблица 2.14. Количество хостов в Интернете WWW Год Количество хостов, млн. 1994 2,4 1995 4,9 1996 9,5 1997 16,1 1998 29,7 1999 43,2 2000 72,4 Источник данных: организация Internet Software Consortium, б) Объясните, какие выводы следуют из найденной модели, при условии, что количество хостов будет расти с той же скоростью. 82. Ожидаемая продолжительность жизни, В табл. 2.15 приведены значения ожидаемой продолжительности жизни (в годах) жителей Соединенных Штатов Америки, родившихся с 1970 по 1997 год. Обозначим через х количество лет, которое прошло с 1970 года. Найдите показательное регрессионное уравнение, наиболее точно описывающее табличные данные. С помощью найденного уравнения оцените среднюю ожидаемую продолжительность жизни человека, который родится в 2010 году. Таблица 2.15. Средняя ожидаемая продолжительность жизни WWW Год рождения Продолжительность жизни, лет 1970 70,8 1975 72,6 1980 73,7 1985 74,7 1990 75,4 1995 75,9 1997 76,5
Глава 2. Другие элементарные функции 181 2.3. Логарифмическая функция ■ Обратная функция ■ Логарифмическая функция ■ Свойства логарифмических функций ■ Вычисление логарифмов при помощи калькулятора ■ Решение практических задач и ех , соответствующие одно- Найдите на панели калькулятора клавиши 10д именным показательным функциям. Рядом с ними расположены клавиши LOG и LN которыми обозначены одноименные логарифмические функции (logarithmic functions). Логарифмические и показательные функции, которые соответствуют расположенным по соседству клавишам, тесно связаны между собой. Действительно, показательная и соответствующая ей логарифмическая функция, являются обратными по отношению друг к другу. Понятие обратной функции (inverse function) детально рассматривается в этом разделе. Оно будет использовано при определении логарифмической функции как обратной по отношению к показательной. Вслед за определением будут рассмотрены основные свойства логарифмических функций. Далее вы познакомитесь с примерами вычисления значений логарифмических функций при заданном значении х с помощью калькулятора. В конце раздела приведены примеры решения практических задач, которое сводится к вычислению логарифмических выражений. Логарифмические функции играют важную роль. В частности, логарифмическими функциями описывают различные процессы. Решения многих прикладных задач сводятся к вычислению логарифмических выражений. Например, шкала, которой пользуются при измерении интенсивности звука (в децибелах), задается в логарифмических единицах измерения. Сила землетрясений измеряется по шкале Рихтера, которая также является логарифмической. В финансах часто возникает задача нахождения периодов времени, по истечении которого удваивается сумма инвестиций, на которую непрерывно или заданное количество раз в год начисляются сложные проценты. Искомая величина задается показательным уравнением, решением которого будет логарифмическое выражение. Обратная функция Рассмотрим графики функций /(ж) = f и д(х) = Они изображены на рис. 2.21. Поскольку fug — функции, то любому элементу из области определения каждой функции соответствует единственное число из области значений соответствующей функции. А для какой из этих функций справедливо обратное утверждение: каждому значению функции соответствует единственный элемент из области
182 Часть I. Элементарные функции Рис. 2.21. Обратные функции определения? Такое утверждение справедливо лишь для функции /. Так, значению функции /, равному 2, соответствует единственный элемент — число —4. Значению функции д, равному 2, соответствуют два числа: —4 и 4. Функцию f называют однозначной (one-to-one). В общем случае можно дать следующее определение однозначной функции. Однозначная функция Функция f называется однозначной, если каждому ее значению соответствует единственное значение независимой переменной. Можно показать, что любая непрерывная функция, которая монотонно возрастает или убывает на всей области ее существования, является однозначной. Если непрерывная функция ведет себя немонотонно, например, возрастает в каком-либо одном интервале значений независимой переменной и убывает в остальных интервалах, то такая функция не может быть однозначной. Примеры графиков таких функций приведены на рис. 2.17. Задание 2.7. Постройте графики функций /(х) — 2х и д(х) = х2. Для каждой функции определите, при каких значениях независимой переменной значение функции равно 4? Какая из этих функций является однозначной? Объясните, почему. ■ Используя понятие однозначной функции /, можно определить новую функцию, обратную функции /.
Глава 2. Другие элементарные функции 183 Обратная функция Если f — однозначная функция, то функция, обратная функции /, образуется путем перестановки независимых и зависимых переменных функции /. Иначе говоря, если точка (а, Ь) принадлежит графику функции /, точка (Ь, а) принадлежит графику функции, обратной /. {Примечание. Если функция f не является однозначной, то она не имеет обратной функции.) В любой библиотеке элементарных функций существует целый ряд важных функций, которые являются обратными к другим базовым функциям, входящим в эту же библиотеку. В данной книге в качестве примера рассматривается логарифмическая функция, которая является обратной по отношению к показательной. Логарифмическая функция Пусть дана показательная функция /, заданная уравнением: У = 2х. (2.2) После перестановки независимой и зависимой переменной получим функцию, обратную по отношению к функции /. х = 2У. (2.3) Полученная таким образом обратная функция называется логарифмической функцией по основанию 2 и записывается следующим образом. у = log2 х тогда и только тогда, когда х = 2У. Поскольку эти две записи эквивалентны, то, чтобы построить график функции у = log2 х, достаточно построить график функции х = 2У. Любая упорядоченная пара чисел, принадлежащая графику показательной функции, принадлежит и графику логарифмической функции. Для этого достаточно всего лишь переставить числа, составляющие упорядоченную пару. Например, если точка с координатами (3, 8) удовлетворяет уравнению (2.2), точка с координатами (8, 3) удовлетворяет уравнению (2.3). Графики функций у = 2х и у = log2 х изображены на рис. 2.22. Обратите внимание на то, что, если сложить чертеж вдоль прямой линии у = х, которая обозначена на рис. 2.22 пунктиром, то графики функций совпадут. Для графиков этих функций прямая у = х является линией симметрии.
184 Часть I. Элементарные функции Рис. 2.22. Графики показательной и логарифмической функций Показательная функция Логарифмическая функция X У = 2Х х = 2У У -3 1 8 1 8 -3 —2 1 4 1 4 —2 -1 1 2 1 2 -1 0 1 1 0 1 2 2 1 2 4 4 2 3 8 8 3 Упорядоченные пары значений полностью обратимы Вообще говоря, поскольку график показательной функции вида /(ж) = Ьх при b ф 1, b > 0 монотонно возрастает либо монотонно убывает (в зависимости от значения основания) на всей области определения функции (см. раздел 2.2), значит, показательная функция имеет обратную. Логарифмическая функция Функция, обратная показательной, называется логарифмической. Если 6 > 0 и b / 1, то логарифмическая форма записи у = log6x эквивалентна показательной форме записи х = ЬУ, Логарифмом числа х по основанию b называется показатель степени, в которую следует возвести число Ь, чтобы получить число х. (Помните: логарифм — это показатель степени.) Область определения логарифмической функции — множество всех положительных действительных чисел, которые являются областью значений соответствующей показательной функции; область значений логарифмической функции — множество всех действительных чисел, которые являются областью определения соответствующей показательной функции. Типичный график показательной функции, а также график обратной по отношению к ней логарифмической функции изображены на рисунке. В приведенных ниже примерах преобразуйте исходные логарифмические выражения в эквивалентные показательные выражения, или наоборот. Пример 2.13 (Преобразование логарифмических выражений в эквивалентные показательные выражения). Преобразуйте данные логарифмические выражения в эквивалентные показательные выражения:
Глава 2. Другие элементарные функции 185 1. log5 25 = 2. 2. log9 3 = ^. 3. log2 Q) = —2. Решение. 1. Выражение log5 25 = 2 эквивалентно выражению 25 = 52. 2. Выражение log9 3 = j эквивалентно выражению 3 = 91/2. 3. Выражение log2 (|) = —2 эквивалентно выражению | = 2-2. ■ Упражнение 2.13. Преобразуйте указанные логарифмические выражения в эквивалентные показательные выражения: 1- log3 9 = 2. 2. log4 2 = | 3. log3 Q) = —2. Пример 2.14 (Преобразование показательных выражений в логарифмические выражения). Преобразуйте данные показательные выражения в эквивалентные логарифмические выражения: 1. 64 = 43. 2. 6 = г/36. 3. | = 2-3. О Решение. 1. Выражение 64 = 43 эквивалентно выражению log4 64 = 3. 2. Выражение 6 = >/36 эквивалентно выражению log36 6 = |. 3. Выражение | = 2~3 эквивалентно выражению log2 (|) = —3. ■ Упражнение 2.14. Преобразуйте показательные выражения в эквивалентные логарифмические выражения. 1. 49 = 72 . 2. 3 = V9. 3. | = З"1. о ■ Ниже приведено несколько примеров, решив которые, вы получите более полное представление о логарифмической функции, а также более наглядно представите ее взаимосвязь с показательной функцией. В каждом примере дано логарифмическое выражение типа у = logb х. Необходимо найти одну из величин: х, Ь или у. Значение двух остальных величин известно. Значения подобраны так, что задача имеет точное решение, определяемое без использования калькулятора. Пример 2.15 (Решения логарифмического уравнения у = logb х). Найдите значения одной из величин: у, b или х. 1. Вычислите значение у: у — log416.
186 Часть I. Элементарные функции 2. Найдите значение х: log2 х = —3. 3. Определите значение у: у = log8 4. 4. Вычислите значение b: logb 100 = 2. Решение. 1. Логарифмическое уравнение у = log4 16 эквивалентно показательному уравнению 16 = 42/. Отсюда находим, что У = 2- 2. Логарифмическое уравнение log2x = —3 эквивалентно показательному уравнению х = 2-3. Отсюда находим, что 1 1 Х~ 23 ~ 8’ 3. Логарифмическое уравнение у = log8 4 эквивалентно показательному уравнению 4 = 8У или 22 = 23у. Отсюда находим, что Зу = 2, 2 у=з- 4. Логарифмическое уравнение log^lOO = 2 эквивалентно показательному уравнению 100 = Ь2. Отсюда находим, что Ь = 10. Основание логарифма не может быть отрицательным д Упражнение 2.15. Найдите значение одной из величин: у, Ь или х. 1. Вычислите значение у: у = log9 27. 2. Найдите значение х: log3 х = — 1. 3. Определите значение Ь\ logb 1000 = 3. ■
Глава 2. Другие элементарные функции 187 Свойства логарифмических функций Логарифмические функции обладают многими примечательными и необычайно полезными свойствами. Опишем восемь основных свойств логарифмических функций в виде теоремы 2.2. Теорема 2.2 (Свойства логарифмических функций). Пусть b, М и N — положительные действительные числа, причем Ь / 1,арих — действительные числа. Тогда справедливы следующие соотношения. 1. logbl = 0. 2. logb6 = l. 3. logftb® = х. 4. blogbx = х, х > 0. М 5. logb MN = logb M + logb N. 6. logb — = logb M - log6 N. 7. logbA/P =plogbM. 8. logb M = logb N тогда и только тогда, когда М = N. а Из приведенных в виде теоремы 2.2 свойств логарифмических функций первые четыре следуют непосредственно из определения логарифмической функции. Упрощенное доказательство свойства 5 приведено ниже. Остальные свойства доказываются аналогичным образом. Итак, пусть и = logb М и v = logb N. Преобразуем эти логарифмические выражения в эквивалентные им показательные выражения. М = Ьи и N = bv. Теперь посмотрите на последовательность приведенных ниже преобразований и убедитесь, что вам понятен каждый логический переход. logb MN = logb bubv = logb b“+r = и + v = log6 M + logb N. Пример 2.16 (Применение свойств логарифмических функций). 1. logb — := iogb wx - logb yz = logb w + logb x - (log6 у + log6 z);= yz = logb w + logb x - log6 у - logb z. 2. log6(wx)3/5;= |logbwx\= |(logbw + logb;r). B Упражнение 2.16. Упростите данные логарифмические выражения аналогично примеру 2.16. 1 1 Д т 1 (RV/3 1,ogbsr' 2’ ogb\sJ ‘
188 Часть I. Элементарные функции Приведенные ниже примеры и упражнения выглядят несколько надуманно. Тем не менее они помогут вам более глубоко изучить основные свойства логарифмических функций. Пример 2.17 (Решение логарифмических уравнений). Найдите значения х, для которых справедливо равенство: 3 2 - logb 4 - - logb 8 + logb 2 = log6 x. Решение. 3 2 - logb 4 - - logb 8 + logb 2 = log6 x, logb 43^2 — logb 82^3 + logb 2 = logb x, Свойство 7 log6 8 - logb 4 + log6 2 = logb x, , 8,2 , logfe = logb хч Свойства 5 и 6 logb 4 = logb ж, X = 4. Свойство 8 ■ Упражнение 2.17. Найдите значения х, для которых справедливо равенство: з logb 2 + | logb 25 - logb 20 = logb x- Пример 2.18 (Решение логарифмических уравнений). Решите уравнение logio х + logio(z + 1) = logio 6. Решение. logio х + logio О + 1) = logio 6, log10[a;(a? + 1)] = log10 6, Свойство 5 x(x + 1) = 6, Свойство 8 X2, + x — 6 = 0, Разложим на множители (т + 3)Сг-2) = 0, х = — 3 и х = 2. Значение х, равное —3, следует исключить, поскольку функция log10(x +1) существует, только если х > —1, т.е. на интервале (—1, сю). Следовательно, значение х = 2 является единственным решением уравнения. ■
Глава 2. Другие элементарные функции 189 Упражнение 2.18. Решите уравнение log3 х + log3(z - 3) = log3 10. в Задание 2.8. Ниже приведено несколько пар логарифмических выражений. Объясните, как связаны между собой выражения каждой пары. Если выражения, составляющие пару, эквивалентны, докажите, что это так. Если нет — приведите примеры эквивалентных выражений. i.log^-log^;^. 3. logb М + log6 АГ; log6 MN. 2. logbM-log6AT; logb —. 4. logb M + logb AT; log6(M + N). ■ Вычисление логарифмов при помощи калькулятора Из всех возможных значений наиболее часто приходится работать с логарифмами, имеющими основание е или 10. Прежде чем приступить к решению прикладных задач, следует научиться находить приближенное значение логарифма любого положительного числа по основанию 10 или е. Важное значение имеет и обратная задача — определение числа по известному значению его логарифма с основанием 10 или е. В недалеком прошлом значения логарифмов находили при помощи специальных таблиц. В настоящее время для выполнения этой задачи используют калькуляторы. Вычисление значений логарифмов при помощи калькулятора намного быстрее и точнее, чем нахождение значения того же логарифма в специальных таблицах. Десятичным логарифмом (или логарифмом Бриггса) называется логарифм числа по основанию 10. Натуральным логарифмом (или логарифмом Непера) называется логарифм числа по основанию е. Большинство моделей калькуляторов имеет клавиши, обозначенные символами ‘log’ (или ‘LOG’) и ‘1п’ (или ‘LN’). Первый набор символов обозначает десятичный логарифм (логарифм по основанию 10), а второй — натуральный логарифм (логарифм по основанию е). Фактически оба символа, ‘log’ и ‘In’, широко используются в математической литературе. Всякий раз, когда в книге встречается логарифм, основание которого не указано, такую запись следует интерпретировать следующим образом. Обозначения логарифмов Десятичный логарифм: lg х = log10 х. Натуральный логарифм: In а; = loge х. Вычисление значения логарифма при помощи калькулятора — задача достаточно простая. На некоторых моделях калькуляторов для этого достаточно ввести число, принадлежащее области определения логарифмической функции, и нажать
190 Часть I. Элементарные функции клавишу LOG или |LN . При использовании других моделей калькуляторов последовательность действий выглядит несколько иначе: сначала нужно нажать клавишу LOG или LN , затем ввести число, принадлежащее области определения логарифмической функции, а затем нажать клавишу ENTER |. Чтобы узнать, какой способ вычисления значений логарифмов реализован на вашем калькуляторе, обратитесь к руководству пользователя, которое прилагается к калькулятору. Пример 2.19 (Вычислите значения логарифмов при помощи калькулятора). Используя калькулятор, вычислите значение указанных логарифмов с точностью до шести десятичных знаков: 1. Ig3184. 3. lg(—3,24). 2. In 0,000349. Решение. 1. lg 3184 = 3,502973. 3. lg(—3,24) = Error3 2. In 0,000349 = -7,960439. Число —3,24 не принадлежит области определения логарифмической функции Упражнение 2.19. При помощи калькулятора вычислите значение данных логарифмов с точностью до шести десятичных знаков. 1. lg 0,013529. 2. In 28,69328. 3. ln(—0,438). Теперь мы рассмотрим вторую из упомянутых выше задач: задачу определения числа по известному значению его логарифма. При решении подобного рода задач широко используется взаимосвязь, существующая между логарифмическими и показательными функциями. Эта взаимосвязь следует непосредственно из определения логарифмической функции, которое было приведено в начале раздела. Взаимосвязь между логарифмическими и показательными функциями Функция Igx = у соответствует функции х = 1О27. Функция In ж = у соответствует функции х = еу. Пример 2.20 (Поиск числа х по известному значению его логарифма logb х = = у). Найдите значение х с точностью до четырех десятичных знаков при условии, что известны значения следующих логарифмов. 3В некоторые калькуляторы заложено более сложное определение логарифмической функции, значениями которой могут быть комплексные числа. При вычислении значения логарифма отрицательного числа, например lg(—3,24), ответ на экране такого калькулятора будет представлен в виде упорядоченной пары действительных чисел. Такой результат является признаком того, что число, логарифм которого вычисляется, не принадлежит области существования традиционной логарифмической функции, определение которой дано в книге.
Глава 2. Другие элементарные функции 191 1. Igx = —2,315. 2. In ж = 2,386. Решение. 1. lgz = -2,315 Преобразуем логарифмическое выражение в эквивалентное показательное выражение. х = IO-2-316 = 0,0048. Воспользуемся калькулятором. 2. In а: = 2,386 Преобразуем логарифмическое выражение в эквивалентное показательное выражение. т _ р2,386 = 10,8699. Воспользуемся калькулятором. ■ Упражнение 2.20. Найдите значение х с точностью до четырех десятичных знаков при условии, что известны значения следующих логарифмов. 1. In ж = -5,062. 2. Igs = 2,0821. ■ Пример 2.21 (Решение показательных уравнений). Найдите значение х с точностью до четырех десятичных знаков. 1. 10® = 2. II 00 r*) co II ri Решение. 1. 10® = 2 Вычислим десятичный логарифм обеих частей уравнения. 2. IglO® = lg2 х = 1g 2 = 0,3010. е® = 3 Воспользуемся свойством 3. Воспользуемся калькулятором. Вычислим натуральный логарифм обеих частей уравнения. 3. In е® = In 3 х = 1пЗ = 1,0986. 3® = 4 Воспользуемся свойством 3. Воспользуемся калькулятором. Вычислим натуральный либо десятичный логарифм обеих Ig3® = lg4 х 1g 3 = 1g 4 lg4 x = T~z = lg3 = 1,2619. частей уравнения (например, десятичный логарифм). Воспользуемся свойством 7. Решим уравнение относительно переменной ®. Воспользуемся калькулятором. ■
192 Часть I. Элементарные функции Существует также и графический способ решения показательных уравнений. Чтобы решить такое уравнение, нужно построить графики обеих частей показательного уравнения и найти точки их пересечения. Графический способ решения уравнений из примера 2.21 проиллюстрирован на рис. 2.23. у2 = 2 у2=3 у2 =4 Рис. 2.23. Графический способ решения показательных уравнений Упражнение 2.21. Найдите значение х с точностью до четырех десятичных знаков: 1. 10х = 7. 2. ех = 6. 3. 4х = 5. ■ Задание 2.9. Объясните, как с помощью калькулятора, на панели которого имеется клавиша, обозначающая натуральный либо десятичный логарифм, найти значение логарифмического выражения у = log5 38,25. [Указание. Сначала преобразуйте логарифмическое выражение в эквивалентное показательное выражение.] ■ Решение практических задач Е Привлекательность разного рода инвестиций весьма удобно оценивать по их времени удвоения, т.е. периода времени, по истечении которого основная сумма удваивается. Решая пример 2.22, вы убедитесь, что в задачах на определение времени удвоения вложенных средств логарифмические функции находят чрезвычайно широкое применение. И Пример 2.22 (Время удвоения капитала). Через какой период времени капиталовложения удвоятся, если на их сумму ежегодно начисляются сложные проценты со ставкой 20%? Ответ округлите до следующего полного года. Решение. Воспользуемся формулой начисления сложных процентов, которая обсуждалась в разделе 2.2: (р \ mt 1 Н ) Формула начисления сложных процентов т/
Глава 2. Другие элементарные функции 193 Очевидно, что задача состоит в нахождении значения величины t при заданных значениях остальных величин: г = 0,20, т = 1 и А = 2Р. Следовательно, задача сводится к решению показательного уравнения. 2Р = Р(1 + 0,2)\ 2 = 1,2‘, 1,2* = 2, 1п1,2* = 1п2, tin 1,2 = In 2, In 2 t = = In 1,2 = 3,8 года « 4 года. Решим уравнение относительно t. Для этого следует вычислить натуральный или десятичный логарифм обеих частей уравнения. Вычислили натуральный логарифм. Воспользуемся свойством 7. Воспользуемся калькулятором. [Примечание: (1п2)/(1п 1,2) =/ 1п2 — 1п1,2] Ответ округляем до следующего полного года. Таким образом, если проценты выплачиваются в конце 3-го года, то к этому времени сумма капиталовложеений еще не успеет вырасти в два раза; если же проценты выплачиваются в конце 4-го года, то сумма, накопленная сумма, будет превосходить основную сумму чуть больше чем в два раза. ■ Уравнение из примера 2.22 можно решить графическим способом. Для этого следует построить графики обеих частей уравнения 2 = 1,2* и найти точку их пересечения (рис. 2.24). Рис* 2.24. Определение точки пересечения графиков правой, у\ = 1,2®, и левой, у2 = 2, частей уравнения В Упражнение 2.22. Через какой интервал времени основная сумма будет удвоена, если на нее ежегодно начисляются сложные проценты с годовой процентной ставкой 13%? Ответ округлите до следующего полного года. ■ Любопытно, а с практической точки зрения и поучительно, будет построить график зависимости времени удвоения суммы вложенных средств от величи-
194 Часть I. Элементарные функции ны ежегодно начисляемых сложных процентов. Выполним следующие преобразования. А = Р(1 + г)‘, 2Р = Р(1 + г)‘, 2 = (1 + т)‘, (1 + т)‘ = 2, 1п(1 + т)‘= 1п2, Яп(1 + г) = In 2, _ In 2 ln(l + r)’ График последнего уравнения приведен на рис. 2.25. Время удвоения, отложенное по оси ординат, измеряется в годах. Годовая процентная ставка, отложенная по оси абсцисс, меняется в диапазоне от 1 до 70% и на рисунке представлена в виде десятичного числа. Сложные проценты начисляются ежегодно. Обратите внимание на то, что время удвоения уменьшается чрезвычайно быстро при изменении величины процентной ставки от 1 до 20% (или от 0,01 до 0,20 при выражении десятичным числом). Рис. 2.25. График зависимости времени удвоения вложенных средств от величины процентной ставки Ответы к упражнениям 2.13. 1) 9 = З2. 2) 2 = 41/2. 2.14. 1) log7 49 = 2. 2)log93=J. А 3) \ = г-. 3) logs Q) = -1.
Глава 2. Другие элементарные функции 195 2.15. 1) у=|. 2) х = |. Z о 3) b = 10. 2.16. 1) logb R - logb S - logb T. 2) |(logbfl-log65). О 2.17. х = 2. 2.18. х = 5. 2.19. 1) —1,868734. 2) 3,356663. 3) Значение не определено. 2.20. 1) 0,0063. 2) 120,8092. 2.21. 1) 0,8451. 2) 1,7918. 2.22. 9 лет. 3) 1,1610. Практикум 2.3 А В задачах 1-6 преобразуйте указанные логарифмические выражения в эквивалентные показательные выражения. 1. log327 = 3. 3. log10 1 = 0. 5- 1Qg4 8 = |- 2. log2 32 = 5. 4. loge 1 = 0. 6. log927 =|. В задачах 7—12 преобразуйте указанные показательные выражения в эквивалентные логарифмические выражения. 7. 49 = 72. 9. 8 = 43/2. 11. A = bu. 8. 36 = 62. 10. 9 = 272/3. 12. M = bx. В задачах 13-24, не пользуясь калькулятором, вычислите значения указанных выражений. 13. log10 1. 15. loge e. 17. logo 2 0,2. 19. log10 103. 21. log22-3. 23. log10 1000. 14. loge 1. 16. log10 10. 18. log13 13. 20. log10 10-5. 22. log335. 24. log6 36. В задачах 25-30 упростите указанные логарифмические выражения, используя свойства логарифмических функций (см. пример 2.16). 25.1ogb£. 27. logbL5. 29. logb-^. qrs 26. logb FG. 28. logbw15. 30. logb PQR.
196 Часть I. Элементарные функции Б В задачах 31-42, не пользуясь калькулятором, найдите значения х, у или Ь. 31. log3 х = 2. 33. log7 49 = у. 35. logb 10~4 = —4. 37. log4x = -. 32. log2 x = 2. 34. log3 27 = y. 36. logb e~2 = -2- 38. log25 x = £ 39. log1/3 9 = у. 40. log49 Q) = y. 41. logb ЮОО = 5 42. logb 4 = z- О В задачах 43-54 максимально упростите указанные логарифмические выражения, используя свойства логарифмических функций (см. пример 2.16). X5 43. logb ^3 • 45. logb 44. logb(x23/3). 46. logb VQ- 1 47. logb (х2Уу)- 48.Юёь^. 49. logb(50 • 2-°-2t). 51- logb[P(l + r)‘], 53. loge 100e-°’olt. 50. logb (100 • 1.06‘). 52. loge Ле-0,34. 54. log10(67 • IO'0’12*). В задачах 55-62 найдите значение х. 2 1 55. logb х = - logb 8 + о 1о8ь9 “ 1о£ь 6- о Z 2 56. logb х = - logfc 27 + 2 logb 2 - logfc 3. О 3 2 57. logb x = - log6 4 - - logb 8 + 2 log6 2. z о 58. logb x = 3logb 2 + x logb 25 - logb 20. £ 59. logb x + logb(a: - 4) = logb 21- 61. log10(x - 1) - logio(z + 1) = 1. 60. logb(x + 2) + logb x = l°gb 24. 62. log10(a: + 6) - log10(o; - 3) = 1. В задачах 63 и 64 постройте графики указанных логарифмических уравнений, предварительно преобразовав их в показательные. 63. у = log2(a: - 2). 64. у = log3(a:+ 2). * 65. Объясните, с помощью каких простых преобразований (см. раздел 1.2) из графи¬ ка функции у = log2 х можно получить график функции, заданной уравнением из задачи 63. * 66. Объясните, с помощью каких простых преобразований (см. раздел 1.2) из графи¬ ка функции у = log3 х можно получить график функции, заданной уравнением из задачи 64.
Глава 2. Другие элементарные функции 197 67. Найдите область определения и область значений функции у = 1 -I- In (я: + 1). 68. Найдите область определения и область значений функции у = lg(x — 1) — 1. В задачах 69 и 70 при помощи калькулятора найдите значение указанных выражений с точностью до пяти десятичных знаков. 69. а) 1g 3527,2. б) In 0,0069132. в) In 277,63. г) In 0,040883. 70. а) 1g 72,604. б) 1g 0,033041. в) In 40 257. г) In 0,0059263. В задачах 71 и 72 найдите значение х с точностью до четырех десятичных знаков. 71. a) Igx = 1,1285. б) Igo; = -2,0497. в) In я = 2,7763. г) In ж = -1,8879. 72. a) Igx = 2,0832. б) Igz = -1,1577. в) In ж = 3,1336. г) In ж = —4,3281. В задачах 73-80 найдите решения указанных уравнений с точностью до четырех десятичных знаков. 73. 10х = 12. 74. 10® = 153. 75. ех = 4,304. 76. ех = 0,3059. 77. 1,03® = 2,475. 78. 1,075® = 1,837. 79. 1,00512* = 3. 80. l,024t = 2. ic. Проверьте правильность своих ответов на задачи 73-80, решив их графически. В задачах 81-88, используя калькулятор, постройте по точкам графики указанных логарифмических функций. Определите интервалы возрастания и убывания данных функций. 81. у = In ж. 83. у = | In ж|. 85. у = 21п(я; + 2). 87. у = 4 In х — 3. 82. у = — In х. 84. у = In |ж|. 86. у = 2 In х + 2. 88. у = 41п(я — 3). let Убедитесь в правильности графиков логарифмических функций, созданных при решении задач 81-88, построив их в графической утилите. В * 89. Объясните, почему логарифм единицы с любым допустимым основанием равен нулю. * 90. Объясните, почему единица не может быть основанием логарифма. 91. Преобразуйте логарифмическое выражение log10 у — log10 с = 0,8# в эквивалентное показательное выражение. 92. Преобразуйте логарифмическое выражение loge х — loge 25 = 0,2t в эквивалентное показательное выражение.
198 Часть I. Элементарные функции 93. * Пусть р(х) = In я, q(x) = \Jx и г (я) = х. Постройте графики всех трех функ¬ ций в одном и том же окне графической утилиты на отрезке 1 х < 16. Объясните, что подразумевают, когда говорят, что какая-либо функция больше другой в каком-либо интервале значений независимой переменной. Перечислите приведенные выше функции в порядке их уменьшения на отрезке 1 < х < 16. 94. * Пусть р(х) = Igx, q(x) = $х и г(х) = х. Постройте графики всех трех функ¬ ций в одном и том же окне графической утилиты на отрезке 1 < х < 16. Объясните, что подразумевают, когда говорят, что какая-либо функция больше другой в каком-либо интервале значений независимой переменной. Перечислите приведенные выше функции в порядке их уменьшения на отрезке 1 < х < 16. Применение математики Экономика и бизнес 95. WWW Время удвоения. Среднегодовой доход фонда Gabelli Growth Fund за первые 10 лет его существования составил 21,36%. Предположим, что деньги, инвестированные в этот фонд, продолжают ежегодно приносить 21,36% дохода, начисляемого по сложным процентам. Через какой интервал времени основная сумма инвестиций будет удвоена? 96. WWW Время удвоения. Среднегодовой доход фонда Janus Flexible Income Fund за его первые 10 лет его существования составил 9,58%. Предположим, что деньги, инвестированные в этот фонд, продолжают ежегодно приносить приносить 9,58% дохода, начисляемого по сложным процентам. Через какой интервал времени основная сумма инвестиций увеличится вдвое? 97. Инвестиции. Через сколько лет (с точностью до двух десятичных знаков) основная сумма размером 1000 долл, вырастет до 1800 долл, при ежеквартальном начислении сложных процентов с годовой ставкой 6%? Как изменится ответ, если сложные проценты будут начисляться непрерывно? 98. Инвестиции. Через сколько лет (с точностью до двух десятичных знаков) основная сумма размером 5000 долл, увеличится до 7500 долл, при условии, что на нее раз в полгода начисляются сложные проценты с годовой ставкой 8%? Как изменится ответ, если сложные проценты будут начисляться непрерывно? 99. Капиталовложение. Новобрачные планируют в течение 6 лет погасить кредит в размере 30000 долл, взятый на покупку дома. Какова должна быть годовая ставка (с точностью до трех десятичных знаков) при непрерывной начислении сложных процентов на основную сумму 20 000 долл.?
Глава 2. Другие элементарные функции 199 100. Капиталовложение. Родители новорожденного планируют за 17 лет накопить 60 000 долл., чтобы иметь возможность оплатить обучение своего ребенка в колледже. Под какую годовую ставку (с точностью до трех десятичных знаков) с непрерывным начислением сложных процентов нужно инвестировать подаренные бабушкой и дедушкой 20 000 долл., чтобы получить необходимую сумму в оговоренный срок? 101. Спрос и предложение. Новая модель шуруповерта реализуется через сеть строительных магазинов, торгующих по сниженным ценам. Маркетинговый отдел управляющей компании подготовил таблицы таблицы с данными зависимости спроса и предложения от цены (табл. 2.16 и табл. 2.17 соответственно) этого вида товара, В табл. 2.16 переменная х указывает количество покупателей, которые в течение месяца приобрели бы новый шуруповерт по цене р долларов за штуку, а в табл. 2.17 она обозначает количество ежемесячно предлагаемых к продаже шуруповертов новой модели по цене р долларов за штуку. Таблица 2.16. Зависимость Таблица 2.17. Зависимость спроса от цены предложения от цены X р = D(x), долл. X р = S(®), долл. 1000 91 1000 9 2000 73 2000 26 3000 64 3000 34 4000 56 4000 38 5000 53 5000 41 а) Найдите коэффициенты логарифмического уравнения регрессии (у = а + + Мпя), описывающего данные табл. 2.16. Используя найденное уравнение, оцените количество покупателей (с точностью до одного человека), которые приобрели бы шуруповерт по цене 50 долл, за штуку. б) Найдите коэффициенты логарифмического уравнения регрессии (у = а + + Ыпж), которое описывает данные табл. 2.17. Используя найденное уравнение, оцените количество шуруповертов (с точностью до одной штуки), которое предлагается к продаже через сеть строительных магазинов по цене 50 долл, за штуку. *в) Будет ли цена в размере 50 долл, за шуруповерт стабильной. Или при определенных обстоятельствах она все же будет изменяться? Объясните, чем обусловлено колебание цены. 102. Точка равновесия. Используя уравнения регрессии, найденные в задаче 101, определите равновесную цену шуруповерта новой модели. Вычислите ее с точностью до цента. Определите равновесное количество товара с точностью до единицы. Биологические науки 103. Громкость звука: децибелы. Человеческое ухо способно воспринимать звуки, интенсивность которых меняется в чрезвычайно широком диапазоне (более чем в 1000 триллионов раз). Измерять интенсивность звука в настолько широком
200 Часть I. Элементарные функции диапазоне значений лучше не в абсолютных, а логарифмических единицах. Единица измерения интенсивности звука называется децибелом. Она названа в честь изобретателя телефона, А. Г. Белла (A. G. Bell). Пусть N — громкость звука (в децибелах), I — его мощность (в ваттах на квадратный сантиметр), a Iq — мощность звука чуть ниже порога слышимости (порогу слышимости соответствует мощность звукового сигнала, равная 10“16 ватт на квадратный сантиметр). Тогда взаимосвязь между указанными величинами описывается следующим уравнением: I = /o10N/1°. Докажите, что эта формула эквивалентна формуле, приведенной ниже. 104. Громкость звука: децибелы. Используя формулу из задачи 103 (в которой I = 10“16 Вт/см2), определите громкость звука в децибелах следующих источников. а) Речь шепотом: 10“13 Вт/см2. б) Обычная речь: 3,16 • 10“10 Вт/см2. в) Автомагистраль: 10“8 Вт/см2. г) Реактивный самолет: 10“1 Вт/см2. 105. Сельское хозяйство. В табл. 2.18 приведены данные об урожайности (бушелей на акр) и валового сбора кукурузы (млн. бушелей) в США в указанные годы, начиная с 1950 г. (1 бушель кукурузы = 25 кг). Обозначим через х количество лет, прошедших с 1900 г. Найдите коэффициенты логарифмического уравнения регрессии у = а + Мпя, описывающего динамику роста урожайности кукурузы. Оцените (с точностью до одного десятичного знака) урожайность кукурузы в 2010 г. Таблица 2.18. Производство кукурузы в США www Год X Урожайность, бушелей на акр Валовый сбор, млн. бушелей 1950 50 37,6 2782 1960 60 55,6 3479 1970 70 81,4 4802 1980 80 97,7 6867 1990 90 115,6 7802 2000 100 139,6 10192
Глава 2. Другие элементарные функции 201 106. Сельское хозяйство. Обратитесь к данным табл. 2.18. Найдите коэффициенты логарифмического уравнения регрессии у = а + b In х9 которое описывает динамику роста валового сбора кукурузы. Оцените (с точностью до миллиона бушелей) валовой сбор кукурузы в 2010 г. Социальные науки 107. Численность населения планеты. При условии, что численность населения планеты составляет 5,8 млрд, человек, а ежегодный прирост составляет 1,14%, определите, через какой период времени численность населения земного шара достигнет такого значения, что на каждого жителя будет приходится всего лишь один кв. ярд земли. (Площадь суши планеты составляет примерно 1,68 • 1014 кв. ярдов.) 108. Археология: датировка радиоуглеродным методом. Содержание радиоактивного изотопа 14С в организме после его смерти описывается уравнением А = A0e-0’000124t, где t — время (лет) и Ао “ количество изотопа 14С в организме в момент смерти t = 0 (см. пример 2.9 в разделе 2.2.). Оцените возраст черепа, обнаруженного в ходе археологических раскопок, если содержание изотопа 14С в нем составляет 10% от первоначального значения (т.е. содержания изотопа в момент смерти). (Указание. Найдите значение t, для которого справедливо равенство А = 0,1Aq.) Ключевые слова, основные обозначения и формулы 2.1. Полиномы и рациональные функции. Полином; степень полинома; непрерывность; точка экстремума; корень уравнения; нуль функции; старший коэффициент полинома; рациональная функция; точки разрыва функции; вертикальные и горизонтальные асимптоты графика. f(x) = апхп + an-ixn-1 Ч h + а±х -I- а0, ап / 0; а(х) 2.2. Показательные функции. Показательная функция; основание степени; графики показательных функций; горизонтальная асимптота; основные свойства показательной функции; иррациональное число е; показательная функция с основанием е; уравнение экспоненциального роста; уравнение экспоненциального распада; сложные проценты; основная сумма (текущая стоимость); начисленная сумма (будущая стоимость); непрерывное начисление сложных процентов. f(x) = bx, Ь>0, Ь/1; у = ех\ N = Noekt-, , , / Т \ А = Аое~к\ А = Р(1 + —) ; \ т/ А = Pert.
202 Часть I. Элементарные функции 2.3. Логарифмические функции. Обратная функция; однозначная функция; логарифмическая функция; основание логарифма; эквивалентное показательное выражение; свойства логарифмов; десятичный логарифм; натуральный логарифм; вычисление логарифмов при помощи калькулятора; решение логарифмических и показательных уравнений; время удвоения инвестиций. у = logb х эквивалентно х = Ьу\ logb#, b > 0, b / 1; Igrr; 1пд; Упражнения для повторения Выполните все упражнения этого обзорного раздела и сравните результаты с ответами, помещенными в конце книги. Ответы ко многим упражнениям на повторение приводятся вместе с номером соответствующего раздела (курсивом). Если у вас возникают затруднения при решении какой-либо задачи, повторите материал соответствующего раздела. А 1. Прологарифмируйте выражение по основанию е: и = ev. 2. Прологарифмируйте выражение по основанию 10: х = 1О27. 3. Выполните потенцирование выражения: In М = N. 4. Выполните потенцирование выражения: Igu = v. В задачах 5, 6 упростите указанные выражения. 5^+4 ( еи \и 5‘ 54-х* 6' ) В задачах 7-9 найдите точное значение х, не пользуясь калькулятором. 7. log3 х = 2. 8. log^. 36 = 2. 9. log2 16 = х. В задачах 10-12 найдите значение х с точностью до трех десятичных знаков. 10. 10х = 143,7. 11. ех = 503 000. 12. Igz = 3,105. 13. 1пя = -1,147. В задачах 14 и 15 для каждого полинома найдите следующие характеристики. 1) Степень полинома. 2) Максимально возможное для полинома данной степени количество точек экстремума. 3) Максимально возможное для полинома данной степени количество точек пересечения его графика с осью х. 4) Наименьшее для полинома данной степени количество точек пересечения его графика с осью х. 5) Максимально возможное для полинома данной степени количество точек пересечения его графика с осью у.
Глава 2. Другие элементарные функции 203 6) Наименьшее для полинома данной степени количество точек пересечения его графика с осью у. 14. р(х) = ах3 + bx2 + ex -I- d, а 0. 15. р(х) = ах4 + Ьх3 + сх2 -И dx + е, а 0. На рисунках к задачам 16 и 17 изображены графики полиномов. Для каждого графика определите следующие характеристики. 1) Количество точек экстремума. 2) Минимальную степень полинома, который может иметь график, изображенный на рисунке. 3) Знак старшего коэффициента полинома. Б В задачах 18 и 19 для каждой рациональной функции выполните следующие задания. 1) Найдите координаты точек пересечения ее графика с осями координат. 2) Определите область существования функции. 3) Найдите уравнения вертикальных и горизонтальных асимптот функции. 4) Схематически изобразите асимптоты графика пунктирной линией. Затем нарисуйте эскиз графика функции f в масштабе —10 х 10 и —10 у 10. 5) Используя графическую утилиту, постройте график функции у = f{x) в масштабе, предложенном по умолчанию. 18. /Ы = 19. /W = В задачах 20-27 найдите точное значение х, не пользуясь калькулятором. 20. lg(x + 5) = lg(2® - 3). 22. 9х-1 = 31+*. 24. 2х2ех = Зхех. 21. 21n(ir — 1) = 1п(х2 — 5). 23. е2х = е*2"3. 25. log1/3 9 = х. , 3 27. log9x = 26. log*8 = -3. В задачах 28-37 найдите значение х с точностью до четырех десятичных знаков. 28. х = Зе1’49. 29. х = 230 • 10“°’161. 30. Igz = -2,0144. 31. In ж = 0,3618. 32. 35 = 7 • 3х. 33. 0,01 = е"0’05*.
204 Часть I. Элементарные функции 34. 8000 = 4000 • 1,08ж. 35. 52х“3 = 7,08. 36. х = log2 7. 37. х = log0 2 5,321. * 38. Как соотносятся графики функций f (х) = т4 — 4т2 +1 и у = х4 при уменьшении масштаба изображения? 39. Сравните графики функций у = х4 и у = х4 — 4т2+ 1, построенные в следующем масштабе. а) — 5 х С 5; — 5 у 5. б) — 5 х 5; —500 С у С 500. 40. Пусть р(х) = 2х4 — 11т3 — 15т2 — 14т — 16. Найдите приближенные действительные нули полинома р(т) с точностью до двух десятичных знаков. 41. Пусть /(т) = ех — 1 и </(т) = 1п(т + 2). Найдите координаты всех точек пересечения графиков функций f и д. Ответ округлите до двух десятичных знаков. В задачах 42 и 43 упростите указанные выражения. 42. ех (е~х + 1) - (ех + 1) (е~х - 1). 43. (е® - е-®)2 - (е® + е"®) (е® - е"®). В задачах 44-46 постройте графики функций, заданных соответствующими уравнениями в указанных интервалах изменения независимой переменной. Определите интервалы возрастания и убывания каждой функции. 44. у = 2Х~\ [-2; 4]. 45. /(t) = lOe”0’08*; t > 0. 46. у = 1п(т -h 1); (—1; 10]. В 47. Учитывая, что значение числа тг равно 3,141592654, а значение л/2 равно 1,414213562, докажите, что значения выражений, вычисленные с помощью калькулятора, на самом деле очевидны. Приведите примеры выражений с натуральным логарифмом и экспонентой, значение которых также очевидно. 1о9(10Ал) 3.141592654 10А1од(Т<2)) 1.414213562 Найдите точные решения уравнений 48-51, не пользуясь калькулятором. 48. IgT — lg3 = lg4 — lg(T + 4). 49. 1п(2т — 2) — 1п(т — 1) = 1пт. 50. 1п(т + 3) — 1п т = 2 In 2. 51. lg Зт2 = 2 + lg 9т. 52. Преобразуйте логарифмическое равенство In?/ = — 5t + Inc в эквивалентное показательное равенство. Затем выразите у через остальные переменные. * 53. Объясните, почему единица не может быть основанием логарифма.
Глава 2. Другие элементарные функции 205 Применение математики Экономика и бизнес Формулы, которые приведены ниже, будут использованы при решении некоторых последующих задач, \ mt 1 ) Формула начисления сложных процентов т/ А = Pert Формула непрерывного начисления сложных процентов 54. WWW Начисление процентов. Банк Provident (г. Цинциннати, штат Огайо) недавно предложил своим вкладчикам депозитные сертификаты, по которым выплачивается 6,59% сложного процента, начисляемого непрерывно. Какую сумму банк должен выплатить вкладчику через 5 лет, если он приобрел депозитный сертификат стоимостью 5000 долл.? 55. WWW Начисление процентов. Банк Capital One Bank (г. Глен-Аллен, штат Вирджиния) недавно предложил своим вкладчикам депозитные сертификаты, по которым выплачивается 6,58% сложного процента, начисляемого ежедневно. Какую сумму банк должен выплатить вкладчику через 5 лет, если он приобрел депозитный сертификат стоимостью 5000 долл.? 56. Начисление процентов. Через какой интервал времени основная сумма, на которую непрерывно начисляются сложные проценты из расчета 6,59% годовых, увеличится втрое? 57. Начисление процентов. Через какой интервал времени основная сумма, на которую ежедневно начисляются сложные проценты из расчета 6,58% годовых, увеличится вдвое? 58. Минимальные средние затраты. Финансовым отделом компании, производящей роликовые коньки, было установлено, что постоянные затраты компании в день составляют 300 долл. При объеме производства, равном 100 пар роликовых коньков величина общих затрат равна 4300 долл. Предположим, что величина общих затрат С(х) линейно связана с количеством произведенной продукции х. а) Найдите для рассматриваемого случая уравнение зависимости С(х), а также функцию средних затрат С(х) = С(х)/х. б) Схематически изобразите график функции средних затрат на отрезке 5 х ^ 200. в) Найдите асимптоты функции.
206 Часть I. Элементарные функции г) Определите, к какому значению стремится функция средних затрат при росте объема производства. 59. Минимальные средние затраты. Обслуживание больницы ежегодно обходится в сумму С(х) (тыс. долл.), определяемую с помощью следующей формулы. С(х) = 20х3 - ЗбОх2 + 2300л: - 1000, где х — количество пациентов (в тыс. чел.), которым в течение года была оказана медицинская помощь. Функция средних затрат С определяется формулой С(х) = С(х)/х. а) Напишите уравнение функции средних затрат. б) Постройте график функции средних затрат на отрезке 1 < х С 12. в) Используя графическую утилиту, постройте график функции средних затрат. Попеременно переходя в режимы изменения масштаба изображения и определения координат исходных данных, определите, какому количеству пациентов соответствует минимальное значение функции средних затрат. Чему равно это минимальное значение? 60. Точка равновесия. Компания планирует выпустить на рынок набор противопригарной посуды из 10 предметов. Маркетинговый отдел собрал статистические данные о зависимости спроса и предложения от цены (табл. 2.19 и 2.20 соответственно) на этот товар. В табл. 2.19 переменная х обозначает количество покупателей, которые на протяжении месяца будут приобретать посуду по цене р долларов, а в табл. 2.20 ею обозначено количество наборов, ежемесячно предлагаемых к продаже по цене р долларов. а) Найдите коэффициенты уравнения квадратичной регрессии, которое описывает данные табл. 2.19. Затем оцените количество покупателей, приобретающих набор посуды по цене 180 долл. Таблица 2.19. Зависимость Таблица 2.20. Зависимость спроса от цены предложения от цены X р = D(x), долл. X р = D(x), долл. 985 330 985 30 2145 225 2145 75 2950 170 2950 110 4225 105 4225 155 5100 50 5100 190 б) Найдите коэффициенты линейного регрессионного уравнения, которое описывает данные табл. 2.20. Определите количество наборов посуды, проданных по цене 180 долл. в) Является ли цена в размере 180 долл, за набор стабильной. Возможен ли ее дальнейший рост или снижение? Объясните, чем обусловлено колебание цены. г) Используя уравнения, найденные при решении пп. а и б, определите равновесную цену набора посуды. Вычислите ее с точностью до цента. Определите равновесное количество товара с точностью до единицы.
Глава 2. Другие элементарные функции 207 61. Телекоммуникации, Согласно данным Ассоциации телекоммуникационной промышленности (Telecommunications Industry Association) количество абонентов мобильной связи выросло примерно с 4 млн. человек в 1990 г. до более чем 76 млн. в 1999 г. (табл. 2.21). Пусть х — количество лет, прошедших с 1990 года. Таблица 2.21. Количество абонен¬ тов мобильной связи «ч Количество абонентов, WWW Год млн. чел. 1990 4 1991 6 1992 9 1993 13 1994 19 1995 28 1996 38 1997 49 1998 60 1999 76 а) Найдите показательное регрессионное уравнение, описывающее приведенные в таблице данные. Оцените (с точностью до миллиона) количество абонентов мобильной связи в 2000 и 2010 гг. *б) Фактическое количество абонентов мобильной связи в 2000 г. составило примерно 97 млн. человек. Как фактическое значение соотносится с оценочным значением, полученным при решении п. al Как повлияет информация о фактическом количестве абонентов в 2000 г. на прогноз их количества в 2010 г.? Биологические науки 62. Медицина. Здоровой мыши прививается одна лейкозная клетка. Примерно через полдня она делится пополам. К концу дня обе лейкозные клетки снова делятся и их становится 4. Деление раковых клеток продолжается до тех пор, пока не достигнет 1 миллиарда; при таком их количестве подопытное животное умирает. а) Напишите уравнение, связывающее количество лейкозных клеток в организме мыши N и количество дней, прошедшее с момента заражения t. б) Определите с точностью до дня, через какой период времени мышь погибнет? 63. Биология морей. Уменьшение интенсивности света, падающего на поверхность воды, с увеличением расстояния от поверхности, описывается показательным уравнением I = Ioe~kd,
208 Часть I. Элементарные функции где I — интенсивность света на глубине d футов от поверхности воды, Iq — интенсивность света, падающая на поверхность воды, к — коэффициент поглощения. Измерения, проведенные в Саргассовом море в Вест-Индии показали, что интенсивность света, падающего на поверхность воды, уменьшается в два раза на глубине 73,6 фута. Найдите величину коэффициента поглощения к, (с точностью до пяти десятичных знаков) и толщину слоя воды (с точностью до фута), который ослабляет интенсивность падающего света до 1%. 64. Сельское хозяйство. В табл. 2.22 приведены данные об объемах потребления кукурузы (млн. бушелей) в Соединенных Штатах Америки в указанные годы, начиная с 1975 г. Пусть х — количество лет, прошедшее с 1900 г. Таблица 2.22. Совокупное потребление кукурузы WWW Год X Совокупное потребление кукурузы (млн. бушелей) 1975 75 522 1980 80 659 1985 85 1152 1990 90 1373 1995 95 1690 а) Найдите коэффициенты логарифмического уравнения регрессии у = а + + b In х, описывающего табличные данные. Оцените (с точностью до млн. бушелей) совокупное потребление кукурузы в 1996 и 2010 гг. *б) Фактическое совокупное потребление кукурузы в 1996 г. составило 1583 млн. бушелей. Как это значение соотносится с оценочным, полученным при решении п. а? Как повлияет информация об объеме фактического потребления кукурузы в 1996 г. на прогноз объема потребления кукурузы в 2010 г.? Аргументируйте свой ответ. Социальные науки 65. Рост численности населения. Во многих странах ежегодный прирост численности населения составляют более 3%. Оцените, через сколько лет население страны увеличится в два раза при условии, что в ближайшей перспективе темп роста численности населения будет сохраняться на указанном уровне. Воспользуйтесь формулой, описывающей начисление сложных процентов: Р = Ро(1 4- г)1. 66. Рост численности населения. Решите задачу 65 при условии, что рост численности населения описывается формулой непрерывного начисления сложных процентов: Р = P$ert. 67. Программа медицинской помощи престарелым. Данные о ежегодных расходах (млрд, долл.) американского правительства по программе бесплатной медицинской помощи престарелым в указанные годы, начиная с 1980 г., приведены в табл. 2.23. Обозначим через х количество лет, прошедшее с 1980 г. а) Найдите коэффициенты показательного уравнения регрессии у = abx9 описывающего табличные данные. Оцените (с точностью до млрд, долл.) совокупные расходы в 2010 г. б) В каком году совокупные расходы достигнут 500 млрд, долл.?
Глава 2. Другие элементарные функции 209 Таблица 2.23. Расходы по программе бесплатной медицинской помощи престарелым Гад Совокупные расходы, млрд, долл. 1980 37 1985 72 1990 111 1995 181 Домашнее задание 2.1. Сравнение скоростей возрастания показательных функций и полиномов, логарифмической функции и функции извлечения корня Показательная функция, например функция /(ж) = 2х, при больших значениях х возрастает чрезвычайно быстро, гораздо быстрее любого полинома. Покажите, что графики функций /(ж) = 2х и д(х) = х2 пересекаются в трех точках. Абсциссы точек пересечения графиков делят ось х на четыре интервала. Для каждого интервала определите, какая из двух функций принимает большие значения. Логарифмическая функция, например функция г(х) = 1пх, при больших значениях х растет чрезвычайно медленно, медленнее, чем функция извлечения корня, например, функция в (х) = \/х. Изобразите схематически графики обеих функций в одной системе координат в интервале х > 0 и определите количество точек пересечения графиков. Определите, какая из этих функций принимает большие значения в каждом из интервалов, образованных абсциссами точек пересечения графиков. Домашнее задание 2.2. Сравнение регрессионных моделей До этих пор уравнение регрессии, описывающее набор исходных данных, представлялось нами в виде полинома разных степеней, показательной или логарифмической функции. Однако уравнение регрессии может выражаться зависимостями любого другого типа. (В графическом калькуляторе TI-83 Plus меню STAT-CALC содержит 10 различных вариантов уравнений регрессии.) Но как определить, какая функциональная зависимость точнее всего описывает исходные данные? Существует два принципиально разных способа выбора регрессионной модели. Согласно первому выбор модели обуславливается природой аппроксимируемых данных. Например, логично предположить, что вес рыбы пропорционален кубу ее длины. Аналогичным образом рост численности населения описывается экспоненциальной функцией, по крайней мере в коротком временном промежутке. Согласно второму способу уравнение регрессии подбирается, исходя из близости кривой его графика к точкам, представляющим исходные данные. Последний способ лучше проиллюстрировать на конкретном примере. Рассмотрим набор данных, представленный в виде таблицы на рис. 2.26. В столбце L1 указаны абсциссы, а в столбце L2 — ординаты точек. На координатной плоскости эти точки
210 Часть I. Элементарные функции располагаются так, как показано на рис. 2.27. Выберем произвольное уравнение, например у\ = 0,6я + 2, и построим ее график так, чтоб он проходит через область точек, представляющих исходные данные (рис. 2.28). и L2 L3 3 ч б в г 3 в 5 L3(1)= Рис. 2.26. Набор данных Рис. 2.27. Исходные данные на координатной плоскости Рис. 2.28. График регрессионного уравнения у± = 0,6х + 2 Чтобы количественно оценить степень близости функции yi к аппроксимируемым данным, нужно вычислить разность между ординатами точек исходного набора данных и графика функции у± (на рис. 2.29 и рис. 2.30 указаны в столбце L3), с одними и теми же абсциссами. Каждая такая разность называется остатком. Наиболее часто оценка близости регрессионного уравнения к описываемому набору данных выполняется по величине суммы квадратов остатков. Ее достаточно просто вычислить при помощи графического калькулятора (рис. 2.31) или в электронной таблице (рис. 2.32). L1 L2 L3 £ г 2 з.г ч 3 ч.ч в В 5.6 5 6. в L1(5)= Рис. 2.29. Регрессионные остатки Рис. 2.30. + L2; ■ L3 sum(<Lz-L5>2> 12.4 sum<(L2-Vi<Li>>г 12.4 Рис. 2.31. Два способа вычисления суммы квадратов остатков А 1 В _1 С I D I Е 1 Data Set 2 X У у1=0.6х + 2 Residual Residual* 2 3 2 2 3.2 -1.2 1.44 4 4 3 4.4 -1.4 1.96 5 6 8 5.6 2.4 5.76 6 8 5 6.8 -1.8 3.24 7 SSR 12.4 Рис. 2.32. Результаты вычислений
Глава 2. Другие элементарные функции 211 1. Найдите уравнение линейной регрессии, описывающей данные, приведенные в таблице, на рис. 2.26. Для найденного уравнения вычислите сумму квадратов остатков и сравните ее с величиной, найденной для функции yi. Оказывается, среди всех возможных полиномов первой степени уравнение линейной регрессии характеризуется минимальной величиной суммы квадратов остатков. По этой причине процедуру построения модели линейной регрессии часто называют методом наименьших квадратов. Подобное утверждение справедливо для полиномов любой заданной степени. Иначе говоря, уравнение квадратичной регрессии имеет минимальное значение суммы квадратов остатков среди всех квадратичных полиномов, уравнение кубической — минимальное значение суммы квадратов остатков среди всех кубических полиномов и т.д. При этом оно оказывается недействительным для показательного или логарифмического уравнений регрессии. Тем не менее при выборе наиболее приемлемого регрессионного уравнения среди показательной, логарифмической и полиномиальной функций, достаточно сравнить сумму квадратов остатков выбранных моделей. 2. Найдите показательное и логарифмическое уравнения регрессии, описывающие зависимость, представленную данными в таблице на рис. 2.26. Для каждой модели вычислите сумму квадратов остатков и сравните ее с суммой квадратов остатков, полученной для уравнения линейной регрессии. 3. Ежегодные расходы на рекламу товаров и услуг в указанные годы, начиная с 1950 года, в США приведены в табл. 2.24. Переменная х обозначает количество лет, прошедших с 1950 г., а переменная у — суммарные расходы на рекламу (млрд. долл.). Какое уравнение регрессии наиболее точно описывает исходные данные: квадратичное, кубическое или показательное? Аргументируйте свой ответ, вычислив для каждого из них сумму квадратов остатков. Таблица 2.24. Ежегодные расходы на рекламу в США в 1950-1995 гг. WWW _ “ ..— ' х, годы у, млрд. долл. 0 5,7 5 9,2 10 12,0 15 15,3 20 19,6 25 27,9 30 53,6 35 94,8 40 128,6 45 160,9
Часть II Конечная математика
3 Финансовая математика ■ 3.1. Простые проценты ■ 3.2. Сложные проценты ■ 3.3. Будущая стоимость аннуитета: амортизационные фонды ■ 3.4. Текущая стоимость аннуитета: погашение долга ■ Ключевые слова, основные обозначения и формулы ■ Упражнения для повторения ■ Домашнее задание 3.1. Снижение выплачиваемых процентов по закладной на дом ■ Домашнее задание 3.2. Итоговая доходность и внутренняя норма прибыли Введение Содержание этой главы не зависит от содержания других глав; ее можно изучать в любой момент. В частности, авторы не рассчитывают на то, что вы изучали главу 2, в которой кратко, на примере применения показательных и логарифмических функций, затрагивались некоторые из рассматриваемых в этой главе тем. Низкая стоимость и удобство использования делает калькулятор прекрасным инструментом решения задач по расчету сложных процентов, аннуитета, амортизации и т.п. Для целей этой главы подойдет любой калькулятор, имеющий клавиши вычисления показательной и логарифмической функции. Графические
214 Часть II. Конечная математика утилиты предоставляют специальные возможности по визуализации зависимостей, описывающих скорость роста капитала на депозитном счете или погашения основной суммы взятого кредита. Если позволяет время, перед началом изучения этой главы ознакомьтесь с арифметическими и геометрическими последовательностями, рассмотренными в приложении Б.2. Знание из свойств упростит понимание некоторых из рассматриваемых далее вопросов. Чтобы каждый раз не повторяться, вынесем следующее замечание в начало главы. Процентные ставки указываемые в формулах этой главы, пред- ставлены десятичными числами. 3.1. Простые проценты Простые проценты, как правило, используются только для краткосрочных счетов — в большинстве случаев срок их существования не превышает одного года. Концепция простых процентов, однако, служит основой для большей части остального материала, рассматриваемого в этой главе. Если человек положил определенную сумму денег Р на сберегательный счет, или если он взял взаймы Р денежных единиц в кредитной организации, то величина Р называется капиталом. При займе денег — независимо от того, кладет человек свои деньги на сберегательный счет или же сам одалживает деньги в кредитной организации — на одолженную сумму начисляются выплаты. Эти выплаты являются рентой за использование чужих денег. Она мало отличается от ренты за использование чужого дома. Выплаты называются процентами. Они, как правило, вычисляются как процентная доля (которую называют процентной ставкой)1 от капитала за определенный период времени. Процентная ставка, если не оговорено другое условие, будет считаться годовой. Простые проценты рассчитываются по следующей формуле. Простые проценты I = Prt, (3.1) где Р — капитал, г — годовая простая процентная ставка (в десятичном представлении), t — время в годах. ]Если г — процентная ставка, записанная в десятичном представлении, то 100г% — это процентная ставка, записанная с использованием процентов. Например, если г = 0,12, то, используя знак процента, получаем 100г% = 100 • 0,12% = 12%. Выражения 0,12 и 12% эквивалентны.
Глава 3. Финансовая математика 215 Например, за 9 месяцев проценты по ссуде в размере 100 долл, при ставке 12% составят: I = Prt = = 100 • 0,12 • 0,75 = = 9 долл. Представляем 12% десятичным числом, а 9 месяцев выражаем в количестве лет— в 9/12 = 0,75 года. В конце девятого месяца заемщик должен будет вернуть капитал (100 долл.) плюс проценты (9 долл.), т.е. 109 долл. Вообще говоря, если капитал Р занимается под процентную ставку г, то через t лет заемщик будет должен вернуть кредитору сумму А, которая будет равна капиталу Р (номинальной стоимости векселя) плюс проценты I (рента за использование денег). Поскольку Р — это сумма, которую одалживают в текущий момент, а А — сумма, которую нужно будет вернуть в будущем, величину Р часто называют текущей стоимостью, а величину А — будущей стоимостью. Суммы А и Р связаны следующей формулой. Сумма: простые проценты А = Р + Prt = Р(1 + rt), (3.2) где Р — капитал, или текущая стоимость, г — годовая простая процентная ставка (записанная в десятичном представлении), t — время в годах, А — сумма, или будущая стоимость. Зная любые три из четырех переменных А, Р, г или t, можно решить уравнение (3.2) относительно четвертой переменной. В следующих примерах демонстрируется несколько типов наиболее распространенных задач, которые можно решать, используя формулу (3.2). ■W Пример 3.1 (Общая сумма к получению по ссуде). Вычислите общую сумму, которую должен получить кредитор, предоставивший заемщику ссуду в размере 800 долл, под девять простых процентов, по истечении четырех месяцев. Решение. Чтобы найти будущую стоимость А суммы, которую кредитор должен получить через четыре месяца, воспользуемся формулой (3.2), подставив в нее следующие значения: Р = 800, г = 0,09 и t — | года. Таким образом, А = Р(1 + rf) = 800 • 1 + 0,09^ О = 800 • 1,03 = 824 долл.
216 Часть II. Конечная математика И Упражнение 3.1. Вычислите общую сумму, которую должен получить кредитор, предоставивший заемщику ссуду в размере 500 долл, под 12 простых процентов, по истечении 30-ти месяцев. В Задание 3.1. 1. Сестра ссудила своему брату 1000 долл., договорившись, что он вернет капитал плюс четыре простые процента, как только сможет. Сколько брат будет должен сестре, если он вернет ссуду через год, два года, пять лет или десять лет? 2. Как проценты, выплачиваемые через десять лет, связаны с процентами, выплачиваемыми через один или два года или через пять лет? 3. Объясните, почему полученные ответы согласуются с тем фактом, что для простых процентов график будущей стоимости, как функции времени, является прямой (рис. 3.1). ■ Рис. 3.1. График будущей стоимости П Пример 3.2 (Текущая стоимость капиталовложения). Сколько (с точностью до цента) необходимо заплатить за вексель, который через девять месяцев будет стоить 5000 долл., если инвестор хочет получить 10% годовых? Решение. Снова воспользуемся формулой (3.2), но только теперь нужно найти капитал (текущую стоимость) при условии, что А = 5000 долл., г = 0,1 и t = = — 0,75 года. A = P(l + rt); 5000 = Р(1 + 0,1 • 0,75); Заменяем А, г и t указанными значениями 5000 = 1,075Р; Решаем уравнение относительно Р Р = 4651,16 долл. ■
Глава 3. Финансовая математика 217 яге Упражнение 3.2. ■“ Повторите вычисления из примера 3.2 для шести месяцев. ■ яр? www Пример 3.3 (Процентная ставка, полученная по векселю). Кратко- ■■ срочные казначейские векселя — это один из инструментов, который используется Министерством финансов США для оплаты государственных долгов. Какой будет годовая простая процентная ставка, если купить 180-дневный казначейский вексель номинальной стоимостью 10000 долл, за 9693,78 долл.? (Запишите ответ в виде процентов с точностью до трех десятичных знаков.) Решение. Снова воспользуемся формулой (3.2), но в этот раз необходимо будет найти число г при условии, что Р = 9693,78, А = 10000 и t = 180/360 = = 0,5 года2. A = P(l + rt), 10 000 = 9693,78 (1 + 0,5г); 10000 = 9693,78 + 4846,89т; 306,22 = 4846,89т; ЧПА 99 г = - ’ А « 0,06318, или 6,318%. WW Упражнение 3.3. Повторите вычисления из примера 3.3, считая, что за цена векселя равна 9668,74 долл. ■ ; \ ; Пример 3.4 (Проценты по капиталовложению). Предположим, что после покупки нового автомобиля человек решил продать свой старый автомобиль другу. В качестве оплаты он получил 270-дневный вексель на сумму 3500 долл., которая была выдана под 10 простых процентов. (И капитал, и проценты будут выплачены по истечении 270-дневного срока.) Семь дней спустя этот человек обнаружил, что ему нужны деньги, и продал вексель третьей стороне за 3550 долл. Какую годовую процентную ставку получит третья сторона при таком капиталовложении? (Запишите ответ в виде процентов с точностью до трех десятичных знаков.) Решение. Этап 1. Найдем сумму, которая будет выплачена спустя 270 дней владельцу векселя. ( 270\ 1 + = 3762,50 долл. 360 / Различные организации часто используют 360-, 364- или 365-дневный год. Для простоты в этом разделе будет использоваться 360-дневный год. В других разделах будет использоваться 365-днев- ный год. Выбранный вариант всегда будет явно оговариваться.
218 Часть II. Конечная математика Этап 2. Для третьей стороны необходимо вычислить годовую процентную ставку г, при которой сумма 3550 долл, вырастет за 270 — 60 = 210 дней до величины 3762,50 долл. Иначе говоря, необходимо найти значение г при заданных величинах А = 3762,50, Р = 3550 и t = А = Р + Prt, А — Р г = ——— = Pt Решаем уравнение относительно переменной г 3762,50 - 3550 3550 • = 0,10262, или 10,262%. Упражнение 3.4. Повторите вычисления из примера 3.4, считая, что вексель был продан третьей стороне спустя 90 дней с момента его выдачи за 3500 долл. ■ Некоторые брокерские фирмы, работающие в Интернете, при продаже акций со скидкой предлагают постоянную процентную ставку, однако многие все же взимают комиссионные исходя из суммы продажи. В табл. 3.1 показан перечень комиссионных для одной из таких фирм. Таблица 3.1. Комиссионные выплаты WWW Объем транзакции, долл. Комиссионное вознаграждение, долл. 0-2499 26,25 + 1,4% от капитала 2500-5999 45 + 0,54% от капитала 6000-19999 60 + 0,28% от капитала 20000-49,999 75 + 0,1875% от капитала 50000-499999 131,25 + 0,09% от капитала 500000 + 206,25 4- 0,0075% от капитала И Пример 3.5 (Проценты по капиталовложению). Инвестор купил 1000 паев акционерного капитала по цене 47,52 долл, за штуку. Через 200 дней он продал акции по цене 52,19 долл, за штуку. Пользуясь перечнем комиссионных, приведенным в табл. 3.1, найдите годовую процентную ставку, полученную при таком капиталовложении. (Запишите ответ в виде процентов с точностью до трех десятичных знаков.) Решение. Капитал, указанный в табл. 3.1, — это стоимость акций. Общие затраты инвестора — это затраты на акции плюс комиссионные. 47,52 1000 = 47 520 долл. 75 + 0,001875 • 47520 = 164,10 долл. 47,520 + 164,10 = 47684,10 долл. Капитал Комиссионные (строка 4 в табл. 3.1) Суммарный вклад
Глава 3. Финансовая математика 219 При продаже акций комиссионные начисляются на вырученную от продажи сумму, а остаток возвращается инвестору. 52,19 • 1000 = 52190 долл. 75 + 0,0009 • 52190 = 178,22 долл. 52190 - 178,22 = 52011,78 долл. Капитал Комиссионные (строка 5 в табл. 3.1) Суммарная выручка Теперь, используя формулу (3.2) при А = 52 011,78, Р = 47684,10 и t = = |, получаем следующий результат. А = Р(1 + г<); 52011,78 = 47684,10 • (1 + |г) = = 47 684,10+ 26 491,17г 4327,68 = 26491,17г 4327,68 г = 26491 17 ~°’16336’ ™ 16,336 В Упражнение 3.5. Повторите вычисления из примера 3.5, если 1000 паев акционерного капитала были куплены по цене 18,78 долл, за штуку и проданы через 270 дней по цене 21,43 долл, за штуку. ■ Задание 3.2. 1. Исходя из формулы (3.2), докажите следующие формулы. „ А А — Р А — Р р = ТУ7е г=рГ^ ^~рГ- 2. Объясните, почему не обязательно помнить формулы, приведенные выше для Р, г и t, если знаешь формулу (3.2). ■ Ответы к упражнениям 3.1. 650 долл. 3.4. 15,0%. 3.2. 4761,90 долл. З.б. 17,095%. 3.3. 6,852%. Практикум 3.1 А В задачах 1-4 необходимо выполнить указанные преобразования, предполагая, что в году 360 дней. 1. 9,5% = ? (в десятичном представлении); 60 дней = ? года. 2. 8,75% = ? (в десятичном представлении); 3 квартала = ? года.
220 Часть II. Конечная математика 3. 0,18 = ? (в процентах); 5 месяцев = ? года. 4. 0,0525 = ? (в процентах); 240 дней = ? года. Используя формулу (3.1) для простых процентов, вычислите неизвестные значения, указанные в задачах 5-8. 5. Р = 500 долл.; г = 8%; t = 6 месяцев; 1 = 7. 6. Р = 900 долл.; г = 10%; t = 9 месяцев; I = ?. 7. I = 80 долл.; Р = 500 долл.; t = 2 года; г = ?. 8. I = 40 долл.; Р = 400 долл.; t = 4 года; г = ?. Б Используя формулу (3.2), вычислите указанные в задачах 9-12 неизвестные величины. 9. Р = 100 долл.; г = 8%; t = 18 месяцев; А = ?. 10. Р = 6000 долл.; г = 6%; t = 8 месяцев; А = ?. 11. А = 1000 долл.; г = 10%; t = 15 месяцев; Р = ?. 12. А = 8000 долл.; г = 12%; t = 7 месяцев; Р = 7. kci. Проверьте результаты задач 11 и 12, решив их с помощью графической утилиты. о В В задачах 13-16 решите каждое уравнение относительно указанной переменной. 13. I = Prt относительно г. 14. I = Prt относительно Р. 15. А = Р + Prt относительно Р. 16. А = Р + Prt относительно г. * 17. Проанализируйте схожесть и различие графиков будущей стоимости А как функ¬ ции времени t, если под простые проценты с процентной ставкой 4, 8 и 12% вложено 1000 долл. (рис. 3.2). * 18. Проанализируйте схожесть и различие графиков будущей стоимости А как функ¬ ции времени t для ссуд в размере 400, 800 и 1200 долл, соответственно, каждая из которых выдается под 7,5 простого процента (рис. 3.3). Рис. 3.2. Иллюстрация к задаче 17
Глава 3. Финансовая математика 221 Время, лет Рис. 3.3. Иллюстрация к задаче 18 Применение математики3 Экономика и бизнес Во всех задачах предполагается, что в году 360 дней. Если ответом должна быть процентная ставка, ее нужно выражать в виде процентов с точностью до трех десятичных знаков. 19. Сколько будет получено процентов, если 3000 долл, ссужены на четыре месяца под 8% годовых? 20. Сколько будет получено процентов, если 5000 долл, ссужены на 10 месяцев под 10% годовых? 21. Сколько придется выплатить процентов по балансу кредитной карточки в размере 554 долл., который просрочен на один месяц, если взимаемая годовая процентная ставка составляет 20%? 22. Универмаг взимает с просроченных счетов годовую процентную ставку в размере 18%. Сколько процентов должен будет заплатить покупатель, просрочивший оплату счета на сумму 835 долл, на два месяца? 23. Кредит на сумму 7250 долл, был возвращен по истечении восьми месяцев. На какую сумму был выписан чек для оплаты (капитал и проценты), если взимаемая процентная ставка составляла 9% годовых? 24. Кредит на сумму 10000 долл, был возвращен по истечении шести месяцев. Какая была возвращена сумма (капитал и проценты), если взимаемая процентная ставка составляла 12% годовых? ’Авторы хотят поблагодарить профессора Роя Лука (Roy Luke) из колледжа Пирса (Pierce College) за большое количество полезных предложений.
Ill Часть II. Конечная математика 25. Кредит на сумму 4000 долл, был оплачен чеком на сумму 4270 долл, по истечении восьми месяцев. Какая при этом взималась годовая процентная ставка? 26. Для оплаты пятимесячного кредита в размере 3000 долл, был использован чек на сумму 3122,50 долл. Какая при этом взималась годовая процентная ставка? 27. Каков размер годовой процентной ставки, взимаемой кредитной организацией, если за использование 1000 долл, в течение 60 дней было уплачено 30 долл.? 28. Каков размер годовой процентной ставки, взимаемой кредитным сообществом, если за использование 2000 долл, в течение 90 дней было уплачено 120 долл.? 29. В радиорекламе кредитной организации утверждается: “Вы платите всего лишь 50 центов в день за каждые одолженные 500 долл.”. Какую сумму нужно будет вернуть, если занять 1500 долл, на 120 дней, и какую в действительности процентную ставку взимает кредитная организация? 30. Джордж нашел компанию, которая взимает 70 центов в день за каждые одолженные 1000 долл. Какую сумму ему нужно будет вернуть, если он одолжит 3000 долл, на 60 дней, и какую в действительности процентную ставку взимает эта компания? 31. www Каков размер годовой процентной ставки по 13-недельному казначейскому векселю стоимостью 1000 долл., если он был продан за 984,37 долл.? 32. www Каков размер годовой процентной ставки по 33-недельному казначейскому векселю стоимостью 1000 долл., если он был продан за 994,16 долл.? 33. www Какова покупная цена 50-дневного казначейского векселя стоимостью 1000 дол, если он приносит 6,53% годовых? 34. www Какова покупная цена 26-недельного казначейского векселя стоимостью 1000 долл., если он приносит 6,203% годовых? 35. В качестве платы за услуги адвокат приняла от клиента 90-дневный вексель на сумму 5500 долл, под 12 простых процентов. (И проценты, и капитал будут выплачены по истечении 90 дней.) Желая воспользоваться своими деньгами раньше, адвокат продала долговую расписку спустя 30 дней третьей стороне по цене 5540 долл. Каков размер годовой процентной ставки, полученной третьей стороной, при таком капиталовложении? 36. Чтобы закончить с продажей дома, продавец принял 180-дневный вексель на сумму 10 000 долл, под 10 простых процентов. (И проценты, и капитал
Глава 3. Финансовая математика 223 будут выплачены по истечении 180 дней.) Желая воспользоваться своими деньгами раньше для покупки другого дома, продавец спустя 60 дней продал вексель третьей стороне по цене 10100 долл. Каков размер годовой процентной ставки, полученной третьей стороной, при таком капиталовложении? —W— 11 ■ ■— Keyes Realty Показанный ниже перечень комиссионных при покупке и продаже принадлежит широко известной брокерской фирме, торгующей в Интернете со скидкой. Учитывая комиссионный сбор при покупке и продаже из этого перечня, в задачах 37-40 необходимо найти годовую процентную ставку по каждому капиталовложению. WWW Объем транзакции, долл. Комиссионное вознаграждение, долл. 0-2500 22 + 1,4% от капитала 2501-6000 38 + 0,45% от капитала 6001-20000 55 + 0,23% от капитала 20001-50000 79 + 0,15% от капитала 50001-500000 119 + 0,07% от капитала 500001+ 169 + 0,06% от капитала 37. www Инвестор купил 500 акций по цене 14,20 долл, за штуку, продержал их у себя 39 недель, а затем продал по цене 16,84 долл, за штуку. 38. www Инвестор купил 450 акций по цене 64,84 долл, за штуку, продержал их у себя 26 недель, а затем продал по цене 72,08 долл, за штуку. 39. www Инвестор купил 2000 акций по цене 23,75 долл, за штуку, продержал их у себя 300 дней, а затем продал по цене 26,15 долл, за штуку. 40. www Инвестор купил 75 акций по цене 31,50 долл, за штуку, продержал их у себя 150 дней, а затем продал по цене 35,40 долл, за штуку. Многие компании, оформляющие декларации о доходах, предлагают своим клиентам заем под ожидаемую сумму возвращенного налога (refund anticipation loan — RAL). Налоговое управление отсылает возвращенную сумму налогов непосредственно компании. Таким образом, плата за услуги компании эквивалентна процентам, взимаемым по кредиту. Приведенный ниже перечень
224 Часть II. Конечная математика принадлежит крупнейшему кредитору, предоставляющему займы RAL. Используя этот перечень, в задачах 41-44 найдите годовую процентную ставку по займам RAL. WWW Сумма RAL, долл. Комиссионные за RAL, долл. 200-500 24,95 501-1500 34,95 1501-2000 54,95 2001-5000 64,95 41. www Клиент получил заем RAL на сумму 400 долл., которая вернется через 20 дней. Какова годовая процентная ставка по этому кредиту? 42. www Клиент получил заем RAL на сумму 1100 долл., которая вернется через 30 дней. Какова годовая процентная ставка по этому кредиту? 43. www Клиент получил заем RAL на сумму 1900 долл., которая вернется через 15 дней. Какова годовая процентная ставка по этому кредиту? 44. www Клиент получил заем RAL на сумму 2100 долл., которая вернется через 25 дней. Какова годовая процентная ставка по этому кредиту? 3.2. Сложные проценты ■ Сложные проценты ■ Скорость роста инвестиций ■ Годовая доходность Сложные проценты Если в конце срока платежа причитающиеся проценты снова вложить под ту же процентную ставку, то эти проценты вместе с исходным капиталом будут приносить проценты в течение следующего срока. Проценты, начисленные на повторно вложенные проценты, называются сложными. Например, предположим, что человек вложил 1000 долл, в банк, который платит восемь сложных процентов, начисляемых ежеквартально. Сколько банк будет должен выплатить этому человеку в конце года? Ежеквартальное начисление сложных процентов означает, что причитающиеся проценты поступают на счет в конце каждого трехмесячного периода, а начисленные проценты наряду с капиталом приносят дополнительный доход в течение следующего квартала. Используя формулу (3.2) для простых процентов, вычислим сумму, начисленную в конце первого квартала после того, как были выплачены проценты. А = Р(1 + rt) = 1000 1 + 0,08 • = 1000 • 1,02 = 1020 долл.
Глава 3. Финансовая математика 225 Во втором квартале новый капитал равен 1020 долл. В конце второго квартала после выплаты процентов на счету будет лежать следующая сумма. А = Р(1 + rt) = 1020 + 0,08 • = 1020 • 1,02 = 1040,40 долл. Аналогично в конце третьего квартала имеем следующий результат. А = Р(1 + rt) = 1040 (1 + 0,08 • = 1040,40 • 1,02 = 1061,21 долл. Наконец, в конце четвертого квартала на счету будет лежать такая сумма. А = Р(1 + rt) = 1061,21 (1 + 0,08 • = 1061,21 • 1,02 = 1082,43 долл. Как же эта сумма, полученная при начислении сложных процентов, соотносится с суммой, полученной при начислении простых процентов? Сумма с простыми процентами составила бы следующую величину. А = Р(1 + rt) = 1000(1 + 0,08 • 1) = 1020 • 1,02 = 1080 долл. Очевидно, что начисляемые ежеквартально сложные проценты приносят на 2,43 долл, больше, чем принесли бы простые проценты. Проанализируем процесс вычисления сложных процентов, приведенный выше, попытавшись установить закономерность, которая могла бы дать общую формулу для вычисления сложных процентов в произвольном случае: А = 1000 • 1,02; А = (1000 • 1,02) • 1,02 = 1000 • 1,022; А = (1000 • 1,022) • 1,02 = 1000 • 1,023; А = (1000 • 1,023) • 1,02 « 1000 • 1,024. Конец первого квартала Конец второго квартала Конец третьего квартала Конец четвертого квартала Итак, в конце n-го квартала получаем следующий результат. А = 1000 • 1,02п Конец n-го квартала ИЛИ / I \ п / л по А = 1000 • 1 + 0,08 • - ) = 1000 • I 1 + \ V 4 где = 0,02 — процентная ставка за квартал. Поскольку годовая процентная ставка обычно называется годовой номинальной процентной ставкой, ставка, приходящаяся на один период начисления сложных процентов, вычисляется путем деления годовой номинальной процентной ставки на количество периодов начисления сложных процентов, приходящихся на один год.
226 Часть II. Конечная математика Вообще говоря, если Р — это капитал, приносящий сложные проценты т раз в году при годовой процентной ставке г (процентная ставка на период начисления равна i = r/т), то сумма А в конце каждого периода будет равна 4 = Р(1 + г); А = [Р(1 + г)](1 + 0 = Л1-Н)2; А = [Р(1 + г)2](1 + г) = Р(1 + г)3; Конец первого периода Конец второго периода Конец третьего периода А — [Р(1 + i)n 1] (1 + i) = Р(1 + i)n. Конец п-го периода Этот важный результат подытожен в приведенной ниже в врезке. Сумма: сложные проценты А = Р(1 + г)п, (3.3) где i = г/т, г — годовая номинальная процентная ставка 4, т — количество периодов начисления сложных процентов в году, i — процентная ставка на один период начисления сложных процентов, п — общее количество периодов начисления сложных процентов, Р — капитал (текущая стоимость), А — сумма (будущая стоимость) в конце n-го периода. Различные способы применения формулы будут продемонстрированы на нескольких примерах. Если даны любые три переменные, упомянутые в формуле (3.3), то, используя определенные алгебраические преобразования и калькулятор, можно вычислить четвертую. В частности, если известны величины А, Р и г, то, используя свойства логарифмов и калькулятор, можно вычислить показатель п. Пример 3.6 (Сравнение процентов для разных периодов начисления сложных процентов). Какой будет сумма спустя пять лет, если вложить 1000 долл, под восемь сложных процентов, начисляемых по следующим правилам? 1. Ежегодно. 2. Раз в полгода. 3. Ежеквартально. 4. Ежемесячно. Ответ запишите с точностью до цента. 4Ее часто сокращенно называют “годовой процентной ставкой” или просто “процентной ставкой”.
Глава 3. Финансовая математика 227 Решение. 1. Начисление сложных процентов ежегодно означает, что в году есть один период выплаты процентов. Таким образом, п = 5, ai = г = 0,08. А = Р(1 + г)п = = 1000 • (1 4- 0,08)® = Воспользуемся калькулятором. = 1000 • 1,469328 = = 1469,33. Полученные проценты = А — Р = 469,33 долл. 2. Начисление сложных процентов раз в полгода означает, что в году есть два периода выплаты процентов. Таким образом, количество периодов выплат в течение пяти лет равно п = 2 • 5 = 10, а процентная ставка на один период составляет • г 0,08 i = — = 4— = 0,04. т 2 Следовательно, А = Р(1 + г)п = = 1000 ■ (1 + 0,04)10 = Воспользуемся калькулятором. = 1000 • (1,480244) = = 1480,24. Полученные проценты = А — Р = 480,24 долл. 3. Ежеквартальное начисление сложных процентов означает, что в году есть четыре периода выплаты процентов. Таким образом п = 4 • 5 = 20, а г = = = 0,02. Следовательно, А = Р(1 + г)п = = 1000 • (1 + 0,02)^^ = Воспользуемся калькулятором. = 1000 • 1,485947 = = 1485,95. Полученные проценты = А — Р = 485,95 долл. 4. Ежемесячное Начисление сложных процентов означает, что в году есть двенадцать периодов выплаты процентов. Таким образом п = 12 • 5 = 60, a i = = 0,006666.5 Следовательно, А = Р(1 + г)п = = 1000 • Воспользуемся калькулятором. 5Напомним, что черточка над 6 указывает на периодичность десятичной дроби. Округление i до меньшего числа десятичных знаков, например до 0,007 или 0,0067, может привести к ошибке округления. Чтобы этого избежать, следует для г использовать столько десятичных знаков, сколько может отобразить используемый калькулятор.
228 Часть II. Конечная математика = 1000 • 1,489846 = = 1489,85. Полученные проценты = А — Р = 489,85 долл. ■ Упражнение 3.6. Повторите решение примера 3.6 при условии, что годовая процентная ставка равна 6%, а сумма вкладывается на восемь лет. ■ Следует отметить существенный рост полученных процентов при переходе от ежегодного начисления сложных процентов к ежемесячному. Возникает вопрос: что произойдет, если начать начислять сложные проценты ежедневно, или ежеминутно, или ежесекундно и т.д.? На рис. 3.4 показано следствие роста числа периодов начисления сложных процентов в году при вложении 1000 долл, на пять лет под восемь сложных процентов. Рис. 3.4. Проценты на 1000 долл., вложенных на пять лет под 8%, при различном числе периодов начисления сложных процентов Исходя из рис. 3.4, можно отметить, что разница между процентами, полученными при ежегодном начислении сложных процентов и начислении раз в полгода, составляет 480,24 долл. — 469,33 долл. = 10,91 долл., а разница между процентами, полученными при начислении сложных процентов раз в полгода и ежеквартальном начислении, равна всего лишь 5,71 долл. Более того, разница между ежеквартальным и ежемесячным начислением составляет всего лишь 3,90 долл., несмотря на то, что количество периодов утраивается. Разница же между ежемесячным и ежедневным начислением равна только 1,91 долл., хотя количество периодов существенно возрастает. Этот факт указывает на то, что получаемые
Глава 3. Финансовая математика 229 проценты стремятся к какому-то пределу. Этот предел достигается при непрерывном начислении сложных процентов, при котором будет получено 491,82 долл. — всего на шесть центов меньше, чем при ежедневном начислении. (Как указывалось в разделе 2.2, если сложные проценты начисляются непрерывно, то А = Pert.) Сравним результаты, показанные на рис. 3.4, с простыми процентами, полученными за тот же период времени. I = Prt 1000 • 0,08 • 5 = 400 долл. Задание 3.3. 1. Как лучше вложить 1000 долл.: под девять простых процентов на десять лет или под семь сложных процентов, начисляемых ежемесячно в течение десяти лет? 2. Объясните, почему график будущей стоимости, являющейся функцией вре¬ мени для простых процентов, является прямой линией, а для сложных процентов загибается вверх (рис. 3.5). ■ Рис. 3.5. График будущей стоимости Еще один способ применения формулы сложных процентов — это задача о том, сколько денег нужно вложить сейчас, чтобы к определенной дате в будущем получить заданную сумму. И Пример 3.7 (Вычисление текущей стоимости). Какую сумму нужно вложить сейчас под десять сложных процентов, начисляемых ежеквартально, чтобы получить 8000 долл, для покупки автомобиля через пять лет? Решение. Итак, известна будущая стоимость А = 8000 долл, для капиталовложения под сложные проценты. Требуется найти текущую стоимость (капитал Р)
230 Часть II. Конечная математика при условии, что i — = 0,025 и п = 4 • 5 = 20. 8000 = Р(1 + О,О25)20; р 8000 _ ” (1 + О,О25)20 ~ 8000 = = 4888,17 долл. 1,638616 Используем калькулятор Таким образом, исходное капиталовложение в размере 4882,17 долл, через пять лет возрастет до 8000 долл. ■ И Упражнение 3.7. Сколько денег должны вложить молодые родители в данный момент под восемь сложных процентов, начисляемых каждые полгода, чтобы получить 8000 долл, к моменту, когда их ребенок будет поступать в колледж через 17 лет? ■ Полезным инструментом для изучения сложных процентов может стать графическая утилита. В частности, для создания рис. 3.6, иллюстрирующего рост капиталовложений, описанных в примере 3.7, была использована электронная таблица. Аналогичные результаты можно получить с помощью графических калькуляторов. Общие принципы и формулы, рассмотренные в этой главе, широко используются в электронных таблицах для построения таблиц и графиков для широкого класса задач, возникающих в деловой сфере. Задание 3.4. 1. Чтобы стать миллионером, двадцатипятилетний человек намерен вложить сумму Р под ставку г сложных процентов, начисляемых ежеквартально, и получить один миллион долларов, достигнув 75 лет. Сколько денег он должен вложить, если г — 4, 8 или 12%? 2. Предположим, что человек вложил 2500 долл, на свое 25-летие. Какую став¬ ку сложных процентов, начисляемых ежеквартально, должен приносить этот вклад, чтобы эта сумма выросла до одного миллиона долларов к его 75-летию? ■ Скорость роста инвестиций Решение уравнения для сложных процентов относительно величины г позволяет найти скорость роста инвестиций.
Глава 3. Финансовая математика 231 А В С 1 Period Intereft Amount 2 0 «4 £82.17 3 1 «122.05 $5 £04.22 4 2 «125.11 $5,129.33 5 3 «128.23 $5 £57.56 6 4 tl31.44 $5,889.00 7 5 «134.78 to to 04 Ю ID 8 6 $138.09 $5,661.82 9 7 $141.55 $5,803.37 10 8 $145.08 $5,948.45 1 1 9 $148.71 $6,097.16 12 10 «152.48 «6,249.59 13 11 «156.24 $6,405.83 14 12 «160.15 $6,565.98 15 13 $164.15 $6,730.13 16 14 $168.25 «6,898.38 17 15 «172.46 $7,070.84 18 16 «176.77 $7J47.61 19 17 $181.19 $7,428.80 20 18 «185.72 $7 £14.52 21 19 $190.36 $7,804.88 22 20 «195.12. $8 £00.00 Рис. 3.6. Рост суммы 4882,17 долл, при вложении на пять лет под десять сложных процентов, начисляемых ежеквартально D Пример 3.8 (Вычисление скорости роста). На рис. 3.7 показано, что капиталовложение в размере 10 000 долл, в один из ориентированных на развитие взаимных фондов за последний 10-летний период принесло бы 126000 долл. Какая номинальная годовая ставка сложных процентов, начисляемых ежегодно, дала бы такой же рост? Выразите ответ в процентах с точностью до трех десятичных знаков. 140 000 120 000 100 000- 80 000 60 000 40 000 - 20000 + 63 000 0 1 2 3 4 5 6 7 8 9 10 Годы Рис. 3.7. Рост капитала, равного 10000 долл. 126 000
232 Решение. Часть II. Конечная математика 126000 = 10000- (1 +г)10; 12,6 = (1 + г)10; = 1 + г; г= >/12^5 — 1 = 0,28836, или 28,836%. И Упражнение 3.8. Инвестиционный фонд Frank Russell Company отслеживает среднюю эффективность различных акций. На рис. 3.7 показано, что в среднем инвестор, вложивший 10000 долл, в инвестиционный фонд, нацеленный на долгосрочный рост капитала, за последний 10-летний период получил бы 63 000 долл. Какая номинальная годовая ставка сложных процентов, начисляемых ежегодно, дала бы такой же рост? Выразите ответ в процентах с точностью до трех десятичных знаков. В Наконец, если решить уравнение для сложных процентов относительно величины п, можно найти время роста инвестиций, т.е. время, которое понадобится для того, чтобы данный капитал вырос до определенного значения (чем меньше это время, тем выше прибыль на инвестиционный капитал). В примере 3.9 проиллюстрированы два метода выполнения таких вычислений. П Пример 3.9 (Вычисление времени роста). Сколько понадобится времени для того, чтобы 10 000 долл, выросли до 12 000 долл., если они вложены под девять сложных процентов, начисляемых ежемесячно? Решение. Метод 1. Используем логарифмы и калькулятор: Л = Р(1 + г)п; 12000 = 10000- (1 + ^ 1,2 = 1,0075п. Теперь решим это уравнение относительно п, взяв логарифм от обеих сторон: In 1,2 = 1п 1,0075™; In 1,2 = nln 1,0075; Можно использовать логарифм с любым основанием Здесь выбран натуральный логарифм
Глава 3. Финансовая математика 233 In 1,2 In 1,0075 Использовано свойство logb = plogb М = 24,40 « 25 месяцев, или 2 года и 1 месяц. {Примечание. Чтобы гарантировать достижение суммы, равной 12000 долл., число 24,40 округляется до 25, поскольку проценты выплачиваются в конце каждого месяца.) Метод 2. Воспользуемся графической утилитой. Чтобы решить эту задачу, используя методы графической аппроксимации, построим графики обеих сторон уравнения 12 000 = 10 000(1,075)”. Они пересекаются в точке х = п = 24,40 месяцев (рис. 3.8, а). Таким образом, время роста равно 25 месяцам. К такому же заключению можно прийти, используя команду решения уравнений (рис. 3.8, б). ■ у2“ 12000 А-Р<1 + 1)л№0 А=12000 Р=10000 1=.0075 N=24.400588158... bound=£-lE99,1... left-rt=0 б) Рис. 3.8. Время роста инвестиций В Упражнение 3.9. Сколько понадобится времени для того, чтобы сумма, равная 10000 долл., выросла до 25 000 долл., если она вложена под восемь сложных процентов, начисляемых ежеквартально? ■ Годовая доходность В табл. 3.2 перечислены процентные ставки и периоды начисления сложных процентов для депозитных сертификатов, предложенных недавно тремя банками. Как определить, какой из этих сертификатов принесет наибольший доход? Таблица 3.2. Депозитные сертификаты www Банк Ставка, % Период начисления Advanta 6,95 ежемесячно DeepGreen 6,96 ежедневно Charter One 6,97 ежеквартально
234 Часть II. Конечная математика Задание 3.5. Найдите стоимость депозитных сертификатов, купленных у каждого банка из табл. 3.2 за 1000 долл., спустя один год. Какой депозитный сертификат принесет наибольший доход? Какой принесет наименьший доход? ■ Если капитал Р вложен под г сложных процентов, начисляемых т раз в год, то спустя один год сумма будет равна А = Р Т \ 7П 1 4 ) т/ Простая процентная ставка, которая дала бы спустя один год ту же сумму А, называется годовой доходностью (annual percentage yield — APY). Чтобы найти величину APY, выполним следующие вычисления. Сумма при \ / Сумма при \ простых процентах I = I сложных процентах I спустя 1 год / у спустя 1 год / (Т \ т 14 ) ; Разделим обе стороны на Р. 772/ (Т \ т 14 ) ; Выделим слева величину АРУ. 772/ (Г \ т 1 + — ) -1. 772/ Годовая доходность Если капитал Р вложен под (номинальную) годовую процентную ставку, равную г сложным процентам, начисляемым т раз в год, то годовая доходность равна (г \ т 1 + — ) -1. 772/ Годовую процентную доходность называют также эффективной ставкой или истинной процентной ставкой. Сложные проценты с различными периодами начисления нельзя сравнивать непосредственно (см. задание 3.5). Однако, поскольку годовой процентный доход — это простая процентная ставка, то всегда можно сравнить эту величину для двух разных сложных процентных ставок. И www Пример 3.10 (Использование годовой доходности для сравнения инвестиций). Найдите годовую доходность (выраженную в процентах с точно¬ стью до трех десятичных знаков) для каждого из банков, перечисленных в табл. 3.2, и сравните депозитные сертификаты.
Глава 3. Финансовая математика 235 Решение. ( 0,0695 \12 ~ Advanta : APY = (1 + — 1 - 1 = 0,07176, или 7,176%; ( 0,0696 \365 _ DeepGreen : APY = (1 + Ц— I - 1 = 0,07207, или 7,207%; \ 365 ) ( 0,0697\4 _ Charter One : APY = (1 + ) - 1 = 0,07154, или 7,154%. Сравнивая эти величины, можно прийти к выводу, что депозитный сертификат банка DeepGreen приносит наибольший доход, а депозитный сертификат банка Charter One — наименьший. ■ Е Упражнение ЗЛО. www Недавно банк Southern Pacific Bank предложил клиентам однолетние депозитные сертификаты, по которым ежедневно выплачивается 6,8 сложного процента, а банк Washington Savings Bank выпустил депозитные сертификаты, по которым ежеквартально выплачивается 6,85 сложного процента. Вычислите годовую доходность (выраженную в процентах с точностью до трех десятичных знаков) для каждого из депозитных сертификатов. Какой из них имеет большую доходность? И Пример 3.11 (Вычисление годовой номинальной процентной ставки по заданной эффективной ставке). Банк планирует выпуск депозитных сертификатов с ежемесячным начислением сложных процентов по эффективной ставке 7,5%. Какую он должен использовать номинальную годовую сложную процентную ставку, начисляемую ежемесячно? Ч Проверьте результат с помощью графической утилиты. Решение. / Г \ 12 0,075 =(1 + -) - 1; х £ Z / / Г \ 12 1,075 =11 + —) ; \ IX / VTo75 = i + ~; 1а W75-i = ^; £ А г = 12 • ( ‘{/Г075 - 1) = = 0,072539, или 7,254%. Воспользуемся калькулятором.
236 Часть II. Конечная математика Таким образом, номинальная годовая процентная ставка, равная 7,254 сложного процента и начисляемая ежемесячно, эквивалентна эффективной ставке на уровне 7,5%. Проверка. *s Чтобы проверить этот результат, воспользуемся программой для решения уравнений (рис. 3.9). ■ R-<1+R/M>AM+1=0 R=.075 ■ R=. 07253902829... М=12 bound=C -1 е99? 1... ■left-rt=0 Рис. 3.9. Решение с помощью калькулятора ян» Упражнение 3.11. Какой будет номинальная годовая сложная процентная ставка, начисляемая ежеквартально, для облигации с эффективной ставкой, равной 5,8%? Проверьте результат с помощью графической утилиты. ■ Предупреждение. В каждой задаче на сложные проценты есть две процентные ставки. Скажем, в примере 3.9 величина г = 0,09, или 9%, — это номинальная годовая сложная процентная ставка, а величина i = г/12 = 0,0075, или 0,75%, — это месячная процентная ставка. Не следует путать эти две ставки, используя величину г вместо величины i в формуле для вычисления сложных процентов. Если сложные проценты начисляются ежегодно, то г = г/1 = г. Во всех остальных случаях величины г и г не совпадают. Ответы к упражнениям 3.6. 1) 1593,85 долл.; 2) 1604,71 долл.; 3) 1610,32 долл.; 4) 1614,14 долл. 3.7. 21084,17. 3.8. 20,208%. 3.9. 47 кварталов, или 11 лет и 3 квартала. 3.10. Банк Southern Pacific Bank: 7,036%; банк Washington Savings Bank: 7,02 долл.; банк Southern Pacific Bank дает больший доход. 3.11. 5,678%.
Глава 3. Финансовая математика 237 Практикум 3,2 Все суммы в долларах следует находить с точностью до цента. Когда в ответе требуется найти процентную ставку, ее следует выражать в процентах с точностью до двух десятичных знаков, если не оговорено противное. Во всех задачах предполагается, что год состоит из 365 дней. А В задачах 1-8 необходимо найти все указанные значения, используя формулу для вычисления сложных процентов (3.3). 1. Р = 100 долл.; i « 0,01; п = 12; А = ?. 2. Р = 1000 долл.; i — 0,015; п = 20; А = 2. 3. Р = 800 долл.; г = 0,06; п = 25; А = ?. 4. Р = 10000 долл.; i = 0,08; п ~ 30; А = 2. 5. А = 10000 долл.; i — 0,03; п — 48; Р = ?. 6. А = 1000 долл.; i 0,015; п = 60; Р = 2. 7. А = 18000 долл.; i — 0,01; п = 90; Р = 2. 8. А = 50 000 долл.; i 0,005; п = 70; Р = 2. В задачах 9-12, зная годовую ставку и период начисления сложных процентов, найдите величину i — процентную ставку, приходящуюся на один период начисления. 9. 9% ежемесячно. 10. 15% ежегодно. И. 7% ежеквартально. 12. 11% раз в полгода. В задачах 13-16, зная ставку на один период начисления сложных процентов, найдите величину г — годовую процентную ставку. 13. 0,8% ежемесячно. 14. 4,5% раз в полгода. 15. 5% ежегодно. 16. 2,3% ежеквартально. Б 17. Какой будет сумма спустя четыре года, если 100 долл, вложено под шесть сложных процентов, начисляемых по следующим правилам? а) ежегодно; б) ежеквартально; в) ежемесячно. Сколько всего будет получено процентов? 18. Какой будет сумма спустя пять лет, если 2000 долл, вложены под семь сложных процентов, начисляемых по следующим правилам? а) ежегодно; б) ежеквартально; в) ежемесячно. Сколько всего будет получено процентов? 19. Какой будет сумма спустя указанный период времени, если 5000 долл, вложены под девять сложных процентов, начисляемых ежемесячно? а) через два года; б) через четыре года. 20. Какой будет сумма спустя указанный период времени, если 20 000 долл, вложены под шесть сложных процентов, начисляемых ежемесячно? а) через пять лет; б) через восемь лет.
238 Часть II. Конечная математика *21. Проанализируйте сходства и различия между графиками будущей стоимости А, зависящей от времени t, если 1000 долл, вложено на восемь лет, а сложные проценты начисляются ежемесячно при годовых ставках 4, 8 и 12% соответственно (рис. 3.10). Рис. 3.10. Иллюстрация к задаче 21 22. Проанализируйте сходства и различия между графиками будущей стоимости А, зависящей от времени t, ддя кредитов в размере 4000, 8000 и 12 000 долл, соответственно, каждый из которых выдан под 7,5 сложного процента, начисляемого ежемесячно в течение восьми лет (см. рис. 3.11). Рис. 3.11. Иллюстрация к задаче 22 23. Вычислите проценты, полученные в течение каждого года и сумму на счету в конце каждого года, если 1000 долл, положены на счет, который приносит 9,75 сложного процента, начисляемого ежегодно в течение шести лет. Представьте результаты в виде таблицы. 24. Вычислите проценты, полученные в течение каждого года и сумму на счету в конце каждого года, если 2000 долл, положены на счет, который приносит 8,25 сложного процента, начисляемого ежегодно в течение пяти лет. Представьте результаты в виде таблицы.
Глава 3. Финансовая математика 239 их Проверьте результаты решения задач 23 и 24, построив таблицы с помощью графической утилиты. 25. Сколько денег нужно вложить сейчас, чтобы получить 10000 долл, спустя указанный период времени, если инвестиционная компания платит восемь сложных процентов, начисляемых раз в полгода? а) 5 лет; б) 10 лет. 26. Сколько денег нужно вложить сейчас, чтобы получить 6000 долл, спустя указанный период времени, если инвестиционная компания платит 10 сложных процентов, начисляемых ежеквартально? а) 3 года; б) 6 лет. 27. Какова эффективная ставка для процентов, начисляемых на деньги, вложенные на указанных условиях? а) 10 сложных процентов, начисляемых ежеквартально; б) 12 сложных процентов, начисляемых ежемесячно. 28. Какова эффективная ставка для процентов, начисляемых на деньги, вложенные на указанных условиях? а) 6 сложных процентов, начисляемых ежемесячно; б) 14 сложных процентов, начисляемых раз в полгода. 29. Сколько понадобится времени, чтобы сумма в размере 4000 долл, выросла до 9000 долл., если она вложена под 15 сложных процентов, начисляемых ежемесячно? 30. Сколько понадобится времени, чтобы сумма в размере 5000 долл, выросла до 7000 долл., если она вложена под восемь сложных процентов, начисляемых ежеквартально? В В задачах 31 и 32, используя формулу для вычисления сложных процентов (3.3), найдите величину п с точностью до ближайшего наибольшего целого числа. 31. А = 2Р; i = 0,06; п = ?. 32. А = 2Р; i =± 0,05; п = ?. 33. Сколько понадобится времени, чтобы деньги удвоились, если они вложены на следующих условиях? а) 10 сложных процентов, начисляемых ежеквартально; б) 12 сложных процентов, начисляемых ежеквартально. 34. Сколько понадобится времени, чтобы деньги удвоились, если они вложены на следующих условиях? а) 14 сложных процентов, начисляемых раз в полгода; б) 10 сложных процентов, начисляемых раз в полгода.
240 Часть II. Конечная математика Применение математики Экономика и бизнес 35. Новорожденный ребенок получил от своих бабушки с дедушкой в подарок 5000 долл., предназначенных для оплаты обучения в колледже. В какую сумму превратятся эти 5000 долл., когда ему исполнится 17 лет, если они вложены под семь сложных процентов, начисляемых ежеквартально? 36. Человек, обладающий 8000 долл., пытается решить, купить ли себе автомобиль сейчас или вложить деньги под 6,5 сложного процента, начисляемого раз в полгода, а затем купить более дорогой автомобиль. Сколько денег он сможет потратить на автомобиль спустя три года? 37. Сколько через 10 лет будет стоить дом, цена которого в данный момент составляет 110000 долл., если темпы инфляции за этот период будут в среднем составлять три сложных процента, начисляемых ежегодно? 38. Сколько через пять лет будет стоить автомобиль, цена которого в данный момент составляет 10000 долл., если в течение следующих пяти лет темпы инфляции в среднем составят четыре сложных процента, начисляемых ежегодно? 39. За прошедшие пять лет арендная плата за офисное помещение росла со скоростью 4,8 сложного процента в год, начисляемого ежегодно. Какой будет арендная плата за офисное помещение спустя пять лет, если в данный момент она составляет 20 долл, за кв. фут? 40. На окраине города стоимость жилья за прошедшие восемь лет росла со скоростью 5,2 сложного процента в год, начисляемого ежегодно. Сколько спустя восемь лет будет стоить дом, цена которого в данный момент составляет 160000 долл.? 41. Сколько понадобится времени, чтобы население определенной страны увеличилось вдвое, если оно растет со скоростью 2,2 сложного процента ежегодно? (Если результат неточен, то его следует округлить до следующего большего года.) 42. Сколько понадобится времени, чтобы население Земли выросло до 10 миллиардов человек, если в данный момент оно составляет 6 миллиардов и растет со скоростью 1,33 сложного процента ежегодно? (Если результат неточен, то его следует округлить до следующего большего года.) 43. Какая инвестиция лучше и почему: девять сложных процентов, начисляемых ежемесячно, или 9,3 сложного процента, начисляемого ежегодно? 44. Какая инвестиция лучше и почему: восемь сложных процентов, начисляемых ежеквартально, или 8,3 сложного процента, начисляемого ежегодно? * 45. Ответьте на следующие вопросы. а) Сколько стоила бы инвестиция в размере 100 долл, в 1998 году, если бы она была сделана в год подписания Декларации независимости и приносила три сложных процента ежеквартально?
Глава 3. Финансовая математика 241 б) Проанализируйте результат ежемесячного, ежедневного и непрерывного (в сравнении с ежеквартальным) начисления сложных процентов на инвестицию в размере 100 долл. в) Используя графическую утилиту, постройте график роста инвестиции, указанной в п. а. * 46. Выполните следующие задания. а) Исходя из формулы (3.3), докажите каждую из следующих формул. р = —— • (1 + г)п’ _ 1пА-1пР 1п(1 + г) б) Объясните, почему не обязательно помнить приведенные выше формулы для Р, i и п, если знаешь формулу (3.3)? 47. Человек накопил 7000 долл, для покупки автомобиля стоимостью 9000 долл. На какой срок нужно вложить 7000 долл, под девять сложных процентов, начисляемых ежемесячно, чтобы они выросли до 9000 долл.? (Если результат неточен, то его необходимо округлить до следующего большего месяца.) 48. У молодоженов есть 15 000 долл, для покупки жилья. Они вычислили, что для покупки дома, интересующего их типа, понадобится 20 000 долл, для первого взноса при оплате в рассрочку. На какой срок нужно вложить эту сумму под десять сложных процентов, начисляемых ежеквартально, чтобы она выросла до 20000 долл.7 (Если результат неточен, то его необходимо округлить до следующего большего квартала.) 49. На индивидуальном пенсионном счету находится 20 000 долл., и его владелец решил больше не пополнять этот счет деньгами, за исключением восьми сложных процентов, начисляемых ежедневно. Сколько денег будет на счету спустя 35 лет, когда Владелец достигнет пенсионного возраста? 50. Предположим, что один доллар был положен на банковский счет в год рождения Христа. Сколько бы было на счету в конце 2010 г., если бы эти деньги приносили два сложных или два простых процента, начисляемых ежегодно? (Теперь читатель может оценить возможности сложного начисления процентов и понять, почему неактивные счета закрываются по истечении относительно короткого промежутка времени.) 51. Сколько понадобится времени, чтобы денежная сумма удвоилась, если она вложена под семь сложных процентов, начисляемых ежедневно, или 8,2 сложного процента, начисляемого ежегодно? 52. Сколько понадобится времени, чтобы денежная сумма удвоилась, если она вложена под пять сложных процентов, начисляемых ежедневно, или шесть сложных процентов, начисляемых ежегодно?
242 Часть II. Конечная математика 53. В разговоре с другом один человек упомянул, что у него есть два капиталовложения в недвижимое имущество, стоимость одного удвоилась за прошедшие девять лет, а стоимость второго удвоилась за прошедшие 12 лет. Его друг тут же сказал, что первое капиталовложение росло со скоростью приблизительно восемь сложных процентов, начисляемых ежегодно, а второе — со скоростью шесть сложных процентов, начисляемых ежегодно. Как его друг сделал такие выводы? Правило 72 (rule of 72) гласит, что годовая сложная скорость роста капиталовложения г, при которой оно удвоится в течение п лет, приблизительно равна г = 72/п. Постройте таблицу, в которой будут сравниваться точная скорость роста и приблизительная скорость роста, полученная согласно правилу 72 для периодов удвоения капитала п = б, 7, ..., 12 лет. Значения обеих скоростей округлите до одного десятичного знака. 54. Вернемся к задаче 53. Покажите, что точная годовая сложная скорость роста капиталовложения, которое удвоится спустя п лет, равна г = 100(21/п - 1). Постройте график этого уравнения и уравнения для правила 72 с помощью графической утилиты для 5 п < 20. Решите задачи 55-58, используя метод графической аппроксимации, 55. Сколько понадобится времени для того, чтобы инвестиция в размере 2400 долл., сделанная под 13 сложных процентов, начисляемых ежеквартально, стала стоить больше, чем инвестиция в размере 3000 долл., сделанная под девять сложных процентов, начисляемых ежеквартально? 56. Сколько понадобится времени для того, чтобы инвестиция в размере 4800 долл., сделанная под 10 сложных процентов, начисляемых ежемесячно, стала стоить больше, чем инвестиция в размере 5000 долл., сделанная под семь сложных процентов, начисляемых ежемесячно? *57. По одной инвестиции выплачивается 10 простых процентов, а по другой — семь сложных процентов, начисляемых ежегодно. Какую инвестицию следует выбрать? Почему? * 58. По одной инвестиции выплачивается девять простых процентов, а по другой — шесть сложных процентов, начисляемых ежемесячно. Какую инвестицию следует выбрать? Почему? 59. Чему равна номинальная годовая сложная процентная ставка, начисляемая ежедневно, для облигации, которая приносит 6,8% в год? 60. Чему равна номинальная годовая сложная процентная ставка, начисляемая ежемесячно, для депозитного сертификата, который приносит 5,9% в год? 61. Какая номинальная годовая сложная процентная ставка, начисляемая ежемесячно, приносит столько же процентов в год, что и ставка в размере семи сложных процентов, начисляемых ежеквартально? 62. Какая номинальная годовая сложная процентная ставка, начисляемая ежедневно, приносит столько же процентов в год, что и ставка в размере шести сложных процентов, начисляемых ежемесячно?
Глава 3. Финансовая математика 243 Задачи 63-66 касаются облигаций с нулевым купоном. Облигация с нулевым купоном — это облигация, которая в текущий момент продается со скидкой, а ее номинальная стоимость выплачивается в будущий момент погашения — никакие проценты в данном случае не начисляются. 63. WWW Облигация с нулевым купоном и номинальной стоимостью 30000 долл, подлежит оплате через 15 лет. По какой цене она должна продаваться сейчас, чтобы ее доходность составила 6,348 сложного процента, начисляемого ежегодно? 64. WWW Облигация с нулевым купоном и номинальной стоимостью 20 000 долл, подлежит оплате через 10 лет. По какой цене она должна продаваться сейчас, чтобы ее норма прибыли составила 6,194 сложного процента, начисляемого ежегодно? 65. WWW Чему будет равна годовая сложная норма прибыли, если заплатить 3,126 долл, за 20-летнюю облигацию с нулевым купоном и номинальной стоимостью 10000 долл.? 66. WWW Чему будет равна годовая сложная норма прибыли, если заплатить 30 000 долл, за пятилетнюю облигацию с нулевым купоном номинальной стоимостью 40 000 долл.? 67. WWW Онлайновая финансовая служба недавно зарегистрировала следующие депозитные счета денежного рынка. а) Банк Republic Bank'. 6,31 сложного процента, начисляемого ежемесячно. б) Банк Chase Bank: 6,35 сложного процента, начисляемого ежедневно. в) Банк BankFirst: 6,36 сложного процента, начисляемого ежемесячно. Сколько процентов в год приносит каждый из них? 68. WWW Онлайновая финансовая служба недавно зарегистрировала следующие однолетние депозитные сертификаты. а) Компания Banking: 6,5 сложного процента, начисляемого ежеквартально. б) Банк Wingspan Bank: 6,6 сложного процента, начисляемого ежемесячно. в) Банк Discover Bank: 6,6 сложного процента, начисляемого ежедневно. Сколько процентов в год приносит каждый из них? Показанный ниже перечень комиссионных при покупке и продаже принадлежит широко известной онлайновой брокерской фирме, торгущей акциями со скидками. Учитывая комиссионный сбор при покупке и продаже, указанный в таблице, найдите годовую сложную процентную ставку по каждому капиталовложению. WWW Объем транзакции, долл. Комиссионное вознаграждение, долл. 0-1600 29 + 2,5% от капитала 1501-6000 57 + 0,6% от капитала 6001-22000 75 + 0,3% от капитала 22001-50000 97 + 0,2% от капитала 50001-500000 147 + 0,1% от капитала 500001 + 247 + 0,08% от капитала
244 Часть II. Конечная математика 69. WWW Инвестор купил 100 акций по цене 65 долл, за штуку, продержал их у себя пять лет, а затем продал по цене 125 долл, за штуку. 70. WWW Инвестор купил 300 акций по цене 95 долл, за штуку, продержал их у себя три года, а затем продал по цене 156 долл, за штуку. 71. WWW Инвестор купил 200 акций по цене 28 долл, за штуку, продержал их у себя четыре года, а затем продал по цене 55 долл, за штуку. 72. WWW Инвестор купил 400 акций по цене 48 долл, за штуку, продержал их у себя шесть лет, а затем продал по цене 147 долл, за штуку. 3.3. Будущая стоимость аннуитета: амортизационные фонды ■ Будущая стоимость аннуитета ■ Амортизационные фонды ■ Приближенный метод вычисления процентных ставок Будущая стоимость аннуитета Аннуитет — это последовательность периодических выплат. Если выплаты производятся в конце каждого временного интервала, то аннуитет называется простым. В этой книге будет рассматриваться только простой аннуитет. Сумма, или будущая стоимость аннуитета, — это сумма всех выплат плюс все начисленные проценты. Предположим, что один человек решил каждые шесть месяцев класть 100 долл, на счет, по которому выплачивают шесть сложных процентов раз в полгода. Сколько денег будет на счету после того, как будет сделан последний вклад, если в течение трех лет он шесть раз внес деньги на счет, по одному разу в конце каждого периода выплаты процентов? Чтобы решить эту задачу, рассмотрим процесс на временной прямой. Используя формулу сложных процентов для суммы А = Р(1 + г)п, можно найти значение каждого вклада после того, как он принес сложные проценты на протяжении всех шести вкладов, как показано на рис. 3.12. Первый Второй Третий Годы 0 1 2 3 4 5 6 Периоды I 1 1 1 1 1 1 ► Рис. 3.12. Стоимость вклада
Глава 3. Финансовая математика 245 Конечно, величины, представленные на рис. 3.12, можно было бы вычислить, используя калькулятор, а затем сложить их и найти сумму, лежащую на счету после внесения всех шести вкладов — этот план в лучшем случае громоздкий. Вместо этого, воспользуемся другим подходом, который непосредственно приведет к формуле, дающей тот же результат за два шага (даже Тогда, когда количество вкладов очень велико). Для начала запишем общую сумму На счету после того, как было сделано шесть вкладов, в следующем виде S = 100 + 100 • 1,03 + 100 • 1,032 + 100 • 1,033 + 100 • 1,034 + 100 • 1,035. (3.4) Хотелось бы найти простой способ для вычисления этой суммы. Умножим левую и правую части уравнения (3.4) иа 1,03. 1,035 = 100 • 1,03 + 100 • 1,032 + 100 • 1,033 + 100 • 1,034 + 100 • 1,035 + 100 • 1,036. (3.5) Выделив в уравнении (3.5) уравнение (3.3), получим следующий результат. 1,035- S = 100 • 1,03е — 100; Новая запись намного компактнее. 0,035 = 100 • (1,03е - 100); 5 = 100 • (1 + 0,ОЗ)6 - 1 0,03 Формула вычисления величины S опре- - (3.6) деляет структуру общего уравнения v 7 В общем случае, если R — это периодический взнос, i — ставка за один период, а п — количество периодов, То будущая стоимость определяется следующим образом. S = R 4- 7?(1 + г) + /?(1 4- г)2 + • • • + R(1 + i)n 1. Сравните с уравнением 3.4. Повторив эти вычисления, можно получить общую формулу для будущей стоимости простого аннуитета: (1 4- i)n — 1 S = R- ; . Сравните с уравнением 3.6 (3.7) Задание 3.6. Проверьте формулу (3.7), применив к уравнению S R + Я(1 + г) + R(1 + г)2 + • • • + Я(1 + г)п-1 тот же метод, что был использован выше для суммирования в формуле (3.4). ■ Возвращаясь вышеописанному примеру, воспользуемся калькулятором для решения следующей задачи. 1 03® _ 1 S = 100 • * — од— « Для повышения уровня точности сохраним все зна- ’ чения в калькуляторе до самого конца; = 646,84 ДОЛл. округлим до необходимого количества десятичных знаков.
246 Часть II. Конечная математика Альтернативный способ вычисления величины S реализуется с помощью таблиц значений определенных функций, используемых в финансовой математике. Одной из таких функций является множитель в виде дроби из формулы (3.7), обозначаемый символом j. (1 + i)n - 1 Snli = : • ' г В таблицах, которые можно найти в учебниках по финансовому делу и математике, указаны значения $п-ц для различных чисел п и г. Чтобы вычислить окончательное значение S, число R следует умножить на величину sn-|i, как показано в формуле (3.7.). (Основным преимуществом калькулятора перед таблицами является то, что с его помощью можно справиться с гораздо большим количеством задач, чем при помощи таблицы, какой бы большой она ни была.) В формуле (3.7) довольно часто переменную S называют параметром FV (от future value — будущая стоимость), а переменную R — параметром РМТ (от payment — выплата). После учета этих обозначений получится формула, представленная в следующей врезке. Будущая стоимость простого аннуитета fl + — 1 FV = РМТ^—4 = РМТ8пЪ, (3.8) г 1 где РМТ — периодический взнос выплаты, i — ставка на один период, п — количество выплат (периодов), FV — будущая стоимость (сумма). (Примечание. Выплаты производятся в конце каждого периода.) Пример 3.12 (Будущая стоимость простого аннуитета). Какой будет величина аннуитета спустя 20 лет, если каждый год на счет, приносящий 8,5 сложного процента, начисляемого ежегодно, вносилось по 2000 долл.? Сколько из этой суммы составляют проценты? Решение. Чтобы найти величину аннуитета, воспользуемся формулой (3.8), в которой РМТ = 2000 долл., i = г = 0,085 и п = 20. FV = РМТ^^—- = г 1.O8520 - 1 = 2000 • ’ ——— = 96 754,03. 0,085 Воспользуемся калькулятором. Чтобы найти количество полученных процентов, после осуществления 20-го взноса из суммарной величины аннуитета следует вычесть общую сумму взносов (20 платежей по 2000 долл.) Взносы = 20 • 2000 = 40 000 долл. Проценты = стоимость - вклады = 96 754,03 — 40 000 = 56 754,03. На рис. 3.13, который был построен при помощи электронной таблицы, показан рост этого счета в течение 20 лет. ■
247 Глава 3. Финансовая математика А В . С D 1 Period Payment Interest Balance 2 1 $2,000 00 $0.00 $2,000.00 3 2 $2,000 00 i 170.00 $4,170.00 4 3 $2,000.00 $354.45 $6,524.45 5 4 $2,000.00 $554.58 . $9,079.03 6 5 $2,000 00 >771.72 111,850.75 7 6 $2,000 00 $1x007.31 $14,858.06 8 7 $2,000.00 $1,262.94 $16,120.99 9 8 $2,000.00 >1x540.28 $21,661.28 ■га 9 $2,000.00 >1,841.21 $25,502.49 ш 10 $2,000 00 $2,167.71 $29,670.20 UJ I 11 $2,000.00 >2,521.97 $34,192.17 ш 12 $2,000.00 $2,906.33 $39,090.50 ш 13 $2,000.00 $3,323.37 $44,42J .87 14 $2,000 00 $3,775 06 $50,197.73 ки 15 $2,000.00 $4,266.91 $56,464.54 ш 16 $2,000.00 $4,799.49 $63,264.02 ■п 1 17 $2,000.00 ♦5.577.44 $70,64J .47 ш 16 $2,000 00 f£o04.5i $76,645.99 ко 19 $2,000 00 $6,684.91 $67,330.90 20 $2,000.00 $7,423.13 .. $96,754.03 ез | Totals $40,000.00 WV54.Qj $100,000 $90,000 $80,000 $70,000 ~ $60,000 | $50,000 $40,000 $30,000 $20,000 $10,000 $0 1 3 5 7 9 11 13 15 17 19 Period Рис. 3.13. Простой айнуитет под 8,5 сложного процента, начисляемого ежегодно в течение 20 лет Упражнение 3.12. Какой будет величина аннуитета спустя 10 лет, если каждый год на счет, приносящий восемь сложных процентов, начисляемых раз в полгода, вносилось по 1000 долл, каждые шесть месяцев? Сколько из этой суммы составляют проценты? ■ Таблица на рис. 3.13 называется бухгалтерским балансом. Рассмотрим структуру этой таблицы более подробно. Первая строка — это особый случай, поскольку платеж был осуществлен в конце периода и проценты не начислялись. Каждая последующая строка таблицы вычислялась следующим образом. Платеж + проценты + старый баланс = новый баланс 2000 + 0,085 • 2000 + 2000 = 4170 Период 2 2000 + 0,085 • 4170 + 4170 = 6524,45 Период 3 Суммы внизу каждого столбца бухгалтерского баланса соответствуют результатам, полученным при использовании формулы (3.8), как и следовало ожидать. Хотя бухгалтерский баланс и удобен в некоторых ситуациях, основное внимание здесь будет уделено формуле (3.8). Существует множество важных задач, которые можно решить, только используя эту формулу. Задание 3.7. 1. Проанализируйте сходства и различия между графиками будущей стоимости простого аннуитета FV, зависящей от времени t, если каждый месяц в течение восьми лет на счет вносятся 1000 долл., а сложные проценты начисляются ежемесячно при годовых процентных ставках 4, 8 и 12% соответственно (рис. 3.14). 2. Проанализируйте связь между графиком уравнения у = 1002, где t — время в месяцах, и графиками, построенными при решении п. 1. ■
248 Часть II. Конечная математика Время, лет Рис. 3.14. Будущая стоимость простого аннуитета Амортизационные фонды У формулы для вычисления будущей стоимости простого аннуитета есть еще одно важное применение. Предположим, что родители новорожденного ребенка решили каждый год в день его рождения до 17 лет класть на счет, приносящий шесть сложных процентов, начисляемых ежегодно, сумму в размере РМТ долл. Деньги пойдут на оплату обучения в колледже. Каким должен быть ежегодный вклад РМТ, чтобы сумма на счету после 17-го вклада была равна 80 000 долл.? Здесь известны величины FV, inn, входящие в формулу (3.8), а задача заключается в вычислении суммы взноса РМТ. FV = РМТ^-^~— 80000 = РМТ1,06^ ~ 1; 0,06 /■Л7Т,«<«Ю |г = 2835,58 в год. Решим уравнение относительно РМТ. Воспользуемся калькулятором. Аннуитет, состоящая из 17 ежегодных вкладов по 2835,58 долл, под шесть сложных процентов, начисляемых ежегодно, через 17 лет будет стоить 80000 долл. Это один из многих примеров аналогичного типа, которые называют задачами об амортизационном фонде. Вообще говоря, любой счет, учрежденный для накопления денежных средств с целью погашения будущих финансовых обязательств или долгов, называется амортизационным. Если взносы выплачиваются в форме простого аннуитета, то необходимо просто решить уравнение (3.8) относительно взноса в амортизационный фонд РМТ: РМТ = FV - . (3.9) (1+г)п-1 ' Важно осознавать, что удобная в использовании формула (3.9) — это просто один из вариантов формулы (3.8). Взнос в амортизационный фонд всегда можно найти, сначала подставив соответствующие значения в формулу (3.8), а затем решив полученное уравнение
Глава 3. Финансовая математика 249 относительно величины РМТ, как это было сделано в примере с колледжем, рассмотренном выше. Кроме того, можно выполнить подстановку непосредственно в формулу (3.9), как в следующем примере. Читатель может пользоваться наиболее удобным для него методом. Е Пример 3.13 (Вычисление взноса в амортизационный фонд). Компания установила, что через пять лет ей понадобится заменить часть оборудования на сумму 800 000 долл. Чтобы иметь эти деньги в распоряжении спустя пять лет, она основала амортизационный фонд, делая равные ежемесячные взносы на счет, приносящий 6,6 сложного процента, начисляемого ежемесячно. 1. Каким должен быть каждый взнос? 2. Сколько процентов будет получено за последний год? Решение. 1. Чтобы найти величину РМТ, можно воспользоваться либо формулой (3.8), либо формулой (3.9). Выберем формулу (3.9), в которой FV = 800 000 долл., i = = = 0,0055 и п = 12 • 5 = 60. РМТ = FV- J = (1 +г)п — 1 0,0055 = 11290,42 долл, в месяц. 2. Чтобы найти проценты, полученные в течение пятого года, воспользуемся сначала формулой (3.8), в которой РМТ = И 290,42 долл., i = 0,0055 и п = 12 • 4 = 48, и найдем сумму, которая будет лежать на счету спустя четыре года. FV = РМТ^^-—- = г = И 290,42 • 1,005548 - 1 0,0055 = 618 277,04 долл. Сумма спустя 4 года В течение пятого года сумма на счету вырастет с 618 277,04 до 800 000 долл. Часть этого прироста составят 12 ежемесячных взносов в размере 11 290,42 долл. Остальная часть прироста обеспечена процентами. 800000 - 618277,04 = 181 722,96 12 11290,042 = 135 485,04 181 722,96 - 135 485,04 = 46 237,92 долл. Прирост за 5-й год Взносы в течение 5-го года Проценты за 5-й год «№ Упражнение 3.13. &М Для строительства пристани в городе было решено выпустить облигации. Требовалось, чтобы город каждые три месяца делал регулярные взносы в амортизационный фонд, приносящий 5,4 сложного процента, начисляемого ежеквартально. В конце десятого года облигационные обязательства будут погашены суммой в 5 000 000 долл.
250 Часть II. Конечная математика 1. Каким должен быть каждый взнос? 2. Сколько процентов будет получено в течение десятого года? ■ Задание 3.8. Предположим, что один человек намерен учредить простой аннуитет, приносящую 7,5 сложного процента, начисляемого ежемесячно, которая на его 70-й день рождения будет стоить один миллион долларов. Какими бы были его ежемесячные взносы, если бы он начал их делать в 20 лет? В 35 лет? В 50 лет? ■ И WWW Пример 3.14 (Рост капитала на счету в банке IRA). Джейн ежегодно делает взнос в размере 2000 долл, на пенсионный счет в банке Roth IRA, приносящий 6,85 сложного процента, начисляемого ежегодно. (Проценты, которые приносит счет в банке Roth IRA, освобождены от налогов.) Из-за смены места работы эти взносы были прекращены через десять лет. Однако счет продолжал приносить проценты до тех пор, пока Джейн не вышла на пенсию через 25 лет после того, как был сделан последний вклад. Сколько денег оказалось на счету, когда Джейн вышла на пенсию? Решение. Прежде всего воспользуемся формулой для вычисления будущей стоимости, в которой РМТ = 2000 долл.; i = 0,0685 и п = 10, чтобы найти сумму на счету через 10 лет. FV = РМТ^-^—- = г 1,068510 - 1 = 2000 ■ — = 0,0685 = 27 437,89 долл. Теперь воспользуемся формулой для вычисления сложных процентов из раздела 3.2, в которой Р = 27 437,89 долл.; i = 0,0685 и п = 25, и найдем сумму, которая лежала на счету, когда Джейн вышла на пенсию. А = Р(1-Н)П = = 27437,89 • 1,068525 = = 143 785,10 долл. О Упражнение 3.14. Вернемся к примеру 3.14. Мэри открыла пенсионный счет в банке Roth IRA с той же процентной ставкой в тот момент, когда Джейн перестала делать взносы на свой счет. Сколько должна вкладывать Мэри на свой счет каждый год в течение следующих 25 лет, чтобы при выходе на пенсию у нее была та же сумма, что и у Джейн? ■ Задание 3.9. Вернемся к примеру 3.14 и упражнению 3.14. Сколько всего денег вложила Джейн, чтобы получить 143 785,10 долл, при выходе на пенсию? Сколько всего денег вложила Мэри, чтобы при выходе на пенсию получить ту же сумму? Разумно ли открывать пенсионный счет как можно раньше? ■
Глава 3. Финансовая математика 251 Приближенный метод вычисления процентных ставок Для решения уравнения (3.8) относительно величин РМТ или п можно воспользоваться алгебраическими преобразованиями, однако для его решения относительно величины i этого сделать нельзя. Однако, чтобы приближенно найти значение i с точностью до нужного числа десятичных знаков, можно воспользоваться графическими методами или программой для решения уравнений. кх Пример 3.15 (Приближенный метод вычисления процентной ставки). Человек делает ежемесячные взносы в форме простого аннуитета в размере 100 долл. Спустя 30 лет стоимость аннуитета составила 160000 долл. Какую сложную годовую процентную ставку, начисляемую ежемесячно, приносил такой аннуитет в течение всего 30- летнего периода? Выразите ответ в процентах с точностью до двух десятичных знаков. Решение. Подставляя величины FV = 160 000 долл., РМТ = 100 долл, и п = 30 • 12 = = 360 в формулу (3.8), получим следующее уравнение. (1 + й360 - 1 160000 = 100 • г Приближенное решение этого уравнения можно найти, используя графические методы (рис. 3.11, а и рис. 3.11, б) Или же программу для решения уравнений (рис. 3.15, в). На рис. 3.11, б и рис. 3.11, в показано, что i = 0,0069567; а 12(г) = 0,0834804. Таким образом, годовая процентная ставка (с точностью до двух десятичных знаков) равна г = 8,35%. ■ Plotl Mott Mott xYiB100<<l+X)A36 0-1)/X 20160000 xYh = xYs = xVfi = IFU=PMT<<l+i)AN-l>/i FU=160000 PMT=100 ■i=.0069567004803499 N=360 bound=C-lE99,1e99> ■left-rt=l,26E-6 Рис. 3.15. Вычисление годовой процентной ставки 1;Т;1:НЯ RTTZ1RW1 Упражнение 3.15. Человек делает ежегодные взносы в форме простого аннуитета в размере 1000 долл. Спустя 20 лет стоимость аннуитета составила 55 000 долл. Какую сложную годовую процентную ставку приносил аннуитет в течение этого 20-летнего периода? Выразите ответ в процентах с точностью до двух десятичных знаков. ■
252 Часть II. Конечная математика Ответы к упражнениям 3.12. Величина годового аннуитета: 29 778,08 долл.; проценты: 9778,08 долл. 1) 95094,67. 2) 248628,89. 3.13. 2322,73 долл. 3.14. 9,64%. Практикум 3.3 В задачах 1-12, используя формулу для вычисления будущей стоимости, требуется найти каждую из указанных величин. А 1. п = 20; г = 0,03; РМТ = 500 долл.; FV = ?. 2. п = 25; i = 0,04; РМТ = 100 долл.; FV = ?. 3. п = 40; г = 0,02; РМТ = 1000 долл.; FV = 1. 4. п = 30; г = 0,01; РМТ = 50 долл.; FV = 2. Б 5. FV = 3000 долл.; п = 20; г = 0,02; РМТ = ?. 6. FV = 8000 долл.; п = 30; г = 0,03; РМТ = 2. 7. FV = 5000 долл.; п = 15; г = 0,01; РМТ = 2. 8. FV = 2500 долл.; п = 10; г = 0,08; РМТ = 2. В 9. FV = 4000 долл.; i = 0,02; РМТ = 200 долл.; п = ?. 10. FV = 8000 долл.; г = 0,04; РМТ = 500 долл.; п = 2. И. FV = 7600 долл.; РМТ = 500 долл.; п = 10; i = ?. (Округлите ответ до двух десятичных знаков.) *5 12. FV = 4100 долл.; РМТ = 100 долл.; п - 20; г = ?. (Округлите ответ до двух десятичных знаков.) Применение математики Экономика и бизнес 13. WWW Недавно страховая компания Guaranty Income Life предложила аннуитет, приносящий 6,65 сложного процента, начисляемого ежемесячно. Сколько денег будет на счету спустя 10 лет, если каждый месяц на этот счет вносится 500 долл.? Сколько из них составят проценты? 14. WWW Недавно страховая компания USG Annuity and Life предложила аннуитет, приносящий 7,25 сложного процента, начисляемого ежемесячно. Сколько денег будет на счету спустя 15 лет, если каждый месяц на этот счет вносится 1000 долл.? Сколько из них составят проценты?
Глава 3. Финансовая математика 253 15. Чтобы собрать достаточную сумму денег для оплаты дома в рассрочку, семейная пара кладет 300 долл, в месяц на счет, приносящий шесть сложных процентов, начисляемых ежемесячно. Сколько денег будет на счету через пять лет, если взносы делаются в конце каждого периода? 16. У человека, занятого собственным делом, есть пенсионный план. (Этот вид плана освобожден от налогов до тех пор, пока деньги не будут сняты.) Сколько денег будет на счету спустя 20 лет, если на счет, приносящий восемь сложных процентов, начисляемых ежегодно, каждый год вносится 7500 долл.? 17. WWW Страховая компания Sun America недавно предложила аннуитет, приносящий 6,35 сложного процента, начисляемого ежемесячно. Какие равные ежемесячные взносы нужно делать на такой вклад, чтобы через 15 лет иметь 200 000 долл.? 18. WWW Недавно страховая компания The Hartford предложила аннуитет, приносящий 5,5 сложного процента, начисляемого ежемесячно. Какие равные ежемесячные взносы нужно делать на такой вклад, чтобы через десять лет иметь 100 000 долл.? 19. Компания установила, что через восемь лет для замены компьютера ей понадобится 100000 долл. Какими были бы взносы, если бы руководство решило учредить амортизационный фонд, ежемесячно внося фиксированную сумму на счет, приносящий 7,5 сложного процента, начисляемого ежемесячно? 20. Родители учредили амортизационный фонд, чтобы через 15 лет получить 120000 долл, для оплаты обучения их ребенка в колледже. Какую сумму необходимо раз в полгода вносить на счет, приносящий 6,8 сложного процента, начисляемого раз в полгода? 21. Составить бухгалтерский баланс, показывающий проценты, полученные в течение года, и баланс в конце каждого года, если в качестве простого аннуитета, приносящего 8,32 сложного процента, начисляемого ежегодно, в конце каждого года в течение пяти лет на счет вносится 1000 долл. 22. Составить бухгалтерский баланс, показывающий проценты, полученные в течение квартала, и баланс в конце каждого квартала, если в качестве простого аннуитета, приносящего 7,9 сложного процента, начисляемого ежеквартально, в конце каждого квартала в течение двух лет на счет вносятся 2000 долл. Kl Проверьте результаты задач 21 и 22, построив бухгалтерский баланс с помощью графической утилиты. 23. Начиная с января человек планирует вкладывать 100 долл, в конце каждого месяца на счет, Приносящий девять сложных процентов, начисляемых ежемесячно. Каждый год Необходимо выплачивать налог на проценты, полученные в течение этого года. Вычислите проценты, полученные в течение каждого когда, для первых трех Лет. 24. Вычислите проценты, полученные в течение каждого из трех лет, если на счет, приносящий 12 сложных процентов, начисляемых ежемесячно, в течение этого периода вносились 500 долл.
254 Часть II. Конечная математика 25. Боб сделал свой первый вклад, равный 1000 долл., на счет, приносящий 6,4 сложного процента, начисляемого ежегодно, на свое 24-летие, а свой последний вклад, равный 1000 долл., — на свой 35-й день рождения (всего 12 одинаковых вкладов). Хотя дополнительные деньги и не вносились, сумма на счету продолжала приносить 6,4 сложного процента, начисляемого ежегодно, до тех пор, пока Боб не вышел на пенсию в 65 лет. Сколько денег было на счету, когда Боб вышел на пенсию? 26. Вернемся к задаче 25. В отличие от Боба, Джон замешкался и не делал своего первого вклада, равного 1000 долл., до тех пор, пока ему не исполнилось 36 лет, но затем он продолжал вносить по 1000 долл, каждый год до тех пор, пока ему не исполнилось 65 лет (всего 30 вкладов). Сколько денег было бы у Джона на счету, если бы он сделал свой последний вклад на 65-й день рождения, а счет приносил 6,4 сложного процента, начисляемого ежегодно? 27. Вернемся к задачам 25 и 26. Сколько должен был бы вносить Джон каждый год, чтобы получить при выходе на пенсию ту же сумму, что и Боб? 28. Вернемся к задачам 25 и 26. Предположим, чтоб Боб решил продолжать вносить на свой счет по 1000 долл, каждый год до своего 65-го дня рождения. Сколько денег должен был бы вносить Джон ежегодно, чтобы получить в 65 лет ту же сумму, что и Боб, если бы он все же отложил открытие своего счета до 36 лет? 29. WWW Банковская служба Compubank предлагает депозитный счет денежного рынка с APY в размере 4,86%. а) Какой будет соответствующая годовая номинальная процентная ставка, если сложные проценты начисляются ежемесячно? б) Какие равные взносы необходимо делать человеку каждый месяц, если он хочет спустя четыре года иметь на счету 10 000 долл.? 30. WWW Онлайновое банковское отделение компании American Express предлагает депозитный счет денежного рынка с APY в размере 5,65%. а) Какой будет соответствующая годовая номинальная процентная ставка, если сложные проценты начисляются ежемесячно? б) Какие равные взносы необходимо делать человеку каждый месяц, если он хочет спустя восемь лет иметь на счету 1000 000 долл.? 31. Человек может позволить себе ежемесячно вносить 200 долл, на счет, приносящий 5,7 сложного процента, начисляемого ежемесячно. Сколько времени понадобится для того, чтобы получить 7000 долл, на покупку лодки. (Неточный результат следует округлить до следующего большего месяца.) 32. Компания учредила амортизационный фонд для модернизации офисного оборудования с ежемесячным взносом в размере 2000 долл, на счет, приносящий 6,6 сложного процента, начисляемого ежемесячно. Сколько времени уйдет на то, чтобы на счету накопилось 100000 долл.? (Неточный результат следует округлить до следующего месяца.)
Глава 3. Финансовая математика 255 zl В задачах 33-36, используя методы графической аппроксимации или программу для решения уравнений, найдите приблизительное значение требуемой процентной ставки. Выразите каждый ответ в процентах с точностью до двух десятичных знаков. 33. Человек делает ежегодные взносы в размере 1000 долл, в качестве простого аннуитета. В конце пятого года сумма аннуитета составила 5840 долл. Какую номинальную годовую сложную процентную ставку приносит этот аннуитет? 34. Человек ежегодно вкладывает 2000 долл, на счет. В конце шестого года сумма в пенсионном фонде составила 14000 долл. Какую номинальную годовую сложную процентную ставку приносит этот фонд? 35. В конце каждого месяца служащий вносит 50 долл, в фонд Рождественского клуба. В конце года в фонде будет находится 620 долл. Какую номинальную годовую сложную процентную ставку, начисляемую ежемесячно, приносит этот фонд? 36. В конце каждого месяца служащий вносит 80 долл, на счет в кредитном союзе. В конце второго года на счету будет находиться 2100 долл. Какую номинальную годовую сложную процентную ставку, начисляемую ежемесячно, приносит этот счет? В задачах 37 и 38 нужно ответить на вопросы, используя методы графической аппроксимации. 37. При каких условиях простой аннуитет, состоящий из ежеквартальных взносов в размере 500 долл, при ставке в шесть сложных процентов, начисляемых ежеквартально, будет стоить больше, чем капитал в размере 5000 долл., вложенный под четыре простых процента? 38. При каких условиях простой аннуитет, состоящая из ежеквартальных взносов в размере 200 долл, при ставке в пять сложных процентов, начисляемых ежемесячно, будет стоить больше, чем капитал в размере 10 000 долл., вложенный под 7,5 сложного процента, начисляемого ежемесячно? 3.4. Текущая стоимость аннуитета: погашение долга ■ Текущая стоимость аннуитета ■ Погашение долга ■ График погашения долга ■ Общая стратегия решения задач Текущая стоимость аннуитета Какую сумму необходимо внести на счет, приносящий шесть сложных процентов, начисляемых ежегодно, чтобы иметь возможность снимать по 1000 долл, каждые шесть месяцев в течение следующих трех лет? (После последней выплаты денег на счету остаться не должно.)
256 Часть II. Конечная математика В данном случае требуется найти текущую стоимость каждой тысячи долларов, которая выплачивается в течение трех лет. Это можно сделать, решив уравнение сложных процентов относительно величины Р. Л = Р(1 + г)п; '’=7ТТ7?=Л,‘ -1! ’ Ставка за период равна i = = 0,03. Текущая стоимость первой выплаты равна 1000 • • 1,03-1, текущая стоимость второй выплаты равна 1000 • 1,03-2 и т.д. На рис. 3.16 этот процесс отображен на временной прямой. Годы Сейчас Первый Второй Третий Периоды 0 1 2 3 4 5 6 I 1 1 1 1 1 1 ► Текущая стоимость Выплаты, долл. 1000 1000 1000 1000 1000 1000 Г 1000 1.03-1 1000 1. оз-2 1000 • 1 .оз-3 1000 • 1.03-4 1000 • 1 .оз-5 1000 1. оз-6 Рис. 3.16. Текущая стоимость аннуитета Чтобы найти суммарные текущие стоимости всех выплат (которые будут равны сумме, необходимой для того, чтобы купить аннуитет сейчас), можно было бы рассчитать величину каждой текущей стоимости на рис. 3.16, используя калькулятор, а затем просуммировать результат. Поскольку этот процесс в общем случае довольно сложен, в частности, если количество выплат велико, воспользуемся методом, который был применен в предыдущем разделе, для выведения формулы, дающей тот же результат за два шага. Для начала запишем общую сумму текущих стоимостей в следующем виде. Р = 1000 • 1,03-1 + 1000 • 1,03-2 + • • • + 1000 • 1,03~6. (3.10) Умножая каждую часть уравнения (3.10) на 1,03, получим следующий результат. 1,03Р = 1000 + 1000 • 1,03-1 + • • • + 1000 • 1,03-5. (3.11) Вычленим теперь уравнение (3.10) в уравнении (3.11). 1,03Р - Р = 1000 - 1000 • 1,03-6; 0,03Р = 1000 • (1000 - 1,03-6); 1 (14-0 03) ~6 Формула вычисления величины Р = 1000 • ■—-—. Р определяет структуру общего (3.12) U,Uo ’ уравнения
Глава 3. Финансовая математика 257 В общем случае, если R — это размер периодического взноса, i — ставка за один период, ап — количество периодов, то текущая стоимость всех выплат определяется следующим образом. Р == Я(1 + г)-1 + Я(1 + г)-2 + • • • + Л(1 + г)"". Проделав то же, что и в приведенном выше примере, можно получить общую формулу для текущей стоимости обычного аннуитета. P = R1~(1 + ^ П. (3.13) г Задание ЗЛО. Проверьте формулу (3.13), применив к уравнению Р *= Я(1 + г)-1 + Я(1 + г)-2 + ••• + /?(! + г)"п тот же метод, что был использован выше для суммирования уравнения (3.10). ■ Возвращаясь к вышеописанному примеру, воспользуемся калькулятором. = 5417,19 долл. Множитель в виде дроби из формулы (3.13) обозначается символом ап^. а — (1 + г)п anli = . • Альтернативный приведенному выше способ вычисления величины Р заключается в использовании таблиц, в которых даны значения ап^ для различных значений п и г. Чтобы вычислить значение Pt нужно величину R умножить на число ап^, как показано в формуле (3.13). (Как отмечалось ранее, основным преимуществом использования калькулятора перед таблицами является то, что с помощью калькулятора можно справиться с намного большим количеством задач, чем с помощью любой таблицы.) В формуле (3.13) Довольно часто величину Р называют параметром PV (от present value — текущая стоимость), а величину R — параметром РМТ (от payment — выплата). Внеся эти изменения, Получаем следующее. Текущая стоимость простого аннуитета PV « РМТ—= PMTan] i, (3.14) где РМТ — периодические выплаты, i — ставка на один период, п — количество периодов, PV — текущая стоимость всех выплат. (Примечание. Выплаты производятся в конце каждого периода.)
Часть II. Конечная математика 258 Пример 3.16 (Текущая стоимость аннуитета). Чему равна текущая стоимость аннуитета, ежемесячные взносы которого равны 200 долл, в месяц в течение пяти лет, если деньги приносят шесть сложных процентов, начисляемых ежемесячно? Решение, Чтобы решить эту задачу, воспользуемся формулой (3.14), в которой РМТ = = 200 долл.; i = = 0,005 и п = 12 • 5 = 60. PV = РМТ-—(1 + г) = i Воспользуемся калькулятором. = 10 345,11 долл. И Упражнение 3.16. Какую сумму нужно положить на счет, приносящий восемь сложных процентов, начисляемых ежеквартально, чтобы получать ежеквартальные выплаты в размере 1000 долл, в течение следующих четырех лет? ■ И WWW Пример 3.17 (Составление пенсионного плана). Недавно страховая компания Lincoln Benefit Life предложила простой аннуитет, приносящий 6,5 сложного процента, начисляемого ежемесячно. Человек планирует в течение 25 лет делать на этот счет равные ежегодные взносы, чтобы затем осуществить 20 равных ежегодных выплат в размере 25 000 долл., снизив баланс на счету до нуля. Сколько денег нужно вкладывать ежегодно, чтобы накопить капитал, позволяющий осуществлять такие выплаты? Сколько всего процентов будет получено в течение этого 45-летнего процесса? Решение, В этой задаче рассматривается как будущая, так и текущая стоимость. На рис. 3.17 проиллюстрировано движение денег на счете. Годы 1 2 ... 25 12 ... 20 I 1 1 1 1 1 > Выплаты, долл. РМТ РМТ РМТ 25 000 25 000 25 000 I I L_M -J 1 I Будущая стоимость: Текущая стоимость: увеличивающийся баланс уменьшающийся баланс Рис. 3.17. Движение денег на счете Поскольку в задаче нужно выполнять выплаты, для начала вычислим текущую стоимость, необходимую для того, чтобы обеспечить их. Используя формулу (3.14), в которой РМТ = 25 000 долл.; i = 0,065 и п = 20, получаем PV = = 1 - 1,О65-20 = 25 000 • ——— = Воспользуемся калькулятором. = 275462,68 долл.
Глава 3. Финансовая математика 259 Теперь найдем вклады, которые за 25 лет дадут будущую стоимость, равную 275 462,68 долл. Используя формулу (3.9) из раздела 3.3, в которой FV = 275462,68; i = 0,065 и п — 25, получаем следующий результат. РМТ = FV ~ = (1 + г)п-1 = 276462'08'Ы^1 = 4677,76 долл. Воспользуемся калькулятором. Таким образом, ежегодные вклады в размере 4677,67 долл, в течение 25 лет обеспечат 20 ежегодных выплат В размере 25 000 долл. Проценты, полученные в течение всего 45- летнего процесса, вычисляются следующим образом. Проценты = (суммарные выплаты) — (суммарные вклады) = = 20 *25 000 - 25 -4677,76 = = 383,056 долл. а И Упражнение 3.17. Вернемся к примеру 3.17. Сколько можно будет выплачивать ежегодно в течение последующих 20 лет, если в течение первых 25 лет ежегодно вкладывать по 2000 долл?и Погашение долга У формулы (3.14) для текущей стоимости обычной ренты есть еще одно важное применение. Предположим, что человек взял ссуду в банке на сумму 5000 долл., чтобы купить автомобиль, и договорился оплатить эту ссуду 36 равными ежемесячными выплатами, включая все причитающиеся проценты. Какой должна быть каждая выплата, чтобы погасить весь долг, включая проценты, за 36 месяцев, если банк взимает 1% в месяц с непогашенного остатка (12 сложных процентов в год, начисляемых ежемесячно)? Фактически банк Купил у этого человека аннуитет. Вопрос же заключается в следующем: какими будут ежемесячные выплаты (РМТ), если банк выплатит человеку 5000 долл, (текущую стоимость) за аннуитет, приносящий ему РМТ долл, в месяц в течение 36 месяцев, при ставке в 12 сложных процентов, начисляемых ежемесячно? (Следует отметить, что величина аннуитета в конце 36-го месяца будет равна нулю.) Чтобы вычислить параметр РМТ, необходимо просто воспользоваться формулой (3.14), в которой PV = 5000 долл.; i = 0,01 И п = 36. PV = РМТ^^^-; I 1 — 0 01 “36 РМТ = 166,07 долл, в месяц. Решим уравнение относительно РМТ и воспользуемся калькулятором. За 16607 долл, в месяц спустя 36 месяцев машина будет принадлежать вкладчику. Иначе говоря, этот человек погасил долг 36 равными ежемесячными выплатами. Вообще
260 Часть II. Конечная математика говоря, погашение долга означает, что долг возвращается в течение заданного промежутка времени равным периодическими платежами, которые включают сложные проценты. Как правило, бывает интересно рассчитать равные периодические выплаты. Решая уравнение (3.14) для будущей стоимости относительно величины РМТ, получим следующую формулу погашения. PMT = PV-—* ,ч . (3.15) 1- (l+i)~n v Формула (3.15) — это просто один из вариантов формулы (3.14). Следовательно, для вычисления размеров периодических взносов РМТ можно использовать любую из формул. И Пример 3.18 (Ежемесячные взносы и суммарные проценты при погашении долга). Предположим, что некто купил телевизор за 800 долл, и договорился оплатить его стоимость 18 равными ежемесячными взносами при ставке 1,5% в месяц, начисляемой на непогашенный остаток. 1. Каков размер его выплат? 2. Сколько он заплатит процентов? Решение. 1. Воспользуемся формулой (3.15), в которой PV = 800 долл.; i = 0,015 и п = 18. РМТ = PV i 1 - (1 + г)~п = 800- 0,015 1 - 1,015~18 Воспользуемся калькулятором. = 51,04 долл, в месяц. 2. Суммарные проценты = (сумма всех выплат) — (исходный кредит) = 18-51,04-800 = = 118,72 долл. И Упражнение 3.18. Один человек продал другому автомобиль за 2400 долл, и договорился продать его в кредит под 1% в месяц, начисляемый на непогашенный остаток. Сколько денег должен выплачивать покупатель ежемесячно, чтобы кредит был погашен в течение 24 месяцев? Сколько процентов получит продавец? ■ Задание 3.11. Чтобы купить дом, семья планирует подписать закладную на сумму 70 000 долл, при ставке 8%, начисляемой на непогашенный остаток. Проанализируйте преимущества и недостатки 20-летней закладной по сравнению с 30-летней закладной. Сравните месячные платежи и выплаченные суммарные проценты. ■
Глава 3. Финансовая математика 261 График погашения долга Что происходит, когда Человек погашает долг равными периодичными платежами и в какой-то момент решает выплатить остаток долга единовременной выплатой? Такое происходит, когда продают дом с неоплаченной закладной. Чтобы понять, что происходит в такой ситуации, нужно подробнее изучить процесс погашения долга. Начнем с достаточно простого примера, чтобы выяснить, как каждая выплата влияет на долг. Е Пример 3.19 (Построение графика выплат). Человек одолжил 500 долл., которые он договорился Вернуть шестью равными ежемесячными выплатами при ставке 1% в месяц, начисляемой на Непогашенный остаток. Какая часть каждой ежемесячной выплаты шла на оплату процентов, а какая — на оплату непогашенного остатка? Решение. Прежде всего, используя формулу (3.14) или (3.15), вычислим требуемые ежемесячные выплаты. Возьмем формулу (3.15), в которой PV = 500 долл.; i = 0,01 и п = 6. 7 РМТ = PV-—- — 1 - (1 4- г)-‘ 0,01 1 - 1,01 -6 = 500* Воспользуемся калькулятором. =» 86,27 долл, в месяц. В конце первого месяца проценты равны 500 • 0,01 = 5,00 долл. Выплаты при погашении долга делятся на две части: оплата причитающихся процентов и оплата непогашенного остатка (выплата основной суммы). Ежемесячная выплата, долл. 86,27 Причитающиеся проценты, долл. 5,00 Оплата непогашенного остатка, долл. 81,27. Неоплаченный остаток в следующем месяце вычисляется следующим образом. Предыдущий Оплата непогашенный непогашенного Новый непогашенный остаток, Долл. остатка, долл. 500,00 - 81,27 остаток, долл. = 418,73. В конце второго месяца проценты, начисленные на непогашенный остаток в размере 418,73 долл., равны следующей сумме. 418,73 • 0,01 = 4,19 долл. Таким образом, в конце второго месяца ежемесячная выплата в размере 86,27 долл, следующим образом покрывает проценты и оплату непогашенного остатка. 86,27 = 4,19 + 82,08 долл.,
262 Часть П. Конечная математика а неоплаченный остаток в третьем месяце составит 418,73 - 82,08 = 336,65 долл. Этот процесс продолжается до тех пор, пока не будут сделаны все выплаты, а непогашенный остаток не станет равным нулю. Расчеты для каждого месяца приведены в табл. 3.3, которую называют также графиком погашения. Таблица 3.3. График погашения долга Номер выплаты Сумма выплаты, долл. Проценты, долл. Оплата непогашенного остатка, долл. Непогашенный остаток, долл. 0 500,00 1 86,27 5,00 81,27 418,73 2 86,27 4,19 82,08 336,95 3 86,27 3,37 82,90 253,75 4 86,27 2,54 83,73 170,02 5 86,27 1,70 84,57 85,45 6 86,30 0,85 85,45 0,00 Итого 517,65 17,65 500,00 Следует отметить, что последнюю выплату пришлось увеличить на 0,03 долл., чтобы непогашенный остаток стал равен нулю. Это маленькое несоответствие возникло из-за ошибок округления в процессе вычисления. Практически всегда последнюю выплату необходимо корректировать, чтобы результирующий неснижаемый остаток в точности был равен нулю. ■ В Упражнение 3.19. Постройте график погашения долга в размере 1000 долл., который должен быть погашен шестью равными ежемесячными выплатами при ставке 1,25% в месяц, начисляемой на непогашенный остаток. ■ П Пример 3.20 (Собственный капитал в рыночной стоимости дома). Семья десять лет назад купила дом за 80 000 долл. Дом они купили в кредит, оплатив первый взнос в размере 20% и подписав 30-летнюю закладную при ставке 9%, начисляемой на погашенный остаток. Чистая рыночная стоимость дома (сумма, полученная после вычитания всех затрат, связанных с продажей) сейчас составляет 120 000 долл., и семья хочет его продать. Какой собственный капитал (с точностью до ближайшего доллара) есть сейчас у семьи, после того, как она сделала 120 ежемесячных выплат? [Собственный капитал = = (текущая чистая рыночная стоимость) — (непогашенный остаток по кредиту).]6 63десь понятие собственный капитал используется в его обычном смысле. Если семья хочет продать этот дом и купить дом подороже, то стоимость нового дома, который они могут позволить себе купить, часто зависит от их собственного капитала в первом доме, где собственный капитал определяется приведенным здесь уравнением. При рефинансировании кредита, взятого для покупки дома, или получении “кредита под собственный капитал”, новая закладная (или вторая закладная) часто основывается на собственном капитале в рыночной стоимости дома. Другое, более формальное, определение собственного капитала здесь рассматриваться не будет.
Глава 3. Финансовая математика 263 Решение. Как же найти непогашенный остаток по кредиту спустя десять лет или после 120 ежемесячных выплат? Один из способов заключается в построении графика погашения, но для этого Понадобилась бы таблица с 120 строками. К счастью, есть более простой способ. Неоплаченный остаток после 120 выплат — это сумма кредита, которая должна быть выплачена с помощью остальных 240 ежемесячных выплат (в течение 20 лет, оставшихся ДЛЯ погашения кредита). Поскольку в кредитном учреждении кредит рассматривают как аннуитет, который они купили у семьи, непогашенный остаток по кредиту при оставшихся п выплатах равен текущей стоимости этого аннуитета и может быть вычислен с помощью формулы (3.14). Поскольку в формуле (3.14) необходимо знать ежемесячные выплаты, сначала, используя формулу (3.15), необходимо вычислить величину PMTt Этап 1. Найдем ежемесячные выплаты: РМТ = PV-.—— - 1 - (1 + i)~n « 64000- 0,0075 1 - 1,0075—360 PV= 0,80 • 80 000 долл. = = 64 000 долл.; 0,09 г= —— = 0,0075; 12 п= 12*30 = 360. Воспользуемся калькулятором. 514,96 долл, в месяц. Этап 2. Найдем текущую стоимость для 20-летней ренты с ежемесячными выплатами 514,96 долл. pv = Рмт-—(1Д212 = г = 514,96 • 1 - 1 ,ОО75-240 0,0075 = 57 235 долл. РМТ= 514,96 долл.; п= 12 • 20 = 240; 0,09 г= —— = 0,0075. 12 Воспользуемся калькулятором. Непогашенный остаток по кредиту. Этап 3. Найдем собственный капитал. Собственный капитал / Текущая чистая у рыночная стоимость Непогашенный остаток по кредиту 120000- 57235 = зз 62 765 долл. В Упражнение 3.20. Семейная пара 20 лет назад купила дом за 65 000 долл. Дом они купили в кредит, оплатив первый взнос в размере 20% и подписав 30-летнюю закладную при ставке 8%, начисляемой на погашенный остаток. Чистая рыночная стоимость дома (сумма, полученная после вычета всех затрат, связанных с продажей) сейчас
264 Часть II. Конечная математика составляет 130 000 долл., и пара хочет его продать. Каким собственным капиталом (с точностью до ближайшего доллара) обладает семья после 240 ежемесячных выплат? ■ Непогашенный остаток по кредиту в примере 3.20 кажется на удивление большим по сравнению с долгом, оставшимся после выплат в течение 10 лет, однако долгосрочное погашение долга начинается с очень небольших сокращений непогашенного остатка. Например, проценты, причитающиеся в конце самого первого периода погашения кредита в примере 3.20, были равны следующей величине. 64000 • 0,0075 = 480,00 долл. Первая ежемесячная выплата делилась следующим образом. Ежемесячная Причитающиеся Оплата непогашенного выплата, долл. проценты, долл. остатка, долл. 514,96 - 480,00 = 34,96. Таким образом, на оплату непогашенного остатка ушло только 34,96 долл. Задание 3.12. 1. У семьи есть 30-летняя закладная на 85 000 долл, при ставке 9,6 сложного процента, начисляемого ежемесячно. Показать, что ежемесячные выплаты по ней составляют 720,94 долл. 2. Объясните, почему уравнение 1 - 1,ОО8-12(зо-а:) у = 720,94 У ’ 0,008 описывает непогашенный остаток по кредиту спустя х лет. 3. Найдите непогашенный остаток спустя 5, 10 и 15 лет. 4. Когда непогашенный остаток будет в точности равен половине исходной суммы 85 000 долл.? 5. Решите задачу 4, используя методы графической аппроксимации в графической утилите (рис. 3.18). ■ Рис. 3.18. График непогашенного остатка по кредиту
Глава 3. Финансовая математика 265 Общая стратегия решения задач После выполнения заданий раздела 3.4, следует переходить к упражнениям из общего практикума. Это даст Ценный опыт в классификации различных типов задач, рассмотренных в данной главе. Хотя все встречающиеся типы задач классифицировать невозможно, читателю могут помочь следующие принципы. Следует знать, что для решения некоторых задачах может понадобиться несколько этих формул, а для решения других не понадобится ни одна. Стратегия решения математических задач в финансовом деле Этап 1. Определите, рассматриваются ли в задаче одиночные выплаты или же последовательность равных периодических выплат. Задачи на простые и сложные проценты содержат одну текущую и одну будущую стоимость. Простые аннуитеты могут рассматриваться как с текущей, так и с будущей стоимостью, однако в этом случае всегда есть последовательность равных периодических выплат. Этап 2. Если рассматривается одиночная выплата, следует установить, какие используются проценты: простые или сложные. Простые проценты, как правило, используются для периодов длительностью год или менее, а сложные проценты применяются для долгосрочных периодов. Этап 3. Если рассматривается последовательность периодических выплат, следует определить, Вносятся ли деньги на счет, сумма на котором растет, — задача на будущую стоимость — или же выплаты снимаются со счета, сумма на котором убывает, — задача на текущую стоимость. Следует помнить, что в задачах на погашение долга всегда есть текущая стоимость простого аннуитета. Этапы 1 и 3 помогут выбрать правильные формулы для решения задач (см. рис. 3.19). Затем следует известные и неизвестные параметры и решить задачу. А = P(l+rt) А = Р(1 + О" (1 + ол -1 FV = РМТ Простой Единовременная^^ процент выплата С Сложный процент Последовательность выплат Будущая стоимость обычной ренты Текущая стоимость обычной ренты PV = PMT Рис. ЗЛ9. Выбор правильных формул для решения задач
266 Часть II. Конечная математика Ответы к упражнениям 3.16. 13 577,71 долл. 3.17. 10688,87 долл. 3.18. РМТ = 112,98 долл, в месяц; суммарные проценты = 311,52 долл. 3.19. Номер выплаты Сумма выплаты, долл. Проценты, долл. Оплата непогашенного остатка, долл. Непогашенный остаток, долл. 0 1000,00 1 174,03 12,50 161,53 838,47 2 174,03 10,48 163,55 674,92 3 174,03 8,44 165,59 509,33 4 174,03 6,37 167,66 341,67 5 174,03 4,27 169,76 171,91 6 174,06 2,15 171,91 0,00 Итого 1044,21 44,21 1000,00 3.20. 98551 долл. Практикум 3.4 Решить все задачи, используя формулу (3.14) или (3.15). А 1. п = 30; г = 0,04; РМТ = 200 долл.; PV = 1. 2. п = 40; i = 0,01; РМТ = 400 долл.; PV = ?. 3. п = 25; г = 0,025; РМТ = 250 долл.; PV = 1. 4. п = 60; г = 0,0075; РМТ = 500 долл.; PV = 1. Б 5. PV = 6000 долл.; п = 36; г = 0,01; РМТ = ?. 6. PV = 1200 долл.; п = 40; i = 0,025; РМТ = ?. 7. PV = 40000 долл.; п = 96; i = 0,0075; РМТ = 1. 8. PV = 14000 долл.; п = 72; г = 0,005; РМТ = ?. В 9. PV - 5000 долл.; i — 0,01; РМТ = 200 долл.; п = ?. 10. PV = 20 000 долл.; г = 0,0175; РМТ = 500 долл.; п = ?. «s 11. PV = 9000 долл.; РМТ = 600 долл.; п = 20; г = ?. (Окрутите ответ до трех десятичных знаков.) 12. PV = 12000 долл.; РМТ = 400 долл.; п — 40; г = ?. (Округлите ответ до трех десятичных знаков.)
Глава 3. Финансовая математика 267 Применение математики Экономика и бизнес 13. WWW Страховая компания American General предлагает 10-летний простой аннуитет с гарантированной ставкой на уровне 6,65 сложного процента, начисляемого ежегодно. Сколько нужно заплатить за один из таких аннуитетов, чтобы ежегодно получать выплаты в размере 5000 долл, в течение 10-летнего периода? 14. WWW Страховая компания American General предлагает также семилетний простой аннуитет с гарантированной ставкой на уровне 6,35 сложного процента, начисляемого ежегодно. Сколько нужно было бы заплатить за одну из таких рент, чтобы ежегодно получать выплаты в размере 10000 долл, в течение семилетнего периода? 15. WWW Компания E-Loan, онлайновая кредитная служба, недавно предложила 36- месячные кредиты на покупку автомобилей со ставкой на уровне 7,56 сложного процента, начисляемого ежемесячно, желающим с отличной оценкой кредитоспособности. Сколько должен занять человек у компании E-Loan, если он обладает отличной оценкой кредитоспособности и может себе позволить ежемесячные выплаты в размере 350 долл.? Сколько всего процентов выплатит этот человек по такому кредиту? 16. WWW Компания E-Loan предложила 36-месячные кредиты на покупку автомобилей со ставкой на уровне 9,84 сложного процента, начисляемого ежемесячно, желающим с хорошей оценкой кредитоспособности. Сколько должен занять человек у компании E-Loan> если он обладает хорошей оценкой кредитоспособности и может себе позволить ежемесячные выплаты в размере 350 долл.? Сколько всего процентов выплатит этот человек по такому кредиту? 17. WWW Человек купил компьютер непосредственно у производителя за 2500 долл, и договорился оплатить его 48 равными платежами при ставке 1,25% в месяц, начисляемой на непогашенный остаток. Каков размер его ежемесячных выплат? Сколько всего будет выплачено процентов? 18. WWW Человек купил компьютер непосредственно у производителя за 3500 долл, и договорился оплатить его 60 равными платежами при ставке 1,75% в месяц, начисляемой на непогашенный остаток. Каков размер его ежемесячных выплат? Сколько всего будет выплачено процентов? 19. WWW Парусная яхта стоит 35 000 долл. Человек заплатил первый взнос в размере 20% и погашал остаток равными ежемесячными выплатами в течение 12-летнего периода. Каков размер его ежемесячных выплат, если он должен был платить 8,75 сложного процента, начисляемого ежемесячно? Сколько всего процентов он заплатил? 20. WWW Дом на колесах стоит 80 000 долл. Человек заплатил первый взнос в размере 10% и Погашал остаток равными ежемесячными взносами в течение семилетнего периода. Каков размер его ежемесячных выплат, если он должен был платить 9,25 Сложного процента, начисляемого ежемесячно? Сколько всего процентов он заплатил?
268 Часть II. Конечная математика 21. Постройте график погашения долга в размере 5000 долл., который должен быть погашен восьмью равными ежеквартальными взносами при ставке 2,8% в квартал, начисляемой на непогашенный остаток. 22. Постройте график погашения долга в размере 10000 долл., который должен быть погашен шестью равными ежеквартальными взносами при ставке 2,6% в квартал, начисляемой на непогашенный остаток. 23. Женщина взяла в долг 6000 долл, под девять сложных процентов, начисляемых ежемесячно. Долг нужно вернуть в течение трех лет равными ежемесячными взносами. Для уплаты налогов ей нужно знать проценты, выплачиваемые в течение каждого года погашения кредита. Вычислите проценты, выплачиваемые в течение первого, второго и третьего года погашения кредита. {Подсказка. Найдите непогашенный остаток после 12, а затем после 24 выплат.) 24. Мужчина учредил ренту по выходу на пенсию, положив 50000 долл, на счет, приносящий 7,2 сложного процента, начисляемого ежемесячно. Равные ежемесячные изъятия будут осуществляться каждый месяц в течение пяти лет, после чего баланс у счета станет нулевым. Каждый год необходимо будет платить налог на проценты, которые принес счет в течение этого года. Сколько всего процентов будет получено в течение первого года? {Подсказка. Сумма на счету в конце первого года — это текущая стоимость четырехлетней ренты.) 25. Одни друзья рассказали другим, что они заплатили первый взнос в размере 25 000 долл, при покупке нового дома в кредит, и должны платить по 525 долл, в месяц в течение 30 лет. За сколько был продан дом, если ставка составила 7,8 сложного процента, начисляемого ежемесячно? Сколько процентов они выплатят за 30 лет? 26. Семья собирается купить новый дом стоимостью 120000 долл. Они должны заплатить первый взнос в размере 20%, а остальную часть оплатить в течение 30 лет равными ежемесячными взносами. Какими будут их ежемесячные взносы, если деньги им обходятся в 7,5 сложного процента, начисляемого ежемесячно? Сколько всего процентов будет выплачено в течение 30 лет? 27. Студент получил гарантируемый государством студенческий кредит в размере 6000 долл, при ставке 3,5 сложного процента, начисляемого ежемесячно. После окончания колледжа через два года студент должен вернуть кредит в течение следующих четырех лет, делая равные ежемесячные взносы. Каков размер этих взносов и сколько всего процентов заплатит студент? {Подсказка. Эта задача состоит из двух частей. Сначала нужно вычислить сумму долга по истечении первых двух лет, а затем погасить долг на эту сумму в течение следующих четырех лет.)
Глава 3. Финансовая математика 269 28. Человек учредил амортизационный пенсионный фонд, вкладывая по 7500 долл, в конце каждого года на протяжении 20 лет. В течение следующих 20 лет он будет ежегодно изымать равные суммы, после чего баланс счета станет равным нулю. Какие ежегодные выплаты будет получать этот человек в течение последних 20 лет, если На его вклад ежегодно начисляются девять сложных процентов? 29. У семьи есть 30-летняя закладная на сумму 75 000 долл, при ставке 8,1 сложного процента, начисляемого ежемесячно. Вычислите ежемесячные выплаты. Кроме того, найдите непогашенный остаток через указанный период времени. а) 10 лет. б) 20 лет. в) 25 лет. 30. У семьи есть 20-летняя закладная на сумму 50 000 долл, при ставке 7,2 сложного процента, начисляемого ежемесячно. Вычислите размер ежемесячных взносов. Кроме того, вычислите непогашенный остаток через указанный период времени, а) 5 лет. б) 10 лет. в) 15 лет. 31. У семьи есть 20-летняя закладная на сумму 80000 долл, при ставке на уровне восьми сложных процентов, начисляемых ежемесячно. а) Вычислите размер ежемесячных взносов и общую сумму выплаченных процентов. б) Предположим, Что семья решила каждый месяц добавлять к выплатам по закладной дополнительные 100 долл., начиная с самой первой выплаты. Сколько понадобятся Времени для того, чтобы семья расплатилась по закладной? Сколько процентов сбережет семья? 32. На момент выхода на пенсию у семейной пары был счет на 200000 долл., приносящий 8,4 сложного процента, начисляемого ежемесячно. а) Сколько ОНИ будут снимать каждый месяц, если решат делать это равными ежемесячными долями в течение 10 лет, после чего баланс на счету должен будет стать равным нулю? б) Сколько изъятий они смогут сделать, если решат снимать по 3000 долл, до тех пор, пока баланс на счету не станет равным нулю? 33. Текущий баланс обычной ренты, приносящей 7,5 сложного процента, начисляемого ежемесячно, равен 500 000 долл. Владелец счета, который вот-вот выйдет на пенсию, должен решить, какую сумму снимать со счета каждый месяц. Вычислите количество изъятий для каждого из следующих вариантов. а) 5000 долл, ежемесячно. б) 4000 долл, ежемесячно. в) 3000 долл, ежемесячно. 34. Вернемся к задаче 33. Сколько денег будет на счету спустя 10, 20 и 30 лет, если владелец счета решит снимать по 3000 долл, ежемесячно? 35. Простой аннуитет приносит 7,44 сложного процента, начисляемого ежемесячно, а) Человек ежемесячно Вкладывает по 100 долл, в течение 30 лет, а затем делает равные ежемесячные изъятия в течение следующих 15 лет, сокращая баланс ДО нуля. Какими будут ежемесячные изъятия? Сколько процентов будет получено в течение всего 45-летнего процесса?
270 Часть II. Конечная математика б) Сколько нужно ежемесячно вкладывать в течение первых 30 лет, если человек хочет делать ежемесячные изъятия в размере 2000 долл, в течение остальных 15 лет? 36. Простой аннутет приносит 6,48 сложного процента, начисляемого ежемесячно. а) Человек хочет ежемесячно класть деньги на счет в течение 15 лет, чтобы затем в течение следующих 20 лет снимать деньги со счета равными долями по 1500 долл, в месяц, сокращая баланс до нуля. Сколько нужно было бы вкладывать каждый месяц в течение первых 15 лет? Сколько всего процентов будет получено в течение всего 35-летнего процесса? б) Сколько можно было бы снимать ежемесячно в течение следующих 20 лет, если бы в течение первых 15 лет человек ежемесячно вкладывал по 1000 долл.? 37. Семейная пара хочет одолжить деньги, используя для залога собственный дом. Кредитная компания предлагает заем, достигающий 70% его стоимости. Дом куплен в кредит 12 лет назад за 79000 долл. Первый взнос был равен 20% стоимости, а остальная сумма была гарантирована 30-летней закладной при ставке 12%, начисляемой на непогашенный остаток. Семья делала ежемесячные выплаты, чтобы погасить кредит в течение 30-летнего периода. В данный момент чистая рыночная стоимость дома равна 100 000 долл. Сделав свою 144 выплат, семья обратилась в кредитную компанию за получением максимальной ссуды. Какую ссуду получит семья (с точностью до доллара)? 38. Человек купил дом 10 лет назад за 100 000 долл. Дом был куплен в кредит после оплаты первого взноса в размере 20% и подписания 30-летней закладной при ставке 9,6%, начисляемой на непогашенный остаток. Кредит погашался в течение 30-летнего периода равными ежемесячными выплатами. Теперь (сделав 120 выплат) владелец хочет рефинансировать дом, поскольку ему нужна дополнительная наличность. Сколько наличных денег (с точностью до доллара) получит владелец после погашения остатка по исходной закладной, если кредитная компания согласилась на новую 30-летнюю закладную на 80% от новой оценочной стоимости дома, которая составляет 136000 долл.? 39. Человек 10 лет назад купил дом за 120000 долл., заплатив первый взнос в размере 20% и подписав 30-летнюю закладную при ставке 10,2 сложного процента, начисляемого ежемесячно. Процентные ставки снизились, и владелец хочет рефинансировать непогашенный остаток, подписав новую 20-летнюю закладную при ставке 7,5 сложного процента, начисляемого ежемесячно. Сколько процентов сбережет рефинансирование? 40. Человек 20 лет назад купил дом за 200000 долл., заплатив первый взнос в размере 20% и подписав 30-летнюю закладную при ставке 13,2 сложного процента, начисляемого ежемесячно. Процентные ставки снизились, и владелец хочет рефинансировать непогашенный остаток, подписав новую 10-летнюю закладную при ставке 8,2 сложного процента, начисляемого ежемесячно. Сколько процентов сбережет рефинансирование? * 41. Проанализируйте сходство и различие между графиками непогашенного остатка, зависящего от времени, для 30-летних закладных на сумму 50 000, 75 000
Глава 3. Финансовая математика 271 и 100000 долл, соответственно, каждая из которых имеет ставку, равную девяти сложным процентам, начисляемым ежемесячно (см. рис. 3.20). Сделайте расчеты ежемесячных платежей и выплаченных суммарных процентов. Рис. 3.20. Рисунок к задаче 41 * 42. Проанализируйте сходство и различие между графиками непогашенного остатка, зависящими от времени, для 30-летних закладных на сумму 60 000 долл, при ставках 7, 10 и 13% соответственно (см. рис. 3.21). Сделайте расчеты ежемесячных платежей и выплаченных суммарных процентов. jcl В задачах 43 и 44, используя методы графической аппроксимации или программу для решения уравнений, вычислите приблизительное значение необходимой процентной ставки. Выразите каждый ответ в процентах с точностью до двух десятичных знаков. 43. Магазин уцененного электронного оборудования предлагает заплатить за стереосистему 1000 долл., Выплатив 12 равных частей по 90 долл. В магазине утверждают, Что, поскольку человек выплатит за год 1080 долл., стоимость кредита 80 долл, соответствует годовой ставке, равной 8%. Это было бы так, если бы человек оплатил кредит единовременной выплатой в конце года. Однако, поскольку он начинает возвращать деньги спустя месяц, то это — погашение
272 Часть II. Конечная математика долга, и, следовательно, 8% — неправильная ставка. Чему равная номинальная годовая сложная процентная ставка для этого кредита? (Можно ли ожидать, что процентная ставка будет выше? Кредиты такого типа называются кредитами с добавленными процентами, и они были очень распространены до того, как Конгресс США принял Закон о честном кредитовании (Truth in Lending Act). Теперь кредитные соглашения должны полностью раскрывать все издержки, связанные с кредитом, а процентные ставки должны выражаться через ставки, используемые в процессах погашения.) 44. Компьютер стоимостью 2000 долл, можно купить в кредит, выплачивая по 100 долл, в месяц в течение двух лет. Какова номинальная годовая сложная процентная ставка для этого кредита? 45. Владелец небольшого предприятия получил два предложения о покупке. Первый предполагаемый покупатель предлагает заплатить владельцу 100 000 долл, наличными сейчас. Второй предлагает заплатить владельцу 10000 долл, сейчас и ежемесячно выплачивать по 1200 долл, в течение 10 лет. В сущности, второй покупатель просит у владельца кредит на сумму 90 000 долл. Если владелец примет второе предложение, то какую номинальную годовую сложную процентную ставку он получит при финансировании такой покупки? 46. На момент выхода на пенсию у пары есть 200000 долл., вложенных в аннуитет. Они могут забрать всю сумму единовременно или получать ежемесячные выплаты в размере 2000 долл, в течение 15 лет. Какую номинальную годовую сложную процентную ставку они получат за деньги, вложенные в аннуитет, если выберут второй вариант? Ключевые слова, основные обозначения и формулы 3.1. Простые проценты. Капитал; проценты; процентная ставка; простые проценты; номинальная стоимость; текущая стоимость; будущая стоимость. I = Prt-, A = P(l + rt). 3.2. Сложные проценты. Сложные проценты; ставка на период начисления сложных процентов; капитал (текущая стоимость); сумма (будущая стоимость); номинальная годовая ставка; годовой процентный доход (или эффективная ставка); время роста; правило 72; облигация с нулевым купоном. 7* / V \ 771 А = Р(1 + г)п; i = —; АРУ=(1 + —) -1. т \ т/ 3.3. Будущая стоимость аннуитета; амортизационные фонды. Аннуитет; простой аннуитет; будущая стоимость; бухгалтерский баланс; амортизационные фонды; взнос в амортизационный фонд. (1 _|_ г)7* — 1 Будущая стоимость: FV = РМТ- ; г Амортизационный фонд: РМТ = FV — .
Глава 3. Финансовая математика 273 3.4. Текущая стоимость аннуитета; погашение. Текущая стоимость; погашение долга; график погашения; собственный капитал; стратегия решения задач. 1 _ (1 + Л-п Текущая стоимость: PV = РМТ ——-— г Погашение: РМТ = PV -—. 1 - (1 + г)~п Упражнения для повторения Выполните все упражнения этого обзорного раздела и сравните результаты с ответами, помещенными в конце книги. Ответы ко многим упражнениям на повторение приводятся вместе с номером соответствующего раздела (курсивом). Если у вас возникают затруднения при решении какой-либо задачи, повторите материал соответствующего раздела, А В задачах 1-4 вычислите указанные величины по формуле А = Р(1 + rt). 1. А = ?; Р = 100 ДОЛЛ.; г — 9%; t = 6 месяцев. 2. А = 808 долл.; Р * ?; г 12%; t = 1 месяц. 3. А = 212 долл.; Р 200 долл.; г = 8%; t = ?. 4. А = 4120 долл.; Р » 4000 долл.; г = ?; t = 6 месяцев. Б В задачах 5 и б вычислите указанные величины по формуле А = Р(1 + i)n. 5. А = 7\Р = 1200 долл.; i = 0,005; п = 30. 6. А = 5000 долл.; Р ?; i = 0,0075; п = 60. В задачах 7 и 8 вычислите указанные величины по формуле FV = РМТ^+^~Г. 7. FV = ?; РМТ = 1000 долл.; г = 0,005; п = 60. 8. FV = 8000 долл.; РМТ ^7;i = 0,015; п = 48. В задачах 9 и 10 вычислите указанные величины по формуле PV = РМТг~^^—. 9. PV = Т, РМТ = 2500 долл.; i = 0,02; п = 16. 10. PV = 8000 долл.; РМТ ^ ?; г = 0,0075; п = 60. В 11. Решите уравнение 2500 1000 • 1,06™ относительно величины п с точностью до ближайшего целого: а) с помощью логарифма; б) с помощью методов графической аппроксимации или графической утилиты. 12. Решите уравнение 5000 » 100 • 1?q101~1 относительно величины п с точностью до ближайшего целого: а) с помощью логарифма; б) с помощью методов графической аппроксимации или графической утилиты.
274 Применение математики Часть II. Конечная математика Экономика и бизнес Все суммы, выраженные в долларах, нужно вычислить с точностью до цента. Если в ответе требуется вычислить процентную ставку, ее нужно выражать в процентах с точностью до двух десятичных знаков. 13. Сколько будет должен человек через 10 месяцев, если он одолжил на этот срок 3000 долл, под 14 простых процентов? Сколько процентов он заплатит? 14. Дедушка с бабушкой положили на счет внука 6000 долл, для оплаты его обучения в колледже. Сколько денег (с точностью до ближайшего доллара) будет на счету через 17 лет, если счет приносит семь сложных процентов, начисляемых ежемесячно? 15. Сколько пришлось бы заплатить за промышленную облигацию, приносящую 6,6 сложного процента, начисляемого ежемесячно, чтобы через 10 лет получить 25 000 долл.? 16. Сберегательный счет приносит 5,4 сложного процента, начисляемого ежегодно. Составьте бухгалтерский баланс, показывающий проценты, полученные в течение каждого года, а также баланс в конце каждого года в течение четырех лет при следующих условиях. а) В начале первого года был сделан единовременный вклад в размере 400 долл. б) Было сделано четыре вклада по 100 долл, в конце каждого года. 17.* Одна инвестиция приносит 13 простых процентов, а другая — девять сложных процентов, начисляемых ежегодно. Какую инвестицию следует выбрать? Почему? 18. Пенсионный счет, содержащий 10 000 долл., должен приносить проценты при ставке, равной семи сложным процентам, начисляемым ежедневно. Сколько денег будет на счету через 40 лет, когда владелец в 65 лет выйдет на пенсию? (Следует использовать 365-дневный год и округлить ответ до доллара.) 19. Какая инвестиция лучше и почему: 9 сложных процентов, начисляемых ежеквартально, или 9,25 сложного процента, начисляемого ежегодно? 20. Вычислите стоимость обычного аннуитета по истечении восьми лет, если на счет, приносящий 7,2 сложного процента, начисляемого ежемесячно, каждый месяц вносилось по 200 долл. Сколько из этой стоимости составляют проценты? 21. Компания, продающая товары по кредитным картам, взимает с просроченных счетов годовую ставку в размере 22%. Сколько процентов долга будет начислено на счет в размере 635 долл., просроченный на один месяц? 22. Вычислите стоимость автомобиля ценой 8000 долл, (с точностью до доллара) спустя пять лет, если уровень инфляции за этот период в среднем составит пять сложных процентов, начисляемых ежегодно. 23. Какова бы была стоимость автомобиля ценой 8000 долл, в задаче 22 (с точностью до доллара) пять лет назад, если бы уровень инфляции за этот период в среднем составил пять сложных процентов в год?
Глава 3. Финансовая математика 275 24. Кредит на сумму 2500 долл, был оплачен в конце 10-го месяца чеком на сумму 2812,50 долл. Каков был размер взимаемой годовой процентной ставки? 25. Продавец автомобилей говорит человеку, что автомобиль, на который он смотрит, можно купить в кредит, заплатив первый взнос в размере 3000 долл, и выплачивая по 200 долл, в месяц в течение 48 месяцев. По какой цене продавался автомобиль и сколько процентов должен заплатить человек в течение этих 48 месяцев, если процентная ставка составляет 14 сложных процентов, начисляемых ежемесячно? 26. Для покупки лодки, которая стоит 3000 долл., у человека есть 2500 долл. Сколько времени понадобится для того, чтобы сумма в 2500 долл, выросла до 3000 долл., если вложить эти деньги под девять сложных процентов, начисляемых ежеквартально? (ПрИ необходимости ответ следует округлить до следующего большего квартала.) 27. Сколько времени понадобится на то, чтобы деньги удвоились, если они вложены под шесть или девять сложных процентов, начисляемых ежемесячно? (При необходимости ответ следует округлить до следующего большего месяца.) 28. Свой первый взнос на пенсионный счет Джон сделал, когда ему исполнилось 21 год. Пока ему не исполнилось 65 лет, Джон каждый год в день своего рождения клал на пенсионный счет 2000 долл. Какая сумма (с точностью до доллара) будет лежать на счету, когда Джону исполнится 65 лет, если счет приносит следующие проценты? а) 7 сложных процентов, начисляемых ежегодно. б) 11 сложных процентов, начисляемых ежегодно. 29. Человек только что продал акции за 17388,17 долл, (чистыми), которые обошлись ему три года назад в 12 903,28 долл, (чистыми). Какую годовую сложную норму прибыли смог получить человек по этой инвестиции? 30. WWW В таблице показана плата за ссуды под ожидаемый возврат налогов (RAL), предлагаемые онлайновой компанией, занимающейся вопросами возврата налогов. Вычислите годовую процентную ставку для каждого из следующих кредитов. а) Ожидаемый возврат налогов на сумму 400 долл, произойдет через 15 дней. б) Ожидаемый возврат налогов на сумму 1800 долл, произойдет через 21 день. Сумма RAL, долл. Комиссионные за RAL, долл. 100-500 29,00 501-1000 39,00 1001-1500 49,00 1501-2000 69,00 2001-5000 82,00 31. WWW Недавно страховая компания Lincoln Benefit Life предложила аннуитет, приносящий 5,5 сложного процента, начисляемого ежемесячно. Какие равные ежемесячные взносы нужно делать, чтобы через 5 лет иметь 50 000 долл.?
Часть II. Конечная математика 276 32. Человек хочет учредить аннуитет, предназначенный для выхода на пенсию. Он будет делать ежеквартальные вклады на протяжении 20 лет, чтобы затем иметь возможность ежеквартально снимать по 5000 долл, в течение 10 лет. Аннуитет приносит 7,32 сложного процента, начисляемого ежеквартально. а) Сколько денег должно быть на счету на момент его выхода на пенсию? б) Сколько денег нужно вкладывать каждый квартал, чтобы в течение 20 лет накопить требуемую сумму? в) Сколько всего процентов будет получено в течение этого 30-летнего периода? 33. WWW Человек одолжил у онлайновой кредитной организации 4000 долл, для покупки компьютера и договорился выплатить эту сумму, сделав 48 равных взносов при ставке 0,9 процента в месяц, начисляемой на непогашенный остаток. Сколько он должен будет платить в месяц? Сколько всего процентов он заплатит? 34. В компании решили учредить амортизационный фонд для замены через шесть лет части оборудования оценочной стоимостью 50000 долл. Чтобы сделать это, они решили делать фиксированные ежемесячные взносы на счет, приносящий 6,12 сложного процента, начисляемого ежемесячно. Каким должен быть каждый взнос? 35. Сколько времени понадобится для того, чтобы сумма удвоилась, если деньги были вложены под 7,5 сложного процента, начисляемого ежедневно или под 7.5 сложного процента, начисляемого ежегодно? 36. Студентка получила студенческий кредит на сумму 8000 долл, под 5,5 сложного процента, начисляемого ежемесячно, который поможет ей закончить последние 1.5 года обучения в колледже. Начиная с первого года после окончания колледжа, студентка должна возвращать кредит в течение следующих пяти лет, делая равные ежемесячные выплаты. Какими будут эти выплаты и сколько всего процентов заплатит студентка? 37. Компания каждый месяц делает взносы в размере 1200 долл, в амортизационный фонд, приносящий шесть сложных процентов, начисляемых ежемесячно. Используя графическую утилиту, определите, когда величина этого фонда достигнет 100000 долл. М 38. У семейной пары есть 20-летняя закладная на сумму 50000 долл, при ставке, равной девяти сложным процентам, начисляемым ежемесячно. Используя графическую утилиту, определите, когда непогашенный остаток упадет ниже 10000 долл. 39. Кредитная компания дала рекламу в газете о том, что за каждые одолженные у нее 100 долл, нужно будет платить всего лишь восемь центов в день. Какую годовую процентную ставку они взимают? (Воспользуйтесь 360-дневным годом.) 40. Постройте график погашения для долга в размере 1000 долл., который должен быть погашен четырьмя равными ежеквартальными платежами при ставке 2.5 процента в квартал, начисляемой на непогашенный остаток. 41. Человек может позволить себе ежемесячно класть только 200 долл, на счет, приносящий 7,98 сложного процента, начисляемого ежемесячно. Сколько времени
Глава 3. Финансовая математика 277 пройдет, прежде чем у него появятся 2500 долл, для покупки автомобиля? (При необходимости ответ следует округлить до следующего большего месяца.) 42. Компания учредила амортизационный фонд для переоборудования фабрики через шесть лет, что, по оценкам, обойдется в 850000 долл. Сколько нужно раз в полгода класть йа счет, приносящий 8,76 сложного процента, начисляемого раз в полгода? Сколько процентов принесет счет спустя шесть лет? 43. Какова номинальная годовая сложная процентная ставка, начисляемая ежемесячно, у депозитного сертификата, приносящего годовой процентный доход в размере 6,48%? 44. Какую годовую процентную ставку получит человек, если он купил у министерства финансов США 13-недельный казначейский вексель с выплачиваемой суммой 5000 долл, за 4922,15 долл.? 45. Чтобы собрать сумму, необходимую для первого взноса по кондоминиуму, молодая пара кладет каждый месяц по 200 долл, на счет, приносящий 7,02 сложного процента, начисляемого ежемесячно. Сколько они должны будут сделать вкладов, если для первого взноса необходимо 10000 долл.? 46. Фирма взяла в кредит на восемь лет 80 000 долл, под 9,42 сложного процента, начисляемого ежемесячно. а) Каковы ее ежемесячные выплаты? б) Каким будет непогашенный остаток в конце первого года? в) Сколько процентов было выплачено в течение первого года? * 47. Человек неожиданно унаследовал 10 000 долл, как раз после того, как он сделал 72 ежемесячных выплат по 30-летней закладной на сумму 60 000 долл, при ставке 8,2 сложного процента, начисляемого ежемесячно. Проанализируйте относительные выгоды при использовании наследства либо для сокращения основной суммы кредита, Либо для покупки депозитного сертификата, приносящего семь сложных процентов, начисляемых ежемесячно. * 48. Семейная пара размышляет над 30-летней закладной на сумму 75 000 долл, для покупки нового Дома. Банк, с которым они ведут дела уже много лет, предлагает ставку в размере 7,54 сложного процента, начисляемого ежемесячно. Конкуренты предлагают ставку 6,87 сложного процента, начисляемого ежемесячно. Стоит ли этой семейной паре сменить банк? 49. За какую сумму должна быть продана облигация с нулевым купоном и номинальной стоимостью 5000 долл., срок платежа по которой наступит через пять лет, если ее доходность составляет 5,6 сложного процента, начисляемого ежегодно? 50. Каков будет размер годовой сложной нормы прибыли, если заплатить 5695 долл, за облигацию с нулевым купоном и номинальной стоимостью 10 000 долл., срок платежа по которой наступит через 10 лет? 51. По какой цене инвестор должен купить 26-недельный казначейский вексель с номинальной стоимостью 5000 долл., если он хочет получить годовую процентную ставку 6,4%?
278 Часть II. Конечная математика 52. Два года назад человек взял взаймы 10 000 долл, под 12 сложных процентов, начисляемых ежемесячно, которые он должен вернуть в течение пяти лет. Сейчас он получил кое-какие дополнительные средства и решил, что хочет расплатиться по этому кредиту. Каким будет непогашенный остаток после того, как он в течение двух лет делал равные ежемесячные выплаты? 53. Какая номинальная годовая процентная ставка даст такой же годовой процентный доход, что и 7,28 сложного процента, начисляемого ежеквартально? 54. Решите следующие задачи. а) Когда человеку исполнилось 21 год, он положил 2000 долл, на свой пенсионный счет. Он продолжал делать взносы каждый год в свой день рождения вплоть до 29 лет включительно (всего девять взносов). Счет приносит восемь сложных процентов, начисляемых ежегодно. Сколько денег будет у него, когда ему исполнится 65 лет, если он оставит деньги на счету, не делая больше вкладов, предполагая, что счет будет продолжать приносить ту же процентную ставку? б) Сколько денег было бы на счету (с точностью до доллара), когда вкладчику исполнилось бы 65 лет, если бы он начал делать вклады в 30 лет и продолжал бы делать их на каждый день своего рождения вплоть до 65 лет (включительно)? 55. В новом поселке дома продаются по 100000 долл. Первый взнос равен 20%. У покупателя есть выбор: 30- или 15-летний кредит, оба со ставкой 7,68 сложного процента, начисляемого ежемесячно. а) Каков будет размер ежемесячных выплат, если клиент возьмет 30- или 15- летний кредит? б) Вычислите непогашенный остаток спустя 10 лет, если клиент возьмет 30- или 15-летний кредит. 56. Кредитная компания дает ссуды на сумму до 60% от стоимости дома. Семья купила свой дом восемь лет назад за 83 000 долл. Дом был куплен в кредит после выплаты первого взноса в размере 20% и подписания 30-летней закладной при ставке 8,4%, начисляемой на остаток. Для погашения кредита за 30-летний период семья делала равные ежемесячные выплаты. Рыночная стоимость дома сейчас составляет 95 000 долл. Сделав 96 выплат, семья обратилась в кредитную компанию для получения максимального кредита. Сколько они получат (с точностью до доллара)? 57. Стереосистема стоимостью 600 долл, была куплена в кредит на шесть месяцев при ежемесячных выплатах, равных 110 долл. Какова номинальная годовая сложная процентная ставка у этого кредита? 58. Женщина каждый год в течение 25 лет кладет по 2000 долл, на пенсионный счет. Когда она выйдет на пенсию, сделав 25 вкладов, на счету будет 220 000 долл. а) Вычислите процентную ставку, которую приносит пенсионный счет в течение 25-летнего периода подготовки к выходу на пенсию. б) Предположим, что пенсионный счет продолжает приносить процентную ставку, вычисленную при решении задачи 1. Как долго пенсионерка сможет сни-
Глава 3. Финансовая математика 279 мать со счета по 30000 долл, в год? Как долго она сможет снимать по 24 000 долл, в год? Домашнее задание 3.1. Снижение выплачиваемых процентов по закладной на дом В благодарность за финансовую поддержку со стороны родителей при получении образования в колледже недавняя выпускница собралась сэкономить им немного денег при выплате кредита за дом, Ее родители сделали 180 выплат по 30-летнему кредиту на сумму 80000 долл, при ставке 11%. 1. Сколько было бы сэкономлено на выплатах процентов, если бы родители смогли найти кредитора, пожелавшего бы рефинансировать непогашенный остаток в течение 15 лет при ставке 9 или 8%? 2. При какой процентной ставке нужно было бы провести рефинансирование, чтобы снизить суммарную выплату процентов на 10000 долл.? 3. Вместо рефинансирования выпускница решила вкладывать по 100 долл, каждый месяц, чтобы снижать основную сумму до тех пор, пока кредит не будет выплачен. Сколько сбережет выпускница своим родителям при выплате процентов? Сколько ей придется вложить? 4. Вернемся к задаче 3. Сколько пришлось бы выпускнице вкладывать каждый месяц, чтобы сберечь своим родителям 10000 долл, при выплате процентов? Домашнее задание 3.2. Итоговая доходность и внутренняя норма прибыли Один продавец в течение четырех лет получал годовую премию (табл. 3.4), вкладывая ее во взаимный фонд. Проценты, которые приносил этот фонд, в разные года были разными. Таблица 3.4. Годовые премии Год Премия, долл. “1 4000 2 7000 3 4000 4 8000 Через год после последнего взноса продавец закрыл фонд и получил 30000 долл. Ему бы хотелось вычислить сложную процентную ставку, соответствующую процентам, которые принесли его инвестиции в течение всего четырехлетнего периода. Эта ставка называется итоговой Доходностью. Поскольку продавец каждый год вкладывал разные суммы, то в этой ситуации Нельзя использовать ни одну из формул для расчета ренты. Однако можно воспользоваться формулой для вычисления сложных процентов, рассмотренной в разделе 3.2. Если г — итоговая доходность, то формула для вычисления сложных процентов выражает через г стоимость каждой инвестиции в конце 4 года (рис. 3.22).
280 Часть II. Конечная математика Годы 1 2 3 4 5 Вклады 3000 7000 1 4000 8000 L 8000(1 +г) ► 4000(1 +г)2 — ► 7000(1 +г)3 — 3000(1 + г)4 Рис. 3.22. Расчет доходности Сумма всех этих величин должна быть равна 30 000, т.е. сумме, находящейся в фонде в конце четвертого года. Таким образом, доходность на момент погашения должна удовлетворять следующему уравнению. 8000 • (1 + г) + 4000 • (1 + г)2 + 7000 • (1 + г)3 + 3000 • (1 + г)4 = 30 000. Используя методы графической аппроксимации для решения этого уравнения (рис. 3.23), находим, что итоговая доходность (с точностью до двух десятичных знаков) равна г = 14,39%. 50 000 0 InUrStChOn N=.14390939 -7=30000 Рис. 3.23. Расчет итоговой доходности с помощью графической утилиты 1. В предыдущих рассуждениях для нахождения итоговой доходности использовались методы графической аппроксимации. Если у читателя есть доступ к программе для решения уравнений, то он должен воспользоваться ею для вычисления конечной отдачи. 2. Предположим, что на 16-летие своего внука бабушка с дедушкой дали ему 1000 долл, в виде свидетельства на акцию, на 17-летие — 1500 долл., а на 18-летие — 2000 долл. На свое 21-летие внук продал все акции за 7000 долл. Какова итоговая доходность этой последовательности инвестиций? 3. Единовременная инвестиция в размере 10000 долл, в малый бизнес принесла 3000 долл, в конце первого года, 4000 долл, в конце второго года и 5000 долл, в конце третьего года. Если эта инвестиция приносила прибыль 100г%, то сумма текущих стоимостей этих ежегодных выплат должна быть равна исходной инве-
Глава 3. Финансовая математика 281 стиции. Покажите, что это приводит к уравнению 3000 4000 5000 10000 = —— + + 1 + г (1 + г)2 (1 + г)3 Решение этого уравнения называется внутренней нормой прибыли. Найдите решение, используя методы графической аппроксимации или программу для решения уравнений. 4. Человек заплатил 160 000 долл, за дом на две семьи, что приносит ему ежегодную прибыль в размере 15 000 долл. Спустя четыре года эта собственность была продана за 190000 долл. Вычислите внутреннюю норму прибыли для этой инвестиции
4 Системы линейных уравнений и матрицы ■ 4.1. Системы линейных уравнений с двумя переменными ■ 4.2. Системы линейных уравнений и расширенные матрицы ■ 4.3. Метод исключения Гаусса-Жордана ■ 4.4. Матрицы: основные операции ■ 4.5. Нахождение обратной квадратной матрицы ■ 4.6. Матричные уравнения и системы линейных уравнений ■ 4.7. Анализ межотраслевых связей по Леонтьеву ■ Основные термины и обозначения ■ Упражнения для повторения ■ Домашнее задание 4.1. Применение матриц для вычисления расходов, доходов и прибыли ■ Домашнее задание 4.2. Переменные и косвенные производственные издержки Введение В этой главе рассматриваются элементарные методы решения систем уравнений с двумя независимыми переменными (основные свойства прямых и линейных уравнений приведены в разделе 1.3 и в приложении А.8, т. 2). Для решения систем уравнений с большим количеством переменных методы, приведенные разделе 1.3, менее удобны, поэтому для этой цели используется метод расширенной матрицы, который совершенно естественно возникает в таких задачах. Далее изучаются свойства и основные операции с такими новыми математическими объектами как матрицы. Изучив эти операции, можно рассматривать системы уравнений с другой точки зрения. В заключение, обсуждаются работы
Глава 4. Системы линейных уравнений и матрицы 283 лауреата Нобелевской премии Василия Леонтьева по применению матричных методов к важным задачам экономики. lex Многие методы работы с матрицами, приведенные в этой главе, реализованы в различных программах, в том числе в графических калькуляторах, в программах Matlab и Explorations in Finite Mathematics. Большинство задач можно решать с помощью как приближенных графических методов, так и матричных. Однако существуют задачи, которые можно решить только с помощью одного из методов. Во избежание путаницы метод решения задачи будет Всегда указываться явно. Разумеется, методы решения с помощью графической утилиты, как и раньше, не являются обязательными (такие задачи отмечены пиктограммой, как и в остальных главах этой книги). 4.1. Системы линейных уравнений с двумя переменными ■ Системы уравнений с двумя переменными ■ Решение системы уравнений с помощью графика ■ Решение системы уравнений методом подстановки ■ Решение системы уравнений методом исключения ■ Решение практических задач Системы уравнений с двумя переменными Рассмотрим основные подходы на следующем простом примере. Пусть два “взрослых” билета в кино и один “детский” вместе стоят восемь долларов, а один “взрослый” билет и три “детских” стоят девять долларов. Вопрос: какова цена каждого билета? Дано: х — стоимость взрослого билета, у — стоимость детского билета. Легко видеть, что эти величины связаны следующими соотношениями. 2х + у = 8, х + Зу = 9. Мы получили систему уравнений относительно двух независимых переменных. Достаточно просто найти упорядоченную пару (ж, у), которая удовлетворяет каждому уравнению по отдельности. Так, например, упорядоченная пара (4, 0) удовлетворяет первому уравнению, однако не удовлетворяет второму. Упорядоченная пара (6, 1) удовлетворяет второму уравнению, но не является решением первого. Чтобы решить систему уравнений, нужно найти все упорядоченные пары, которые удовлетворяют обоим уравнениям одновременно. В общем случае решение системы уравнений определяется следующим образом. Система двух уравнений с двумя переменными Задана система линейных уравнений ах + by = h, ex + dy = fc,
284 Часть II. Конечная математика где а, Ь, с, d, Н,ик — действительные константы. Пара чисел х = xq и у = у0 (ее также можно записать как упорядоченную пару (хо, уо)) является решением этой системы уравнений, если она удовлетворяет каждому уравнению по отдельности. Множество всех таких упорядоченных пар называется множеством решений системы уравнений. Решить систему — значит найти множество решений. Ниже будут рассмотрены три метода решения систем уравнений: графический, метод подстановки и метод исключения. Каждый из методов по-своему хорош — выбор зависит от конкретной ситуации. Решение системы уравнений с помощью графика Напомним, что график уравнения прямой состоит из всех упорядоченных пар чисел, которые удовлетворяют уравнению прямой. Для того чтобы решить задачу с билетами, построим графики обеих прямых на одном и том же рисунке. Координаты точек, которые принадлежат обоим графикам, являются решением системы уравнений, так как удовлетворяют обоим уравнениям одновременно. Пример 4.1 (Решение системы уравнений с помощью графика). Решите задачу о билетах с помощью графика. 2х + у = 8, х + Зт/ = 9. Решение. Построим графики обоих уравнений на одном рисунке (оба графика — прямые линии). Затем приближенно найдем точку пересечения прямых. Проверка. 2х + у = 8 2 • 3 + 2 = 8 х + Зу = 9 3 + 32 = 9 Пара (3, 2) удовлетворяет обоим уравнениям одновременно 8 = 8 9 = 9
Глава 4. Системы линейных уравнений и матрицы 285 Упражнение 4.1. Найдите решение следующей системы уравнений и проверьте его аналитически. 2х — у = —3, х + 2у = -4. Из примера 4.1 видно, что система имеет одно решение, если линии (графики уравнений) пересекаются а одной точке. В общем случае взаимное расположение прямых в декартовой системе Координат может описываться тремя случаями, как показано в следующем примере. Пример 4.2 (Решение системы уравнений с помощью графика). Найдите решение каждой из следующих систем с помощью графиков. 1. х — 2у = —2; х + у = 5. Решение. 1. 2» х + 2у = -4; 2х + 41/ = 8. 3. 2а? + 41/= 8; х + 2у = 4. Линии пересекаются — система имеет единственное решение Линии параллельны — система не имеет ни одного решения 3. Линии совпадают — система имеет бесконечно много решений Упражнение 4.2. Решите графически следующие системы уравнений: 1. х + у = 4; 2. 6х — Зу = 9; 2х — у = 2. 2х — у = 3. 3. 2х — у = 4; 6х — Зу = —18. Теперь определим некоторые термины, которые будут использоваться для описания различных решений систем уравнений. Системы линейных уравнений: основные термины Система уравнений является совместной, если она имеет одно или несколько решений, и несовместной, если она не имеет ни одного решения. Совместная система уравнений называется независимой, если она имеет только одно решение (которое называется единственным), и зависимой, если она имеет несколько решений. В примере 4.2 первая система уравнений является совместной и независимой. Ее решение: х = 4 и у = L Вторая система уравнений является несовместной. Третья система — совместна И зависима, поскольку допускает бесконечно большое количество решений (точки совпадающих прямых).
286 Часть II. Конечная математика Задание 4.1. Пусть система уравнений имеет два решения. Какая она — совместная или зависимая? Аргументируйте ответ. а Рассматривая систему двух уравнений с двумя неизвестными с помощью геометрических методов, мы получили основные подходы, которые применяются к решению таких систем. Две произвольные прямые могут пересекаться в одной точке, быть параллельными или совпадать. Системы, приведенные в примере 4.2, иллюстрируют три возможности существования решений системы двух уравнений в двух переменных. Теорема 4.1 (Возможные решения системы линейных уравнений). Система линейных уравнений ах + by = Л, ex + dy = к может обладать следующими свойствами. 1. Иметь одно решение. Совместная и независимая 2. Не иметь решений. Несовместная 3. Иметь бесконечно много решений. Совместная и зависимая Этими вариантами исчерпываются все возможные случаи. ■ Раньше графики строили вручную, с большой неточностью. Ситуация поменялась с появлением графических утилит. С их помощью решения линейных систем с двумя переменными можно наглядно изображать на рисунке и вычислять с достаточно высокой точностью. Такого рода решение приведено в примере 4.3. кх Пример 4.3 (Решение системы уравнений с помощью графической утилиты). Решите следующую систему с точностью до двух десятичных знаков с помощью графической утилиты. Ьх + 2у = 15, 2х — Зу = 16. Решение. Сначала решим каждое уравнение относительно переменной у: Ьх + 2у = 15, 2х — Зу = 16, 2у = 15 — 5х, —Зу — —2х + 16, 2 16 у = -2,5 4- 7,5. у=-х-—. О о Далее введем уравнения в графическую утилиту (рис. 4.1, а), построим графики в окне вывода и приблизительно определим точку пересечения (рис. 4.1, б). Округляя значения, полученные с помощью графической утилиты до двух десятичных знаков, найдем, что решением системы является х = 4,05 и у = —2,63, или (4,05, -2,63). Проверка. 5х + 2у = 15; 2х — Зу = 16; 5 • 4,05 + 2 • (-2,63) = 15; 2 • 4,05 - 3 • (-2,63) = 16; 14,99 » 15. 15,99 «16.
Глава 4. Системы линейных уравнений и матрицы 287 Y1B-2.5X+7.5 УгВ<2>3>Х“16/3 Ys = Yh = Ys = Ys= ?7 = Yb= ... б) точка пересечения а) формулы уравнений Рис. 44. Решение системы уравнений с помощью графической утилиты Контрольная подстановка в уравнение полученного решения не дает точного совпадения, так как решение получено приближенно. ■ ю. Упражнение 4.3. Найдите решение следующей системы уравнений с точностью до двух десятичных знаков с помощью графической утилиты. 2х — Ьу = —25, 4х + Зу = 5. ■ С помощью графических методов можно представить систему уравнений наглядно (на рисунке). Довольно часто особенности системы легко выявить именно этим способом. Кроме того, с помощью построения графиков можно приближенно получить ее решение с достаточно хорошей точностью. Решение системы уравнений методом подстановки Теперь перейдем к рассмотрению простого алгебраического метода, с помощью которого можно получать Точные решения системы двух линейных уравнений (если таковые существуют). В этом методе одну из переменных выражают относительно другой, используя одно из уравнений системы (уравнение выбирают так, чтобы избежать неудобных выражений с дробями). Далее результат вычисления подставляют в оставшееся уравнение и решают получившееся линейное уравнение с одной переменной. Наконец, подставляя найденное значение обратно 0 выражение, полученное на первом шаге, вычисляют значение второй переменной. Эта процедура станет полностью понятной после разбора следующего примера. Пример 4.4 (Решение системы уравнений с помощью подстановки). Найдите решение следующей системы с помощью метода подстановки. 5х + у — 4, 2х — Зу = 5.
288 Часть II. Конечная математика Решение. Будем выражать одну переменную через другую в одном из уравнений. Чтобы избежать выражений с дробями, возьмем первое уравнение и выразим переменную у через х. Зх + у = 4, у = 4 — 5ж, 2х — Зу = 5, 2х - 3 • (4 - 5х) = 5, 2х — 12 4- 15х = 5, 17z = 17, Выразим переменную у через а? из первого уравнения Подставим результат во второе уравнение Второе уравнение Решаем относительно ® х = 1 . Чтобы вычислить значение переменной у, заменим переменную х в выражении у = = 4 — 5х единицей: у = 4 — Зх, т/= 4-5-1, У = -1 Таким образом, решением системы являются числа ж = 1ит/ = —1, или точка (1, —1). Проверка. 5х + т/ = 4, 2х — Зу = 5, 5-1 + (-1) = 4, 2 -1 — 3 • (—1) = 5, 4 = 4. —5 = 5. Упражнение 4.4. Найдите решение следующей системы с помощью метода подстановки. Зх + 2у = —2, 2х — у = —6. Задание 4.2. Вернитесь к примеру 4.2 и решите систему методом подстановки. На основе полученных результатов объясните, как с помощью метода подстановки определить, является система зависимой или несовместной. ■ Решение системы уравнений методом исключения Рассмотренные выше графический метод и метод подстановки одинаково хороши для поиска решения системы двух уравнений. Однако для систем из большего числа уравнений они оказываются менее эффективными. Перейдем к методу исключения. Это наиболее универсальный метод решения систем линейных уравнений. Его легко
Глава 4. Системы линейных уравнений и матрицы 289 распространить на большие системы уравнений, и, кроме того, на его основе удобно создавать компьютерные методы решения. Мы уже рассматривали операции над линейным уравнением, которые не изменяют множества его решений (см, приложение А.8). В методе исключения систему уравнений преобразовывают К эквивалентной системе с тем же множеством решений. Системы уравнений называются эквивалентными, если они имеют одинаковые множества решений. В теореме 4.2 приведены операции, с помощью которых получаются эквивалентные системы уравнений: Теорема 4,2 (Операции, приводящие к получению эквивалентных систем уравнений). Система уравнений переходит в эквивалентную систему при следующих преобразованиях. 1. Изменение порядка записи уравнений. 2. Умножение уравнения на константу, не равную нулю. 3. Умножение уравнения на константу и сложение с другим уравнением системы. ■ Применение всех трех операций, перечисленных в теореме 4.2, приводит к эквивалентной системе. Сейчас нам в основном понадобятся операции пп. 1 и 2. Операция п. 1 используется, как Правило, при работе с большими системами. Рассмотрим примеры, иллюстрирующие применение теоремы 4.2. Пример 4.5 (Решение системы уравнений методом исключения). Найдите решение следующей системы уравнений методом исключения. Зх - 2у = 8, 2х + by = —1. Решение. Используем теорему 4.2 для исключения одной из переменных и получения системы уравнений с очевидным решением. Зх — 2у = 8, 2х + by = —1. Умножим первое уравнение на число 5, а второе уравнение — на число 2 (теорема 4.2, п. 2) ; 5 • (Зх - 2у) = 5 • 8, ; ' 2 • (2х + 5у) = 2 • (—1).; 15х — 101/ = 40, 4х + Ют/ = -2. Прибавим первое уравнение ко второму, а затем исключим переменную у (теорема 4.2, п. 3) 19х = 38, Разделим обе части получившегося уравнения на число 19 — это эквивалентно умножению обеих частей на (теорема 4.2, п. 2) х = 2 . Это уравнение вместе с каким-либо уравнением исходной системы образует эквивалентную систему
290 Часть II. Конечная математика Зная, что х = 2, подставим это значение в какое-либо уравнение исходной системы (выберем второе) и решим получившееся уравнение относительно переменной у. 2-2 + 5i/ = -1, 51/ = -5, У = -1 Таким образом, решением системы являются числа х = 2 и у = -1, или точка (2, -1). Проверка. Зх — 2у = 8, 2х + Зу = —1, 3 • 2 - 2 • (-1) = 8, 2 • 2 + 5 • (-1) = -1, 8 = 8. -1 = -1. ■ Упражнение 4.5. Найдите решение следующей системы уравнений методом исключения. Зх — 2у = 12, 2х + Зу = 1. в Определим, к чему приводит метод исключения в случаях, если система уравнений не имеет решения или допускает бесконечно много решений. Рассмотрим следующую систему. 2х + Зу = —3, х + Зу = 2. Умножая первое уравнение на —2 и складывая со вторым, получим следующие результаты. 2х + Зу = —3 —2х — Зу = —4 0 = — 7 Невозможно В результате мы пришли к противоречию. Допущение, что исходная система имеет решение, оказалось ошибочным (в противном случае 0 = — 7). Поэтому система не имеет решения и множество решений пусто. Графики уравнений системы являются параллельными прямыми и система несовместна. Теперь рассмотрим следующую систему. ^У = 4, -2х + у = -8. Умножая первое уравнение на число 2 и прибавляя результат ко второму уравнению, получаем следующую систему. 2х — у = 8, -2х + у = -8, 0 = 0.
Глава 4. Системы линейных уравнений и матрицы 291 Если после сложения уравнений получается равенство 0 = 0, то уравнения являются эквивалентными. Это значит, что их графики совпадают и система зависима. Если положить х = к, где к — произвольное действительное число, и решить любое уравнение относительно переменной у, то получается равенство у = 2к — 8. Это значит, что решениями системы будут пары (fc, 2к — 8) при любом действительном к. Переменная к в этом случае называется параметром, подстановка вместо к произвольного действительного числа дает частное решение системы. Например, частными решениями системы являются следующие точки. к = —1®, к = 2, к = 5, к — 9,4, (-1, -10) (2, -4) (5, 2) (9,4, 10,8) Решение практических задач Большое количество практических задач можно свести к системе двух уравнений, решение которой можно найти с помощью любого из рассмотренных выше методов. Рассмотрим два примера. ■w WWW Пример 4.6 (Диета). Для увеличения количества кальция и витамина А в еже- шм дневной диете женщина намерена пить молоко и апельсиновый сок. В одной унции молока содержится 37 мг кальция и 57 мкг1 витамина А. Одна унция апельсинового сока содержит 5 мг кальция и 65 мкг витамина А. Сколько унций молока и апельсинового сока нужно употреблять в пищу ежедневно, чтобы получать 500 мг кальция и 1200 мкг витамина А? Решение, На первом этапе решения этой задачи введем подходящие переменные, руководствуясь вопросом, Поставленным в задаче. В примере 4.6 требуется определить точное количество унций молока и апельсинового сока. Введем переменные, представляющие эти неизвестные величины. х — количество унций молока, у — количество унций апельсинового сока. Далее соберем все приведенные в условии данные в таблице. Ее удобно построить так, как показано ниже. Количества, которые соответствуют переменным, расположены по строкам, а не по столбцам. Молоко Апельсиновый сок Необходимое количество Кальций, мг 37 5 500 Витамин А, мкг 57 65 1200 Теперь построим уравнения для переменных х и у, используя данные таблицы. Количество кальция в х унциях молока 37а: Количество витамина Ава: унциях молока 57а: (Количество кальция в у унциях апельсинового сока 4- 5у (Количество витамина А в у унциях апельсинового сока + 65у Требуемое количество кальция 500, Требуемое количество витамина А 1200. Микрограмм — одна миллионная (10 6) часть грамма.
292 Часть II. Конечная математика Таким образом, мы пришли к следующей модели: 37ж + 5у = 500, 57х + 65?/ = 1200. Умножая первое уравнение на число —13 и используя исключение, получаем следующий результат. -481х - 651/ = -6500, 37 • 12,5 + Бу = 500; 57х + 651/ = 1200, 51/ = 37,5; —424z = -5300, у = 7,5 . х = 12,5 . Таким образом, требуемые количества кальция и витамина А получаются, если употреблять 12,5 унций молока и 7,5 унций апельсинового сока. Проверка. 37х + 51/ = 500; 57я + 651/ = 1200; 37 • 12,5 + 5 • 7,5 = 500; 57 • 12,5 + 65 • 7,5 = 1200; 500 = 500. 1200 = 1200. Иг На рис. 4.2 показано решение этой задачи методом графической аппроксимации. ■ Рис. 4.2. Решение задачи с помощью графической утилиты И WWW Упражнение 4.6. Для увеличения количества протеина и кальция в ежедневной диете мужчина намерен есть обезжиренный творог и йогурт. В одной унции обезжиренного творога содержится три грамма протеина и 15 мг кальция. Одна унция йогурта содержит один грамм протеина и 41 мг кальция. Сколько унций творога и йогурта нужно употреблять в пищу ежедневно, чтобы получать 62 г протеина и 760 мг кальция? ■ DWWW Пример 4.7 (Предложение и спрос). Количество продукта, которое потребители готовы приобрести за определенный промежуток времени, зависит от его цены. Обычно, чем выше цена, тем меньше спрос, и наоборот, чем ниже цена, тем больше спрос. Аналогично количество продукта, предлагаемого для продажи за определенный промежуток времени, зависит от его цены. Обычно количество предлагаемого продукта
Глава 4. Системы линейных уравнений и матрицы 293 по высоким ценам больше, чем по низким. Простейшая модель предложения и спроса линейна, графики уравнений предложения и спроса являются прямыми линиями. Допустим, что речь идет о ежедневных продажах вишен в некотором городе. С использованием специальных методов (регрессионного анализа) были найдены следующие уравнения зависимости предложения и спроса от цены. р — — 0,2g 4“ 4; Зависимость спроса от цены р — 0,07(7 4“ 0,76. Зависимость предложения от цены Здесь q — количество вишен (тыс. фунтов), р — цена (долл.). Например, если клиенты покупают 10 тыс. фунтов вишен (q =* 10), то цена за фунт будет р = —0,2 -10 + 4 = 2 долл, за фунт. С другой стороны, по такой цене поставщики могут предложить 17,714 тыс. фунтов вишен (корень уравнения 2 = 0,07д 4- 0,76). Иначе говоря, по цене два доллара за фунт поставщики предлагают гораздо больше товара, чем могут приобрести покупатели. Поскольку предложение превышает спрос при такой цене, то последняя будет падать. При какой цене продажи стабилизируются? Иначе говоря, при какой цене предложение будет таким же, как и Спрос? Такая цена (если она существует) называется равновесной, а объем продаж при такой цене называется равновесным количеством. Точка пересечения кривых для уравнений зависимости предложения и спроса от цен называется точкой равновесия. Как найти эти величины? Решим следующую систему уравнений. р = —0,2g + 4; Уравнение спроса р = 0,07g + 0,76. Уравнение предложения Применим метод подстановки (подставляя р = —0,2g + 4 во второе уравнение). -0,2g 4- 4 = 0,07д 4- 0,76; -0,27д == -3,24; q » 12 ТЫС. фунтов. Равновесное количество Теперь подставим величину q = 12 в любое уравнение исходной системы (возьмем первое уравнение) и вычислим р, р -0,2 • 12 4- 4; р я; 1,60 доллара за фунт. Равновесная цена Геометрическая интерпретация этих результатов приведена на рис. 4.3. Если цена выше равновесной, т.е. больше 1,60 долл, за фунт, то предложение будет больше спроса, и цена будет падать. Если цена ниже равновесной, т.е. меньше 1,60 долл, за фунт, то спрос будет больше предложения, и цена будет расти. Так или иначе, цена достигнет равновесного значения 1,60 долл, за фунт. При такой цене поставщики будут предлагать 12 тыс. фунтов вишен, и потребители будут покупать тоже 12 тыс. фунтов вишен. ■ Е Упражнение 4.7. Повторите решение примера 4.7 (включая построение графиков) при следующих условиях. р = -0,lq 4“ 3; Уравнение спроса р = 0,08д 4- 0,66. Уравнение предложения ■
294 Часть II. Конечная математика р Количество, тыс. фунтов Рис. 4.3. Экономическое равновесие Ответы к упражнениям 4.1. х = —2, у = —1. Проверка. 2х — у = —3 2. (_2)-(-1)1-3 —3 = 3 х + 2у = — 4 —2 + (—1) = —4 -4= -4 4.2. 1) х = 2, у = 2. 2) Бесконечно много решений. 3) Нет решений. 4.3. х = -1,92, у = 4,23. 4.4. х = —2, у = 2. 4.5. х = 2, у = —1. 4.6. 16,5 унций творога, 12,5 унций йогурта. 4.7. Равновесное количество = 13 тыс. фунтов, равновесная цена = 1,70 долл, за фунт.
Глава 4. Системы линейных уравнений и матрицы 295 Практикум 4.1 А В задачах 1-4 требуется определить, какому из приведенных графиков соответствует указанная система, и решить ее, используя график. 1. — 4х + 2у = 8, 2х - у = 0. 3. -х + 2у = 5, 2х + Зу = —3. 2. х 4- у = 3, 2х — у = 0. 4. 2х — 4у = —10, —х + 2у = 5.
296 Часть II. Конечная математика Решите задачи 5-8 с помощью графиков. 5. Зх — у = 2, 6. Зх — 21/ = 12, х + 2у = 10. 7х 4- 21/ = 8. 7. т + 2п = 4, 8. Зи 4- 51; = 15, 2т + 4п = —8. би 4-101; = —30. Решите задачи 9-12 методом подстановки. 9. у = 2х — 3, 10. у = х — 4, х + 2у = 14. х 4- Зу = 12. 11. 2х + у = 6, 12. Зх - у = 7, х - у = -3. 2х 4- Зу = 1. Решите задачи 13-16 методом исключения. 13. Зи - 2v = 12, 14. 2х — 31/= —8, 7и -h 2v = 8. 5х 4- Зу = 1. 15. 2т — п = 10, 16. 2x4-31/ = 1, т — 2п = —4. Зх — у = 7. Б В задачах 17-30 требуется решить системы методом подстановки или исключения. 17. 9х - Зу = 24, 18. 4х 4- Зу = 26, Их -I- 2у = 1. Зх - 111/ = -7. 19. 2х — Зу = —2, 20. Зх — 61/ = —9, —4х 4- бу = 7. —2х 4- 41/ = 12. 21. Зх 4- 8у = 4, 22. 7т 4- 12п = —1, 15х 4-101/ = —10. 5т — Зп = 7. 23. —6х 4- 101/ = —30, 24. 2х 4- 41/ = —8, Зх — Зу = 15. х 4- 2у = 4. 25. х 4- у = 1, 26. х 4- у = 1, 0,Зх — 0,41/ = 0. 0,5х — 0,41/ = 0. 27. 0,2х — 0,51/= 0,07; 28. 0,Зи - 0,61; = 0,18; 0,8х - 0,31/ = 0,79. 0,5и 4- 0,21; = 0,54. _ 2 3 7 5 29, + = 2; о / 30. ~х - -у = 10; Z о 7 5 АЛ XJ 2 4 „ -х - -у = -5. -х 4- ~У = 6. 3 4 5 3* ICl В задачах 31-34 требуется вычислить приближенное решение систем уравнений с точностью до двух десятичных знаков с помощью графического калькулятора. 31. Зх — 2у = 5, 32. Зх — 7у = —20, 4х + Зу = 13. 2х + Зу = 8.
Глава 4. Системы линейных уравнений и матрицы 297 33. -2,4ж + 3,5?/ = 0,1; 34. 4,2ж + 5,4?/ = -12,9; -1,7ж + 2fiy = -0,2. 6,4ж + 3,7?/ = -4,5. В В задачах 35-40 постройте графики всех трех уравнений на одном рисунке. Найдите координаты точек, в которых пересекаются несколько прямых линий. 35. х-2у = -6, 2х + у = 8, х + 2у = -2. 36. х + у = 3, х + Зу = 15, Зх — у = 5. 37. х + у = 1, х — 2у — —8, Зх + у = —3. 38. х — у = 6, х - 2у = 8, х + 4?/ = —4. 39. 4х — Зу = —24, 2х + Зу = 12, 8х — бу = 24. 40. 2х 4- Зу = 18, 2х — Бу = —6, 4х + Бу = —24. *41. В приведенных ниже системах уравнений коэффициенты очень похожи. Можно предположить, что и множества решений этих систем почти одинаковы. Подтвердите или опровергните это предположение, рассматривая графики данных систем уравнений и решая системы методом подстановки или исключения. а) 5я + 4?/ = 4, б) 5ж + 4?/ = 4, 11# + 9у = 4. Их + 8?/ = 4. в) Бх -I- 4у 2= 4, Юж + Зу «= 4. * 42. Повторите решение задачи 41 для таких систем: а) Бх — Бу — 10, б) 6ж — Бу = 10, — 13ж + 11у = “20. —13ж + 10г/ = -20. в) 6ж - Бу == 10, -12я + 10?/ = -20. Применение математики Экономика и бизнес 43. Предложение и спрос. Пусть уравнения спроса и предложения футболок в курортном городе на некоторой неделе имеют следующий вид. р = 0,7g + 3; Уравнение предложения р — -1,7g + 15; Уравнение спроса где р — цена в долларах, q — количество в тысячах штук. ♦а) Найдите предложение и спрос (с точностью до единицы) для футболок при цене четыре доллара за каждую. Обсудите стабильность рынка футболок при таком уровне цен.
298 Часть II. Конечная математика *б) Найдите предложение и спрос (с точностью до единицы) для футболок при цене девять долларов за каждую. Обсудите стабильность рынка футболок при таком уровне цен. в) Найдите равновесную цену и равновесное количество. г) Постройте графики обоих уравнений на одном рисунке. Покажите точку равновесия, кривые спроса и предложения. 44. Предложение и спрос. Пусть уравнения спроса и предложения бейсбольных кепок в курортном городе в течение конкретной недели имеют следующий вид. р = 0,4д + 3,2; Уравнение предложения р = — 1,9q + 17; Уравнение спроса где р — цена в долларах, q — количество в сотнях штук. *а) Найдите предложение и спрос (с точностью до единицы) для бейсбольных кепок при цене четыре доллара за каждую. Обсудите стабильность рынка бейсбольных кепок при таком уровне цен. *б) Найдите предложение и спрос (с точностью до единицы) для бейсбольных кепок при цене девять долларов за каждую. Обсудите стабильность рынка бейсбольных кепок при таком уровне цен. в) Найдите равновесную цену и равновесное количество. г) Постройте графики обоих уравнений на одном рисунке. Покажите точку равновесия, кривые спроса и предложения. 45. WWW Предложение и спрос. При цене 0,60 долл, за бушель ежедневное предложение пшеницы составляет 450 бушелей, а спрос — 570 бушелей. Когда цена выросла до 0,75 долл, за бушель, ежедневное предложение выросло до 600 бушелей, а спрос упал до 495 бушелей. Допустим, что уравнения спроса и предложения — линейны. а) Найдите уравнение спроса. (Подсказка. Запишите искомое уравнение в форме р = aq + Ь и решите относительно а и 6.) б) Найдите уравнение предложения. в) Найдите равновесную цену и равновесное количество. г) Постройте графики обоих уравнений на одном рисунке. Покажите точку равновесия, кривые спроса и предложения. 46. WWW Предложение и спрос. При цене 1,40 долл, за бушель ежедневное предложение овса составляет 850 бушелей, а спрос — 580 бушелей. Когда цена снизилась до 1,20 долл, за бушель, дневное предложение упало до 350 бушелей, а спрос вырос до 980 бушелей. Допустим, что уравнения спроса и предложения — линейны. а) Найдите уравнение спроса. б) Найдите уравнение предложения. в) Найдите равновесную цену и равновесное количество. г) Постройте графики обоих уравнений на одном рисунке. Покажите точку равновесия, кривые спроса и предложения.
Глава 4. Системы линейных уравнений и матрицы 299 47. Анализ безубыточности. Небольшая компания производит ноутбуки. Постоянные затраты (аренда, страхование и т.п.) составляют 48 000 долл, в месяц, а переменные издержки (оплата труда, материалы и т.п.) — 1400 долларов на каждую произведенную единицу. Ноутбуки продаются по цене 1800 долл. Следовательно, уравнения затрат и доходов имеют такой вид. у =: 48 000 + 1400т, у — 1800т, Уравнение затрат Уравнение доходов где х — ежемесячный объем производства (шт.), у — ежемесячные затраты и доходы (долл.). а) При каком объеме производство будет безубыточным? *б) Постройте графики обеих функций на одном рисунке и найдите точку безубыточности. Опишите области между прямыми слева и справа от точки безубыточности. 48. Анализ безубыточности. Повторите решение задачи 47 для следующих уравнений затрат и доходов. у = 65 000+ 1100т, у — 1600т. Уравнение затрат Уравнение доходов 49. Анализ безубыточности. Фирма, доставляющая заказы по почте, продает видеокассеты по цене 19,95 долл, за единицу, включая перевозку и доставку. Месячные постоянные затраты (реклама, арендная плата и т.п.) составляют 24 000 долл., а переменные издержки (перевозка, материалы) — 7,45 долл, за каждую кассету, а) При каком объеме продаж достигается уровень безубыточности? *б) Постройте графики уравнений затрат и доходов на одном рисунке и найдите точку безубыточности. Опишите области между прямыми слева и справа от точки безубыточности. 50. Анализ безубыточности. Повторите решение задачи 49, если месячные постоянные затраты выросли до 2700 долл., переменные издержки — до 9,15 долл., и фирма увеличила цену за единицу товара до 21,95 долл. 51. Стоимость доставки. Компания United Express установила базовую цену доставки пакета весом в один фунт или меньше, а за каждый дополнительный
300 Часть II. Конечная математика фунт (или его часть) веса предусмотрела доплату. Получателю пришел счет на 27,75 долл, за доставку пакета весом в пять фунтов, и 64,50 долларов доставки 20-фунтового пакета. Найдите базовую цену доставки и сумму доплаты за каждый дополнительный фунт. 52. Стоимость доставки. Вернемся к задаче 51. Компания Federal Shipping информировала получателя (см. задачу 51) о том, что доставка пакета весом в пять фунтов стоит 29,95 долларов, а 20-фунтового — 59,20 долларов. а) Считая, что стоимость услуг этой компании определяется по тем же правилам, что и в компании United Express, найдите базовую цену за доставку и сумму доплаты за каждый дополнительный фунт веса при доставке груза компанией Federal Shipping. *б) Определите простое правило, позволяющее выбрать из этих вариантов более дешевый. Аргументируйте свой ответ. 53. Распределение ресурсов. Производитель кофе использует колумбийские и бразильские кофейные зерна для выпуска двух смесей: робусты и мягкого кофе. Для получения фунта робусты требуется 12 унций колумбийских и четыре унции бразильских зерен. Для фунта мягкого кофе требуется шесть унций колумбийских и десять унций бразильских зерен. Кофейные зерна поставляются в мешках по 132 фунта. У компании имеется 50 мешков колумбийских и 40 бразильских зерен. Сколько фунтов каждой смеси можно выпустить, используя имеющееся сырье? 54. Распределение ресурсов. Вернемся к задаче 53. а) Предположим, что компания приняла решение прекратить выпуск робусты и производит только одну смесь мягкого кофе. Сколько фунтов этой смеси может производить компания, и какое количество зерен каждого сорта для этого требуется? Все ли имеющиеся запасы сырья будут использованы? б) Повторите решение п. а, если компания прекратила производство мягкого кофе, выпуская только робусту. Биологические науки 55. Рацион питания. Подопытные животные содержатся на жесткой диете. Помимо всего прочего, каждое животное должно получать 20 г белков и 6 г жиров. В лаборатории производят две пищевые смеси: смесь А, в которой содержится 10% белков и 6% жиров, и смесь Б, содержащую 20% белков и 2% жиров. Сколько граммов каждой смеси нужно использовать для поддержания правильной диеты?
Глава 4. Системы линейных уравнений и матрицы 301 56. Удобрение растений. Для ухода за апельсиновыми деревьями используются два вида удобрений ~ сортов А и Б. В каждой пачке сорта А содержится восемь фунтов азотных И четыре фунта фосфорных удобрений. В пачке сорта Б содержится семь фунтов азотных и шесть фунтов фосфорных удобрений. Исследования показали, что Дереву требуется 720 фунтов азотных и 500 фунтов фосфорных удобрений. Сколько пачек удобрения каждого сорта нужно, чтобы обеспечить дерево требуемым количеством полезных веществ? Социальные науки 57. Психология: стремление и уклонение. Некоторые ситуации люди воспринимают со “смешанными эмоциями”. Например, публичная речь приносит положительные эмоции в случае одобрения, и отрицательные — в случае неудачи. Какая из них доминирует? Дж. С. Браун (J. S. Brown) в одном из экспериментов тренировал крыс, Кормив их из коробки. Затем из той же коробки крысы получали электрический разряд. В этом и проявлялся конфликт стремления и уклонения, в данном случае связанный с коробкой. Используя специальную аппаратуру, Браун получил следующие закономерности. р - -“d + 70; о 4 р — --d + 230. 3 Уравнение стремления Уравнение уклонения Здесь d — расстояние от коробки, причем 30 < d < 172,5. Уравнение стремления выражает вес (г), который крыса прилагает как усилие, чтобы приблизиться к коробке, находясь на на расстоянии d (см) от нее. Уравнение уклонения выражает вес (г), который крыса прилагает как усилие, чтобы убежать от коробки, находясь на расстоянии d сантиметров от нее. а) Постройте графики уравнений стремления и уклонения в одной и той же системе координат. б) Найдите, при каком значении d графики уравнений пересекаются. *в) Как вы считаете, что будет делать крыса, если ее поместить на расстоянии d от коробки? (Более подробно это явление обсуждается в работе J. S. Brown, “Gradients of Approach and Avoidance Responces and Their Relation to Motivation”, Journal of Comparative Physiological Psychology, 1948, 41:450-465.) 4.2. Системы линейных уравнений и расширенные матрицы ■ Матрицы ■ Решение систем уравнений с помощью расширенных матриц ■ Резюме Обычно системы уравнений состоят из большого числа уравнений и неизвестных. Решать их вручную непрактично. Раньше подобные задачи решались на больших компьютерах, но теперь появилось много программ для решения линейных систем. Выбор
302 Часть II. Конечная математика достаточно широк: графические калькуляторы (вроде Texas Instruments TI-83), пакеты программ (например, Explorations in Finite Mathematics или Matlab), процессоры электронных таблиц (например, Excel). Все эти программы имеют нечто общее: для их использования вы должны уметь решать системы линейных уравнений вручную. В этом разделе мы изучим матричные методы решения систем уравнений, эффективность которых в первую очередь проявляется при выполнении расчетов в графических утилитах. Но не стоит думать, что специальное программное обеспечение будет решать математические задачи самостоятельно. Ни одна графическая утилита не найдет решение системы уравнений без участия человека. В основном наше внимание будет сосредоточено на построении математических моделей и интерпретации получаемых результатов — это как раз те задачи, которые не под силу ни одной графической утилите. Как и раньше, задачи, для решения которых требуется применение графической утилиты, будут отмечаться пиктограммой. Матрицы В рассматриваемом ранее методе исключения центральную роль играли множители при неизвестных переменных и постоянные слагаемые в правой части. Решение систем легче описать, если определить специальный математический объект, который называется матрицей. Матрица — это прямоугольный массив чисел, записанный в скобках. Приведем два примера матриц: -4 5 12 1-451 018 7 0 —2 _| ’ ~ -3 10 9 -6 0 -1 (4.1) Каждое число в матрице называется элементом. Матрица А состоит из шести элементов, расположенных в двух строках и трех столбцах. Матрица В состоит из 12 элементов, расположенных в четырех строках и трех столбцах. Если матрица состоит из т строк и п столбцов, то она называется “матрицей т X п” (читается: “матрица т на п”). Выра¬ жение т X п называется размером матрицы, а числа т и п — ее размерностями. Обра¬ тите внимание на то, что количество строк всегда приводится первым. В уравнении (4.1) матрица А имеет размер 2 х 3, а матрица В — размер 4x3. Матрица с п строками и п столбцами называется квадратной матрицей порядка п. Матрица с одним столб- 1X4 1 0 цом называется матрицей-столбцом, а матрица с одной строкой называется матрицей- строкой. Приведем примеры. 3X3 ■ 0,5 0,2 1,0 ' 0,0 0,3 0,5 _ 0,7 0,0 0,2 _ Квадратная матрица третьего 4X1 3 ' —2 1 0 _ Матри ца-стол бец Матрица-строка порядка Положение элемента в матрице определяется номером строки и столбца, на пересечении которых находится элемент, и обозначается с помощью двойного индекса ,
Глава 4. Системы линейных уравнений и матрицы 303 где i — номер строки, j — номер столбца. 1-4 5 7 0-2 ап = 1, а21 = 7, ai2 = —4, ai3 = 5, а22 = 0, &23 = — 2. Обратите внимание на то, что число а 12 — это “а с индексами один и два” (а не “а с индексом двенадцать”). Элементы ац 1 и й22 = 0 образуют главную диагональ матрицы А. В общем случае, главная диагональ матрицы А состоит из элементов ап, 0,22, азз,... Замечание. 0 ТЧ Л 5 В большинстве графических утилит есть средства для хранения матриц и операции с ними. На рис. 4.4 показана матрица на экране графического калькулятора. Наверху указан размер матрицы, а внизу — положение и значение текущего элемента. Обратите внимание на то, что индексы элемента разделены запятой. Так обычно делают во всех графических утилитах, однако это обозначение не является стандартным в математической литературе. В компьютерных таблицах элементы матриц нумеруются или целыми числами по строкам И столбцам (рис. 4.5, а)), или числами по строкам и прописными буквами по столбцам (рис. 4.5, б)) ffiTRfXtRl 2 хЗ : ? о -г з г=-4 1 1 2 1 1 : -4 1Л 2 7i 0 к гэ Рис. 4.4. Матрица в графическом калькуляторе: нумерация элементов б) Матрица хранится в ячейках: D5:F6 а) Матрица хранится в ячейках: R1C1:R2C3 Рис. 4.5* Матрица в электронной таблице: нумерация элементов Коэффициенты и постоянные слагаемые в системах уравнений можно использовать для образования специальных видов матриц. Рассмотрим систему уравнений. 2х — Зу = 5, х 4- 2у = —3. (4.2) Введем такие матрицы. Матрица коэффициентов Г 2 -3 “ 1 2 Матрица свободных членов Г 5 ■ -3 Расширенная матрица 2—3 5 1 2 -3
304 Часть II. Конечная математика В расширенной матрице содержится наиболее существенная информация о системе уравнений — коэффициенты при неизвестных и свободные члены. Вертикальная черта используется для того, чтобы их визуально разделить. (В графических калькуляторах и электронных таблицах эта черта не изображается.) Позднее в этой главе мы будем разделять матрицы коэффициентов и свободных членов, но сейчас нашей целью будет построение расширенных матриц. Чтобы обобщить получаемые результаты на системы с большим количеством уравнений, изменим обозначения в системе (4.2), переходя к индексам. (Вскоре мы сможем обходиться без букв, используя только индексы.) Иначе говоря, вместо хну будем писать соответственно и х2. Тогда система (4.2) принимает следующий вид. 2^1 — Зх2 = 5, Xi + 2х2 = —3. В общем случае, с каждой системой уравнений вида ацХ1 + Я12Х2 = fci, . L. (4«3) ^21^1 Т Cb22x2 = ^2- где xi и х2 — переменные, связана расширенная матрица. Столбец 1 (Ci) Столбец 2 (С2) | Столбец 3 (Сз) 011 012 ^1 * Строка 1 (Hi) 021 а22 к>2 * Строка 2 (R2) В этой матрице содержится наиболее важная информация о системе уравнений (4.3). Нашей целью является изучение основных операций с расширенными матрицами, с помощью которых можно найти решение системы (4.3), если таковое существует. Методы решения уравнений с помощью расширенных матриц являются прямым следствием метода исключения, рассмотренного в разделе 4.1. Напомним, что системы уравнений называются эквивалентными, если их множества решений совпадают. Как преобразовать систему уравнений в эквивалентную? Для этого используются операции, перечисленные ниже (теорема 4.2 раздела 4.1). Операции, приводящие к получению эквивалентных систем уравнений Система линейных уравнений переходит в эквивалентную систему при следующих преобразованиях. 1. Изменение порядка записи уравнений. 2. Умножение уравнения на константу, не равную нулю. 3. Умножение уравнения на константу и сложение с другим уравнением системы. Возвращаясь к предыдущему обсуждению, можно говорить, что две расширенные матрицы являются эквивалентными по строкам, если соответствующие им системы уравнений эквивалентны. (Этот факт обозначается значком помещенным между матрицами). Какие действия можно выполнить над матрицами, эквивалентными по строкам? Любые, рассмотренные выше и следующие из теоремы 4.3.
Глава 4. Системы линейных уравнений и матрицы 305 Теорема 4.3 (Операции, Приводящие к получению расширенных матриц, эквивалентных по строкам). Перечисленные ниже операции над строками позволяют преобразовать расширенную матрицу в матрицу, эквивалентную по строкам. 1. Перестановка местами двух строк (Ri <-► ЯД 2. Умножение любой строки на ненулевую константу (kRi —> Ri). 3. Умножение любой строки на ненулевую константу и прибавление к другой строке (kRj + Ri —► Я<). ■ Решение систем уравнений с помощью расширенных матриц Использование теоремы 4.3 для решения систем уравнений, записанных в форме (4.3), лучше всего проиллюстрировать на примерах. Пример 4.8 (Решение систем уравнений с помощью расширенной матрицы). Решите систему уравнений, используя операции над расширенной матрицей. 3#1 -h 4a?2 = 1, (4.4) X! -2X2=7. Решение. Сначала построим расширенную матрицу для системы (4.4). 3 1 4 1 —2 7 (4.5) Наша цель — привести матрицу (4.5) к следующему виду. 1 0 (4.6) где т и п — действительные числа. Тогда решение системы (4.4) будет очевидным, поскольку расширенная матрица (4.6) соответствует такой системе уравнений (строка расширенной матрицы переходит в уравнение системы). xi = т «1 + 0®2 = т Х2 = П 0®i + Х2 = п Теперь выполним операции над строками матрицы (4.5), преобразовывая ее к виду (4.6). Этап 1. Чтобы в левом верхнем углу стояла единица, поменяем местами строки R\ и R2 (теорема 4.3, п. 1). ’ 3 4 1' 1 -2 7 Ri 4—► R2 1 —2 7 3 4 1 Этап 2. Чтобы в левом нижнем углу стоял нуль, умножим строку Ri на число —3 и прибавим результат к строке Яд (теорема 4.3, п. 3). Это приводит к изменению строки Яг, а не Я1« Иногда удобно выносить коэффициент — ЗЯ1 за пределы матрицы для облегчения Поиска арифметических ошибок, как показано ниже. 1—2 7 “ 3 4 1 — ЗЯ1 “ЬЯг —► R2 -3 в —21* ■ 1 —2 7 ' 0 10 -20
306 Часть II. Конечная математика Этап 3. Чтобы на пересечении второй строки и второго столбца стояла единица, умножим строку Т?2 на (теорема 4.3, п. 2). 1 —2 0 10 ^2 1 —2 7 0 1 —2 Этап 4. И, наконец, чтобы на пересечении первой строки и второго столбца стоял нуль, умножим строку Т?2 на число 2 и прибавим результат к первой строке (теорема 4.3, п. 3). Это приводит к изменению строки , но не Т?2- О 1 о 7 —2 2R% +-R1 —► Ri 1 0 3 0 1 —2 Таким образом, наша цель достигнута! Последняя расширенная матрица соответствует следующей системе. =3, «1 + 0®2 = 3, Х2 = —2. 0X1 + = ~2 . Поскольку система (4.7) эквивалентна исходной системе (4.4), мы нашли решение системы (4.4), т.е. = 3 и Х2 = —2. Проверка. 3xi + 4x2 = 1, Xi - 2x2 = 7, 3-3 + 4-(-2) = 1, 3-2-(-2) = 7, 1 = 1. 7 = 7. а Рассмотренный процесс можно записать в более компактной форме, как показано ниже. 4 1 —2 7 —2 7 4 1 6 —21- R± 4—► R% —3.R1 Н“/?2 —► R2
307 Глава 4. Системы линейных уравнений и матрицы Таким образом, xi *= 3 И Жд = —2. Упражнение 4.8. Решите следующую систему уравнений, используя операции над расширенными матрицами. 2x1 — = —7, Xi + 2х2 =4. . icl WWW Во многих Графических утилитах предусмотрены команды, выполняющие операции над строками. На рис. 4.6 показаны результаты применения таких операций, применяемых при решении примера 4.8. Инструкции по выполнению этих операций приведены в справочном руководстве графической утилиты. rm [[3 4 11 [1 -2 71] rowSwapC[A], 1,2) * [R] ([1 "2 ?] [3 4 1J] ТТЛ Щ -2 7] [R], 1>2 1§ ?20J] ТН1 [[1-27 ] [0 10 -20JJ *row<. 1 я [Al ? 2)-> [ А] ТН1 [[1-27] [0 1 -2]] *row+<2,[А],2,1) [[1 -27] [0 1 -2J1 [[103] [0 1 -2]] a) R^R2 б) —ЗЯ. + Л,-Я, г) 2/^+^ Рис. 4.6. Операции над строками, выполненные в графической утилите Задание 4.3. В примере 4.8 фигурируют пять расширенных матриц. Вычислите системы линейных уравнений, которые соответствуют этим матрицам, и найдите решение каждой из них графически. Обсудите связь между этими решениями. ■ Пример 4.9 (Решение систем уравнений с помощью расширенной матрицы). Решите систему уравнений, выполняя операции над расширенной матрицей. 2x1 — 3x2 = 6, 3xi + 4х2 = (4.8)
308 Часть II. Конечная математика Решение. — ЗК1 Н“Л2 —► I-R2 +Н1 —► Ri —► Я1 Таким образом, Xi = | и Х2 = — 1. Проверку оставляем читателю. Упражнение 4.9. Решите систему уравнений, выполняя операции над расширенной матрицей. 5xi — 2x2 = И5 5 2x1 4-3x2 = х- 4 Пример 4.10 (Решение систем уравнений с помощью расширенной матрицы). Решите систему уравнений, выполняя операции над расширенной матрицей. 2xi — ^2 = 4, —6x1 4- 3x2 = —12. Решение. 2 -1 -6 3 4 -12 |Л1 —► R\ (получаем единицу в верхнем левом углу) |Т?2 “► R2 (та же процедура для Я2)
Глава 4. Системы линейных уравнений и матрицы 309 1 —2 -1 -12 . 4* 2Й1 + R2 —► R2 (получаем нуль в нижнем левом углу) 1 2 2 1 1 0 1 2 2 0 Последняя матрица эквивалентна системе: - ^2 = 2, 0 = 0. ОЖ1 + 0®2 = О • (4.9) Эта система уравнений эквивалентна исходной системе. С геометрической точки зрения графики уравнений исходной системы совпадают, и задача допускает бесконечное количество решений. Вообще, если в результате преобразований получается расширенная матрица с нулевой строкой в системе двух линейных уравнений, то система является зависимой и имеет бесконечно много решений. Представим бесконечное Множество решений как в разделе 4.1, т.е. используя параметр. Начнем с решения первого уравнения системы (4.9), xi - ±х2 = 2, выражая одну переменную через другую. Выразим х± через х2, поскольку это проще: = -Х2 + 2. (4.Ю) Теперь введем параметр t (для обозначения параметра можно использовать и другие буквы, например к, s, р, д). Если для произвольного действительного значения t мы положим х2 = t9 то пара + 2, (4.И) х2 =t является решением системы (4.8). С помощью обозначения упорядоченных пар это выражение можно записать так: для произвольного действительного значения t -t + 2, Z (4.12) является решением системы (4.8). Иначе говоря, t еЯЛ . (4.13) В дальнейшем для записи решений такого типа мы будем пользоваться менее формальными обозначениями (4.11) и (4.12).
310 Часть II. Конечная математика Проверка. Ниже приведена проверка того, что система уравнений (4.11) позволяет найти решение для системы уравнений (4.8) для любого действительного числа t: 2xt - х2 = 4; /1 \ ? 2 -t + 2 - * = 4; \ & j t + 4 - t = 4; 4 = 4. —6x1 + Зжг = —12; /1 \ ? —6 ( — t н- 2 J = —12; -3t - 12 + 3t = -12; — 12 = -12. Упражнение 4.10. Решите систему уравнений, выполняя операции над расширенной матрицей. -2д?1 -|- 6^2 = 6, 3xi — 9x2 = —9. Задание 4.4. В решении примера 4.10 фигурируют три расширенные матрицы. Вычислите системы линейных уравнений, которые соответствуют этим матрицам и найдите решение каждой из них графически. Обсудите связь между этими решениями. ■ Пример 4.11 (Решение систем уравнений с помощью расширенной матрицы). Решите систему уравнений, выполняя операции над расширенной матрицей. 2xi + 6х2 = —3, Xi 4" 3x2 — 2. Решение. 2 6-3 1 3 2 ‘ 1 3 2 2 6-3 -2 -6 -4 ’ 1 3 2 0 0-7 R± ► R>2 —2R1 R2 —► R2 Уравнение R2 эквивалентно равенству О = —7 Второе уравнение не может выполняться ни для одной упорядоченной пары действительных чисел. Как известно из раздела 4.1, эта система является несовместной и не имеет решений, поскольку равенство 0 = — 7 заведомо ошибочно. Таким образом, если в расширенной матрице в результате преобразований слева от вертикальной черты получаются нули, а справа — отличное от нуля число, то система является несовместной и не имеет решений. ■
Глава 4. Системы линейных уравнений и матрицы 311 Упражнение 4.11. Решите систему уравнений) выполняя операции над расширенной матрицей. 2xi —^2 = 3, 4xi — 2x2 = — 1* Резюме В примерах 4.8-4.11 мы рассмотрели все возможные типы решений системы уравнений с двумя независимыми переменными, которые упоминались в теореме 4.1 раздела 4.1. Классифицируем их заново, но на этот раз для матричного представления. Способы матричного представления системы двух линейных уравнений с двумя неизвестными Форма 1 Единственное решение (совместная и независимая система) 10т 0 1 п Форма 2 Бесконечно много решений (совместная и зависимая система) 1 т п ООО Форма 3 Нет решений (несовместная система) 1 т п 0 0 р Здесь т, п и р — действительные числа, причем р / 0. Метод решения линейных систем, рассмотренный в этом разделе, обычно называется методом исключения Гаусса-Жордана. В следующем разделе этот метод будет обобщен для решения линейных систем произвольного размера, в том числе и для таких систем, у которых количество уравнений и неизвестных различно. Ответы к упражнениям 4.8. xi = —2, Х2 = 3. 4.9. xi = 2, х2 = - j. 4.10. Система является зависимой. Для произвольного действительного числа t решения имеют вид: xi — 3t — 3, Х2 — t. 4.11. Система является несовместной и не имеет решений. Практикум 4.2 А В задачах 1-10 речь идет о следующих матрицах. -4 1 0 ' -5 С = [ 2 -3 0 ] , -1 —4 2 -5 8 А = 2 6 В - D - 9 8 4 0 7 0 1. Какой размер у матрицы А? У матрицы С?
312 Часть II. Конечная математика 2. Какой размер у матрицы В? У матрицы D? 3. Найдите все матрицы-строки. 4. Найдите все матрицы-столбцы. 5. Найдите все квадратные матрицы. 6. Для матрицы В найдите 62х и 613. 7. Для матрицы А найдите 012 и а2з- 8. Для матриц С и D найдите С13 и (/21 • 9. Найдите все элементы главной диагонали матрицы В. 10. Найдите все элементы главной диагонали матрицы А. В задачах 11 и 12 речь идет о следующих матрицах. 1-2 3 9 -5 0 7 -8 11. Ответьте на следующие вопросы. а) Какой размер у матрицы Е? б) Сколько столбцов нужно добавить к матрице F, чтобы дополнить ее до квадратной матрицы? в) Найдите е2з и Д2. 12. Ответьте на следующие вопросы. а) Какой размер у матрицы F? б) Сколько строк нужно добавить к матрице Е, чтобы дополнить ее до квадратной матрицы? в) Найдите е 14 и/з1. В задачах 13-24 выполните указанные операции над строками со следующей матрицей. 1-3 2 4 -6 -8 13. R\ <—> В2. 14. |я2 r2- 15. -4Л1 -► Вь 16. —2Ri —► Ri. 17. 2R2 R2. 18. —1R2 —> r2. 19. — 4Bi 4- R2 —> R2. 20. — 2^2 + —► J?i 21. — 2Ri 4- R2 —> R2. 22. —3J?i + R2 —► r2. 23. — l/?i 4- R2 R2. 24. Ri 4- R2 ~> R2 • Матрицы, приведенные в задачах 25-30, получаются в результате выполнения одной операции над строками матрицы А. Какая это операция? -1 2 -3 6 -3 12
Глава 4. Системы линейных уравнений и матрицы 313 25. ■ -1 2 2 -1 -3 4 27. ' -1 0 2 - 9 - -3 ’ -6 29. ’ 1 6 1 1 ■ -3 12 26. -2 6 4 -3 -6 12 28. ’ 3 0 5 ' 6 -3 12 30. ’ -1 2 -3 ' 2 5 0 Et. WWW Проверьте свои выводы, проводя вычисления с помощью графической утилиты. Решите системы уравнений в задачах 31 и 32 с помощью расширенной матрицы. В каждом случае запишите систему, соответствующую расширенной матрице, и решите эту систему графически. Обсудите связь между решениями. * 31. xi + Х2 = 5, * 32. xi — Х2 = 2, Х1 — Х2 = 1. Xi + Х2 = 6. Б В задачах 33-52 найдите решения систем уравнений с помощью расширенной мат- рицы. 33. Х\ — 2X2 — 1) 34. XI + 3X2 = 1, 2xi —^2 = 5. 3xi — 2x2 = 14. 35. xi - 4х2 ®= -2, 36. xi — 3x2 = —5, -2xi + ^2 — —3. —3x1 — Х2 = 5. 37. 3xi — Х2 = 2, 38. 2xi + х2 = 0, xi -F 2x2 = Ю. xi — 2x2 = —5. 39. xi + 2х2 — 4, 40. 2xi - 3x2 = ~2, 2xi + 4x2 = ^8. —4x1 4- 6х2 = 7. 41. 2X1 + ^2 = б) 42. 3xi — Х2 = —5, Xi — Х2 = —3. xi 4- 3x2 = 5. 43. 3X1 — 6x2 “9, 44. 2xi — 4x2 = —2, -2xi + 4x2 = 6. —3X1 4- 6x2 = 3. 45. 4xi — 2x2 2, 46. —6X1 4- 2x2 = 4, —6x1 4- 3x2 — 3. 3xi — Х2 = —2. 47. 2xi + х2 = 1, 48. 2xi — %2 = —8, 4xi - Х2 = —7. 2xi 4- Х2 = 8. 49. 4xi - 6x2 — 8, 50. 2xi — 4x2 = —4, —6x1 + 9x2 » -10. —3X1 4- 6x2 = 4. 51. -4xi + 6x2 ** “8, 52. —2x1 4- 4x2 = 4, 6xi - 9x2 — 12. 3xi — 6X2 = —6.
314 Часть II. Конечная математика В В задачах 53-58 найдите решения систем уравнений с помощью расширенной мат- рицы. 53. 3xi — Х2 = 7, 54. 2xi ~ 3x2 = —8, 2xi + 3x2 = 1. 5xi + 3x2 = 1. 55. 3xi + 2x2 = 4, 56. 4xi + 3x2 = 26, 2xi — х2 = 5. 3xi — 11^2 = —7. 57. 0,2xi + 0,5x2 = 0,07, 58. 0,3xi — 0,6x2 = 0Д8, 0,8xi - 0,Зх2 = 0,79. 0,5xi — 0,2х2 = 0,54. Jex, В задачах 59-62 найдите решения систем уравнений с помощью расширенной матрицы. Для вычислений используйте графическую утилиту. 59. 0,8a:i + 2,88x2 = 4, 60. 2,7xi - 15,12x2 = 27, l,25xi + 4,34x2 = 5. 3,25xi - 18,52x2 = 33. 61. 4,8xi - 40,32x2 = 295,2, 62. 5,7xi - 8,55x2 = -35,91, -3,75xi + 28,7x2 = -211,2. 4,5xi + 5,73x2 = 76,17. 4.3. Метод исключения Гаусса-Жордана ■ Приведенные матрицы ■ Решение систем уравнений методом исключения Гаусса-Жордана ■ Решение практических задач Теперь, имея некоторый опыт операций над отдельными расширенными матрицами, рассмотрим системы уравнений с более чем двумя неизвестными. Кроме того, мы не будем требовать, чтобы количество уравнений и количество неизвестных совпадали. Оказывается, выводы относительно свойств системы двух линейных уравнений с двумя неизвестными, приведенные в теореме 4.1 раздела 4.1, можно распространить на системы уравнений произвольной размерности. Возможные решения системы линейных уравнений Система линейных уравнений может иметь однозначное решение, не иметь решения или допускать бесконечное количество решений, независимо от количества уравнений и неизвестных. Как и для системы двух уравнений с двумя неизвестными, эти варианты можно описать терминами единственное решение, совместная, несовместная, зависимая и независимая система. Приведенные матрицы В предыдущем разделе мы использовали операции над сроками для преобразования расширенной матрицы для системы двух линейных уравнений с двумя неизвестными Оц Я12 &1 011^1+012^2 = ^1, а21 «22 &2 J «21X1 + 022^2 = ^2-
Глава 4. Системы линейных уравнений и матрицы 315 в одну из следующих упрощенных форм. Форм а 1 Форма 2 ■ 1 0 т 1 т п 0 1 п 0 0 0 Форма 3 1 т п О 0 р (4.14) Здесь т, п и р — действительные числа, причем р / 0. Каждая из этих приведенных форм соответствует отдельному классу решений и никакие две из них не являются эквивалентными по строкам. Теперь мы перейдем к рассмотрению систем с большим количеством уравнений и неизвестных. Задание 4.5. Формы матриц (4.14) соответствуют линейным системам с единственным решением, с бесконечным количеством решений и без решений соответственно. Определите количество решений для следующих расширенных матриц трех линейных уравнений с тремя неизвестными. ‘ 1 2 3 5 ‘ ’ 1 2 3 5 ' " 1 0 0 5 1. 0 0 0 6 2. 0 0 0 0 3. 0 1 0 6 0 0 0 0 0 0 0 0 0 0 1 7 Поскольку теперь Мы не ограничиваем количество уравнений и неизвестных, то привести все возможные “упрощенные формы”, как было сделано для системы двух линейных уравнений с двумя неизвестными, при использовании большой системы уже довольно сложно. Удобнее дать определение общей “упрощенной формы”, которая называется приведенной матрицей (reduced matrix) и годится для всех матриц и систем, независимо от их размера. Приведенная матрица Матрица называется Приведенной или записанной в приведенной форме в следующих ситуациях. 1. Каждая строка, состоящая полностью из нулей, находится ниже строки, содержащей по крайней мере один отличный от нуля элемент. 2. Крайний слева ненулевой элемент строки равен единице. 3. Все остальные элементы в столбце, содержащем единицу (крайний слева ненулевой элемент данной строки), равны нулю. 4. Крайний слева единичный элемент в данной строке находится правее, чем крайний слева единичный элемент в Предыдущей строке. Следующие матрицы представлены в приведенной форме. Убедитесь в том, что они удовлетворяют определению, сформулированному выше. 1 0 2 0 1 -3 1 4 0 0 0 1 ООО 1 0 0 0 10 0 0 1 0 -3 " 0 2 , 1 6 3 -1 0
316 Часть II. Конечная математика Пример 4.12 (Приведенные матрицы). Перечисленные ниже матрицы не являются приведенными. Определите, какой из пунктов определения приведенной матрицы нарушается в каждом случае. Найдите последовательность операций над строками, которые преобразуют каждую матрицу к приведенной форме. 0 1 —2 ' 'У ’02-2 3 ' 1 0 3 L. 1 0 1 -1 " 1 0 -3 ■ ■ 1 0 0 -1 ■ 3. 0 0 0 4. 0 2 0 3 0 1 —2 0 0 1 -5 Решение. 1. Нарушается условие 4: во второй строке крайний слева ненулевой элемент не расположен правее ненулевого элемента в первой строке. С помощью операции Ri <-► R2 получаем следующее. ’ 1 0 3 ' ° 1 —2 2. Нарушается условие 3: во втором столбце над крайней слева единицей, стоящей во второй строке, находится отличный от нуля элемент. После строковой операции 2Я2 + Я1 —► Я1 получаем следующую матрицу. 12 0 1 0 0 1-1 3. Нарушается условие 1: во второй строке все элементы равны нулю, а в следующей строке — нет. После строковой операции Я2 <-► R3 получаем следующую матрицу. 1 0 -3 0 1 —2 0 0 0 4. Нарушается условие 2: во второй строке крайний слева отличный от нуля элемент не равен единице. После строковой операции |/?2 —► Я2 получаем следующую матрицу. ’ 1 0 0 -1 ■ 0 1 0 3 2 0 0 1 -5 Упражнение 4.12. Перечисленные ниже матрицы не являются приведенными. Определите, какой пункт определения приведенной матрицы нарушается в каждом случае. Найдите операцию или операции над строками и преобразуйте каждую матрицу к приведенной форме. 0 1 0 -3 1 2 0 3 3. 1 0 0 0 4. 0 0 0 0 0 0 1 2 0 0 1 4 ■
Глава 4. Системы линейных уравнений и матрицы 317 Решение систем уравнений методом исключения Гаусса-Жордана Теперь мы полностью готовы рассмотреть схему процесса исключения Гаусса-Жордана. В этом методе решения расширенная матрица, представляющая систему линейных уравнений, преобразовывается к приведенной форме. Система уравнений, которая соответствует приведенной расширенной матрице, называется приведенной системой уравнений. Как мы узнаем ниже, приведенные системы довольно просто решать. Название метода исключения Гаусса походит от имени немецкого математика Карла- Фридриха Гаусса (1777—1885) (Karl Friedrich Gauss). Гаусс, один из величайших математиков своего времени, использовал метод решения систем уравнений для астрономических задач. Позднее этот метод был обобщен немецким геодезистом Вильгельмом Жорданом (1842-1849) (Wilhelm Jordan)2 и применен для решения задач, связанных с крупномасштабными измерениями. Пример 4.13 (Решение системы уравнений методом исключения Гаусса-Жордана). Решите систему уравнений методом исключения Гаусса-Жордана. 2x1 - 2х2 + я3 = 3, 3xi + ^2 — = 7, Xi — Зх2 + 2хз = 0. Решение, Этап 1. Выберем отличный от нуля столбец был равен единице сделаем так, чтобы его верхний элемент Здесь должна стоять единица -2 1 -3 2 3 1 1 -1 2 и ► Лз Этап 2. Подберем множитель для строки, содержащей единицу (этап 1), и сделаем так, чтобы в столбце, Содержащем эту единицу, остальные элементы были равны -3 2 0 1 -1 7 —2 1 3 —ЗЯ1 + R2 —► R2 —2R\ + Д3 —> R3 Этап 3. Повторим процедуру этапа 1 для подматрицы, которая получается вычеркиванием (воображаемым) первой строки О.1Я2 —► R2 Этап 4. Повторим этап 2 для полной матрицы ЗЛ2 Ч” Ri —► Ri —4R2 + R3 —> R& 2Не путайте немецкого геодезиста Вильгельма Жордана (точнее, Йордана) с французским математиком Камилем Жорданом (Camille Jordan). — Прим. ред.
318 Часть II. Конечная математика Этап 5. Повторим процедуру этапа 1 для подматрицы, которая получается вычеркиванием (воображаемым) первых двух строк Здесь должна стоять единица ":Т •о о о -од 1 -0,7 I 0,7 : 0 —0,2 0,2 2,1!" —5Нз —► К3 Этап 6. Повторим этап 2 для полной матрицы ОДЛ2 И- -Ri —► Ri 0,7Лз + R2 —► R2 Теперь можно перейти к решению приведенной системы 1 0 0 2 0 1 0 0 0 0 1 -1 £1 = 2, Х2 = 0, хз = -1. Решением последней системы является xi = 2, Х2 = 0 и х$ = — 1. Читатель может убедиться самостоятельно, что этот набор является решением исходной системы. ■ Процесс исключения Гаусса-Жордана Этап 1. Найти крайний слева отличный от нуля столбец и использовать операции над строками для получения верхнего элемента, равного единице. Этап 2. Подобрать множители для строки, содержащие единицу (этап 1), и преобразовать все элементы столбца, содержащего эту единицу, так, чтобы они были равны нулю. Этап 3. Повторить этап 1 для подматрицы, которая получается вычеркиванием (воображаемым) строки этапа 2 и всех строк, расположенных выше нее. Этап 4. Повторить этап 2 для полной матрицы, в которую входят “вычеркнутые” строки. (Замечание. Если на каком-либо этапе этого процесса получается строка с нулевыми элементами до вертикальной черты и отличным от нуля элементом после нее, то можно далее не преобразовывать матрицу к приведенной форме, поскольку получено противоречие вида 0 — п, п 7^ 0. Это значит, что система не имеет решений.) Замечания. 1. Несмотря на то что каждой матрице соответствует единственная приведенная форма, последовательность шагов (алгоритм), приводящая матрицу к этому виду, не является единственной. Иначе говоря, существует и другая последовательность шагов (с использованием строковых операций), приводящая к приведенной матрице
Глава 4. Системы линейных уравнений и матрицы 319 (например, вычисления можно организовать так, чтобы уменьшить количество операций с дробями). Впрочем, отметим еще раз, что сейчас мы не будем описывать эффективные методы для преобразования небольших матриц к приведенному виду при вычислениях вручную. Основной нашей целью является получение представления об общих методах использования их для решения больших систем с помощью графической утилиты, Большинство графических утилит имеет возможность вычислять приведенные матрицы или непосредственно или с помощью небольшой подпрограммы. На рис. 4.7 показано решение примера 4.13 на графическом калькуляторе, в котором есть встроенная программа преобразования матриц к приведенной форме. Обратите внимание на то, что элемент на пересечении второй строки и четвертого столбца в приведенной матрице после вычисления на графическом калькуляторе оказался равным очень малому числу —3,5Е — 13, тогда как точное значение — нуль. Такая ситуация характерна для графических калькуляторов и на самом деле проблем не вызывает, Нужно только заменить очень маленькие числа в научных обозначениях на 0. Рис. 4.7. Применение метода исключения Г аусса-Жордана на графическом калькуляторе Упражнение 4.13. Решите систему уравнений методом исключения Гаусса-Жордана. 3x1 + х2 - 2х3 = 2, Х\ - 2х2 + а?3 = 3, 2x1 — ^2 — Зх3 = 3. Пример 4.14 (Решение системы уравнений методом исключения Гаусса-Жордана). Решите систему уравнений методом исключения Гаусса-Жордана. 2xi ~ 4х2 + = -4, 4xi — 8x2 + 7хз = 2, -2xi + 4х2 - Зх3 = 5.
320 Часть II. Конечная математика Решение. 2 —4 1 —4 ' 0,5-Ri —► R± 4 -8 7 2 —2 4 -3 5 1 —2 0,5 —2 4 -8 7 2 —4Л1 4“ Т?2 —► Rz —2 4 -3 5 2R± + Лз —► -Из f —2 0,5 —2;' 0 0 5 10 0,21^2 —► Rz Заметим, что третий столбец является 0 0 —2 1 _ крайним слева отличным от нуля столб¬ цом в этой подматрице. 1 —2 0,5 —2 ’ —0,5К2 “h Ri —► Ri 0 0 1 2 0 0 —2 1 _ 2R2 + -R3 —► -R3 1 —2 0 -3 ' 0 0 1 2 Здесь процесс Гаусса-Жордана можно прервать, посколь¬ 0 0 0 5 ку в последней строке мы получили противоречие. ■ Упражнение 4.14. Решите систему уравнений методом исключения Гаусса-Жордана. 2xi — 4хг — хз = — 8, 4xi ~ 8x2 + Зхз = 4, —2x1 + 4x2 + х3 = 11. Предупреждение. ю. На рис. 4.8 показано решение примера 4.14 на графическом калькуляторе с помощью встроенной программы. Заметим, что программа не останавливается, когда возникает противоречие, и продолжает вычисления. Не путайте приведенную форму с совместной системой (см. рис. 4.7). А [ 12 14 [ -2 rref А -4 1 -8 7 4 -4J 2 J -3 5 11 [ [1 [0 (0 -2 0 01 0 1 0] 0 0 111 Рис. 4.8. Возникновение противоречий при работе с графическим калькулятором
Глава 4. Системы линейных уравнений и матрицы 321 Пример 4.15 (Решение системы уравнений методом исключения Гаусса-Жордана). Решите систему уравнений методом исключения Гаусса-Жордана. Решение. 3xi -I- 6x2 — 9хз = —15, 2xi + 4x2 — бхз = 10, -2X1 — Зх2 + 4х3 = -6. 3 6 -9 15 ‘ —► -Ri 2 4 -6 10 _ —2 -3 4 -6 1 2 -3 5 ‘ 2 4 -6 10 —2Й1 -|- J?2 —► -R2 -2 -3 4 -е 22?i + -R3 —► 7?з 1 2 -3 5 ‘ Поменять местами вторую и третью стро- 0 0 0 0 _ ки нужно, чтобы получить отличный от нуля XL2 -гьз * 0 1 -2 4 _ матричный элемент сверху во втором столб¬ це подматрицы. 1 2 -3 5 ‘ —2Я2 + Ki —► R± 0 1 -2 4 0 0 0 0 _ 1 0 1 -3 ' Теперь матрица представлена в приведенной форме. Оста¬ 0 1 -2 4 ется записать соответствующую ей систему и найти ее ре¬ 0 0 0 0 _ шение. Х1 + %з = —3, Уравнение, соответствующее третьей строке, удовлетворяется произвольными ®i, ®2, ®з, поэтому его можно пропу¬ Х2 ~ 2хз - 4. стить. Заметим, что крайняя слева переменная в каждом из уравнений фигурирует в одном и только в одном уравнении. Это значит, что крайние слева переменные xi и Х2 можно выразить через оставшуюся переменную х3. xi = —х3 — 3, х2 = 2х3 + 4. Если мы положим хз » где t — произвольный действительный параметр, то xi = — t — 3, Х2 = 2t + 4, хз = t. Легко проверить, что (—t ~ 3, 2£ + 4, t) является решением исходной системы для произвольного действительного параметра t t а 0 t = -2 t = 3,5 (-3; 4; 0), (-1; 0; -2), (-6,5; И; 3,5). а
322 Часть II. Конечная математика Если количество крайних слева единиц в приведенной расширенной матрице меньше, чем количество неизвестных, и в системе уравнений нет противоречий, то такая система является зависимой и имеет бесконечное множество решений. Описать множество решений таких систем достаточно просто. В приведенной системе крайние слева переменные соответствуют крайним слева единицам в приведенной расширенной матрице. В определении приведенной формы расширенной матрицы подразумевается, что каждая крайняя слева переменная в соответствующей системе появляется в одном и только одном уравнении. Выразить каждую крайнюю слева переменную через оставшиеся переменные и записать общее решение системы довольно просто. (В примере 4.16 рассматривается более запутанный случай.) Упражнение 4.15. Решите систему уравнений методом исключения Гаусса-Жордана. 2^1 — 2x2 — 4хз = —2, 3xi — 3x2 — бхз = —3. —2x1 3x2 4- хз = 7. ■ Задание 4.6. Объясните, почему в определении приведенной формы гарантируется, что каждая крайняя слева переменная в приведенной системе фигурирует в одном и только в одном уравнении, и в каждом уравнении содержится только одна крайняя слева переменная. Обсудите метод, с помощью которого на основе анализа приведенной формы можно определить тип системы (зависимая, независимая). ■ Пример 4.16 (Решение системы уравнений методом исключения Гаусса-Жордана). Решите систему уравнений методом исключения Гаусса-Жордана. Х1 4- 2х2 4- 4х3 + Х4 - Х5 = 1, 2xi 4- 4x2 4- 8x3 4- Зхд — 4x5 — 2, Xi 4- 3x2 4~ 7хз 4" 3x5 = —2. Решение. ■ 1 2 4 1 -1 1 ■ 2 4 8 3 —4 2 —27?1 -|- R2 —> -R2 _ 1 3 7 0 3 —2 —Ri 4" R'3 —► R3 " 1 2 4 1 -1 1 ' 0 0 0 1 —2 0 R2 R3 _ 0 1 3 -1 4 -3 ’ 1 2 4 1 -1 1 ‘ —27^2 4~ Ri —► Ri 0 1 3 -1 4 -3 0 0 0 1 —2 0
Глава 4. Системы линейных уравнений и матрицы 323 ■ 1 0 —2 3 -9 7 ' 0 1 3 -1 4 -3 0 0 0 1 -2 0 ■ 1 0 -2 0 -3 7 ‘ 0 1 3 0 2 -3 0 0 0 1 -2 0 *1 —ЗЯ3 -|- Bi —► Ri R3 + R2 —► R2 Теперь матрица представлена в приведенной форме. - 2х3 £2 + Зхз - 3^5 = 7, + 2x5 = — 3, Х4 — 2x5 — О- Выразим неизвестные xi, Х2 и Х4 через оставшиеся переменные хз и Х5: Xi = 2хз + 3x5 + 7, $2 = ^Зхз — 2X5 ~ 3, Х4 = 2х5. Если положить хз = в и х& — t, где в и t — произвольные действительные числа, то получим вектор, являющийся решением системы. Xi = 2s + 3t + 7, Х2 = —3$ — 2t — 3, х3 = s, Х4 = 2t, Х5 = t. Проверка предоставляется читателю. Упражнение 4.16. Решите систему уравнений методом исключения Гаусса-Жордана. Х1 - Х2 + 2х3 - 2х5 = 3, -“2x1 4- 2хг — 4х3 — Х4 + Хб = —5, 3X1 — 3x2 + 7хз + Х4 — 4x5 = 6. Решение практических задач И Зависимые системы линейных уравнений позволяют обсудить вопросы математического моделирования более подробно. В общем случае процедура решения практических задач состоит из трех этапов (рис. 4.9). Этап 1. Постройте математическую модель, решение которой даст информацию о практической задаче. Этап 2. Решите математическую модель. Этап 3. Интерпретируйте решение математической модели на языке практической задачи. При решении сложных задачах эта процедура может повторяться по несколько раз.
324 Математическое решение Часть II. Конечная математика Практическая задача 2. Решить Математическая модель Рис. 4.9. Процедура решения задач WWW Пример 4.17 (Покупка). Компания, сдающая в аренду небольшие грузовики, намерена купить 25 грузовиков общей вместимостью 28 000 куб. футов. Существует три типа грузовиков разной длины: 10-футовые грузовики вместительностью 350 куб. футов, 14-футовые грузовики вместительностью 700 куб. футов и 24-футовые Решение. Введем следующие обозначения. xi — количество 10-футовых грузовиков, Х2 — количество 14-футовых грузовиков, хз — количество 24-футовых грузовиков. Теперь сформулируем математическую модель: Х1 + х2 + Хз = 25, 350x1 -I- 700х2 + 1400х3 = 28000. Общее количество грузовиков Суммарная вместимость (4.15) Построим расширенную матрицу и решим систему методом исключения Гаусса- Жордана. 1 1 1 25 ' 350 700 1400 28000 1 1 1 25 ‘ 1 2 4 80 1 1 1 25 ' 0 1 3 55 350 —Ri -J- R2 —► R2 —R2 + Ri —> -Ri
Глава 4. Системы линейных уравнений и матрицы 1 0 -2 -30 ' 0 1 3 56 Матрица представлена в приведенной форме - 2x3 = —30 или xi = 2х3 — 30, ■4- Зхз = 55 или х2 = — Зхз + 55. Положим хз = t, где t — произвольное действительное число. xi = 2t — 30, Х2 = —+ 55, хз = t 325 (4.16) Вектор (xi, д?2, хз) является решением математической модели (4.15). Интерпретируем полученное решение на языке исходной задачи. Поскольку переменные xi, Х2 и хз описывают количество грузовиков разного типа, эти числа не могут быть отрицательными» Более того, эти значения не должны быть дробными, каждое из них — неотрицательное целое число. Поскольку t = я3, то число t, очевидно, является неотрицательным целым числом. Другие ограничения на возможные значения параметра t даются первым и вторым уравнениями (4.16). xi = 2t — 30 > 0 подразумевает, что t 15, 55 1 Х2 = ~-3£ + 55 > 0 подразумевает, что t — = 18-. о о Таким образом, задача имеет осмысленные решения при значениях параметра 15, 16, 17 и 18. Иначе говоря, единственная комбинация 25 грузовиков, обеспечивающая суммарную вместительность, равную 28 000 куб. футов, определяется формулами: х^ = 2t — 30, Х2 = = —32 + 55 > 0 и я3 = t, где параметр t может принимать значения 15, 16, 17 и 18. Для представления конечных результатов удобно воспользоваться таблицей. 10-футовые грузовики 14-футовые грузовики 24-футовые грузовики t «1 Х2 Хз 15 0 10 15 16 2 7 16 17 4 4 17 18 6 1 18 Упражнение 4Л 7. WWW Компания, сдающая в аренду небольшие грузовики, намерена купить 16 грузовиков общей вместимостью 19200 куб. футов. Существует три типа грузовиков: грузовые микроавтобусы вместительностью 300 куб. футов, 15-футовые грузовики вместительностью 900 куб. футов и 24-футовые грузовики вместительностью 1500 куб. футов. Сколько грузовиков каждого типа может купить компания? ■ Задание 4.7. Вернемся к примеру 4.16» Расценки за аренду транспорта таковы: 19,95 долл, в день за 10-футовый грузовик, 29,95 долл, в день за 14-футовый грузовик и 39,95 долл, в день за 24-футовый грузовик. При каком выборе в таблице дневной доход компании будет максимальным? ■
326 Часть II. Конечная математика Ответы к упражнениям 4.12. 1) Нарушено условие 1. Вместо числа 3 на пересечении второй строки и второго столбца должна стоять единица. После преобразования |/?2 —► R2 получаем следующую матрицу. Г 1 0 2 ' 0 1-2 2) Нарушено условие 3. Вместо числа 5 на пересечении первой строки и второго столбца должен стоять нуль. После преобразования — 57?2 + получаем следующую матрицу. ■ 1 0 -6 8 ‘ 0 1 2 -1 0 0 0 0 3) Нарушено условие 4. Крайняя слева единица во второй строке не расположена правее крайней слева единицы в первой строке. После преобразования R\ <-* R2 получаем следующую матрицу. 10 0 0 0 10-3 0 0 1 2 4) Нарушено условие 1. Полностью состоящая из нулей вторая строка должна быть внизу. После преобразования R2 R3 получаем следующую матрицу. 12 0 3 0 0 14 0 0 0 0 4.13. xi = 1, х2 = —1, хз = 0. 4.14. Несовместная, решений нет. 4.15. xi = 5t + 4, х2 = 3t + 5, Х3 = t, где t — произвольное действительное число. 4.16. д?1 = s + 7, х2 = 5, хз = t — 2, Х4 = —3t — 1, х5 = t, где t и s — произвольные действительные числа. 4.17. t — 8 грузовых микроавтобусов, — 2t + 24 15-футовых грузовиков и t 24-футовых грузовиков, где t может принимать значения 8, 9, 10, И или 12. Практикум 4.3 А В задачах 1-10 требуется определить, представлены ли перечисленные матрицы в приведенной форме. Если нет, то необходимо найти операцию (операции) над строками, с помощью которой (которых) матрица преобразуется к приведенной форме.
Глава 4. Системы линейных уравнений и матрицы 327 ’ 1 О *3. О О О 1 ■ О 1 *5. О О О О ■ 1 1 *7. О О _ О О *9. ■ 1 О О О ■ 1 О О —2 ' *4. О 1 О О О О 1 1 _ ■ 1 2 -3 1' *6. О О L 4 О О О О ’ 1 0 -1 3 ‘ *8. 0 2 1 1 _ 0 0 0 0 *10. ■ 1 —2 0 0 1 ■ 0 0 1 1 0 В задачах 11-18 запишите системы линейных уравнений, которые соответствуют перечисленным приведенным матрицам, и решите их. И. -2 3 О 12. 1 о о о о 1 о о о о 1 о о о о 1 —2 о 1 3 ■ 1 0 -2 3 ‘ ■ 1 -2 0 -3 ' 13. 0 1 1 -5 14. 0 0 1 5 0 0 0 0 0 0 0 0 ■ 1 0 0 ‘ ■ 1 0 5 ' 15. 0 1 0 16. 0 1 -3 0 0 1 0 0 0 17. —2 О О 1 1 о -3 3 1 СП 1 1 я ’ 1 0 —2 3 4 ‘ 2 1о. 0 1 -1 2 -1 1 о о о 1 о О О 1 Б В задачах 19-24 с помощью операций над строками преобразуйте перечисленные матрицы к приведенной форме. 19. ’ 1 0 2 1 -1 ■ 3 • 20. ' 1 0 3 2 1 ' -4 ' 1 0 -3 1 ‘ ’ 1 0 4 0 ' 21. 0 1 2 0 22. 0 1 -3 -1 _ 0 0 3 -6 _ _ 0 0 —2 2 ■ 1 2 -2 -1 ■ 0 2 8 1 ' 23. 0 3 -6 1 24. 2 2 6 -4 0 — 1 2 _1 3 0 — 1 4 1 2 _ В задачах 25-44 требуется решить системы уравнений методом исключения Гаусса- Жордана. 25. 2a?i + 4x2 — Юхз *= —2, 3xi + 9x2 — 21ггз = О, Х1 + 5X2 — 12хз а» 1. 26. 3xi + 5x2 — хз = —7, Х1 + Х2 + Хз - -1, 2xi 4" Ихз = 7.
328 Часть II. Конечная математика 27. 3xi + 8^2 — хз = —18. 2xi 4- х2 4- 5х3 = 8, 2xi + 4х2 -1- 2х3 = -4. 28. 2xi 4- 6х2 + 15хз = —12, 4xi 4- 7х2 4- 13х3 = -10, 3xi 4~ 6х2 4~ 12хз = —9. 29. 2xi — х2 — Зхз = 8, xi — 2х2 = 7. 30. 2xi 4- 4х2 - 6х3 = 10, 3xi 4- Зх2 — Зхз = 6. 31. 2xi — ^2 = 0, 3xi + 2х2 = 7, Х1 — х2 = —1. 32. 2xi ~ х2 = 0, 3xi 4- 2х2 = 7, xi - х2 = -2. 33. 3xi — 4х2 — хз = 1, 2xi — Зх2 + х3 = 1, xi — 2х2 + Зхз = 2. 34. 3xi 4- 7х2 — х3 = 11, xi 4- 2х2 — Хз = 3, 2xi 4- 4х2 — 2хз = 10. 35. 3xi — 2х2 + Хз = —7, 2xi 4- х2 - 4х3 = 0, Xi + х2 — Зхз = 1. 36. 2xi 4- Зх2 4- 5х3 = 21, Xi Х2 6x3 — 2, 2xi 4- х2 - х3 = И. 37. 2xi + 4х2 — 2хз = 2, —3x1 — бх2 -|- Зхз = —3. 38. 3xi - Эх2 4- 12х3 = 6, —2x1 4- 6х2 — 8x3 = —4. 39. 4xi — х2 4- 2хз = 3, -4xi + - Зх3 = -10, 8xi - 2х2 4- 9х3 = -1. 40. 4xi ~ 2х2 4- 2х3 = 5, —6x1 4- Зх2 — Зхз = —2, lOxi — 5х2 4- 9хз = 4. 41. 2xi ~ 5х2 — Зхз = 7, —4x1 4- Юх2 4- 2х3 = 6, 6x1 — 15х2 — хз — —19. 42. -4xi 4- 8х2 4- Юх3 = -6, 6xi — 12х2 - 15хз = 9, —8x1 4- 14х2 4- 19хз = —8. 43. 5xi - Зх2 4- 2хз = 13, 2xi — х2 — Зх3 = 1, 4xi — 2х2 4- 4хз = 12. 44. 4xi — 2х2 4- Зхз = 3, 3xi — х2 — 2хз = —10, 2xi4-4x2- хз = -1. * 45. Рассмотрите систему трех линейных уравнений с тремя неизвестными. Обсу¬ дите свойства системы и ее решений, если в приведенной форме расширенная матрица удовлетворяет следующим условиям. а) Содержит одну крайнюю слева единицу. б) Содержит две крайние слева единицы. в) Содержит три крайние слева единицы. г) Содержит четыре крайние слева единицы. * 46. Рассмотрите систему трех линейных уравнений с тремя неизвестными. Приве¬ дите по два примера приведенных неэквивалентных форм, если система удовлетворяет следующим условиям. а) Является совместной и зависимой. б) Является несовместной.
Глава 4. Системы линейных уравнений и матрицы 329 В задачах 47-50 определите связь параметра к и количества решений системы уравнений. *47. Xi - #2 == 4, 3xi + кх2 ~ 7. *48. Xi -и 2х2 = 4, —2x1 4- кх2 = —8. *49. xi 4- кх2 — 3, 2^1 + 6X2 =« 6. *50. xi 4- кх2 = 3, 2xi 4- 4x2 = 8. В В задачах 51-56 требуется решить системы уравнений методом исключения Гаусса- Жордана. 51. xi + 2x2 4хз — Х4 = 7, 2xi + 5х2 — 9хз “ 4x4 — 16, xi -1- 5x2 — 7хз — 7x4 = 13. 52. 2xi 4- 4x2 + 5х3 4- 4x4 = 8, xi 4- 2x2 4- 2хз 4- Х4 = 3. 53. Х1 — Х2 + Зхз — 2x4 = 1) —2x1 + 4x2 Зхз 4- х< = 0,5; 3xi — х2 + Юхз — 4x4 = 2,9; 4xi ~ 3x2 + 8X3 — 2x4 = 0,6. 54. xi 4- Х2 + 4х3 + х4 = 1,3; -xt 4- х2 - х3 = 1,1; 2xi 4- Х3 + 3x4 = —4,4; 2xi 4- 5x2 4- Ихз 4- ЗХ4 = 5,6. 55. Х1 - 2X2 + #3 + + 2X5 = —2x1 + 4x2 + 2хз + 2x4 — 2xs = 3X1 — 6X2 + #3 4* Х4 4- 5x5 = —xi 4- 2x2 + Зхз 4м Х4 4- х$ = 2, 0, 4, 3. 56. xi — 3x2 4- х3 4- Х4 + 2х5 = 2, —Xi 4- 5x2 4- 2х3 4- 2x4 — 2x5 = 0, 2xi — 6x2 4- 2х3 + 2x4 4- 4x5 = 4, -Xi 4- Зх2 - хз - Х5 = -3. Применение математики В каждой из задач постройте математическую модель. (В ответах в конце книги приведена как математическая модель, так и интерпретация ее решения.) Решите уравнения модели методом исключения Гаусса-Жордана и дайте интерпретацию полученному решению. Экономика и бизнес * 57. Планирование производства. Небольшое предприятие производит надувные лодки трех типов: одно-, двух- и четырехместные. Для изготовления каждой лодки задействованы три отдела, перечисленные в таблице. В отделах резки, сборки и упаковки затрачиваются максимум 380, 330 и 120 рабочих часов в неделю соответственно. а) Сколько лодок Каждого типа будет производить предприятие, работая в полную МОЩНОСТЬ? б) Как изменится план производства, если не задействован отдел упаковки? в) Как изменится план производства, если производство четырехместных лодок прекратить?
330 Часть II. Конечная математика Отдел Одноместная лодка, ч Двухместная лодка, ч Четырехместная лодка, ч Резки 0,5 1,0 1,5 Сборки 0,6 0,9 1,2 Упаковки 0,2 0,3 0,5 * 58. Планирование производства. Повторите решение задачи 57, если в отделах рез¬ ки, сборки и упаковки затрачиваются максимум 350, 330 и 115 рабочих часов в неделю соответственно. 59. WWW Аренда транспорта. Химическое предприятие намерено арендовать 24 железнодорожные цистерны суммарной вместимостью 520 000 галлонов. Имеется три типа цистерн: вместимостью 8000, 16000 и 24000 галлонов. Сколько цистерн каждого типа может арендовать предприятие? 60. WWW Аренда транспорта. Корпорация намерена арендовать парк из 12 самолетов суммарной вместимостью 220 пассажиров. Имеется три типа самолетов: вместимостью 10, 15 и 20 пассажиров. Сколько самолетов каждого типа может арендовать корпорация? * 61. WWW Аренда техники. Вернемся к задаче 59. Стоимость аренды цистерны зави¬ сит от ее вместимости. Для цистерны на 8000 галлонов она составляет 450 долл, в месяц, для цистерны на 16000 галлонов — 650 долл, в месяц, а для цистерны на 24 000 галлонов — 1150 долл, в месяц. При каком выборе решения в задаче 59 расходы на аренду будут минимальными? * 62. WWW Аренда техники. Вернемся к задаче 60. Стоимость аренды зависит от вме¬ стимости самолета. Если самолет имеет 10 мест, его аренда равна 8000 долл, в месяц, если 15 мест — 14000 долл, в месяц, а если 20 мест — 16000 долл, в месяц. При каком выборе решения в задаче 60 расходы на аренду будут минимальными? 63. Подоходный налог. Корпорация имеет облагаемую налогом прибыль в размере 7 650000 долл. При таком уровне доходов федеральная налоговая ставка составляет 50%, налоговая ставка штата — 20% и местная налоговая ставка — 10%. Каждая из этих ставок относится к общему доходу, так что суммарный подоходный налог корпорации составляет 80%. Однако обычно уплаченный корпорацией налог вычитают из общей суммы еще до того, как будет определен налоговый взнос в другие фонды. Допустим, что федеральный налог платится с суммы прибыли после того, как из нее уже вычтены налоги штата и местные налоги, причем последние определяются аналогичным способом. Какие налоговые обязательства имеет корпорация (в процентах от суммы дохода) при таком способе уплаты? 64. Подоходный налог. Повторите решение задачи 63, если при уплате федерального налога и налога штата сумма местного налога не вычитается из общего дохода. 65. Доход, облагаемый налогом. В результате слияний и приобретений четыре компании распределили свои фонды. В приведенной ниже таблице в каждой строке представлен процент от общего капитала, принадлежащий каждой компании, и ее годовой доход (в миллионах долларов).
Глава 4. Системы линейных уравнений и матрицы 331 Так, компания А владеет 71 % капитала, компания Б — 8%, компания В — 3% и т.д. Для определения суммы налога штата доходы каждой из компаний определены как доля в собственных годовых доходах плюс доля в доходах других компаний, как показано в таблице. Какой облагаемый налогом доход имеет каждая компания (в Миллионах долларов)? Компания Доля общего капитала, принадлежащая каждой компании, % Годовой доход, - млн. долл А Б В Г А 71 8 3 7 3,2 Б 12 81 11 13 2,6 В 11 9 72 8 3,8 Г 6 2 14 72 4,4 66. Доход, облагаемый налогом. Повторите решение задачи 65, если правило налога изменено следующим образом. Доходы каждой из компаний определены как весь собственный годовой доход плюс доля в доходах других компаний. Биологические науки * 67. Диета. В больнице разработана специальная диета на основе трех базовых блюд. Диета должна обеспечивать пациентов 340 единицами кальция, 180 единицами железа и 220 единицами витамина А. Количество единиц на унцию каждого компонента для трех блюд приведены в таблице. а) Сколько унций каждого блюда можно использовать для соблюдения условий диеты? б) Как изменится ответ на п. а, если блюдо В не используется? в) Как изменится ответ на п. а, если витамин А в требованиях диеты пропущен? Единиц на унцию Блюдо А Блюдо Б Блюдо В Кальций 30 10 20 Железо 10 10 20 Витамин А 10 30 20 * 68. Диета. Повторите решение задачи 67, если диета должна содержать 400 единиц кальция, 160 единиц железа и 240 единиц витамина А. 69. Удобрение растений. Фермер может приобрести четыре вида удобрительных смесей. В каждом барреле смеси А содержится 30 фунтов фосфорных, 50 фунтов азотных и 30 фунтов калийных удобрений; в барреле смеси Б содержится 30 фунтов фосфорных, 75 фунтов азотных и 20 фунтов калийных удобрений; в барреле смеси В содержится 30 фунтов фосфорных, 25 фунтов азотных и 20 фунтов калийных удобрений; и, наконец, в каждом барреле смеси Г содержится 60 фунтов фосфорных, 25 фунтов азотных и 50 фунтов калийных удобрений. Тестирование показало, что для типичного поля требуется 900 фунтов фосфорных, 750 фунтов азотных и 700 фунтов калийных удобрений. По сколько баррелей каждой смеси нужно использовать для удобрения поля, учитывая рекомендуемые количества?
332 Часть II. Конечная математика 70. Рацион питания. В лабораторном эксперименте крысы за раз съедают 5 пакетов корма, содержащих суммарно 80 единиц витамина Е. Используются четыре различных смеси. В пакете смеси А содержится 5 единиц витамина Е; в пакете смеси Б содержится 10 единиц витамина Е; в пакете смеси В содержится 15 единиц витамина Е; и в пакете смеси Г — 20 единиц витамина Е. Сколько пакетов каждого вида смеси можно смешать для обеспечения правильной диеты? Социальные науки 71. Социология. Два социолога располагают достаточной суммой денег для исследования школьной нагрузки в некотором городе. Они намерены составить отчет на основе опроса, взяв при этом 600 телефонных и 400 домашних интервью. Участники группы А могут взять 30 телефонных и 10 домашних, а участники группы Б — 20 телефонных и 20 домашних интервью в час. Сколько часов потребуется каждой группе, чтобы взять нужное количество интервью? 72. Социология. Повторите решение задачи 71, если требуется провести 650 телефонных и 350 домашних интервью. 73. Транспортный поток. На рис. 4.10 показан транспортный поток в городе по четырем односторонним улицам. Числа рядом с каждой улицей описывают количество въезжающих и выезжающих автомобилей в час. Переменные х\, Х2, хз и х± описывают транспортные потоки между пересечениями улиц. 5-я 6-я улица улица 700 600 Вашингтон- 800 А 600 ’ авеню *4 *2 Линкольн- 900 хз 500 авеню 400 500 Рис. 4.10. Иллюстрация к задаче 73 а) При однородном транспортном потоке количество въезжающих на перекресток автомобилей совпадает с количеством выезжающих. Например, если на перекресток 5-й улицы и Вашинггон-авеню каждый час въезжает 1500 автомобилей, то х\ + х± автомобилей покидают перекресток, поэтому х± + х4 = = 1500. Найдите уравнения, которые описывают транспортные потоки через три других перекрестка. б) Решите систему уравнений, полученную в п. а. в) Какое максимальное количество автомобилей может проехать с Вашингтон- авеню на Линкольн-авеню через 5-ю улицу? г) Предположим, что светофоры настроены так, что с Вашингтон-авеню на Линкольн-авеню через 5-ю улицу может проехать 1000 автомобилей. Определите транспортные потоки на остальных участках этой сети дорог.
Глава 4. Системы линейных уравнений и матрицы 333 74. Транспортный поток. Вернемся к задаче 73. На рис. 4.11 показаны изменения транспортного потока, если Вашинггон-авеню перекрыта восточнее 6-й улицы. Повторите решения пп. a-в задачи 73 для этого транспортного потока. 5-я 6-я 700' 800 800 *4’ «1 0 Х1 1100 600 х> 500 500 Линкольн- авеню Вашингтон- авеню Рис. 4.11. Иллюстрация задаче 74 4.4. Матрицы: основные операции ■ Сложение и вычитание ■ Умножение числа к на матрицу М ■ Произведение матриц WWW В двух предыдущих разделах мы рассматривали определение такого нового математического понятия, Как матрица. В настоящей, как и в последующих главах, мы будем развивать концепции, связанные с матрицами. Идеи, связанные с матрицами, с одной стороны довольно старые, а с другой — вполне современные. Упоминания о матрицах и системах уравнений были найдены в китайских рукописях, датированных приблизительно 200 годом до н.э. С недавним изобретением компьютеров матрицы стали успешно использоваться для решения многих практических задач. Как правило, программное обеспечение предусматривает средства для вычислений с матрицами. Как мы увидим далее, сложение и умножение матриц во многом напоминает аналогичные операции с обычными действительными числами, однако между ними есть несколько существенных отличий. Для выявления сходных черт и различий в приложении А.2 приводятся основные операции с действительными числами. Сложение и вычитание Прежде чем рассматривать основные арифметические операции с матрицами, определим понятие равенства матриц. Две матрицы называются равными, если их размеры и соответствующие элементы совпадают. Например: 2X3 2x3 а Ь с _ и v w def х у z
334 Часть II. Конечная математика тогда и только тогда, когда c — w f = Z а = и d = х b = v е = у Суммой двух матриц, имеющих одинаковые размеры, называется матрица, элементы которой являются суммами соответствующих элементов данных двух матриц. Для матриц различных размеров операция сложения не определена. Пример 4.18 (Сложение матриц). 1. а с ' 2 1 а. о- ОО 1 | + г/ 1 0 ' ) X / z . 3 (а + га) (с-1-2/) 1 2 1 - (Ь + я) (d + z) 5 - 2 2 ‘ 2. 1 2 — 5 + -3 2 5 J — —2 4 0 -1 Результат не определен. 3. 5 0-2 1 -3 8 0 —2 7 6 8 Упражнение 4.18. Вычислите сумму матриц. Для решения задач с матрицами можно использовать графические калькуляторы. На рис. 4.12 показано решение примера 4.18, б на графическом калькуляторе. [А] [[2-301 -51J [-3 [А] + [BJ [[5 [-2 4 0J] Рис. 4.12. Решение задачи с помощью графического калькулятора Вычисляя суммы матриц, мы складываем их соответствующие элементы. Из свойств действительных чисел следует, что операция сложения матриц одного размера является коммутативной и ассоциативной. Если А, В и С — матрицы одного размера, то Коммутативность А + В = В + А, Ассоциативность (А + В) + С = А + (В + С).
Глава 4. Системы линейных уравнений и матрицы 335 Матрица, у которой все элементы равны нулю, называется нулевой. Например, матрицы являются нулевыми. (Замечание. Для обозначения нулевой матрицы произвольного размера часто используется упрощенное обозначение “0”.) Матрица, отрицательная по отношению к матрице М, обозначается как —М9 а ее элементы получаются заменой знаков элементов матрицы М. Это значит, что а Ь с d если М = , то — М = —а —Ь —с —d Заметим, что М + (—М) = 0 (нулевая матрица). Если матрицы Аи В одного размера, то вычитание определяется следующим образом. Иначе говоря, вычесть матрицу В из матрицы А значит прибавить к матрице А матрицу, отрицательную по отношению к В. Пример 4.19 (Вычитание матриц). -2 2 ' 3 4 3 -2 5 О —2 О 2 -2 -3 -4 5 -4 2 -4 3 5 Упражнение 4.19. Вычислите разность: [ 2 — 3 5 ] — [ 3 — 2 1 ]. ■ Умножение числа к на матрицу М Произведение числа к И матрицы М обозначается кМ — это матрица, которая получается умножением каждого элемента матрицы М на число к. Пример 4.20 (Умножение матрицы на число). -2 3-10 -2 1 3 0 -1 -2 -6 2 4 -2 0 2 -6 4 0 3 Упражнение 4.20. Вычислите произведение. 1,3 10 0,2 3,5
336 Часть II. Конечная математика Задание 4.8. Произведение действительных чисел можно записать как повторную сумму, если одно из них — положительное целое число. 2а = а + а, За = а 4- а 4- а, 4а = а4-а4-а4-а и так далее. Распространите это представление на случай произведения целого числа к и матрицы М. Приведите примеры, иллюстрирующие ваши выводы. ■ В следующем примере матричные методы используются для решения практической задачи. Е Пример 4.21 (Комиссионный сбор). Миссис Смит и мистер Джонс работают менеджерами по продаже в автосалоне, в котором выставлено всего две модели автомобилей. Август — последний месяц в текущем году, когда продают именно эти модели, в сентябре будут выставлены модели следующего года. Приблизительная оценка ежемесячных продаж выполняется с помощью матриц. Объем продаж в августе Стандарт Премиум 54000 88000 126000 0 Объем продаж в сентябре Стандарт Премиум ’ 228 000 368 000 304000 322 000 (Например, миссис Смит обеспечила 54 000 долл, продажей модели стандартной комплектации в августе, а мистер Джонс — 322 000 долл, продажей премиального автомобиля в сентябре.) 1. Какой общий объем продаж в августе и в сентябре у менеджеров и по каждой модели? 2. На сколько отличаются объемы продаж в августе от продаж в сентябре? 3. Пусть оба менеджера получают 5% комиссионных от продаж. Вычислите комиссионные для них обоих и по каждой модели в сентябре. Решение, 1. А 4- В = 2. В-А = 3. 0,05В = Стандарт Премиум 282 000 456 000 Миссис Смит 430 000 322 000 Мистер Джонс Стандарт Премиум 174000 280 000 Миссис Смит 178 000 322000 Мистер Джонс 0,05-228000 0,05-368000 0,05-304000 0,05-322000 11400 18 400 Миссис Смит 15 200 16 100 Мистер Джонс Е Упражнение 4.21. Повторите решение примера 4.21 для следующих начальных условий. 45 000 77000 190000 345 000 106 000 22 000 J И В~[ 266 000 276 000 о На рис. 4.13 показано решение примера 4.21 на электронной таблице.
337 Глава 4. Системы линейных уравнений и матрицы 1 2_ 3 4 I 1 5 6 1 1 7 1 August Sabs September Sales September Commissions 2 Compact Luxury Compact Luxury Compact Luxury 3 Smith $54.000 $88,000 $228.000 $368,000 $11 ,400 $18,400 4 Jones $126,000 to $304,000 $322,000 $15,200 $16,100 5 Combined dales Sales Increase 6 Smith $282.000 $456.000 $174,000 $280,000 7 Jones $430.000 $822.000 $178,000 $322,000 Рис. 4.13. Решение задачи с помощью электронной таблицы Произведение матриц Теперь перейдем к определению произведения матриц. На первый взгляд оно может показаться странным. Несмотря на кажущуюся странность, такое определение органично для общей Теории матриц и очень полезно для решения многих практических задач. Впервые произведение матриц было определено в работах английского математика Артура Кэли (Arthur Cayley, 1821—1895) при исследовании систем линейных уравнений и их решений. В разделе 4.6 мы увидим, что произведение матриц является основным понятием в процедуре представления систем линейных уравнений в виде матричных уравнений и при решении матричных уравнений. Матричные уравнения и их решения — это альтернативный подход К решению систем линейных уравнений в случае, если количество неизвестных совпадает с количеством уравнений. Сначала дадим определение двух специальных видов матриц — строки и столбца. Произведение строки и столбца Произведение 1 х п Матрицы-строки и п х 1 матрицы-столбца является матрицей 1x1 и определяется как г 1 ХП 1 [ aj а2 • • • ап J п х 1 “ Ь1 ' &2 — [а1^1 “Ь <^2^2 + ‘ ‘ + ®п^п] ♦ Ьп Заметим, что произведение строки и столбца определено, если количество элементов в строке совпадает с количеством элементов в столбце. Пример 4.22 (Произведение строки и столбца). [2 -3 0 ] -Б 2 = [2 • (-5) + (-3) • 2 + 0 • (-2)] = [-10 - 6 + 0] = [-16]. -2
338 Часть II. Конечная математика Упражнение 4.22. [-1 0 3 2 ] 2 3 4 Вернемся к примеру 4.22. Отличие действительного числа —16 от матрицы [—16] размером 1x1 является чисто формальным, обычно матрицы размером 1x1 записывают как действительные числа без скобок. Ниже мы будем записывать такие матрицы как числа, пропуская скобки в тех случаях, когда это будет уместно. Пример 4.23 (Затраты на рабочую силу). Производство пары водных лыж занимает три рабочих часа в цехе сборки и один рабочий час в цехе отделки. Сотрудники цеха сборки получают девять долларов в час, а цеха отделки — шесть долларов в час. Тогда величину затрат на производство одной пары лыж можно выразить следующим произведением. [3 1] = [3 • 9 + 1 • 6] = [27 + 6] = [33], или 33 доллара за пару. В Упражнение 4.23. Предположим, что на фабрике из примера 4.23 изготавливаются также спортивные водные лыжи, на производство пары которых уходит пять рабочих часов в цехе сборки и 1,5 рабочего часа в цехе отделки. Запишите произведение матриц, которое описывает величину затрат на производство спортивных водных лыж. Вычислите эту величину. ■ Теперь распространим определение произведения строки и столбца на более общий случай. Произведение матриц Если А — матрица т х р и В — матрица р х п, то произведением матриц Ли В (обозначается АВ) называется матрица т х п, у которой элемент на пересечении г-й строки и j-ro столбца равен произведению г-й строки матрицы А и j-ro столбца матрицы В. Если количество столбцов матрицы А не совпадает с количеством строк матрицы В, то матричное произведение АВ не определено. Прежде чем вычислять произведение, нужно проверить соответствие размеров матриц. Если А является матрицей а х 6, а В — матрицей с х d, то произведение АВ существует, только если b = с и является матрицей а х d (рис. 4.14). Если же b с, то произведение АВ не существует. Определение произведения матриц не является таким уж сложным, как может показаться на первый взгляд. В следующем примере процесс вычисления произведений станет более понятным. Если А = 2 3-1 —2 1 2 1 3 0 2 и В = 2 -1 то А является матрицей 2 х 3, а В — матрицей 3x2. Чтобы найти первую строку произведения АВ, последовательно умножим первую строку матрицы А на все столбцы матрицы В
Глава 4. Системы линейных уравнений и матрицы 339 Должны быть одинаковыми Ь = с I I а' b с' d t t Размер произведения a'd Рис. 4.14. Произведение двух матриц и запишем результаты как действительные числа (а не как матрицы размером 1 х 1). Вторая строка матрицы АВ вычисляется аналогичным способом. Четыре произведения строк и столбцов заданных матриц составляют четыре элемента матрицы АВ. Эти четыре произведения вычисляются в уме или с помощью калькулятора, поэтому их обычно не пишут. 2X2 9 И -2 -2 Пример 4.24 (Произведении матриц). 3X2 2 1 1 О -1 2 2X4 1-101' 2 12 0 3X4 4-12 2 1-10 1 3 3 4 -1 не определено. 5. [
340 Часть II. Конечная математика 6. -5 2 [2-30] = —2 -10 4 —4 15 -6 6 0 0 О Упражнение 4.24. Найдите следующие произведения матриц, если они определены. jcl На рис. 4.15 показано решение примера 4.24, п. 1 на калькуляторе. Как вы считаете, к какому результату можно прийти, решая на калькуляторе пример 4.24, п. 2? [А] [В1 ЕЕ1 12 (Я1* [В] [14 -1 [1 -1 [3 3 112 [1 [ -1 -1 0 1 ' 11 0] 211 11 0]] 2 ] 1 1 -11J 2 2 0 4 Рис. 4.15. Произведение матриц на калькуляторе В арифметике действительных чисел порядок множителей не имеет значения, например: 5 • 7 = 7 • 5. В операции произведения матриц это не так. Произведение АВ не равно произведению В А даже в том случае, если оба произведения и обе матрицы имеют одинаковый размер (см. пример 4.24, пп. 3-4). Произведение матриц некоммутативно. Более того, произведение АВ может быть равным нулю даже тогда, когда множители А и В — отличны от нуля (см. пример 4.24, п. 4).
Глава 4. Системы линейных уравнений и матрицы 341 Свойства нуля Не распространяются на произведение матриц. (Свойства нуля для действительных чисел обсуждаются в приложении А.2, т. 2). Задание 4.9. Помимо коммутативности и свойств нуля между произведениями действительных чисел и матриц имеются и другие принципиальные различия. 1. Для действительных чисел существует только одно число, квадрат которого равен нулю — это число 0 (Т.е. О2 » 0). Найдите хотя бы одну матрицу А размером 2x2 с отличными от нуля элементами такую, что А2 = О3, где 0 — нулевая матрица размером 2x2. 2. Для действительных чисел Имеется только одно отличное от нуля число, квадрат которого равен ему самому — это число 1 (т.е. I2 = 1). Найдите хотя бы одну матрицу А размером 2 х 2 с отличными от нуля элементами такую, что А2 = А. ■ Мы вернемся к свойствам матричного произведения далее в этой главе. А сейчас рассмотрим практические приложения матричного произведения. И Пример 4.25 (Оплата Труда). Обобщим условия для производства обычных и спортивных водных лыж (Пример 4.23 и упражнение 4.23) в одной матрице. Время производства пары лыж, ч Сборочный Отделочный цех цех обычные лыжи 5 1,5 спотивные лыжи 3 1 Теперь допустим, что компания имеет два завода — один в Калифорнии (СА), другой — в Мэриленде (MD). Почасовая оплата на каждом из заводов представлена с помощью следующей матрицы. Почасовая оплата, долл. СА МЬ Сборочный цех 12 13 _ Отделочный цех 7 8 Поскольку L и Н являются матрицами размером 2 х 2, то их произведение в том или ином порядке даст матрицу такого же размера. Я£ = ‘ 12 7 13 ' 8 ’ 5 3 1,5 ' 1 = ’ 99 59 31 18,5 — LH - ‘ 5 1.5 12 13 ‘ ’ 70,5 77 3 1 7 8 _ 43 47 Как интерпретировать матричные элементы произведений? Начнем с произведения HL, Элемент 99 в этом произведений является произведением первой строки матрицы Н и первого столбца матрицы L. [12 13 ] [ 5 7Р’“,Н“П“Ж" =12-5 + 13.3 = 60 + 39 = 99 L J L 3 J обычные лыжи 3 Согласно стандартным алгебраическим обозначениям будем писать А2 = А А, А3 — ААА и т.п.
342 Часть II. Конечная математика Отметим, что 60 долл. — это цена за сборку пары спортивных лыж на калифорнийском заводе, а 39 долл. — пары обычных лыж на мэрилендском. Несмотря на то что оба числа представляют собой расходы на рабочую силу, нет смысла их складывать вместе, поскольку они не относятся к одному типу лыж на одном заводе. Поэтому, хотя произведение HL корректно определено с математической точки зрения, оно не является полезным для интерпретации задачи. Перейдем теперь к произведению LH. Элемент 70,5 в первой строке и первом столбце матрицы LH соответствует следующему произведению. Сборка Отделка [5 1,5 ] 12 7 Сборка Отделка = 5 • 12 + 1,5 • 7 = 60 + 10,5 = 70,5. В этом случае 60 долл. — величина затрат на сборку пары спортивных лыж на калифорнийском заводе, 10,5 долл. — величина затрат на отделку пары спортивных лыж на калифорнийском заводе. Таким образом, сумма определяет затраты на производство пары спортивных лыж на калифорнийском заводе. Остальные элементы произведения LH также определяют общие затраты на производство лыж, как указано в комментариях к строкам и столбцам следующей матрицы. Общие затраты на производство лыж, долл. СА МЬ LH = 70,50 77 43 47 Спортивные лыжи Обычные лыжи zl Решение примера 4.25 с помощью электронной таблицы показано на рис. 4.16. А в I 1 c D E F 1 Labor-hours per ski Hourly wages 2 Assembly Fi nishi ng California Wisconsin 3 Trick ski 5 1.5 Assembly $12 $13 4 Slalom ski 3 1 Fi nishi ng $7 $8 5 Labor costs per ski 6 California Wisconsin 7 Trick ski $70.50 $77.00 8 Slalom ski $43.00 $47.00 Рис. 4.16. Перемножение матриц процессором электронных таблиц: команда MMULT(B3:C4, E3:F4) вычисляет произведение матриц в В7:С8 И Упражнение 4.25. Вернемся к примеру 4.25. Пусть требуется узнать, сколько времени на каждом заводе затрачено на производство 2000 пар спортивных и 1000 пар обычных лыж. Исходные данные можно представить в виде следующих матриц. Пар спортивных Пар обычных ЛЫЖ ЛЫЖ ’ 2000 ' P=\ ' 2000 1000 ] Q = 1000 Пар спортивных лыж Пар обычных лыж
Глава 4. Системы линейных уравнений и матрицы 343 Используя матрицу рабочих часов L из примера 4.25, вычислите произведение PL или LQ — то, которое дает осмысленную информацию о задаче. Дайте соответствующие комментарии к строкам и столбцам матрицы. ■ Предупреждение. Из примера 4.25 и упражнения 4.25 можно сделать важные выводы о произведениях матриц. Используя графические утилиты для матричных вычислений, получаемые результаты всегда нужно интерпретировать с точки зрения определения матричных операций и условий решаемой задачи. Ответы к упражнениям 1 о 2 5 —2 1 4.19. [ -1 -1 4 ] . 4.20. 13 2 35 4.21. 1) 2) 3) 235000 372000 145000 160000 9500 13300 422000 298000 268000 254000 17250 ‘ 13800 4.22. [ 8 ] . Q 4.23. Г 5 1,5 1 * L J о [ 54 ] , или 54 долл. 4.24. 1) Не определено. 2) -1 -1 2 ‘ 12 -4 3 -2 2 1 2 6 0 0 0 0 0 3) 4) -6 -12 ‘ 3 6 5) [ И ] • 6) 12 -8 6 —4 9 -6 4 2 3 Сборка Отделка 4.25. PL = [ 13 000 4000 ] Время производства, ч
344 Часть II. Конечная математика Практикум 4.4 А В задачах 1—14 требуется выполнить указанные операции, если они определены. 8. 10 -1 -4 -1 —2 J [ 2 2 -3 1 Г 1 -1 1 2 J [ 0 —2 1 -1 1 Г 2 -3 0-2 12 Б В задачах 15-22 вычислите произведения матриц. 15. [ 5 -2 ] J 16. [-4 3] ! 1 ■ 2 ’ 19. [3 —2 —4 ] 2 20. [ 1 —2 2 ] -1 -3 1 В задачах 23-40 речь идет о таких матрицах. -1 4 —2 Выполните следующие операции, если они определены. 23. АС. 24. С А. 25. АВ. 26. В А. 27. В2. 28. С2.
Глава 4. Системы линейных уравнений и матрицы 345 29. В + AD. 32. 0,2(70. 35. -2ВА + 6CD. 30. С + DA. 33. ЗВ А + 4АС. 36. -1АС + 3DB. 31. О,1Г>В. 34. 2DB + 5CD 37. ACD. 38. CD А. 39. DBA. 40. BAD. jcl В задачах 41 и 42с помощью графической утилиты вычислите матрицы В, В2, В3... и АВ, АВ2, АВ3. Опишите закономерности, которые наблюдаются в каждой последовательности матриц. *41. А= [ 0,3 0,7 ] И В = ’ 0,4 0,2 0,6 ' 0,8 *42. А = [ 0,4 0,6 ] И В = ' 0,9 0,3 0,1 ' 0,7 43. Найдите числа а, Ь, с и d, для которых выполняется равенство a b 1 Г 2 -3 I _ Г 1 —2 с d ] + 0 1 J — L 3 —4 44. Найдите числа w, X, у и z, для которых выполняется равенство 4—2 w х -3 0 + I у z 2 -3 0 5 45. Найдите числа х и у, для которых выполняется равенство 2х 4 ' 1 з?/ —2 ' ’ -5 2 ‘ -3 бх 1 -2 ~У . -5 13 46. Найдите числа х и у, для которых выполняется равенство ' 5 Зх 1 ■ 1 -4у ' ' 6 -7 ' 2х -4 4 5 0 В 47. Найдите числа х и у, для которых выполняется равенство х -1 2 1 10 4 1 У У 2 1 48. Найдите числа х и у, для которых выполняется равенство 1 3 ' X 1 ■ ■ У 7 ‘ -2 -2 3 2 . у -б 49. Найдите числа а, Ь, с и d, для которых выполняется равенство 1 —2 1 Г а Ь 2 -3 ] [ с d 1 0 3 2
346 Часть II. Конечная математика 50. Найдите числа а, Ь, с и d, для которых выполняется равенство 1 3 I Г а b 14 cd 6 -5 7 —7 *51. Квадратная матрица называется диагональной, если все ее элементы, не расположенные на главной диагонали, равны нулю. Диагональная матрица размером 2x2 имеет вид л Г а 0 [« <d’ где а и d — произвольные действительные числа. Проверьте правильность приведенных ниже утверждений. Если утверждение является всегда верным, объясните почему. Если неверным, приведите примеры. а) Если А и В — диагональные матрицы 2 х 2, то А+В — диагональная матрица 2x2. б) Если А и В — диагональные матрицы 2 х 2, то А + В = В + А. в) Если А и В — диагональные матрицы 2 х 2, то АВ — диагональная матрица 2 х 2. г) Если А и В — диагональные матрицы 2 х 2, то АВ = В А. * 52. Квадратная матрица называется верхней треугольной матрицей, если все ее элементы, расположенные ниже главной диагонали, равны нулю. Верхняя треугольная матрица размера 2x2 имеет вид где а, b и d — произвольные действительные числа. Проверьте правильность приведенных ниже утверждений. Если утверждение является всегда верным, объясните почему. Если неверным, приведите примеры. а) Если А и В — верхние треугольные матрицы 2 х 2, то А + В — верхняя треугольная матрица 2x2. б) Если А и В — верхние треугольные матрицы 2 х 2, то А + В = В + А. в) Если А и В — верхние треугольные матрицы 2 х 2, то АВ — верхняя треугольная матрица 2x2. г) Если А и В — верхние треугольные матрицы 2 х 2, то АВ = В А. Применение математики Экономика и бизнес 53. Анализ затрат. На двух различных заводах компания производит гитары и банджо. Затраты на изготовление каждого инструмента приведены в следующей матрице. Завод X Гитара Банджо Сырье, долл. 30 25 Оплата труда, долл. 60 80 ’
Глава 4. Системы линейных уравнений и матрицы 347 Завод Y Сырье, долл. Оплата труда, долл. Гитара Банджо ’ 30 25 Найдите среднюю величину затрат на производство инструментов j (А + В). 54. Анализ затрат. Если затраты на сырье и работу на заводе X увеличить на 20%, то средняя величина затрат на производство инструментов станет равной | (1,2А + В). Найдите это значение. 55. Наценка. Автомобильный салон продает три модели импортных машин. Розничные и фактурные Цены на базовые модели и дополнительные опции представлены следующими двумя матрицами (сокращение “кондиц.” обозначает кондиционер в салоне автомобиля). Розничные цены, долл. Базовая комплектация Кондиц. AM/FM- радио Круиз- контроль Модель А ’ 10900 683 253 195 “ Модель Б 13000 738 382 206 = М Модель В 16300 867 537 225 Фактурные цены, долл. Базовая комплектация Кондиц. AM/FM- радио Круиз- контроль Модель А 9400 582 195 160 ' Модель Б 11500 621 295 171 = W Модель В 14100 737 420 184 Определим матрицу наценки как M—N (наценка — это разность между розничными и фактурными ценами). Допустим, что курс доллара резко упал, и фактурные цены снизились на 15% для всех моделей на следующий год. Для сохранения уровня конкуренции розничные цены были снижены на 10%. Найдите матрицу наценки на следующий год при этих условиях с точностью до доллара. 56. Наценка. Вернемся к задаче 55. Какой будет матрица наценки, если фактурные цены снизить на 20%, а розничные на 15%? (Результат округлить с точностью до доллара.)
348 Часть II. Конечная математика 57. WWW Оплата труда. Компания, производящая надувные резиновые лодки, имеет два завода в штатах Массачусетс и Вирджиния. Данные об объеме выполненных работ и оплате труда приведены в следующих матрицах. Время производства лодки, ч М = отдел резки 0,6 1,0 1,5 отдел сборки 0,6 0,9 1,2 отдел упаковки 0,2 “ 0,3 0,4 1- местная лодка 2- местная лодка 4-местная лодка Оплата в час, долл. а) б) *в) *г) МА VA 14 10 12 8 7 7 Отдел резки Отдел сборки Отдел упаковки Вычислите затраты на оплату труда при производстве одноместных лодок на массачусетском заводе. Вычислите затраты на оплату труда при производстве четырехместных лодок на заводе в Вирджинии. Обсудите возможные интерпретации матричных произведений MN и NM. Если каждое из произведений MN и NM допускает осмысленную интерпретацию, найдите произведения их строк и столбцов. 58. Количество товара. Компания производит и реализует в розницу пять различных моделей компьютеров через три магазина, расположенных в одном городе. В матрице М приведены данные о количестве компьютеров всех моделей в каждом магазине, а в матрице N — розничные и оптовые цены на эти модели. М = Количество, шт. Модель С 3 5 3 А 4 2 10 В 2 3 4 Ь 7 0 4 Е 1 6 3 Магазин 1 Магазин 2 Магазин 3
Глава 4. Системы линейных уравнений и матрицы 349 Опт. Розн. цена цена 700 840 ' А 1400 1800 В 1800 2400 С Модель 2700 3300 Ь 3500 4900 Е а) Какова розничная цена для товаров в магазине 2? б) Какова оптовая цена для товаров в магазине 3? *в) Обсудите возможные интерпретации элементов произведений матриц MN uNM. *г) Если каждое из произведений MN и NM допускает осмысленную интерпретацию, найдите произведения их строк и столбцов. *д) Какие матричные методы можно использовать для поиска запасов товара для каждой модели во всех трех магазинах? Укажите, какие матрицы нужны для этих вычислений. *е) Какие матричные методы можно использовать для поиска запасов товара для всех пяти моделей в каждом из магазинов? Укажите, какие матрицы нужны для этих вычислений. * 59. WWW Воздушные Перевозки. Национальная компания грузовых перевозок соединяет пять городов, как показано на диаграмме. Для представления этого плана в матричном виде строится матрица инцидентности Л, в строках которой указываются пункты вылета, а в столбцах — пунктам назначения. Для отображения полета из i-ГО города в J-Й поместим 1 на пересечении г-й строки и j-ro столбца. Если полеты между городами не предусмотрены, введем 0. Элементы главной диагонали также равны нулю, поскольку полеты в пределах одного города не имеют смысла. Представив теперь план полетов в виде матрицы, произведем с ней преобразования для получения некоторой информации о полетах.
350 Часть II. Конечная математика Пункт назначения 1 2 3 4 5 1 '00111' 2 10 10 0 Пункт вылета 3 0 0 0 0 1 = А. 4 0 10 0 0 5 0 0 0 1 0 а) Вычислите матрицу А2. Какой смысл имеет единица во второй строке и третьем столбце матрицы А2 в контексте нашей задачи? Какой смысл имеет нуль во второй строке и пятом столбце матрицы А2 в контексте выполненной операции? Как интерпретировать недиагональные элементы матрицы А2? (Подсказка. Учтите возможные перелеты между г-м и j-m городом на диаграмме.) б) Вычислите матрицу А3. Какой смысл имеет единица в четвертой строке и втором столбце матрицы А3 в контексте нашей задачи? Какой смысл имеет нуль во второй строке и четвертом столбце матрицы А3 в контексте выполненной операции? Как интерпретировать недиагональные элементы матрицы А3? в) Последовательно вычислите матрицы А, А + А2, А + А2 + А3, ..., пока не получите матрицу, все недиагональные элементы которой отличны от нуля. Дайте интерпретацию этой матрицы. * 60. Воздушные перевозки. Постройте матрицу инцидентности для приведенной ниже диаграммы. Последовательно вычислите матрицы А, А+А2, А+А2+А3,..., пока не получите матрицу, все недиагональные элементы которой отличны от нуля. Дайте интерпретацию этой матрицы. Биологические науки 61. Диета. Диетолог компании, которая занимается зерновыми культурами, составил три рецепта для хлеба (X, Y и Z), выпекаемых из муки двух сортов: А и В. Содержание белков, углеводов и жиров в муке каждого вида описывается матрицей М. Количество муки сорта А и В в каждом рецепте приведено в матрице N. Сорт муки А 4 20 3 В 2 16 1 М = Протеины, г/унция Углеводы, г/унция Жиры, г/унция
Глава 4. Системы линейных уравнений и матрицы 351 Рецепт X У Z 15 10 5 Мука сорта А 5 10 15 Мука сорта В а) Найдите количество белков в рецепте X. б) Найдите количество жиров в рецепте Z. *в) Обсудите возможные интерпретации матричных элементов произведений MNи NM. *г) Если какое-либо из произведений MN и NM допускает осмысленную интерпретацию, вычислите его и сделайте соответствующие подписи к строкам и столбцам. *62. WWW Наследственность, Открытия Грегора Менделя (Gregor Mandel, 1822— 1884), австрийского монаха и ботаника, в корне изменили такую дисциплину, как генетика, В одном из экспериментов он скрестил желтый круглый горох (желтый и круглый — это доминирующие признаки, в горохе также содержатся гены рецессивных характеристик зеленого и морщинистого) и получил урожай, описанный В следующей матрице: Круглый Сморщенный Желтый Г 315 101 ' = М Зеленый 108 32 Допустим, что во втором эксперименте такого же рода он получил следующий результат Круглый Сморщенный Желтый 370 128 _ „ Зеленый ИО 36 Обсудите матричные методы, Которые можно использовать при совместном анализе обоих экспериментов для получения ответов на следующие вопросы. Постройте необходимые для этого матрицы и выполните вычисления. а) Полное количество горошин каждой категории. б) Полное количество горошин всех четырех категорий. в) Процентное содержание горошин в каждой категории. Социальные науки 63. Политика, На местных выборах в Калифорнии группа поддержки способствует выдвижению своего кандидата тремя способами агитации: звонками по телефону, посещениями на дому и почтовой рассылкой. Расценки за агитацию описываются матрицей М, а количество избирателей, с которыми была проведена работа, в двух соседних городах указано в матрице N. Оплата, долл. 0,40 1,00 0,35 М = Телефонный звонок Посещение на дому Почтовая рассылка
352 Часть II. Конечная математика Телефонный Посещение Почтовая ЗВОНОК на дому рассылка N = ■ 1000 500 5000 ' Беркли 2000 800 8000 Окленд а) Вычислите затраты на агитацию в Беркли. б) Вычислите затраты на агитацию в Окленде. *в) Обсудите возможные интерпретации матричных элементов произведений MN и NM. *г) Если какое-либо из произведений MN и NM допускает осмысленную интерпретацию, вычислите его и сделайте соответствующие подписи к строкам и столбцам. *д) Обсудите матричные методы, которые позволяют найти полное количество телефонных звонков, посещений дома и писем. Постройте необходимые матрицы и выполните вычисления. *е) Обсудите матричные методы, которые позволяют найти полное количество контактов в Окленде и Беркли. Постройте необходимые матрицы и выполните вычисления. * 64. Тестирование. Преподаватель провел четыре теста в классе из пяти студентов и представил результаты в виде следующей матрицы: Оценка за тест Энн ’ 78 84 81 86 ' Боб 91 65 84 92 Кэрол 95 90 92 91 = М Дэн 75 82 87 91 Эрик 83 88 81 76 Обсудите матричные операции, с помощью которых преподаватель может получить следующую информацию. В каждом из случаев постройте необходимые матрицы и выполните вычисления. а) Средняя оценка по тестам для каждого студента, если считать, что тесты — одинакового уровня сложности. б) Средняя оценка по тестам для каждого студента, если считать, что первые три теста — одинакового уровня сложности, а четвертый — вдвое сложнее. в) Средняя оценка по тестам для всей группы студентов. 65. Уровень квалификации. Чтобы присвоить каждому игроку ранг в приближающемся теннисном турнире, было решено, чтобы все теннисисты сыграли по сету друг с другом. Результаты приведены в следующей таблице. Игрок Проигрыш Аарон Чарльзу, Дэну, Элвису Барт Аарону, Дэну, Элвису Чарльз Барту, Дэну Дэн Фрэнку Элвис Чарльзу, Дэну, Фрэнку Фрэнк Аарону, Барту, Чарльзу
Глава 4. Системы линейных уравнений и матрицы 353 а) Запишите эти результаты в виде матрицы инцидентности А, помещая единицу на Пересечении г-й строки и j-ro столбца матрицы А, если г-й игрок проиграл J-му, И нуль в противном случае (см. задачу 59). б) Вычислите матрицу В = А + А2, *в) Обсудите матричные методы, с помощью которых можно найти сумму строк матрицы В, Постройте необходимые матрицы и выполните вычисления. *г) Присвойте каждому игроку характеристику (от наиболее сильного до наиболее слабого). Объясните, по какому принципу это делается. 66. Уровень квалификации. Все шахматисты провели по матчу друг с другом. Результаты приведены в следующей таблице. Игрок Проигрыш Энн Диане Бриджит Энн, Кэрол, Диане Кэрол Энн Диана Кэрол, Элен Элен Энн, Бриджит, Кэрол а) Запишите эти результаты в виде матрицы инцидентности А, помещая единицу на пересечении г-й строки и j-ro столбца матрицы А, если г-й игрок проиграл j-му, И нуль в противном случае (см. задачу 59). б) Вычислите матрицу В = А + А2. *в) Обсудите матричные методы, с помощью которых можно найти сумму строк матрицы В. Постройте необходимые матрицы и выполните вычисления. *г) Присвойте каждому игроку характеристику (от наиболее сильного до наиболее слабого). Объясните, по какому принципу это делается. 4.5. Нахождение обратной квадратной матрицы ■ Единичная матрица для умножения ■ Вычисление обратной матрицы ■ Решение практических задач: криптография Единичная матрица для умножения Как известно, la = al а= а ддя всех действительных чисел а. Число 1 называется единицей по отношению к операциям с действительными числами. Существует ли единичный элемент по отношению к операции умножения матриц одного и того же размера? Иначе говоря, если М — матрица т х п, то существует ли единичный элемент I такой, что IM =z MI *= Ml В общем случае ответ отрицательный. Однако по отношению к операции умножения квадратных матриц порядка п (матрицы с п строками и п столбцами) единичный элемент существует и определяется следующим образом.
354 Часть II. Конечная математика Единичный элемент по отношению к операции умножения квадратных матриц порядка п является квадратной матрицей порядка п и обозначается через I. Все элементы главной диагонали матрицы I (с левого верхнего угла до правого нижнего) равны 1, а остальные — 0. Например, матрицы 1 0 0 0 1 0 0 0 1 являются единичными матрицами порядка 2 и 3 соответственно. |ci. Во многих графических утилитах существует команда для генерации единичной матрицы заданного размера (рис. 4.17). identity 2 [ [1 _[0 0 1 0 identity 3 t [1 [0 [0 01 1)1 0] 0] 1]] Рис. 4.17. Единичные матрицы Пример 4.26 (Умножение на единичную матрицу). Упражнение 4.26. Вычислите произведения следующих матриц. 1. 1 0 0 1 2 2 -3 1 Г 1 0 5 7 0 1 5 ’ 1 0 0 ' ’4 2 " ’4 2 ■ 1 А 2. 0 1 0 3 -5 И 3 -5 1 и А 1 0 0 1 6 8 6 8 U 1
Глава 4. Системы линейных уравнений и матрицы 355 В общем случае можно Показать, что если М — квадратная матрица порядка п, а I — единичная матрица порядка п, то IM = MI = м. Если же М — матрица m х п, т.е, не квадратная (т / п), то ее можно умножить слева и справа на единичные матрицы разных размеров (см. пример 4.26, пп. 3 и 4.). Сейчас мы сосредоточимся на квадратных матрицах, избегая тем самым возможной путаницы, связанной с двумя единичными матрицами, которые не являются квадратными. Задание 4.10. Уравнение х2 = 1 имеет только два действительных решения х = 1 и т = — 1. 0 1' 1 0 1. Покажите, что Матрица А единичная матрица размером 2x2. 0 -1 -1 удовлетворяет уравнению А2 = /, где I — 2. Покажите, что Матрица В — удовлетворяет уравнению В2 = I. 0 3. Найдите матрицу размером 2x2, все элементы которой отличны от нуля, а квадрат равен единичной матрице. ■ Вычисление обратной матрицы Как известно, каждому действительному числу а (кроме 0) можно сопоставить обратное число а-1, причем a~la = 1. Действительное число а-1 называется обратным к числу а относительно операции умножения. Например, число 2^1 является обратным к числу 2 относительно умножения, поскольку 2-1 -2 = 1. Используем эту идею для определения обратной матрицы. Нахождение обратной квадратной матрицы Пусть М — квадратная матрица порядка п и I — единичная матрица порядка п. Если существует матрица Л/”1 (читается “обратная к М”) такая, что М~ГМ = ММ'1 = I, то матрица М~х называется обратной к матрице М относительно операции умножения. Действительное число, обратное к числу а, можно записать как 1/а. Для матриц это обозначение не используется» Используем теперь это определение для поиска обратной матрицы. Пусть 2 1 3 2 Требуется найти такую матрицу М-1 = а с b d
356 что Часть II. Конечная математика мм-1 = м~хм = I. Поэтому запишем М I 2 3 а с _ 1 О 1 2 ] [ b d ] ~ |_ 0 1 и будем искать такие числа а, Ь, с и d, чтобы произведение М и М~г было равно единичной матрице. Вычисляя произведение матриц М и М~1 в левой части, получим следующий результат. (2а + 36) (а -|- 26) (2с 4- 3d) 1 _ Г 1 О (с 4-2d) ] “ [ 0 1 Это равенство выполняется, если справедливы следующие соотношения. 2а 4~ 36 — 1, а 4~ 26 — 0. 2с 4“ 3d — 0, с 4~ 2d — 1. Решая эти две системы уравнений, находим: а = 2, 6 = — 1, с = — 3 и d = 2. Итак, Это легко проверить. м "2 3 ‘ 1 2 м-1 ’ 2 -3 ‘ -1 2 = I ■ 1 О' 0 1 = оо см к 1 1 1 м ■ 2 3 1 2 В отличие от действительных чисел, для квадратных матриц не всегда существует обратная. Например, если то, поступая как и выше, мы получим следующие системы уравнений. 2а 4~ 6 = 1, 4а 4~ 26 — 0. 2с 4“ d = 0, 4с 4~ 2d = 1. Обе системы являются несовместными и не имеют решений, поэтому матрица 7V-1 не существует. Умение находить обращение матрицы, если оно существует, дает возможность находить прямые решения во многих практических задачах. В следующем разделе мы рассмотрим, как использовать обратные матрицы для решения систем линейных уравнений. Метод, с помощью которого была найдена матрица М~г (если она существует), становится довольно громоздким для матриц с размерностью больше двух. Рассмотрим более эффективную процедуру поиска обратной матрицы с помощью расширенной матрицы (см. разделы 4.2 и 4.3).
Глава 4. Системы Линеййых уравнений и матрицы 357 Пример 4.27 (Вычисление Обратной матрицы). Вычислите обратную матрицу, если она существует: 1 -1 О Решение. Как и раньше, ДЛЯ начала запишем произведение матриц. М м-1 I “1-1 1 ■ a d д ’10 0' 0 2—1 b е h = 0 10 2 3 0 с f г 0 0 1 Это равенство будет верным, только если выполняются следующие соотношения. а — Ь + с = = 1, d- е + f = 0, д — h + i = 0, 2Ь - с 0, 2e + f = ■■ 1, 2h — i = 0, 2о -|- ЗЬ - — 0. 2d -I- Зе — 0. 2g 3h — . 1. Запишем все три системы с помощью расширенных матриц. Первая Вторая Третья 1 -1 1 1 “1-1 1 0 ’1-1 1 0 0 2-1 0 0 2-1 1 0 2-1 0 2 3 0 0 _ 2 3 0 0 2 3 0 1 Во всех трех системах матрицы слева — одни и те же, поэтому в расширенных матрицах будут производиться одни и те же преобразования со строками для представления расширенной матрицы в приведенной форме. Для ускорения процесса преобразований скомбинируем все три расширенные матрицы в одну. о о 1 1 -1 О 2 2 3 1 1 О -10 1 ООО = [«!'] (4.17) Теперь будем преобразовывать матрицу (4.17) до тех пор, пока не получим эквивалентную матрицу вида I В • 1 0 0 a d 9 ' 0 1 0 b e h 0 0 1 c f i =[/|В] (4.18) По окончании этой процедуры новая матрица В справа от вертикальной черты совпадет с матрицей М~г. Рассмотрим преобразование матрицы (4.17) в форму (4.18). Последо¬ вательность шагов — та же, что и в решении систем уравнений методом исключения Гаусса-Жордана (раздел 4.3). M I 1 -1 1 1 0 0 ’ 0 2 -1 0 1 0 2 3 0 0 0 1 —2Hi + Вз —► В>з
358 Часть II. Конечная математика ■ 1 -1 1 1 0 0 ' 0 2 -1 0 1 0 _ 0 5 -2 —2 0 1 ■ 1 -1 1 1 0 0 “ R2 4“ Ri —► Ri 0 1 1 2 0 1 2 0 _ 0 5 —2 —2 0 1 -5R2 + Rs —► Rs ’ 1 0 1 2 1 1 2 0 " 0 1 1 2 0 1 2 0 0 0 1 2 —2 5 2 1 2Rs —► Rs " 1 0 1 2 1 2 1 1 1 2 1 2 -5 0 " 0 _ 0 1 0 0 —4 0 2 _ — | R3 + Ri Ri %Rs 4- R2 —► R2 ■ 1 0 0 3 3 -1 ‘ 0 1 0 -2 -2 1 0 0 1 —4 -5 2 Преобразовывая исходные системы уравнений в эквивалентные, получаем следующие результаты (на практике этот шаг на самом деле не нужен). а = 3, d = 3, b = —2, е = -2, с = —4. 9 = -1, Л = 1, i = 2. Они как раз и представляют элементы искомой матрицы М 1! Заметим, эта матрица записана справа от вертикальной линии в обобщенной матрице, т.е. М~х = В, Поскольку в определении обратной матрицы требуется выполнение условий MM~r=I и М~ГМ = 1, (4.19) то для проверки вычисления, вообще говоря, нужно вычислить оба произведения ММ~{ и М~1М. Однако можно показать, что если хотя бы одно из уравнений (4.19) удовлетворяется, то второе будет выполняться автоматически. Поэтому для проверки достаточно вычислить одно из произведений ММ~1 и М~гМ, а не оба. Проверка. 1 0 0 М~гМ = 3 3 -2 -2 —4 -5 -1 1 2 1 -1 1 О 2—1 2 3 0 10 =1. 0 1 0 0
Глава 4. Системы линейных уравнений и матрицы 359 Упражнение 4.27. Пусть 1. Постройте расширенную матрицу [М 11]. 2. Для преобразований матрицы [М 11] в матрицу [I | В] используйте операции над строками. 3. Убедитесь с помощью умножения, что В = М~г (т.е. ВМ = I). ■ Процедура, описанная в примере 4.27, может быть использована для нахождения обратной квадратной матрицы любого размера или привести к заключению, что обратная матрица не существует. Резюмируем эти результаты в теореме 4.4. Теорема 4.4 (Нахождение обратной квадратной матрицы М). Если расширенная матрица [М | /] может быть преобразована в форму [/ | В], то получающаяся в результате матрица В совпадает с М“Ч Однако если слева от вертикальной черты получается матрица с одной или несколькими строками (или столбцами), состоящими полностью из нулей, то матрица М-1 не существует. ■ Задание 4.11. 1. Допустим, что В квадратной матрице М есть строка, полностью состоящая из нулей. Объясните, почему в таком случае матрица М не имеет обратной. 2. Допустим, что В квадратной матрице М есть столбец, полностью состоящий из нулей. Объясните, почему в таком случае матрица М не имеет обратной. ■ Пример 4.28 (Вычисление обратной матрицы). Вычислите М-1, если М = Решение. 4 -1 1 0 ‘ |Я1 —► Ri -6 2 0 1 1 _ 1 4 1 4 0 ‘ -в 2 0 1 6В1 4" R>2 —► Н>2 1 _ 1 4 1 4 0 ' 0 1 2 3 2 1 2R2 —► R2 1 _ 1 4 1 4 0 ' \R2 + Я1 —► Ri 0 1 3 2 Г 1 0 14 1 О 13 2 Итак, М-1 = Г 1 3 2 Проверьте этот результат, показав, что М~гМ = I.
360 Часть II. Конечная математика Упражнение 4.28. Вычислите Л/-1, если М = Ю, В большинстве графических утилит можно вычислять обратные матрицы, как показано на рис. 4.18. а) в графическом калькуляторе обратная матрица вычисляется с помощью команды М-1 Рис. 4.18. Вычисление обратной матрицы в графической утилите *1 В | С 1 D| [ Е J F | G 1 м МInverse 2 4 -1 1 0.5 3 -6 2] з г| 4 б) в электронной таблице обратная матрица вычисляется с помощью функции MINVERSE(B2:C3) Задание 4.12. Обратной к матрице А = b d а с является следующая матрица. ' d -b - А^ = ad — be —с ad — be a “ D d -b' —c a _ ad — be ad —be _ D = ad — be. Она существует только тогда, когда D 0. 1. Умножьте матрицы для проверки этой формулы. Что можно сказать о матрице А-1, если D = 0? 2. Воспользуйтесь этой формулой для поиска обратной матрицы в примере 4.28. ■ Пример 4.29 (Вычисление обратной матрицы). Вычислите Л/-1, если М = 2 -4 -3 6 Решение. 2 —4 1 0 ‘ -3 6 0 1 J 1 —2 1 O' -3 6 0 1 J ■ 1 -2 1 O' 0 0 1 1J ^Ri —► Ri ЗЛ1 4- R2 —► R% Во второй строке матрицы слева от вертикальной черты все элементы равны нулю, поэтому обратная матрица не существует. в Упражнение 4.29. Вычислите 7V-1, если N = 3 1 6 2
Глава 4. Системы линейных уравнений и матрицы 361 из. Матрицы, для которых не существует обратных, называются сингулярными, или вырожденными. Графические утилиты распознают сингулярные матрицы и выдают сообщение об ошибке, как показано на рис. 4.19. [ [2 -4J [‘3 6 11 М-i А В С D Е F G 1 И И Inverse ERROR 03 SINGUCfiOSt 2 2 -4 -3 6 ### ### 3 ■ ■ ! Г- М 1 4 а) графический калькулятор выдает однозначное сообщение об ошибке б) сообщение об ошибке в электронной таблице выглядит более загадочно Рис. 4.19. Сообщение об ошибке Решение практических задач: криптография E WWW С помощью вычисления обратных матриц можно построить эффективную процедуру шифрования и расшифровки сообщений. Для этого сопоставим буквам английского алфавита целые числа от 1 до 26, как показано ниже. Число 0 будет соответствовать пробелу между словами (в более сложной кодировке можно использовать как строчные, так и прописные буквы, а также символы пунктуации). Пробел ABCDEFJHIJ KLMNOPQRS TUVWXYZ О 1 2 3 4 5 б 7 8 9 10 11 12 13 14 15 16 17 18 19 20 21 22 23 24 25 26 Тогда фразе “SECRET CODE” будет соответствовать последовательность 19 б 3 18 5 20 0 3 15 4 5 Любая обратимая матрица, Элементы которой являются положительными целыми числами, может быть использована как матрица шифрования. Например, чтобы закодировать приведенное выше сообщение с помощью матрицы размером 2x2 разделим числа в последовательности на группы по два числа и запишем их в столбцы матрицы В. 19 3 5 0 15 5 5 18 20 3 4 0 Пары чисел записываются сверху вниз поко- лоночно (не построчно) (Обратите внимание на то, ЧТО в конце сообщения добавлен лишний пробел, чтобы число элементов матрицы было четным). Затем умножим эту матрицу слева на А. АВ = 4 3 1 1 19 3 5 0 5 18 20 3 15 5 4 0 91 66 80 9 72 20 24 21 25 3 19 5
362 Часть II. Конечная математика При этом зашифрованное сообщение имеет вид 91 24 66 21 80 25 9 3 72 19 20 5 Фразу можно расшифровать, преобразовав ее обратно в матричную форму и умножив слева на матрицу расшифровки А~1. Матрица А-1 легко вычисляется по известной матрице А, поэтому в качестве ключа к расшифровке достаточно указать матрицу кодировки А. Несмотря на простую концепцию, сообщения, закодированные этим способом, злоумышленнику довольно сложно расшифровать. Пример 4.30 (Криптография). Сообщение 46 84 85 28 47 46 4 5 10 30 48 72 29 57 38 38 57 95 закодировано с помощью приведенной ниже матрицы. Расшифруйте его. А = 1 2 2 1 1 3 1 2 1 Решение. Поскольку матрица кодировки имеет размер 3x3, запишем закодированное сообщение по столбцам матрицы С с тремя строками. ■ 46 28 4 30 29 38 ' с = 84 47 5 48 57 57 85 46 10 72 38 95 Если в матрице В записано незакодированное сообщение, то она связана с матрицей С как С = АВ. Для восстановления В найдем А-1 (подробности вычислений пропустим) и умножим обе части уравнения С = АВ на А-1. В = А~1С = -5 2 1 ' ’ 46 28 4 30 2 -1 0 84 47 5 48 4 -1 -1 . 85 46 10 72 23 0 0 18 7 19 ' 8 9 3 12 1 19 15 19 1 0 21 0 29 38 57 57 38 95 Преобразовывая матричные элементы в числовую последовательность и учитывая соответствие между числами и буквами, получим расшифрованное сообщение. 23 8 15 0 9 19 0 3 1 18 12 0 7 1 21 19 19 0 WHO IS CARL GAUSS И Упражнение 4.30. Приведенная ниже фраза зашифрована с помощью матрицы из примера 4.30. Рас¬ шифруйте это сообщение. 46 84 85 28 47 46 32 41 78 25 42 53 25 37 63 43 71 83 19 37 25
Глава 4. Системы линейных уравнений и матрицы 363 Ответы к упражнениям 4.26. 1) 2) 4.27. 1) 2) 3) 4.28. 4.29. 2 5 4 3 6 3 -1 1 1 1 -1 3 1 -3 7 2 ' -5 8 _ -1 1 О О О 1 1 2 -1 1 1 -1 -1 -1 2 о 1 о 1 2 -1 О о 1 _ -1 -1 2 3 -1 1 -1 1 О 1 О О О 1 о 1 о 1 1 0 О 1 о 1 1 о о о 1 о о о 1 -1 1 2 Не существует. 4.30. WHO IS WILHELM JORDAN. Практикум 4.5 А В задачах 1-8 требуется выполнить указанные операции. 2. 4. 6. 8. 1 О О 1 -1 О 1 О О 3 1 6 6 2 О 1 О —4 2 -3 -1 О ' 1 О О -5 -1 6 2 О 1 3 1 6 —4 2 -3 1 О О О -5 -1 О О 1 о о 1 О 1 о В задачах 9-18 необходимо определить, являются ли указанные матрицы взаимно обратными, вычисляя их произведение. 1 2 9. 11 13. 3 —2 2 -1 -5 -8 —4 3 2 -1 2 ' 3 3 4 2 3 ’ 1 1 -1 -1 ’ 3 -2 ‘ 8 -5 • 10. 12. 14. -2 -4 5 —2 7 -5 -1 2 —7 3 4 -3 -1 —2 7 ' 5 ' 3 4 -5 —7 3 2
364 Часть II. Конечная математика 1 2 О О 1 О -1 -1 1 1 -1 1 0 2-1; 2 3 0 1-2 0 1 О 1 0 . 16. -3 1 -1 0 ] [о 3 3 -1 ' -2-2 1 . —4 -5 2 1 0 -1 3 1 -1 О 0 1 О 1 О 1 -2 ; 1 1 О 3 1 18. о о 1 0 -1 -3 1 —2 О 0 1 Б В задачах 19-28 для заданной матрицы М требуется вычислить матрицу М 1 и показать, что М-1М = I. 20. 1 -5 О -1 ’ 2 3 0 “ 26. 1 2 3 _ 0 -1 -5 _ ’ 1 0 -1 ' 28. 2 -1 0 1 1 —2 В задачах 29-34 требуется вычислить обратную матрицу, если она существует. 4 3 —2 2 3 9 2 4 3 В В задачах 35-42 требуется вычислить обратную матрицу, если она существует. " -5 -2 -2 ’ " 2 -2 4 ‘ 35. 2 1 0 36. 1 1 1 1 0 1 _ 1 0 1 2 1 1 " 1 -1 0 37. 1 1 0 . 38. 2 -1 1 -1 -1 0 0 1 1 — ’ -1 —2 2 ’ 4 2 2 39. 4 3 0 . 40. 4 2 0 4 0 4 5 0 5
Глава 4. Системы линейных уравнений и матрицы 365 2 -1 —2 ‘ ■ -1 -1 4 " 41. —4 2 8 42. 3 3 -22 6 —2 -1 —2 -1 19 43. Покажите, что (А 1) 1 == А дпя А = 44. Покажите, что (АВ)-1 = В-1 А-1 для 2 5 3 7 и В = * 45. Обсудите вопрос существования М 1 для диагональной матрицы а О О d М = Обобщите полученные выводы для произвольной диагональной матрицы с размерами П X и. *46. Обсудите вопрос существование М-1 для верхней треугольной матрицы а b О d М = Обобщите полученные выводы для произвольной верхней треугольной матрицы с размерами и х п. В задачах 47-49 вычислите матрицы А 1 и А2. 47. А = 48. А = 2 49. А = * 50. Основываясь На наблюдениях, сделанных в задачах 47-49, дайте ответ на следующий вопрос. Если для квадратной матрицы А выполняется равенство А-1 = А, то чему равна матрица Аа? Ответ аргументируйте математически. ьсъ В задачах 51-54 с помощью графической утилиты вычислите обратные матрицы (если они существуют). 51. ’ 6 5 0 2 3 5 2 0 3 2 -1 1 4 ‘ 1 -2 0 4 2 4 ‘ 1 52. ’ -2 2 0 2 1 2 4 -1 2 -3 2 6 0 2 -1 0 3 4 -1 ' 5 7 5 4 5 5 ' 6 -3 1 2 3 4 3 53. -1 1 2 -2 3 54. -1 -2 -1 2 3 3 3 1 0 1 1 6 1 6 4 1 1 2 0 2 1 —4 3 —7 —4
366 Часть II. Конечная математика Применение математики Социальные науки В задачах 55-58 речь идет о следующей матрице кодировки 55. Криптография. Зашифруйте фразу “THE SUN ALSO ARISES” с помощью матрицы А. 56. Криптография. Зашифруйте фразу “THE GRAPES OF WRATH” с помощью матрицы А. 57. Криптография. Приведенная ниже фраза зашифрована с помощью матрицы А. Расшифруйте ее. 37 52 24 29 46 69 49 69 8 8 36 44 5 5 41 50 22 26 58. Криптография. Приведенная ниже фраза зашифрована с помощью матрицы А. Расшифруйте ее. 9 13 40 49 29 34 2 3 22 26 6 9 43 57 29 34 54 74 XX В задачах 59-62 нужно использовать графический калькулятор или компьютер. Ниже приведена матрица шифрования В. Постройте матрицу с пятью строками и необходимым количеством столбцов для каждой из фраз. 10 10 1 0 110 3 2 1111 0 0 10 2 1112 1 59. Криптография. Зашифруйте фразу “THE BEST YEARS OF OUR LIVES” с помощью матрицы В. 60. Криптография. Зашифруйте фразу “THE BRIDGE OF RIVER KWAI” с помощью матрицы В. 61. Криптография. Приведенная ниже фраза зашифрована с помощью матрицы В. Расшифруйте ее. 32 34 60 19 40 24 21 67 11 69 27 44 85 16 85 29 65 82 28 82 21 66 44 41 62 8 0 16 0 8 62. Криптография. Приведенная ниже фраза зашифрована с помощью матрицы В. Расшифруйте ее. 28 22 56 11 36 30 27 75 15 78 30 51 64 30 62 39 30 58 25 44
Глава 4. Системы линейных уравнений и матрицы 367 4.6. Матричные уравнения и системы линейных уравнений ■ Матричные уравнения ■ Матричные уравнения и системы линейных уравнений ■ Решение практических задач С помощью единичной И обратной матриц, обсуждавшихся в предыдущем разделе, можно построить метод решения некоторых простых матричных уравнений. Методы решения матричных уравнений — это другой важный подход к решению систем линейных уравнений, применимый для независимых систем, у которых количество уравнений совпадает с количеством неизвестных. Если же система зависима, или количество неизвестных не совпадает с количеством уравнений, то нужно воспользоваться методом исключения Гаусса. Матричные уравнения Прежде чем приступить к решению матричных уравнений, целесообразно коротко повторить основные свойства действительных чисел, приведенные в приложении А.2, т. 2, и методы решения линейных уравнений (приложение А.8, т. 2). Задание 4.13. Пусть а, b и с — действительные числа, причем а 0. Решите приведенные ниже уравнения относительно переменной х. 1. ах = b. 2. ах + b = с. ■ Подходы к решению простых Матричных уравнений очень похожи и на самом деле следуют из методов решения линейных уравнений с действительными числовыми коэффициентами. Однако для матричных уравнений свободы в действиях меньше, поскольку матричное произведение некоммутативно. Основные свойства матриц, которые используются для решения матричных уравнений, резюмированы в теореме 4.5. Теорема 4.5 (Основные свойства матриц). Свойства сложения Ассоциативность А + (В + С) = (А + В) + С. Коммутативность А + В = В + А. Аддитивная единица А + 0 = 0 + Д = А. Аддитивный обратный элемент Свойства умножения А+(—А) = (—А) + А = 0. Ассоциативность А(ВС) = (АВ)С. Мультипликативная единица А1 = 1А = А. Мультипликативный обратный элемент Если А — квадратная матрица и существует А-1, то АЛ"1 = А-1 А = I.
368 Часть II. Конечная математика Комбинированные свойства Дистрибутивность слева Дистрибутивность справа Равенство Сложение Умножение слева Умножение справа А(В + С) = АВ 4- АС. (В + С) А = ВА + С А. Если А = В, то А -I- С — В + С. Если А = В, то С А = СВ. Если А = В, то АС = ВС. Использование этих свойств при решении матричных уравнений лучше всего показать на примере. Пример 4.31 (Решение матричного уравнения). Задана матрица А размером п х п и столбцы В и X размером п х 1. Решите матричное уравнение АХ = В, считая, что все необходимые обратные матрицы существуют. Решение. Требуется найти столбец X, удовлетворяющий уравнению АХ — В. Для этого умножим обе части уравнения на матрицу А-1 (считая, что эта матрица существует) и выделим неизвестную матрицу X в левой части. АХ = В Используем свойство умножения слева А-1 (АХ) = А-1В Используем свойство ассоциативности (А~1А)Х = А'1 В А-1 А = I IX = А-1 В IX = 1 X = А-1 В Упражнение 4.31. Задана матрица А размером п х п и столбцы В, С и X размером п х 1. Решите матричное уравнение АХ + С = В, считая, что все необходимые обратные матрицы существуют. ■ Предупреждение. Нельзя путать между собой умножение слева и умножение справа. Если АХ = В, то А"1 (АХ) ^ВА-1. Матричные уравнения и системы линейных уравнений Теперь мы переходим к рассмотрению способов решения независимых систем линейных уравнений, у которых количество неизвестных совпадает с количеством уравнений. В первую очередь запишем такую систему в матричной форме вида АХ = В, а затем воспользуемся решением этого уравнения X = А-1 В, полученным в примере 4.31. Пример 4.32 (Использование обратных матриц для решения систем линейных уравнений). Решите систему уравнений с помощью обратной матрицы. Xi — Х2 + Х3 = 1, 2ж2-х3 = 1, (4.20) 2a?i 4- 3^2 = 1.
Глава 4. Системы линейных уравнений и матрицы 369 Решение. Быстрее всего эта система решается с помощью вычисления матрицы, обратной по отношению к Матрице коэффициентов. А = О 2 -1 1 2 -1 3 О Представим систему (4.20) В матричной форме. А 1 -1 0 2 2 3 xi %3 В 1 1 1 (4.21) Проверьте, действительно ЛИ матричное уравнение (4.21) эквивалентно системе (4.20), вычислив произведение в левой части и приравнивая соответствующие элементы в левой и правой частях. В этом проявляется еще одна важная причина определения матричного произведения. Теперь нам нужно Найти столбец X, удовлетворяющую уравнению АХ = В. В примере 4.31 было показано, что если матрица А-1 существует, то Х = А~ГВ. Матрица, обратная к матрице А, была найдена в примере 4.27 раздела 4.5. 3 А-1 = —2 3 -1 ‘ -2 1 -5 2 Итак, 5 -3 —7 Следовательно, решение исходной системы имеет вид: х± = 5, = -3 и хз = —7. Проверьте это решение подстановкой в систему (4.20). ■ Упражнение 4.32. Решите систему уравнений с помощью обратной матрицы. 3i?i *- Х2 4- хз = 1, -£1+^2 =3, Х1 + я3 = 2. (Замечание: матрица, обратная к матрице коэффициентов, была найдена в упражнении 4.27 раздела 4.5.) ■
370 Часть II. Конечная математика На первый взгляд может показаться, что решение систем методом обратной матрицы является более трудоемким, чем с помощью процесса исключения Гаусса-Джордана. Преимущество использования обратной матрицы становится очевидным при решении нескольких систем уравнений с одинаковыми матрицами коэффициентов и постоянных членов. Пример 4.33 (Использование обратных матриц для решения систем линейных уравнений). Решите системы уравнений с помощью обратной матрицы. 1. хг — х2 + х3 = 3, 2. xi — х2 4- хз = —5, 2х2 - х3 = 1, 2х2 - х3 = 2, 2xi + Зх2 = 4. 2xi + Зх2 = -3. Решение. Обратите внимание на то, что матрица коэффициентов А в обеих системах такая же, как и в системе (4.20). Отличие от этой системы — только в постоянном слагаемом правой части. Поэтому для решения данных систем можно воспользоваться матрицей А-1, как это было сделано в примере 4.32. X А-1 В 3 3 -1 ■ ■ 3 ' Г 8' 1. — —2 -2 1 1 = —4 —4 -5 2 4 -9 Иначе говоря, xi = 8, х2 = — 4 и х3 = -9. Иначе говоря, xi = —6, х2 = 3 и х3 = 4. Упражнение 4.33. Решите системы уравнений с помощью обратной матрицы (см. упражнение 4.32). 1. 3xi х2 1 Х3 — 3, -xi 4- х2 = -3, xi + х3 = 2. 2. 3xi — х2 4- х3 = —5, —xi+x2 = 1, £1 + х3 = —4. ю. Из примеров 4.32 и 4.33 можно сделать вывод, что методы обратной матрицы очень эффективны для вычислений вручную. Достаточно один раз найти обратную матрицу, и можно решать любую систему уравнений, которая отличается от исходной системы правой частью. Поскольку большинство графических утилит имеют средства для поиска обратных матриц, их можно использовать для решения матричных уравнений (рис. 4.20). Однако если утилита имеет встроенную программу для преобразования расширенной матрицы в приведенную форму, то более удобным будет метод исключения Гаусса-Джордана. Более того, метод исключения Гаусса-Джордана может быть использован во всех случаях, как было отмечено выше, в том числе и тогда, когда матричные методы не применимы.
Глава 4. Системы линейных уравнений и матрицы 371 А 3 С 1 о гп 1 F 1 1 G | 1 Н 1 1 1 1 1 J 1 А В X в X 2 1 -1 1 3 8 -5 -6 3 0 2 -1 1 -4 2 3 4 1Ч> 3 0 4 -9 -3 4 Рис. 4.20< Использование метода обратной матрицы в электронной таблице: значения полей G2:G4 получены с помощью функции МУМНОЖ(МОБР(В2:П4);Г2:Р4) Использование метода обратной матрицы для решения систем линейных уравнений Предположим, что количество уравнений совпадает с количеством неизвестных, и матрица коэффициентов имеет обратную матрицу. Тогда система уравнений имеет единственное решение, которое можно найти с помощью обратной матрицы коэффициентов, решая соответствующее матричное уравнение. Матричное уравнение Решение АХ = В Х = А'1 В А что будет, если Обратной матрицы коэффициентов не существует? В таком случае не будет единственного решения, а сама система может быть или зависимой, или несовместной. Для определения типа решения нужно воспользоваться методом исключения Гаусса. Кроме того, как упоминалось выше, метод исключения Гаусса всегда используется для решения систем, у которых количество неизвестных не совпадает с количеством уравнений. Решение практических задач Следующий пример — иллюстрация эффективности применения обратной матрицы для решения систем линейных уравнений. И Пример 4.34 (Анализ капиталовложений). Консультант по инвестициям может предложить вкладчикам два способа вложения денежных средств: умеренное капиталовложение А с 10% годовых, и более рискованное капиталовложение В с 20% годовых. Клиенты могут разделить свои инвестиции между предложенными схемами и получить доход между 10 и 20%. Однако чем выше доход, тем больше риск. Каким образом каждый из клиентов разделяет свои Инвестиции для получения указанного дохода? Вкладчик к 1 2 3 Общий объем инвестиций, долл. 20000 50000 10000 fcl Годовой доход, долл. 2400 7500 1300 (12%) (15%) (13%) Решение, В ответе на вопрос задачи будет шесть величин — по две на каждого клиента. Их удобнее всего найти с помощью обратной матрицы. Мы будем решать задачу для произвольного вкладчика, Который инвестирует неопределенную сумму к\ и получает
372 Часть II. Конечная математика годовой доход к2. (Обратите внимание на то, что и к2 не являются переменными. Их значения заданы в условии для каждого клиента.) Введем следующие обозначения: xi — сумма инвестиций по схеме А для данного клиента, х2 — сумма инвестиций по схеме В для данного клиента. Получим следующую математическую модель. Х\ + х2 = к\ Общий объем инвестиций 0,lxi + 0,2x2 — к2 Годовой доход Представим эту систему в виде следующего матричного уравнения. А X В ■ 1 1 Xi _ Г fci ' 0,1 0,2 . Х2 . [ к2 _ Если существует матрица А \ то решение будет иметь такой вид. X = А~ХВ. Для вычисления матрицы А 1 построим расширенную матрицу [А | 1] и воспользуемся процедурой, описанной в разделе 4.5. ■ 1 1 1 0 ‘ о,1 0,2 0 1 10^2 —► В>2 ■ 1 1 1 0 ‘ 1 2 0 10 —.Ri + В>2 —► В2 " 1 1 1 0 ' —2R2 + Ri —► R± 0 1 -1 10 ■ 1 0 2 -10 ' 0 1 -1 10 Таким образом, 2 -10 ' 2 -10 ‘ 1 1 ■ 1 0 ‘ -1 10 Проверка: -1 10 . О’1 0,2 — 0 1 X А"1 в XI ' 2 -10 ' ■ кг ’ Х2 . -1 10 Чтобы решить задачу для каждого из клиентов, замените параметры fci и к2 соответствующими значениями, приведенными в таблице, и вычислите произведение с матрицей А-1. Вкладчик 1 Xi - [ 2 -10 ' ■ 20000 ' _ Г 1бооо ' х2 ~ 1. -1 10 2400 ~ [ 4000
Глава 4. Системы линейных уравнений и матрицы 373 Решение'. х± = 16 000 Долл, для инвестиций по схеме А, Х2 = 4000 долл, для инвестиций по схеме В. Вкладчик 2 ’ *1 ' 2 -10 ' ’ 50000 ‘ ’ 25000 ' «2 -1 10 7500 25000 Решение', xi = 25 000 Долл, для инвестиций по схеме А, Х2 = 25 000 долл, для инвестиций по схеме В. Вкладчик 3 а?1 1 Г 2 -10 1 Г 10000 а?2 ] [ -1 Ю И 1300 7000 3000 Решение'. = 7000 долл. ДЛЯ инвестиций по схеме А, Х2 = 3000 долл, для инвестиций по схеме В. ■ ■га Упражнение 4,34. ШЙ Повторите решение примера 4.34, если для инвестиций по схеме А предусмотрено 8% годовых, а по схеме В — 24% годовых. ■ Ю- На рис. 4.21 показано решение примера 4.34 в электронной таблице. L _b . 1 1 C 1 1 D | 1 E| F | G 1 2 . 1 Clients 2 3 A 3 Total Investment $2o,ooo $50,000 $10,000 1 1 4 Annual Return 12,400 $7,500 $1,300 0.1 0.2 5 Amount Invested in A $16,000 $25,000 $7,000 6 Amount Invested in В $4,000 $25,000 $3,000 Рис. 4.2 L Решение примера с помощью электронной таблицы Задание 4.14. Вернемся к математической модели из примера 4.34. А 1 1 0,1 0,2 X *1 Х2 В к2 (4.22) 1. Всегда ли уравнение (4.22) имеет решение для произвольной постоянной матрицы В? 2. Все ли такие решения имеют смысл в контексте данной задачи? Если нет, приведите примеры. 3. Опишите возможные значения доходов к2, если общий объем инвестиций fci равен 10000 долл. ■
374 Часть II. Конечная математика Ответы к упражнениям 4.31. АХ + С = В, : (АХ + с) - с = в - с,: !АХ + (С-С) = В-С,; : ах + о = в-с,\ АХ = в'-'с, \ A-\Axj = А-1 (В-С),] \(А~гА)Х = А-1 (В -С),: ; IX = А~1(В-С),\ X = А~}(В -С). 4.32. xi = 2, Х2 = 5, хз = 0. 4.33. 1) xi = —2, Х2 = —5, хз = 4. 2) xi = 0, х2 = 1, хз = -4. 1,5 —6,25 -0,5 6,25 4.34. А-1 = ; вкладчик 1:15 000 долл, по схеме А, 5000 долл, по схеме В; вкладчик 2: 28 125 долл, по схеме А, 21 875 долл, по схеме В; вкладчик 3: 6875 долл, по схеме А, 3125 долл, по схеме В. Практикум 4.6 А В задачах 1-4 требуется записать матричные уравнения в виде систем уравнений. 1 0 0 1 3 “2 . -1 0 3 -1 0 3 5 —4 Xi Х2 Х3 _ ^1 Х2 хз 3 -4 2. -2 1 -3 4 Xi Х2 2 В задачах 5-8 требуется записать каждую систему уравнений в матричной форме вида АХ — В. 5. 3xi — 4x2 = 1, 2xi 4- Х2 = 5. 7. xi — 3x2 + 2хз = -3, —2xi-h3x2 = 1, Х1 4* Х2 4- 4хз = —2. 6. 2xi 4- Х2 = 8, —5x1 + 3x2 = —4. 8. 3xi 4~ 2хз = 9. -Х1 + 4х2 4- Хз = -7. —2x1 4- 3x2 = 6. В задачах 9-12 требуется вычислить xi и Х2. 9. Xi Х2 3 —2 1 4 10. Х1 ■ —2 1 ' 3 ' Х2 -1 2 -2
Глава 4. Системы линейных уравнений и матрицы 375 X! 3 -1 -2 Х2 J [0 2 J L 1 В задачах 13-16 требуется вычислить xi и Х2- 13. ■ 1 1 -1 ' -2 Х1 . Х2 . а= 1 1 1П> ь- 1 1 14. 15. ■ 1 2 1 ‘ -3 Х2 ■ 15 10 16. 13 xi _ 9 1 4 J L Ж2 J “ [ 6 3 —2 х2 20 1 Х1 _ 10 Б В задачах 17-24 требуется записать каждую систему в матричном виде и решить ее с помощью обратной матрицы. (Замечание: обратные матрицы были найдены в задачах 21-28 практикума 4,5.) 17. Х\ 4~ 2x2 — fci, Х1 4- 3^2 = a) k\ = 1, к% = 3. в) ki = -2, fc2 — !• 18. 2xi 4- х2 = fci, 5xi 4- Зх2 = fc2. 6) fci = 3, fc2 = 5. a) ki = 2, к% = 13. в) fci = 1, fc2 = “3. 19. Xi 4~ Зх2 fci, 2xi + 7х2 = fc2. 6) fci = 2, fc2 = 4. a) fci = 2, fc2 = —1. в) fci = 3, fc2 = —1. 20. 2xi 4- x2 = fci, Xi 4- x2 = fc2. 6) fci = 1, fc2 = 0- a) fci = —1, fc2 — “2. в) fci = 2, fc2 = 0. 21. xi — 3x2 = fci, x2 4- Хз =» fc2, 2xi- x2 + 4x3 — кз. 6) fci = 2, fc2 = 3. a) fci = 1, fc2 = 0, кз = 2. в) fci = 2, fc2 = —2, fc3 — 1. 22. 2xi4-3x2 » fci, xi 4- 2x2 + 3x3 — fc2, —x2 - 5x3 = fc3. 6) fci = -1, fc2 = 1, fc3 = 0. a) fci = 0, fc2 = 2, fc3 = 1. в) fci — 3, fc2 = 1, кз = 0. 23. xi 4- x2 —fci, 2xi + 3x2 — x3 = fc2, xi 4- 2x3 = fc3. 6) fci = —2, fc2 = 0, кз = 1.
376 Часть II. Конечная математика а) к± =2, к2 = 0, кз = 4. в) fci = 4, &2 = 2, кз = О. 24. х\ — хз = ki, 2а?1 — — к2у Х1+х2- 2х3 = к3. a) ki = 4, = 8, кз = О. в) fci = О, к2 = 8, кз — -8. б) fci = О, к2 = 4, кз = -2. б) fci = 4, к2 = О, кз — —4. В задачах 25-30 необходимо объяснить, почему приведенные системы нельзя решить с помощью обратной матрицы. Приведите метод, с помощью которого можно получить решение системы, и вычислите это решение. *25. —2xi-h 4x2 =—5, 6xi - 12x2 = 15. *27. xi — 3x2 — 2х3 = —1, —2xi Н- 5x2 И- 4х3 = 3. * 29. xi — 2x2 + Зхз = 1, 2xi — 3x2 — 2х3 = 3, xi — х2 — 5хз = 2. * 26. —2x1 + 4x2 = 5, 6x1 — 12x2 = 15. * 28. xi - 3x2 - 2х3 = -1 —2xi "И 7х2 + Зхз = 3. * 30. xi — 2x2 4- Зхз = 1, 2xi — 3x2 - 2х3 = 3, xi — х2 — 5х3 = 4. В В задачах 31-36 требуется решить указанные матричные уравнения, где А и В — матрицы размера п х п, С, D и X — матрицы размера п х 1, считая, что все необходимые обратные матрицы существуют. 31. АХ - ВХ = С. 32. АХ + ВХ = С. 33. АХ + X = С. 34. АХ - X = С. 35. АХ -С = D — ВХ. 36. АХ + С = ВХ + D. * 37. С помощью обратной матрицы решите следующую систему для указанных значений параметров ki и к2. xi -I- 2,001x2 = fci; xi + 2 х2 = к2. a) fci = 1, к2 = 1. б) ki = 1, к2 = 0. в) fci = 0, к2 = 1. Обсудите влияние малых изменений постоянных к± и к2 на множество решений системы. *38. Повторите решение задачи 37 для следующей системы. xi — 3,001x2 = fci; xi - 3 х2 = к2.
Глава 4. Системы линейных уравнений и матрицы 377 В задачах 39-42 необходимо представить каждую систему в матричном виде и вычислить ее решение с помощью обратной матрицы. Для вычислений используйте графическую утилиту 39. xi + &z?2 + Тхз =* 135, 6x1 + 6^2 4- 8x3 == 155, 3xi 4- 4x2 4- бхз = 75. 41. 6xi -I- 9x2 4- 7хз 4- 6x4 = 250, 6xi 4- 4x2 4“ 7хз 4- ЗХ4 = 195, 4xi 4- 5x2 4- Зхз 4- 2x4 — 145, 4xi 4- 3x2 4- 8x3 4- 2x4 =* 125. 40. 5xi 4* 3x2 — 2хз = 112, 7xi 4- 5x2 = 70, 3xi 4- Х2 - 9хз = 96. 42. 3xi 4" 3x2 4* 6x3 4~ 5x4 ~ Ю, 4xi 4- 5x2 4- 8х3 4- 2х4 = 15, 3xi 4- 6x2 + 7хз 4- 4x4 = 30, 4xi + Х2 4- 6x3 4- 3x4 = 25. Применение математики В каждой из представленных ниже задач постройте математическую модель. (В ответах в конце книги приведены как формулировки математических моделей, так и их интерпретации в контексте задачи.) Используйте матричные методы для решения уравнений модели и интерпретируйте полученные решения. Экономика и бизнес 43. Организация концертов. В концертном зале имеется 10000 мест, которые разделены на две категории по цене билетов — 4 и 8 долл. Будем считать, что все места каждой категорий могут быть проданы. Концерт 1 2 3 Продано билетов 10000 10000 10000 Вырученная сумма 56000 60000 68000 а) Сколько билетов каждой категории нужно продать, чтобы получить суммы, указанные в таблице? б) Можно ЛИ вырулить сумму 90 000 долл.? 30 000 долл.? Аргументируйте ответ. в) Опишите все возможные значения вырученных сумм. 44. Плата за парковку, В муниципальном зоопарке установлены следующие расценки за парковку: пять долларов для местных посетителей и 7,50 долл, для приезжих. В конце дня фиксируется общее количество парковавшихся автомобилей и общая сумма, уплаченная за парковку, однако количество автомобилей каждой категории ие учитывается. В следующей таблице приведена информация за четырехдневный период. День 1 2 3 4 Количество парковавшихся машин, шт. 1200 1550 1740 1400 Вырученная сумма, долл. 7125 9825 11100 8650 а) Сколько автомобилей Каждой категории парковалось под зоопарком в каждый из дней?
378 Часть II. Конечная математика б) Допустим, что в некоторый день парковалось 1200 автомобилей. Возможна ли выручка 5000 или 10000 долл, в этом случае? в) Опишите все возможные значения вырученных сумм, если общее количество парковавшихся автомобилей равно 1200. 45. Планирование производства. Поставщик автомобильных комплектующих производит рамы к автомобилям и прицепам на двух различных заводах. Данные об объеме выпуска (рам в час) представлены в следующей таблице. Завод Рамы машин, шт/ч Рамы прицепов, шт/ч А 16 5 Б 8 8 В какой срок каждый из заводов выполнит следующие объемы заказов? Заказы 1 2 3 Рамы машин, шт 3000 2800 2600 Рамы прицепов, шт 1600 2000 2200 * 46. Планирование производства. Затраты на покупку материалов и оплату труда при производстве двух моделей гитар приведены в следующей таблице. Модель Оплата Стоимость гитары труда материалов А 30 20 Б 40 30 а) Предположим, что общие затраты на материалы и оплату труда ограничены суммой 3000 долл. Сколько гитар каждой модели можно выпустить, если эта сумма разделена так, как указано в следующей таблице? Распределение затрат по моделям, долл. 1 2 3 Оплата труда, долл. 1800 1750 1720 Стоимость материалов, долл. 1200 1250 1280 б) Можно ли распределить средства следующим образом: 1600 долл, для оплаты труда и 1400 долл. — на материалы? Можно ли разделить ее так, что 2000 долл, будут потрачены на оплату труда и 1000 долл. — на материалы? Аргументируйте ответ. 47. Поощрительные вознаграждения. Маленькая компания обеспечивает поощрительными вознаграждениями некоторых высших руководителей. Каждый из них получает премию в виде процентов с суммы ежегодной прибыли, после того как с нее будет вычтена премия других руководителей (см. таблицу). Вычислите с точностью до 100 долл, премии руководителей компании, если ежегодная прибыль равна 2 млн. долл.
Глава 4. Системы линейных уравнений и матрицы 379 Руководящая должность Премия, % Президент Исполнительный вице-президент Ассоциированный вице-президент Помощник вице-президента 3 2,5 2 1,5 48. Поощрительные вознаграждения. Повторите решение задачи 47, если в список премий включен менеджер по продажам с 1% премиальных. Биологические науки *49. www Диета, Процентное содержание белков и жиров в двух промышленно выпускаемых пищевых смесях приведено в следующей таблице. Смесь Белки, % Жиры, % А 20 4 Б 14 3 а) Сколько унций каждой смеси необходимо использовать для приготовления блюд, перечисленных в следующей таблице? Блюдо 1 2 Белки, унции 80 90 100 Жиры, унции 17 18 21 б) Можно ЛИ приготовить блюдо, содержащее 100 унций белков и 22 унции жиров или 80 унций белков и 15 унций жиров? Аргументируйте ответ. Социальные науки 50. Образование: распределение ресурсов. Университет огласил вакансии на должности лекторов и преподавателей практических занятий некоторых из двухгодичных курсов. Количество востребованных сотрудников и их годовая зарплата приведены в следующей таблице. Должность Лектор Преподаватель Количество преподавателей 3 4 Годовая зарплата, тыс, долл. 20 25 Количество групп На каждом курсе и зарплатный фонд (тыс. долл.) приведены в следующей таблице. Сколько лекторов и преподавателей можно пригласить в соответствии с бюджетными и кадровыми требованиями? Курс __ 12 3 Количество групп 30 33 35 Зарплатный фонд, тыс, долл. 200 210 220
380 Часть II. Конечная математика 4.7. Анализ межотраслевых связей по Леонтьеву ■ Двухотраслевая модель ■ Трехотраслевая модель WWW Одно из важных прикладных применений матриц и обратных матриц называется анализом межотраслевых связей. Наиболее весомый вклад в эту область внес Василий Леонтьев, лауреат Нобелевской премии по экономике 1973 года. Разработанные им методы нашли успешное применение в экономическом планировании в индустриальных странах. Помимо всего прочего, он тщательно исследовал 500 секторов экономики Соединенных Штатов и связи между ними. Конечно, в этом исследовании широко применялись мощные компьютеры. Цели нашего нынешнего исследования будут более скромными. Сначала мы рассмотрим модель экономики, состоящую из двух отраслей. Основные определения и идеи, которые будут представлены в этом простом введении, можно обобщить и на более реалистичные модели экономики. Целью анализа затрат и результатов является поиск условий равновесия, при котором отрасли экономики выпускают достаточно продукции, чтобы обеспечить как внешние, так и внутренние потребности. При заданном объеме спроса внутри различных отраслей задача состоит в поиске объема производства, который удовлетворяет требованиям внешнего спроса. Двухотраслевая модель Для определенности рассмотрим гипотетическую систему экономики, состоящую из двух отраслей, одну из них представляет электроэнергетическая компании Е, а другую — гидротехническая компания W. Продукцию каждой компании будем измерять в долларах. Электроэнергетическая компания потребляет электричество и воду (потребление) для производства электричества (выпуск). Допустим, что на производство одного доллара электроэнергетическая компания затрачивает 0,30 долл, в виде потребленного электричества и 0,10 долл, потребленной воды, а один доллар, произведенный гидротехнической компанией, обходится в 0,20 долл, в виде потребленного электричества и 0,40 долл, потребленной воды. Если общий спрос остальных секторов экономики (т.е. спрос всех других потребителей воды и электричества) составляет di = 12 млн. долл, на электричество, d,2 = 8 млн. долл, на воду, то какой объем производства сможет покрыть указанные потребности? Для начала допустим, что объем производства электроэнергетической компании равен 12 млн. долл., а гидротехнической — 8 млн. долл. (т.е. как раз столько, сколько нужно для покрытия внешнего спроса). Для его достижения компаниям потребуются следующие средства. Для производства электричества нужно электричество 0,3 • 12 Для производства электричества нужна вода + 0,2-8 5,2 млн. долл, на электричество
Глава 4. Системы линейных уравнений и матрицы 381 Для производства воды нужно электричество 0,1 • 12 Для производство воды нужно вода + 0,4-8 = 4,4 млн. долл, на воду Иначе говоря, внешним Потребителям придется довольствоваться электричеством, стоимостью 6,8 млн. долл, и водой стоимостью 3,6 млн. долл. Таким образом, для удовлетворения внутреннего й внешнего спроса одновременно компании должны производить больше, чем требуют внешние потребители. В сущности, компании должны производить продукцию в объеме, удовлетворяющем внутреннее потребление плюс внешний спрос. Задание 4.15. Допустим, что электроэнергетическая компания удвоила объем производства до 24 млн. долл., а гидротехническая — до 16 млн. долл. Как теперь будут расходоваться электричество и вода на производственные нужды? Сколько воды и электричества останется для внешних потребителей? Попытайтесь подобрать методом проб и ошибок уровень производства, при котором внешним потребителям выделяется электричество стоимостью 12 млн. долл, и вода стоимостью 8 млн. долл. ■ Теперь мы можем сформулировать постановку основной задачи в методе анализа затрат и результатов. Основная задача межотраслевых связей При заданном внутреннем спросе на продукцию каждой отрасли нужно определить объем производства каждой из отраслей для удовлетворения заданного внешнего спроса. Введем следующие обозначения. xi — общий объем производства электроэнергетической компании, Х2 — общий объем производства гидротехнической компании. Следовательно, внутренний спрос будет равен следующим величинам. 0,3^1 + 0,2^2 Внутренний спрос на электричество 0,1X1 + 0,4хг Внутренний спрос на воду Объединив внутренний и внешний спрос, получим требуемый объем производства компаний в виде системы уравнений. Общий объем Внутренний Внешний производства спрос спрос xi = 0,Зж1 + 0,2хз “И di Х2 = 0,1X1 + 0,4X2 + ^2 В матричной форме она записывается так. Xi _ 0,3 0,2 хд “ 0,1 0,4 Xi di х2 d2 т.е. X = MX + О, (4.24)
382 Часть II. Конечная математика где введены следующие обозначения: Матрица внешнего спроса Матрица объема производства Технологическая матрица Технологическая матрица — базовое понятие в анализе межотраслевых связей. Элементы технологической матрицы можно определить так (читается слева направо и затем вверх). Производство Е W т Потребление (Потребление Е ддя производства электричества стоимостью 1 долл. у (Потребление W ддя производства электричества стоимостью 1 долл. Потребление Е для^ производства воды ^стоимостью 1 долл.у Потребление W для' производства воды стоимостью 1 долл. Теперь нужно решить уравнение (4.24) относительно матрицы X. Как и в разделе 4.6, получаем следующие матричные уравнения. X = MX + D X - MX = D IX - MX = D (/ - M)X = D X = (I-M^D Считаем, что матрица I — M имеет обратную Пропуская промежуточные вычисления, получаем следующий результат. 1-М = 0,7 -0,2 -0,1 0,6 Итак, 1,5 0,5 1 Г di 0,25 1,75 d2 (1-М)-1 = 1,5 0,25 0,5 1,75 ■ 1,5 0,5 г 12 ' ’ 22 0,25 1,75 8 17 (4-25) (4.26) = М 1 0 ^1 Х2 В результате внутренний и внешний спрос полностью удовлетворяются одновременно при объеме производства электроэнергетической компании, равном 22 млн. долл., и объеме производства гидротехнической компании, равном 17 млн. долл.
Глава 4. Системы линейных уравнений и матрицы 383 Проверка. Проверим Полученный ответ подстановкой в уравнение (4.24). X = MX + D. ' 22 ‘ 17 ]_ ’ 22 ‘ 17 — ' 22 ‘ 17 — 0,3 0,2 22 0,1 0,4 ] 17 22 17 Ю 1 Г 12 9 ] + 8 12 8 Kl Для решения этой задачи с помощью графической утилиты нужно просто ввести матрицы М, D и I в память, а затем использовать уравнение (4.25) для поиска X и уравнение (4.24) для проверки ответа. Процесс решения на графическом калькуляторе показан на рис. 4,22. м [[.3 .2] [.1 .411 0 [[12] Ее ]] I [[1 0] [0 и а) сохраните матрицы М, D и 1 в памяти графического калькулятора (122] [171J M*X+D [ [221 [171 ] б) вычислите вектор X и проверьте ответ в уравнении (4.24) Рис. 4.22. Решение системы уравнений с помощью графического калькулятора Очевидно, уравнение (4.26) пригодно для решения задачи с произвольно заданными значениями внешнего спроса d\ и cfo- Это очень удобно, поскольку уравнение (4.26) позволяет быстро получить решения не только для значений внешнего спроса исходной задачи. Если бы для решения системы уравнений (4.23) мы использовали процедуру исключения Гаусса-Жордана, то при изменении значений внешнего спроса уравнения пришлось бы решать заново. Допустим, что через пять лет объемы внешнего спроса будут равны di = 24 (млн. долл.) и d2 = 16 (млн. Долл.). Для определения уровня производства компаний достаточно подставить новые значения В уравнение (4.26) и вычислить следующее произведение. #2 1,5 0,5 0,25 1,75 24 16 44 34 Решение задачи анализа двухотраслевого баланса Предположим, что две отрасли С\ и С% описываются следующими матрицами. Технологическая матрица С1 с2 м Ci Г ап ai2 1 , Cz2 <^21 ^22 Матрица производства Матрица внешнего спроса di d>2 *2
384 Часть II. Конечная математика где aij — продукция Сг, которая требуется для производства одного доллара продукции Cj. Решением матричного уравнения межотраслевых связей Объем Внутренний Внешний производства спрос спрос X = MX + D (4.27) является X = (I-Mj-'D при условии, что для матрицы I — М существует обратная. (4.28) Трехотраслевая модель Уравнения (4.27) и (4.28), полученные для двухотраслевой модели экономики, не изменятся по форме и для трехотраслевой модели, четырехотраслевой и, вообще, для экономики с п отраслями (где п — произвольное натуральное число). Если матрицы имеют корректные размеры и матрица (/ — М)-1 существует, то алгоритм получения уравнения (4.28) из уравнения (4.27) не изменяется. Задание 4.16. Определите, какие должны быть размеры матриц в уравнениях (4.24) и (4.25) для экономики с п отраслями. ■ В следующем примере уравнения (4.24) и (4.25) используются для трехотраслевой модели экономики. И Пример 4.35 (Анализ межотраслевых связей). Экономика состоит из трех отраслей: сельское хозяйство (А), энергетика (Е) и машиностроение (М). Для производства продукции, стоимостью 1 долл., в сельскохозяйственной отрасли требуется 0,20 долл, затрат сельскохозяйственной отрасли и 0,40 долл, затрат отрасли энергетики. Для производства продукции на 1 долл, в отрасли энергетики требуется 0,20 долл, затрат отрасли энергетики и 0,40 долл, отрасли машиностроения. Для производства продукции, стоимостью 1 долл, в отрасли машиностроения требуется 0,10 долл, затрат сельскохозяйственной отрасли, 0,10 долл, затрат отрасли энергетики и 0,30 долл, затрат отрасли машиностроения. Вычислите объемы производства в каждой из отраслей, если внешний спрос составляет 20 млрд. долл, для сельскохозяйственной отрасли, 10 млрд. долл, для отрасли энергетики и 30 млрд. долл, для отрасли машиностроения. Решение, Поскольку в данной задаче рассматривается три отрасли, то технологическая матрица будет иметь размер 3 х 3, а матрицы объема производства и внешнего спроса — размер 3x1. Построим их, используя условие задачи. Матрица внешнего Матрица объема Технологическая матрица спроса производства А Е М А ' 0,2 0 ОД " Г 20 ‘ Г М = Е 0,4 0,2 ОД , D = 10 , х = Х2 М 0 0,4 0,3 30
Глава 4. Системы линейных уравнений и матрицы 385 где матрицы М, X и D удовлетворяют основному уравнению X = MX + D, Поскольку решением этого уравнения является матрица X = (Z — то сначала нужно найти матрицу I — М, а затем — матрицу (/ — М)-1. Пропуская подробности вычислений, получим следующий результат. 0,8 0 -0,1 ' ' 1,3 0,1 0,2 " 1-М = -0,4 0,8 -0,1 и 0,7 1,4 0,3 0 -0,4 0,7 0,4 0,8 1,6 Следовательно, матрица общего объема производства будет иметь следующий вид. А (I - М)-1 D 1,3 0,1 0,2 20 33 »Г2 0,7 1,4 0,3 10 = 37 0,4 0,8 1,6 30 64 Таким образом, при Объеме производства в 33 млрд. долл, для сельскохозяйственной отрасли, 37 млрд. долл. — для энергетики и 64 млрд. долл. — для машиностроительной отрасли удовлетворяются все требования внутреннего и внешнего спроса. ■ * Решение примера 4.35 с помощью электронной таблицы показано на рис. 4.23. aJ 1 IL 1 С , J D. 1 1 е| Г F 1 G 1 Н 1 III 1 J 1 И L L 1 Im 1 Technology Matrix М Fi nal Output 2 А _ Е .. М 1 - м Demand 3 А 0,2 0 0.1 0.8 0 -0.1 20 33 4 Е 0.4 0.2 0.1 -0.4 0.8 -0.1 10 37 5 М 0 0.4 0.3 0 -0.4 0.7 30 64 Рис. 4.23. Ответ, записанный в ячейках L3:L5, получается с помощью функции МУМНОЖ(МОБР(РЗ:Н5);13:15) ■га Упражнение 4.35. Bid Экономика состоит из трех отраслей: угледобывающей, нефтедобывающей и транспортной. Для производства продукции на 1 долл, в угледобывающей отрасли требуется 0,20 долл, затрат угледобывающей отрасли и 0,40 долл, затрат транспортной отрасли. Для производства продукции, стоимостью 1 долл., в нефтедобывающей отрасли требуется 0,10 долл, затрат нефтедобывающей отрасли и 0,20 долл, затрат транспортной области. Для предоставления услуг на 1 долл, в транспортной отрасли требуется 0,40 долл, затрат угледобывающей отрасли, 0,20 долл, затрат нефтедобывающей отрасли и 0,20 долл, затрат транспортной отрасли. 1. Вычислите технологическую матрицу М. 2. Вычислите матрицу (/ — М)”1. 3. Вычислите объемы производства в каждой из отраслей, если внешний спрос со¬ ставляет 20 млрд. долл, на продукцию угледобывающей отрасли, 10 млрд. долл, на продукцию нефтедобывающей отрасли и 20 млрд. долл, на услуги транспортной отрасли. в
386 Часть II. Конечная математика Ответы к упражнениям 4.35. 1) 2) ' 0,2 0 0,4 ' 0 0,1 0,2 . °>4 0,2 0,2 _ ’ 1,7 0,2 0,9 ' 0,2 1,2 0,4 0,9 0,4 1,8 3) 71 млрд. долл, для угледобывающей отрасли, 26 млрд. долл, для нефтедобывающей отрасли и 67 млрд. долл, для транспортной отрасли. Практикум 4.7 А В задачах 1-6 идет речь о модели экономики с двумя отраслями: сельского хозяйства А и энергетики Е. Ниже приведена технологическая матрица М и матрицы внешнего спроса (млрд. долл). Di = 6 4 А Е А 0,4 0,2 Е 0,2 0,1 ' 8 = 5 = м, D3 = 12 9 1. Какие затраты в отраслях А и Е требуются для производства продукции на 1 долл, в отрасли А? 2. Какие затраты в отраслях А и Е требуются для производства продукции на 1 долл, в отрасли Е? 3. Вычислите матрицы I — М и (I — М)-1. 4. Вычислите объем производства в каждой отрасли для обеспечения внешнего спроса D\. 5. Повторите решение задачи 4 для внешнего спроса D%. 6. Повторите решение задачи 4 для внешнего спроса D3. Б В задачах 7-12 идет речь о модели экономики с тремя отраслями: сельского хозяйства А, строительства В и энергетики Е. Ниже приведена технологическая матрица М и матрицы внешнего спроса (млрд. долл.). А В м А ’ 0,3 0,2 0,2 ‘ В о,1 0,1 0,1 = м, М 0,2 0,1 ОД 5 ' 20 ‘ 10 1 £>1 = 15 15 10 7. Какие затраты в отраслях А, В и Е требуются для производства продукции на 1 долл, в отрасли В?
Глава 4. Системы линейных уравнений и матрицы 387 8. Какая часть Производства отрасли В расходуется на внутренний спрос во всех трех отраслях? 9. Покажите, ЧТО 0,7 -0,2 -0,2 ’ 1~М = -0,1 0,9 -0,1 -0,2 -0,1 0,9 10. Пусть ’ 1,6 0,4 0,4 (7-М)"1 = 0,22 1,18 0,18 0,38 0,22 1,22 Покажите, что (/ - М) — М) = I. 11. Воспользуйтесь матрицей (Z — М)~г из задачи 10 для определения объема производства в каждой отрасли для обеспечения внешнего спроса Di. 12. Повторите решение задачи 11 для внешнего спроса D2. В В задачах 13-16 требуется вычислить матрицы (I — М) 1 и X, 10 25 ■ 0,3 0,1 0,3 ' ' 20 ' 15. М = 0,2 о,1 0,2 , D = 5 . О’1 0,1 0,1 _ . 10 ’ 0,3 0,2 0,3 ' ' 10 ' 16. М = 0,1 0,1 0,1 , D = 25 0,1 0,2 о,1 15 17. Технологическая матрица для экономики, состоящей из двух отраслей — сельского хозяйства А и промышленного производства М, имеет следующий вид. А М А Г 0,3 0,25 М ~ М 0,1 0,25 а) Вычислите общий объем производства, который необходим, чтобы обеспечить внешний спрос 40 млрд. долл, для сельского хозяйства и 40 млрд. долл, для промышленного производства. *б) Как повлияет на внешний спрос увеличение общего объема производства в отрасли сельского хозяйства на 20 млрд. долл, при неизменном уровне промышленного производства? 18. Технологическая матрица для экономики, состоящей из двух отраслей — энергетической Е и транспортной Т, имеет следующий вид. Е Т м _Е Г 0,25 0,25 М ~ Т |_ 0,4 0,2
388 Часть II. Конечная математика а) Вычислите общий объем производства, который необходим, чтобы обеспечить внешний спрос 50 млрд. долл, для энергетической и 50 млрд. долл, для транспортной отраслей. *б) Как повлияет на внешний спрос увеличение общего объема производства в отрасли транспорта на 40 млрд. долл, при неизменном уровне производства в энергетике? * 19. Технологическая матрица для экономики, состоящей из двух отраслей — энер¬ гетики Е и горной промышленности М, имеет следующий вид. Е М м _ Е Г 0,2 0,3 ' М [ 0,4 0,3 / Руководители этих двух отраслей намерены установить такой уровень производства, чтобы внешний спрос составлял 40% от уровня производства. Придумайте метод, с помощью которого можно произвести расчет. * 20. Технологическая матрица для экономики, состоящей из двух отраслей — авто¬ мобильной А и строительной промышленности С, имеет следующий вид. А С А Г 0,1 0,4 ' М ~ С 0,1 0,1 ’ Руководители этих двух отраслей намерены установить такой уровень производства, чтобы внешний спрос составлял 70% от уровня производства. Придумайте метод, с помощью которого можно произвести расчет. *21. Во всех технологических матрицах, которые встречались в тексте, элементы принимают значения между 0 и 1. Почему? Может ли элемент технологической матрицы быть отрицательным, нулем, единицей или превышать единицу? * 22. Во всех технологических матрицах сумма элементов в столбце меньше единицы. Почему? Может ли существовать столбец с суммой элементов, равной единице или превышающей ее? Что можно сказать об экономической системе, для которой сумма элементов в любом столбце равна единице? Применение математики Экономика и бизнес 23. Экономика состоит из двух индустриальных секторов: угольной и металлургической промышленности. Для производства продукции, стоимостью один доллар в угольной промышленности требуется 0,10 долл, затрат угольной отрасли и 0,20 долл, затрат металлургической отрасли. Для производства продукции, стоимостью один доллар в металлургической промышленности требуется 0,20 долл, затрат угольной отрасли и 0,40 долл, затрат металлургической отрасли. Вычислите объемы производства в каждой из отраслей, которые необходимы для удовлетворения внешнего спроса 20 млрд. долл, для угольной промышленности и 10 млрд. долл, для металлургической промышленности.
Глава 4. Системы линейных уравнений и матрицы 389 24. Экономика состоит из двух частей: транспортной и промышленной. Для производства услуг, стоимостью один доллар в транспортной отрасли требуется по 0,10 долл, затрат в каждой отрасли. Для производства продукции стоимостью один доллар В промышленности требуется по 0,40 долл, затрат в каждой отрасли. Вычислите объемы производства в каждой из отраслей, которые необходимы для удовлетворения внешнего спроса, равного 5 млрд. долл, для транспортной отрасли и 20 млрд, долл, для промышленности. 25. Экономика маленького островного государства состоит из двух секторов: сельского хозяйства и туризма, Для производства сельхозпродукции на 1 долл, требуется 0,20 долл, затрат сельскохозяйственной отрасли и 0,15 долл, затрат туристической отрасли. Для предоставления туристических услуг на 1 долл, требуется 0,40 долл, затрат сельскохозяйственной отрасли и 0,30 долл, затрат туристической отрасли. Вычислите объемы производства в каждой из отраслей, которые необходимы Для удовлетворения внешнего спроса 60 млн. долл, для сельскохозяйственной Отрасли и 80 млн. долл, для туристической отрасли. 26. Экономика страны состоит из двух отраслей: сельского хозяйства и нефтедобычи. Для производства сельхозпродукции на 1 долл, требуется 0,40 долл, затрат сельскохозяйственной отрасли и 0,35 долл, затрат нефтедобывающей отрасли. Для производства продукции на 1 долл, в нефтедобывающей отрасли требуется 0,20 долл, затрат сельскохозяйственной отрасли и 0,05 долл, затрат нефтедобывающей отрасли. Вычислите объемы производства в каждой из отраслей, которые необходимы Для удовлетворения внешнего спроса 40 млн. долл, долларов для сельскохозяйственной отрасли и 250 млн. долл, для нефтедобывающей отрасли. 27. Экономика состоит из трех отраслей: сельского хозяйства, промышленности и энергетики. Для производства сельхозпродукции на 1 долл, в сельском хозяйстве требуется 0,20 долл, затрат в сельском хозяйстве, 0,20 долл, затрат в промышленности и 0,20 долл, затрат в энергетике. Для производства промышленной продукции на 1 долл, требуется 0,40 долл, затрат в сельском хозяйстве, 0,10 долл, затрат в промышленности и 0,10 долл, затрат в энергетике. Для производства энергии на 1 долл, требуется 0,30 долл, затрат в сельском хозяйстве, 0,10 долл, затрат в промышленности и 0,10 долл, затрат в энергетике. Вычислите объемы производства в каждой из отраслей, которые необходимы для удовлетворения внешнего спроса 10 млрд. долл, для сельского хозяйства, 15 млрд. долл, для промышленности И 20 млрд. долл, для энергетики. 28. Большая энергетическая компания поставляет электричество, природный газ и нефть. Для производства электричества стоимостью один доллар требуется 0,30 долл, затрат на электричество, 0,10 долл, затрат на природный газ и 0,20 долл. Затрат на нефть. Для производства природного газа стоимостью один доллар требуется 0,30 долл, затрат на электричество, 0,10 долл, затрат на природный газ и 0,20 долл, затрат на нефть. Для производства одного доллара нефти требуется по 0,10 долл, затрат в каждом секторе. Вычислите объемы производства в каждом из секторов, которые необходимы для удовлетворения внешнего спроса, равного 25 млрд. долл, для электричества, 15 млрд. долл, для природного газа и 20 млрд. долл, для нефти.
390 Часть II. Конечная математика jci. В задачах 29 и 30 следует использовать графическую утилиту. 29. Экономика состоит из четырех отраслей: сельского хозяйства Л, энергетики Е, сферы услуг L и промышленности М. В таблице приведены стоимость производства продукции и предоставления услуг на 1 долл, в каждой из отраслей, а также внешний спрос (млрд, долл.) за трехлетний период. Вычислите объемы производства в каждом из секторов, которые необходимы для удовлетворения внешнего спроса. Ответ округлите до миллиарда. Производство Внешний спрос A E L M 1 2 3 A 0,05 0,17 0,23 0,09 23 32 55 __ E 0,07 0,12 0,15 0,19 41 48 62 Потребление T L 0,25 0,08 0,03 0,32 18 21 25 M 0,11 0,19 0,28 0,16 31 33 35 30. Повторите решение задачи 29 для следующей таблицы. Производство Внешний спрос A E L M 1 2 3 A 0,07 0,09 0,27 0,21 18 22 37 E 0,14 0,07 0,21 0,24 26 31 42 Потребление r Li 0,17 0,06 0,02 0,21 12 19 28 M 0,15 0,13 0,31 0,19 41 45 49 Основные термины и обозначения 4.1. Обзор: системы линейных уравнений с двумя переменными. Линейная система; решение системы; множество решений; графический метод; совместная, несовместная, независимая система; единственное решение; зависимая система; метод подстановки; метод исключения с помощью сложения; эквивалентные системы; параметр; частное решение; равновесная цена; точка равновесия. 4.2. Системы линейных уравнений и расширенные матрицы. Матрица; элемент матрицы; размер; матрица т х п; размерность; матрица-столбец; матрица-строка; положение элемента в матрице; обозначение с помощью двойного индекса аг;; главная диагональ; матрица коэффициентов; матрица постоянных членов; расширенная матрица; эквивалентные системы; построчно-эквивалентные матрицы; операции над строками; метод исключения Гаусса-Жордана Ri <-> Rj', kRi R^ kRj + Ri —> Ri.
Глава 4. Системы линейных уравнений и матрицы 391 4.3. Метод исключения Гаусса-Жордана. Приведенная форма; приведенная система; метод исключения Гаусса-Жордана; подматрица. 4.4. Матрицы: основные операции. Равные матрицы; сумма матриц; нулевая матрица; отрицательная матрица; разность матриц; произведение числа и матрицы; произведение строки и столбца; произведение двух матриц; диагональная матрица; верхняя треугольная матрица; матрица инцидентности. 4.5. Нахождение обратной квадратной матрицы. Единичный элемент для операции умножения; Обратная матрица для операции умножения; сингулярная матрица; матрица шифрования; матрица расшифровки. 4.6. Матричные уравнения и системы линейных уравнений. Матричное уравнение; основные свойства матриц. 4.7. Анализ затрат и результатов Леонтьева. Анализ затрат и результатов; технологическая матрица; матрица объема производства; матрица внешнего спроса. Упражнения для повторения Выполните все упражнения этого обзорного раздела и сравните результаты с отве- томи, помещенными в конце книги. Ответы ко многим упражнениям на повторение приводятся вместе с номером соответствующего раздела (курсивом). Если у вас возникают затруднения при решении какой-либо задачи, повторите материал соответствующего раздела. А 1. Решите систему уравнений графическим методом: 2х — у = 4, х — 2у = —4. 2. Решите систему, указанную в задаче 1, методом подстановки. * 3. Определите, представлены ли перечисленные ниже матрицы в приведенной фор¬ ме. Если нет, укажите последовательность операций, необходимых для преобра¬ зования матриц в приведенную форму. 4. Заданы матрицы А и В. . _ Г 5 3-102 —4 8 13 0 2 4 7 В = а) Какие размеры у матриц Аи В?
392 Часть II. Конечная математика б) Вычислите 6124, ^15, Ьз1 И &22- в) Определены ли произведения АВ или ВА1 5. Вычислите величины х\ и Х2 из следующего соотношения. 1-2 xi _ 4 1 -3 J I. х2 ] “ [ 2 В задачах 6-14 требуется выполнить указанные операции с перечисленными матрицами, если эти операции определены. С= [ 2 3 ], D= • б. А + В. 7. В + D. 8. А - 2В. 9. АВ. 10. АС. 11. AD. 12. DC. 13. CD. 14. С 4- D. 15. Вычислите матрицу, обратную к приведенной ниже матрице А, с помощью подходящего преобразования строк матрицы [А | /]. Покажите, что А-1 А = I. 4 3 2 16. Решите систему уравнений методом исключения. 4xi 4“ 3x2 = 3, 3xi + 2x2 — 3. 17. Решите систему уравнений задачи 16 с помощью преобразования строк расширенной матрицы, соответствующей системе. 18. Решите систему уравнений задачи 16, представляя ее в виде матричного уравнения и используя обратную матрицу коэффициентов (см. задачу 15). Найдите решение системы, если константы 3 и 5 в правой части заменить на константы 7 и 10 соответственно или константы 2 и 4 соответственно. Б В задачах 19-24 требуется выполнить указанные операции с перечисленными матрицами, если эти операции определены. 19. А + D. 22. ВС. 20. Е + DA. 23. СВ. 21. DA - ЗЕ. 24. AD - ВС.
Глава 4. Системы Линейных уравнений и матрицы 393 25. Вычислите матрицу, обратную к приведенной ниже матрице А с помощью подходящего преобразования строк матрицы [А | I]. Покажите, что А-1 А = I. 1 2 1 2 3 3 4 2 1 А = 26. Решите системы уравнений методом исключения Гаусса-Жордана. a) xi 4- 2х2 + 3#з = 1 б) 4- 2x2 — £з = 2 2x1 + 3#а + 4хд = 3 2xi + 3x2 + хз = —3 xi + 2ха + Хз = 3 3xi + 5x2 = —1 27. Решите систему уравнений задачи 26, п. а, представляя ее в виде матричного уравнения и используя обратную матрицу коэффициентов (см. задачу 25). Найдите решение системы, если константы 1, 3 и 3 в правой части заменить на О, О и —2 или —3, —4 и 1 соответственно. * 28. Определите отношение между количеством решений приведенной ниже системы и значениями константы к. 2xi — 6x2 = 4, —xi + кх2 = —2. 29. Вычислите (/ — М)"1 и матрицу объема производства X по заданным технологической матрице М и матрице внешнего спроса D (в млрд. долл.). Г 0,2 0,15 1 о Г 30 1 "= ОД 0,3 • °= 20 ■ 30. Используя изменения масштаба изображения и другие приближенные методы графических утилит, вычислите решение приведенной ниже системы с точностью до двух десятичных знаков. х — 5у = —5, 2х + Зу = 12. В 31. Вычислите матрицу, обратную к приведенной ниже матрице А. Убедитесь в том, что А 1А — 4 5 6 4 5-4 1 1 1 32. Вычислите решение системы уравнений 0,04xi + 0,05x2 + 0,06хз = 360, 0,04xi + 0,05x2 — 0,04хз = 120, xi + х2 + хз = 7000, представляя ее в виде матричного уравнения и используя обратную матрицу коэффициентов. (Прежде чем начинать вычисления, умножьте первые два уравнения на 100, чтобы устранить десятичные знаки. См. также задачу 31.)
394 Часть II. Конечная математика 33. Решите задачу 32 методом исключения Гаусса-Жордана. 34. Задана технологическая матрица М и матрица внешнего спроса D (в млрд. долл.). Вычислите (/ — М)-1 и матрицу объема производства X. ■ 0,2 0 0,4 ' ' 40 ‘ м = о,1 0,3 0,1 , D = 20 0 0,4 0,2 30 * 35. Определите, сколько может быть решений у системы п уравнений с п неизвест¬ ными, если для матрицы коэффициентов выполняются следующие условия. а) Существует обратная матрица. б) Не существует обратной матрицы. * 36. Определите, сколько может быть решений у системы уравнений, расширенная матрица которой в приведенной форме указана ниже, для следующих случаев, а) т 0. б) т = 0 и п ф 0. в) т = 0 и п = 0. 10-25 0 1 3 3 0 0 т п * 37. Решение уравнения анализа затрат и результатов X = MX + D равно X = (7 — — M)~1D. Ниже приведены два варианта решения этого уравнения. Обсудите законность каждого шага в этих решениях. (Считайте, что все необходимые обратные матрицы существуют.) Являются ли оба решения правильными? a) X = MX + D, б) X = MX + D, X - MX = D, X(I — M) = D, X = D(I - М)-1. -D = MX - X, —D = (M — I)X, X = (M - Применение математики Экономика и бизнес 38. Анализ безубыточности. Производитель кухонного оборудования готовит к реа- лизации новую машину для производства макаронных изделий. Постоянные затраты разработку, конструирование и тому подобное составляют 243 000 долл., а переменные издержки — 22,45 долл, за машину. Цена машины составляет 59,95 долл. а) Вычислите уравнения функций расходов и прибыли. б) Найдите точку безубыточности. в) Постройте графики обеих функций в одной системе координат и укажите на них точку безубыточности. Определите с помощью графика условия, при которых производство будет прибыльным или убыточным.
Глава 4. Системы линейных уравнений и матрицы 395 39. WWW Распределение ресурсов. Транснациональной горнодобывающей компании принадлежат две шахты, в Войс-Бэй (Voice Вау) и Хоук-Ридж (Hawk Ridge), Канада. Состав рудной породы для каждой шахты представлен в таблице. Сколько тонн руды нужно добыть на каждой шахте, чтобы получить шесть тонн никеля и восемь тонн меди? Шахта Никель (%) Медь (%) Войс-Бэй 2 4 Хоук-Ридж 3 2 40. www Распределение ресурсов. а) Решите задачу 39 с помощью матричного уравнения, используя метод обратной матрицы. б) Решите задачу 39 способом» указанным в п. а, если требуется добыть 7,5 тонн никеля и 7 тонн меди. * 41. WWW Аренда техники. Компания, продающая зерно, намерена арендовать 20 кры¬ тых вагонов общей вместимостью 108 000 куб. футов. Существует три типа вагонов разных размеров: 3000 куб. футов, 4500 куб. футов и 6000 куб. футов. а) Сколько вагонов каждого типа можно арендовать? б) Пусть стоимость месячной аренды вагонов составляет: 180 долл, для 3000- футового Вагона, 225 долл, для 4500-футового и 325 долл, для 6000-футового. Какое решение п. а соответствует минимальной стоимости аренды? * 42. Стоимость сырья. Компании для производственных целей нужно получить два типа бронзовых сплава. Требующиеся для этого количества меди, олова и цинка приведены в матрице М, а стоимость этих материалов (долл, за фунт) от двух поставщиков — в матрице N. Компании нужно выбрать одно из предложений. Медь, Олово, Цинк, фунты фунты фунты м = ‘ 4800 600 300 ' Сплав 1 . 6000 1400 700 Сплав 2 Поставщик А Поставщик Б 0,75 0,70 Медь N = 6,50 6,70 Олово 0,40 0,50 Цинк а) Обсудите возможные интерпретации матричных элементов произведений МN и NM. б) Если какое-либо из произведений MN и NM имеет смысл, вычислите его и определите смысл строк и столбцов. в) Придумайте метод, с помощью которого можно определить более выгодное предложение. 43. WWW Оплата труда. Компания, производящая бюджетные калькуляторы, владеет двумя заводами в Калифорнии (СА) и Техасе (ТХ). В матрицах М и N
396 Часть II. Конечная математика приведены данные о времени выполнения работ и оплате труда в разных производственных отделах и на разных заводах. Отдел обработки,ч Отдел сборки,ч Отдел упаковки,ч „ Г м = 0,10 0,05 Модель А 0,25 0,20 0,05 Модель Б СА, тх, долл. долл. 12 10 ‘ Отдел обработки N = 15 12 Отдел сборки 7 6 Отдел упаковки а) Вычислите стоимость производства модели Б на заводе в Калифорнии. *б) Обсудите возможные интерпретации матричных элементов произведений MN и NM. *в) Если какое-либо из произведений MN и NM имеет смысл, найдите его и определите смысл строк и столбцов. 44. Анализ капиталовложений. Имеется сумма инвестиций 5000 долл., одна часть вкладывается под 5% годовых, а другая — под 10%. Сколько денег нужно вложить под каждую процентную ставку, чтобы получить за год 400 долл.? Решите задачу с помощью расширенной матрицы. 45. Анализ капиталовложений. Решите задачу 44 с помощью матричного уравнения и обратной матрицы коэффициентов. * 46. Анализ капиталовложений. Можно ли при условиях задачи 44 получить за год 200 или 600 долл.? Опишите все возможные варианты. 47. Организация концерта. В открытом амфитеатре имеется 25 000 мест. Билеты продаются по ценам 8, 12 и 20 долл., причем количество проданных 8-долларо- вых билетов должно равняться числу 20-долларовых. Сколько билетов каждого типа может быть продано (считая, что все места заняты) согласно информации о выручке, приведенной в таблице? Решите задачу с помощью обратной матрицы коэффициентов. Концерт 1 2 3 Проданных билетов, шт 25000 25000 25000 Сумма выручки, долл 320000 330000 340000 * 48. Организация концертов. Насколько изменится решение задачи 47, если не требовать равенства количеств проданных 8- и 20-долларовых билетов? 49. Анализ межотраслевых связей. Экономика состоит из двух отраслей: сельского хозяйства и промышленности. Для производства сельхозпродукции на 1 долл, требуется 0,30 долл, в сельскохозяйственной отрасли и 0,20 долл, в промыш-
Глава 4. Системы линейных уравнений и матрицы 397 ленности. Для производства промышленной продукции на 1 долл, требуется 0,10 долл, в сельскохозяйственной отрасли и 0,40 долл, в промышленности. а) Вычислите объемы производства в каждой из отраслей, которые необходимы для удовлетворения внешнего спроса 50 млрд. долл, для сельскохозяйственной отрасли и 20 млрд. долл, для промышленной. б) Вычислите объемы производства в каждой из отраслей, которые необходимы для удовлетворения внешнего спроса 80 млрд. долл, для сельскохозяйственной отрасли и 60 млрд. долл, для промышленной. Социальные науки 50. Криптография. Следующая фраза закодирована с помощью приведенной ниже матрицы В. Расшифруйте ее. 25 8 26 24 25 33 21 14 21 41 30 50 21 32 41 25 25 25 1 1 1 1 0 0 1 1 1 51. Транспортный поток. На рисунке приведена схема транспортного потока в час пик на перекрестке четырех дорог с односторонним движением (автомобилей в час). 1-я 2-я улица улица 700 200 600 Х4 *1 600 Х2 300 500 *3 300 400 Эльм-стрит Оак-стрит Рис. 4.24. Иллюстрация к задаче 51 а) Запишите систему уравнений, которая описывает транспортный поток через все четыре перекрестка. б) Найдите решение системы. в) Какое максимальное количество автомобилей может проехать с Оак-стрит (Oak Street) на Эльм-стрит (Elm Street) через 1-ю улицу? г) Пусть транспортный поток с Оак-стрит на Эльм-стрит через 1-ю улицу ограничен 500 автомобилями в час. Определите транспортные потоки на других частях магистрали.
398 Часть II. Конечная математика 52. Уровень квалификации. В турнире принимают участие четыре футбольные команды. Все команды провели по одной игре друг с другом с такими результатами: команда А проиграла команде Б, команда Б проиграла командам В и Г, команда В проиграла командам А и Г, команда Г проиграла команде А. а) Запишите результаты турнира в виде матрицы инцидентности. б) Вычислите матрицу М + М2 и используйте эту матрицу для расположения команд по рангу в турнирной таблице. Объясните, по какому принципу определяется ранг команды. Домашнее задание 4.1. Применение матриц для вычисления расходов, доходов и прибыли Производитель игрушек закупает комплектующие у разных поставщиков и собирает из них три готовые модели железной дороги: “Limited”, “Empire” и “Comet”. Компоненты каждой модели, а также количество минут, необходимое для ее производства, перечислены в табл. 4.1. Таблица 4.1. Детали детской железной дороги Деталь Модель “Limited” “Empire” “Comet” Локомотив, шт 1 1 2 Вагон, шт 5 6 8 Рельсовый прогон, шт 20 24 32 Переключатель стрелки, шт 1 2 4 Модуль электропитания, шт 1 1 1 Работа, мин. 15 18 24 Стоимость деталей приведена в табл. 4.2, а цены на модели железной дороги — в табл. 4.3. Продавец получил заказ из розничного магазина игрушек, приведенный в табл. 4.4. Информацию, которая содержится в каждой из таблиц, нужно представить в виде матрицы и затем, выполняя матричные операции, определить следующие величины: а) Количество деталей и время, необходимое для выполнения заказа. б) Стоимость выполнения заказа. Таблица 4.2. Стоимость деталей Деталь Цена, долл. Локомотив 12,52 Вагон 1,43 Рельсовый прогон 0,25 Переключатель стрелки 2,29 Модуль электропитания 12,54 Работа (за минуту) 0,15
Глава 4. Системы линейных уравнений и матрицы 399 Таблица 4.3« Цены на модели Модель Цена, долл. “Limited” 54,60 “Empire” 62,28 “Comet” 81,15 Таблица 4.4. Объем заказов Модель Количество, шт “Limited” 48 “Empire” 24 “Comet” 12 в) Доход, получаемый при выполнении заказа. г) Прибыль с заказа. 1. Используйте по одной букве для обозначения матриц, соответствующих таблицам. В этих обозначениях запишите матричные выражения, с помощью которых можно получить требуемую информацию. 2. Выполните вычисления, указанные в домашнем задании 4.1, а. Вскоре после выполнения заказа, приведенного в табл. 4.4, поставщик сообщает производителю, что вагоны и локомотивы используемого в моделях типа отсутствуют. У производителя на складе есть всего 30 локомотивов и 134 вагона. 3. Сколько игрушек каждого типа может собрать производитель из имеющихся в его распоряжении локомотивов и вагонов? Считается, что количество всех остальных компонентов не ограничено. 4. Какой доход получит производитель, если продаст эти игрушки? Если есть несколько вариантов комплектации, то какая из них наиболее выгодна? Домашнее задание 4.2. Переменные и косвенные производственные издержки Компания по производству офисного оборудования имеет три производственных отдела для выпуска трех различных видов товара: копировальных аппаратов, принтеров и факсов. Существуют также четыре сервисных отдела: бухгалтерии, рекламный, исследовательский и обслуживания, которые выполняют определенные задачи для производственных отделов. Кроме того непроизводственные отделы также выполняют задачи для других сервисных отделов. Постоянные ежемесячные затраты на содержание отделов а также процент времени, затрачиваемого на выполнение задач других непроизводственных отделов (включая собственные задачи), приведены в табл. 4.5. Пусть xi, ^3 и #4 обозначают ежемесячные затраты отделов бухгалтерии, рекламы, исследований и обслуживания, a yi, и уз - отделов производства копировальных аппаратов, принтеров И факсов соответственно. 1. Общие затраты xi отдела бухгалтерии описываются уравнением хг = 100000 + 0,15x1 4- 0,1x4. Напишите аналогичные уравнения для других сервисных отделов. 2. Представьте систему уравнений, указанную в п. 1, в виде матричного уравнения X = D + АХ.
400 Часть II. Конечная математика Таблица 4.5. Ежемесячные производственные затраты Отдел Общие затраты, долл. Время выполнения задач непроизводственных отделов, % Бухгалтерия Рекламный отдел Исследовательский отдел Отдел обслуживания Бухгалтерия 100000 15 0 0 10 Рекламный отдел Исследовательский 150000 15 0 0 10 отдел Отдел обслужива¬ 120000 15 0 25 20 ния Отдел производства копироваль¬ 60000 10 0 0 0 ных аппаратов Отдел производ¬ 270000 15 20 40 20 ства принтеров Отдель производ¬ 140 000 15 20 20 20 ства факсов 190 000 15 60 15 20 3. Общие расходы у\ отдела производства копировальных аппаратов описываются таким уравнением. т/i = 270000 -I- 0,15^1 + 0,2x2 + 0,4хз -I- 0,2x4. Напишите аналогичные уравнения для других отделов производства. 4. Представьте систему уравнений, указанную в п. 3, в виде матричного уравнения Y = С + ВХ. 5. С помощью матричных операций найдите решение системы уравнений, указанной в п. 2, и используйте его для решения системы, приведенной в п. 4. 6. Сравните общие затраты всех семи отделов, перечисленных в табл. 4.5, с общими затратами трех производственных отделов, вычисленных в п. 5. Интерпретируйте полученный результат.
5 Линейные неравенства и линейное программирование ■ 5.1. Системы линейных неравенств с двумя переменными ■ 5.2. Линейное Программирование в двух измерениях: геометрический подход ■ 5.3. Геометрическое введение в симплекс-метод ■ 5.4. Симплекс-Метод: максимизация с ограничениями вида “меньше или равно” ■ 5.5. Двойственная задана: минимизация с ограничениями вида “больше или равно” ■ 5.6. Задачи максимизации и минимизации со смешанными ограничениями ■ Ключевые слова, основные обозначения и термины ■ Упражнения для повторения ■ Домашнее задание 5.1. Двухэтапный метод: альтернатива М-методу ■ Домашнее задание 5.2. Планирование производства Введение В этой главе рассматриваются линейные неравенства с двумя и более переменными. В дополнение к этому вводится новый и мощный математический аппарат, называемый линейным программированием^ который используется для решения различных практических задач. Операции Над строками матриц, введенные в главе 4, будут особенно полезны в разделах 5.4-5.6.
402 Часть II. Конечная математика 5.1. Системы линейных неравенств с двумя переменными ■ Графическое изображение линейных неравенств с двумя переменными ■ Графическое решение систем линейных неравенств ■ Решение практических задач Многие приложения математики используют системы неравенств, а не уравнений. Наиболее удобным способом представления решения системы линейных неравенств с двумя переменными является график. В этом разделе описаны методы графического изображения как одного линейного неравенства, так и системы линейных неравенств с двумя переменными. Графическое изображение линейных неравенств с двумя переменными Известно, как изобразить графически уравнения первой степени, например, у = 2х — 3 и 2х — Зу = 5, но как изобразить графически неравенство первой степени, скажем, у < 2х — 3 и 2х — Зу > 5? Оказывается, графическое изображение этих неравенств почти так же просто, как и графическое изображение уравнений, но сначала необходимо рассмотреть некоторые важные подмножества плоскости в прямоугольной системе координат. Прямая делит плоскость на две половины, называемые полуплоскостями. Вертикальная линия делит ее на левую и правую полуплоскости, а любая невертикальная линия — на верхнюю и нижнюю полуплоскости (рис. 5.1). Задание 5.1. Рассмотрим следующее линейное уравнение и связанные с ним линейные неравенства: Зх — 41/ = 24, (5.1) Зх — 41/ < 24, (5.2) Зх - 41/ > 24. (5.3) 1. Изобразите прямую, задаваемую уравнением (5.1). 2. Найдите на этой прямой точку, имеющую координату х, равную 4, и проведите вертикальную прямую через эту точку. Рассмотрите взаимосвязь между координатами у точек на прямой и соотношениями (5.1), (5.2) и (5.3). 3. Повторите решение задачи 2 для х = — 4 и х = 12. 4. Основываясь на наблюдениях, сделанных при решении задач 2 и 3, опишите все точки плоскости, удовлетворяющие уравнению (5.1), неравенству (5.2) и неравенству (5.3) соответственно. I
403 Глава 5. Линейные неравенства и линейное программирование Рис. 5.1. Полуплоскости Для исследования полуплоскостей, определенных линейным уравнением вида у — — х = —2, перепишем уравнение в виде у — х — 2. Для любого заданного значения х существует единственное значение у такое, что (ж, у) лежит на прямой. Например, для х = 4 существует у = 4—2 — 2. Для того же значения х и меньших значений у точка (гг, у) будет лежать ниже прямой, Поскольку у < х — 2. Таким образом, нижняя полуплоскость соответствует неравенству у > х — 2, как изображено на рис. 5.2. у * (4, У); У > 4 — 2: точка на верхней полуплоскости (4, у); у = 4 — 2: точка на границе i у); У < 4 — 2: точка на нижней полуплоскости Рис. 5.2. Полуплоскости и неравенства Четыре неравенства, образованные из равенства у = х — 2 заменой знака = на знаки > , ^ , < и соответственно имеют следующий вид. у > х — 2^ у х -2, у < х — 2, у х — 2.
404 Часть II. Конечная математика Графиком каждого из них является полуплоскость, не включающая границу для неравенств < и > , и включающая границу для неравенств < и . На рис. 5.3 полуплоскости определены маленькими стрелками на графике у = х — 2 и затем изображены как залитые серым цветом области. Исключенная граничная прямая изображена пунктирной линией, а включенная граничная прямая — непрерывной линией. Рис. 5.3. Полуплоскости, соответствующие разным неравенствам В результате предшествующего обсуждения вырисовывается следующая теорема. Теорема 5.1 (Графики линейных неравенств). Графическим образом линейного неравенства Ах 4- By < С или Ах -F By > С, где В 0, является верхняя полуплоскость либо нижняя полуплоскость (но не обе), определенные прямой линией Ах -I- By = С. Если В = 0 и А 0, то графическим образом линейного неравенства Ах < С или Ах > С есть либо левая полуплоскость, либо правая полуплоскость (но не обе), определенные прямой Ах = С. ■ В качестве вывода из этой теоремы обозначим простую процедуру быстрого графического изображения линейных неравенств. Процедура графического изображения линейных неравенств Этап 1. Сначала изобразим график Ах 4- By = С в виде пунктирной прямой линии, если равенство не включается в исходное соотношение, или в виде сплошной прямой, если равенство включается. Этап 2. Выберем контрольную точку где-либо на плоскости, но не на прямой (начало координат требует обычно наименьшего объема вычислений), и подставим ее координаты в неравенство. Этап 3. Графический образ исходного неравенства представляет собой полуплоскость, содержащую контрольную точку, если координаты данной точки удовлетворяют неравенству, или полуплоскость, не содержащую контрольную точку, если координаты ее не удовлетворяют неравенству. Пример 5.1 (Графическое изображение линейного неравенства). Изобразите неравенство 2х — Зу 6. Проверьте решение с помощью графической утилиты.
Глава 5. Линейные неравенства и линейное программирование 405 Решение. Этап 1. Изобразим график 2х — Зу = 6 в виде сплошной прямой линии, поскольку равенство включается а исходное соотношение. Этап 2. Выберем удобную контрольную точку выше или ниже прямой линии. Начало координат (0, 0) требует наименьшего объема вычислений. Следовательно, подставляя точку (0, 0) в Неравенство, получим 2х - Зу 6, 2-0 — 30 = 0^6. Это утверждение верно, поэтому точка (0, 0) принадлежит множеству, являющемуся решением. Этап 3. Прямая линия 2х — Зу = 6 И полуплоскость, содержащая начало координат, образуют графический образ неравенства 2х — Зу 6, что изображено на рис. 5.4. Рйс. 5.4. Полуплоскости, соответствующие разным неравенствам
406 Часть II. Конечная математика Проверка. WWW Проверка решения с помощью графической утилиты изображена на рис. 5.5. Маленький треугольник слева от Yi на рис. 5.5, а показывает, что заштриховывается верхняя часть графика. Процедура штриховки полуплоскостей описана в справочном руководстве графической утилиты. ■ PUtl Hot* ^YiB(2/3)X-2 \Yz = \Y? = х?Ч = \Ys = \Vfi = \Y? = 10 10 6) Рис. 5.5. Проверка неравенства с помощью графической утилиты Упражнение 5.1. Изобразите неравенство бх — Зу > 18. Проверьте решение с помощью графической утилиты. Пример 5.2 (Графическое изображение неравенств). Изобразите следующие неравенства. 1. у > —3. 2. 2х < 5. 3. х С Зу. Решение. 1- У 5- 1 1 Ч.1 1 1 1 1 -5--
Глава 5. Линейные неравенства и линейное программирование 407 Упражнение 5.2. Изобразите неравенства 1. у < 4. 2. 4х —9. 3. Зх 2у. Графическое решение систем линейных неравенств Теперь рассмотрим системы линейных неравенств. я + У 6, и 2х — у 0 2х + у 22, х + у 13, 2х + Зу 50, х 0, 2/ > 0. Требуется решить эти неравенства графически, т.е. найти множество всех упорядоченных пар действительных чисел (я, у), которые подходят одновременно для всех неравенств системы. Это Множество называется областью решений системы. (Во многих приложениях область решения называют также областью допустимых решений). Для того чтобы найти область решений, изобразим каждое неравенство в системе и затем найдем пересечение всех полуплоскостей. Для упрощения последующих рассуждений рассматриваются системы линейных неравенств, у которых включается знак равенства. Пример 5.3 (Графическое решение систем линейных неравенств). Решите следующую систему линейных неравенств графически. х + у б, 2х — у 0. Решение. Изобразим Прямую х + у = 6 и заштрихуем область, удовлетворяющую линейному неравенству х + у 6. Эта область заштрихована темно-серыми линиями на рис. 5.6, а. Затем нарисуем прямую 2х — у = 0 и заштрихуем область, удовлетворяющую неравенству 2х ~ у 0. Данная область заштрихована светло-серыми линиями на рис. 5.6, а. Область решений системы неравенств представляет собой пересечение этих двух областей. Эта область заштрихована на рис. 5.6, а более густо, а на рис. 5.6, б изображена в виде единственной заштрихованной области решения. Координаты любой точки в заштрихованной области рис. 5.6, б задают решение неравенств. Например, точки
408 Часть II. Конечная математика (2; 4), (6; 3) и (7,43; 8,56) представляют собой три из бесконечного множества решений, что может быть легко проверено. Точка пересечения (2; 4) является решением обоих уравнений х + у = 6и2х — у = 0. Решение может быть получено с помощью любого метода, рассмотренного в главе 4. I х + у = 6 2х —у = 0 Рис. 5.6. Решение системы неравенств х + у = 6 2х-у = 0 Упражнение 5.3. Решите графически следующую систему линейных неравенств: Зх + у < 21, х — 2у < 0. Задание 5.2. Вернемся к примеру 5.3. Изобразите все границы и заштрихуйте область, содержащую точки, которые не удовлетворяют всем неравенствам. Иначе говоря, заштрихуйте область плоскости, соответствующую неравенству х 4- у < 6, а затем заштрихуйте область плоскости, соответствующую неравенству 2х — у < 0. Какая часть плоскости останется незаштрихованной? Сравните этот метод с методом, используемым в решении к приме- РУ 5.3. а kx WWW Метод решения неравенств, исследованный в задании 5.2, хорошо работает в сочетании с графической утилитой, позволяющей заштриховать верхнюю и нижнюю части графика. В примере 5.3 незаштрихованная область на рис. 5.7, б соответствует области решения на рис. 5.6, б. Точки пересечения прямых, которые образуют границу области решения, будут играть основную роль в решении задач линейного программирования, которые рассматриваются в следующем разделе.
Глава 5. Линейные неравенства и линейное программирование 409 Мф11 МфСЭ kVi|6-X ^Yal2X чУи* \YSte \7йк а) Рис. 5.7. Решение системы неравенств с помощью графической утилиты Угловая точка Угловой точкой (comer point) области решения называется точка в области решения, которая является пересечением двух граничных прямых. Например, точка (2; 4) является единственной угловой точкой области решения в примере 5.3 (рис. 5.6). Пример 5.4 (Графическое решение системы линейных неравенств). Решите следующую систему линейных неравенств графически и найдите угловые точки. 2х + у 22, х + у 13, 2х + 5у < 50, х 0, 2/^0. Решение. Неравенства х 0 и у Э 0 свидетельствуют о том, что область решения лежит в первом квадранте.1 Таким образом, можно ограничить внимание частью плоскости. Сначала изобразим прямые. 2х + у з= 22, х + у = 13, 2х + 5у « 50. Найдем точки пересечения всех прямых с осями координат, а затем проведем прямые через эти точки Затем, выбирая начало координат (0; 0) в качестве контрольной точки, увидим, что графики каждого из первых трех неравенств системы состоят из соответствующей прямой и полуплоскости, лежащей ниже ее. Это обозначено маленькими стрелочками на рис. 5.8. Таким образом, область решения системы состоит из точек первого квадранта, которые лежат ниже одновременно всех трех прямых (см. заштрихованную область на рис. 5.8). Неравенства х 0 И у 0 часто встречаются в приложениях, включающих системы неравенств, поскольку переменные х и у часто представляют величины, которые не могут быть отрицательными (количество единиц продукции, количество рабочих часов и т.д.).
410 Часть II. Конечная математика Рис. 5.8. Графическое решение системы линейных неравенств Угловые точки (0; 0), (0; 10) и (11; 0) могут быть определены по графику. Две другие угловые точки определены следующим образом. Решив систему Решив систему 2х 4- 5у = 50, 2х + у = 22, х + у = 13, х + у = 13, получим точку (5; 8). получим точку (9; 4). Отметим, что прямые 2х + 5у = 50 и 2х 4- у = 22 также пересекаются, но точка пересечения не является частью области решения, и, следовательно, не является угловой точкой. в Упражнение 5.4. Решите следующую систему линейных неравенств графически и найдите угловые точки. 5х -F у 20, х 4- у 12, я 4- Зт/ > 18, х 0, 2/^0. Если сравнить области решений примеров 5.3 и 5.4, то можно увидеть, что между ними существует значительная разница. Вокруг области решения в примере 5.4 можно описать окружность. Однако невозможно заключить все точки области решения примера 5.3 в какой-то круг, независимо от того, насколько большим его нарисовать. Эти рассуждения приводят к следующему определению.
Глава 5. Линейные неравенства и линейное программирование 411 Ограниченные и неограниченные области решений Область решения системы линейных неравенств ограничена, если ее можно заключить внутрь круга. Если она не может быть заключена внутрь круга, она не ограничена. Таким образом, область решения в примере 5.4 ограничена, а область решения в примере 5.3 не ограничена. Это определение будет играть важную роль в следующем разделе. Решение практическим задач ■W Пример 5.5 (Медицина). Пациенту в больнице необходимо получать ежедневно по ШйЗ крайней мере 84 единицы лекарства Л и 120 единиц лекарства Б (предположим, что передозировка каждого лекарства безопасна для организма). Каждый грамм вещества М содержит десять единиц лекарства А и восемь единиц лекарства Б, а каждый грамм вещества Н содержит две единицы лекарства А и четыре единицы лекарства Б. Сколько граммов веществ М и Н можно смешать, чтобы получить минимальную требуемую дневную дозу лекарств? Решение. Чтобы ответить на поставленный в примере вопрос, необходимо ввести следующие переменные. х — количество использованных граммов вещества М, у — количество использованных граммов вещества Н. Соберем информацию по задаче в таблице, в столбцах которой расположены переменные. Количество вещества в грамме Вещество А/, ед. Вещество Я, ед. Минимальная суточная потребность, ед. Лекарство Л 10 2 84 Лекарство Б 8 4 120 Поскольку в одном грамме вещества М содержится 10 единиц лекарства Л, а в дневную дозу лекарства входит х граммов вещества А/, в дневной дозе содержится 10х единиц лекарства Л. Используй аналогичные рассуждения, можно рассмотреть остальные компоненты дневной дозы лекарств. 10х — количество единиц лекарства А в х граммах вещества М, 2у — количество единиц лекарства Л в у граммах вещества Н, 8х — количество единиц лекарства Б в х граммах вещества М, ку — количество единиц лекарства Б в у граммах вещества Н. /Число единицХ лекарства Л в х граммах у вещества М / /Число единицХ лекарства Б в х граммах у вещества М / /Число единицХ лекарства Л в у граммах у вещества Я / /Число единицХ лекарства Б в у граммах у вещества Я ) 84, 120, Число используемых граммов вещества М Число используемых граммов вещества Н 0, 0.
412 Часть II. Конечная математика Превращая эти словесные утверждения в математические утверждения с использованием переменных хиу, которые введены выше, получим следующую модель, состоящую из системы линейных неравенств: 10х 4- 2у 84, 8# + 4т/ 120, х 0, 2/^0. Ограничение на лекарство А Ограничение на лекарство Б Нельзя использовать отрицательное количество вещества Л1 Нельзя использовать отрицательное количество вещества Н Изображая графически систему линейных неравенств, получим область допустимых решений (рис. 5.9). Таким образом, любая точка заштрихованной области (включая прямолинейные границы) будет отвечать условиям задачи; а любая точка вне заштрихованной области не будет отвечать требованиям. Например, 4 единицы вещества М и 23 единицы вещества Н будут отвечать условиям задачи, а 4 единицы вещества М и 21 единица вещества Н — нет. (Отметим, что область решений не ограничена.) ■ Упражнение 5.5. (Распределение ресурсов). Завод выпускает два типа открытых лодок — двухместные и четырехместные. Выпуск каждой двухместной лодки тре¬ бует затрат 0,9 рабочего часа в цехе раскройки и 0,8 рабочего часа в сборочном цехе. Выпуск каждой четырехместной лодки требует затрат 1,8 рабочего часа в цехе раскрой¬ ки и 1,2 рабочего часа в сборочном цехе. Количество рабочих часов в цехе раскройки и сборочном цехе составляет соответственно 864 и 672 в месяц. 1. Представьте эту информацию в виде таблицы.
Глава 5. Линейные неравенства и линейное программирование 413 2. Если х — количество двухместных лодок и у — количество четырехместных лодок, выпускаемых ежемесячно, запишите систему линейных неравенств, отражающих производственный процесс. Найдите множество возможных решений графически. ■ Замечания, Вернемся к примеру 5.5 и упражнению 5.5. В примере 5.5, конечно, есть смысл рассматривать Нецелые значения х и у. В упражнении 5.5 ситуация не столь однозначна. Как интерпретировать производство 214,5 двухместных и 347,75 четырехместных лодок в месяц? Всегда можно усреднить величины 214,5 и 314,75 для определения реалистичного объема производства. В общем, можно предполагать, что все точки в допустимой области представляют собой приемлемые решения, хотя нецелые решения могут требовать специальной интерпретации. Ответы к упражнениям 5.1. Изобразим прямую 6х — Зу = 18 пунктирной линией (поскольку равенство не включается). Выбирая в качестве контрольной точки начало координат (0; 0), получим, что 6 0 — 3 0 > 18 — неверное утверждение. Таким образом, нижняя полуплоскость, определенная равенством 6х — Зу = 18, представляет собой полуплоскость 6х - Зу > 18.
414 Часть II. Конечная математика Затрачиваемые рабочие часы Общее количество рабочих часов в месяц Двухместная лодка Четырехместная лодка Цех раскройки 0,9 1,8 864 Цех сборки 0,8 1,2 672 2) 0,9х + 1,8?/ С 864, 0,8х 4-1,22/ < 672, х > 0, у 0. Практикум 5.1 А Изобразите графически неравенства в задачах 1-10. 1. у х — 1. 3. Зх - 2у > 6. 2. у > х 4-1. 4. 2х — 5у < 10. 5. х —4. 7. 6х 4- 4у > 24. 6. у < 5. 8. 4х 4- 8у 32.
Глава 5. Линейные неравенства и линейное программирование 415 9. 5х < — 2у. 10. 6ж > 4т/. В задачах 11—14 подберите область решения систем линейных неравенств из четырех областей, изображенных на рисунке. 11. Зя - 2т/ > 0. 13. х + 2у 8, Зх — 2у 0. 14. х + 2у < 8, Зх — 2у 0. В задачах 15-18 решите системы линейных неравенств графически. 15. Зх -I- у 6, 17. х-2у^ 12, 2х + у 4. 16. Зх + 4т/ < 12, х < 4. -3. 18. 2х + Зу < 20, х — Зу —5. При решении задач 19-22 следует использовать графическую утилиту, дающую пользователю возможность заштриховать верхнюю или нижнюю части графика. 1. Изобразите граничную линию в стандартном окне и заштрихуйте область, которая содержит точки, удовлетворяющие всем неравенствам. 2. Повторите задание 1, но на этот раз заштрихуйте область, содержащую точки, которые не удовлетворяют всем неравенствам. (См. задание 5.2 и рис. 5.7.) Объясните, как можно распознать область решения на каждом графике. 19. я + г/^5, 20. х — 2у 1, 2х - у 1. х + Зу 12. 21. 2х + у > 4, 22. Зх + у -2, Зх — у 7. х — 2у —6. Б В задачах 23-26 выберите одну из четырех областей решения системы линейных неравенств, изображенных на рисунке. Определите угловые точки каждой области решения.
416 Часть II. Конечная математика 23. 25. х + 32/ 18, 2х + ?/ > 16, х О, 2/^0. х + Зу 18, 2х + у 16, х О, У 0. 2х -|- у < 16, х О, 2/^0. 26. х -I- Зу 18, 2х -I- у 16, х О, 2/ > 0. jc_ Решите системы неравенств в задачах 27-36 графически и определите, какая это область: ограниченная или неограниченная. Найдите координаты всех угловых то- чек. 27. 2х -I- Зу : 12, 28. Зх + 42/ 3 S 24, х 3 j 0, х J г 0, У * i 0. у г г 0. 29. 2х + у $ : ю, 30. 6х + Зу S 24, х + 2у $ : 8, Зх -|- бу з $30, х 3 г 0, х J г 0, У * i 0. У 5 >, 0. 31. 2х + 2/5 i 10, 32. 4х + Зу; г 24, х + 2у ; J 8, Зх + 42/ J г 8, х 5 г 0, х J г 0, У 5 г 0. У г 0. 33. 2х -Ь у $ U0, 34. Зх -к 2/ - $ 21, х+ у S S 7, х-Ь 2/ > $ 9, х + 2у s J 12, х + Зу $ $ 21, х J г 0, х J S 0, У 5 г 0. У 'i г 0.
Глава 5. Линейные неравенства и линейное программирование 417 35. 2х + у 2 S 16, 36. Зх + у t >, 24, х+ у; г 12, х+ у t >, 16, х 4- 2у f г 14, х + Зу; ъ 30, X f s 0, X J £ 0, У > о. у 0. ю. Проверьте решение задач 27-36 с помощью графической утилиты. О В Решите задачи 37-46 графически и определите, какой является каждая из областей решения: ограниченной либо неограниченной. Найдите координаты всех угловых точек. 37. х + 4у 32, Зх + у 30, 38. у < х + 5у 2х + у 11, 15, 12. 4я + 5т/ J г 51. 39. 4х + Зу s £48, 40. 2х + Зу 24, 2х + у J г 24, х + Зу 15, X S £ 9. У > 4. 41. х-у 0, 42. 2х + Зу 12, 2х - у 4, -х + Зу < з, 0 х 8. 0 < у < 5. 43. -х + Зу >1, 44. х+ У : 10, 5х — у >9, 5х + Зу : 15, я + у <9, -2х + Зу : 15, X s£5. 2х — 5у : s£ 6. 45. 16х + 13у 119, *=s 46. 8х + 4у s£41, 12х + 16у > 101, —15х + 5у s£ 19, -4х + 3j 11. 2х + бу 37. В задачах 47 и 48 найдите угловые точки области решения алгебраически без изображения графика. 47. Рассмотрите следующую систему неравенств и соответствующие границы. Зх + 4у С 36, Зх + 4т/ = 36, Зх 4- 2у < 30, Зх + 2у = 30, я > 0, х = 0, У 0. у = 0. а) Используйте алгебраические методы для определения точек пересечения (если они существуют) каждой возможной пары граничных линий. (Существует шесть различных возможных пар.) б) Проверьте каждую точку пересечения во всех четырех неравенствах для определения угловых точек.
418 Часть II. Конечная математика * 48. Повторите решение задачи 47 для следующих неравенств. 2х -I- у 16, 2х -I- Зу < 36, х О, 2/ > 0. 2х + у = 16, 2х + Зу = 36, х = 0, У = 0. Применение математики Экономика и бизнес 49. Производство: распределение ресурсов. Фирма-производитель выпускает два вида водных лыж: спортивные и обычные. Изготовление пары спортивных лыж занимает шесть рабочих часов, а их отделка — один час. Изготовление пары обычных лыж соответственно требует четырех рабочих часов, а отделка — одного. Всего на изготовление и отделку продукции ежедневно тратиться 108 и 24 часов. Обозначив через х количество пар спортивных лыж и через у — количество пар обычных лыж, производимых ежедневно, запишите систему линейных неравенств, которая определяет соответствующие ограничения на переменные хну. Найдите количество допустимых решений графически для числа пар лыж каждого типа, которое может быть произведено. 50. Производство: распределение ресурсов. Компания по производству мебели изготавливает столы и стулья. Изготовление стола занимает восемь рабочих часов, а отделка - два рабочих часа. Для производства стула соответственно необходимо два и один рабочий час. Всего ежедневно на изготовление и отделку мебели тратиться соответственно 400 и 120 часов. Обозначив через х количество столов, а через у — количество стульев, производимых ежедневно, запишите систему линейных неравенств, которая определяет соответствующие ограничения на переменные х и у. Найдите графически множество допустимых решений для количества столов и стульев, которое может быть произведено. * 51. Производство: распределение ресурсов. Вернемся к задаче 49. Компания получает прибыль 50 долл, от реализации пары спортивных лыж и прибыль 60 долл, от реализации пары обычных лыж. а) Если компания производит по 10 спортивных и обычных лыж ежедневно, дневная прибыль будет составлять 1100 долл. Существуют ли другие гра-
Глава 5. Линейные неравенства и линейное программирование 419 фики производства, которые дадут дневную прибыль 1100 долл.? Как они соотносятся с графиком прямой 50ж + 60?/ = 1100? б) Найдите Графики производства, которые дадут дневную прибыль, превышающую 1100 долл., и повторите решение п. а для этих показателей. в) Рассмотрите методы применения прямых линий, подобных использованным при решении пп. а и б для выявления наибольшей возможной дневной прибыли. * 52. Производство: распределение ресурсов. Вернемся к примеру 50. Компания получает прибыль 50 долл, от производства каждого стола и 15 долл, от производства каждого стула. а) Если компания выпустит 20 столов и 20 стульев в день, дневная прибыль составит 1300 долл. Существуют ли другие графики производства, дающие дневную прибыль, равную 1300 долл.? Как они соотносятся с графиком прямой линии 50ж + 15?/ = 1300? б) Найдите графики производства, которые дадут дневную прибыль, превышающую 1300 долл., и повторите решение п. а для этих показателей. в) Рассмотрите методы использования прямых линий, подобных рассмотренным в пп« а и бу для выявления наибольшей возможной дневной прибыли. Биологические науки 53. Удобрение растений. Фермер может купить два типа удобрений, смесь А и смесь Б. Каждый кубический ярд смеси А содержит 20 фунтов фосфорных, 30 азотных и 5 фунтов калийных удобрений. Каждый кубический ярд смеси Б содержит 10 фунтов фосфорных, 30 фунтов азотных и 10 фунтов калийных удобрений. Минимальная ежемесячная потребность составляет 460 фунтов фосфорных, 960 фунтов азотных и 220 фунтов калийных удобрений. Обозначив через х количество используемых кубических ярдов смеси Л, а через у — количество используемых кубических ярдов смеси Б, запишите систему линейных неравенств, которая определяет соответствующие ограничения на переменные хну. Найдите графически множество допустимых решений для объемов смеси А и смеси Б, которые могут быть использованы. 54. Диета. Диетолог в больнице должен разработать специальную диету из двух блюд. Каждая унция блюда М содержит 30 единиц кальция, 10 единиц железа и 10 единиц витамина А. Каждая унция блюда Н содержит 10 единиц кальция, 10 единиц железа и 30 единиц витамина А. Диета должна обеспечивать больных 360 единицами кальция, 160 единицами железа и 240 единицами витамина А. Обозначив через х количество унций блюда М, а через у — количество унций блюда Н, запишите систему линейных неравенств, которая отражает заданные условия. Найдите графически множество допустимых решений для размеров порции каждого блюда. Социальные науки 55. Поведение животных. В эксперименте по изучению поведения животных психолог использует два типа ящиков Скиннера (Skinner), в которых находятся крысы и мыши. Каждая мышь ежедневно проводит 10 мин. в ящике Л и 20 мин.
420 Часть II. Конечная математика в ящике Б. Каждая крыса ежедневно проводит 20 мин. в ящике Л и 10 мин. в ящике Б. Общее время эксперимента составляет 800 мин. ежедневно для ящика Л и 640 мин. для ящика Б. Сколько мышей и крыс может быть задействовано в эксперименте при сформулированных условиях? Обозначив через х количество мышей, а через у — количество крыс, запишите систему линейных неравенств, которая определяет соответствующие ограничения на переменные хну. Найдите множество допустимых решений, используя графики. 5.2. Линейное программирование в двух измерениях: геометрический подход ■ Задача линейного программирования ■ Линейное программирование: общее описание ■ Геометрическое решение задач линейного программирования ■ Решение практических задач Некоторые задачи, которые рассматривались в предыдущем разделе, связаны с задачей более общего вида, называемой задачей линейного программирования. Линейное программирование представляет собой математическую процедуру, которая была разработана в помощь менеджерам для принятия решения. Она стала одним из наиболее широко используемых и хорошо известных инструментов теории управления. Изучим эту тему, используя интуитивный геометрический подход. Это облегчит нам в дальнейшем освоение алгебраического подхода, который является менее интуитивным, но необходим при решении большинства реальных задач. Замечания. Для удобства в последующих разделах вместо символьных переменных, таких как х и у, используются индексированные формы записи, например, zi и я2.. Задача линейного программирования Начнем наши рассуждения с конкретного примера. С помощью геометрического метода получим две важные теоремы и простую общую геометрическую процедуру решения задач линейного программирования с двумя переменными. И Пример 5.6 (Производственное планирование). Производитель туристических палаток выпускает стандартную и экспедиционную модели, продавая их на общенациональном рынке. Для выпуска стандартной палатки необходимо затратить один рабочий час в цехе раскройки и три рабочих часа — в цехе пошива. На выпуск экспедиционной палатки затрачивается два рабочих часа в цехе раскройки и четыре рабочих часа — в цехе пошива. Общее производственное время цеха раскройки и цеха пошива составляет 32
Глава 5. Линейные неравенства и линейное программирование 421 и 84 часа соответственно. Сколько палаток каждого типа следует выпускать ежедневно для того, чтобы максимизировать общую дневную прибыль (предполагая, что все палатки проданы), если прибыль компании от реализации стандартной палатки равна 50 долл., а от реализации экспедиционной — 80 долл.? Решение. Это — пример задачи линейного программирования. Начнем с анализа поставленного вопроса. Искомые переменные Итак, цель менеджмента — получить максимальную прибыль. Поскольку прибыль от реализации стандартной и экспедиционной палаток различна, необходимо решить, сколько единиц каждого типа палаток производить. Таким образом, естественно ввести следующие искомые Переменные. Пусть xi — количество стандартных палаток, производимых ежедневно, Х2 — количество экспедиционных палаток, производимых ежедневно. Представим производственные требования, цели и ограничения в табл. 5.1. Таблица 5.1. Производственные требования и ограничения Время изготовления одной Общее производственное время, ч палатки, ч Стандартная модель Экспедиционная модель Цех раскройки 1 2 32 Цех пошива 3 4 84 Прибыль от одной палатки, долл. 50 80 Целевая функция Используя искомые переменные и информацию из табл. 5.1, можно записать целевую функцию. Р = 50xi + 80x2- Цель задачи — найти значения искомых переменных, при которых целевая функция достигает оптимального значения (в данном случае — максимального). Ограничения Из вида целевой функции следует, что прибыль может быть сколь угодно большой, если выпустить неограниченное количество палаток. Однако любая компания-производи-
422 Часть II. Конечная математика тель имеет конечные ресурсы, производственную мощность, спрос и другие параметры, определяющие ограничения задачи. Используя информацию из табл. 5.1, можно определить два ограничения задачи. Производственные ограничения цеха раскройки / Ежедневные \ / Ежедневные \ затраты времени на раскройку xi + затраты времени на раскройку Х2 стандартных \ палаток / 1X1 + экспедиционных палаток У 2х2 / Общее \ производственное у время у 32 Производственные ограничения цеха пошива / Ежедневные \ затраты времени на раскройку xi стандартных у палаток / 3xi / Ежедневные \ затраты времени на раскройку Х2 экспедиционных палаток 4X2 / Общее \ производственное у время у 84 Неотрицательность Поскольку невозможно произвести отрицательное количество палаток, то необходимо ввести условия неотрицательности xi > 0, Х2 0, которые записываются в виде Х1, х2 0. Математическая модель Рассмотрим математическую модель данной задачи. Максимизировать функцию при условиях Р = 50xi + 80x2 Целевая функция Xi + 2x2 С 32, Первое ограничение задачи 3xi -1- 4x2 84, Второе ограничение задачи Xi,X2 0. Условия неотрицательности Графическое решение Графически решая множество линейных неравенств, описывающих производственные ограничения, получим область допустимых решений (рис. 5.10). Выбирая из области допустимых решений производственный план (xi, Х2), можно вычислить прибыль фабрики. Р = 50xi -|- 80x2- Например, если х^ = 12 и Х2 = 10, прибыль за день составляла бы Р = 50 • 12 + 80 • 10 = 1400 долл.
Глава 5. Линейные неравенства и линейное программирование 423 Рис 5.10. Область допустимых решений Если же xi = 23 и xj = 2, прибыль за день была бы равной Р = 50 • 23 + 80 • 2 = 1300 долл. Вопрос заключается в Том, какой производственный план (xi, Х2) из допустимой области обеспечивает максимальную прибыль? Таким образом, возникает задача максимизации. Поскольку поточечная проверка невозможна (пришлось бы проверить бесконечно много точек), необходимо найти иной путь. Присваивая переменной Р в равенстве Р = 50xi + 8ОХ2 конкретное значение целевой функции и изображая полученное уравнение в декартовой системе координат (см. рис. 5.10), получим линию постоянной прибыли. Точки, лежащие на этой линии в допустимой области, представляют собой производственные планы, обеспечивающие одинаковую прибыль. Проделывая эту процедуру для множества значений Р, получим семейство линий постоянной прибыли (рис. 5.11), которые являются параллельными друг другу, поскольку все они имеют один и тот же наклон. Чтобы убедиться в этом, запишем Р = 50xi + 80x2 в виде уравнения с угловым коэффициентом Отметим, что для любого значения Р линия постоянной прибыли имеет наклон — Кроме того, если величина Р возрастает, значение точки пересечения с осью хг(Р/80) также увеличивается, И линия постоянной прибыли удаляется от начала координат. Таким образом, максимальная прибыль достигается в точке, где линия постоянной прибыли наиболее удалена от начала координат, но все же находится в контакте с допустимой областью. В нашей задаче это происходит в точке (20, 6), как показано на рис. 5.11. Таким образом, если производитель изготавливает 20 стандартных и 6 экспедиционных палаток ежедневно, прибыль будет максимальной в точке (20, 6). Р = 50 • 20 + 80 • 6 = 1480 долл. Точка (20, 6) называется оптимальным решением задачи, поскольку она максимизирует целевую функцию (прибыль) и принадлежит допустимой области. В общем случае оказывается, что максимальная прибыль достигается в одной из угловых точек. Отметим также, что минимальная прибыль (Р = 0) достигается в угловой точке (0; 0). ■
424 Часть II. Конечная математика 20 ^2000 долл. Линия постоянной прибыли (о, 16); Ц480долл?'хх PV°V80x>’ v х где Р= 2000 долл. ю; ■1200дол^<х ■ 800 долл. хх(20,6) Оптимальное решение ЛОО долл. (0,0)1 10 20 (28,0) 30 40 Рис. 5.11. Линии постоянной прибыли к Упражнение 5.6. Фирма выпускает два типа резиновых надувных лодок — двух- и четырехместные. Для выпуска двухместной лодки необходимо затратить 0,9 рабочего часа в цехе раскройки и 0,8 рабочего часа в сборочном отделении. Для выпуска четырехместной лодки необходимо 1,8 рабочего часа в цехе раскройки и 1,2 рабочего часа в сборочном отделении. Общее производственное время первого и второго цехов составляет 864 и 672 часов соответственно. Компания получает 25 долл, от продажи двухместной и 40 долл, от продажи четырехместной лодки. 1. Определите искомые переменные. 2. Представьте производственные ограничения в таблице, аналогичной табл. 5.1 из примера 5.6. 3. Запишите целевую функцию Р. 4. Запишите ограничения задачи и условия неотрицательности. 5. Изобразите область допустимых решений, а также графики целевой функции для Р = 5000, Р = 10 000, Р = 15 000 и Р = 21600. 6. С помощью графика и линий постоянной прибыли определите, сколько лодок нужно производить ежемесячно, чтобы получить максимальную прибыль. Какова максимальная прибыль? ■ Прежде чем двигаться дальше, обозначим последовательность действий по созданию модели в примере 5.6. Построение модели для решения задачи прикладного линейного программирования Этап 1. Определяем искомые переменные. Этап 2. Представляем ограничения задачи в виде таблицы, ставя в соответствие искомым переменным столбцы таблицы (см. табл. 5.1). Этап 3. Определяем цель и записываем линейную целевую функцию. Этап 4. Записываем ограничения задачи, используя линейные уравнения и/или неравенства. Этап 5. Записываем условия неотрицательности.
Глава 5. Линейные неравенства и линейное программирование 425 Задание 5.3. Обратимся к области Допустимых решений S, изображенной на следующем рисунке. 1. Пусть Р = х\ +х2. Нарисуйте линии постоянной прибыли, проходящие через точки (5, 5) и (10, 10). Нарисуйте отрезок линии с наименьшей прибылью и двигайте его в направлении возрастания прибыли, не изменяя наклона. Каково максимальное значение прибыли Р? Где достигается этот максимум? 2. Повторите решение задачи 1 для равенства Р = х^ + 10#2- 3. Повторите решение задачи 1 для Р = 10xi + #2- ■ Линейное программирование: общее описание В примере 5.6 и упражнении 5.6 оптимальное решение достигается в угловой точке области допустимых решений. Всегда ли так происходит? Теорема 5.2 утверждает, что ответ положителен. Сначала дадим несколько общих определений. Целью задачи линейного программирования является поиск оптимального значения (максимального или минимального) линейной целевой функции вида z = cixi + с2х2 Ч \-спхп, где искомые переменные #i, x2i..., хп подчинены ограничениям задачи в виде линейных неравенств и уравнений. Кроме того, переменные должны удовлетворять условиям неотрицательности Xi 0, i = 1, 2, ...,п. Множество точек, удовлетворяющих как ограничениям задачи, Так и условиям неотрицательности, называется областью допустимых решений задачи. Любая точка этой области, в которой целевая функция достигает оптимального значения, называется оптимальным решением. Теорема 5.2 (Основная теорема линейного программирования). Если оптимальное значение целевой функции в задаче линейного программирования существует, оно должно достигаться в одной (или нескольких) угловых точках допустимой области. ■ Теорема 5.2 позволяет сформулировать простую процедуру решения задачи линейного программирования при условии, что задача имеет оптимальное решение. Следует иметь в виду, что не все задачи имеют оптимальное решение. Дря того чтобы использовать теорему 5.2, необходимо знать, что рассматриваемая задача имеет оптимальное решение. Условия, обеспечивающие существование оптимального решения задачи линейного программирования, определяются теоремой 5.3.
426 Часть II. Конечная математика Теорема 5.3 (Существование оптимальных решений). 1. Если область допустимых решений задачи линейного программирования ограничена, то как максимальное, так и минимальное значение целевой функции всегда существует. 2. Если область допустимых решений не ограничена и коэффициенты целевой функции положительны, то минимальное значение целевой функции существует, а максимальное — нет. 3. Если область допустимых решений пуста (т.е. не существует точек, удовлетво¬ ряющих всем ограничениям), то как максимальное, так и минимальное значения целевой функции не существуют. ■ Теорема 5.3 не охватывает все варианты. Например, что происходит в том случае, если областиь допустимых решений не ограничена и один (или оба) коэффициента целевой функции отрицательны? Задачи такого типа должны решаться с помощью внимательного изучения графика целевой функции для различных допустимых решений, как это делалось в примере 5.6. Геометрическое решение задач линейного программирования Рассуждение, приведенное выше, приводит к следующей процедуре для геометрического решения задач линейного программирования с двумя искомыми переменными. Геометрическое решение задачи линейного программирования с двумя переменными Этап 1. Нарисуем область допустимых решений. Затем, если оптимальное решение существует (т.е. выполняются условия теоремы 5.3), найдем координаты всех угловых точек. Этап 2. Составим таблицу, перечисляя значения целевой функции во всех угловых точках. Этап 3. Определим оптимальное решение (решения) из таблицы, построенной на этапе 2. Этап 4. Для применяемой задачи дадим интерпретацию оптимального решения (решений) в терминах исходной задачи. Применим эту процедуру для решения некоторых задач линейного программирования, в которых модель уже определена. Пример 5.7 (Решение задачи линейного программирования). 1. Минимизировать и максимизировать функцию z = 3xi + х2 при условиях 2xi + х2 20, 10X1 + #2 > 36, 2xi + 5x2 36, xi,x2 > 0.
Глава 5. Линейные неравенства и линейное программирование 427 2. Минимизировать и максимизировать функцию z — 10xi -I- 20^2 при условиях 6xi -I- 2х2 > 36, 2xi + 4x2 > 32, Х2 < 20, Х1,Х2 0. Решение. Задача 1. Этап 1. Изобразим Область допустимых решений S. Затем после проверки условий существования оптимального решения с помощью теоремы 5.3 вычислим координаты всех угловых точек. Поскольку область S ограничена, функция z будет имеет как максимум, так и минимум в этой области (теорема 5.3.1), и они оба достигаются в угловых точках (теорема 5.2). Угловая точка (®х; х2) z = За?1 + (3;6) 15 (2; 16) 22 (8; 4) 28 Этап 2. Найдем значение целевой функции во всех угловых точках, как показано в таблице. Этап 3. Определим Оптимальные решения с помощью этапа 2. Изучая значения, приведенные в Таблице, видим, что минимальное значение функции z равно 15 и достигается в точке (3; 6), а максимальное значение функции z равно 28 и достигается в точке (8; 4). Задача 2. Этап 1. Изобразим область допустимых решений S. Затем, после проверки условий существования оптимального решения с помощью теоремы 5.3, находим координаты всех угловых точек. Поскольку область S не ограничена и коэффициенты целевой функции положительны, функция z имеет минимальное значение на этой области, но не имеет максимального (теорема 5.3.2).
428 Часть II. Конечная математика Угловая точка (хх; х2) z = 10о?1 + 20x2 (0; 20) 400 (0; 18) 360 (4; 6) 160 (16; 0) 160 Этап 2. Найдем значения целевой функции во всех угловых точках, как показано в таблице. Этап 3. Определим оптимальное решение с помощью этапа 2. Минимальное значение функции z равно 160 и достигается в точках (4; 6) и (16; 0). ■ Решение примера 5.7 представляет собой кратное оптимальное решение. Вообще говоря, если две угловые точки являются оптимальными решениями задачи линейного программирования, то любая точка прямолинейного отрезка, соединяющего их, также является оптимальным решением. Это единственный случай, когда оптимальные решения также находятся в неугловых точках. Упражнение 5.7. 1. Максимизируйте и минимизируйте функцию z = 4xi + 2x2 при ограничениях, указанных в примере 5.7.1. 2. Максимизируйте и минимизируйте функцию z = 20xi 4- 5д?2 при ограничениях, данных в примере 5.7.2. ■ Задачи линейного программирования полезно изучать с помощью компьютеров. На рис. 5.12 изображено решение примера 5.7.1 на графическом калькуляторе, а на рис. 5.13 — решение примера 5.7.2 с помощью аплета, написанного Крисом Джонсом (Chris Jones) на языке Java. Задание 5.4. В примере 5.7.2 не было оптимального решения задачи максимизации целевой функции Р в области допустимых значений S. Чтобы оптимальное решение задачи максимизации существовало, желательно добавить дополнительное ограничение и модифицировать область допустимых решений. Какое из следующих ограничений позволяет достигнуть поставленной цели?
Глава 5. Линейные неравенства и линейное программирование 429 Рис. 5Л2. Решение примера 5.7.1 на графическом калькуляторе Рис. 5.13* Решение примера 5.7.2 на графическом калькуляторе 1. xi 20 2. х2 4 3. Xi С Х2 4. Х2 < Xi Для иллюстрации Теоремы 5.3.3 рассмотрим следующую задачу. Максимизировать функцию Р = 2х± + 3x2 При УСЛОВИЯХ Xi + Х2 8, xi + 2х2 8, 2x1 + #2 Ю) Xl, Х2 > 0. Пересечением неравенств, задающих ограничения, является пустое множество (рис. 5.14). Следовательно, область допустимых решений пуста (см. приложение А.1, т. 2). В этом случае необходимо заново сформулировать задачу, например, пересмотреть количество рабочих часов, бюджет и объем снабжения, установленные в проекте, чтобы получить непустую область допустимых решений.
430 Часть II. Конечная математика Рис. 5.14. Пустая область допустимых решений Решение практических задач И Пример 5.8 (Медицина). Теперь преобразуем пример 5.5 из предыдущего раздела в задачу линейного программирования. Пациенту в больнице необходимо получать по крайней мере 84 единицы лекарства Л и 120 единиц лекарства Б ежедневно (предполагается, что передозировка обоих лекарств полностью безопасна). Грамм вещества М содержит 10 единиц лекарства Л и 8 единиц лекарства Б, грамм вещества Н содержит две единицы лекарства Л и четыре единицы лекарства Б. Теперь предположим, что как вещество М, так и вещество Н содержат нежелательное лекарство Д: три единицы на грамм вещества М и одну единицу на грамм вещества Н. Сколько граммов каждого из веществ М^Н следует смешать, чтобы удовлетворить минимальную суточную потребность и в то же время минимизировать влияние лекарства Д? Сколько единиц нежелательного лекарства Д будет в этой смеси? Решение. Сначала построим математическую модель. Этап 1. Введем искомые переменные. В соответствии с вопросами, поставленными в этом примере, нам требуется решить, сколько граммов веществ М и Н следует смешать, чтобы получить дневную дозу лекарства. Таким образом, переменными являются следующие величины. xi — количество граммов вещества М, Х2 — количество граммов вещества Н. Этап 2. Подытожим соответствующий материал в таблице, поместив в столбцы количество веществ М и Н. Количество вещества в грамме Минимальная суточная потребность Вещество Af, ед. Вещество Я, ед. Лекарство А 10 2 84 Лекарство Б 8 4 120 Лекарство Д 3 1
Глава 5. Линейные неравенства и линейное программирование 431 Этап 3. Определим цель и целевую функцию. Целью является минимизация количества лекарства Д 1 суточной дозе лекарства. Используя переменные и информацию, приведенную в таблице, получим линейную целевую функцию С — 3xi + Этап 4. Запишем ограничения задачи. lOxi + 2х2 84 8xi 4- 4x2 120 Ограничение на лекарство А Ограничение на лекарство Б Этап 5. Добавим условия неотрицательности и подведем итог. Минимизировать функцию С = 3xi + Х2 Целевая функция при условиях lOxi 4- 2х2 > 84 8xi -I- 4x2 > 120 Х1,Х2 0 Ограничение на лекарство А Ограничение на лекарство Б Ограничения неотрицательности Теперь используем геометрический метод решения задачи (рис. 5.15). Этап 1. Нарисуем область допустимых решений. Затем после проверки условий существования оптимального решения с помощью теоремы 5.2 найдем координаты всех угловых точек. Решая систему неравенств ограничений графически, получим допустимую область, изображенную на рис. 5.15. Поскольку допустимая область не ограничена и коэффициенты целевой функции положительны, эта задача минимизации имеет решение. X 40; 30; 20: 10; 2 (0,42) : \ Область - у4,22) допустимых - \ значений : \(15,0) Угловая точка (®i; С = 3a?i + (0; 42) 42 (4; 22) 34 (15; 0) 45 -k-i »- у 0 10 20 30 4 0 1 Рис. 5.15. Область допустимых решений Этап 2. Вычислим значения целевой функции во всех угловых точках, как показано в таблице. Этап 3. Определим оптимальное решение с помощью этапа 2. Оптимальным решением является число С = 34 в угловой точке (4; 22).
432 Часть II. Конечная математика Этап 4. Дадим интерпретацию оптимальному решению в терминах исходной задачи. Если используются 4 единиц вещества М и 22 единицы вещества Я, будут обеспечены минимальные суточные потребности в лекарствах А и Б и одновременно минимизировано нежелательное воздействие лекарства Д, получаемого в количестве 34 единиц. (Любая другая комбинация М и Н из области допустимых решений приведет к большему количеству нежелательного лекарства Д.) ■ к Упражнение 5.8 (Сельское хозяйство). Фермер, занимающийся разведением кур, может купить специальную пищевую смесь А по 20 центов за фунт и специальную пищевую смесь Б по 40 центов за фунт. Фунт смеси А содержит 3000 единиц питательного вещества Hi и 1000 единиц питательного вещества Н?; фунт смеси Б содержит 4000 единиц питательного вещества Hi и 4000 единиц питательного вещества Яг - Сколько фунтов каждой смеси нужно использовать ежедневно, чтобы минимизировать дневные расходы на питание, удовлетворяя (или превышая) минимальные дневные потребности в питательных веществах, если минимальные дневные потребности для совместного кормления цыплят составляют 36 000 единиц питательного вещества Hi и 20 000 единиц питательного вещества Яг? Чему равна минимальная величина ежедневных затрат? Постройте математическую модель и решите задачу, используя геометрический метод. ■ Замечания. Обратитесь к примеру 5.8. Если изменить минимальную потребность в лекарстве Л со 120 до 125 г, оптимальное решение изменится до 3,6 г вещества А/ и 24,1 г вещества Я с точностью до одной десятой. Теперь обратитесь к примеру 5.6. Если изменить общее производственное время в сборочном отделении с 84 до 79, оптимальное решение изменится до 15 стандартных и 8,5 экспедиционных палаток. Можно отмерить 3,6 г вещества М и 24,1 г вещества Я, но как можно изготовить 8,5 палатки? Следует сделать 8 или 9 палаток? Если решение задачи должно быть целым, а оптимальное решение, найденное графически, включает десятичные дроби, приближение десятичной дроби к ближайшим целым не всегда дает оптимальное целое решение (см. задачу 36, упражнение 5.2). Процесс поиска оптимальных целых решений задачи линейного программирования называется целочисленным программированием и требует специальной техники, изучение которой выходит за рамки этой книги. Как отмечалось выше, если мы столкнемся с дробным решением (например, 8,5 палатки в день), будем рассматривать его как среднее значение для длинных производственных сроков.
Глава 5. Линейные неравенства и линейное программирование 433 Ответы к упражнениям 5.6. 1) х\ — количество двухместных лодок, производимых ежемесячно, Х2 — количество четырехместных лодок, производимых ежемесячно. 2) Время изготовления, ч Максимальное количество рабочих часов в месяц Двухместная лодка Четырехместная лодка Цех раскройки 0,9 1,8 864 Сборочный цех 0,8 1,2 672 Прибыль от одной лодки, долл> 25 40 3) Р = 25xi + 40д?2‘ 4) 0,9xi + 1,8^2 < 864, 0,8xi -F 1,2x2 С 672, xi, х2 > 0. 6) 480 двухместных лодок, 240 четырехместных лодок, ежемесячный максимальный доход Р — 21600 долл. 5.7. 1) Минимальное значение z = 24 в точке (3; 6); максимальное значение z = 40 в точках (2; 16) и (8; 4) (кратное оптимальное решение). 2) Минимальное значение z — 90 в точке (0; 18); нет максимального значения. 5.8. min С = 0,2xi + 0,4хд при условиях 3000X1 + 4000x2 > 36000, 1000X1 + 4000x2 20000, Х1, Х2 > 0. Восемь фунтов смеси Л, три фунта смеси Б\ минимальные затраты С = 2,80 долл, ежедневно.
434 Часть II. Конечная математика Практикум 5.2 А Найдите максимальные значения всех целевых функций в задачах 1-4 в области допустимых значений S, изображенной на рисунке. 1. z — х + у. 2. z = 4х + у. 3. z = Зх + 7у. 4. z = 9х + 31/. Найдите минимальные значения всех целевых функций в задачах 5-8 в области допустимых значений Т, изображенной на рисунке. 5. z — 7х + 41/. 6. z — 7х + 91/. 7. z = Зх + Sy. 8. z = 5х 4- 41/. Б Решите задачи линейного программирования, сформулированные в пунктах 9-26. 9. Максимизировать функцию Р = 5xi + 5x2 при условиях 2д?1 + ^2 С Ю, Х1 + 2х2 8, Xi, Х2 0. 10. Максимизировать функцию Р = 3xi 4- 2x2 при условиях 6xi + 3x2 24, 3xi 4- 6x2 30, Xi, Х2 > 0. 11. Минимизировать и максимизировать функцию z — 2xi 4- Зхг при условиях 2xi 4- #2 Ю, Xi 4- 2x2 8, Xi, Х2 0.
Глава 5. Линейные неравенства и линейное программирование 435 12. Минимизировать и максимизировать функцию z = 8xi -I- 7хг при условиях 4xi + Зхг 24, 3xi 4* 4x2 8, Xi, Х2 > 0. 13. Максимизировать функцию Р = 30xi + 40x2 при условиях 2xi + #2 С Ю, xi + Х2 С 7, Xi 4- 2x2 С 12, Xi, Х2 > 0. 14. Максимизировать Р = 20xi + 10x2 при условиях 3X1 + Х2 С 21, Xi + Х2 С 9, xi 4- 3x2 С 21, Xi, Х2 0. 15. Минимизировать и максимизировать функцию z = 10xi + 30x2 при условиях 2X1 + Х2 > 16, Х1 + х2 > 12, xi + 2x2 14, Х1, 22 > 0. 16. Минимизировать и максимизировать функцию z = 400xi + 100х2 при условиях 3X1 + Х2 24, Х1 + Х2 16, Xi + 3x2 30, Xi, Х2 > 0. 17. Минимизировать и максимизировать функцию Р = 30xi + 10x2 при условиях 2xi + 2x2 4, 6X1 + 4x2 С 36, 2xi 4- Х2 С Ю, Xi, х2 > 0. 18. Минимизировать и максимизировать функцию Р = 2xi -I- Х2 При УСЛОВИЯХ Xi + Х2 2, 6xi -Ь 4x2 С 36, 4xi 4 2x2 С 20, Xl, Х2 > 0. 19. Минимизировать и максимизировать функцию Р = 3xi 4- 5x2 при условиях Х1 + 2x2 С 6, Xi 4- Х2 С 4, 2xi 4* 3x2 12, Xi, Х2 > 0.
436 Часть II. Конечная математика 20. Минимизировать и максимизировать функцию Р = — xi 4- 3^2 при условиях 2xi “ х2 4, -xi + 2x2 4, х2 6, Х1, Х2 0. 21. Минимизировать и максимизировать функцию Р = 20xi + 10х2 при условиях 2xi + Зх2 > 30, 2xi + ^2 С 26, —2x1 + 5x2 < 14, Xi, х2 0. 22. Минимизировать и максимизировать функцию Р = 12xi + 14х2 при условиях —2X1 + х2 6, #1 4- х2 С 15, 3xi — х2 0, Xi, Х2 0. 23. Максимизировать функцию Р = 20xi + 30х2 при условиях 0,6xi + 1,2x2 960; 0,03xi + 0,04x2 < 36; 0,3xi + 0,2x2 С 270; Xi, х2 0. 24. Минимизировать функцию С = 30xi + 10х2 при условиях l,8xi 4- 0,9х2 270; 0,3xi + 0,2x2 > 54; 0,01xi + 0,03x2 39; Х1, Х2 0. *5 25. Максимизировать функцию Р = 525xi 4- 478х2 при условиях 275xi + 322х2 < 3381, 350xi 4- 340x2 С 3762, 425x1 4-306x2 ^4114, Xi, Х2 0. 26. Максимизировать Р = 300xi + 460х2 при условиях 245xi + 452х2 С 4181, 290x1 + 379х2 3888, 390xi 4- 299x2 4407, Xi, х2 0.
Глава 5. Линейные неравенства и линейное программирование 437 Почему в задачах 27 и 28 нельзя использовать теорему 5.2, чтобы сделать вывод о том, что максимальное или минимальное значение существуют? Нарисуйте области допустимых решений и рассмотрите вопрос существования максимального и минимального значений целевой функции z = xi — Х2 с помощью графиков. * 27. Минимизировать и Максимизировать функцию z = xi — Х2 При условиях Х1 ““ 2X2 С О, 2#i Х2 С 6, #1, Х2 0. *28. Минимизировать и максимизировать функцию z = х± — Х2 при условиях - 2X2 “6, 2#i — Х2 0, $1, Х2 > 0. в * 29. Угловыми точками области допустимых решений, определенной системой ли¬ нейных неравенств х± + 2x2 IO? 3a7i + Х2 15, Xl, Х2 0, являются О ~ (0; 0), А = (0; 5), В = (4; 3) и С = (5; 0). Если Р = ах± + Ьх2 и а, 6 > 0, Определите условия, которым должны удовлетворять числа а и 6, чтобы максимальное значение Р существовало в следующих точках. а) Только в точке А. б) Только в точке В. в) Только в точке С. г) Как в А, так и в В. д) Как в В, Так и в С. * 30. Угловыми точками области допустимых решений, определенными системой ли¬ нейных неравенств xi + 4я?2 > 30, 3x1 + Х2 24, Xi, Х2 > 0, являются А & (0; 24), В = (6; 6) и D = (30; 0). Если С = ах\ +Ъх2 иа,Ь > 0, определите, каким условиям должны удовлетворять а и Ь, чтобы минимальное значение С существовало существовало в следующих точках. а) Только в точке А. б) Только в точке В. в) Только в Точке D. г) Как в А, так и в В. д) Как в В, Так и В D.
438 Часть II. Конечная математика Применение математики В задачах 31-46 требуется построить математическую модель в форме задачи линейного программирования и решите ее с помощью геометрического метода. Экономика и бизнес 31. Производство: распределение ресурсов. Компания-производитель выпускает два типа водных лыж — обычные лыжи и лыжи для слалома. Соответствующая производственная информация приведена в следующей таблице. Цех Время производства пары лыж, ч Общее производственное время, ч Обычные лыжи Лыжи для слалома Цех сборки 6 4 108 Цех отделки 1 1 24 а) Сколько пар лыж каждого типа необходимо производить, чтобы прибыль была максимальной, если прибыль от продажи обычных лыж составляет 4 долл., а от продажи лыж для слалома — 30 долл.? *б) Как изменится производственный план и максимальная прибыль, если прибыль от продажи пары лыж для слалома уменьшится до 25 долл.? *в) Как изменится производственный план и максимальная прибыль, если прибыль от продажи пары лыж для слалома увеличится до 45 долл.? 32. Производство: распределение ресурсов. Мебельная фабрика выпускает обеденные столы и стулья. Соответствующая производственная информация приведена в следующей таблице. Цех Время производства единицы продукции, ч Общее производственное время, ч Стол Стул Отдел сборки 8 2 400 Отдел комплектации 2 1 120 Прибыль, долл. 90 25 а) Сколько столов и стульев необходимо производить ежедневно, чтобы добиться максимальной прибыли? Чему равна максимальная прибыль? *б) Как изменится производственный план и максимальная прибыль, если отдел маркетинга решит, что количество производимых стульев должно быть по крайней мере в четыре раза больше числа производимых столов? 33. Планирование производства. Мебельная компания имеет два завода, которые производят пиломатериалы, используемые при изготовлении столов и стульев. За один рабочий день завод А может произвести пиломатериалы, требуемые для изготовления 20 столов и 60 стульев, а завод Б может произвести пиломатериалы, требуемые для изготовления 25 столов и 50 стульев. Компании необходимо достаточно пиломатериалов для производства по крайней мере 200 столов и 500 стульев. а) Определите, сколько дней должен работать каждый завод, чтобы произвести достаточное количество пиломатериалов с минимальными затратами, если
Глава 5. Линейные неравенства и линейное программирование 439 один день работы завода А стоит 1000 долл., а один день работы завода Б — 900 долл.? Чему равны минимальные затраты? *б) Как изменится производственный план и максимальная прибыль, если минимальная величина ежедневных затрат завода А уменьшится до 600 долл., а все остальные данные в задаче А останутся прежними? *в) Как изменится производственный план и максимальная прибыль, если минимальная величина ежедневных затрат завода Б уменьшится до 800 долл., а все остальные показатели в задаче А останутся прежними? 34. Производство: распределение ресурсов. Компания выпускает два типа компьютеров — настольные и ноутбуки. Производство настольного компьютера требует 400 долл, капитальных вложений и 40 рабочих часов. Производство ноутбука требует 250 Долл, капитальных вложений и 30 рабочих часов. Капитал фирмы равен 20000 долл., а на производство настольных компьютеров и ноутбуков затрачиваются 2160 рабочих часов. а) Какое максимальное количество компьютеров способна производить компания? *б) Каждый настольный компьютер приносит 320 долл, прибыли, а ноутбук — 220 долл. Какова будет прибыль компании от производства максимального количества компьютеров, определенного при решении п. а? Является ли эта прибыль максимальной? Если нет, то какова максимальная прибыль? 35. Логистика. Школьные учителя планируют арендовать автобусы и микроавтобусы для экскурсии. Каждый автобус может перевозить 40 учеников и трех сопровождающих, причем его аренда стоит 1200 долл. Каждый мироактобус может перевозить восемь учеников и одного сопровождающего, а его аренда стоит 100 долл. Поскольку в путешествие могут поехать 400 учеников, воспитатели должны планировать отправить по крайней мере 400 человек. Поскольку сопровождать учащихся вызвались только 36 родителей, воспитатели должны планировать использовать по крайней мере 36 сопровождающих. Сколько техники каждого типа должна арендовать школа, чтобы минимизировать транспортные расходы? Чему равны минимальные транспортные расходы? *36. Логистика. Вернемся к рассмотрению задачи 35. Покажите, что оптимальное решение, найденное графически, включает десятичные знаки, если каждый микроавтобус может перевезти семь человек, а количество сопровождающих равно 35. Найдите все допустимые решения с целыми значениями и найдите решение, обеспечивающее минимизацию транспортных расходов. Можно ли получить оптимальное целое решение, округлив оптимальное десятичное решение? Поясните. 37. Капиталовложение, У инвестора есть 60 000 долл, для вложения в фонд депозитных сертификатов и во взаимный фонд. Фонд депозитных сертификатов приносит доход, равный 5%, а взаимный фонд приносит в среднем 9%. Взаимный фонд требует минимального вложения в 10 000 долл., а инвестору нужно, чтобы в фонд депозитных сертификатов было вложено в два раза больше средств, чем во взаимный фонд. Сколько средств следует вложить в фонд депозитных серти-
440 Часть II. Конечная математика фикатов, а сколько — во взаимный, чтобы максимизировать доход? Чему равен максимальный доход? 38. Капиталовложение. У инвестора есть 24 000 долл, для вложения в облигации, имеющие рейтинги ААА и В. Облигации, имеющие рейтинг ААА, приносят в среднем 6% дохода, а облигации В — 10%. Инвестору необходимо, чтобы в облигации, имеющие рейтинг ААА, было вложено по крайней мере в три раза больше денег, чем в облигации с рейтингом В. Сколько денег нужно вложить в облигации каждого типа, чтобы максимизировать доход? Каков максимальный доход? 39. Борьба с загрязнением. Под влиянием новых требований федерального законодательства о загрязнении окружающей среды химический завод заменил старый производственный процесс на новый, более дорогостоящий. При использовании старого процесса в атмосферу выбрасывалось 20 г двуокиси серы и 40 г пылевидных частиц на каждый галлон произведенной химической продукции. В результате использования нового процесса в атмосферу выбрасывается 5 г двуокиси серы и 20 г пылевидных частиц на каждый производимый галлон. Компания получает 60 и 20 центов прибыли на галлон при использовании старого и нового процессов соответственно. а) Сколько галлонов химического вещества нужно произвести с помощью каждого из процессов, чтобы максимизировать дневную прибыль, если правительство позволяет заводу выбрасывать в атмосферу не больше, чем 16 000 г двуокиси серы и 30 000 г пылевидных частиц ежедневно? Какова максимальная дневная прибыль? *б) Как изменится производственный план и максимальная прибыль, если правительство решит ограничить выброс двуокиси серы до 11 500 г ежедневно, а остальные данные останутся неизменными? *в) Как изменится производственный план и максимальная прибыль, если правительство решит ограничить выброс двуокиси серы до 7200 г ежедневно, а остальные показатели останутся неизменными? 40. Наращивание капитала. Сеть предприятий быстрого питания планирует расшириться, открыв несколько новых ресторанов. Сеть включает два типа ресторанов — самообслуживания и полного обслуживания. Строительство ресторана самообслуживания стоит 100000 долл., ему требуется пять работников, а ожидаемая ежегодная прибыль составляет 200000 долл. Строительство ресторана полного обслуживания стоит 150 000 долл., ему требуется 15 работников, а ожидаемая ежегодная прибыль составляет 500000 долл. Предполагаемый капитал для расширения равен 2 400 000 долл. По трудовому соглашению владельцы ресторана могут нанять не более 210 работников, а лицензионные ограничения требуют, чтобы они открыли не более 20 новых ресторанов. Сколько ресторанов каждого типа должна открыть компания, чтобы максимизировать ожидаемый годовой доход? Какую часть своего капитала они используют и сколько работников они наймут?
Глава 5. Линейные неравенства и линейное программирование 441 Биологические науки 41. Удобрение растений. Садовод в своем апельсиновом саду может использовать удобрительные смеси двух типов — марки А и марки Б. Содержимое (в фунтах) азота, фосфора и калия в упаковке каждой марки приведено в таблице. В результате тестирования выяснилось, что для удобрения сада необходимо по крайней мере 1000 фунтов фосфора и 400 фунтов калия. Смесь Марка А Марка Б Азот, фунт/уп. 8 3 Фосфорная кислота, фунт/уп. 4 4 Калий, фунт/уп. 2 1 а) Сколько упаковок каждой смеси нужно использовать, если садовник хочет максимизировать количество азота, которым удобряется сад? б) Сколько упаковок каждой смеси нужно использовать, если садовник хочет минимизировать количество азота, которым удобряется сад? Сколько азота будет внесено? 42. Диета. Специалист по питанию должен составить специальную диету из двух блюд — М и Н. Каждая унция блюда М содержит 30 единиц кальция, 10 единиц железа, 10 единиц витамина А и восемь единиц холестерина. Каждая унция блюда Н содержит 10 единиц кальция, 10 единиц железа, 30 единиц витамина Л и четыре единицы холестерина. Сколько унций каждого блюда нужно использовать, чтобы удовлетворить минимальные потребности и в то же время минимизировать Потребление холестерина, если минимальные суточные потребности составляют 360 единиц кальция, 160 единиц железа и 240 единиц витамина Л? Каково минимальное потребление холестерина? 43. Удобрение растений. Фермер может купить два вида удобрительных смесей — Л и Б. Каждый кубический ярд смеси Л содержит 20 фунтов фосфорных, 30 фунтов азотных И 5 фунтов калийных удобрений. Каждый кубический ярд смеси Б содержит 10 фунтов фосфорных, 30 фунтов азотных и 10 фунтов калийных удобрений. Минимальные ежемесячные потребности составляют 460 фунтов фосфорных, 960 фунтов азотных и 220 фунтов калийных удобрений. Сколько кубических ярдов каждого вида смесей должен смешивать фермер, чтобы удовлетворить ежемесячные потребности при минимальных затратах, если смесь Л стоит 30 долл., а смесь Б — 35 долл, за кубический ярд? Чему равна стоимость такой смеси? 44. Рацион питания. Лаборанта в медицинском исследовательском центре попросили составить рецепты двух пищевых смесей, предназначенных для неких животных, — Л И Б. Каждая унция смеси Л содержит восемь единиц жиров, 16 единиц углеводов и две единицы белков. Каждая унция смеси Б содержит четыре единицы жиров, 32 единицы углеводов и восемь единиц белков. Минимальная суточная потребность составляет 176 единиц жиров, 1024 единицы углеводов и 384 единицы белков. Сколько унций каждой смеси нужно использовать, чтобы удовлетворить минимальные суточные потребности при наименьших затратах,
442 Часть II. Конечная математика если каждая из смесей стоит 5 центов за унцию? Чему равна стоимость этого количества корма? Социальные науки 45. Поведение животных. В эксперименте по изучению поведения животных психолог использует два типа ящиков Скиннера с мышами и крысами. Количество времени (мин.), которое каждая крыса и мышь проводят в каждом ящике ежедневно, приведено в таблице. Какое максимальное количество мышей и крыс может быть использовано в этом эксперименте? Сколько мышей и сколько крыс составляют этот максимум? Время, мин. Мыши Крысы Максимальное время эксперимента, мин. Ящик Скиннера А 10 20 800 Ящик Скиннера Б 20 10 640 46. Социология. Городской совет собрался для того, чтобы обсудить социальные проблемы. Для этого из близлежащего университета были приглашены социологи и ассистенты. Еженедельное распределение платы за их работу дано в таблице. Сколько социологов и ассистентов следует пригласить, чтобы минимизировать затраты на оплату работы и удовлетворить еженедельные потребности в рабочих часах? Социолог Ассистент Минимальное рабочее время в неделю, ч Полевые исследования, ч 10 30 180 Исследовательский центр, ч 30 10 140 Еженедельные затраты, долл. 500 300 5.3. Геометрическое введение в симплекс-метод ■ Стандартная задача максимизации ■ Фиктивные переменные ■ Базисные и небазисные переменные ■ Базисные допустимые решения и симплекс-метод WWW Геометрический метод решения задач линейного программирования дает общий взгляд на предмет и некоторую полезную терминологию. Однако с практической точки зрения этот метод полезен только для задач, содержащих две переменные и относительно мало ограничений. Что происходит в том случае, если необходимо больше искомых переменных? В этих ситуациях используется алгебраический метод, называемый симплекс- методом, который был разработан Джорджем Б. Данцигом (Georg В. Dantzig) в 1947 году. Этот метод предназначен для прикладных задач, содержащих сотни и даже тысячи
Глава 5. Линейные неравенства и линейное программирование 443 переменных и ограничений, поэтому он идеально подходит для использования на компьютерах. Алгебраическая процедура, лежащая в основе симплекс-метода, требует, чтобы ограничения задачи были записаны в виде уравнений, а не неравенств. В связи с этим необходимо ввести новую терминологию. Проиллюстрируем симплекс-метод следующим простым примером. Стандартная задача максимизации Вернемся к задаче о производстве палаток, описанной в примере 5.6 из предыдущего раздела. Напомним математическую модель для задачи. Максимизировать функцию Р = 50xi + 80x2 Целевая функция при условиях х\ 4- 2хч С 32 Ограничения цеха раскройки 3x1 + 4x2 84 Ограничения цеха пошива Xi,X2 0 Условия неотрицательности Здесь переменные xi И Х2 обозначают количество стандартных и экспедиционных палаток соответственно, выпускаемых ежедневно. Отметим, что ограничения задачи содержат неравенства вида “меньше или равно” с положительными постоянными в правой части. Задачи максимизации, удовлетворяющие этому условию, называются стандартными. В этом и следующем разделе наше внимание будет сосредоточено именно на стандартных задачах максимизации. Стандартная задача максимизации в стандартной форме Говорят, что задача линейного программирования является стандартной задачей максимизации в стандартной форме, если ее математическая модель имеет следующий вид. Максимизировать целевую функцию Р = cixi 4- с2х2 4 Н спхп при ограничениях aiXi 4- (I2X2 4 h апхп Ь, где b 0, и условиях Х1, Х2, ..., хп 0. (Замечание. Математическая модель (5.4) является стандартной задачей максимизации. Отметим также, что коэффициентами целевой функции могут быть любые действительные числа.) Задание 5.5. Найдите пример стандартной задачи максимизации, содержащей две переменные и одно ограничение задачи, причем выполняются следующие условия. 1. Область допустимых решений ограничена. 2. Область допустимых решений не ограничена. Возможно ли, чтобы стандартная задача максимизации не имела решения? Поясните. ■
444 Часть II. Конечная математика Фиктивные переменные Чтобы привести задачу линейного программирования к матричному виду, который используется в симплекс-процессе, преобразуем неравенства ограничений задачи в систему линейных уравнений, используя простой аппарат, который называется фиктивной переменной. В частности, чтобы преобразовать систему неравенств ограничений задачи из модели (5.4) Х\ + 2x2 32, Ограничения цеха раскройки 3^1 + 4x2 С 84, Ограничения цеха пошива в систему уравнений, прибавим к левым частям неравенств (5.5) переменные <$1 и $2- В результате получим следующую систему уравнений. X! + 2х2 + «1 = 32, (5 .о) 3xi + 4x2 “F ^2 = 84. Переменные si и $2 называются фиктивными потому, что каждая из них составляет разность (вычисленную фиктивно) между левой и правой частями неравенства в системе (5.5). Например, если производится 20 стандартных (xi = 20) и пять экспедиционных палаток (х2 = 5), то количество рабочих часов, затраченных в цехе раскройки, равно 20 + 2 • 5 = 30, что оставляет неиспользованными два рабочих часа из 32 имеющихся. Таким образом, величина «1 должна равняться 2. Отметим, что если переменные xi и Х2 удовлетворяют системе неравенств (5.5), то фиктивные переменные $i и $2 должны быть неотрицательными. Базисные и небазисные переменные Отметим, что система (5.6) имеет бесконечно много решений. Чтобы убедиться в этом, достаточно решить ее относительно $1 и $2 в терминах xi и Х2, а затем присвоить переменным xi и Х2 произвольные значения. Определенные решения системы (5.6), называемые базисными, связаны с точками пересечения (расширенных) граничных линий области допустимых решений, изображенной на рис. 5.16. xi Рис. 5.16. Базисные решения Как определяются базисные решения системы (5.6)? Эта система содержит четыре переменные и два уравнения. Разделим четыре переменные на две группы, называемые базисными и небазисными переменными. Базисные переменные отбираются произвольно.
Глава 5. Линейные неравенства и линейное программирование 445 Следует лишь учитывать, что их количество должно быть равным количеству уравнений в системе. Оставшиеся переменные объявляются небазисными. Поскольку система (5.6) состоит из двух уравнений, в качестве базисных можно выбрать любые две из четырех переменных. Тогда остальные две являются небазисными. Решение, которое вычисляется, если положить две небазисные переменные равными нулю и решить систему относительно двух базисных переменных, называется базисным. (Заметим, что, положив две переменные в системе (5.6) равными нулю, мы получим систему, состоящую из двух уравнений с двумя переменными, которые могут иметь (см. главу 4) одно решение, бесконечно много решений или не иметь решений.) Пример 5.9 (Базисные решения и допустимая область). 1. Найдите два базисных решения системы (5.6), сначала выбирая в качестве базисных переменных si и 82, а затем — Х2 и «1. 2. Свяжите каждое базисное решение, найденное в ходе решения задачи 1, с точкой пересечения (расширенных) граничных линий области допустимых решений (см. рис. 5.16). 3. Определите, какие из точек пересечения, найденных при решении задачи 2, принадлежат области допустимых значений. Решение. 1. Если в качестве базисных переменных выбраны переменные $i и «2, то переменные х\ и Х2 являются иебазисными. Базисное решение вычисляется, если положить небазисные переменные равными нулю и решить систему уравнений относительно базисных переменных. Если х\ = 0 и Х2 = 0, то система (5.6) принимает следующий вид. si = 32, 82 = 84. О О = 32 + 82 = 84 Базисным решением этой системы является следующий набор чисел. Xi ~ О, Х2 = 0, 81 = 32, 82 = 84. (5-7) Если выбрать в качестве базисных переменные Х2 и si, то переменные х± и 82 станут небазисными. Полагая небазисные переменные равными нулю, приведем систему (5.6) к следующему виду. 2x2 4“ 81 = 21, 4j?2 = 84. О 4“ 2®2 4~ 81 3®i 4“ 4®2 О ~1= 32 + 82 — 84 Решая ее, получим числа Х2 = 21 и 81 = —10. Базисным решением является следующий набор чисел. Х1 = 0, Х2 — 21, 81 = —10, 82 = 0. (5-8)
446 Часть II. Конечная математика 2. Решение (5.7) является базисным, поскольку числа xi = 0 и Х2 = 0 соответствуют началу координат 0(0; 0) (см. рис. 5.16). Оно является пересечением граничных линий xi = 0 и х2 = 0. Решение (5.8) также является базисным, поскольку точка Ху = 0 и Х2 = 21 является точкой пересечения 0(0; 21) граничных линий х± = О и 3^1 -I- 4^2 = 84. 3. Точка пересечения 0(0; 0) принадлежит области допустимых решений. Следо¬ вательно, естественно назвать соответствующее базисное решение допустимым базисным решением (basic feasible solution). Точка пересечения 0(0; 21) не принадлежит допустимой области. Таким образом, соответствующее базисное решение не является допустимым. ■ Упражнение 5.9. 1. Найдите два базисных решения системы (5.6), сначала выбирая в качестве базисных переменные xi и $i, а затем переменные х± и 2. Свяжите все базисные решения, найденные при решении задачи 1, с точкой пересечения (расширенных) граничных линий допустимой области на рис. 5.16. 3. Определите, какие из точек пересечения, найденные при решении задачи 2, принадлежат области допустимых решений. ■ Систематически продвигаясь в расчетах, как в примере 5.9 и упражнении 5.9, можно получить все базисные решения системы (5.6). Результаты вычислений подытожены в табл. 5.2, которая также включает геометрические интерпретации базисных решений соответственно рис. 5.16. Эти интерпретации продемонстрированы на рис. 5.17. Стоит внимательно изучить табл. 5.2 и рис. 5.17. (Чтобы быть уверенным, что в табл. 5.2 перечислены все возможные базисные решения, постройте таблицу с учетом нулевых значений небазисных переменных.) WWW Таблица 5.2. Базисные решения Базисные решения Точка пересечения Пересекающиеся граничные линии Допустимость Ж1 Х2 «1 «2 0 0 32 84 0(0; 0) xi = 0, х2 = 0 Да 0 16 0 20 А(0; 16) = 0, xi + 2х2 = 32 Да 0 21 -10 0 0(0; 21) xi = 0, 3xi -1- 4х2 = 84 Нет 32 0 0 -12 0(32; 0) х2 = 0, Xi + 2x2 = 32 Нет 28 0 4 0 0(28; 0) х2 = 0, 3xi 4х2 = 84 Да 20 6 0 0 В(20;6) xi + 2х2 = 32, 3xi -1- 4х2 = 84 Да
Глава 5. Линейные неравенства и линейное программирование 447 Рис. 5.17. Базисные допустимые решения: О, Л, В, С. Базисные решения, которые не являются допустимыми: D, Е Важные выводы 1. Из табл. 5.2 следует, что базисное решение, которое не является допустимым, содержит по крайней мере одно отрицательное значение, а базисное допустимое решение не содержит отрицательных значений. Иначе говоря, можно определить, является ли базисное решение допустимым, просто изучая знаки переменных в решении. 2. Из табл. 5.2 и рйс. 5.17 следует, что базисные допустимые решения связаны с угловыми точками допустимой области, которые содержат оптимальное решение исходной задачи линейного программирования. Прежде чем двигаться дальше, формализуем определения, на которые делались ссылки в приведенных выше рассуждениях, чтобы их можно было применить к стандартным задачам максимизаций без каких-либо ссылок на геометрические формы. Базисные переменные и небазисные переменные; базисные решения и допустимые базисные решения Предположим, что система линейных уравнений связана с задачей линейного программирования (в такой системе количество переменных всегда больше, чем количество уравнений). Переменные делятся на две взаимоисключающие группы: базисные (их должно быть столько же, сколько уравнений) и небазисные. Решение, которое вычисляется, если положить небазисные переменные равными нулю и решить систему уравнений относительно базисных переменных, называется базисным. Если базисное решение не имеет отрицательных значений, оно является базисным допустимым решением. Пример 5.10 (Базисные переменные и базисные решения). Рассмотрим систему трех уравнений относительно восьми переменных (искомых и фиктивных), связанную со стандартной задачей максимизации. 1. Сколько базисных и небазисных переменных связано с системой? 2. Положим небазйсные переменные равными нулю. Сколько уравнений и неизвестных будет входить в полученную систему?
448 Часть II. Конечная математика 3. Является ли базисное решение допустимым, если оно состоит только из неотрицательных чисел? Решение. 1. Поскольку система состоит из трех уравнений, должны быть три базисные переменные и пять небазисных переменных. 2. Три уравнения с тремя переменными. 3. Да, является допустимым. ■ Упражнение 5.10. Рассмотрим систему из пяти уравнений относительно 11 переменных (искомых и фиктивных), связанную со стандартной задачей максимизации. 1. Сколько базисных и небазисных переменных связано с системой? 2. Положим небазисные переменные равными нулю. Сколько линейных уравнений и неизвестных будет входить в полученную систему? 3. Является ли допустимым базисное решение, если оно имеет один или несколько отрицательных элементов? ■ Базисные допустимые решения и симплекс-метод Сформулируем без доказательства следующую важную теорему, эквивалентную основной теореме линейного программирования (теорема 5.2 предыдущего раздела). Теорема 5.4 (Основная теорема линейного программирования (вариант 2)). Если оптимальное значение целевой функции в задаче линейного программирования существует, то оно должно достигаться на одном (или нескольких) базисных допустимых решениях, и Теперь становится понятным, почему изучение базисных и небазисных переменных, а также базисных решений и базисных допустимых решений так важно — эти понятия являются центральными в процессе вычисления оптимальных решений задач линейного программирования. Задание 5.6. Как использовать таблицу базисных решений для поиска оптимального решения стандартной задачи максимизации, если известно, что она имеет такое решение? ■ Итак, нами сделан первый шаг к описанию общего метода решения задач линейного программирования, содержащих произвольное количество переменных и ограничений задачи. Мы описали метод определения всех угловых точек (базисных допустимых решений) области допустимых решений без изображения графика. Это существенный шаг вперед, позволяющий рассматривать задачи с большим количеством переменных. К сожалению, по мере увеличения числа переменных и ограничений количество угловых точек резко возрастает. В реальных задачах поиск всех угловых точек для нахождения оптимального решения, как правило, не применяется. Таким образом, следующий шаг состоит в разработке метода определения оптимального решения без поиска всех угловых точек. Эта процедура называется симплекс-методом.
Глава 5. Линейные неравенства и линейное программирование 449 Симплекс-метод, использующий специальную матрицу и операции со строками, автоматически передвигается от одного базисного допустимого решения к другому (т.е. от одной точки области Допустимых решений к другой), каждый раз приближаясь к оптимальному решению (если оно существует) до тех пор, пока оно не будет достигнуто. Замечательным свойством симплекс-метода является то, что в больших задачах линейного программирования Он позволяет находить оптимальное решение (если оно существует) проверкой относительно небольшого количества базисных допустимых решений (угловых точек). Ответы к упражнениям 5.9. 1) Базисное решение, соответствующее базисным переменным х\ и $х: х\ = 28, Х2 = 0, $i = 4, 32 ~ 0. Базисное решение, соответствующее базисным переменным Xi И «2- = 32, Х2 = 0, «1 = 0, 32 = —12. 2) Первое базисное решение соответствует точке С(28; 0), которая является точкой пересечения граничных линий Х2 = 0 и Зжх + 4^2 = 84. Второе базисное решение соответствует точке 2*7(32; 0), которая является точкой пересечения граничных линий Х2 = 0 и х\ + 2я2 = 32. 3) Точка С7(28; 0) принадлежит области допустимых значений (следовательно, соответствующее базисное решение является базисным допустимым решением); точка Е(32; 0) не принадлежит области допустимых значений (следовательно, соответствующее базисное решение не является допустимым). 5.10. 1) Пять базисных переменных и шесть небазисных переменных. 2) Пять уравнений и пять переменных. 3) Не является допустимым. Практикум 5.3 А 1. Рассмотрите взаимосвязь между стандартной задачей максимизации с двумя ограничениями и тремя переменными и соответствующей системой уравнений. В частности, найдите следующие величины и объясните, как их вычислить. а) Количество свободных переменных, которые следует ввести, чтобы получить систему уравнений, описывающих ограничения задачи. б) Количество базисных и небазисных переменных, связанных с системой. в) Количество линейных уравнений и количество переменных в системе, возникающей, если положить небазисные переменные равными нулю. * 2. Повторите задачу 1 для стандартной задачи максимизации с тремя ограничени¬ ями и четырьмя переменными. * 3. Рассмотрите взаимосвязь между стандартной задачей максимизации и соответ¬ ствующей системой уравнений, описывающий ограничения, если в систему входят девять переменных, в том числе пять фиктивных. В частности, найдите каждую из следующих величин и объясните, как их вычислить.
450 Часть II. Конечная математика а) Количество уравнений ограничений в системе. б) Количество переменных в системе. в) Количество базисных и небазисных переменных, связанных с системой. г) Количество линейных уравнений и количество переменных в системе, которая получена, если положить небазисные переменные равными 0. * 4. Повторите решение задачи 3, если система уравнений, описывающая ограничения, имеет 10 переменных, в том числе четыре фиктивных. 5. Ниже в таблице приведены все базисные решения системы 2xi 4" Зз?2 4~ 51 — 24, 4xi -И Зхз 4- «2 = 36. Для каждого базисного решения определите небазисные и базисные переменные. Затем классифицируйте каждое базисное решение как допустимое или недопустимое. Х1 х2 31 «2 1. 0 0 24 36 2. 0 8 0 12 3. 0 12 -12 0 4. 12 0 0 -12 5. 9 0 6 0 6. 6 4 0 0 6. Повторите решение задачи 5 для системы 2xi 4- Х2 4- 51 = 30, 4- 5x2 4- 52 = 60, базисные решения которой даны в следующей таблице: Х2 31 32 1. 0 0 30 60 2. 0 30 0 -90 3. 0 12 18 0 4. 15 0 0 45 5. 60 0 -90 0 6. 10 10 0 0 7. Ниже в таблице перечислены все возможные выборы небазисных переменных для системы 2xi 4- Х2 4- 51 = 50, xi 4- 2х2 4- 52 = 40.
Глава 5. Линейные неравенства и линейное программирование 451 В каждом случае найдите значения базисных переменных и определите, является ли базисное решение допустимым. «2 31 32 1. 0 0 ? ? 2. 0 ? 0 ? 3. 0 ? ? 0 4. ? 0 0 ? 5. ? 0 ? 0 6. ? ? 0 0 8. Повторите задачу 7 для системы Х\ + 2х2 + «1 = 12, 3^1 -I- 2х2 4- 52 = 24. Б Изобразите графически системы неравенств в задачах 9-12. Введя свободные переменные, преобразуйте системы неравенств в системы уравнений и найдите все базисные решения системы. Постройте таблицу, содержащую все базисные решения, соответствующие точкам на графике, и определите, является ли базисное решение допустимым. (Вам нет необходимости перечислять пересекающиеся линии.) 9. Xi + Х2 16, 2xi + Х2 20, Xi, Х2 > 0. 11. 2X1 + 22, Х1 + Х2 12, xi 4- 2x2 20, Xi, Х2 0. 10. 5xi + #2 35, 4xi + #2 32, Xi, Х2 0. 12. 4xi 4- #2 < 28, 2xi + Х2 < 16, #1 + %2 С 13, Xi, Х2 > 0. 5.4. Симплекс-метод: максимизация с ограничениями вида “меньше или равно” ■ Исходная система ■ Симплекс-таблИца ■ Поиск опорного плана ■ Геометрическая интерпретация симплекс-метода ■ Резюме ■ Решение практических задач Теперь мы готовы описать симплекс-метод для решения стандартной задачи максимизации. Отдельные детали в описании метода изменяются от одной книги к другой, однако структура процесса сохраняется.
452 Часть II. Конечная математика Как отмечалось в предыдущем разделе, симплекс-метод наиболее удобен при использовании на компьютерах. Однако для того, чтобы отточить мастерство создания моделей и интерпретации результатов, необходимо научиться формулировать и решать вручную несколько достаточно простых задач линейного программирования. Именно этим мы и займемся в следующих разделах. Для решения задач линейного программирования можно применять как специальное программное обеспечение, так и электронные таблицы, например, Excel. Исходная система Введем понятия и процедуры, содержащиеся в симплекс-методе, на примере задачи производства палаток. Для удобства напомним постановку задачи в стандартной форме. Максимизировать функцию Р = 50xi + 80^2 Целевая функция при условиях xi + 2x2 32, 3xi + 4x2 84, Х1,Хг 0. Первое ограничение задачи Второе ограничение задачи Условия неотрицательности (5.9) Вводя фиктивные переменные .$1 и S2, преобразуем неравенства ограничений задачи (5.9) в следующую систему уравнений. Xi -|- 2x2 4" $i = 32, 3xi + 4х2 + «2 = 84, (5.Ю) Х1, ^2, $1, «2 > 0 Поскольку базисное решение системы (5.10) не является допустимым, если оно содержит какую-либо отрицательную величину, необходимо также включить условия неотрицательности как для исходных переменных xi и хг, так и для фиктивных переменных si и $2- Из рассуждений последнего раздела известно, что среди бесконечного множества решений системы (5.10) оптимальное решение находится среди базисных допустимых решений, соответствующих угловым точкам допустимой области. Добавим к системе (5.10) уравнение целевой функции Р = 50xi + 80x2 в виде —50x1 — 80x2 + Р = 0. Таким образом, мы получим исходную систему. Xi + 2x2 4- $i — 32, 3xi 4- 4х2 4- $2 = 84, —50x1 — 80x2 4- Р = 0, Xi, Х2, Si, S2 0. (5.И) Когда к системе (5.10) добавляется уравнение целевой функции, необходимо слегка модифицировать данное ранее определение базисного допустимого решения применительно к исходной системе (5.11).
Глава 5. Линейные неравенства и линейное программирование 453 Базисные решения и базисные допустимые решения для исходных систем 1. Переменная Р, соответствующая значениям целевой функции, всегда выбирается в качестве базисной переменной и никогда не выбирается в качестве небазисной. 2. Отметим, что базисное решение системы (5.11) является также базисным решением системы (5.10) после удаления переменной Р. 3. Если базисное решение системы (5.11) является базисным допустимым решением системы (5.10) после удаления переменной Р, то базисное решение системы (5.11) называется базисным допустимым решением системы (5.11). 4. Базисное допустимое решение системы (5.11) может содержать отрицательное число, но только в случае, если оно является значением переменной Р. Эти изменения приводят к небольшой корректировке второй версии основной теоремы линейного программирования (см. раздел 5.3). Теорема 5.5 (Основная теорема линейного программирования (вариант 3)). Если оптимальное значение целевой функции в задаче линейного программирования существует, то это значение должно достигаться в одном (или нескольких) базисных допустимых решениях исходной системы. в Итак, начнем изучать симплекс-процесс с базисным допустимым решением исходной системы (5.11), которое будем рассматривать как исходное базисное допустимое решение. Исходным базисным допустимым решением, которое легко найти, является решение, соответствующее началу координат. Поскольку система (5.11) состоит из трех уравнений относительно пяти переменных, она имеет три базисные и две небазисные переменные. Изучая систему, можно увидеть, что х\ и Х2 появляются во всех уравнениях, a si, «2 и Р появляются только однажды и в разных уравнениях. Базисное решение может быть найдено путем перебора переменных $i, «2 и Р в качестве базисных переменных (напомним, Р всегда выбирается в качестве базисной переменной), в то время как небазисные переменные х\ и Х2 полагаются равными 0. Полагая переменные xi и Х2 равными 0 и решая систему (5.11) относительно базисных переменных, получим базисное решение. xi =0, Х2 = 0, «1 = 32, S2 = 84, Р = 0. Это базисное решение является допустимым, поскольку ни одна из переменных (исключая Р) не является отрицательной. Таким образом, первоначальное базисное решение найдено. Теперь очевидно, почему мы добавили уравнение целевой функции к системе (5.10): базисное допустимое решение системы (5.11) содержит не только базисное допустимое решение системы (5.10), но и значение Р, соответствующее этому решению Найденное исходное допустимое базисное решение связано с началом координат. Конечно, мы не производили никаких палаток и не получали прибыли, поэтому Р = 0 долл. Начиная с этого легко Полученного исходного базисного допустимого решения, симплекс- процесс движется через итерации (повторения) к другому базисному допустимому решению, каждый раз повышая прибыль, и процесс продолжается до тех пор, пока не будет достигнута максимальная прибыль. Затем процесс останавливается.
454 Часть II. Конечная математика Симплекс-таблица Чтобы обеспечить поиск оптимального решения, обратимся к матричному методу, который рассматривался в главе 4. Первым шагом является запись расширенной матрицы для исходной системы (5.9). Эта матрица называется исходной симплекс-таблицей. Она представляет собой таблицу, составленную из коэффициентов системы (5.11). Ж1 Х2 51 52 р «1 1 2 1 0 0 32 ‘ 52 3 4 0 1 0 84 Исходная симплекс-таблица Р -50 -80 0 0 1 0 (5.12) В матрице (5.12) строка ниже пунктирной линии всегда соответствует целевой функции. Каждая из выбранных выше базисных переменных $i, $2 и Р также записывается слева от матрицы, так что элементы в пересечении их строк и столбцов не равны нулю. Например, поместим базисную переменную si слева так, чтобы элемент пересечения $1 строки и 51 столбца был равен единице, а не нулю. Аналогично располагается переменная §2- Переменная Р целевой функции всегда помещается внизу. Такое размещение базисных переменных позволяет считывать некоторые допустимые базисные решения прямо из матрицы. Если xi = 0 и Х2 = 0, то значения базисных переменных, расположенных слева от матрицы (5.12), т.е. числа 32, 84 и 0 отделяются вертикальной линией. Изучая матрицу (5.12), соответствующую выбору переменных si, $2 и Р в качестве базисных, легко увидеть, что каждая базисная переменная располагается выше столбца, все элементы которого равны нулю и только один — единице, и что никакие два таких столбца не содержат единицы в одной и той же строке. Эти наблюдения приводят к формализации процесса выбора базисных и небазисных переменных, который является важной частью симплекс-метода. Выбор базисных и небазисных переменных в симплекс-таблице Этап 1. Определим количество базисных и небазисных переменных. Это число не изменяется в течение симплекс-процесса. Этап 2. Выбор базисных переменных. Переменная может быть выбрана в качестве базисной переменной, только если она соответствует столбцу, который содержит лишь один ненулевой элемент (обычно единицу), а ненулевой элемент в столбце не находится в той же строке, что и ненулевой элемент в столбце, соответствующем другой базисной переменной. (В этой процедуре обычно в качестве базисной переменной выбирается переменная Р, поскольку столбец Р никогда не изменяется во время симплекс-процесса.) Этап 3. Выбор небазисных переменных. После выбора базисных переменных оставшиеся переменные считаются небазисными. (Столбцы таблицы, соответствующие небазисным переменным, обычно содержат несколько ненулевых элементов.) Выбор переменных $1, 52 и Р в качестве базисных, а переменных х\ и х% — в качестве небазисных полностью соответствует описанному выше соглашению о выборе базисных и небазисных переменных для симплекс-метода.
Глава 5. Линейные неравенства и линейное программирование 455 Поиск опорного плана Заменим одну из базисных переменных $1 или «2 (но не Р) небазисной переменной xi или Х2- Это приведет к увеличению прибыли. Для небазисной переменной, заменяющей базисную, необходимо произвести соответствующие операции над строками матрицы так, чтобы вновь выбранная базисная переменная содержала один ненулевой элемент в своем столбце. В результате Этого процесса старая базисная переменная обычно будет получать дополнительный ненулевой элемент в своем столбце, становясь небазисной. Какую небазисную переменную следует превратить в базисную? Есть смысл выбрать небазисную переменную, которая будет максимально увеличивать прибыль при ее изменении на единицу. Рассматривая целевую функцию Р = 50^1 4- 80x2, увидим, что если xi остается небазисной переменной (положим ее равной нулю), а Х2 становится новой базисной переменной, то Р = 50 • 0 + 80x2, и на каждую единицу увеличения переменной Х2 значение Р будет увеличиваться на 80 долл. Если Х2 остается небазисной переменной, a xi становится новой базисной переменной, то на каждую единицу увеличения переменной х\ значение Р будет возрастать только на 50 долл. По этой причине в качестве новой базисной переменной выберем небазисную переменную #2- Назовем ее включаемой переменной. (Базисная переменная, покидающая множество базисных переменных и становящаяся небазисной, называется исключаемой переменной.) Назовем столбец, Соответствующий включаемой переменной, ведущим. Рассматривая нижнюю строку в матрице (5.12), т.е. строку целевой функции, расположенную ниже пунктирной линии, видим, что ведущий столбец расположен слева от столбца Р и содержит наибольший По абсолютной величине отрицательный нижний элемент. Вообще говоря, наибольший по абсолютной величине отрицательный элемент в нижней строке, расположенный слева от столбца Р, определяет переменную, обеспечивающую наибольшее возрастание Р при увеличении на единицу. По этой причине назовем элементы, записанные в нижней строке матрицы слева от столбца Р, индикаторами. Проиллюстрируем индикаторы, ведущий столбец, включаемую переменную и исход- ное дс 52 Р ►пустимо Включае Xi 1 3 е баз» мая пс 1 Х2 2 4 юное peinei временная *1 32 Р 1 0 0 0 1 0 1ие. 32 ' 84 Исходная симплекс-таблица Индикаторы обозначены полужирным шрифтом —50 -80 0 0 1 0 Ведущии столбец Xi = 0, Х2 = 0, «1 = 32, 82 = 84, Р = 0 Исходное допустимое базисное решение Теперь, когда небазисная переменная Х2 стала включаемой (т.е. небазисная переменная стала базисной), Какую из двух базисных переменных (si или «г) следует выбрать
456 Часть II. Конечная математика в качестве исключаемой (какая базисная переменная должна стать небазисной)? Выше показано, что для значения xi — 0 каждое единичное увеличение входящей переменной Х2 приводит к увеличению значения Р на 80 долл. Можно ли увеличивать переменную Х2 неограниченно? Нет! Предел налагается условиями неотрицательности переменных $1 и S2- (Напомним, что, если какая-то базисная переменная, кроме переменной Р, становится отрицательной, у нас больше нет допустимого решения.) Поэтому перефразируем вопрос и спросим: “На сколько может увеличиваться значение х2, если х± = 0, не опасаясь того, что переменные $1 или з2 станут отрицательными?”. Чтобы увидеть, на сколько может увеличиваться Х2, обратимся к матрице (5.13) или системе (5.11) и запишем два уравнения, задающие ограничения задачи при условии, что Xi = 0. 2x2 + si = 32, 4^2 + s2 = 84. Решая эту систему относительно переменных «1 и s2, получим выражения «1 = 32 — 2ж2, s2 = 84 - 4ж2. Для того чтобы переменные si и s2 были неотрицательными, переменную ж2 следует выбирать так, чтобы числа 32 — 2ж2 и 84 — 4ж2 были неотрицательными. 32 - 2х2 > 0, —2x2 —32, 84 — 4х2 > 0, -4и2 -84, Для того чтобы удовлетворялись оба неравенства, переменная х2 должна быть меньше или равна наименьшей из величин, т.е. 16. Таким образом, переменная Х2 может увеличиваться до 16, не вызывая того, что либо «1, либо s2 станут отрицательными. Теперь рассмотрим, как найти эти величины (16 и 21) по симплекс-таблице. Включаемая переменная 1 3 2 4 $1 82 Р 32 84 = 16 (наименьшая величина) (5.14) 1 0 0 0 1 0 0 0 1 Ведущий столбец Используя матрицу (5.14) несложно определить величину, на которую может увеличиваться включаемая переменная, выбирая наименьший из коэффициентов, полученных делением каждого элемента последнего столбца, расположенного выше пунктирной линии, на соответствующий положительный элемент ведущего столбца. Строка с наименьшим коэффициентом называется ведущей строкой. В данном случае включаемой переменной является переменная $1, а переменные ж2 и «1 меняются ролями. Элемент, стоящий на пересечении ведущего столбца и ведущей строки, называется ведущим. Он обведен кружком. Поскольку отрицательный или нулевой элемент в ведущем столбце не налагает ограничений на прирост включаемой переменной, нет необходимости вычислять коэффициенты для отрицательных или нулевых значений, расположенных в ведущем столбце.
Глава 5. Линейные неравенства и линейное программирование 457 Отрицательный или нулевой элементы никогда не выбираются в качестве ведущих. Включаемая переменная Ведущий элемент —ч. | Nfri S1 52 Р Исключаемая переменная —* •| 1 1 0 0 32 F Ведущая строка 3 4 0 1 0 84 (5.15) Р -50 -80 0' 0 1 1 0 Ведущий столбец Выбор ведущего элемента Этап 1. Найдем наибольший по абсолютной величине отрицательный индикатор в нижней строке Таблицы слева от столбца Р (отрицательное число с наибольшей абсолютной величиной). Столбец, содержащий этот элемент, является ведущим. Если существует еще один элемент, равный наименьшему отрицательному, выбираем любой из них. Этап 2. Разделим каждый положительный элемент в ведущем столбце выше пунктирной линии на соответствующий элемент в последнем столбце. Ведущей строкой является строка, соответствующая наименьшему полученному коэффициенту. Если существует несколько одинаковых наименьших коэффициентов, выбираем любой из них. Если ведущий столбец не имеет положительных элементов выше пунктирной линии, решение не существует, и процесс останавливается. Этап 3. Ведущим является элемент, стоящий на пересечении ведущих столбца и строки. (Замечание. Ведущий элемент всегда положителен и никогда не стоит в нижней строке.) (Необходимо помнить: включаемая переменная стоит в основном столбце вверху, а исключаемая переменная — в ведущей строке слева от ведущего столбца.) Для того чтобы классифицировать переменную х% как базисную, произведем операции со строками матрицы (5.15) так, чтобы ведущий элемент превратился в единицу, а все остальные элементы столбца — в нуль. Эта процедура преобразования небазисной переменной в базисную называется поиском опорного плана. www Поиск опорного плана Поиск опорного плана состоит в выполнении следующих операций над строками. Этап 1. Поделим ведущую строку на ведущий элемент. Тем самым мы преобразуем ведущий элемент в единицу. (Если ведущий элемент уже равен единице, пропускаем этот этап.) Этап 2. Сложим ведущую строку, поделенную на ведущий элемент, с другими строками таблицы. В результате все остальные элементы ведущего столбца станут равными нулю. (Замечание. В процессе поиска опорного плана строки не меняются. Единственный способ, с помощью которого ведущий элемент может быть преобразован в единицу (если он не был равен ей), состоит в том, что ведущая строка делится на ведущий элемент.) Процесс поиска опорного плана завершается получением следующих результатов.
458 Часть II. Конечная математика 1. (Включаемая) небазисная переменная превращается в базисную. 2. (Исключаемая) базисная переменная превращается в небазисную. 3. Значение целевой функции увеличивается или (в некоторых случаях) остается тем же. Теперь выполним операцию поиска опорного плана для матрицы (5.15). (Для ускорения процесса не будем повторно указывать переменные после выполнения первой операции над строками и будем использовать термины “вход” и “выход” вместо “включаемая переменная” и “исключаемая переменная” соответственно.) Вход Х1 Х2 «1 52 р Выход — 51 1 1 0 0 32 ' —► -Ri $2 3 4 0 1 0 84 Р -50 -80 0 0 1 0 1 2 Ф 1 2 0 0 16 ' 3 4 0 1 0 84 —4Д1 4" R2 —► R2 _ “50 -80 0 0 1 0 _ 8ОЛ1 4" R& —► R3 1 2 1 1 2 0 0 16 ' 1 0 —2 1 0 20 -10 0 40 0 1 1280 Операция поиска опорного плана завершена, и теперь необходимо вставить соответствующие переменные в новую таблицу. Поскольку переменная х2 заменила переменную si, базисными переменными теперь являются переменные х2, s2 и Р. Отметим, что это полностью согласуется с процедурой выбора базисных переменных. Запишем новое допустимое базисное решение, полагая небазисные переменные х^ и si равными нулю и решая систему относительно базисных переменных. (Напомним, что значениями базисных переменных, записанных слева, являются соответствующие числа, записанные справа от вертикальной линии. Чтобы увидеть это, подставим х\ = 0 и si = 0 в соответствующую систему, указанную после симплекс-таблицы.) Х2 1 1 | 0 0 16 52 10-210 20 Р -10 0 40 0 1 1280 + Х2 4“ 1^1 — 16 х± — 231 -|- 32 — 20 —lOxi 4“ 4031 4" Р = 1280 xi — 0, х2 = 16, 31 = 0, 32 = 20, Р = 1280 долл. Прибыль, равная 1280 долл., является существенным улучшением в сравнении с ее отсутствием. Однако это не предел. Прибыль Р можно повышать еще, поскольку в нижней строке есть отрицательный индикатор. Для того чтобы понять причину этого явления, запишем целевую функцию. -10х14-40з1+Р = 1280, или Р = 10X1 - 4031 4-1280.
Глава 5. Линейные неравенства и линейное программирование 459 Если «1 остается небазисной переменной (положим ее равной нулю), a xi становится новой базисной переменной, то Р = 10^1 - 40 • 0 + 1280 = 10^1 + 1280. Следовательно, на каждую единицу прироста переменной значение Р будет увеличиваться на 10 долл. Теперь проведем Поиск опорного плана еще раз (т.е. повторим приведенную выше последовательность шагов), для другого ведущего элемента. Вход #1 Х2 31 32 Р Л?2 1 1 | 0 0 16 ' Выход —* 32 ©0-210 20 р -10 0 40 0 1 1280 ^=32 = 20 На этот раз процедура повторяется для обведенной кружком единицы. Иначе говоря, теперь поиск опорного плана проводится для ведущего элемента, представленного единицей. В этом случае нет необходимости выполнять первый этап процедуры, поэтому перейдем сразу ко второму этапу и получим нули выше и ниже ведущего элемента. Вход Х2 «1 з2 р Х2 1 2 1 1 2 0 0 16 ■ Выход — 32 ф 0 —2 1 0 20 р -10 0 40 0 1 1280 _ 0 1 3 2 _1 2 0 6 ' 1 0 —2 1 0 20 0 0 20 10 1 1480 — 4- Ri —► Ri ЮЛэ 4- R3 —► R3 Поскольку в нижней строке больше не существует отрицательных индикаторов, процесс завершен. Вставим соответствующие переменные в последнюю таблицу и запишем соответствующее допустимое базисное решение. Базисными теперь являются переменные xi, Х2 и Р. Следовательно, чтобы получить соответствующее допустимое базисное решение, необходимо положить небазисные переменные 31 и «2 равными нулю и решить систему относительно базисных переменных. Х1 а?2 81 82 Р Х2 ‘01 | 4 о 6 ' XI 10-2 10 20 Р 0 0 20 10 1 1480 xi = 20, х2 = 6, si = 0, з2 = 0, Р = 1480. Чтобы убедиться, что мы получили точку максимума, перепишем целевую функцию. 20«1 4“ 10з2 4- Р = 1480, P = 1480-20si-10s2.
460 Часть II. Конечная математика Поскольку переменные $1 и не могут быть отрицательными, любое увеличение каждой из них будет уменьшать прибыль. Наконец, возвращаясь к нашей исходной задаче, приходим к выводу, что производственный план, предусматривающий выпуск 20 стандартных и 6 экспедиционных палаток, будет обеспечивать максимальную ежедневную прибыль, равную 1480 долл. Этот результат совпадает с геометрическим решением, полученным в разделе 5.2. Тот факт, что обе фиктивные переменные равны нулю, означает, что при таком производственном плане завод будет работать на полную мощность. Геометрическая интерпретация симплекс-метода Теперь можно дать геометрическую интерпретацию симплекс-метода, используя понятие области допустимых значений. В табл. 5.3 перечислены три только что найденных базисных допустимых решения (в порядке, в котором они были вычислены). В таблицу включены также соответствующие угловые точки области допустимых значений, изображенной на рис. 5.18. Таблица 5.3. Базисное допустимое решение ®2 si 32 Р, долл. Угловая точка 0 0 32 84 0 0(0; 0) 0 16 0 20 1280 А(0; 16) Рис. 5.18. Область допустимых решений Анализ табл. 5.3 и рис. 5.18 показывает, что симплекс-процесс, начавшийся в начале координат, движется к соседней угловой точке А(0; 16) и затем к оптимальному решению В(20; 6) в следующей угловой точке. Это типично для симплекс-процесса. Резюме Рассмотрим схему алгоритма симплекс-метода, изображенную на рис. 5.19.
Глава 5. Линейные неравенства и линейное программирование 461 Рис. 5.19. Алгоритм симплекс-метода решения стандартных задач максимизации. (Ограничения задачи имеют форму “меньше или равно” и содержат неотрицательные постоянные в правой части. Коэффициентами целевой функции могут быть любые действительные числа) Пример 5.11 (Использование симплекс-метода). Решите следующую задачу линейного программирования, используя симплекс-метод. Максимизировать функцию Р = 10xi + 5x2 при условиях 4x1 + #2 28, 2xi + 3x2 < 24, Х1,Х2 > 0.
462 Часть II. Конечная математика Решение. Введем фиктивные переменные si и S2 и запишем исходную систему: 4x1 4" #2 4" ^1 = 28, 2xi + 3x2 4-52 = 24, -lOxi - 5x2 4- Р = 0, Х1, ^2, 51, 52 0. Запишем симплекс-таблицу и определим первый ведущий элемент, а также включаемые и исключаемые переменные. Вход г Х2 «1 52 р Выход —>■ S1 1 0 0 0 28 ¥ = 7 52 2 3 0 1 0 24 =12 Р -10 -5 0 0 1 0 Выполним поиска опорного плана. Вход Х1 Х2 51 «2 р Выход —> $1 (4) 1 0 0 0 28 —► R1 52 2 3 0 1 0 24 Р -ю -5 0 0 1 0 ф 0,25 0,25 0 0 7 ' 2 3 0 1 0 24 —2Ri -|- R2 — Я2 . “10 -5 0 0 1 0 1ОК1 + Яз — Яз Х1 1 0,25 0,25 0 0 7 ‘ ~52 0 2,5 -0,5 1 0 10 Р 0 -2,5 2,5 0 1 70 Поскольку в последней строке все еще есть отрицательный индикатор, повторим процедуру, находя новый ведущий элемент. Вход Х2 51 52 р Х1 1 0J5 0,25 0 0 7 ‘ Выход —► «2 0 -0,5 1 0 10 р 0 -2,5 2,5 0 1 70 0,26 — 28 то = 4 2,5 * Проведя новый поиск опорного плана, получим следующий результат. Х1 Выход —► S2 р Х1 Вход Х2 81 52 р 1 0,25 0 0 7 0 ..й._ -0,5 1 0 10 2^5 —► R2 0 -2,5 2,5 0 1 70
Глава 5. Линейные неравенства и линейное программирование 463 1 0,25 0,25 0 0 7 ' -0,25Яа + Я1 —► Я1 0 Ф -0,2 0,4 0 4 0 -2,5 2,5 0 1 70 2,5Яа 4“ Яз —► Яз Х1 1 0 0,3 -0,1 0 6 ‘ ~х2 0 1 -0,2 0,4 0 4 р 0 0 2 1 1 80 Поскольку все индикаторы в последней строке неотрицательны, остановимся и интерпретируем оптимальное решение. Мах Р *= 80 достигается при х\ = 6, Х2 = 4, $i = 0, s2 = 0. (Чтобы убедиться в этом, запишите целевую функцию, соответствующую последней строке, и посмотрите, что происходит с величиной Р, когда увеличивается переменная $i или s2.) ■ Упражнение 5.11. Решите следующую задачу линейного программирования, используя симплекс-метод. Максимизировать функцию Р = 2#i + х2 при условиях bxi + х2 9, £1+^2^ 5, Xl, Х2 0. ■ Задание 5.7. Изобразите область допустимых решений для задачи линейного программирования в примере 5.11 и найдите путь к оптимальному решению, определенному симплекс-методом.и Пример 5.12 (Использование симплекс-метода). Решите задачу, используя симплекс- метод. Максимизировать функцию Р = 6xi + Зх2 при условиях —2^1 + Зх2 < 9, —xi + Зх2 12, xi,x2 > 0. Решение, Запишем исходную систему, используя фиктивные переменные $i и s2: “2j?i + Зх2 Н- «si = 9, —Т1 + Зх2 + $2 = 12, —6x1 — Зх2 + Р = 0. Запишем симплекс-таблицу и определим первый ведущий элемент. Xi Х2 $1 S2 Р «1 ’ —2 3 10 0 9 ‘ *2 -1 3010 12 Р 1 : w; о о: »—1' 0 Ведущий столбец
464 Часть II. Конечная математика Поскольку оба элемента ведущего столбца, расположенные выше пунктирной линии, отрицательны, невозможно выбрать основную строку. Остановимся и сделаем вывод, что оптимального решения нет. ■ Упражнение 5.12. Решите задачу, используя симплекс-метод. Максимизировать функцию Р = 2xi + 3^2 при условиях — 3xi + 4x2 С 12, Х2 < 2, Xi, Х2 0. Задание 5.8. В примере 5.12 мы столкнулись с таблицей, содержащей ведущий столбец и не содержащей ведущей строки. Это значит, что задача не имеет оптимального решения. 1. Что случится, если нарушить правило выбора ведущей строки, выбрать в качестве ведущего элемента отрицательное число —1, а затем продолжить применение симплекс-метода? 2. В симплекс-таблице из примера 5.12 был еще один отрицательный индикатор. Что произойдет, если нарушить правило выбора ведущего столбца, выбрать в качестве ведущего второй столбец, а затем продолжить применение симплекс-метода? 3. Изобразите область допустимых решений в примере 5.12, покажите прямые, соот¬ ветствующие значениям Р = 36 и Р = 66, и объясните, почему эта задача не имеет оптимального решения. ■ Вернемся к примерам 5.11 и 5.12. В примере 5.11 сделан вывод, что оптимальное решение найдено, поскольку невозможно выбрать ведущий столбец. В примере 5.12 сделан вывод, что задача не имеет оптимального решения, поскольку был выбран ведущий столбец, но не было возможности выбрать ведущую строку. Отметим, что не следует использовать симплекс-метод, выбирая отрицательный ведущий элемент или неверный ведущий столбец. Если нет возможности выбрать ведущий столбец, работа симплекс- метода прекращается — оптимальное решение найдено. Если ведущий столбец выбран, но нет возможности выбрать ведущую строку, применение симплекс-метода прекращается — оптимального решения не существует. Решение практических задач П Пример 5.13 (Сельское хозяйство). Фермер владеет 100 акрами земли. Он планирует выращивать не больше трех видов сельскохозяйственных культур. Семена для посева на одном акре сельскохозяйственных культур Л, Б и В стоят 40, 20 и 30 долл, соответственно. На семена может быть использовано максимум 3200 долл. Для того чтобы засеять один акр земли культурой А, требуется один день, культурой Б — два дня, а культурой В — один день. Максимальное время, которым располагает фермер, — 160 рабочих
Глава 5. Линейные неравенства и линейное программирование 465 дней. Сколько акров земли должно быть засеяно каждой культурой, чтобы максимизировать прибыль, если фермер может получить прибыль 100 долл, с акра для культуры А, 300 долл, с акра для культуры Б и 200 долл, с акра для культуры В? Решение, Фермер должен выбрать количество акров, которое должно быть засеяно каждой культурой. Таким образом, переменными являются следующие величины. х\ — количество акров, засеянных культурой А, Х2 — количество акров, засеянных культурой Б, хз — количество акров, засеянных культурой В. Цель фермера — максимизировать прибыль, которая вычисляется по следующей формуле. Р = lOOxi + 300х2 + 200х3. Фермер ограничен площадью посева, объемом денежных средств на приобретение семян и рабочим временем. Все рассматриваемые в условии задачи ограничения выражаются следующим образом. Xi + Х2 + х3 С ЮО, 40xi + 20х2 -И ЗОх3 С 3200, xi -I- 2x2 + #з С 160. Ограничение площади посева Ограничение денежных средств Ограничение рабочего времени Добавляя условия неотрицательности, получим следующую модель задачи линейного программирования. Максимизировать функцию Р = 100xi 4- 300x2 + 200х3 Целевая функция при условиях Х1 + Х2 + х3 < 100, 40xi 4- 20x2 + ЗОх3 3200, ► xi 4- 2x2 + £з С 160, Ограничение задачи Xi, ^2, #з > 0. Условия неотрицательности Затем введем фиктивные переменные и составим исходную систему: XI + Х2 4- Х3 + 51 = 100, 40 а; i + 20x2 + ЗОхз + S2 = 3200, Х1 + 2х2 + Хз + S3 = 160, -100x1 —300x2 — 200x3 4- Р = 0, Xi, Х2, Х3, $1, $2, 53 0.
466 Часть II. Конечная математика Отметим, что исходная система имеет 7 — 4 = 3 небазисные и 4 базисные переменные. Теперь построим симплекс-таблицу и решим задачу с помощью симплекс-метода. Выход Вход Xl x2 хз Si 82 S3 P 51 1 1 1 1 0 0 0 100 ' \ 52 40 20 30 0 1 0 0 3200 ^*53 1 1 0 0 1 0 160 P _ -100 -300 -200 0 0 0 1 0 1 1 1 1 0 0 0 100 ‘ 40 20 30 0 1 0 0 3200 0,5 Ф 0,5 0 0 0,5 0 80 -100 -300 -200 0 0 0 1 0 0,5Лз —► Rq —Яз -I- Ri —► -Ri —2ОТ?з 4~ R>2 —* R2 300R& 4" R& —► Л4 Выход Вход 51 Xi 0,5 X2 0 0 Si 1 82 0 S3 -0,5 p 0 20 52 30 0 20 0 1 -10 0 1600 0,5 1 0,5 0 0 0,5 0 80 P 50 0 -50 0 0 150 1 24000 - 1 0 Ф 2 0 -1 0 40 30 0 20 0 1 -10 0 1600 0,5 1 0,5 0 0 0,5 0 80 50 0 -50 0 0 150 1 24000 X3 1 0 1 2 0 -1 0 40 S2 10 0 0 -40 1 10 0 800 X2 0 1 0 -1 0 1 0 60 P 100 0 0 100 0 100 i 26000 2Hi —► Ri —20.R1 4- Rs —► R% —0,5-Ri 4" -Re —► Нз 50Ri 4~ R& —► R& Все индикаторы в нижней строке неотрицательны, и можно получить оптимальное решение. = О, Х2 = 60, х3 = 40, «1 = 0, $2 = 800, «з = О, Р = 26000 долл. Таким образом, если фермер засеет 60 акров сельскохозяйственной культурой Б, 40 акров культурой В, а культурой А ничего засевать не будет, то будет достигнута максимальная прибыль, равная 26000 долл. Равенство «2 = 800 свидетельствует о том (см. вторую строку начальных уравнений), что эта максимальная прибыль достигается при использовании только 2400 долл, из имеющихся для посева 3200 долл. Иначе говоря, 800 долл, оказываются свободными и могут быть использованы для другой цели. ■ kEL WWW На рис. 5.20 иллюстрируется решение примера 5.13 с помощью программы Excel. Для решения этой задачи можно также использовать пакет прикладных программ Explorations in Finite Mathematics.
Глава 5. Линейные неравенства и линейное программирование 467 А В I C 1 D E F 1 Resources Crop A Crop В Crop C Available Used 2 Acres 1 1 1 100 100 3 Seed($) 40 20 30 3,200 2,400 4 Workdays 1 2 1 160 160 5 Profit Per Acre 100 300 200 26,000 <-Total 6 Acres to plant 0 60 40 Profit Рис. 5.20. Решение задачи с помощью программы Excel В Упражнение 5.13. Повторите пример 5.13, модифицированный следующим образом. Затраты на акр Доступно ресурсов Культура А Культура Б Культура В Стоимость семян, долл. 24 40 30 3600 Рабочие дни 1 2 2 160 Прибыль, долл. 140 200 160 Замечания. 1. Можно показать, что область допустимых решений задачи линейного программирования в примере 5.13 имеет восемь угловых точек, хотя с помощью симплекс- метода решение находится только за два шага. Теперь становится видна мощь симплекс-метода. В больших задачах разность между общим количеством угловых точек и числом операций поиска опорного плана является еще более существенной. Область допустимых значений может иметь сотни и даже тысячи угловых точек, хотя с помощью симплекс-метода оптимальное решение часто находится за 10 или 15 шагов. 2. Сошлемся на второе ограничение задачи в примере 5.13. 40xi + 20x2 + ЗОхз < 3200. Умножение обеих частей этого неравенства на до введения фиктивной переменной упрощает последующие вычисления. Однако выполнение этой операции имеет побочный эффект — оно превращает единицы фиктивной переменной из долларов в десятки долларов. Чтобы увидеть, почему это происходит, сравним следующие уравнения. 40^1 4- 20хг + ЗОх3 + $2 = 3200, «2 представляет доллары 4xi + 2x2 + Зхз + $2 = 320. «2 представляет десятки долларов Итак, если вы умножаете ограничение задачи на число, примите во внимание изменение интерпретации значений фиктивной переменной. 3. Для простоты изложения мы избегали сложных задач, в которых могут возникать вырожденные решения. Заинтересованный читатель найдет описание таких задач в специализированной литературе.
468 Часть II. Конечная математика Ответы к упражнениям 5.11. МахР = 6, если xi = 1 и х2 = 4. 5.12. Оптимального решения нет. 5.13. Сорок акров следует засеять семенами А, 60 акров — семенами Б, а семена В не сеять вообще. Р = 17600 долл, (поскольку s2 = 240, 240 долл, из 3600 долл, использовано не будет). Практикум 5.4 А В задачах 1-4 выполните следующее. 1) Определите базисные и небазисные переменные. 2) Найдите соответствующее базисное допустимое решение. 3) Определите, найдено ли оптимальное решение, нужна ли дополнительная итерация симплекс-метода, либо задача не имеет оптимального решения. 1. Х2 «1 52 р 2. 11 Х2 S1 32 Р 2 1 0 3 0 12 ’ 1 4 -2 0 0 10 3 0 1 —2 0 15 0 2 3 1 0 25 -4 0 0 4 1 20 0 5 6 0 1 35 3. аг2 *з 51 52 S3 Р 4. а?3 81 52 53 р " —2 0 1 3 1 0 0 5 ' ’ 0 2 -1 1 4 0 0 5 “ 0 1 0 —2 0 0 0 15 0 1 2 0 -2 1 0 2 -1 0 0 4 1 1 0 12 1 3 0 0 5 0 0 11 —4 0 0 2 4 0 1 45 0 -5 4 0 ■ -3 0 1 27 В задачах 5-8 найдите основной элемент, определите включаемые и исключаемые переменные и выполните одну итерацию симплекс-метода. 5. Х\ Х2 «1 52 Р 6. Х\ Х2 51 S2 Р 1 4 10 0 4 1 6 10 0 36 3 5 0 1 0 24 3 10 10 5 -8 -5 0 0 1 0 —1 —2 0 0 1 0 7. Xl Х2 31 32 S3 р 8. а?1 Х2 si з2 з3 Р ’21 100 0 4 1 ‘ ’ 0 0 2 1 1 0 2 3 0 110 0 1 10-40 1 0 3 0 0 2 0 1 0 2 0 1 5 0 2 0 ] 11 -4 0 -3 0 0 1 5 0 0 -6 0 -5 1 18 В задачах 9-12 выполните следующее. 1) Используя свободные переменные, запишите исходную систему для каждой задачи линейного программирования. 2) Запишите симплекс-таблицу, укажите первый ведущий элемент и определите включаемые и исключаемые переменные. 3) Примените симплекс-метод.
Глава 5. Линейные неравенства и линейное программирование 469 9. Максимизировать функцию Р = 15xi + 10x2 при условиях 2X1 + С 10, Xi -I- 3X2 < 10, Х1,Х2 0. 10. Максимизировать функцию Р = 3xi 4- 2x2 при условиях 5xi -I- 2x2 С 20, 3xi + 2х2 16, Х1,Х2 > 0. 11. Повторите решение задачи 9, если целевая функция изменена следующим образом Р = 30xi 4- Х2- 12. Повторите решение задачи 10, если целевая функция изменена следующим образом Р = xi + Зх2. Б Решите задачи линейного программирования 13-28, используя симплекс-метод. 13. Максимизировать функцию Р = 30xi 4- 40x2 при условиях 2xi + #2 С Ю, xi + х2 7, xi 4- 2x2 17, Х1,Х2 > 0. 14. Максимизировать функцию Р = 15xi + 20x2 при условиях 2xi -|- #2 С 9, xi 4- х2 $ 6, Xi 4- 2x2 Ю, xi,x2 > 0. 15. Максимизировать функцию Р = 2xi 4- Зх2 при условиях —2X1 + ^2 2, -xi + х2 5, Х2 < 6, Х1,Х2 0. 16. Повторите решение задачи 15 с целевой функцией Р = —xi 4- 3x2.
470 Часть II. Конечная математика 17. Максимизировать функцию Р = -х^ 4- 2x2 при условиях —xi 4- Х2 С 2, —X} 4~ 3x2 12, xi — 4х2 4, Ж1, Х2 0. 18. Повторите решение задачи 17 с целевой функцией Р = х^ 4- 2x2. 19. Максимизировать функцию Р = 5хх 4- 2x2 ~ при условиях xi 4- %2 — з?3 Ю, 2x1 4- 4x2 + Зх3 < 30, £1,^2, #3 > 0. 20. Максимизировать функцию Р = 4xi — Зхг 4- 2хз при условиях xi 4- 2x2 — #з 5, 3xi 4- 2x2 4- Зхз 22, Я1,Я2,ЯЗ 0. 21. Максимизировать функцию Р = 2xi 4- Зхг 4- 4х3 при условиях xi 4- хз < 4, Х2 4- х3 < 3, Я1,Я2,ЯЗ > 0. 22. Максимизировать функцию Р = xi 4- Х2 4- 2хз при условиях Х1 — 2X2 4- Хз 9, 2xi 4- х2 4- 2х3 28, X1, Х2, Х3 0. 23. Максимизировать функцию Р = 4xi 4- 3x2 4- 2хз при условиях 3xi 4- 2хг 4- 5хз < 23, 2xi 4- Х2 4- хз 8, xi 4~ Х2 4- 2хз 7, Я1,Я2,373 0. 24. Максимизировать функцию Р = 4хг 4- 2x2 4- Зхз при условиях xi 4- Х2 4- Хз 11, 2xi Н- 3x2 4~ Хз 20, Xi 4~ 3x2 4~ 2хз 20, 3'1,372,3?з 0.
Глава 5. Линейные неравенства и линейное программирование 471 В 25. Максимизировать функцию Р = 20xi 4- 30х2 при условиях 0,6^1 + 1,2X2 960; 0,03xi + 0,04х2 36; 0,3xi + 0,2х2 < 270; xi,x2 > 0. 26. Повторите решение задачи 25 с целевой функцией Р = 20xi 4- 20х2. 27. Максимизировать функцию Р = xi 4- 2х2 4- Зхз при условиях 2xi 4- 2х2 4- 8x3 < 600, xi + Зх2 4- 2хз < 600, 3xi 4- 2х2 4 Хз 400, х 1, х2,Х3 0. 28. Максимизировать функцию Р = 10xi 4- 50х2 4- Юхз при условиях 3xi 4- Зх2 4- Зхз < 66, 6xi — 2х2 4- 4хз 48, 3xi 4- 6х2 -4 9хз 108, £1,^2, 0. В задачах 29 и 30 сначала решите задачу линейного программирования с помощью симплекс-метода, отслеживая базисное допустимое решение на каждом шаге. Затем изобразите область допустимых решений и проиллюстрируйте поиск соответствующего оптимального решения. * 29. Максимизировать функцию Р = 2xi 4- 5х2 при условиях xi 4- 2х2 < 40, xi 4- Зх2 48, xi 4- 4х2 С 60, х2 14, xi,x2 > 0. *30. Максимизировать функцию Р = 5xi 4- Зх2 при условиях 5xi 4- 4х2 < 100, 2xi 4~ 28, 4xi + ^2 42, xi < 10, xi,x2 > 0.
472 Часть II. Конечная математика В задачах 31-34 возможен неоднозначный выбор первого ведущего столбца. Используя симплекс-метод, решите задачу двумя различными способами: сначала выбирая в качестве ведущего столбец 1, а затем — столбец 2. Рассмотрите взаимосвязь между этими двумя решениями. *31. Максимизировать функцию Р = + х2 При УСЛОВИЯХ 2X1 + Х2 16, xi < 6, Х2 < 10, Х1,Х2 > 0. * 32. Максимизировать функцию Р = х± + Х2 при условиях Х1 + 2X2 С 10, Х1 < 6, Х2 4, xi,x2 0. * 33. Максимизировать функцию Р = 3xi + Зх2 + 2хз при условиях Xi + Х2 + 2Хз < 20, 2^1 + х2 + 4х3 32, Xi, Х2, Х3 0. * 34. Максимизировать функцию Р = 2xi + 2х2 + хз при условиях Х1 + х2 + Зх3 < 10, 2xi + 4x2 + 5х3 24, Х1,х2,х3 > 0. Применение математики Сформулируйте задачи линейного программирования. (Ответы приведены в конце книги.) Затем решите задачу, используя симплекс-метод. Дайте интерпретацию каждой ненулевой фиктивной переменной в оптимальном решении. Бизнес и экономика 35. Производство: распределение ресурсов. Маленькая компания производит три разных электронных компонента для компьютеров. Для выпуска компонента А необходимо два часа для изготовления и один час для сборки; для выпуска компонента Б необходимо три часа для изготовления и один час для сборки; для выпуска компонента В необходимо два часа для изготовления и два часа для сборки. Компания может использовать до 1000 рабочих часов для изготовления и 800 рабочих часов для сборки еженедельно. Прибыль от реализации каждого компонента А, Б и В составляет 7, 8 и 10 долл, соответственно. Сколько
Глава 5. Линейные неравенства и линейное программирование 473 компонентов каждого типа должна произвести компания еженедельно, чтобы получить максимальную прибыль (предполагается, что все произведенные компоненты могут быть проданы)? Чему равна максимальная прибыль? *36. Производство: распределение ресурсов. Решите задачу 35, добавив дополнительное ограничение, состоящее в том, что общее количество всех компонентов, производимых еженедельно, не может превышать 420. Рассмотрите влияние этого ограничения на решение задачи 35. 37. Капиталовложение. У инвестора есть около 100000 долл, для вложения в государственные облигации, взаимные фонды и фонды денежного рынка. Доходность государственных облигаций, взаимных фондов и фондов денежного рынка составляют 8, 13 и 15% соответственно. Стратегия инвестора требует, чтобы сумма средств, вложенных во взаимные фонды и фонды денежного рынка, не превышала сумму, вложенную в государственные облигации. Сколько денег следует вложить в каждый из фондов, чтобы получить максимальный доход? 38. Капиталовложение. Повторите задачу 37, предположив, что в фонды денежного рынка может быть вложено не более 30 000 долл. 39. Реклама. У компании, управляющей сетью магазинов, есть около 20 000 долл, для размещения рекламы на телевидении. Все рекламные ролики будут демонстрироваться одной телевизионной станцией. В дневное время 0,5 мин. рекламы стоят 1000 долл., а объем аудитории достигает 14000 зрителей. В вечернее время 0,5 мин. рекламы стоят 2000 долл., а объем аудитории увеличивается до 24000 зрителей. В ночное время 0,5 мин. рекламы стоят 1500 долл., а объем аудитории снижается до 18 000. Телевизионная станция не может показать больше 15 рекламных роликов во всех трех временных периодах. Сколько рекламных роликов должно быть помещено в каждый временной период так, чтобы максимизировать количество потенциальных покупателей, которые увидят рекламу? (Не обращайте внимание на повторный просмотр рекламы одним и тем же потенциальным покупателем.) 40. Реклама. Повторите решение задачи 39 для случая, когда отдел рекламы увеличивает свой бюджет до 24 000 долл, и требует, чтобы по крайней мере половина рекламных роликов была показана в прайм-тайм. 41. Строительство: распределение ресурсов. Подрядчик планирует построить новую серию домов в колониальном стиле, многоуровневых домов и усадеб. Для строительства дома в колониальном стиле требуется 1/2 акра земли, 60 000 долл, и 4000 рабочих часов. Такое строительство приносит 20000 долл, прибыли. Для строительства многоуровневого дома требуется 1/2 акра земли, 60 000 долл, и 3000 рабочих часов. Такое строительство приносит 18 000 долл, прибыли. Для строительства усадьбы требуется один акр земли, 80 000 долл, и 4000 рабочих часов. Такое строительство приносит 24 000 долл, прибыли. У подрядчика есть 30 акров земли, 3 200000 долл, и 180000 рабочих часов. а) Сколько домов каждого типа следует построить, чтобы максимизировать прибыль подрядчика? Чему равна максимальная прибыль? *б) Уменьшение спроса на дома в колониальном стиле приводит к падению прибыли от строительства такого дома с 20 000 до 17 000 долл. Рассмотрите
474 Часть II. Конечная математика влияние этого изменения на количество построенных домов и максимальную прибыль. *в) Увеличение спроса на дома в колониальном стиле приводит к увеличению прибыли от строительства такого дома с 20 000 до 25 000 долл. Рассмотрите влияние этого изменения на количество построенных домов и максимальную прибыль. 42. Производство: распределение ресурсов. Компания производит трех-, пяти- и десятискоростные велосипеды. Каждый велосипед проходит три этапа — изготовления, покраска и окончательной сборки. Соответствующие данные приведены в таблице. а) Сколько велосипедов каждого типа следует компании производить еженедельно, чтобы максимизировать прибыль? Какова максимальная прибыль? ♦б) Как влияет на решение п. а увеличение прибыли от производства десятискоростного велосипеда до ПО долл.? *в) Как влияет на решение п. а увеличение прибыли от производства пятискоростного велосипеда до 110 долл.? Рабочее время, затраченное на производство велосипеда, ч Общее рабочее время, ч Трехскоростной Пятискоростной Десятискоростной Изготовление 3 4 5 120 Покраска 5 3 5 130 Сборка 4 3 5 120 Прибыль от одного велосипеда, долл. 80 70 100 43. Упаковка: ассортимент изделий. Кондитерская фабрика производит три сорта конфет — с твердой начинкой, фруктовой и кремовой начинкой и упаковывает их в коробки. Коробка ассортимента I содержит 4 конфеты с твердой начинкой, 4 — с фруктовой и 12 — с кремовой. Она продается за 9,40 долл. Коробка ассортимента II содержит 12 конфет с твердой начинкой, 4 — с фруктовой и 4 — с кремовой. Она продается за 7,60 долл. Коробка конфет ассортимента III содержит 8 конфет с твердой начинкой, 8 — с фруктовой и 8 — с кремовой. Она продается за 11,00 долл. Затраты на производство одной конфеты составляют 0,20 долл, для конфеты с твердой начинкой, 0,25 долл. — для конфеты с фруктовой начинкой и 0,30 долл. — для конфеты с кремовой начинкой. Еженедельно фабрика может
Глава 5. Линейные неравенства и линейное программирование 475 производить 4800 конфет с твердой начинкой, 4000 — с фруктовой и 5600 — с кремовой. а) Сколько коробок конфет каждого типа следует производить фабрике еженедельно, чтобы максимизировать прибыль? Чему равна максимальная прибыль? *б) Как изменится решение п. а, если количество производимых еженедельно конфет с фруктовой начинкой увеличится до 5000, а все остальные показатели останутся без изменения? *в) Как изменится решение п. а, если количество производимых еженедельно конфет как с фруктовой, так и с твердой начинкой увеличится до 6000, а все остальные показатели останутся без изменения? 44. Планирование: распределение ресурсов. Малая бухгалтерская фирма готовит налоговые декларации для трех типов плательщиков: индивидуальных, коммерческих и промышленных. Процесс подготовки декларации начинается с одночасового опроса плательщика налогов. Данные, собранные во время этого опроса, вводятся в компьютерную систему, работающую в режиме разделения времени. Для того чтобы ввести данные об индивидуальном плательщике требуется один час, коммерческом — два часа, а промышленном — полтора часа. Расчет налогов индивидуального плательщика занимает 10 мин., коммерческого — 25 мин., промышленного — 20 мин. У фирмы есть один работник, проводящий первоначальный опрос, и два работника, которые вводят данные в компьютер. Опрашивающий может работать не более 50 ч в неделю, а каждый из сотрудников, вводящих данные, может работать максимум 40 ч в неделю. Компьютер доступен 1025 мин. еженедельно. Каждый индивидуальный плательщик налогов приносит бухгалтерской фирме 50 долл, прибыли, коммерческий — 65 долл., а промышленный — 60 долл. а) Сколько Плательщиков налогов каждого типа должна принять фирма еженедельно, чтобы максимизировать прибыль? Чему равна максимальная прибыль? *б) Как изменится решение п. а, если общее время опроса увеличится до 35 в неделю, а все другие данные останутся без изменения? *в) Как изменится решение п. а, если общее время работы компьютера уменьшится до 450 мин в неделю, а все остальные данные останутся без изменения? Биологические науки 45. Рацион питания. Естественный рацион питания некоего животного состоит из трех пищевых продуктов — А, Б и В. Количество единиц кальция, железа и белков в одном грамме каждого пищевого продукта и среднее суточное потребление приведены в таблице. Ученый хочет узнать, как повлияет на животное увеличение уровня потребления белков, если кальций и железо потребляются на среднесуточном уровне. Сколько граммов каждого пищевого продукта следует использовать, чтобы максимизировать количество белков в рационе? Каково максимальное количество белков?
476 Часть II. Конечная математика Пищевой продукт А, ед/г Пищевой продукт Б, ед/г Пищевой продукт В, ед/г Среднесуточное потребление, ед. Кальций 1 3 2 30 Железо 2 1 2 24 Белки 3 4 5 60 46. Рацион питания. Повторите решение задачи 45 для ситуации макисмизации суточного потребления кальция, не позволяя железу или белкам превышать уровень среднесуточного потребления. Социальные науки 47. Опрос общественного мнения. Политолог получил грант на исследовательский проект по изучению тенденций в голосовании. Согласно условиям гранта на поквартирный опрос избирателей выделяется сумма 3200 долл. Для проведения опроса наняты студенты, аспиранты и сотрудники факультета. Каждый студент проводит 18 опросов за 100 долл. Каждый аспирант проводит 25 опросов за 150 долл. Каждый сотрудник факультета проводит 30 опросов за 200 долл. Из-за ограниченных транспортных возможностей может быть нанято не более 20 опрашивающих. Сколько студентов, аспирантов и сотрудников факультета нужно набрать, чтобы максимизировать количество опросов? Чему равно максимальное количество опросов? 48. Опрос общественного мнения. Повторите решение задачи 47, если одно из требований гранта состоит в том, чтобы по крайней мере 50% опрашивающих были аспирантами. 5.5. Двойственная задача: минимизация с ограничениями вида “больше или равно” ■ Постановка двойственной задачи ■ Решение задач минимизации ■ Применение: транспортная задача ■ Резюме Материал предыдущего раздела был ограничен стандартной задачей максимизации (ограничения задачи имели вид “меньше или равно”, в правой части допускались только неотрицательные постоянные, а коэффициентами целевой функции могли быть любые действительные числа). Теперь будем рассматривать задачу минимизации с ограничениями задачи вида “больше или равно”. Оказывается, что эти два типа задач очень тесно связаны между собой.
Глава 5. Линейные неравенства и линейное программирование 477 Постановка двойственной задачи Каждая задача минимизации с ограничениями вида “больше или равно” связана с задачей максимизации, которая называется двойственной. Для иллюстрации формулировки двойственной задачи рассмотрим следующую задачу минимизации. Минимизировать функцию С = 16xi + 45x2 при условиях 2xi 4- 5^2 50, xi 4- 3x2 > 27, Xi, Х2 > 0. (5.16) Первый шаг в получении двойственной задачи состоит в том, чтобы построить матрицу, используя ограничения задачи и целевую функцию, записанную в следующем виде. 2xi + 5x2 50, xi + 3x2 27, 16x1 4- 45x2 = С. 2 5 50 ' А = 1 3 27 16 45 1 Предупреждение. Не путайте матрицу А с симплекс-таблицей. Для того чтобы отличать двойственную матрицу от симплекс-таблицы, в матрице А используется непрерывная горизонтальная линия. Матрица А не содержит фиктивных переменных, а коэффициент С стоит в том же столбце, что и постоянные из ограничений задачи. Теперь образуем другую матрицу, называемую транспонированной к матрице А, Вообще говоря, транспонированной к данной матрице А является матрица Ат, полученная заменой соответствующих строк соответствующими столбцами (первой строки — первым столбцом, второй строки — вторым столбцом и т.д.). А = 2 5 1 3 50 ’ 27 Я1 в А= С1 в Ат Яа в А = С3 в Ат 16 45 1 Яз в А = С3 в Ат I Ат = 1 2 1 5 3 1 , 16 45 Ат — транспонированная матрица А 50 27 1 Если для выполнения матричных операций используется графическая утилита, то информацию о вычислении транспонированной матрицы следует искать в справочном руководстве (рис. 5.21). Теперь для определения новой задачи линейного программирования используем строки Ат. Этой новой задачей всегда будет задача максимизации с ограничениями задачи “меньше или равно”. Чтобы избежать путаницы, в этой новой задаче будем использовать другие переменные. У1 У2 %У1 + У2 С 16, 2 1 16 ‘ 5j/i + 3j/2 45, Ат = 5 3 45 50yi + 27у2 = Р- 50 27 1
478 Часть II. Конечная математика [[2 5 50] [1 3 27] [16 45 1 ]] тнп [[2 1 16] [5 3 45] [50 27 1 ]] Рис. 5.21. Вычисление матрицы Ат с помощью графической утилиты Двойственной к задаче минимизации (5.16) является следующая задача. Максимизировать функцию Р = 501/1 + 27у2 при условиях 21/1 + У2 16, 5?/1 + 3?/2 45, 3/1, 3/2 > О- (5.17) Задание 5.9. Исключая условия неотрицательности, компоненты задачи линейного программирования можно разделить на три категории: коэффициенты целевой функции, коэффициенты ограничений задачи и постоянные в правых частях ограничений задачи. Опишите взаимосвязи между компонентами исходной задачи минимизации (5.16) и двойственной задачи максимизации (5.17). ■ Постановка двойственной задачи Рассмотрим задачу минимизации с ограничениями “больше или равно”. Этап 1. Запишем в последней строке матрицы коэффициенты целевой функции исходной задачи. Этап 2. Транспонируем матрицу А, меняя ролями ее строки и столбцы. Этап 3. В качестве коэффициентов ограничений задачи максимизации с ограничениями “меньше или равно” используем числа, стоящие в строках матрицы Ат. Пример 5.14 (Постановка двойственной задачи). Сформулируйте двойственную задачу. Минимизировать функцию С = 40х i + 12x2 + 40хз при условиях 2^1 + #2 + 5х3 > 20, 4xi + #2 + 30, Х1, #2, #з > 0. Решение. Этап 1. Образуем матрицу А. 2 1 5 20 ' А = 4 1 1 30 40 12 40 1
Глава 5. Линейные неравенства и линейное программирование 479 Этап 2. Найдем матрицу Ат, транспонированную к А. 2 4 40 1 1 12 5 1 40 20 30 1 Этап 3. Поставим двойственную задачу. Максимизировать функцию Р = 20a/i + ЗО3/2 при условиях 2^/1 + 4?/2 С 40, У1 + У2 12, 5у1 + У2 40, 2/1, У2 0. Упражнение 5.14. Сформулируйте двойственную задачу. Минимизировать функцию С = 16a?i -I- 9х2 4- 21хз при условиях Xi 4- Х2 + Зхз > 12, 2а?1 4- #2 4- Хз 16, xi, я2, > 0. Решение задач минимизации Следующая теорема устанавливает связь между решениями задачи минимизации и двойственной к ней задачи* Теорема 5.6 (Основной принцип двойственности). Задача минимизации имеет решение тогда и только тогда, когда двойственная к ней задача имеет решение. Если решение существует, оптимальное значение задачи минимизации совпадает с оптимальным значением двойственной задачи. ■ Доказательство теоремы 5.6 выходит за рамки этой книги. Однако ее можно проиллюстрировать с помощью геометрического решения задачи минимизации (5.16) и ее двойственной задачи максимизации (5.17). (Отметим, что теорема 5.3 раздела 5.2 гарантирует, что обе задачи имеют решения.) Исходная задача (5,16) Минимизировать функцию С = 16xi 4- 45х2 при условиях 2^1 4- 5х2 > 50, xi 4- Зх2 > 27, xi, х2 > 0.
480 Часть II. Конечная математика Угловая точка (xi; х2) С = 16а?1 + 45х2 (0; 10) 450 (15; 4) 420 (27; 0) 432 Min С = 420 в точке (15; 4). Двойственная задача (5.17) Максимизировать функцию Р = 501/1 + 271/2 при условиях + У2 16, 5уг + Зт/2 < 45, 2/1, У2 > 0. Угловая точка (i/i; у2) Р = 50j/i + 27у2 (0;0) 0 (0; 15) 405 (3; Ю) 420 (8;0) 400 МахР = 420 в точке (3; 10). Таким образом, минимальное значение функции С в задаче (5.16) совпадает с максимальным значением функции Р в задаче (5.17). Отметим, что оптимальные решения, на которых достигается максимальное значение, разные: оптимальным решением задачи (5.16) является точка (15; 4), а задачи (5.17) — точка (3; 10). Теорема 5.6 гарантирует
Глава 5. Линейные неравенства и линейное программирование 481 только то, что оптимальные значения задачи минимизации и ее двойственной задачи равны, а не то, что оптимальные решения совпадают. Вообще говоря, изучая допустимое множество двойственной задачи, невозможно определить оптимальное решение задачи минимизации. Однако к двойственной задаче можно применить симплекс-метод и найти как оптимальное значение, так и оптимальное решение исходной задачи минимизации. Чтобы увидеть, как это делается, решим задачу (5.17) с помощью симплекс-метода. По причинам, которые пояснятся позже, в качестве фиктивных переменных в двойственной задаче будем использовать переменные xi и я2 из исходной задачи. 2?/1 + 5yi + У2 +Х1 = 16 = 45 Исходная система для двойственной задачи Зт/2 + Х2 - 50?/1 - 272/2 + Р = 0 У1 У2 Х1 Х2 р Х1 1 1 0 0 16 ■ 0,5Я1 —► Ri 5 3 0 1 0 45 Р -50 —27 0 0 1 0 _ - Ф 0,5 0,5 0 0 8 ' 5 3 0 1 0 45 —5Я1 -|- Rz —► Ri -50 -27 0 0 1 0 50Я1 + Яз —► Яз У1 1 05 0,5 0 0 8 ‘ ~х2 0 -2,5 1 0 5 2 Яг —► R2 Р 0 —2 25 0 1 400 1 0,5 0,5 0 0 8 ' —0,5Яз -|- Я1 —► Я1 0 Ф -5 2 0 10 0 —2 25 0 1 400 2R2 + Яз —► Яз У1 1 0 3 -1 0 3 У2 0 1 -5 2 0 10 р 0 0 15 4 1 420 Поскольку все индикаторы в нижней строке неотрицательны, решением двойственной задачи является следующий набор чисел. У1 =3, у2 = Ю, х\ = 0, х2 = 0, Р = 420. Это согласуется с геометрическим решением, полученным ранее. Более того, изучая нижнюю строку окончательной симплекс-таблицы, мы видим то же оптимальное решение задачи минимизации, которое получено непосредственно с помощью геометрического метода. Min С = 420 при xi = 15, х2 = 4. Это не случайно. Оптимальное решение задачи минимизации всегда может быть получено из нижней строки окончательной симплекс-таблицы для двойственной задачи. Итак, использование переменных х± и х2 в качестве фиктивных переменных двойственной задачи позволяет просто определить решение исходной задачи.
482 Часть II. Конечная математика Задание 5.10. Симплекс-метод можно использовать для решения любой стандартной задачи максимизации. Какая из следующих задач минимизации имеет в качестве двойственной стандартную задачу максимизации? (Задачи решать не требуется.) 1. Минимизировать функцию С = 2xi + Зх2 при условиях 2x1 — 5^2 > 4, Х1 — 3X2 —6, xi, х2 0. 2. Минимизировать функцию С = 2xi — Зх2 при условиях —2x1 + 5х2 4, —xi 4- Зх2 6, Xi, Х2 0. Вообще говоря, каким условиям должна удовлетворять задача минимизации, чтобы ее двойственная задача была стандартной задачей максимизации? ■ Решение задачи минимизации Рассмотрим задачу минимизации с целевой функцией, имеющей неотрицательные коэффициенты. Этап 1. Запишем все ограничения задачи в виде неравенств “больше или равно”. (Это может привести к тому, что в правых частях некоторых ограничений задачи появятся отрицательные числа.) Этап 2. Сформулируем двойственную задачу. Этап 3. Запишем исходную систему двойственной задачи, используя в качестве фиктивных переменные из задачи минимизации. Этап 4. Используем симплекс-метод для решения двойственной задачи. Этап 5. Читаем решение задачи минимизации в нижней строке окончательной сим- плекс-таблицы, полученной на этапе 4. (Замечание. Если двойственная задача не имеет оптимального решения, то и задача минимизации не имеет оптимального решения.) Пример 5.15 (Решение задачи минимизации). Решите следующую задачу минимизации с помощью максимизации двойственной. Минимизировать функцию С = 40xi 4- 12х2 4- 40хз при условиях 2xi 4" х2 4“ 5хз > 20, 4xi + + х3 30, Х1, ^2) 0.
Глава 5. Линейные неравенства и линейное программирование 483 Решение. С помощью метода, использованного в примере 5.14, получаем двойственную задачу. Максимизировать функцию Р = 201/1 + ЗО1/2 при условиях 2i/i + 41/2 40, 2/1 + 2/2 12, 52/1 + 2/2 < 40, 2/ь 2/2 > 0. Используя в качестве фиктивных переменные х±, и х3, получим исходную систему для двойственной задачи. 2?/i + 4j/2 + = 40, У1 + У2 + х2 = 12, + У2 + хз = 40, -20уг - 30у2 + Р = 0. Теперь образуем симплекс-таблицу и решим двойственную задачу: Ж1 У1 2 XI 1 х2 0 Хз 0 р 0 40 0,25.Ri —► Ri Х2 1 1 0 1 0 0 12 хз 5 1 0 0 1 0 40 Р —20 -30 0 0 1 1 0 1 2 ф 1 4 0 0 0 40 1 1 0 1 0 0 12 —R1 4~ R& —► R2 5 1 0 0 1 0 40 —Ri 4~ R3 —► R3 —20 -30 0 0 0 1 0 30Н1 4" R4 —► R& 2/2 г 1 2 1 1 4 0 0 0 10 ' х2 ® 0 1 4 1 0 0 2 2R2 —► R2 Хз 9 2 0 1 4 0 1 0 30 р L -5 0 15 2 0 0 1 300 J г 1 2 1 4 0 0 0 10 ' —-F Ri —► Ri © 0 1 2 2 0 0 4 9 2 0 _ 1 4 0 1 0 30 + Из —► R& 4~ Р4 —* Ra L -5 0 ‘"'15“‘ 2 0 0 1 300 2/2 ■ 0 1 1 2 -1 0 0 8 ' 2/1 1 0 1 2 2 0 0 4 'хз 0 0 2 -9 1 0 12 р L о 0 5 10 0 1 320 J В нижней строке таблицы записан результат. Min С = 320 при xi = 5, х2 = 10, ж3 = 0.
484 Часть II. Конечная математика Упражнение 5.15. Решите следующую задачу минимизации с помощью максимизации двойственной (см. упражнение 5.14). Минимизировать функцию С = 16xi 4- 9^2 + 21хз при условиях xi + #2 + Зхз > 12, 2xi 4- х2 4- х3 > 16, Х1, ^2, ^3^0- ■ Предупреждение. В предыдущем разделе отмечалось, что умножение неравенств на число (обычно с целью упрощения вычислений) изменяет единицы измерения фиктивных переменных. В результате требуется специальная интерпретация их значений в оптимальном решении, но серьезные проблемы не возникают. Однако при использовании двойственного метода умножение неравенств на число в двойственной задаче может иметь весьма серьезные последствия — нижняя строка окончательной симплекс-таблицы может уже не приводить к верному решению задачи минимизации. Чтобы понять это, обратимся к первому ограничению двойственной задачи в примере 5.15. 2т/1 + 41/2 40. Если умножить это ограничение на 1/2, а затем решить систему, окончательная таблица примет следующий вид (проверьте это). 2/1 У2 х2 х3 Р ■ 0 1 1 -1 0 0 8 ' 1 0 -1 2 0 0 4 0 0 4 -9 1 0 12 0 0 10 10 0 1 320 Нижняя строка этой таблицы определяет тот факт, что оптимальное решение задачи минимизации равно С = 320 при xi = 10 и х2 = 10. Это неверный ответ (верный ответ: xi = 5). Таким образом, в задаче максимизации никогда нельзя умножать неравенство на число, если эта задача используется для решения задачи минимизации. Можно просто упростить ограничение в задаче минимизации прежде, чем формулировать двойственную задачу. Пример 5.16 (Решение задачи минимизации). Решите следующую задачу минимизации с помощью максимизации двойственной. Минимизировать функцию С — 5xi 4- Юхг при условиях ^1 - #2 1, -xi 4- х2 2, Решение. xi, х2 0. 1 -1 1 ■ 1 -1 5 А = -1 1 2 лт = -1 1 10 5 10 1 1 2 1
Глава 5. Линейные неравенства и линейное программирование 485 Двойственной является следующая задача. Максимизировать функцию Р = ух + 2у2 при условиях ух — у2 < 5, -У1 + У2 10, 2/1, 2/2 > 0. Введем фиктивные переменные хх и х2 и образуем исходную систему для двойственной задачи. 2/1 - 2/2 4- хх = 5, —2/1 + 2/2 + х2 = 10, —2/1 “22/2 + Р = 0. Образуем симплекс-таблицу и решим систему. Хх Х2 Р Хх ~х2 р Веду У1 У2 Xi. х2 Р 1-1100 -1 ф 0 1 0 5 ‘ 10 4“ 2Дз 4“ -Re —► -Не Выше пунктирной линии в ведущем столбце нет положительных элементов. —1 —2 0 0 1 0 0 110 -1 1010 0 _ 15 ' 10 -3 0021 J к щии столбец 20 Число —3 в нижней строке означает, что первый столбец является ведущим. Поскольку в нем выше пунктирной линии нет положительных элементов, ведущую строку выбрать невозможно. Следовательно, необходимо прекратить итерации симплекс-метода и сделать вывод, что задача максимизации не имеет оптимального решения (см. рис. 5.19, раздел 5.4). Из теоремы 5.6 следует, что исходная задача минимизации не имеет решения. График неравенств в задаче минимизации (рис. 5.22) свидетельствует, что область допустимых решений пуста. Таким образом, не удивительно, что оптимального решения не существует. g Упражнение 5.16. Решите следующую задачу минимизации с помощью максимизации двойственной. Минимизировать функцию С = 2xi -I- 3^2 при условиях хх — 2х2 2, —хх 4- х2 1, Х1,Х2 0.
486 Часть II. Конечная математика Рис. 5.22. Область допустимых решений Применение: транспортная задача Q Одним из первых применений линейного программирования было решение задачи о минимизации затрат на транспортировку материалов. Задачи такого типа называются транспортными. И Пример 5.17 (Транспортная задача). Компания по производству компьютеров имеет два сборочных завода: А и Б, а также два магазина: I и II. Ежемесячно завод А может собирать не больше 700 компьютеров, а завод Б — не больше 900. Кроме того, ежемесячно в магазин I необходимо поставлять по крайней мере 500, а в магазин II — 1000 компьютеров. Расходы на транспортировку одного компьютера с одного завода в каждый из магазинов таковы: с завода А в магазин 1 — 6 долл., с завода А в магазин II — 5 долл., с завода Б в магазин 1 — 4 долл., с завода Б в магазин II — 8 долл. Найдите транспортное расписание, минимизирующее общие затраты на перевозку компьютеров со сборочных заводов в магазины. Чему равна минимальная величина этих затрат? Решение, Чтобы получить транспортное расписание, необходимо решить, сколько компьютеров следует перевозить с каждого завода в каждый магазин (рис. 5.23). Для этого введем четыре переменные. xi — количество компьютеров, перевозимых с завода А в магазин I, Х2 — количество компьютеров, перевозимых с завода А в магазин II, х$ — количество компьютеров, перевозимых с завода Б в магазин I, х± — количество компьютеров, перевозимых с завода Б в магазин II. Подытожим соответствующие данные в таблице. Отметим, что мы не ассоциируем каждую переменную со столбцом. Эта таблица похожа на матрицы инцидентности из раздела 4.4. В ней источники связаны со строками, а пункты назначения — со столбцами.
487 Глава 5. Линейные неравенства и линейное программирование Объем поставок, шт Объем производства, шт Магазин I Магазин II Завод А 6 5 700 Завод Б 4 8 900 Минимальная потребность, шт 500 1000 Рис. 5.23. Транспортная задача Общее количество компьютеров, перевезенных с завода Л, равно xi + #2- Поскольку это количество не должно превышать объем производства завода Л, получаем следующий результат. + #2 700 Количество компьютеров, собираемых заводом А Аналогично общее количество компьютеров, собираемых заводом Б, должно удовлетворять следующему неравенству. х3 + х4 < 900 Количество компьютеров, собираемых заводом Б Общее количество компьютеров, доставленных в каждый из магазинов, должно удовлетворять следующему неравенству. Xi + *з 500 Количество компьютеров, поставленных в магазин I и + #4 Ю00 Количество компьютеров, поставленных в магазин II Используя величину затрат на доставку, указанную в таблице, вычислим суммарную величину затрат на доставку. С = 6x1 + 5x2 + 4х3 + 8x4. Таким образом, необходимо решить следующую задачу линейного программирования. Минимизировать функцию с = 6x1 + 5x2 + 4хз -I- 8x4 при условиях Xi + Х2 700, Объем производства завода А ^3 + ^4 < 900, Объем производства завода Б Х1 + Х3 500, Заказ магазина I Х2 + Х4 1000, Заказ магазина II Xi, Х2 , Х3, Х4 0.
488 Часть II. Конечная математика Сначала необходимо умножить первые два неравенства на —1, чтобы все условия приняли вид “больше или равно”. Эта процедура породит отрицательные постоянные в задаче минимизации, но не в двойственной к ней. Поскольку коэффициенты целевой функции С неотрицательны, постоянные в двойственной задаче будут неотрицательными, и двойственная задача будет стандартной задачей максимизации. Итак, задачу можно представить следующим образом. Минимизировать функцию С = 6xi 4- 5я?2 + 4#з + при условиях —xi — х2 —700, —хз — дц —900, xi + Х3 500, х2 + #4 > 1000, #1, #2, ^3, #4 0. ■ -1 -1 0 0 -700 ‘ -1 0 1 0 6 0 0 -1 -1 -900 -1 0 0 1 5 А = 1 0 1 0 500 , Ат = 0 -1 1 0 4 0 1 0 1 1000 0 -1 0 1 8 6 5 4 8 1 -700 -900 500 1000 1 Двойственная задача выглядит следующим образом. Максимизировать функцию Р = —700т/1 — 900^/2 + 500?/з + IOOO1/4 при условиях — yi + уз ^6, ~У1 + 2/4^5, —2/2 Н-2/з ^4, —2/2 + 2/4 8, 2/1, 2/2, 2/3, 2/4 > 0. Введем фиктивные переменные х±9 х2, хз и х± и образуем исходную систему для двойственной задачи. —2/1 + 2/з + Ж1 = 6, ~У1 + У4 ~У2 + Уз ~У2 + 2/4 700г/1 + 900т/2 — 500уз — IOOO1/4 + %2 — 5, + Хз -4, + Х4 =5, + Р = 0.
Глава 5. Линейные неравенства и линейное программирование 489 Составим симплекс-таблицу и решим соответствующую систему. У1 У2 Уз 1/4 Х1 Х2 Хз а?4 Р Я1 -1 0 1 0 1 0 0 0 0 6' Х2 -1 0 0 фо 1 0 0 0 5 хз 0 -1 1 0 0 0 1 0 0 4 Х± 0 -1 0 1 0 0 0 1 0 8 —R"2 4“ Ra —► Ra р 700 900 -500 -1000 0 0 0 0 1 0 ЮООЛ2 + R& —► R& -1 0 1 0 1 0 0 0 0 6" —R3 4~ Ri —► Ri 1/4 -1 0 0 1 0 1 0 0 0 5 •хз 0 -1 ф 0 0 0 1 0 0 4 X д 1 -1 0 0 0 -1 0 1 0 3 Р -300 900 -500 0 0 1000 0 0 1 5000 500Яз 4“ R& —► R& Х1 -1 0 0 0 1 0 -1 0 0 2 R& 4~ Ri —► Ri У4 -1 0 0 1 0 1 0 0 0 5 R4 4~ R2 —► R2 •Уз 0 -1 1 0 0 0 1 0 0 4 х^ ф -1 0 0 0 -1 0 1 0 3 р -300 400 0 0 0 1000 500 0 1 7000 ЗООА4 4“ R& —► R& Х1 0 0 0 0 1 -1 -1 1 0 5' У4 0 -1 0 1 0 0 0 1 0 8 •Уз 0 -1 1 0 0 0 1 0 0 4 У1 1 -1 0 0 0 -1 0 1 0 3 р 0 100 0 0 0 700 500 300 1 7900 Решение задачи записано в нижней строке этой таблицы. Min С — 7900 при х\ = 0, Х2 = 700, хз = 500, х± = 300. Расписание, минимизирующее затраты на перевозки, предусматривает поставку 7000 компьютеров с завода Л в магазин II, 500 компьютеров с завода в магазин I и 300 компьютеров с завода Б в магазин II. Величина общих затрат на перевозку составляет 7900 долл.и ю. WWW На рис. 5.24 изображено решение примера 5.17, полученное с помощью программы Excel. Программа Excel позволяет представить исходные данные и решение в формате, который понятен и легок для чтения. В этом состоит одно из основных преимуществ использования электронных таблиц для решения задач линейного программирования. Для решения транспортной задачи можно также применить пакет прикладных программ Explorations in Finite Mathematics. И Упражнение 5.17. Решите пример 5.17 для варианта, когда затраты на перевозку с завода Л в магазин I увеличиваются до 7 долл., а затраты на перевозку с завода Б в магазин II увеличиваются до 3 долл. ■
490 Часть II. Конечная математика А в | с D I E| I E I G | H | 1 I 1 DATA SHIPPING SCHEDULE 2 DISTRIBUTION OUTLET 1 II ASSEMBLY CAPACITY DISTRIBUTION OUTLET I II TOTAL 3 4 5 PLANT А PLANT В $6 $5 $4 $8 700 900 PLANTA PLANT В 0 700 500 300 700 800 6 7 MINIMUM REQUIRED 500 1,000 TOTAL 500 1,000 8 TOTAL COST $7,900 Рис. 5.24. Решение задачи о перевозках с помощью программы Excel Резюме В этом и предыдущем разделах решались как задачи максимизации, так и задачи минимизации, но с определенными условиями, налагаемыми на неравенства, константы, стоящие в правой части, и/или коэффициенты целевой функции. Классификация типов задач и методов решения, рассмотренных нами, приведена в табл. 5.4. Таблица 5.4. Классификация типов задач и методов решения Тип Ограниче¬ Постоянные Коэффициенты задачи ния задачи правой части целевой функции 1. Максимизация Меньше Неотрица¬ Любые действи¬ или равно тельные тельные числа 2. Минимизация Больше или равно Любые действительные числа Неотрицательные Метод решения Использовать симплекс-метод с фиктивными переменными Сформулировать двойственную задачу и решить с помощью симплекс-метода с фик¬ тивными переменными В следующем разделе разработана обобщенная версия симплекс-метода, которая позволяет решать как задачи максимизации, так и задачи минимизации с любыми комбинациями знаков равенства и неравенства в ограничениях. Ответы к упражнениям 5.14. Максимизировать функцию Р = 12у^ + 16т/2 при условиях У1 + %У2 < 16, У1 4- У2 9, tyi +У2 21, 1/1, У2 > О- 5.15. Min С — 136 при xi = 4, #2 = 8, х^ = 0.
Глава 5. Линейные неравенства и линейное программирование 491 5.16. Двойственная задача. Максимизировать функцию Р = 2t/i + 7/2 при условиях у\ — у2 С 2, “2j/i + ?/2 3, У1, У2 0. Не имеет оптимального решения. 5.17. 600 шт. — с завода А в магазин II, 500 шт. — с завода Б в магазин I, 400 шт. — с завода Б в магазин II; величина общих затрат на перевозку составляет 6200 долл. Практикум 5.5 А В задачах 1-8 найдите транспонированную матрицу. 1. [-5 0 3 -1 8]. 1‘ 3- ’о • 4 [2 1—6 0 -1* |5 2 0 1 3 ‘ "1 2 —Г 0 2-7 8 0 1 4-1 3 2. [10-7 3 -2]. 9' . 5 4’ —4 • 0 Г 7 з -1. з [-6 1 0-9/ ’1-1 3 2 1-402 8. 4-5 6 1 -3 8 0 -1 2 7-31 В задачах 9 и 10 необходимо выполнить следующее. 1) Сформулировать двойственную задачу. 2) Записать исходную систему для двойственной задачи. 3) Записать исходную симплекс-таблицу для двойственной задачи и обозначить столбцы таблицы. 9. Минимизировать функцию С = 8xi + 9x2 при условиях xi 4- Зх2 > 4, 2а?1 + я2 > 5. Xi, Х2 0. 10. Минимизировать функцию С = 12xi 4- 5x2 при условиях 2xi + Х2 7, 3xi +#2^9, Xi, Х2 > 0.
492 Часть II. Конечная математика Ниже сформулированы задачи минимизации, соответствующая двойственная задача и окончательная симплекс-таблица, полученная при решении двойственной задачи. 1) Найдите оптимальное решение двойственной задачи. 2) Найдите оптимальное решение задачи минимизации. 11. Минимизировать функцию С = 21xi + 50x2 при условиях 2xi + 5x2 ^12, 3xi + 7х2 17, Xi, Х2 0. Максимизировать функцию Р = 12yi + 171/2 при условиях 2г/1 + 3?/2 ^21, 5i/i + 7у2 50, 2/1, У2 > 0. 2/1 2/2 Х1 Х2 Р ’ 0 1 5 -2 0 5 1 0 —7 3 0 3 0 0 1 2 1 121 12. Минимизировать функцию С = 16xi 4- 25x2 при условиях 3xi + 5x2 > 30, 2xi 4- 3x2 19, Xi, Х2 0. Максимизировать функцию Р = 301/1 4-191/2 при условиях 31/1 4- 21/2 16, 5i/i + 31/2 25, 2/1, 2/2 > 0. 2/1 2/2 Х1 Х2 р " 0 1 5 -3 0 5 1 0 -3 2 0 2 0 0 5 3 1 155 В задачах 13-20 необходимо выполнить следующее. 1) Поставить двойственную задачу. 2) Найти решение исходной задачи, применяя симплекс-метод к двойственной задаче. 13. Минимизировать функцию С = 9xi 4- 2хг при условиях 4xi + #2 13, 3xi + х2 > 12, Xi, Х2 > 0.
Глава 5. Линейные неравенства и линейное программирование 493 14. Минимизировать функцию С = xi 4- 4х2 при условиях #1 + 2X2 5, Xi + 3X2 6, Xi, Х2 0. 15. Минимизировать функцию С = 7xi 4- 12х2 при условиях 2д?1 4- Зх2 15, Xi 4- 2х2 8, xi, х2 > 0. 16. Минимизировать функцию С = 3xi 4- 5х2 при условиях 2xi + Зх2 > 7, xi 4- 2х2 4, xi, х2 0. 17. Минимизировать функцию С = llxi 4- 4х2 при условиях 2xi “И я2 > 8, —2x1 4- Зх2 > 4, xi, х2 > 0. 18. Минимизировать функцию С = 40xi + Юх2 при условиях 2xi + #2 12, 3xi —#2^3, xi, х2 > 0. 19. Минимизировать функцию С = 7xi 4- 9х2 при условиях — 3xi 4- я2 6, xi — 2х2 4, xi, х2 > 0. 20. Минимизировать функцию С = 10xi 4- 15х2 при условиях — 4xi + #2 12, 12xi ~ Зх2 > 10, xi, х2 > 0. Б Решите задачи линейного программирования 21-32, применяя симплекс-метод к двойственной задаче. 21. Минимизировать функцию С = 3xi 4- 9х2 при условиях 2xi + Х2 8, xi 4- 2x2 8, xi, х2 > 0.
494 Часть II. Конечная математика 22. Минимизировать функцию С = 2xi 4- при условиях Х\ 4- Ж2 > 8, Х\ 4- 2х2 4, Х1, Х2 0. 23. Минимизировать функцию С = 7xi 4- 5^2 при условиях xi 4- х2 > 4, xi — 2x2 —8, -2xi + Х2 -8, Xi, Х2 0. 24. Минимизировать функцию С = 10xi 4- 4хг при условиях 2xi + #2^6, xi — 4x2 —24, —8x1 4- 5x2 —24, Xi, Х2 0. 25. Минимизировать функцию С = 10xi 4- ЗОхг при условиях 2xi 4- #2 16, xi 4- %2 12, xi 4- 2х2 > 14, Xi, Х2 > 0. 26. Минимизировать функцию С = 40xi 4- 10x2 при условиях 3xi 4- Х2 > 24, xi 4- х2 > 16, Xi 4“ 4x2 30, xi, х2 0. 27. Минимизировать функцию С = 5xi 4- 7хг при условиях xi 4, xi 4- х2 8, Xi 4- 2x2 Ю, Xi, Х2 0. 28. Минимизировать функцию С = 4xi 4- 5x2 при условиях 2xi 4- Х2 12, + ^2 > 9, %2 4, Xi, Х2 0.
Глава 5. Линейные неравенства и линейное программирование 495 29. Минимизировать функцию С = 10xi + 7^2 + 12хз при условиях xi 4- Х2 4- 2хз > 7, 2xi + 2х2 + > 4, Xi, Х2, Хз 0. 30. Минимизировать функцию С = 14xi + 8x2 + 20хз при условиях Х1 + Х2 + Зхз > 6, 2X1 + х2 + Хз 9, Xi, Х2, Хз > 0. 31. Минимизировать функцию С = 5xi + 2хг + 2хз При условиях Х1 — 4X2 + Хз 6, -Х1 4- х2 — 2х3 4, Xi, Х2) Хз 0. 32. Минимизировать функцию С = 6x1 4- 8х2 4- Зхз при условиях - 3xi - 2x2 4- хз > 4, Х1 + х2 - Хз > 2, хь х2, х3 0. * 33. Задача минимизации содержит четыре переменные и два ограничения. Сколько переменных и ограничений содержится в двойственной задаче? *34. Задача минимизации содержит три переменные и пять ограничений. Сколько переменных и ограничений содержится в двойственной задаче? * 35. Сколько переменных и ограничений должно быть в исходной задаче, если необ¬ ходимо решить задачу минимизации с помощью применения геометрического метода к двойственной задаче? * 36. Сколько переменных и ограничений должно быть в исходной задаче, если необ¬ ходимо решить задачу минимизации с помощью применения геометрического метода к исходной задаче? В задачах 37-40 необходимо определить, можно ли задачу минимизации с заданными условиями свести К двойственной и решить симплекс-методом. При положительном ответе опишите, Какие преобразования необходимо сделать прежде, чем формулировать двойственную задачу. При отрицательном объясните, почему. * 37. Коэффициент целевой функции отрицателен. * 38. Коэффициент ограничения отрицателен. * 39. Ограничение задачи имеет вид “меньше или равно”. * 40. Ограничение задачи имеет отрицательную константу в правой части.
496 Часть II. Конечная математика В Решите задачи линейного программирования 41-44, применяя симплекс-метод к двойственной задаче. 41. Минимизировать функцию С — 16xi + 8^2 + 4х3 при условиях 3xi -I- 2х2 + 2х3 > 16, 4xi + 3x2 + хз > 14, 5xi + 3x2 + хз > 12, xi, х2, х3 0. 42. Минимизировать функцию С — 6x1 + 8x2 + 12х3 при условиях Х1 -|- 3X2 + Зх3 6, xi -I- 5х2 + 5х3 4, 2xi 4" 2x2 4“ Зхз 8, xi, х2, хз 0. 43. Минимизировать функцию С = 5xi + 4хг 4- 5х3 -I- 6x4 при условиях xi 4- Х2 < 12, хз + Х4 25, xi + хз 20, Х2 4- Х4 15, Xi, Х2, Хз, Х4 0. 44. Повторите решение задачи 43 с целевой функцией С = 4xi + 7x2 4- 5х3 + 6x4. Применение математики Постройте математическую модель в форме задачи линейного программирования. (Ответы приведены в конце книги.) Решите задачу, применяя симплекс-метод к двойственной задаче. Экономика и бизнес 45. Планирование производства. Компания по производству продуктов питания изготавливает обычное и элитное мороженое на трех заводах. За час работы завод в Седарбурге производит 20 галлонов обычного и 10 галлонов элитного мороженого, завод в Графтоне — 10 галлонов обычного и 20 галлонов элитного мороженого, а завод в Вест-Бенде — 20 галлонов обычного и 20 галлонов элитного мороженого. Час работы завода в Седабурге стоит 70 долл., в Графтоне — 75 долл, и в Вест-Бенде — 90 долл. а) Компании необходимо производить по крайней мере 300 галлонов обычного и 200 галлонов элитного мороженого ежедневно. Сколько часов должен работать ежедневно каждый завод, чтобы производить требуемое количество мороженого и минимизировать производственные затраты? Чему равна минимальная величина производственных затрат?
Глава 5. Линейные неравенства и линейное программирование 497 *б) Как изменятся производственный план и минимальная величина производственных затрат, если потребность в элитном мороженом возрастет до 300 галлонов ежедневно, а все остальные данные в п. а останутся прежними? *в) Решите п. б для варианта, когда потребность в элитном мороженом увеличивается до 400 галлонов ежедневно. 46. Горнодобывающая промышленность: производственное планирование. Компания имеет два рудника, на каждом из которых добываются три сорта руды. На руднике в Вест-Саммите за один час работы можно добыть две тонны руды низкого сорта, три тонны руды среднего сорта и одну тонну руды высшего сорта. На руднике в Нос-Ридже за один час работы можно добыть две тонны руды низкого сорта, одну Тонну руды среднего сорта и две тонны руды высшего сорта. Для удовлетворения существующих потребностей компании необходимо по крайней мере 100 тонн руды низкого сорта, 60 тонн среднего сорта и 80 тонн руды высшего сорта. Затраты на работу каждого рудника изменяются в зависимости от условий добычи руды. а) Сколько часов должен работать каждый рудник, чтобы удовлетворять минимальные Потребности в количестве руды и минимизировать производственные затраты, если час работы рудника в Вест-Саммите стоит 400 долл., а час работы рудника в Нос-Ридже — 600 долл.? Чему равна минимальная величина производственных затрат? *б) Как изменятся производственный план и минимальная величина производственных затрат, если час работы рудника в Вест-Саммите будет стоить 300 долл., час работы рудника в Нос-Ридже — 700 долл., а все остальные данные останутся прежними? *в) Повторите решение п. б, если час работы рудника в Вест-Саммите стоит 800 долл., а час работы рудника в Нос-Ридже — 200 долл. 47. Закупки. Компания Acme Micros торгует компьютерами, оснащенными дисководами для Носителей стандартной и повышенной емкости. Дисководы поставляются двумя другими компаниями — Associated Electronics и Digital Drives. Стоимость дисководов для носителей стандартной емкости в компании Associated Electronics составляет 250 долл., а повышенной — 350 долл. Стоимость дисководов Для носителей стандартной емкости в компании Digital Drives составляет 290 долл., а повышенной — 320 долл. Компания Associated Electronics может поставлять не больше 1000 дисководов для носителей стандартной и повышенной емкости в любой комбинации. Общая ежемесячная поставка Digital Drives не может превышать 2000 дисководов. Компании Acme Micros требуется ежемесячно по крайней мере 1200 дисководов для носителей стандартной и 1600 дисководов повышенной емкости. Сколько дисководов каждого типа должна заказать компания Acme Micros от каждого поставщика, чтобы удовлетворить свои ежемесячные потребности и минимизировать стоимость закупки? Чему равна минимальная стоимость закупки? 48. Транспортировка. Компания по производству продуктов питания хранит зерно в элеваторах, расположенных в Эймсе, штат Айова, и Бедфорде, штат Индиана. Ежемесячно зерно перевозится на перерабатывающие заводы в Колумбию, штат Миссури, и Дэнвилл, штат Иллинойс. Ежемесячные поставки (в тоннах) зерна
498 Часть II. Конечная математика на каждый элеватор, ежемесячные потребности (в тоннах) каждого перерабатывающего завода и затраты на транспортировку тонны зерна даны в таблице. Определите план перевозок, минимизирующий затраты на транспортировку зерна. Чему равна минимальная величина этих затрат? Стоимость транспортировки, долл./т Объем поставок, т Колумбия Дэнвилл Эймс 22 38 700 Бедфорд 46 24 500 Объем переработки, т 400 600 Биологические науки 49. Диета. Специалисту по питанию в больнице нужно разработать специальную диету, использующую три блюда — А, Б и В. Каждая унция блюда А содержит 20 единиц кальция, 10 единиц железа, 10 единиц витамина А и 20 единиц холестерина. Каждая унция блюда Б содержит 10 единиц кальция, 10 единиц железа, 15 единиц витамина А и 24 единицы холестерина. Каждая унция блюда В содержит 10 единиц кальция, 10 единиц железа, 10 единиц витамина А и 18 единиц холестерина. Сколько унций каждого блюда должно быть использовано, чтобы удовлетворить минимальные потребности и в то же время минимизировать потребление холестерина, если минимальные суточные потребности составляют 300 единиц кальция, 200 единиц железа и 240 единиц витамина А? Чему равно минимальное потребление холестерина? 50. Удобрение растений. Фермер может купить три вида удобрений — смесь Л, смесь Б и смесь В. Каждый кубический ярд смеси А содержит 20 фунтов фосфорных, 10 фунтов азотных и 10 фунтов калийных удобрений. Каждый кубический ярд смеси Б содержит 10 фунтов фосфорных, 10 фунтов азотных и 15 фунтов калийных удобрений. Каждый кубический ярд смеси В содержит 20 фунтов фосфорных, 20 фунтов азотных и 5 фунтов калийных удобрений. Минимальные ежемесячные потребности составляют 480 фунтов фосфорных, 320 фунтов азотных и 225 фунтов калийных удобрений. Сколько кубических ярдов каждой смеси должен смешать фермер, чтобы удовлетворить минимальные ежемесячные потребности при минимальной стоимости, если один кубический ярд смеси А стоит 30 долл., смеси Б — 36 долл, и смеси В — 39 долл.? Чему равна минимальная стоимость? Социальные науки 51. Образование: распределение ресурсов. В неком районе города расположены четыре школы: две переполненные и две с вакантными местами. Чтобы сбалансировать нагрузку на образовательные учреждения, школьный совет решил перевозить учеников из переполненных школ в незаполненные. В средней школе Северного округа на 300, а в средней школе Южного округа на 500 учеников больше, чем они рассчитаны. Средняя школа Центрального округа может принять дополнительно 400 учеников, а средняя школа Вашингтона — 500. Еженедельные затраты на перевозку ученика из Северного округа в Центральный
Глава 5. Линейные неравенства и линейное программирование 499 составляет 5 долл., из Северного округа в Вашингтон — 2 долл., из Северного округа в Центральный — 3 долл., а из Южного округа в Вашингтон — 4 долл. Определите количество учеников, которое следует перевозить из каждой переполненной школы в незаполненную, чтобы сбалансировать количество учеников и минимизировать затраты на их перевозку. Чему равна минимальная величина этих затрат? 52. Образование: распределение ресурсов. Решите задачу 51, если еженедельные затраты на Перевозку ученика из Северного округа в Вашингтон составляют 7 долл., а вся остальная информация остается той же. 5.6. Задачи максимизации и минимизации со смешанными ограничениями ■ Введение в М-метод ■ Общие положения М-метода ■ Использование М-метода для решения задач минимизации ■ Резюме ■ Сложная практическая задача: приложение нефтепереработки В предыдущих двух разделах было дано решение как задачи максимизации, так и задачи минимизации, но с более строгими ограничениями на неравенства, константы в правой части и/или коэффициенты целевой функции (см. табл. 5.4 предыдущего раздела). В этом разделе будет представлена обобщенная версия симплекс-метода, которая позволяет решать как задачи максимизации, так и задачи минимизации с любыми комбинациями знаков “меньше или равно”, “больше или равно” и “равно” в ограничениях. Единственное требование состоит в том, чтобы каждое ограничение задачи имело неотрицательную константу в правой части. (Как будет показано, это ограничение легко выполняется.) Введение в М-метод Опишем М-метод На примере простой задачи максимизации со смешанными ограничениями. Ключевые части метода будут затем обобщены для более сложных задач. Рассмотрим следующую задачу. Максимизировать функцию Р = 2xi + х2 при условиях Х1 + Ю, -Х\ + х2 2, #1, х2 0. (5.18)
500 Часть II. Конечная математика Чтобы образовать равенство из первого неравенства, введем, как ранее, фиктивную переменную «1. xi + + «1 = Ю. Как получить равенство из второго неравенства? Вводим вторую переменную «2 и вычитаем ее из левой части. -Х1 4- х2 - «2 = 2. Переменная s2 называется избыточной, потому что она представляет собой величину (излишек), на которую левая часть неравенства превышает правую. Теперь выразим задачу линейного программирования (5.18) в виде системы уравнений. ^1 + Я2 + $1 = Р, —Xi 4- х2 — s2 =2, (5.19) —2x1 — х2 4- Р = 0, xi, х2, «1, s2 0. Можно показать, что базисное решение системы (5.19) не является допустимым, если какая-либо из переменных (исключая Р) отрицательна. Таким образом, требуется, чтобы избыточная переменная удовлетворяла условию неотрицательности. Базисным решением, которое вычисляется, если положить небазисные переменные х\ и х2 равными 0, является следующий набор чисел. xi = 0, х2 = 0, «1 = 10, s2 = —2, Р = 0. Однако это базисное решение не является допустимым, поскольку избыточная переменная «2 отрицательна (что является нарушением требований неотрицательности всех переменных, исключая Р). Симплекс-метод работает только в том случае, когда базисное решение для таблицы является допустимым, так что поставленную задачу невозможно решить, просто записывая таблицу для задачи (5.19) и проводя итерацию симплекс- процесса. Задание 5.11. Чтобы убедиться в том, что симплекс-метод не будет работать в случае, когда базисное решение не является допустимым, запишите таблицу для системы (5.19) и выполните операцию поиска опорного плана. Является ли новое базисное решение допустимым? Можно ли выбрать другой основной элемент? ■ Чтобы применить симплекс-метод к задачам со смешанными переменными, обратимся к остроумному приему, который использует искусственную переменную. Эта переменная не имеет физического содержания в исходной задаче (что объясняется использованием слова “искусственный”) и введена исключительно с целью получения базисного допустимого решения так, чтобы можно было применить симплекс-метод. Искусственной переменной является переменная, которая введена в каждое равенство, содержащее избыточную переменную. Как и ранее, чтобы убедиться, что рассматриваются только допустимые базисные решения, требуется, чтобы искусственная переменная была неотрицательной. (Как будет показано позднее, искусственные переменные используются также для того, чтобы усилить ограничения задачи.)
Глава 5. Линейные неравенства и линейное программирование 501 Возвращаясь к рассматриваемой задаче, введем искусственную переменную а\ в равенство, включающее избыточную переменную $2. -я?1 + Х2 - 52 + 01 = 2. Чтобы предотвратить ситуацию, когда искусственная переменная становится частью оптимального решения, в целевую функцию вводится очень большой “штраф”. Этот штраф создается путем выбора положительной константы М настолько большой, что в любом окончательном оптимальном решении исходной задачи искусственная переменная вынужденно обращается в нуль. (Поскольку в компьютерных решениях константа М может быть сделана произвольно большой, константа М часто выбиралась максимально большим числом, которое Допускал компьютер!) Затем прибавим слагаемое —Ма^ к целевой функции. Р = 2xi + #2 - Ма^. Получим новую задачу, которая называется модифицированной. Максимизировать функцию Р = 2xi + х2 — Ма\ при условиях Я1 + Х2 + 51 = 10, -xi 4- х2 - s2 -F ai = 2, Xl, Х2, Si, S2, с&1 0. (5.20) Исходная система для модифицированной задачи (5.20) выглядит следующим образом. #1 4” х2 4“ 51 = 10, -Xi + х2 —2X1 ~ — 52 4“ G1 — 2, + Маг + Р = 0, Xi, Х2, 51, S2, <2-1 0. (5.21) Запишем расширенную матрицу коэффициентов для системы (5.21), которую назовем предварительной симплекс'-таблицей для модифицированной задачи. (Причина, по которой эта матрица называется “предварительной”, а не “исходной”, скоро прояснится.) Х1 х2 51 52 Я1 р 1 1 1 0 0 0 10 -1 1 0 -1 1 0 2 -2 -1 0 0 м 1 0 (5.22) Требования к исходной симплекс-таблице Для того чтобы таблица системы рассматривалась как исходная симплекс-таблица, она должна удовлетворять следующим двум требованиям.
502 Часть II. Конечная математика 1. Требуемое количество базисных переменных можно выбрать с помощью процесса, описанного в разделе 5.4. Иначе говоря, переменная может быть выбрана в качестве базисной только тогда, когда она соответствует столбцу, имеющему только один ненулевой элемент, который не находится в той же строке, что и ненулевой элемент в столбце другой базисной переменной. Оставшиеся переменные выбираются в качестве небазисных переменных, которые при определении базисного решения следует положить равными нулю. 2. Базисное решение, которое вычисляется, если положить небазисные переменные равными нулю, является допустимым. Матрица (5.22) удовлетворяет требованию первой исходной симплекс-таблицы, поскольку величины si, «2 и Р могут быть выбраны в качестве базисных переменных в соответствии со сформулированным критерием. (Не все предварительные симплекс- таблицы удовлетворяют первому требованию; см. пример 5.18.) Однако матрица (5.22) не удовлетворяет второму требованию к исходной симплекс-таблице, поскольку базисное решение не является допустимым («2 = —2). Для того чтобы использовать симплекс-метод, необходимо сначала выполнить операции над строками и преобразовать матрицу (5.22) в эквивалентную матрицу, удовлетворяющую обоим требованиям к исходной симплекс-таблице. Отметим, что это преобразование не является итерацией симплекс-процесса. Чтобы понять, как действовать дальше, отметим, что в матрице (5.21) число — 1 в столбце «2 находится в той же строке, что и единица в столбце а±. Это не случайно! Искусственная переменная а± была введена так, что это непременно должно было случиться. Если исключить величину М, стоящую внизу столбца ai, небазисная переменная fli станет базисной, а интересующая нас базисная переменная $2 — небазисной. Таким образом, мы исключаем М из столбца ai, выполняя такие операции над строками: Как видим, в последней матрице базисными переменными являются ai и Р. Базисным решением, которое вычисляется, если положить небазисные переменные a?i, Х2 и $2 равными нулю, является следующий набор чисел. xi = 0, Х2 = 0, si = 10, 32 = 0, «1=2, Р = —2М. Базисное решение является допустимым (напомним, что величина Р может быть отрицательной), и оба требования к исходной симплекс-таблице удовлетворяются. Теперь можем начинать итерации симплекс-метода. Основной столбец определяется с помощью наибольшего по абсолютной величине отрицательного индикатора в нижней строке таблицы. Поскольку М — положительное число, то число — М — 1, конечно, является отрицательным индикатором. Что можно
Глава 5. Линейные неравенства и линейное программирование 503 сказать об индикаторе М — 2? Напомним, что константа М является очень большим положительным числом. Будем предполагать, что число М является настолько большим, что любое выражение вида М — k является положительным. Таким образом, единственным отрицательным индикатором в нижней строке является — М — 1. Х1 Х2 Si s2 ai Р 1 1 10 0 0 10 ‘ = 10 Ведущая строка —*■ -1 ф 0-110 2 Ны, II К5 М-2 -М-1 0 М 0 1 —2М Ведущий столбец Определив ведущий элемент, начнем поиск опорного плана. Х2 1 Ф S1 1 0 S2 0 -1 ai 0 1 р 0 0 10 ' 2 —R2 + Ki —► Я1 *1 ai 1 -1 Р _ М-2 -М- 1 0 М 0 1 —2М _ (TVf + 1)Яг + -Rs —► Яз «1 <2> 0 1 1 -1 0 8 ‘ —► Я1 ~Х2 -1 1 0 -1 1 0 2 Р -3 0 0 -1 м + 1 11 2 = Ф 0 1 2 1 2 _ 1 2 0 4 ' -1 1 0 -1 1 0 2 Ri + R2 —► R2 -3 0 0 -1 М + 1 1 2 _ ЗЯ1 + Яз —► Яз хг 1 0 1 2 1 2 1 2 0 4 ' ~^2 0 1 1 2 1 2 1 2 0 6 Р 0 0 2 2 М - 1 1 14 _ Поскольку все индикаторы в последней строке являются неотрицательными (число М — | является Неотрицательным, поскольку М — очень большая положительная константа), можем остановиться и записать оптимальное решение. МахР 14 при = 4, #2 = 6, $1 = 0, «2 = 0, «1 = 0. Это решение является оптимальным решением модифицированной задачи (5.20). Как оно связано с исходной задачей (5.19)? Поскольку в этом решении ai = 0, решение =4, Х2 — 6, $i = 0, $2 = 0) Р = 14, (5.23) конечно, является допустимым решением для системы (5.19). (Это можно проверить с помощью прямой подстановки решения в систему (5.19).) Неожиданно оказывается, что решение (5.23) является оптимальным решением исходной задачи. Чтобы убедиться в этом, предположим, что можно найти допустимые значения xi, Х2, $i и «2, которые удовлетворяют исходной системе (5.19) и обеспечивают выполнение неравенства Р > 14. Это противоречит тому факту, что Р = 14 является максимальным значением величины Р в модифицированной задаче. Таким образом, решение (5.23) является оптимальным решением исходной задачи.
504 Часть II. Конечная математика Как иллюстрирует этот пример, если число а± = 0 является оптимальным решением модифицированной задачи, то удаление переменной дает оптимальное решение исходной задачи. Что происходит, если в оптимальном решении модифицированной задачи выполняется неравенство ai 0? В этом случае можно показать, что исходная задача не имеет оптимального решения, поскольку множество ее допустимых решений пусто. Задание 5.12. Изобразите допустимую область для задачи (5.18), нанесите координаты (xi, Х2) базисного решения для каждой симплекс-таблицы в решении этой задачи и проиллюстрируйте путь к оптимальному решению. ■ В больших задачах каждое ограничение “больше или равно” требует введения избыточной и искусственной переменных. Если одно из ограничений задачи является равенством, а не неравенством, нет необходимости вводить фиктивную или избыточную переменную. Однако, если в систему ограничений входит равенство, необходимо ввести другую искусственную переменную, поскольку в исходном базисном решении переменные часто оказываются равными нулю, и равенство может оказаться нарушенным (см. пример 5.18). Наконец, каждая искусственная переменная также должна быть включена в целевую функцию модифицированной задачи. Для всех искусственных переменных можно использовать одну и ту же константу М. В силу той роли, которую играет постоянная М, этот метод часто называют Af-методом. Общие положения М-метода Резюмируем ключевые этапы М-метода и применим их для решения нескольких задач. М-метод: введение фиктивных, избыточных и искусственных переменных для получения модифицированной задачи Этап 1. Если какое-то ограничение в правой части содержит отрицательную константу, то, чтобы получить ограничения с неотрицательной константой, умножим обе части на число —1. (Если ограничение представляет собой неравенство, это поменяет его направление.) Этап 2. Введем фиктивную переменную в каждом ограничении “меньше или равно”. Этап 3. Введем избыточную и искусственную переменные в каждом ограничении “больше или равно”. Этап 4. Введем искусственную переменную в каждом ограничении, представленном в виде равенства. Этап 5. Для каждой искусственной переменной аг к целевой функции прибавим —Мсц. Для всех искусственных переменных используем одну и ту же константу М. Пример 5.18 (Поиск модифицированной задачи). Сформулируйте модифицированную задачу для следующей задачи линейного программирования. (Не пытайтесь ее решать.) Максимизировать функцию Р = 2xi + 5^2 + Зхз
Глава 5. Линейные неравенства и линейное программирование 505 При условиях Х1 + 2X2 ~ Х3 7, -xi + Х2 — 2х3 -5, XI + 4x2 + Зя?3 1, 2гг 1 — #2 + 4х3 = 6, xi, Х2) хз 0. Решение. Во-первых, чтобы изменить знак числа —5, умножим второе ограничение на —1. ; - 1 • (-Х1 + х2 - 2х3) -1 • (-5) ; #1 “ #2 + 2х3 5. Во-вторых, введем фиктивные, избыточные и искусственные переменные в соответствии с правилами, описанными выше. xi + 2x2 - х3 + si = 7, Х\ — Х2 4- 2^3 — $2 + G1 =5, £1+4x2+ 3яз - s3 + а2 =1, 2xi - #2 + 4хз + а3 = 6. Наконец прибавим к Целевой функции числа —Mai, —Ма2 и — Ма$. Р = 2xi + 5x2 + Зх3 - Mai — Ма2 — Ма$. Получим следующую модифицированную задачу. Максимизировать функцию Р — 2xi + 5х2 + Зх3 — Mai — Ма2 — Маз при условиях Xi + 2хг — + si = 7, #1 — $2 + 2х3 — $2 + ai =5, Xi + 4x2 4” Зхз — s3 + fl2 =1? Xi - Х2 + 4х3 + а3 = 6, Xi, Х2, Хз, «1, S2, S3, G1, &2, 0. ■ Упражнение 5.18. Повторите решение примера 5.18 для следующей задачи. Максимизировать функцию Р = 3xi - 2хг 4- х3 при условиях xi — 2X2 + хз 5, —xi — 3x2 + 4хз —10, 2xi 4- 4х2 4- 5х3 20, 3xi — Х2 — Хз = —15, Xi, Х2, Хз 0. ■ Теперь перечислим ключевые этапы M-метода. Этапы и замечания, сделанные ниже, основаны на множестве важных теорем, которые принимаются без доказательства.
506 Часть II. Конечная математика В частности, этап 2 основан на том факте (за исключением некоторых вырожденных случаев, не рассматриваемых здесь), что если модифицированная задача линейного программирования имеет оптимальное решение, то предварительная симплекс-таблица будет преобразована в исходную симплекс-таблицу исключением константы М из столбцов, соответствующих искусственным переменным в предварительной симплекс-таблице. Получив исходную симплекс-таблицу, можем начинать поиск оптимального плана. М-метод: решение задачи Этап 1. Образуем предварительную симплекс-таблицу для модифицированной задачи. Этап 2. Выполняем операции над строками, чтобы исключить константу М из нижней строки предварительной симплекс-таблицы в столбцах, соответствующих искусственным переменным. Полученная в результате таблица является исходной симплекс-таблицей. Этап 3. Решаем модифицированную задачу с помощью применения симплекс-метода к исходной симплекс-таблице, найденной на этапе 2. Этап 4. Связываем оптимальное решение модифицированной задачи с исходной задачей. 1) Если модифицированная задача не имеет оптимального решения, исходная задача не имеет оптимального решения. 2) Если все искусственные переменные в оптимальном решении модифицированной задачи равны нулю, то, чтобы найти оптимальное решение исходной задачи, удаляем искусственные переменные. 3) Если какая-то искусственная переменная в оптимальном решении модифицированной задачи не равна нулю, исходная задача не имеет оптимального решения. Пример 5.19 (Использование TVf-метода). Решите следующую задачу линейного программирования, используя М-метод. Максимизировать функцию Р = х± — х2 + Зхз при условиях xi 4- х2 < 20, xi + х3 = 5, хг 4- х3 Ю, ХЬ Х2, х3 0. Решение. Сформулируем модифицированную задачу. Максимизировать функцию Р = xi - х2 4- Зя?з — Mai — Ма2 при условиях xi 4- х2 + .s'i = 20, xi + хз 4- ai = 5, х2 4- хз - s2 4- а2 = 10, Х1, Х2) Хз, 51, 52, Ц1, а2 0.
Глава 5. Линейные неравенства и линейное программирование 507 Запишем предварительную симплекс-таблицу для модифицированной задачи и найдем исходную симплекс-таблицу, исключая М из столбцов, соответствующих искусственным переменным. Х1 Х2 Хз S1 ai 52 а2 р 1 1 0 1 0 0 0 0 20 Исключаем М 1 0 1 0 1 0 0 0 5 из столбца ai 0 1 1 0 0 -1 1 0 10 -1 1 -3 б м б м 1 0 —MR2 + -R4 —► R4 1 1 0 1 0 0 0 0 20 Исключаем М 1 0 1 0 1 0 0 0 5 из столбца аз 0 1 1 0 0 -1 1 0 10 -М-1 1 -М-3 0 0 б м 1 -5М —-|“ К4 —► R& 1 1 0 1 0 0 0 0 20 1 0 1 0 1 0 0 0 5 0 1 1 0 0 -1 1 0 1 -М-1 -М + 1 -2М-3 0 0 м 0 1 -15М Последняя матрица свидетельствует о том, что базисными переменными являются 5i, ai, аг и Р. Базисным решением, найденным в том случае, если небазисные переменные х±, Х2, хз и «2 полагаются равными нулю, является следующий набор чисел. х\ = 0, Х2 = 0, Хз = 0, «1 = 20, а\ =5, «2 = 0, а2 = 10, Р = — 15М. Базисное решение является допустимым, и оба требования относительно исходной сим- плекс-таблицы удовлетворяются. Теперь можно начинать итерацию симплекс-процесса для определения оптимального решения. Х2 яз 51 О1 52 02 р «1 г 1 1 0 1 0 0 0 0 20 ai 1 0 Ф 0 1 0 0 0 5 02 0 1 1 0 0 -1 1 0 10 —Яз+Яз —► Яз Р _ -М-1 -М+1 -2М-3 0 0 м 0 1 -15М (2М+3) Я2 + Ял-♦ Я4 «1 1 1 0 1 0 0 0 0 20 —Яз 4- Я1 —► Я1 52 1 0 1 0 1 0 0 0 5 02 -1 ф 0 0 -1 -1 1 0 5 Р М+2 -М+1 0 0 2М+3 м 0 1 -5М+15 _ (М-1)Яз + Я4->Я4 51 2 0 0 1 1 1 -1 0 15 53 1 0 1 0 1 0 0 0 5 х2 -1 1 0 0 -1 -1 1 0 5 Р 3 0 0 0 М+4 1 М-1 1 10 Поскольку нижняя строка не имеет отрицательных индикаторов, можно остановиться и записать оптимальное решение модифицированной задачи. xi =0, Х2 = 5, Хз = 5, si = 15, ai =0, «2 = 0, а2 = 0, Р = 10. Поскольку ai = 0 и аз = 0, решение исходной задачи выглядит следующим образом. МахР = 10 при xi = 0, Х2 = 5, хз = 5. а
508 Часть II. Конечная математика Упражнение 5.19. Решите следующую задачу линейного программирования, используя М-метод. Максимизировать функцию Р = х\ + 4х2 + 2хз при условиях х2 -I- хз < 4, Х\ — Хз = 6, Х1 — Х2 — Хз 1, Xi, х2, хз 0. Пример 5.20 (Использование Af-метода). Решите следующую задачу линейного программирования, используя М-метод. Максимизировать функцию Р = 3xi + 5х2 при условиях 2X1 + Х2 4, xi + 2x2 10? xi, х2 0. Решение. Вводя фиктивные, избыточные и искусственные переменные, получим моди- фицированную задачу. 2xi + х2 + $1 = 4, Х1 4- 2х2 — s2 4- а 1 = ю, Модифицированная задача —3x1 - 5х2 + М а 1+P = 0. Предварительная симплекс-таблица Х1 2 х2 1 51 1 52 0 ai 0 Р 0 4 Исключаем М 1 -3 2 -5 0 0 -1 0 1 м 0 1 10 0 из столбца ai —АГЯ2 “Ь Рз —► Рз Исходная симплекс-таблица «1 2 ф 1 0 0 0 4 Поиск оптимального плана ~(21 р 1 -М- 2 3 -2М-5 0 0 -1 м 1 0 0 1 10 -ЮМ —2 Л1 -|“ Р2 —► Р2 (2ЛЦ-5)Н14-Яз->Нз х2 2 1 1 0 0 0 4 р -3 2>М 4* 0 7 0 —2 2М + 5 -1 м 1 0 0 1 2 -2М + 20 Оптимальным решением модифицированной задачи является следующий набор чисел. xi = 0, х2 = 4, «1 = 0, S2 = 0, <21 = 2, Р = —2М + 20. Поскольку ai / 0, исходная задача не имеет оптимального решения. Из рис. 5.25 видно, что область допустимых решений исходной задачи пуста. ■
Глава 5. Линейные неравенства и линейное программирование 509 А()В = 0 Рис. 5.25. Область допустимых решений Упражнение 5.20. Решите следующую задачу линейного программирования, используя Л/-метод. Максимизировать функцию Р = 3xi + 2х2 при условиях xi -I- 5^2 С 5, 2xi + х2 12, xi, х2 0. Использование М-метода для решения задач минимзации М-метод может использоваться и для решения задач минимизации. Для минимизации целевой функции необходимо максимизировать ее отрицательную величину. На рис. 5.26 показано, что минимальное значение функции f достигается в той же точке, что и максимальное значение функции —Более того, если т — минимальное значение /, то —т — максимальное значение функции — /, и наоборот. Таким образом, можно найти минимальное значение функции /, находя максимальное значение — /, а затем изменяя знак максимального значения. Рис. 5.26. Максимум и минимум функций
510 Часть II. Конечная математика К Пример 5.21 (Планирование производства: задача минимизации.) Маленькая ювелирная фирма нанимает высококвалифицированного огранщика драгоценных камней. Фирма хочет использовать его труд по крайней мере в течение шести часов ежедневно. С другой стороны, шлифовальные станки могут быть использованы в любом количестве до 10 ч ежедневно. Фирма специализируется по трем видам драгоценных камней А, Б и В. Время, затрачиваемое на огранку и шлифовку, а также стоимость драгоценных камней приведены в таблице. Сколько драгоценных камней каждого вида следует производить ежедневно, чтобы минимизировать стоимость полученных украшений? Чему равна минимальная стоимость? Вид драгоценных камней А Б в Огранка, ч 1 1 1 Шлифовка, ч 2 1 2 Стоимость, долл. 30 30 10 Решение. Поскольку необходимо решить, сколько ювелирных камней каждого типа нужно производить ежедневно, введем следующие переменные. xi — количество ювелирных камней вида А, производимое ежедневно, Х2 — количество ювелирных камней вида Б, производимое ежедневно, хз — количество ювелирных камней вида В, производимое ежедневно. Сформулируем модель. Минимизировать функцию С = 30xi 4- ЗОяг + Юхз Целевая функция при условиях xi + #2 + #з 6, 2xi + ^2 + 2х3 10, Xi, Х2, Хз 0. Ограничение задачи Ограничение задачи Условия неотрицательности Превращаем эту задачу в задачу максимизации, полагая Р = —С = —30x1 — 30x2 — 10х3. Таким образом, получим следующую задачу. Максимизировать функцию Р = — 30xi — 30x2 — Юхз при условиях Xi -I- Х2 -Ь Х3 6, 2xi 4- х2 4- 2х3 10, X1, Х2, хз 0» Min С = — Мах Р. Чтобы решить ее, поставим модифицированную задачу. Xi + Х2 + Хз — $i + fli = 6, 2xi 4~ Х2 И- 2хз + $2 = Ю, 30xi 4" 30x2 4- Юхз 4- Ма\ 4- Р — 0, Xi, Х2, Хз, 31, 32, di 0.
Глава 5. Линейные неравенства и линейное программирование 511 2?1 Х2 из 31 ai S2 Р 1 1 1 -1 1 0 0 6 Исключим М 2 1 2 0 0 1 0 10 из столбца ai 30 30 10 0 М 0 1 0 —+ 7?з —►Яз Начинаем поиск опорного плана. Предположим, что число М настолько велико, что числа М — 30 и — М + 10 являются отрицательными. ai ~$2 1 2 1 1 1 -1 0 1 0 0 1 0 0 6 10 |Я2—Я2 Р -м+зо -м+зо -М+10 м 0 0 1 -6М 1 1 1 1 2 1 ф -1 0 1 0 0 1 2 0 0 6 5 —R2 + Ri —► Ri _ —М-1-30 -М+30 -м+ю м 0 0 1 -6М (М-10)Н2 + Яз ai ~хз 0 1 (D 1 0 1 -1 0 1 0 1 1 2 0 0 1 5 2К1— Р 20 -|М+25 0 м 0 |М-5 1 -М-50 0 1 (D 1 0 1 -2 0 2 0 -1 1 2 0 0 2 5 — 1Я1 + Я2 —► Я2 20 -^М+25 0 м 0 ^М-5 1 —М —50 _ (|М-25)Я1 + Яз-> Яз XI 0 1 1 0 0 1 —2 0 2 -1 -1 1 0 0 2 4 р 20 0 0 50 М-50 20 1 -100 Нижняя строка не имеет отрицательных индикаторов, поэтому оптимальным решением модифицированной задачи является следующий набор чисел. xi =0, Х2 = 2, хз =4, Si = 0, czi=O, $2 = 0, Р = -100. Поскольку ai = 0, удаление переменной ai дает оптимальное решение исходной задачи максимизации, а также задачи минимизации. Min С = — МахР — —(—100) = 100 при х± =0, Х2 — 2, хз = 4. Таким образом, минимальная стоимость ювелирных украшений, равная 100 долл., будет достигнута, если ежедневно производить два камня типа Я, четыре камня типа Б и не производить камни типа В, ■ у - Упражнение 5.21. Повторите пример 5.21, если огранщик камней должен работать по крайней мере в течение 8 часов ежедневно, а все остальные данные остаются прежними. ■ Резюме М-метод может быть использован для решения любой задачи минимизации, включая те, которые могут быть решены двойственным методом. (Отметим, что пример 5.21 мог
512 Часть II. Конечная математика бы быть решен двойственным методом.) Оба метода решения задачи минимизации важны. В практикуме 5.6 большинство задач минимизации рекомендуется решать с помощью М- метода, чтобы достичь его более глубокого освоения. Если метод решения не определен, двойственный метод обычно более прост. Правильно выбрать метод решения задачи поможет табл. 5.5. Таблица 5.5. Классификация задачи и методов Тип задачи Ограничения Правая часть ограничений Коэффициенты целевой функции Метод решения 1. Мак- Меньше Неотрицатель- Любые действи- Использование симплекс- сими- зация или равно ные тельные числа метода с фиктивными переменными 2. Минимизация Больше или равно Любые действительные числа Неотрицательные Составление двойственной задачи и решение с помощью предыдущего метода 3. Мак¬ Смешан¬ Неотрицатель¬ Любые действи¬ Формирование моди¬ сими¬ зация ные (^, =) ные тельные числа фицированной задачи с фиктивными, избыточными и искусственными переменными и решение с помощью М-метода 4. Ми¬ Смешан¬ Неотрицатель¬ Любые действи¬ Максимизация функции, ними¬ зация ные (^, =) ные тельные числа отрицательной по отношению к целевой функции с помощью предыдущего метода Сложная практическая задача: приложение нефтепереработки Вплоть до настоящего момента все рассматриваемые задачи могли быть решены вручную. Однако действительное значение симплекс-метода состоит в его способ¬ ности решать задачи с большим количеством переменных и ограничений, в которых поиск опорного плана выполняется с помощью компьютера. В качестве примера рассмотрим задачу, которая требует применения компьютера.
Глава 5. Линейные неравенства и линейное программирование 513 Пример 5.22 (Производство бензина). Нефтеперерабатывающий завод производит бензин двух сортов, обычного и высшего, смешивая в разных пропорциях два компонента: А и Б. Компонент А имеет октановое число 90 и стоит 28 долл, за баррель. Компонент Б имеет октановое число 110 и стоит 32 долл, за баррель. Октановое число обычного бензина должно быть не ниже 95, а октановое число бензина высшего сорта должно быть не ниже 105. Обычный бензин продается по 34 долл., а бензин высшего сорта — по 40 долл, за баррель. В настоящий момент компания имеет 30 000 баррелей компонента А и 20 000 баррелей компонента Б. Она имеет также заказы на 20 000 баррелей обычного и 10000 баррелей бензина высшего сорта, которые должны быть выполнены. Предполагая, что весь произведенный бензин может быть продан, определите максимальную возможную прибыль. Решение. Эта задача аналогична транспортной задаче предыдущего раздела. Иначе говоря, чтобы максимизировать прибыль, требуется решить, сколько каждого компонента следует использовать для производства каждого сорта бензина. Введем следующие переменные. — количество баррелей компонента А, используемых в обычном бензине, Х2 — количество баррелей компонента Я, используемых в бензине высшего сорта, а?з — количество баррелей компонента Б, используемых в обычном бензине х± — количество баррелей компонента Б, используемых в бензине высшего сорта. Представим исходные данные в табл. 5.6. Таблица 5.6. Исходные данные Компонент Октановое число Стоимость, долл. Объем запасов, баррелей А 90 28 30000 Б 110 32 20000 Минимальное Отпускная Объем заказов, Сорт октановое число цена, долл. баррелей Обычный 95 34 20000 Высший 105 40 10000 Общее количество компонента А, используемого в смесях, равно xi + х%. Оно не должно превышать объем имеющихся поставок. Таким образом, одним из ограничений является следующее неравенство. £1 + #2 30000. Соответствующим ограничением для компонента Б является неравенство £з + ^4 20 000. Производимые объемы производства обычного бензина и бензина высшего сорта должны быть достаточными для удовлетворения существующих заказов. 20000, Ограничение на производство обычного бензина a?2 + 10 000. Ограничение на производство бензина высшего сорта
514 Часть II. Конечная математика Теперь рассмотрим октановое число. Октановое число смеси представляет собой просто среднее пропорциональное октановых чисел компонентов. Таким образом, октановое число обычного бензина определяется по следующей формуле. Х1 хз Октановое число = 90 F 110 , Xi + Хз Xi + х3 где xi/(xi -I- хз) представляет собой процентное соотношение компонента А, используемого в обычном бензине, а хз/(х1 + хз) — процентное соотношение компонента Б. Окончательное октановое число обычного бензина должно составлять по крайней мере 95. Итак, получаем следующие ограничения. Х1 Хз 90 1- 110 95, Умножаем на + ж3 4- Хз + %з 90xi + 110хз 95 (xi 4- хз). Приводим подобные члены 0 5xi — 15хз. Ограничение на октановое число обычного бензина Для бензина высшего сорта должны выполняться следующие ограничения. 90 Х2 + 110 — — 105, Х2 + Х4 Х2 + Х4 90X2 + 110X4 (Xi 4- Х4) , 0 15x2 — 5x4. Ограничение на октановое число бензина высшего сорта Стоимость используемых компонентов вычисляется по следующей формуле. С = 28(xi + х2) 4- З2(х3 + х4). Доход от продажи всего бензина определяется так. R = 34(xi 4- хз) 4- 40(х2 4- х4). Прибыль компании определяется следующей зависимостью. P = R-C = = 34(xi 4- х3) + 40(х2 4- х4) - 28(xi 4- х2) - З2(х3 4- х4) = = (34 - 28)xi + (40 - 28)х2 + (34 - З2)х3 + (40 - 32)х4 = = 6x1 4- 12x2 4- 2хз 4- 8x4. Для того чтобы найти максимальную прибыль, необходимо решить следующую задачу линейного программирования. Максимизировать функцию при условиях Х1 4“ Хз 4- £1 4- х3 Р — 6x1 4- 12x2 4- 2хз 4- 8x4 Прибыль 30 000, Объем запасов компонента А Х4 20 000, Объем запасов компонента Б 20 000, Объем производства обычного бензина
Глава 5. Линейные неравенства и линейное программирование 515 Х2 + #4 > 10000, 5я?1 — 15жз < 0, 15я?2 — 5х4 0, X1, ^3, 3?4 0. Объем производства бензина высшего сорта Октановое число обычного бензина Октановое число бензина высшего сорта WWW Применим компьютерную технологию решения этой задачи. Для этого можно было бы использовать процессор электронных таблиц, программное обеспечение Explorations in Finite Mathematics или один из многочисленных онлайн-сервисов. Нами была выбрана утилита из графического калькулятора TI-86, разработанного компанией “Texas Instruments”. Эта программа, написанная Скоттом Кэмпбеллом (Scott Campbell), основана на М-методе, хотя исходная таблица, используемая Скоттом, слегка отличается от используемой в этой книге. Во-первых, коэффициенты целевой функции должны быть записаны в первой строке, а не в последней. Во-вторых, действительное большое число должно использоваться в модифицированной целевой функции. И, наконец, столбец для целевой переменной Р в эту таблицу не включен. Исходная таблица Скотта для модифицированной задачи показана на рис. 5.27. т {[ -6 -12 -2 -8 0 0 0 1000000 0 1000000 0 0 0 ] (1 1 0 0 1 0 0 0 0 0 0 0 300001 (0 0 1 1 0 1 0 0 0 0 0 0 20000] [1 0 1 0 0 0 -1 1 0 0 0 0 20000] [0 1 0 1 0 0 0 0 -1 1 0 0 10000] [5 0 -15 0 0 0 0 0 0 0 1 0 0 ] [0 15 0 -5 0 0 0 0 0 0 0 1 0 ]] Рис. 5.27. Решение задачи с помощью графического калькулятора Результат выполнения программы приведен в первых двух строках рис. 5.28. Z = 310000 <26250 3750 8750 11250 0 0 15000 0 5000 0 5000 5000} [[0 0 0 0 3 и 0 1000000 0 1000000 .6 .6 310000] [0000 1.5 -.5 1 -1 0 0 ’.I -.1 15000 ] [0 0 0 0 -.5 1.5 0 0 1 -1 .1 .1 5000 ] [0 0 1 0 .375 -.125 0 0 0 0 -.075 -.025 8750 ] [0 0 0 1 -.375 1.125 0 0 0 0 .075 .025 11250 ] [1 0 0 0 1.125 -.375 0 0 0 0 -.025 -.075 26250 ] [0100 -.125 .375 0 0 0 0 .025 .075 3750 ]] Рис. 5.28. Таблица, содержащая решение задачи Интерпретируя результат, представленный на рис. 5.28, видим, что нефтеперерабатывающий завод должен смешать 26250 баррелей компонента А и 8750 баррелей компонента Б, чтобы произвести 35 000 баррелей обычного бензина. Кроме того, завод должен смешать 3750 баррелей компонента Я и 11250 баррелей компонента Б, чтобы произве-
516 Часть II. Конечная математика сти 15 000 баррелей бензина высшего сорта. Это даст максимальную прибыль, равную 310000 долл. ■ Задание 5.13. Дайте интерпретацию фиктивных и избыточных переменных в компьютерном решении, представленном на рис. 5.28. ■ В Упражнение 5.22. Предположим, что нефтеперерабатывающий завод, упомянутый в примере 5.22, располагает 35 000 баррелями компонента Л, который стоит 25 долл, за баррель, и 15 000 баррелями компонента#, который стоит 35 долл, за баррель. Сформулируйте задачу линейного программирования, решением которой является максимальная прибыль, если вся остальная информация остается неизменной. Не пытайтесь решить задачу (до тех пор, пока вы не овладеете программным обеспечением, предназначенным для выполнения задач линейного программирования). ■ Ответы к упражнениям 5.18. Максимизировать функцию Р = 3xi — 2x2 4- хз — Mai — Ма2 — Маз при условиях xi - 2х2 4- х3 - $i + ai = 5, xi + Зх2 - 4х3 — 32 4- а2 — 10, 2xi + 4х2 4- 5х3 4- $з = 20, -3xi 4- х2 4- х3 4- аз = 15, Xi, Х2, Х3, ®1, s2> а2> 53> а3 0. 5.19. МахР = 22 при xi = 6, Х2 = 4, хз = 0. 5.20. Не имеет оптимального решения. 5.21. Минимальная стоимость в 200 долл, достигается, когда камни вида А не производятся, ежедневно производится шесть камней вида Б и два камня вида В. 5.22. Максимизировать Р = 9xi -I-15^2 — хз + 5x4 при условиях Х1 + х2 s $ 35000, Хз 4“ Х4 ; 15000, Х1 4- хз J г 20000, Х2 4- Х4 ; г 10000, 5xi - 15х3 s ; о, 15X2 — 5^4 S S о, Xi, Х2, Х3, Х4 г 0. Практикум 5.6 А В задачах 1-8 выполните следующее. 1) Введите фиктивные, избыточные и искусственные переменные и сформулируйте модифицированную задачу.
Глава 5. Линейные неравенства и линейное программирование 517 2) Запишите предварительную симплекс-таблицу для модифицированной задачи и найдите исходную симплекс-таблицу 3) Найдите оптимальное решение модифицированной задачи, применяя симплекс- метод к исходной симплекс-таблице. 4) Найдите оптимальное решение исходной задачи, если оно существует. 1. Максимизировать функцию Р = 5xi + 2х2 при условиях Xi + 2х2 С 12, xi + х2 > 4, Х1, Х2 > 0. 2. Максимизировать функцию Р = 3xi 4- 7x2 при условиях 2X1 4- х2 16, Xi + Х2 6, Xi, Х2 0. 3. Максимизировать функцию Р = 3xi + 5х2 при условиях 2xi 4- Х2 8, Х1 4* х2 = 6, xi, х2 > 0. 4. Максимизировать функцию Р = 4xi + Зх2 при условиях Xi -|- Зх2 24, Х1 + х2 = 12, xi, х2 0. 5. Максимизировать функцию Р = 4xi -I- Зх2 при условиях —xi 4- 2x2 С 2, xi + х2 4, xi, х2 0. 6. Максимизировать функцию Р = 3xi 4- 4х2 при условиях Х1 — 2x2 2, xi 4- х2 5, xi, х2 0. 7. Максимизировать функцию Р = 5xi 4- 10х2 при условиях xi 4- х2 3, 2xi 4" Зх2 12, Xi, х2 0. 8. Максимизировать функцию Р = 4xi -I- 6х2 при условиях Xi 4- х2 2, 3xi 4“ 5х2 15, xi, х2 0.
518 Часть II. Конечная математика Б Решите задачи 9-22 с помощью М-метода. 9. Минимизировать и максимизировать функцию Р = 2х\ - х2 при условиях + Х2 < 8, 5xi + Зх2 > 30, Х1, х2 > 0. 10. Минимизировать и максимизировать функцию Р = — 4rri + 16x2 при условиях 3xi + х2 28, xi -I- 2x2 16, Xi, Х2 0. И. Максимизировать функцию Р = 2xi + 5х2 при условиях xi + 2x2 С 18, 2xi + Х2 21, xi + х2 10, Xi, Х2 > 0. 12. Максимизировать функцию Р = 6xi + 2х2 при условиях xi + 2х2 20, 2xi + Х2 16, xi 4- х2 > 9, Xi, Х2 0. 13. Максимизировать функцию Р = 10xi 4- 12х2 4- 20хз при условиях 3xi 4- х2 4- 2хз 12, — %2 4- 2хз = 6, %1, хз 0. 14. Максимизировать функцию Р = 5xi 4- 7х2 4- 9хз при условиях xi — х2 4- х3 20, 2xi 4- х2 4- 5хз = 35, xi, х2, хз 0. 15. Максимизировать функцию С = —5xi — 12х2 4- 16хз при условиях xi 4- 2х2 4- хз 10, 2xi 4- Зх2 4- хз > 6, 2xi 4- х2 - х3 = 1, ХЪ Х2, Хз 0. 16. Максимизировать функцию С = —Зхх 4- 15х2 — 4х3 при условиях 2xi 4- х2 4- Зхз < 24, Xi 4- 2х2 4- Хз 6, xi — Зх2 4- хз = 2, xi, х2, х3 0.
Глава 5. Линейные неравенства и линейное программирование 519 17. Максимизировать функцию Р = 3xi + 5х2 + 6x3 при условиях 2xi 4- х2 4- 2х3 < 8, 2xi + х2 — 2хз = О, Х1, х2, х3 0. 18. Максимизировать функцию Р = 3xi + 6х2 -к 2хз при условиях 2x1 + 2х2 4- Зхз 12, 2xi — 2х2 4- хз = 0, xi, х2, хз 0. 19. Максимизировать функцию Р = 2xi 4- Зх2 4- 4хз при условиях Xi 4- 2х2 + х3 25, 2xi + х2 4- 2х3 < 60, Xi 4- 2х2 - х3 10, xi, х2, х3 > 0. 20. Максимизировать функцию Р = 5xi 4- 2х2 4- 9хз при условиях 2xi + 4х2 + х3 150, 3xi 4- Зх2 4- хз 90, —Xi 4- 5х2 4- хз 120, xi, х2, х3 > 0. 21. Максимизировать функцию Р = xi 4- 2х2 4- 5хз при условиях Xi 4- Зх2 + 2х3 < 60, 2xi 4- 5х2 4- 2хз > 50, Xi - 2х2 4- х3 40, xi, х2, х3 0. 22. Максимизировать функцию Р = 2xi 4- 4х2 4- хз при условиях 2xi 4- Зх2 4- 5хз < 280, 2xi 4- 2х2 + хз 140, 2xi + х2 150, xi, х2, хз 0. *23. Решите задачи 5 и 7 геометрическим методом. Сравните условия, при которых М-метод позволяет выяснить, что оптимального решения не существует, с условиями, перечисленными в теореме 5.3 из раздела 5.2. * 24. Повторите решение задачи 23 применительно к задачам 6 и 8. В Задачи 25-32 являются смешанными. Некоторые из них могут быть решены с помощью методов, представленных в разделах 5.4 и 5.5, в то время как другие должны быть решены с помощью М-метода. 25. Минимизировать функцию С = 10xi — 40х2 - 5хз при условиях Xi 4- Зх2 < 6, 4х2 + х3 3, xi, х2, хз 0.
520 Часть II. Конечная математика 26. Максимизировать функцию Р = 7xi — 5x2 + 2хз при условиях — 2х2 4- х3 > —8, Xi — Х2 + Х3 < 10, ХЪ #2, #3 > 0. 27. Максимизировать функцию Р = —5xi 4-10^2 + 15хз при условиях 2xi 4- 3x2 4- хз 24, xi — 2x2 — 2х3 1, Х1, Х2, Хз 0. 28. Минимизировать функцию С = — 5xi 4- 10x2 4- 15хз при условиях 2xi + 3x2 4- хз 24, xi — 2x2 — 2хз 1, хъ х2, хз 0. 29. Минимизировать функцию С = 10xi 4- 40x2 4- 5хз при условиях xi 4- 3x2 > б, 4х2 4- хз > 3, Xi, Х2, Хз 0. 30. Максимизировать функцию Р = 8xi 4- 2x2 — Юхз при условиях xi 4- Х2 — Зхз < 6, 4xi - х2 4- 2х3 -7, Х1, Х2, х3 0. 31. Максимизировать функцию Р = 12xi 4- 9x2 4- 5х3 при условиях xi 4- 3x2 4- хз 40, 2xi 4- Х2 4- Зхз < 60, хь х2, хз 0. 32. Минимизировать функцию С = 10xi 4- 12x2 4- 28хз при условиях 4xi 4- 2x2 4- Зхз 20, 3xi — х2 — 4хз 10, xi, ^2, 0- Применение математики Сформулируйте задачи линейного программирования, а затем решите их с помощью М-метода. (Ответы приведены в конце книги.) Экономика и бизнес 33. Производство: распределение ресурсов. Компания по производству электронных компонентов выпускает два типа модулей памяти для микрокомпьютеров, емкость которых равна 16 Кбайт и 64 Кбайт соответственно. Модуль емкостью 16 Кбайт требует 10 мин. для сборки и две минуты для тестирования. Модуль емкостью 64 Кбайт требует 15 мин. для сборки и четыре минуты для тестирования. Компания получает прибыль 18 долл, от каждого модуля емкостью
Глава 5. Линейные неравенства и линейное программирование 521 16 Кбайт и 30 долл, от каждого модуля емкостью 64 Кбайт. Общее рабочее время цеха сборки составляет 2200 мин. Для цеха тестирования этот показатель равен 500 мин. Для того чтобы удовлетворить текущие потребности, компания должна производить по крайней мере 50 модулей емкостью 16 Кбайт ежедневно. Сколько единиц каждых модулей должна производить компания ежедневно, чтобы максимизировать прибыль? Чему равна максимальная прибыль? *34. Производство: распределение ресурсов. Как изменится решение задачи 33, если ежедневно сборочный цех может работать только 2100 минут? 35. Реклама. Фирма, планируя рекламную кампанию для привлечения новых клиентов, хочет разместить не больше 10 рекламных объявлений в трех газетах. Сообщение в газете Sentinel стоит 200 долл, и будет прочитано 2000 чел. Объявление в газете Journal стоит 200 долл, и будет прочитано 500 чел. Рекламное объявление в газете Tribune стоит 100 долл, и будет прочитано 1500 чел. Компания хочет, чтобы ее рекламные объявления прочитали не менее 16000 чел. Сколько объявлений она должна поместить в каждой из газет, чтобы минимизировать рекламные расходы? Чему равны минимальные расходы? *36. Как изменится решение задачи 35, если газета Tribune не будет принимать от компании больше четырех рекламных объявлений? Биологические науки 37. Диета. Человеку, придерживающемуся богатой белками низкоуглеводной диеты, необходимо получать с пищей не менее 100 единиц белков и не больше 24 единиц углеводов ежедневно. Диета должна состоять из трех специальных жидких блюд А, Б и В. Содержание и стоимость блюд приведены в таблице. Сколько порций каждого блюда нужно потреблять ежедневно, чтобы удовлетворить белковые и углеводные требования диеты при минимальных расходах? Чему равна минимальная стоимость потребляемых блюд? Блюда А Б В Белки, ед. 10 10 20 Углеводы, ед. 2 3 4 Стоимость блюда, долл. 0,60 0,40 0,90 *38. Как изменится решение задачи 37, если стоимость блюда В увеличится до 1,50 долл. 39. Удобрение растений. Фермер может использовать три типа удобрений — смесь А, смесь Б и смесь В. Содержание азота, фосфора и калия в кубическом ярде каждой смеси приведены в таблице. Проведя химический анализ грунта, было установлено, что на поле нужно внести по крайней мере 800 фунтов калия, но не более 700 фунтов фосфора. Фермер планирует выращивать сельскохозяйственную культуру, которая потребляет большое количество азота. Сколькими кубическими ярдами каждой смеси должно быть удобрено поле, чтобы удовлетворить потребности в калии и фосфоре и максимизировать количество азота? Чему равно максимальное вносимое количество азота?
522 Часть И. Конечная математика Удобрительная смесь А Б В Азот, фунт/куб. ярд 12 16 8 Фосфор, фунт/куб. ярд 12 8 16 Калий, фунт/куб ярд 16 8 16 * 40. Удобрение растений. Как изменится решение задачи 39, если предельное содержание фосфора увеличится до 1000 фунтов? В задачах 41-49 сформулируйте задачи линейного программирования, не решая их. Экономика и бизнес 41. Планирование производства. Компания производит рамы для легковых и грузовых автомобилей на заводах в Милуоки и Рейсине. Ежедневный производственный бюджет завода в Милуоки равен 50000 долл., а объем производства — не более 300 рам. Производство рамы для легкового автомобиля на заводе в Милуоки стоит 150 долл., а для грузового — 200 долл. Ежедневный производственный бюджет завода в Рейсине равен 35 000 долл., а объем производства — не более 200 рам. Производство рамы для легкового автомобиля на заводе в Рейсине стоит 135 долл., а для грузового — 180 долл. Основываясь на предыдущих требованиях, компания хочет ограничить производство 250 рамами для легковых автомобилей и 350 рамами для грузовых автомобилей. Сколько рам каждого типа следует производить ежедневно на каждом заводе, чтобы максимизировать дневную прибыль, если компания получает 50 долл, прибыли от каждой рамы для легкового автомобиля и 70 долл, прибыли от каждой рамы для грузового автомобиля. 42. Финансы: распределение кредитов. Сберегательно-кредитная компания имеет три миллиона долларов для предоставления кредитов. Типы кредитов и ежегодных доходов приведены в таблице. Законы штата требуют, чтобы не меньше 50% средств по ипотечным кредитам были предоставлены по первой закладной, и не меньше 30% общего объема кредитов предоставлялись по первой или второй закладной. Политика компании требует, чтобы количество предоставляемых вексельных и автомобильных кредитов не превышало 25% общего объема кредитов, а вексельные кредиты не превышали 15%. Сколько денег следует вложить в каждый тип кредита, чтобы максимизировать доход компании? Тип кредита Ежегодный доход, % Вексельный 18 Первая закладная 12 Вторая закладная 14 Автомобильный 16
Глава 5. Линейные неравенства и линейное программирование 523 43. Производство бензина. Нефтеперерабатывающий завод производит два сорта бензина, обычный и повышенного качества, смешивая три компонента: А, Б и В. Компонент А имеет октановое число 90 и стоит 28 долл, за баррель, компонент Б имеет октановое число 100 и стоит 30 долл, за баррель, а компонент В имеет октановое число 110 и стоит 34 долл, за баррель. Октановое число обычного бензина должно быть не меньше 95, а октановое число бензина повышенного качества должно быть не меньше 105. Обычный бензин продается по 38 долл., а бензин повышенного качества — по 46 долл, за баррель. У компании есть 40 000 баррелей компонента А, 25 000 баррелей компонента Б и 15 000 баррелей компонента В. Она должна произвести по крайней мере 30000 баррелей обычного бензина и 25000 баррелей бензина повышенного качества. Как следует смешать компоненты, чтобы максимизировать прибыль? 44. Смешивание: производство питания. Компания производит два типа сухих завтраков, обычные и элитные, смешивая сухофрукты, орехи и хлебные злаки. Рецепты смесей даны в таблице. У компании есть 1200 фунтов сухофруктов, 750 фунтов орехов и 1500 фунтов хлебных злаков. Компания получает 0,40 долл, прибыли с каждого фунта обычной смеси и 0,60 долл, с каждого фунта улучшенной смеси. Сколько фунтов каждого ингредиента следует использовать в каждой смеси, чтобы максимизировать прибыль компании? Тип смеси Ингредиенты Обычная По крайней мере 20% орехов. По крайней мере 40% злаков. Улучшенная По крайней мере 30% орехов. По крайней мере 25% злаков. 45. Инвестиционная политика. Инвестор планирует разделить свои инвестиции между высокотехнологичными взаимными фондами, глобальными взаимными фондами, корпоративными облигациями, муниципальными облигациями и депозитными сертификатами. Каждый из этих типов инвестиций имеет ежегодную доходность и фактор риска (см. таблицу). Уровень риска представляет собой произведение соответствующего фактора риска и процентного соотношения всех выбранных фондов. Общий уровень риска представляет собой сумму уровней риска для всех вложений. Инвестор хочет, чтобы по крайней мере 20% его вложений составляли вложения в депозитные сертификаты, и не хочет, чтобы уровень риска превышал 1,8. Какое процентное соотношение всех его инвестиций должно быть сделано в каждый из фондов, чтобы максимизировать доходность? Вид инвестиций Ежегодная доходность Фактор риска Фонды высоких технологий 0,11 2,7 Глобальные фонды 0,1 1,8 Корпоративные облигации 0,09 1,2 Муниципальные облигации 0,08 0,5 Депозитные сертификаты 0,05 0
524 Часть II. Конечная математика 46. Инвестиционная политика. Вернемся к задаче 45. Предположим, что инвестор решает минимизировать общий фактор риска прежде, чем доход упадет ниже 9%. Какое процентное соотношение общего количества инвестиций должно быть сделано в каждый из фондов, чтобы минимизировать общий уровень риска? Биологические науки 47. Диета. Специалист по питанию должен составить специальную диету с использованием блюд А9 Б и В. В таблице приведены ингредиенты и стоимость одной унции каждого блюда. Ежедневная диета должна обеспечивать по крайней мере 400 единиц кальция, по 200 единиц железа, 300 единиц витамина А, а также содержать не больше 150 единиц холестерина и 900 калорий. Сколько унций каждого блюда следует использовать, чтобы выполнить требования диеты при минимальной стоимости? Блюдо А Б В Кальций, ед. 30 10 30 Железо, ед. 10 10 10 Витамин А, ед. 10 30 20 Холестерин, ед. 8 4 6 Калории, ед. 60 40 50 Стоимость унции, долл. 0,40 0,60 0,80 48. Рацион питания. Фермер выращивает три сельскохозяйственные культуры — кукурузу, овес и соевые бобы, которые смешивает, чтобы получить корм для коров и свиней. По крайней мере 40% пищевой смеси для коров должна составлять кукуруза. Пищевая смесь для свиней должна содержать по крайней мере вдвое больше соевых бобов, чем кукурузы. Фермер собрал 1000 бушелей кукурузы, 500 бушелей овса и 1000 бушелей соевых бобов. Ему необходимо 1000 бушелей каждой смеси для скота. Неиспользованные кукуруза, овес и соевые бобы могут быть проданы за 4; 3,50 и 3,25 долл, за бушель соответственно. (Таким образом, эти цифры представляют собой стоимость сельскохозяйственных культур, используемых для питания скота.) Сколько бушелей каждой сельскохозяйственной культуры следует использовать в каждой пищевой смеси, чтобы произвести достаточно корма для скота при минимальных затратах? Социальные науки 49. Образование: распределение ресурсов. Три города образуют общий образовательный агломерат с двумя средними школами. Каждая школа может принять максимум 2000 студентов. В городе А проживает 500 учеников, в городе Б — 1200, а в городе В — 1800. Еженедельные затраты на перевозку учеников из каждого города в каждую школу приведены в таблице. Для того чтобы выровнять нагрузку, межгородской совет решил, что каждая школа должна включить в список по крайней мере 40% общего количества учеников. Более того, в каждом городе в одну и ту же школу нельзя отправлять больше 60% учеников. Сколько учеников из каждого города должны быть включены в списки каждой
Глава 5. Линейные неравенства и линейное программирование 525 школы так, чтобы удовлетворить эти требования и минимизировать затраты на перевозку? Еженедельные затраты на перевозку учеников, долл. Школа I Школа II Город А 4 8 Город Б 6 4 Город В 3 9 Ключевые слова» основные обозначения и формулы 5.1. Системы линейных неравенств с двумя переменными. График линейного неравенства с двумя переменными; правая полуплоскость; левая полуплоскость; верхняя полуплоскость; нижняя полуплоскость; графическое решение системы линейных неравенств; область решения; допустимая область; угловая точка; ограниченные области; неограниченные области. 5.2. Линейное программирование на плоскости: геометрический подход. Задача линейного программирования; переменные; целевая функция; ограничения; условия неотрицательности; математическая модель; графическое решение; задача максимизации; линия постоянной прибыли; оптимальное решение; область допустимых решений; основная теорема линейного программирования; множественное оптимальное решение; пустая область допустимых решений; неограниченная целевая функция. 5.3. Геометрическое введение в симплекс-метод. Стандартная задача максимизации; фиктивные переменные; базисные переменные; небазисные переменные; базисное решение; базисное допустимое решение; основная теорема линейного программирования. 5.4. Симплекс-метод: максимизация с ограничениями вида “меньше или равно ”. Исходная система; базисные решения и базисные допустимые решения для исходных систем; основная теорема линейного программирования; исходное допустимое базисное решение; симплекс-таблица; исходная симплекс-таблица; выбор базисных и небазисных переменных для симплекс-метода; итерация симплекс- метода; включаемая переменная; исключаемая переменная; ведущий столбец; индикаторы; Ведущая строка; ведущий элемент; поиск опорного плана. 5.5. Двойственная задача: минимизация с ограничениями вида “больше или равно”. Двойственная задача; транспонирование; транспонированная матрица АТ; решение задачи минимизации; основной принцип двойственности; транспортная задача. 5.6. Максимизация и минимизация со смешанными ограничениями задачи. Избыточная переменная; искусственная переменная; модифицированная задача; предварительная симплекс-таблица; исходная симплекс-таблица; М-метод.
526 Часть II. Конечная математика Классификация задач и методов Тип задачи Ограничения Правая часть ограничений Коэффициенты целевой функции Метод решения 1. Мак- Меньше Неотрицатель- Любые действи- Использование симплекс- сими- зация или равно ные тельные числа метода с фиктивными переменными 2. Минимизация Больше или равно Любые действительные числа Неотрицательные Составление двойственной задачи и решение с помощью предыдущего метода 3. Мак¬ Смешан¬ Неотрицатель¬ Любые действи¬ Формирование моди¬ сими¬ зация ные (^, =) ные тельные числа фицированной задачи с фиктивными, избыточными и искусственными переменными и решение с помощью М-метода 4. Ми¬ Смешан¬ Неотрицатель¬ Любые действи¬ Максимизация функции, ними¬ зация ные (^, =) ные тельные числа отрицательной по отношению к целевой функции с помощью предыдущего метода Упражнения для повторения Выполните все упражнения этого обзорного раздела и сравните результаты с ответами, помещенными в конце книги. Ответы ко многим упражнениям на повторение приводятся вместе с номером соответствующего раздела (курсивом). Если у вас возникают затруднения при решении какой-либо задачи, повторите материал соответствующего раздела. А Решите системы в задачах 1 и 2 с помощью графического метода и определите, являются ли полученные области допустимых решений ограниченными или неограниченными. Найдите координаты всех угловых точек. 1. 2х! + х2 8, 2. 3X1 + х2 > 9, 3xi + 9х2 < 27, 2xi + 4х2 > 16, xi, х2 0. xi, х2 0. 3. Решите задачу линейного программирования геометрически. Максимизировать функцию Р = 6xi + 2х2 при условиях 2xi -I- х2 8, xi -I- 2х2 10, xi, х2 0. 4. Представьте ограничения задачи 3 в виде системы уравнений, используя фиктивные переменные. 5. Сколько базисных и небазисных переменных связаны с системой в задаче 4?
Глава 5. Линейные неравенства и линейное программирование 527 6. Найдите все базисные решения системы в задаче 4 и определите, какое базисное решение является допустимым. 7. Запишите симплекс-таблицу задачи 3 и укажите ведущий элемент. Определите включаемые и исключаемые переменные. 8. Решите задачу 3, используя симплекс-метод. 9. Для приведенной ниже симплекс-таблицы определите базисные и небазисные переменные. Найдите ведущий элемент, включаемую и исключаемую переменную и выполните одну итерацию симплекс-метода. 11 х2 хз «1 S2 «3 Р 2 1 3 -1 0 0 0 20 3 0 4 1 1 0 0 30 2 0 5 2 0 1 0 10 -8 0 -5 3 0 0 1 50 10. Найдите базисное решение для каждой таблицы. Определите, достигнуто ли оптимальное решение, нужна ли дополнительная итерация симплекс-метода или задача не имеет оптимального решения. а) в) Х1 а?2 31 81 Р ’4 10 0 0 2 0 110 2 ' 5 б) ■ -1 3 0 10 0 2 10 0 7 ' 0 —2 10 0 1 22 —2 0 3 0 1 1 -2 0 4 0 0 2 16 0 12 6 ‘ 15 1 ; о! оо; О; к>: ►— 10 11. Решите задачу линейного программирования геометрически. Максимизировать функцию С = 5x± + 2х2 при условиях #1 + 3^2 15, 2xi + #2 20, Xi, Х2 > 0. 12. Сформулируйте двойственную задачу к задаче 11. 13. Запишите исходную систему для двойственной задачи 12. 14. Запишите первую симплекс-таблицу для двойственной задачи 12 и обозначьте столбцы. 15. Используйте симплекс-метод для определения оптимального решения двойственной задачи 12. 16. Используйте окончательную симплекс-таблицу из задачи 15 для определения оптимального решения задачи 11.
528 Часть II. Конечная математика Б 17. Решите задачу линейного программирования геометрически. Максимизировать функцию Р = 3xi + 4х2 при условиях 2x1 + 4x2 < 24, 3xi + 3x2 21, 4xi 4- 2x2 20, Xi, Х2 0. 18. Решите задачу 17, используя симплекс-метод. 19. Решите задачу линейного программирования геометрически. Минимизировать функцию С — 3xi 4- 8х2 при условиях Xi + х2 > 10, xi 4- 2x2 15, х2 > 3, Xi, Х2 > 0. 20. Сформулируйте двойственную задачу к задаче 19. 21. Решите задачу 19, применяя симплекс-метод к двойственной задаче 20. Решите задачи линейного программирования 22 и 23. 22. Максимизировать функцию Р = 5xi + Зх2 — Зх3 при условиях xi — х2 — 2х3 3, 2xi + 2х2 — 5хз < 10, Xi, Х2, Хз 0. 23. Максимизировать функцию Р = 5xi + Зх2 — Зх3 при условиях Х1 — х2 — 2хз < 3, xi + х2 5, xi, х2, хз 0. В задачах 24 и 25 выполните следующее. 1) Введите фиктивные, избыточные и искусственные переменные и поставьте модифицированную задачу. 2) Запишите предварительную симплекс-таблицу для модифицированной задачи и найдите исходную симплекс-таблицу. 3) Найдите оптимальное решение модифицированной задачи, применяя симплекс- метод к исходной симплекс-таблице. 4) Найдите оптимальное решение исходной задачи, если оно существует. 24. Максимизировать функцию Р = xi + Зх2 при условиях xi 4* х2 6, xi -4- 2х2 8, xi, х2 0.
Глава 5. Линейные неравенства и линейное программирование 529 25. Максимизировать функцию Р = xi + Х2 при условиях xi + ж2 5, Х\ + 2X2 4, Х1, Х2 0. 26. Запишите модифицированную задачу для следующей задачи линейного программирования, не решая ее. Максимизировать функцию Р = 2xi + 3x2 + хз при условиях Х1 — 3^2 + -Xi - х2 + 2хз -2, 3xi -I- 2x2 — 4, X1, Х2, *3 0. Кратко опишите задачи, которые можно решить с помощью методов, указанных ниже. В описание включите виды оптимизации, количество переменных, типы ограничений и все условия, наложенные на коэффициенты и константы. * 27. Геометрический метод. * 28. Базисный симплекс-метод с фиктивными переменными. * 29. Двойственный метод. *30. М-метод. В * 31. Решите следующую задачу линейного программирования с помощью симплекс- метода. Затем нарисуйте область допустимых решений и траекторию, ведущую к оптимальному решению, найденному с помощью симплекс-метода. Максимизировать функцию Р = 2х\ + Зх2 при условиях #1 + 2x2 22, 3xi + я?2 С 26, Xi 8, х2 < Ю, Xi, Х2 0. 32. Решите задачу с помощью двойственного метода. Минимизировать функцию С = 3xi + 2x2 при условиях 2x1 + #2 20, 2xi + я2 9, Xi + Х2 6, Xi, Х2 > 0. 33. Решите задачу 32 с помощью М-метода.
530 Часть II. Конечная математика 34. Решите задачу с помощью двойственного метода. Минимизировать функцию С = 15xi + 12^2 + 15х3 + 18x4 при условиях Xi + Х2 240, Тз + #4 500, xi + х3 400, Х2 + #4 > 300, Х1, ^2, %3, х4 0. Применение математики Сформулируйте задачи линейного программирования и решите избранным методом. (Ответы приведены в конце книги.) Экономика и бизнес 35. Производство: распределение ресурсов. Компания South Shore Sail Loft производит обычные и спортивные паруса. Для изготовления обычного паруса необходимо два часа на раскройку и четыре часа на пошив. Для изготовления спортивного паруса необходимо три часа на раскройку и 10 ч для пошив. Цех раскройки работает 150 ч, а цех пошива — 380 ч. Примените геометрический метод. а) Сколько парусов каждого типа должна производить компания, чтобы максимизировать прибыль, если прибыль от реализации обычного паруса составляет 100 долл., а от реализации паруса для соревнований — 200 долл.? Чему равна максимальная прибыль? *б) Увеличение спроса на спортивные паруса приводит к тому, что прибыль от их продажи возрастает до 260 долл. Как изменится количество произведенных парусов и максимальная прибыль? *в) Уменьшение спроса на спортивные паруса приводит к тому, что прибыль от их продажи падает до 140 долл. Как изменится количество произведенных парусов и максимальная прибыль? 36. Капиталовложения. У инвестора есть 150000 долл, для вложения в акции нефтяных компаний, металлургических комбинатов и правительственные облигации. Правительственные облигации гарантируют доходность на уровне 5%, но доходность каждой акции может меняться. Чтобы защитить себя от больших потерь, инвестор решает, что сумма, вложенная в акции нефтяных компаний, не должна превышать 50 000 долл., и что общая сумма, вложенная в акции, не может превышать сумму, вложенную в облигации, более чем на 25 000 долл. Используйте симплекс-метод для ответа на следующие вопросы. а) Каков должен быть инвестиционный портфель, чтобы максимизировать доходность, если акции нефтяных компаний приносят 12%, а металлургических комбинатов — 9%? Чему равен максимальный доход? б) Повторите решение п. а, если акции нефтяных компаний приносят 9%, а металлургические — 12%.
Глава 5. Линейные неравенства и линейное программирование 531 37. Логистика: график перевозок. Компания выпускает электродвигатели для стиральных машин на заводе А и заводе Б. Затем они перевозятся либо на завод X, либо на завод Y, где производится сборка стиральных машин. Максимальное количество электродвигателей, которое может быть произведено на каждом заводе ежемесячно, минимальное количество двигателей, требуемое ежемесячно для удовлетворения потребностей каждого из заводов и затраты на транспортировку одного двигателя приведены в таблице. Определите план перевозок, обеспечивающий минимизацию затрат на транспортировку электродвигателей на сборочные заводы. Используйте двойственный метод. Стоимость доставки Объем электродвигателя, долл. Завод X —давОд y производства, шт. Завод А 5 8 1500 Завод Б 9 7 1000 Требуемое количество, шт. 900 1200 38. Пищевое производство. Компания использует удлиненный и канадский рис для производства двух марок рисовых смесей, марки Л, которая продается под фирменным названием, и марки Б, которая продается под непатентованным названием. Марка А должна содержать по крайней мере 10% удлиненного, а марка Б — 5% канадского риса. Удлиненный рис обходится компании в 0,70 долл., а канадский — 3,40 долл, за фунт. Компания продает смесь марки А по 1,50 долл, за фунт, а марку Б — по 1,20 долл, за фунт. У компании есть 8000 фунтов удлиненного и 500 фунтов канадского риса. Как следует использовать имеющийся рис, чтобы максимизировать прибыль от продажи этих двух смесей? Чему равна максимальная прибыль? Используйте симплекс-метод. Биологические науки 39. Рацион питания. Специальная диета должна обеспечивать лабораторных животных по крайней мере 850 единицами витаминов, 800 единицами минералов и 1150 калориями. Есть две пищевые смеси: А и Б. Грамм смеси А содержит две единицы витаминов, две единицы минералов и четыре калории. Грамм смеси Б содержит пять единиц витаминов, четыре единицы минералов и пять калорий. Используйте геометрический метод линейного программирования. а) Сколько граммов каждой смеси следует использовать, чтобы удовлетворить требованиям диеты при минимальной стоимости, если смесь А стоит 0,04 долл., а смесь Б — 0,09 долл, за грамм? Чему равна минимальная стоимость? *б) Как изменится решение п. а, если цена смеси Б упадет до 0,06 долл, за грамм? *в) Как изменится решение п. а, если цена смеси Б возрастет до 0,12 долл, за грамм?
532 Часть II. Конечная математика Домашнее задание 5.1. Двухэтапный метод: альтернатива М-методу В М-методе, чтобы получить базисное допустимое решение, к ограничениям “больше или равно” и “равно” прибавлялись искусственные переменные, а затем к целевой функции прибавлялись слагаемые вида —Ма^. Предполагалось, что число М было произвольно большим. Это гарантировало, что в любом оптимальном решении искусственные переменные равны нулю. Существует другой популярный метод, называемый двухэтапным методом, который использует искусственные переменные иначе. Чтобы проиллюстрировать этот метод, рассмотрим следующий пример. Максимизировать функцию Р = 2xi + х2 при условиях xi + х2 С Ю, xi 3, Х2 4, Х1, Х2 0. 1. Изобразите область допустимых решений и решите эту задачу геометрическим методом. Если прибавить фиктивные и избыточные переменные, получится следующая система. #1 + х2 + $1 = 10, Xi — s2 = 3, (5.24) %2 -«з -4, -2X1 - %2 + Р = 0. Базисным решением этой системы является следующий набор чисел. xi = 0, х2 = 0, <$1 = 10, s2 = —3, «з = —4, Р = 0. Это решение не является допустимым, поэтому поиск опорного плана невозможен. Как и ранее, прибавим искусственные переменные, чтобы получить базисное допустимое решение, не изменяя целевую функцию. + X? + «1 = 10, Xi — s2 + ai = 3, (5.25) %2 — $3 + а2 = 4, —2X1 - %2 + Р = 0. Базисным решением этой системы является следующий набор чисел. xi = 0, х2 = 0, §1 = 10, 82 = 0, ai = 3, «з = 0, а2 = 4, Р = 0. Это решение является допустимым, следовательно, можно начать симплекс-процесс. 2. Запишите симплекс-таблицу для системы (5.25) и проведите поиск опорного плана. Сравните результат с геометрическим решением.
Глава 5. Линейные неравенства и линейное программирование 533 Применение симплекс-метода к системе (5.25) дает оптимальное решение системы (5.25) с одной или несколькими искусственными переменными, которые имеют ненулевые значения. В большинстве случаев простое удаление искусственных переменных не дает базисного допустимого решения системы (5.24). Однако, если можно найти базисное допустимое решение системы (5.25) со всеми искусственными переменными, равными нулю, то можно удалить искусственные переменные и найти базисное допустимое решение системы (5.24). В М-методе это завершается прибавлением слагаемых к целевой функции. В этом новом методе рассматриваем другую целевую функцию. С = ai + й2« Если минимизировать функцию С, и минимальное значение окажется равным нулю, то, поскольку ai и аг неотрицательны, они обе равны нулю. Для того чтобы минимизировать С, максимизируем функцию Т = —С = — (11 — (12. Добавить функцию Т к системе (5.25) в этом виде нельзя. Чтобы сохранить необходимую форму симплекс-таблицы, необходимо выразить функцию Т через небазисные переменные. Решая второе и третье уравнения системы (5.25) относительно переменных и аг соответственно, получим следующую функцию. Т = xi + х2 - «2 - «з - 7. Перенося все переменные в левую часть, добавим это уравнение к системе (5.25), образуем систему (5.26): Xi 4- х2 -h si = 10, Х1 — S2 + «1 — 3, Х2 - «з + Л2 =4, (5.26) —2х± — х2 + Р =0, —XI — х2 + S2 + S3 + Т = -7. 3. Запишите симплекс-таблицу для системы (5.26) и примените симплекс-метод со следующим ограничением: не выбирать никакие из элементов в строке целевой функции в качестве ведущих. Найдя оптимальное решение, запишите соответствующую систему, которая показана ниже. Si + s2 — ai + S3 — а2 — 3, Х\ — s2 + ai = 3, х2 - «з + а2 = 4, (5.27) — 2s2 2ai — S3 + сь2 P = 10, ai + d2 + T = 0. Базисным допустимым решением системы (5.27) является следующий набор чисел. = 3, х2 = 4, si =3, S2 = 0, ai =0, s3 = 0, а2 = 0, Р = 10, Т = 0.
534 Часть II. Конечная математика Отметим, что максимальным значением функции Т является нуль, и что оно достигается при ai = а2 = 0. Таким образом, удаление уравнения для функции Т и столбца искусственных переменных приводит к системе (5.28), эквивалентной системе (5.24), но имеющей базисное решение, которое является допустимым. xi Х2 $1 + 32 + 33 = 3, - з2 = 3, - Зз =4, - 2s2 - з3 + Р = 10. (5.28) 4. Запишите симплекс-таблицу для системы (5.28) и примените симплекс-процедуру для оптимального решения. Сравните результаты с решением задачи 1. Этот метод называется двухэтапным потому, что он включает решение двух задач. Этап 1. Минимизация суммы искусственных переменных для получения базисного допустимого решения, в котором все искусственные переменные равны нулю. Этап 2. Удаление искусственных переменных и завершение решения исходной задачи. 5. Запишите пошаговую процедуру двухэтапного метода для решения задач линейного программирования. Рассмотрите ситуации, в которых этот метод не будет давать оптимального решения. Примените полученную процедуру для решения задач 13, 15, 17 и 19 из практикума 5.6. Домашнее задание 5.2. Планирование производства io. Компания имеет линию для производства наборов уличной, состоящих из маленьких столов, обычных стульев, вращающихся стульев, шезлонгов и больших столов. Каждый предмет мебели проходит через три цеха: изготовления, сборки и комплектации. В табл. 5.7 приведено время, затрачиваемое на изготовление мебели в каждом из цехов, общее рабочее время каждого цеха, прибыль с каждого предмета и объем заказов на каждый вид продукции. Компания должна выпустить достаточное количество мебели, чтобы удовлетворить существующие заказы. Таблица 5.7. Производство мебели Цех Прибыль с единицы, долл. Объем заказов, шт Изготовления, ч Сборки, ч Комплектация, ч Маленький стол 1 1 1 8 200 Обычный стул 1 2 3 17 160 Вращающийся стул 2 2 3 24 180 Шезлонг 3 4 2 31 240 Большой стол 5 4 5 52 100 Общее рабочее время 2500 3000 3500
Глава 5. Линейные неравенства и линейное программирование 535 1. Сформулируйте задачи линейного программирования и с помощью графической утилиты покажите, что максимальная прибыль, равная 27 830 долл., достигается, когда компания производит 200 маленьких столов, 270 обычных стульев, 180 вращающихся стульев, 240 шезлонгов и 190 больших столов. 2. Как изменится Оптимальное решение задачи 1, если прибыль от реализации маленького стола увеличится до 12 долл.? 3. Как изменится Оптимальное решение задачи 1, если прибыль от реализации большого стола уменьшится до 30 долл.? 4. Как изменится оптимальное решение задачи 1, если заказы на вращающиеся стулья будут отменены? 5. Как изменится Оптимальное решение задачи 1, если объем заказов на шезлонги увеличится до 300 шт.? 6. Как изменится оптимальное решение задачи 1, если общее рабочее время сборочного цеха увеличится доЁЗЗОО? 7. Как изменится оптимальное решение задачи 1, если общее рабочее время цеха изготовления увеличится до 2800?
Теория вероятностей ■ 6.1. Основные принципы счета ■ 6.2. Перестановки и сочетания ■ 6.3. Пространство элементарных исходов, события и вероятность ■ 6.4. Объединение, пересечение и дополнение событий; шансы ■ 6.5. Условная вероятность, пересечение и независимость событий ■ 6.6. Формула Байеса ■ 6.7. Случайная величина, распределение вероятностей и математическое ожидание ■ Ключевые слова, основные обозначения и формулы ■ Упражнения для повторения ■ Домашнее задание 6.1. Победа в телешоу ■ Домашнее задание 6.2. Жеребьевка в профессиональных видах спорта Введение Подобно многим другим областям математики, теория вероятностей возникла как инструмент для решения практических задач. Врач и азартный игрок Джироламо Кардано (Girolamo Cardano) (1501-1576) разработал одну из наилучших математических теорий своего времени, включающую систематический анализ задач, которые возникают в азартных играх. В 1654 году другой игрок, Шевалье де Мере (Chevalier de Меге), преследуемый неудачами, сблизился с известным французским философом и математиком Блэзом Паскалем (Blaise Pascal) (1623-1662), который рассматривал определенные задачи, связанные с игральной костью. Паскаль заинтересовался этими задачами, изучил их и обсудил
Глава 6. Теория вероятностей 537 с Пьером Ферма (Pierre de Fermat) (1601-1665), другим французским математиком. Таким образом, в игровых залах Западной Европы родилась теория вероятностей. Несмотря на свое “низкое” происхождение, теория вероятностей превратилась в высоко ценимую и очень полезную ветвь математики. Она используется практически в каждой области. Теория вероятностей может рассматриваться как наука о неопределенности. Например, неизвестно, Какая карта будет вынута из колоды, состоящей из 52 карт. И все же предположим, что некто вытянул карту и вернул ее в колоду, а затем опять вытянул какую-то карту и вернул ее, и это действие повторили много раз. Определенная карта, скажем, дама пик, на большом промежутке времени будет вытаскиваться с определенной относительной частотой, которую можно приблизительно предсказать. Итак, теория вероятностей связана С определением долгосрочной частоты появления данного события. Как событиям приписать их вероятности? Существует два основных подхода к решению этой задачи, один — теоретический, а другой — эмпирический. Приведенный пример проиллюстрирует разницу между этими двумя подходами. Зададим вопрос: “Какова вероятность выпадения двух очков, если игральная кость подбрасывается один раз?”. При использовании теоретического подхода следует рассуждать следующим образом: поскольку существует шесть равновозможных результатов (если игральная кость идеальная) и только один вариант, при котором выпадает два очка, то вероятность этого События равна |. Здесь вероятность приписана событию даже без повторного подбрасывания игральной кости. Как следует интерпретировать этот результат? Можно ожидать, что через длительный период времени (после многократного подбрасывания кости) два очка выпадут примерно в | случаев от количества подбрасываний. При эмпирическом подходе предположение о равной возможности выпадения игральной кости не делается. Просто проводится эксперимент, при котором подбрасывается игральная кость. Затем вычисляется процентное соотношение количества исходов, когда выпало два очка, и оценивается вероятность этого события при одном подбрасывании игральной кости. Каждый из подходов имеет свои преимущества и недостатки. Сначала рассмотрим теоретический подход и разработаем процедуры, которые приведут к решению большого количества интересных задач. Эти процедуры требуют подсчета количества способов появления определенных событий. Решить эту непростую задачу поможет мощный математический аппарат, рассматриваемый в следующих двух разделах. 6.1. Основные принципы счета ■ Правило сложения ■ Диаграммы Венна ■ Правило умножения Правило сложения Если группа студентов, изучающих химию в колледже, состоит из 13 мужчин и 15 женщин, очевидно, что общее количество студентов в группе равно 28. Это простой пример подсчета, метода определения количества элементов во множестве без перечисления элементов. Операции с множествами играют важную роль во многих ситуациях, в которых
538 Часть II. Конечная математика выполняется подсчет (операции с множествами описаны в приложении А.1, т. 2). Например, если М — множество юношей в группе студентов, a F — множество девушек, то объединением М U F множеств М и F является множество всех студентов в группе. Поскольку эти множества не имеют общих элементов, пересечение М A F множеств М и F является пустым множеством. В этом случае говорят, что М и F — непересекающиеся множества. Количество элементов множества М U F представляет собой общее количество студентов в группе и обозначается как п(М U F). Эта величина задается следующей формулой. п(М U F) = п(М) + n(F) = 13 + 15 = 28. (6.1) Таким образом, в этом примере количество элементов в объединении множеств М и F является суммой количества элементов множеств М и F. Однако в общем случае это утверждение неверно. Для того чтобы увидеть, почему, рассмотрим другой пример. Предположим, что в некоей группе 22 студента специализируются по математике, 16 — по физике и 7 — по обоим предметам. Если М — множество студентов, специализирующихся по математике, а Р — по физике, то множество М П Р представляет собой множество студентов, специализирующихся по обеим наукам. Можно порассуждать, как показано выше, и сделать вывод, что в классе 22 + 16 = 38 студентов, но это неверно. Студенты, имеющие двойную специализацию, учтены дважды — один раз как математики и еще раз — как физики. Чтобы исправить этот двойной счет, вычтем из этой суммы количество студентов, специализирующихся по двум наукам. Таким образом, общее количество студентов в списке группы задается следующей формулой. n(M UP) = п(М) + п(Р) - п(М А Р) = 22 + 16 - 7 = 31. (6.2) Равенства (6.1) и (6.2) иллюстрируют правило сложения, применяемое при подсчете элементов в объединении двух множеств. Правило сложения (для подсчета) Для любых двух множеств Аи В п(А U В) = п(А) 4- п(В) — п(А А В). (6.3) Если множества Ли В не пересекаются, то п(Л U В) = п(А) -|- п(В). (6.4) И Пример 6.1 (Услуги для сотрудников). В результате изучения компаний в некоем городе выяснилось, что 750 фирм предлагают своим служащим страхование здоровья, 640 — стоматологическое страхование и 280 — страхование здоровья и стоматологическое страхование. Сколько фирм предлагают своим служащим страхование здоровья или стоматологическое страхование? Решение, Если Н — множество фирм, которые предлагают своим служащим страхование здоровья, и D — множество фирм, предлагающих стоматологическое страхование, то Н АВ — множество фирм, предлагающих страхование здоровья и стоматологическое страхование; HUD — множество фирм, предлагающих страхование здоровья или стоматологическое страхование.
Глава 6. Теория вероятностей 539 Таким образом, п(Я) = 750, n(D) = 640, п(Я A D) = 280 и п(Н U £>) *= п(Я) + n(D) - п(Я A D) = 750 + 640 - 280 = 1110. Таким образом, 1110 фирм предлагают своим служащим страхование здоровья или стоматологическое страхование. ■ В Упражнение 6.1. Опрос, упомянутый в примере 6.1, позволил также выяснить, что 345 фирм предла¬ гают своим служащим групповое страхование жизни, 285 — страхование по долгосрочной потере трудоспособности и 115 — групповое страхование жизни и страхование по долгосрочной потере трудоспособности. Сколько фирм предлагают своим служащим групповое страхование жизни или страхование по долгосрочной потере трудоспособности? ■ Диаграммы Венна Рассмотрим следующий весьма поучительный пример. D Пример 6.2 (Исследование рынка). В городе выходит две ежедневные газеты — Sentinel и Journal. Опрос 100 жителей города показал: 35 человек подписываются на Sentinel, 60 — на Journal, а 20 — на обе газеты. 1. Сколько человек из опрошенных подписываются на Sentinel, но не подписываются на Journal! 2. Сколько подписываются на Journal, но не подписываются на Sentinel! 3. Сколько не подписываются ни на одну из газет? 4. Соберите эту информацию в таблице. Решение, Пусть множество U представляет собой группу опрошенных людей, a S и J — множество подписчиков газет Sentinel и Journal соответственно. Поскольку множество U содержит все рассматриваемые элементы, оно является универсальным множеством для этой задачи (в математической статистике употребляется также термин “генеральная совокупность”. — Прим. ред.). Дополнением S' к множеству S является совокупность респондентов в множестве U, которые не подписываются на газету Sentinel. Аналогично множеством J' является совокупность респондентов, не подписывающихся на газету Journal. Используя множества S и J, а также их дополнения и пересечение, можно разделить множество U на четыре непересекающихся подмножества, изображенных на диаграмме Венна на рис. 6.1. S A J — множество людей, которые подписываются на обе газеты, S A J' — множество людей, которые подписываются на газету Sentinel, но не подписываются на газету Journal, S' A J — множество людей, которые подписываются на газету Journal, но не подписываются на газету Sentinel, S' A J1 — множество людей, которые не подписываются ни на одну из газет. Данную обзорную информацию можно выразить в терминах обозначений множеств следующим образом. n(t/) = 100, n(S) = 35, n( J) = 60, n(S A J) = 20.
540 Часть II. Конечная математика Рис. 6.1. Диаграмма Венна для задачи учета подписчиков газет Для ответа на вопросы 1-3 можно использовать эту информацию и диаграмму Венна (Venn). Для начала пометим пересечение S A J на диаграмме числом 20 (рис. 6.2). По мере получения ответов на вопросы 1-3 изобразим соответствующие числа на диаграмме. Рис. 6.2. Количество подписчиков газет 1. Поскольку 35 человек подписываются на газету Sentinel и 20 — на обе газеты, количество подписчиков газеты Sentinel, не подписывающихся на газету Journal, равно n(S A J') = 35 - 20 = 15. 2. Аналогично количество подписчиков газеты Journal, не подписывающихся на газету Sentinel, равно n(S" A J) = 60 - 20 = 40. 3. Общее количество подписчиков газет равно 20 + 15 + 40 = 75. Таким образом, количество людей, которые не подписываются ни на одну из газет, равно n(S" A J') = 100 - 75 = 25. 4. Диаграммы Венна являются полезным инструментом для определения количества элементов в различных исследуемых множествах, но часто результат должен быть представлен в форме таблицы, а не диаграммы. Следующая таблица содержит информацию, представленную на рис. 6.2, а также включает суммарное количество элементов во множествах S, S', J, J' и U, не включенных в диаграмму Венна во избежание путаницы. Journal Общее Подписчики J Остальные количество Sentinel Подписчики S 20 15 35 Остальные 40 25 65 Общее количество 60 40 100
Глава 6. Теория вероятностей 541 В Упражнение 6.2. В маленьком городе есть две радиостанции: одна работает в AM, а другая — в FM- диапазоне. Опрос 100 жителей города дал следующие результаты: за последние 30 дней 65 человек слушали АМ-станцию, 45 человек — FM-станцию и 30 человек слушали обе станции. 1. Сколько человек из опрошенных слушали АМ-станцию, но не слушали FM-стан- цию за 30-дневный период? 2. Сколько человек слушали FM-станцию, но не слушали АМ-станцию? 3. Сколько человек не слушали ни одну из станций? 4. Представьте эту информацию в таблице. ■ Задание 6.1. Пусть А, В и С — три множества. Используя диаграмму Венна, объясните следующее равенство. n(AuBUC) - п(А) + п(В) + п(С) - п(А АВ) - п(А АС)- - n(B А С) + п(А А В П С). ■ Правило умножения Как показано выше, если элементы множества определены с помощью операции объединения, для поиска количества элементов во множестве используются операции сложения и вычитания. Теперь рассмотрим множества, которые определены с помощью последовательности операций. Как будет показано ниже, для вычисления количества элементов во множествах, образованных таким образом, используется операция умножения. Рассмотрим пример. П Пример 6.3 (Ассортимент товара). Розничный магазин торгует куртками размеров S, М, L и XL. Все куртки либо синие (В), либо красные (R). Какие возможны варианты выбора куртки и сколько их? Решение. Для того чтобы решить задачу, нарисуем диаграмму в виде дерева. Совместный выбор (исходы) (S, В) (S.R) (М,В) (M.R) (L, В) (L,R) (XL, В) (XL, R) Таким образом, существует восемь возможных способов выбора куртки (исходов). Существует четыре способа выбора размера и два способа выбора цвета. Первый элемент в упорядоченной паре представляет собой выбор размера, а второй элемент представляет собой выбор цвета. ■
542 Часть II. Конечная математика Н Упражнение 6.3. Фирма предоставляет своим служащим несколько вариантов медицинского страхования от трех различных страховых компаний R, S и Т. Каждая компания предлагает два уровня страхового покрытия, А и В, один из которых требует дополнительных вкладов со стороны служащих. Укажите варианты совместного выбора и общее количество возможных вариантов. Решите задачу, используя древовидную диаграмму. ■ Предположим, вас спросили: “Сколькими способами можно расположить 3 из 26 букв алфавита в номерном знаке автомобиля так, чтобы ни одна из букв не повторялась?”. Пытаться подсчитать возможности, используя диаграмму в виде дерева, было бы очень утомительно, если не сказать больше. Следующее правило умножения позволит решить эту задачу довольно просто. К тому же, оно создаст основу для другого вычислительного аппарата, который разработан в следующем разделе. Правило умножения 1. Если последовательно производятся две операции Oi и О2, причем для первой операции возможны N± исходов, а для второй операции — N2, то существует N1 • TV2 возможных способов последовательного выполнения двух действий. 2. В общем случае, если последовательно выполняется п операций Ох, О2, ..., Оп, имеющих Ni,N2, .. .,Nn возможных исходов соответственно, то существует Ni • N2 Nn возможных способов последовательного выполнения п действий. В примере 6.3 существует четыре возможных исхода при выборе размера (первая операция) и два возможных исхода при выборе цвета (вторая операция); следовательно, в силу правила умножения существует 4*2 = 8 возможных способов последовательного выполнения двух действий. Выполните упражнение 6.3, используя правило умножения. (Ответ: 3*2 = 6.) К вопросу о номерном знаке автомобиля: существуют 26 способов выбора первой буквы; после того, как выбрана первая буква, существует 25 способов выбора второй буквы; и после того, как выбраны две буквы, существует 24 способа выбора третьей буквы. Следовательно, используя правило умножения, получим, что существуют 26 • 25 • 24 = 15 600 возможных способов выбора трех букв из алфавита без повторений. Задание 6.2. Государство собирается принять шестисимвольный формат для номерных знаков автомобилей, состоящих из блока букв, за которым следует блок цифр. Форматом нужно снабдить до 20 миллионов автомобилей. Сколько символов нужно включить в каждый блок? ■ И Пример 6.4 (Компьютерное тестирование). Многие колледжи и университеты сейчас используют программы компьютерного тестирования. Предположим, что тест должен состоять из пяти сравнимых по сложности вопросов. В компьютере хранятся пять вариантов первого вопроса, восемь — второго, шесть — третьего, пять — четвертого и 10 — пятого. Сколько различных тестов, состоящих из пяти вопросов, может составить компьютер? (Два теста считаются разными, если они отличаются хотя бы одним вопросом.)
Глава 6. Теория вероятностей Решение. О\: выбор первого вопроса. С>2- выбор второго вопроса. О3: выбор Третьего вопроса. О4: выбор четвертого вопроса. О5: выбор пятого вопроса. 543 N\\ 5 способов. JVg Z 8 способов. 7V3: 6 способов. TV4: 5 способов. 7V5: 10 способов. Таким образом, компьютер может создать 5 • 8 • 6 • 5 • 10 = 12 000 различных тестов. Q Упражнение 6.4, Каждый вопрос теста имеет пять вариантов ответа. Сколько различных вариантов ответа существует, если тест содержит пять вопросов, а студент должен выбрать единственный вариант ответа? ■ Пример 6.5 (Кодовые слова). Сколько различных трехбуквенных паролей можно составить, используя первые восемь букв алфавита при следующих условиях? 1. Буквы не могут повторяться. 2. Буквы могут повторяться. 3. Соседние буквы не могут быть одинаковыми. Решение. Для того чтобы образовать трехбуквенное кодовое слово из восьми имеющихся букв, выберем букву для первой позиции, одну для второй позиции и одну для третьей позиции. В общей сложности есть три операции. 1. Буквы не могут повторяться. 01: выбор первой буквы. М: 8 способов. О2: выбор второй буквы. ЛГ2: 7 способов. О3: выбор третьей буквы. N3*. 6 способов. Ответ: 8 • 7 • 6 =« 336 возможных паролей. После того как выбрана одна буква. После того как выбраны две буквы. Количество способов последовательного выполнения трех действий. 2. Буквы могут повторяться. 01: выбор первой буквы. О2- выбор второй буквы. О3: выбор третьей буквы. N\\ 8 способов. 8 способов. N3: 8 способов. Повторения допускаются. Повторения допускаются. Ответ: 8 • 8 • 8 — 83 = 512 возможных паролей. 3. Соседние буквы не могут повторяться. 01: выбор первой буквы. О2: выбор второй буквы. О3: выбор третьей буквы. Wi: 8 способов. 7 способов. N3: 7 способов. Ответ: 8 • 7 • 7 =* 392 возможных пароля. Не может быть такой же, как первая. Не может быть такой же, как вторая, но может быть такой же, как первая. Упражнение 6.5. Сколько различных четырехбуквенных кодовых слов можно составить, используя первые 10 букв алфавита, при трех различных условиях, которые сформулированы в примере 6.5? ■
544 Часть П. Конечная математика Ответы к упражнениям 6.1. 515. 6.2. 1) 35. 2) 15. 3) 20. 4) — FM-станция Слушатели, чел. Остальные, чел. Общее количество, чел. АМ-станция Слушатели, чел. Остальные, чел. 30 15 35 20 65 35 Общее количество, чел. 45 55 100 6.3. Существует шесть способов выбора. 6.4. 6.5. Выбор компании Выбор покрытия Совместный выбор 3) 10-9-9-9 = 7290. Практикум 6.1 А Задачи 1—12 связаны со следующей диаграммой Венна. Найдите количество элементов в каждом из определенных ниже множеств. 1. А. 2. В. 3. и. 4. А'. 5. В'. 6. А П В. 7. Л U В. 8. А П В'. 9. А' Г) В. 10. А! П В'. 11. (ЛПВ)'. 12. (ЛиВ)'
Глава 6. Теория вероятностей 545 Решите задачи 13-16 двумя способами: 1) используя древовидную диаграмму и 2) используя правило умножения. 13. Сколькими различными способами могут упасть две монеты? Символ О обозначает орел, Р — решку? Смешанный исход (О, Р) нужно отличать от исхода (Р, О). 14. Сколько двухбуквенных кодовых слов можно образовать из первых трех букв алфавита, если никакая из букв не может использоваться более одного раза? 15. Возможные исходы подбрасывания монеты — орел, О, или решка, Р. Затем подбрасывается Одна игральная кость, возможные исходы 1,2,3,4, 5 и 6. Сколькими способами можно последовательно совершить эти два действия? 16. Сколькими способами могут упасть три монеты, если такие исходы, как (О, Р, О), (О, О, Р) и (Р, О, О), должны считаться различными (символ О обозначает орел, Р — решку)? 17. Для проведения досуга путеводитель рекомендует шесть ресторанов и три казино. а) Сколько вариантов выбора имеет семейная пара, если она идет либо обедать, либо играть? б) Пара идет обедать, а затем в казино. Сколько различных способов последовательного выбора времяпрепровождения существует? 18. В колледже предлагается два вводных курса по истории, три — по естествознанию, два — по математике, два — по философии и один — по английскому языку. а) Сколько Вариантов выбора имеет первокурсник, если он выбирает один курс из каждой области знаний? б) Сколько вариантов выбора имеет студент-заочник, если он может выбрать только один вводный курс? Б 19. Ландшафтная служба классифицирует 20 полей для гольфа в порядке возрастающей сложности как бронзовые, серебряные или золотые. Существуют только два золотых Поля и в два раза больше бронзовых, чем серебряных. а) Сколько существует вариантов выбора поля для гольфа, если игрок решает играть на серебряном или золотом поле? б) Игрок в гольф решает играть один раунд в неделю в течение трех недель, первый — на бронзовом поле, затем — на серебряном, а затем — на золотом. Сколько у него вариантов выбора? 20. Список, состоящий из 14 колледжей, включает в себя шесть дорогих колледжей (обучение стоит больше, чем 20000 долл, в год), семь отдаленных (больше 200 миль) и два дорогих и отдаленных одновременно. а) Студент решает выбрать недорогой колледж, расположенный в радиусе 200 миль от дома. Сколько вариантов выбора у него есть? б) Студент решает поступить в недорогой колледж, расположенный недалеко от дома, поучиться в нем два года, а затем перевестись в недорогой колледж, расположенный далеко от дома. Сколько вариантов выбора у него есть?
546 Часть II. Конечная математика Решая задачи 21-26, используйте информацию, приведенную на диаграмме Венна, для определения количества элементов в каждом из четырех непересекающихся подмножеств. 21. п(А) = 80, п(В) = 50, п(А П В) = 20, n(U) = 200. 22. п(А) = 45, п(В) = 35, п(А Г) В) = 15, п(У) = 100. 23. п(А) = 25, п(В) = 55, п(А U В) = 60, n(U) = 100. 24. п(А) = 70, п(В) = 90, п(АиВ) = 120, п([/) = 200. 25. п(А') = 65, п(В') = 40, п(А' П В') = 25, n(U) = 150. 26. п(А') = 35, п(В') = 75, n(A' U В') = 95, п(Г) = 120. Решая задачи 27-32, заполните следующую таблицу. А А' Всего В 9 9 9 В' ? 9 9 Всего ? 9 9 27. п(А) = 70, п(В) = 90, п(А П В) = 30, п([7) = 200. 28. п(А) = 55, п(В) = 65, п(А П В) = 35, п(17) = 100. 29. п(А) = 45, п(В) = 55, n(A U В) = 80, n(U) = 100. 30. п(А) = 80, п(В) = 70, п(АиВ) = 110, n(U) = 200. 31. п(А') = 15, п(В') = 24, n(A' U В') = 32, п(Г) = 90. 32. п(А') = 81, п(В') = 90, п(А' П В') = 63, п(С/) = 180.
Глава 6. Теория вероятностей 547 Решая задачи 33 и 34, обсудите справедливость каждого из утверждений. Если утверждение справедливо всегда, объясните почему. Если нет, приведите контрпример. * 33. Оцените справедливость следующих утверждений. а) Если А или В — пустое множество, то А и В являются непересекающимися. б) Если А И В непересекающиеся множества, то А или В является пустым множеством. * 34. Оцените справедливость следующих утверждений. а) Если Ап В непересекающиеся, то п(А Г\В) = п(А) + п(В). б) Если п(А U В) = п(А) + п(В), то А и В непересекающиеся. 35. Новая модель автомобиля может иметь пять возможных цветов, три возможных коробки передач, четыре типа интерьера и два типа двигателей. Сколько существует возможных комплектаций автомобиля? 36. В сэндвич-баре предлагаются сэндвичи со следующими наполнителями: три вида хлеба, Пять видов мяса, а также салат или капуста. Сколько различных сэндвичей предлагается, если используется один компонент из каждой категории? 37. Сколько различных четырехбуквенных кодовых слов можно составить из шести букв алфавита, если ни одна из букв не повторяется? Если буквы могут повторяться? Если соседние буквы должны быть различными? 38. Сколько различных пятибуквенных кодовых слов можно составить из первых семи букв алфавита, если ни одна из букв не должна повторяться? Если буквы могут повторяться? Если соседние буквы должны быть разными? 39. Секретный замок имеет пять колесиков, каждое из которых снабжено десятью цифрами от 0 до 9. Сколько существует комбинаций, состоящих из пяти цифр, если ни одна из цифр не повторяется? Если цифры могут повторяться? Если последовательные цифры должны быть разными? 40. Маленький секретный замок на дипломате имеет три колесика, каждое из которых снабжено десятью цифрами от 0 до 9. Сколько существует трехразрядных комбинаций, если ни одна из цифр не повторяется? Если цифры могут повторяться? Если последовательные цифры должны быть разными? 41. Сколько существует различных номерных знаков автомобилей, если каждый содержит три буквы (из 26 букв алфавита), за которыми следуют три цифры (от 0 до 9)? Сколько таких номеров содержат неповторяющиеся буквы и неповторяющиеся цифры?
548 Часть II. Конечная математика 42. Сколько существует пятиразрядных почтовых индексов? Сколько из этих чисел содержат неповторяющиеся цифры? * 43. Имеет ли значение в примере 6.3, в каком порядке производятся операции вы¬ бора? Одинаково ли количество вариантов выбора сначала цвета куртки, а затем размера, и способов выбора сначала размера, а затем цвета? Проверьте ваш ответ, используя древовидную диаграмму и правило умножения. * 44. Объясните, как три множества А, В и С должны быть связаны друг с другом, чтобы следующее равенство имело место (используйте диаграммы Венна): п(А U В U С7) = п(А) 4~ п(7?) 4~ п(С)— -п(ЛпС)-п(ВПС). В 45. В группу из 75 человек входят 32 человека, играющих в теннис, 37 человек, играющих в гольф, и 8 человек, играющих и в теннис, и в гольф. Сколько человек в группе не занимаются ни одним из этих видов спорта? 46. В группу, состоящую из 30 студентов консерватории, входят 13 человек, играющих на фортепьяно, 16 человек, играющих на гитаре, и 5 человек, играющих и на фортепьяно, и на гитаре. Сколько студентов в классе не играют ни на одном из этих инструментов? 47. В туристическую группу из 100 человек, путешествующих по Европе, входят 42 человека, владеющих французским языком, 55 человек, владеющих немецким языком, и 17 человек, не говорящих ни на одном из этих языков. Сколько человек в группе говорят как на французском, так и на немецком языках? 48. В футбольную команду высшей школы, состоящую из 40 игроков, входят 16 игроков, игравших в нападении в прошлом году, 17 игроков, игравших в защите, и 12, не игравших в прошлом году. Сколько игроков в прошлом году играли как в нападении, так и в защите? Применение математики Экономика и бизнес 49. Подбор персонала. Служба подбора персонала классифицирует служащих (используя тесты и опросы) по индексу интеллектуального развития: высокий (Н), средний (М) и низкий (L), а также как агрессивных (А) или пассивных (Р). Сколько существует различных вариантов классификации? а) Решите, используя древовидную диаграмму. б) Решите, используя правило умножения. 50. Подбор персонала. Корпорация планирует заполнить две разные вакансии вице- президентов Vi и 14 административными служащими на двух своих производственных предприятиях. На предприятии А есть шесть кандидатов, а на предприятии Б — восемь. Сколькими способами можно заполнить эти две вакансии,
Глава 6. Теория вероятностей 549 если вакансия Vi должна заполняться служащими предприятия Л, a — служащими предприятия Б1 Сколькими способами можно заполнить эти вакансии, если выбор делается без учета предприятия? 51. Логистика. Торговый представитель, живущий в городе А, хочет на самолете посетить три разных города — Б, В и Г. Существует два способа добраться до аэропорта (ехать на собственном автомобиле в аэропорт и из аэропорта или использовать такси для обоих путешествий), и все города связаны авиалиниями. Сколько различных способов путешествия существует, если каждый город нужно посетить один раз и вернуться домой? 52. Логистика. Компания-производитель в городе А хочет перевозить на грузовиках свою продукцию в четыре разных города — Б, В, Г и Д. Все города связаны дорогами. Сколько различных планов транспортировки может быть составлено так, чтобы отдельный грузовик, выехавший из Я, посетив каждый город ровно один раз, вернулся домой? 53. Исследование рынка. В результате опроса 1200 потребителей в одном из городов выяснилось, что 850 человек являются владельцами микроволновых печей, 740 — владельцами видеомагнитофонов, а 580 — имеют и микроволновые печи, и видеомагнитофоны. а) Сколько человек из опрошенных имеют либо микроволновую печь, либо видеомагнитофон? б) Сколько людей не имеют ни микроволновой печи, ни видеомагнитофона? в) Сколько людей имеют микроволновую печь и не имеют видеомагнитофона? 54. Исследование рынка. В результате опроса 800 малых фирм выяснилось, что у 250 из них есть фотокопировальные устройства, у 420 — факсы и у 180 — и то, и другое. а) Сколько фирм из опрошенных имеют либо фотокопировальное устройство, либо факс? б) Сколько фирм не имеют ни фотокопировального устройства, ни факса? в) Сколько фирм имеют факс и не имеют фотокопировального устройства? 55. Связь. Компания кабельного телевидения обслуживает 8000 пользователей в пригородной зоне. Компания предлагает два платных канала, НВО и Showtime: 2450 пользователей принимают НВО, 1940 принимают Showtime и 5180 не принимают ни один из платных каналов. Сколько пользователей принимают и НВО, и Showtime! 56. Связь. Местная телефонная компания предлагает своим 10 000 потребителей две специальные услуги: переадресацию вызова и ожидающий вызов. 3770 потребителей используют переадресацию, 3250 используют ожидающий вызов и 4530
550 Часть II. Конечная математика не используют ни одну из этих услуг. Сколько потребителей используют как переадресацию, так и ожидающий вызов? 57. WWW Минимальный вес. В таблице ниже приведены данные о количестве мужчин и женщин, достигших минимального веса или имеющих вес ниже этого уровня для различных возрастных категорий. Общее ко- Возрастная группа личество, 16-19 лет 20-24 года старше 25 лет тыс. чел Мужчины, достигшие минимального уровня веса, тыс. чел. 343 154 237 734 Мужчины, имеющие вес ниже минимального уровня, тыс. чел. 118 102 159 379 Женщины, достигшие минимального уровня веса, тыс. чел. 367 186 503 1056 Женщины, имеющие вес ниже минимального уровня, тыс. чел. 251 202 540 993 Общее количество, тыс. чел. 1079 644 1439 3162 а) Сколько мужчин в возрасте от 20 до 24 лет имеют вес ниже минимального уровня? б) Сколько женщин в возрасте больше 20 лет достигли минимального веса? в) Сколько человек в возрасте 16-19 лет или среди мужчин достигли минимального уровня веса? г) Сколько человек имеют вес ниже минимального уровня? 58. Минимальный вес. Вернемся к задаче 57. а) Сколько женщин в возрасте 16-19 лет достигли минимального уровня веса? б) Сколько мужчин в возрасте 16-24 лет имеют вес ниже минимального уровня? в) Сколько человек в возрасте 20-24 лет или среди женщин имеют вес ниже уровня минимального? г) Сколько человек достигли минимального уровня веса? Биологические науки 59. Медицина. В процессе медицинских исследований пациентов классифицируют по полу — мужской (М) или женский (Ж), приверженности к курению — курящие (К) или некурящие (Н), а также комплекции — худые (X), обычные (О) или полные (П). Сколько существует различных возможных способов классификации? а) Решите, используя древовидную диаграмму. б) Решите, используя правило умножения. 60. Планирование семьи. Семейная пара планирует иметь трех детей. Сколько возможно различных комбинаций “девочка-мальчик”? Различайте такие комбинации, как (А/, М, Д), (М, Д, М) и (Д, М, М). а) Решите, используя древовидную диаграмму. б) Решите, используя правило умножения.
Глава 6. Теория вероятностей 551 Социальные науки 61. WWW Политика. Политик, избирающийся на третий срок, планирует встретиться со всеми спонсорами первых двух избирательных кампаний. В первую избирательную кампанию сделали вклад 1475 человек, во вторую — 2350, а в первую и вторую — 920. Сколько человек сделали вклады в первую или вторую избирательные кампании? 62. Политика. При первом избрании за политика проголосовали 12457 человек, при втором избрании — 15 322, причем 9345 человека проголосовали за него оба раза. Сколько человек проголосовали за этого политика хотя бы один раз? 6.2. Перестановки и сочетания ■ Факториалы ■ Перестановки ■ Сочетания ■ Решение практических задач Правило умножения, рассмотренное в предыдущем разделе, можно использовать для создания двух дополнительных приемов счета, которые чрезвычайно полезны при решении более сложных вычислительных задач. Оба эти приема используют функцию, называемую факториалом. Факториалы При использовании правила умножения появились выражения вида 26 • 25 • 248 • 7 • 6, в которых каждый натуральный множитель уменьшался на единицу при движении слева направо. Множители в следующем произведении продолжают уменьшаться на единицу до тех пор, пока не станут равными единице. 5-4-3-2-1 Произведения такого вида настолько часто встречаются в вычислительных задачах, что полезно выразить их в краткой форме. Произведение первых п натуральных чисел называется п-факториалом и обозначается как п\. Кроме того, нуль-факториал (0!) считается равным единице. Факториал Факториал для натурального числа п определяется по следующим правилам. п! = п{п - 1)(п -2) 2-1 4! = 4 • 3 • 2 • 1 0! =* 1 п! — п • (и — 1)! {Замечание', на многих калькуляторах есть клавиша п\ или ее эквивалент.)
552 Часть II. Конечная математика Пример 6.6 (Вычисление факториалов). 1. 5! = 5-4-3-2-1 = 120. 7-^ = 7 2. = 6! 8! _ 5! ~ 4. 5!47! 3. М 8- 7- 6-М о п о — = 8 • 7 • о = 336. 52 • 51 • 50 • 49 • 48 • 47! Л = 2 598960. Упражнение 6.6. Вычислите следующие величины. 2. 9! 20! 5' 3!17!‘ 1. 6!. 3. 1* 7! 4. —. 0!3! 5 • 4 - 3 • 2 • 1 • 471 Интересно и полезно отметить, что величина п! растет очень быстро. Сравните следующие величины. 5! = 120, 10! = 3 628 800, 15! = 1307 674 368 000. Вычислите с помощью калькулятора величины 69!, 70! и 71!. Перестановки Определенное (горизонтальное) расположение множества картин на стене называется перестановкой множества картин. Обобщим это определение. Перестановка множества предметов Перестановкой множества определенных предметов называется расположение предметов в определенном порядке без повторения. Предположим, что нужно расположить четыре картины на стене художественной галереи слева направо. Сколько перестановок (упорядоченных расстановок) существует? Используя принцип умножения следует рассуждать так. Существует четыре способа выбора первой картины; после того, как выбрана первая картина, существует три способа выбора второй картины, после того, как первые две картины выбраны, существует два способа выбора третьей картины; и после того, как первые три картины выбраны, существует только один способ выбора четвертой. Таким образом, количество перестановок (упорядоченных расположений) множества из четырех картин равно 4 • 3 • 2 • 1 = 4! = 24. Сколько существует перестановок множества из п предметов в общем случае? Используя рассуждения, приведенные выше, приходим к выводу, что существует п способов выбора первого предмета, п — 1 способов выбора второго предмета и т.д.
Глава 6. Теория вероятностей 553 Количество перестановок п предметов Количество перестановок п определенных предметов без повторения, которое обозначается как Рп,п, равно Рп,п — п(п — 1) 2 ♦ 1 = п! п множителей Пример', количество перестановок 7 предметов равно ^7,7 = 7*6*5*4*3*2*1 = 7! 7 множителей Теперь предположим, что директор художественной галереи решает использовать только два из имеющихся полотен, причем они будут расположены на стене слева направо. Теперь речь идет об определенном расположении двух картин из четырех, которое называется размещением из четырех элементов по два. Размещение из п предметов по г Размещение множества из п определенных предметов по г без повторений — это расположение г из п предметов в определенном порядке. Сколько упорядоченных расположений двух картин можно получить из четырех картин? Иначе говоря, сколько существует размещений из четырех предметов по два? Существует четыре способа выбора первой картины; после того, как выбрана первая, существует три способа выбора второй. Таким образом, количество размещений множества из четырех предметов по два, которое обозначается как Р^з, выражается следующей формулой. Р4,2 =4*3. Используя факториалы, эту формулу можно переписать так. р _ _ 4 • 3 • 2! _ 4! Умножаем числитель 4 ♦ 3 на единицу, 4,2 2! ~ 2! представленную в виде 2!/2! Рассуждая таким Же образом, как в примере, находим, что количество размещений из п определенных предметов по г без повторений (0 г п) выражается следующей формулой. Рп,г — п(п — 1)(п — 2) (п — г + 1) г множителей ^9,6 = 9(9 — 1)(9 — 2) (9 — 6 + 1) = 9*8*7*6*5’4 6 множителей Умножая правую часть равенства для Рп>г на единицу, представленную в виде (п—г) !/(п— - г)!, получим формулу для РП)Г, записанную с помощью факториалов. Рп,г = п(п - 1)(п - 2) (п - г + (п — г)! Однако, поскольку п(п — 1)(п — 2) (п — г 4- 1)(п — г)! = п!, выше приведенное выражение упрощается. р =^- п’г (п-г)Г
554 Часть II. Конечная математика Подытожим эти результаты следующим образом. Количество размещений из п предметов по г Количество размещений из п определенных предметов по г без повторений равно1 Рпг — п(п — 1)(п — 2) (п — г + 1) г множителей ?5,2 = 5*4 Два множителя или п! (п — г)! ’ О г п, РЬ,2 = 5! _ 5! (5 -2)! ” 3! (Замечание: Рп,п = = п! — количество размещений из п предметов по п. Напомним, что по определению 0! = 1.) Пример 6.7 (Вычисление факториалов). Дано множество {А, В, С}. Сколько существует размещений этого множества из трех предметов по два? Найдите ответ на этот вопрос, используя следующие средства. 1. Древовидную диаграмму. 2. Правило умножения. 3. Две формулы для Рп/ Решение. 1. Используя древовидную диаграмму, приходим к следующему выводу. Существует шесть размещений из трех предметов по два. 2. Используя правило умножения, получаем следующие результаты. Оу заполнение первого места. Ny три способа. О2- заполнение второго места. N2: два способа. Таким образом, существует 3*2 = 6 размещений из трех предметов по два. 3. Используя две формулы для Рп,г, получаем следующее. Два множителя I 31 3*2*1 Р3 2 = 3 • 2 = 6 или Рз,2 = * . = — = 6. V5 “ 2 " 1 Таким образом, существует шесть размещений из трех предметов по два. Разумеется, все три метода дают один и тот же ответ. ■ вместо Рп>г часто используются обозначения Р™пРг и Р(п,г).
Глава 6. Теория вероятностей 555 Упражнение 6.7. Дано множество {А, В, С, D}. Сколько существует размещений этого множества из четырех предметов по два? Найдите ответ на этот вопрос, используя разные средства. 1. Древовидную диаграмму. 2. Правило умножения. 3. Две формулы для Рп>г. в Правило умножения, возможно, покажется наиболее простым методом в примере 6.7. Однако для больших значений п и г более удобно использовать факториалы. Действительно, существует много калькуляторов, которые вычисляют п! и РП)Г (см. рис. 6.1 в примере 6.3 и справочное руководство к калькулятору). Пример 6.8 (Размещения). Найдите количество размещений из 13 предметов по 8, используя калькулятор. Решение. Используем формулу в виде факториалов для Рп,г. 13! 13! Р^=(йТ8)! = ^= 51891840' Применение древовидной диаграммы для решения этой задачи потребует громадных усилий. Использование правила умножения потребует вычисления величины 13 • 12 • 11 • 10 • 9 • 8 • 7 • 6, состоящей из восьми множителей, что не так уж плохо. Калькулятор может дать мгновенный результат (рис. 6.3). ■ 13! 6227020800 135/5! 51891840 13 пРг 8 51891840 Рис. 6.3. Решение задачи с помощью графического калькулятора Упражнение 6.8. Найдите количество размещений из 30 предметов по 4, используя калькулятор. ■ Сочетания Предположим, что в художественном музее есть восемь полотен выдающихся мастеров, и другой художественный музей хочет взять на время три из этих полотен для специальной выставки. При выборе трех картин из восьми порядок не имеет значения, просто необходимо выбрать трехэлементное подмножество из множества, содержащего восемь полотен. Иначе говоря, необходимо выбрать то, что называется сочетанием из восьми предметов по три.
556 Часть II. Конечная математика Сочетание из п предметов по г Сочетанием из п предметов по г предметов без повторения является r-элементное подмножество множества, состоящего из п элементов. Порядок элементов в подмножестве не имеет значения. Сколько существует вариантов выбора трех из имеющихся восьми полотен? Иначе говоря, чему равно количество сочетаний из восьми предметов по три? Для того чтобы ответить на вопрос и лучше понять суть общей задачи, вернемся к примеру 6.7. В примере 6.7 было описано множество {А, В, С} и вычислено количество размещений из трех предметов по два с использованием древовидной диаграммы. С помощью этой диаграммы можно также определить количество сочетаний из трех предметов по два (количество двухэлементных подмножеств трехэлементного множества) и сравнить его с количеством перестановок (рис. 6.4). АВ АС ВА ВС (А, В) (А, С) (В, С) (В, А) (С, А) (С, В) {А, В} И, Q {B,Q Шесть перестановок трех объектов, взятых по два. (Порядок имеет значение.) Три перестановки трех объектов, взятых по два. (Порядок не имеет значения.) Рис. 6.4. Сочетания и перестановки Сочетаний существует меньше, чем перестановок, чего и следовало ожидать. Каждому подмножеству (сочетанию) соответствуют две упорядоченные пары (перестановки). Обозначим количество сочетаний, изображенных на рис. 6.4, через Сз,2 ИЛИ Q). Наша конечная цель — вычислить величину С*п,г с помощью факториала. Сначала получим формулу для 0*3,2, а затем обобщим полученный результат. Известно, что количество размещений из трех предметов по два обозначается как Рз,2. Применим формулу для вычисления этого числа. Теперь опишем вычисление величины Рз,2 с помощью двух операций. Oi: выбор подмножества из двух элементов. N^: С*з,2 способов. Ог- упорядочение подмножества в данном порядке. TV2: 2! способов. Последовательное выполнение операций Oi и О2 позволяет вычислить количество размещений из трех предметов по два. рэ о Рз,2 — 0*3,2 • 2! или С*з,2 = ~2!^*
Глава 6. Теория вероятностей 557 Для того чтобы найти количество сочетаний из трех предметов по два, подставим в эту формулу величину Рз$ = (з1’2)| и решим ее относительно Сз^- 3! 3-2.1 3,2 2!(3 - 2)! (2-1)(1) Этот результат согласуется с тем результатом, который получен с использованием диаграммы в виде дерева. Отметим, что количество сочетаний из трех предметов по два равно количеству размещений из трех предметов по два, деленному на количество перестановок элементов двухэлементного подмножества. Это наблюдение можно также сделать, анализируя рис. 6.4. Проводя аналогичные рассуждения, получим, что количество сочетаний из п предметов по г выражается следующей формулой. Р м! Сп.г =* = -77—:—77, Поскольку Рп>г = — . г! ri(n-r)! (п — г)! Количество сочетаний из п предметов по г Количество сочетаний из п определенных предметов по г без повторения вычисляется по формуле2 Рп,г Рб2,5 г! ~ ~~ 5! — п! 52! = О г л = г!(п —г)!’ " " 5!(52 — 5)! Теперь можно ответить на вопрос о картинах. г = 8! = 8! L LIlO* 1 ™ 8,3 3!(8-3)! 3!5!: З-2-l-SfT ' Итак, существует 56 способов выбора трех картин для отправки. Иначе говоря, существует 56 сочетаний из восьми предметов по три. Пример 6.9 (Размещения и сочетания). Комитет состоит из 10 человек. 1. Сколькими способами можно выбрать председателя комитета, вице-председателя и секретаря, предполагая, что один человек не может занимать более одной должности? 2. Сколькими способами можно выбрать подкомитет из трех человек? Решение. Отметим, чем отличаются задачи 1 и 2. В задаче 1 имеет значение порядок выбора членов комитета. В задаче 2 порядок выбора не играет никакой роли. Таким образом, в задаче 1 интерес представляет количество размещений из 10 человек по три, 2Вместо символов Сп,г и (£) часто используются символы и С(п, г).
558 Часть II. Конечная математика а в задаче 2 — количество сочетаний из 10 человек по три. Эти величины вычисляются следующим образом (и поскольку числа не велики, нет необходимости использовать калькулятор). !• Ло,з — 2. Cio,3 = 10! _ 10! (10-3)! ~ ТГ - 10! _ 10! 3!(10 — 3)! ~ 3!7! •10 ”9 - 8 '• if ; — 720 способов. •id "d ”s "if : _ :3<2>b7f f 120 способов. Упражнение 6.9. Комитет состоит из 12 человек. 1. Сколькими способами можно выбрать председателя, вице-председателя, секретаря и бухгалтера, если один человек не может занимать более одной должности? 2. Сколькими способами можно выбрать подкомитет из четырех человек? ■ Если пит — большие числа (как в примере 6.9), то при вычислении выражений, включающих факториалы, следует применять калькулятор. Многие калькуляторы имеют функции, непосредственно вычисляющие величину СП)Г (рис. 6.5). 13 пСг 8 1287 13 пРг 8 51891840 13 пРг 8/8! 1287 Рис. 6.5. Решение задачи с помощью графического калькулятора Пример 6.10 (Сочетания). Найдите количество сочетаний из 13 предметов по 8. Найдите точный ответ, используя калькулятор. Решение, „ /13\ 13! 13! С13,8 “ ( 8 )~ 8!(13 - 8)! “ 8!5! “ 87’ ■ Сравните результат решения примера 6.10 с результатом, полученным при решении примера 6.8, и обратите внимание на то, что С1з,8 значительно меньше, чем Р1з,8 (см. рис. 6.5). Упражнение 6.10. Вычислите количество сочетаний из 30 предметов по четыре. Найдите точный ответ, используя калькулятор. ■ Запомните В размещениях порядок имеет значение. В сочетаниях порядок значения не имеет.
Глава 6. Теория вероятностей 559 Для того чтобы определить характер задачи (с размещениями или с сочетаниями), проверьте, приводит ли перестановка элементов множества к другому результату. Если да, используйте размещения, если нет — сочетания. Задание 6.3. 1. Перечислите все трехбуквенные коды, состоящие из разных букв, которые выбраны из множества букв М, А, Т, Н. Определите, как этот перечень связан с величиной РП)Г. 2. Перепишите перечень из задачи 1 так, чтобы первыми шли все коды, не содержа¬ щие буквы М, Затем — все коды, не содержащие буквы А, затем — все коды, не содержащие буквы Т, и наконец — все коды, не содержащие буквы Н. Как этот перечень иллюстрирует формулу РП)Г = г!СП)Г? ■ Решение практических задач Теперь опишем некоторые приложения рассмотренных выше понятий. Некоторые приложения в этом и следующих разделах рассматривают стандартную игральную колоду из 52 карт, которая описана ниже. Стандартная игральная колода из 52 карт Стандартная колода из 52 карт (рис. 6.6) имеет четыре масти, содержащие по 13 карт: бубны, черви, трефы И пики. Бубны и черви — красные, трефы и пики — черные. Каждая масть содержит 13 карт, среди которых перенумерованные от 2 до 10, валет, дама, король и туз. Валет, дама и король часто называются фигурами (face card). В зависимости от игры туз может считаться самой младшей и/или самой старшей картой масти. А ♦ A < A 4 A i 4 I 4 I 4 ♦ 4 3 A 3 ▲ ♦ A U 4 ♦4 4 u * lA A .4 4 ♦4 4 U 4 7 д Д <A A Ia ♦ h ♦ фА a >.4 ♦4.4 U.4 i£51L ■ ~ 4. ; и; ¥ \' hr-.. Рис. 6.6. Колода карт Пример 6.11 (Техника счета). Сколько карточных раскладов из пяти карт содержат три туза и два короля?
560 Часть II. Конечная математика Решение. Применим правило умножения и сочетания. Представим процедуру выбора расклада из пяти карт с помощью следующих двух операций. Ох : выбор трех тузов из четырех возможных (порядок не важен). О2 • выбор двух королей из четырех возможных (порядок не важен). М : С4,з N2 : С4,2 Количество раскладов = С±^С±,2 = 4! 4! 3!(4 — 3)! * 2!(4 — 2)! = 4 • 6 = 24. Упражнение 6.11. Сколько раскладов из пяти карт содержат три черви и четыре пики? Пример 6.12 (Техника счета). Код товара включает две буквы, за которыми следуют три цифры. Сколько существует кодов товара, в которых буквы должны быть выбраны из первых восьми букв алфавита без повторений, а цифры — из 10 цифр (0-9) без повторений? Решение. Применим правила умножения и размещений. Представим процедуру выбора кода товара с помощью следующих двух операций. Ох- выбор двух букв из восьми имеющихся (порядок важен). TVi: Pg,2 О2: выбор трех цифр из 10 имеющихся (порядок важен). N2: Рю,з Количество кодов товара = Ре,2^10,з = 8! 10! (8-2)! ‘ (10-3)! = 56-720 = 40320. Упражнение 6.12. Повторите решение примера 6.12 при тех же условиях, за исключением того, что код товара должен иметь три буквы, за которыми следуют две цифры (без повторений). ■ Пример 6.13 (Техника счета). У компании есть семь старших и пять младших служащих. Необходимо сформировать профсоюзный комитет. Сколькими способами можно сформировать комитет из четырех служащих при следующих условиях? 1. Необходимо выбрать любых четырех служащих. 2. Необходимо выбрать четырех старших служащих. 3. Необходимо выбрать трех старших служащих и одного младшего служащего. 4. Необходимо выбрать двух старших и двух младших служащих. 5. Необходимо выбрать по крайней мере двух старших служащих. Решение. 1. Поскольку в компании работают 12 служащих, количество разных комитетов из четырех человек равно 12! 12! С12’4 = 4!(12 - 4)! = 4!8! = 495’
Глава 6. Теория вероятностей 561 2. Если в комитет могут входить только старшие служащие, количество различных комитетов равно 7! 7! 677,4 = 41(7-4)! = 413! = 35' 3. Три старших служащих могут быть выбраны способами, а один младший служащий может быть выбран способами. Применяя правило умножения, получим, что количество способов, с помощью которых могут быть выбраны три старших и один младший служащий, равно 7! 5! 7!5! С7’3 ‘ 676,1 = 31(7 — 3)! ‘ 1!(5 — 1)! = 3!4!1!4! = 175’ 7! 5! 7!5! 4. С7>2 • С5>2 = 21(7-2)! ‘ 21(5 — 2)! = 2!5!2!3! = 21°’ 5. Комитеты, состоящие по крайней мере из двух старших служащих, могут быть разделены на три непересекающихся множества: 1) комитеты с четырьмя старшими служащими без младших; 2) комитеты с тремя старшими служащими и одним младшим; 3) комитеты с двумя старшими и двумя младшими служащими. Количество вариантов комитетов типов 1, 2 и 3 было вычислено при решении задач 2-4 соответственно. Общее количество комитетов всех трех типов является суммой этих величин. Тип 1 Тип 2 Тип 3 С7т,4 + Ст,з * ^5,1 “И Ст,2 • Сб,2 = 35-1- 175 -И 210 — 420. ■ Упражнение 6.13. Используя информацию из примера 6.13, дайте ответ на следующие вопросы. 1. Сколько можно сформировать комитетов, состоящих из четырех человек, в которые входили бы один старший служащий и три младших? 2. Сколько можно сформировать комитетов, состоящих из четырех человек, в которые входили бы четыре младших служащих? 3. Сколько можно сформировать комитетов, состоящих из четырех служащих, в которые входили бы по крайней мере два младших служащих? ■ Задание 6.4. 1. Вычислите числа Сп,о, Спд, Сп>2,..., СП)П для и = 4,5,6. Рассмотрите полученные закономерности. 2. Вычислите сумму Cn,o + Cn,i + Сп>2 + • • • + СП)П и знакочередующуюся сумму С*п,о “ Cn,i 4* С*П)2 — • —F (—1)пСП)П для п = 4,5,6. Сделайте предположение о сумме и знакочередующейся сумме для п = 7 и проверьте ваше предположение. в
562 Часть II. Конечная математика Ответы к упражнениям 6.6. 1) 720. 2) 10. 3) 720. 4) 20. BCD А С D А В D АВ AC AD ВА ВС BD СА СВ CD 12 перестановок 4-х объектов, взятых по два 5) 1140. D АВС DA DB DC Двенадцать перестановок из четырех объектов, взятых по два. 2) Oi : заполнение первого места. О2 : заполнение второго места. 4-3 = 12 М : четыре способа. TV2 • 3 способа. 4! 3) Р4,2 = 4 • 3 = 12; Р4,2 = = 12. 6.8. 6.9. 6.10. 30! Рзо,4=(30^4)!= 657720- 12! —-п = 11880 способов. (12 - 4)! 12! 4!(12-4)!= 495 способов. 30! = 27405. 1) Р12,4 = 2) С12>4 = 6.11. 6,30,4 4! (30 - 4)! С13,з • С1з,2 = 22 308. 6.12. Р8,з-Ло,2 = 30240. 6.13. 1) С7,2 • С5)з = 70. 2) С5,4 = 5. 3) Cjt2 • С.5,2 + Суд • С*5,3 + Сб,4 = 285. Практикум 6.2 А В задачах 1-24 вычислите указанные выражения. 10 . 15! 1. 7!. 2. 8!. 3. 77 ■ 4. —. 8! 12! , 100! , 1001! „ 6! „ 7! 5. . 6. . 7. . 8. . (100 - 3)! (1001 - 2)! 3!3! 2!5! „ 9! Ю! .. 9! ,, 18! 9. . 10. . 11. . 12. . 4!(9 - 4)! 5!(10 — 5)! 2!3!4! 3!5!10! 13. Р7,з- 14. С*7,з- 15. С8,8. 16. Р8,8. 17. Сб2,з- 18. С52,49- 19. -7*52,3 • 20. Ps2,5- 21. 22. ,, С'гб.г * £*26,2 ,4 С*13,2^13,3 С52,5' Сб2,3 Сб2,4 С52,5
Глава 6. Теория вероятностей 563 Должны ли в задачах 25-30 рассматриваться размещения, сочетания или ни то и ни другое? Объясните ваши рассуждения. * 25. Новый ректор университета назначил трех новых служащих: проректора по фи¬ нансам, проректора по академической работе и проректора по работе со студентами. * 26. Ректор университета выбрал двух из своих проректоров для участия в церемонии открытия новой ветви университетского городка. * 27. Студент выбрал в библиотеке четыре романа для чтения на каникулах. * 28. В качестве подарка студент приобрел четыре книги: по одной для отца, матери, младшей сестры и старшего брата. * 29. Отец купил рожок мороженого (шоколадного, ванильного или клубничного) для каждого из своих четырех детей. * 30. Клуб любителей книг собирается ежемесячно дома у одного из своих 10 членов. В декабре клуб выбирает места встреч на следующий год. 31. Сколько разных вариантов среди трех первых мест возможны в забеге, если в нем принимают участие 10 лошадей? (Исключите одновременные финиши.) 32. Сколько разных вариантов среди пяти первых мест возможны в забеге на длинную дистанцию, если в нем принимают участие 50 человек? (Исключите одновременные финиши.) 33. Сколькими способами можно выбрать подкомитет, состоящий из трех членов комитета, в состав которого входят семь человек? Сколькими способами можно выбрать председателя, вице-председателя и секретаря комитета, в состав которого входят семь человек? 34. Девять карточек пронумерованы цифрами от 1 до 9. Выбирается расклад из трех карточек: последовательно по одной карточке. Сколько раскладов можно создать при следующих условиях? а) Порядок учитывается. б) Порядок Не учитывается. Б *35. Рассмотрите относительный порядок роста функций х!, З1 и х3. *36. Рассмотрите относительный порядок роста функций х!, 2х и х2. 37. Сколько раскладов, состоящих из пяти карт стандартной колоды, в состав которой входят 52 карты, будут содержать две черви? 38. Сколько раскладов, состоящих из пяти карт стандартной колоды, в состав которой входят 52 карты, будут содержать все фигуры? Все фигуры, но без королей? 39. Сколько раскладов, содержащих семь карт стандартной колоды, в состав которой входят 52 карты, имеют ровно пять пик и две черви? 40. Сколько раскладов, содержащих пять карт стандартной колоды, в состав которой входят 52 карты, содержат две трефы и три черви?
564 Часть II. Конечная математика 41. В ресторане предлагается 8 закусок, 10 основных блюд и 7 десертов. Комитет по организации банкета должен выбрать три закуски, четыре основных блюда и два десерта. Сколькими способами он может это сделать? 42. В трех подразделениях работают 12, 15 и 18 членов профсоюза соответственно. Каждое отделение должно выбрать делегата и его заместителя, которые будут представлять отделение на конференции. Сколькими способами можно это сделать? В задачах 43 и 44 обратитесь к приведенной ниже таблице, изображенной на дисплее калькулятора, где у\ = РП)Г и у2 = Сп?г для п = 6. X Vi ив 0 1 1 1 6 б г 30 15 з 120 20 ч 360 15 5 720 б б 720 1 1 nCr X *43. Рассмотрите и объясните симметрию чисел в столбце у2. ★ 44. Объясните, как таблица иллюстрирует формулу Рп,г = в 45. На окружности выбраны восемь точек. а) Сколько можно провести хорд, соединяя точки всеми возможными способами? б) Сколько треугольников можно образовать, используя эти восемь точек в качестве вершин? в) Сколько четырехугольников можно образовать, используя эти восемь точек в качестве вершин? 46. На окружности выбраны пять точек. а) Сколько хорд можно провести, соединяя точки всеми возможными способами? б) Сколько треугольников можно образовать, используя эти пять точек в качестве вершин? 47. Сколькими способами два человека могут сесть в ряд из пяти стульев? Три человека? Четыре человека? Пять человек? 48. Каждая из двух стран посылает пять делегатов на переговоры. Используется прямоугольный стол с пятью стульями вдоль каждой длинной стороны. Сколько существует способов рассадить делегатов, если каждой стране предоставляется длинная сторона (операция 1)? 49. В баскетбольной команде есть пять специализаций игроков. Сколько можно образовать стартовых команд при следующих условиях. а) Специализация игрока учитывается.
Глава 6. Теория вероятностей 565 б) Специализация игрока не учитывается. в) Специализация игрока не учитывается, но либо Майк, либо Кен (но не оба одновременно) должны входить в стартовый состав. 50. Сколько комитетов из четырех человек можно образовать из группы, состоящей из девяти человек при следующих условиях. а) Нет ограничений. б) И Джим, и Мэри должны войти в состав комитета. в) Либо Джим, либо Мэри должны войти в состав комитета. 51. Найдите наибольшее целое к такое, что ваш калькулятор может вычислить kl без ошибки перегрузки. 52. Найдите наибольшее целое к такое, что ваш калькулятор может вычислить С^к.к без ошибки перегрузки. 53. Обратите внимание на то, что в таблице, созданной графическим калькулятором, наибольшее значение СПуГ при п = 20 равно Сгодо = 184 756. Используйте аналогичную таблицу для поиска наибольшего значения СП)Г при п = 24. X IM 7 В 9 12 12 13 77520 125970 167960 1ВЧ756 167960 125970 77520 ViB20 nCr X 54. Обратите внимание на то, что в таблице, созданной графическим калькулято¬ ром, наибольшее значение Сп>г при п = 21 равно С21Д0 = С*21,и = 352 716. Используйте аналогичную таблицу для поиска наибольшего значения СП)Г при X НИ 7 В 10 11 13 1162B0 203490 293930 352716 352716 293930 203490 ViB21 nCr X Применение математики Экономика и бизнес 55. Контроль качества. Компьютерный магазин получил оборудование, состоящее из 24 лазерных принтеров, среди которых пять сломанных. Для демонстрации в магазине выбирают три принтера. а) Сколько всего существует способов выбора? б) Сколько из выбранных вариантов не будут содержать сломанные принтеры?
566 Часть II. Конечная математика 56. Контроль качества. Магазин электроники получил оборудование, состоящее из 30 графических калькуляторов, среди которых шесть сломанных. Четыре из этих калькуляторов планируется отправить в ближайшую среднюю школу. а) Сколько существует способов выбора? б) Сколько из выбранных вариантов не будут содержать сломанные калькуляторы? 57. Закрытие бизнеса. Компания, управляющая сетью восьми ювелирных магазинов в Джорджии, 12 — во Флориде и 10 — в Алабаме, планирует закрыть 10 магазинов. а) Сколько вариантов существует в этой ситуации? б) Компания решает закрыть два магазина в Джорджии, пять — во Флориде и три — в Алабаме. Сколько вариантов существует в этой ситуации? 58. Увольнение служащих. В компании, торгующей недвижимостью, работают 14 служащих в центральном офисе, 8 — в северном и 6 — в южном. Руководство планирует уволить 12 служащих. а) Сколько вариантов есть у руководства компании? б) Компания решает уволить пять служащих из центрального офиса, четыре — из северного и три — из южного. Сколько вариантов есть у руководства компании? 59. Подбор персонала. Предположим, что из шести претендентов женщин и пяти мужчин были успешно избраны служащие на пять должностей. Сколько существует вариантов выбора следующих составов? а) Три женщины и два мужчины. б) Четыре женщины и один мужчина. в) Пять женщин. г) Пять человек независимо от пола. д) По крайней мере четыре женщины. 60. Выбор комитета. Комитет по рассмотрению конфликтных ситуаций должен быть избран из двух отделений Л и Б, содержащих 15 и 20 человек соответственно Сколько существует вариантов выбора комитетов со следующими составами? а) Три человека из отделения А и один человек из отделения Б. б) Два человека из отделения А и два человека из отделения Б. в) Все из отделения А. г) Четыре человека независимо от отделения. д) По крайней мере три человека из отделения А. Биологические науки 61. WWW Медицина. Существует восемь стандартных классификаций типов крови. В ходе экзамена на должность лаборанта каждый кандидат должен определить типы трех образцов крови. Сколько всего различных наборов образцов может быть предложено, если все образцы, входящие в набор, имеют разные типы? Если несколько образцов могут иметь один тип?
Глава 6. Теория вероятностей 567 62. Медицинское исследование. В силу ограниченности средств из восьми подходящих исследовательских центров для изучения заболеваний сердца выбраны пять. Сколько всего возможно вариантов выбора? Социальные науки 63. Политика. Съезд для выбора кандидатов в президенты и вице-президенты должен сделать выбор из четырех кандидатов. Карточки с именами президента и вице-президента должны быть заготовлены для каждого возможного исхода до выборов. Сколько разных типов карточек должно быть заготовлено? 64. Политика. Сколькими способами шесть кандидатов могут быть перечислены в бюллетене? 6.3. Пространство элементарных исходов, события и вероятность ■ Эксперименты ■ Пространства элементарных исходов и события ■ Вероятность события ■ Предположение о равновозможности В этом разделе дано относительно краткое и неформальное введение в теорию вероятностей. Более детальные и формальные трактовки можно найти в книгах, полностью посвященных этому предмету. Изучение теории вероятностей включает в себя много идей, поэтому сначала необходимо освоить фундаментальные понятия. Эксперименты Первый шаг в построении математической модели теории вероятностей — описание случайных экспериментов. Некоторые эксперименты не приводят к одному и тому же постоянному результату, независимо от того, насколько тщательно они повторяются при одних и тех же условиях. Такие эксперименты называются случайными. Известными примерами случайных экспериментов являются подбрасывание монет, игральной кости, наблюдение за частотой дефектных единиц на сборочной линии или наблюдение за частотой смертей в определенной возрастной группе. Теория вероятностей является ветвью математики, развитие которой связано с исходами случайных экспериментов, как реальных, так и теоретических. Далее слово эксперимент используется для обозначения случайного эксперимента. Пространства элементарных исходов и события С результатами экспериментов связаны пространства элементарных исходов и события. Вторым шагом в построении математической модели для изучения теории вероятностей является определение этих двух терминов. Для этого оказываются полезными понятия теории множеств, которые рассмотрены в приложении А.1, т. 2. Рассмотрим следующий эксперимент: “Колесо с 18 числами, расположенными по периметру (рис. 6.7), раскручивается и крутится до остановки так, чтобы стрелка указала на пронумерованный сектор”.
568 Часть II. Конечная математика Рис. 6.7. Рулетка Какие исходы можно рассматривать в этом эксперименте? Когда колесо останавливается, можно интересоваться, какое число является следующим за указателем, является ли это число четным, делится ли оно на 5, является ли оно простым, остановилась ли стрелка на затемненном или белом секторе и т.д. Перечень возможных исходов бесконечен. В общем случае не существует единственного метода анализа всех возможных исходов эксперимента. Следовательно, прежде, чем производить эксперимент, важно решить, какие исходы представляют интерес. Предположим, что наш интерес ограничен множеством чисел, написанных на колесе, и различными его подмножествами, например, множеством простых или четных чисел. Решив, за чем наблюдать, составим перечень результатов эксперимента, которые называются элементарными исходами или элементарными событиями. Результатом эксперимента может быть один и только один исход, указанный в перечне. Выберем в качестве элементарных событий числа на колесе и образуем множество S = {1,2,3,..., 17,18}. Множество S называется пространством элементарных исходов эксперимента. Теперь рассмотрим следующий исход: “Когда колесо остановилось, число, следующее за указателем, делится на 4”. Этот исход не является элементарным, поскольку он не связан с одним и только одним элементом в пространстве S. Этот исход считается состоявшимся, если указатель останавливается на одном из чисел: 4, 8, 12 или 16, т.е. на элементе множества Е = {4,8,12,16}. Подмножество Е называется составным событием (а исход — сложным исходом). Пространства элементарных исходов и события Если пространство исходов S формируется таким образом, что каждое испытание имеет один и только один исход, множество S называется пространством элементарных событий данного эксперимента. Каждый элемент в множестве S называется элементарным исходом или элементарным событием. Событием Е называется любое подмножество множества S (включая пустое множество 0 и пространство S). Событие Е называется элементарным, если оно содержит только один элемент, и составным, если оно содержит несколько элементов. Говорят, что событие Е происходит, если имеет место элементарный исход, принадлежащий множеству Е. Термины событие и исход эксперимента являются синонимами.
Глава 6. Теория вероятностей 569 Действительность Эксперимент (реальный или теоретический) Исход (элементарный или составной) Математическая модель Пространство элементарных исходов (множество S) Событие (подмножество 5; элементарное или составное) Пример 6.14 (Элементарное и составное события). Рассмотрим пример с колесом, на котором написаны числа (см. рис. 6.7), и пространство элементарных событий S = = {1,2,3,..., 17,18}. Что собой представляет событие Е (подмножество пространства элементарных исходов S), соответствующее каждому из следующих элементарных исходов? Определите, является это событие элементарным или составным. 1. Исходом является простое число. 2. Исходом является квадрат числа 4. Решение, 1. Исход является простым числом, если происходит какое-то из элементарных событий 2, 3, 5, 7, 11, 13 или 17.3 Выпадение простого числа эквивалентно появлению одного из элементов множества Е = {2,3,5,7,11,13,17}. Поскольку событие Е содержит несколько элементов, оно является составным. 2. Исход является квадратом числа 4, только если указатель остановится на числе 16. Таким образом, исход эксперимента принадлежит множеству Е = {16}. Поскольку множество Е содержит только один элемент, оно является элементарным. ■ Упражнение 6.14. Повторите решение примера 6.14 при следующих условиях. 1. Исходом является число, кратное 12. 2. Исходом является четное число, превышающее 15. ■ Пример 6.15 (Пространство элементарных событий). Подбрасываются две монеты — номиналом 5 и 10 центов. Как определить пространство элементарных исходов этого эксперимента? Существует несколько возможностей. Рассмотрим три из них. 1. Если интерес представляет то, какая монета выпала орлом (О) или решкой (Р), то, используя древовидную диаграмму, легко определить соответствующее пространство элементарных событий для эксперимента. Монета 5 центов Монета 10 центов (исходы) Начало (исходы) _____ ° Р ° "С===~~-~ Р Составные (исходы) ОО ОР РО рр 3 Элементарные события представляют собой не числа, а одноэлементные множества, содержащие эти числа. Таким образом, правильнее писать {2}, {3}, {5}, {7}, {11}, {13} и {17}, а не 2, 3, 5, 7, И, 13 и 17. Однако для простоты записи мы будем отождествлять одноэлементное множество и содержащееся в нем число.
570 Часть II. Конечная математика Таким образом, Si = {ОО, OP, РО, РР}, и в пространство элементарных событий входит четыре элементарных события. 2. Если интерес представляет только количество орлов, выпавшее при одном подбрасывании двух монет, можно положить S2 = {0,1,2},ив пространство элементарных событий входят три элементарных события. 3. Если интерес представляет, совпали ли (Д) стороны монет или нет (Н), можно положить S3 = {Д, Н}. Тогда в пространстве элементарных исходов существует только два элементарных события. ■ Какое пространство элементарных событий в примере 6.15 является наиболее общим? Пространство элементарных событий Si содержит больше информации, чем пространство S2 или S3. Если известно, какой исход имел место в пространстве Si, то известно, какой исход имел место в пространствах S2 и S3. Однако обратное неверно. (Отметим, что элементарные события в пространствах S2 и S3 являются составными событиями в пространстве Si.) Говорят, что множество Si является более фундаментальным пространством элементарных исходов, чем множества S2 или S3. Таким образом, в качестве наиболее широкого пространства элементарных исходов следует выбрать множество Si. Выбор пространства элементарных исходов Для каждого конкретного эксперимента не существует единственно верного пространства элементарных исходов. При определении пространства элементарных исходов для эксперимента следует учитывать столько деталей, сколько необходимо для ответа на все интересующие экспериментатора вопросы. Если возникает сомнение, следует выбирать более широкое пространство элементарных исходов. Упражнение 6.15. Экспериментатор записывает состав семьи, имеющей двух детей, учитывая их пол: мальчик (М) или девочка (Д). 1. Что представляет собой соответствующее пространство элементарных исходов, если необходимо определить вероятность определенного пола ребенка, зная порядок рождения детей? Нарисуйте древовидную диаграмму. 2. Что представляет собой соответствующее пространство элементарных исходов, если экспериментатора интересует только количество девочек в семье? 3. Что представляет собой соответствующее пространство элементарных исходов, если экспериментатора интересует только, одинаков (О) пол детей или нет (Р)? 4. Что представляет собой соответствующее пространство элементарных исходов для всех трех вопросов, сформулированных в п.п. 1-3? ■ Пример 6.16 (Пространства элементарных исходов и события). Рассмотрим эксперимент, состоящий в подбрасывании двух игральных костей. Удобное пространство элементарных событий, которое позволит ответить на многие вопросы об интересующих нас событиях, изображено на рис. 6.8. Пусть S — множество всех упорядоченных пар, представленных на рисунке. Элементарное событие (3, 2) следует отличать от элементарного события (2, 3). Первое означает, что число 3 выпало на первой игральной кости,
Глава 6. Теория вероятностей 571 а число 2 — на второй, в то время как второе событие означает, что число 2 выпало на первой игральной кости, а число 3 — на второй. Вторая игральная кость ц (1,1) 0,2) (1,3) (1,4) (1,5) (1,6) ■ (2,1) (2,2) (2,3) (2,4) (2,5) (2,6) и (3,1) (3,2) (3,3) (3,4) (3,5) (3,6) 1111 (4,1) (4,2) (4,3) (4,4) (4,5) (4,6) (5,1) (5,2) (5,3) (5,4) (5,5) (5,6) (6,1) (6,2) (6,3) (6,4) (6,5) (6,6) Рис. 6.8. Пространство элементарных исходов Что собой представляет событие (подмножество пространства элементарных событий S), соответствующее каждому из следующих исходов? 1. Сумма очков равна 7. 2. Сумма очков равна 11. 3. Сумма очков меньше 4. 4. Сумма очков равна 12» Решение. 1. Под высказыванием “Сумма очков равна 7” подразумеваем, что сумма всех выпавших очков на обоих игральных костях равна 7. Этот исход соответствует событию {(6,1), (5,2), (4,3), (3,4), (2,5), (1,6)}. 2. Высказывание ‘‘Сумма очков равна 11” соответствует событию {(6,5), (5,6)}. 3. Высказывание “Сумма очков меньше 4” соответствует событию {(1,1), (2,1), (1,2)}. 4. Высказывание “Сумма очков равна 12” соответствует событию {(6,6)}. ■
572 Часть II. Конечная математика Упражнение 6.16. Обратитесь к пространству элементарных событий, изображенному на рис. 6.8. Что собой представляют события, соответствующие каждому из следующих исходов? 1. Сумма очков равна 5. 2. Сумма очков равна простому числу, которое больше 7. ■ Вероятность события Следующий шаг в разработке математической модели для изучения теории вероятностей — введение функции вероятности. Это функция, которая приписывает произвольному событию, связанному с пространством элементарных исходов, действительное число, которое больше или равно нулю и меньше или равно единице. Рассмотрим способы, с помощью которых элементарным событиям в пространстве элементарных событий S приписываются вероятности. Вероятности элементарных событий Для данного пространства элементарных исходов S = {ei, ег,..., еп} каждому элементарному событию ei приписывается действительное число Р(е;), которое называется вероятностью события е^. Эти числа могут приписываться произвольно до тех пор, пока соблюдены следующие два условия. 1. Вероятность элементарного события является числом, которое больше или равно нулю и меньше или равно единице, т.е. О P(ei) 1. 2. Сумма вероятностей всех элементарных событий в пространстве элементарных событий равна единице, т.е. P(ei) + Р(е2) + • • • + P(en) = 1. Говорят, что любое распределение вероятностей, удовлетворяющее условиям 1 и 2, является приемлемым. Наша математическая теория не объясняет, как приемлемые вероятности приписываются элементарным событиям. Как правило, это зависит от ожидаемого или действительного процентного соотношения количества возникновений элементарного события при многократном повторении эксперимента. Распределение вероятностей, основанное на этом принципе, называется обоснованным. Пусть эксперимент состоит в подбрасывании одной монеты. В качестве пространства элементарных событий выберем множество S = {О,Р}. Если монета правильной формы и сбалансирована, то мы склоняемся к тому, чтобы приписывать вероятности элементарных событий в множестве S следующим образом. P(O) = i и Р(Р) = |. Эти действия аргументируются так. Поскольку монета может лечь двумя способами, в отдаленном будущем орел будет выпадать в половине случаев подбрасывания монеты,
Глава 6. Теория вероятностей 573 следовательно, решка будет выпадать в половине случаев. Эти значения вероятностей являются приемлемыми, поскольку оба условия для приемлемых распределений вероятностей, сформулированные выше, удовлетворяются. 1. О Р(О) 1, 0 Р(Р) 1. 2. Р(О) + Р(Р) = | + | = 1. Если подбросить монету 1000 раз, можно ожидать, что орел появится примерно (но не точно) в 500 случаях. Для моделирования 1000 подбрасываний монеты можно использовать случайные числа, сгенерированные графической утилитой. На рис. 6.9 показаны результаты трех таких сеансов моделирования: 497 орлов первый раз, 495 орлов второй раз и 504 орлов третий раз. randBin<1000?.5? 3) <497 495 504} Рис. 6.9. Результаты моделирования Однако, если в 1000 подбрасываниях монеты выпало 376 орлов, можно подозревать, что монета несбалансирована (или имеет неправильную форму). Тогда можно приписать элементарным событиям в пространстве элементарных событий S следующие вероятности, основанные на полученных экспериментальных результатах. Р(О) = 0,376 и Р(Р) = 0,624. Однако, хотя распределение вероятностей Р(О) = 1 и Р(Р) = 0 также приемлемо, оно не является обоснованным (если монета имеет две стороны). Распределение вероятностей Р(О) = 0,6 и Р(Р) = 0,8 не является приемлемым, поскольку 0,6 -I- 0,8 = 1,4, что нарушает условие обоснованности. Важно иметь в виду, что среди бесконечного количества возможных приемлемых распределений вероятностей элементарным событиям пространства элементарных событий предпочтение отдается распределению, основанному на экспериментальных результатах. Как определить вероятность произвольного события Е, связанного с пространством элементарных событий S, для данного приемлемого распределения вероятностей по элементарным событиям?
574 Часть II. Конечная математика Вероятность события Е Для данного приемлемого распределения вероятностей в пространстве элементарных событий S вероятность произвольного события Е, обозначенную как Р(Е), определим следующим образом. 1. Если Е — пустое множество, то Р(Е) = 0. 2. Если Е — элементарное событие, то величина Р(Е} уже определена. 3. Если Е — составное событие, то величина Р(Е) представляет собой сумму вероятностей элементарных событий, входящих в множество Е. 4. Если множество Е совпадает с пространством элементарных событий S, то Р(Е) = P(S) = 1. Пример 6.17 (Вероятности событий). Обратимся к примеру 6.15, в котором бросаются монеты номиналом 5 и 10 центов. Пространство элементарных событий имеет вид S = {ОО, ОР,РО,РР}. Поскольку существует четыре элементарных исхода, и монеты правильно сбалансированы, должно оказаться, что в отдаленном будущем каждый исход появится в 25% случаев подбрасывания монет. Припишем одну и ту же вероятность, равную |, каждому элементарному событию в пространстве S. Элементарное событие ei 00 ОР РО РР РЫ 1 4 1 4 1 4 1 4 В соответствии с условиями 1 и 2 это приемлемое и обоснованное распределение вероятностей для идеальных (правильно сбалансированных) монет или монет, близких к идеальным. 1. Чему равна вероятность выпадения одного орла (и одной решки)? 2. Чему равна вероятность выпадения по крайней мере одного орла? 3. Чему равна вероятность выпадения по крайней мере одного орла или по крайней мере одной решки? 4. Чему равна вероятность выпадения трех орлов? Решение. 1. Е\ = {Выпадение одного орла} = {OP, РО}. Поскольку событие Е± является составным, используем правило 3, указанное в определении вероятности, и найдем величину Р(Е\) путем суммирования вероятностей элементарных событий в множестве Е\. Р(Е1) = Р(ОО) + Р(РО) = | + | = ^. 2. Е2 = {выпадение по крайней мере одного орла} = {ОО, OP, РО}; 1113 Р{Е2) = Р(0О) + Р(ОР) + Р(РО) =4+5+5=? 3. Ез = {00, ОР, РО, РР} = S. Р(Е3) = P(S) = 1. l + l + l + l = 1’ 4 4 4 4
Глава 6. Теория вероятностей 575 4. Ед = выпадение 3 орлов = 0 Пустое множество Р(0) = 0. Этапы вычисления вероятности события Е Этап 1. Определим для эксперимента соответствующее пространство элементарных исходов S. Этап 2. Припишем приемлемые вероятности элементарным событиям из пространства S, Этап 3. Вычислим вероятность произвольного события Е, складывая вероятности элементарных событий, принадлежащих множеству Е. Функция Р, определенная в п. 2 и 3, называется функцией вероятности, областью определения которой являются все возможные события (подмножества) в пространстве элементарных исходов S, а областью значений — отрезок [0, 1]. Упражнение 6.17. Предположим, что при подбрасывании монет номиналом 5 и 10 центов в примере 6.16 оказалось, что событие ОО произошло 273 раза, ОР — 206 раз, РО — 312 раз и РР — 209 раз. На основании этого факта элементарным событиям в пространстве S приписаны следующие вероятности. Элементарное событие ej ОО ОР РО РР Р(е<) "0,273 0,206 0,312 0,209 Это приемлемое и обоснованное распределение вероятностей элементарных событий из пространства S. Чему равны вероятности следующих событий? 1. Ei = {Выпадение по крайней мере одной решки}. 2. Е2 = {Выпадение двух решек}. 3. Е3 = {Выпадение по крайней мере одного орла или по крайней мере одной решки}. ■ Пример 6.17 и упражнение 6.17 иллюстрируют два важных способа приемлемого и обоснованного распределения вероятностей в пространстве элементарных событий S. Каждый подход имеет свое преимущество в конкретных ситуациях. 1. Теоретический. Для распределения вероятностей элементарных событий используются предположения и процесс дедуктивного рассуждения. В действительности никакие эксперименты не проводятся. Именно так мы решали пример 6.17. 2. Эмпирический. Вероятности приписываются элементарным событиям на основании результатов реальных экспериментов. Именно так необходимо выполнять упражнение 6.17. Более точно понятие эмпирической вероятности формулируется следующим образом: если эксперимент производится п раз и событие Е появляется с частотой f(E), то отношение f(E)/n называется относительной частотой появления события Е в п испытаниях. Определим эмпирическую вероятность Е, обозначаемую через Р(Е), как число
576 Часть II. Конечная математика (если такое существует), к которому стремится относительная частота f(E)/n с увеличением п. Для любого определенного п относительная частота f(E)/n также называется приближенной эмпирической вероятностью события Е. Приближенная эмпирическая вероятность частота появления Е f(E) Р(Е) « — = ±-^-. общее количество испытаний п (Чем больше п, тем точнее приближение.) Предположение о равновозможности В примере с подбрасыванием монет номиналом 5 и 10 центов каждое элементарное событие в пространстве элементарных исходов S = {ОО, OP, РО, РР} имело одинаковую вероятность, равную Приписывая одинаковую вероятность каждому элементарному событию из пространства S, мы в действительности делаем предположение, что появления всех элементарных событий одинаково возможны. Назовем его предположением о равновозможности. Вероятность элементарного события при предположении о равновозможности Если предполагается, что все элементарные события в пространстве элементарных событий S = {ei, в2,..., еп} являются равновозможными, то каждому событию приписывается вероятность 1/п, т.е. P(eJ = Предположение о равновозможности позволяет получить очень полезную формулу для вычисления вероятностей произвольных событий, связанных с пространством элементарных исходов S. Рассмотрим следующий пример. Если бросается игральная кость при предположении, что выпадение каждой грани одинаково вероятно, то в пространстве элементарных исходов S = {1,2,3,4,5,6} каждому элементарному событию приписывается вероятность |, поскольку существует шесть элементарных событий. Вероятность события Е = {Выпадение простого числа} = {2,3,5} равна Количество элементов в Е I Р(Е) = Р(2) + Р(3) + Р(5) = 1 + 1 + 1 = | = 1 0 0 0 0 2 т Количество элементов в S Таким образом, при предположении, что все элементарные события равновозможны, вычисление вероятности появления любого события Е в пространстве элементарных исходов S состоит в делении количества элементов в множестве Е на количество элементов в пространстве S.
Глава 6. Теория вероятностей 577 Теорема 6.1 (Вероятность произвольного события при предположении равновозмож- ности). При предположении, что все элементарные исходы пространства S одинаково вероятны, вероятность произвольного события Е в пространстве S равна Р(Е) = число элементов в Е п(Е) число элементов в S n(S) ’ Пример 6.18 (Вероятности и предположения о равновозможности). Рассмотрим снова подбрасывание двух игральных костей и предположим, что все элементарные исходы в пространстве S, изображенном на рис. 6.8, одинаково вероятны. Найдите вероятности следующих событий. 1. Ех = {Выпала сумма, равная 7}. 2. Еъ = {Выпала сумма, равная 11}. 3. Е$ = {Выпала Сумма меньше 4}. 4. Е4 = {Выпала сумма, равная 12}. Решение. Обращаясь к рис. 6.8 и результатам, полученным в примере 6.16, находим следующие результаты. 1- P(E1) = ^W = ^ = |- Bl = {(6,1), (5,2), (4,3), (3,4), (2,5), (1,6)} п\Ъ) оо о 2. Р{Е2) = Г^- = ^ = у Е2 = {(6,5),(5,6)} ОО 10 ( \ п (Ез) 3 1 3- Р = = ™ = Вз = {(1,1), (2,1), (1,2)} П [D) 00 12 . _ ч п (Е4) 1 4- Р(р4) = ^Г = -Е4 = {(6,6)} ■ П(Ь) 00 Упражнение 6.18. При условиях, сформулированных в примере 6.18, найдите вероятности следующих событий (каждое событие связано с суммой точек, изображенных на обеих игральных костях). 1. Е$ = {Выпала сумма, равная 5}. 2. Ев = {Выпала Сумма, равная простому числу, которое больше 7}. ■ Пример 6.19 (Моделирование и эмпирические вероятности). Используйте случайные числа, полученные с помощью графической утилиты, для моделирования 100 подбрасываний двух игральных костей. Определите эмпирические вероятности следующих случайных событий и сравните их с теоретическими вероятностями. 1. Ех = {Выпала сумма, равная 7}. 2. Е2 = {Выпала сумма, равная И}. Решение. Для выбора случайного целого числа от 1 до 6 можно использовать графическую утилиту. Выбор каждого из чисел данного интервала равновероятен. Следовательно,
578 Часть II. Конечная математика чтобы смоделировать подбрасывание двух игральных костей, необходимо выбрать случайное целое число от 1 до 6, добавить к нему второе случайное целое число от 1 до 6 и записать сумму (см. первую команду, изображенную на рис. 6.10, а и справочное руководство). Вторая команда, изображенная на рис. 6.10, а, моделирует 100 одновременных подбрасываний двух игральных костей. Суммы очков записываются в список Li. Анализируя гистограмму Li, изображенную на рис. 6.10, б, получим эмпирические вероятности.4 randlnt(1, 6)+ran dlnt<l,6) 10 randlnt(l?6>100) +randlnt(l>6?100 )->Li <11 6 9 7 2 6 7... Рис. 6.10. Эмпирические вероятности 1. Эмпирическая вероятность события Е\ равна = 0,15; теоретическая вероятность события Ei (см. пример 6.18, а) равна = 0,167. 2. Эмпирическая вероятность события Е2 равна = 0,06; теоретическая вероятность события Е2 (см. пример 6.18, б) равна = 0,056. ■ ю. Упражнение 6.19. Используйте результаты работы графической утилиты, изображенные на рис. 6.10, б, для определения эмпирических вероятностей следующих событий и сравните их с теоретическими вероятностями. 1. Е$ = {Выпала сумма, которая меньше 4}. 2. Ед = {Выпала сумма, равная 12}. I Задание 6.5. Контейнер содержит 12 графических калькуляторов, среди которых два сломанных. Сначала из контейнера случайным образом вынимается один калькулятор, а затем, без возвращения, — второй. Будут ли приемлемыми предположения о равновозможности элементарных исходов в пространстве S = {XX, ХС, СХ, СС}, где X — исправный калькулятор, а С — сломанный? I Теперь перейдем к некоторым примерам, которые используют приемы, разработанные в предыдущих разделах. Пример 6.20 (Вероятность и равновозможность). Чему равна вероятность вытащить пять карт пиковой масти, вытаскивая без возвращения пять карт из колоды, состоящей из 52 карт? 4Если вы моделируете этот эксперимент с помощью графической утилиты, не следует ожидать тех же эмпирических вероятностей.
Глава 6. Теория вероятностей 579 Решение. Пространство элементарных событий S представляет собой множество всех пятикарточных раскладов, полученных из колоды, состоящей из 52 карт. Поскольку порядок в раскладе не имеет значения, n(S) = 052,5• Пусть событие Е представляет собой множество всех пятикарточных раскладов, полученных из 13 карт пиковой масти. И снова порядок не имеет значения и п(Е) = 613,5. Таким образом, предполагая, что каждый пятикарточный расклад равновероятен с любым другим, получаем следующий результат. Р(Е) = L 1287 О 0005 ’ n(S) С52,5;.2598'960J ’ Упражнение 6.20. Чему равна вероятность вытащить семь карт червовой масти, вытаскивая без возвращения семь карт из колоды, состоящей из 52 карт? ■ Пример 6.21 (Вероятность и равновозможность). Совет правления университета состоит из 12 мужчин и 16 женщин. Случайным образом выбирается комитет, состоящий из шести человек. Чему равна вероятность того, что он будет состоять из трех мужчин и трех женщин? Решение. Пусть S — множество всех комитетов, состоящих из шести человек, избранных из 28 человек. Тогда п(5) = 628,6» Пусть Е — множество всех комитетов, состоящих из шести человек, в которые входят три мужчины и три женщины. Для того чтобы найти число п(Е), используем принцип умножения и следующие две операции. 01: выбор трех мужчин из 12 возможных. М: 612,3- О2: выбор трех женщин из 16 возможных. N2: 61б,з- Таким образом, п(2?) = М • N2 = 612,3 ’ 616,3 И р/рч П (Е) С12.3 • Схб.З Р(Е)-М3)~ Сад « 0,327. Упражнение 6.21. Чему равна вероятность того, что комитет в примере 6.21 будет состоять из четырех мужчин и двух женщин? ■ Необходимо отметить, что существует много вычислительных задач, в которых невозможно получить простую формулу, дающую количество возможных исходов. В ситуациях такого типа часто обращаются к древовидной диаграмме и подсчету ветвей. Задание 6.6. Пять футбольных команд состязаются друг с другом в Высшем дивизионе. В начале сезона всем интересно предугадать относительное расположение пяти команд в турнирной таблице в конце сезона. Одинаковые положения исключаются. Опишите соответствующее пространство элементарных событий для этой задачи. Сколько элементарных событий оно содержит? Какова вероятность определенного окончательного расположения, если все возможные окончательные расположения пяти команд равновероятны? Рассмотрите причины, по которым спортивный комментатор должен или не должен высказывать предположение о равновозможности окончательного расположения пяти команд. ■
580 Часть II. Конечная математика Ответы к упражнениям 6.14. 1) Е = {12}; элементарное событие. 2) Е = {16,18}; составное событие. 6.15. 1) Si = {ММ, МД, ДМ, ДД}; Пол первого Пол второго ребенка ребенка Составные (исходы) ММ мд дм да 2) S2 = {0,1,2}. 3) S3 = {О, Р}. 4) Sj. 6.16. 6.17. 1) 1) 6.18. 1) 2) {(6,5), (5,6)}. 3) 1. 2) Р(Е6) = ± io 6.19. 1) 2) {(4,1), (3,2), (2,3), (1,4)}. 0,727. 2) 0,209. 9 3 = 0,09 (эмпирическая); — = 0,083 (теоретическая). —= 0,01 (эмпирическая); = 0,028 (теоретическая). 100 36 6.20. С1з,7/С52,7«1>3-10-5. 6.21. (712,4^16,2/^28,6 0,158. Практикум 6.3 А * 1. Как интерпретировать выражение Р(Е) = 1? * 2. Как интерпретировать выражение Р(Е) = 0? Идеальная игральная кость раскрашена так, что три стороны красные, две — белые и одна — голубая. Игральная кость бросается один раз. В задачах 3-6 найдите вероятности следующих событий. 3. Верхняя сторона голубая. 4. Верхняя сторона красная или белая. 5. Верхняя сторона красная, белая или голубая. 6. Верхняя сторона не белая. Обратитесь к описанию стандартной колоды, состоящей из 52 карт, и рис. 6.6. Эксперимент состоит в вытаскивании одной карты из стандартной колоды. В задачах 7-12 определите вероятность следующих событий. 7. Извлечена карта красной масти. 8. Извлечена карта бубновой масти. 9. Извлечена фигура. 10. Извлечен туз или король. 11. Извлечен черный валет. 12. Извлечена червонная дама.
Глава 6. Теория вероятностей 581 13. Чему равна вероятность того, что в семье, в которой родились двое детей, дети разнополые? Исключите рождение близнецов. Предположите, что рождения девочки и мальчика равновероятны. 14. Чему равна вероятность того, что в семье, в которой родились двое детей, есть две девочки? Исключите рождение близнецов. Предположите, что рождения девочки и мальчика равновероятны. * 15. В магазине продаются четыре модели CD-плейеров — J, G, Р и S. Из прошлых записей менеджер определил, что относительная частота выбора модели плейера потребителями изменяется. Какие из распределений вероятностей выбора определенной модели плейера потребителями нужно отбросить? Почему? а) Р( J) = 0,15, P(G) = -0,35, Р(Р) = 0,50, P(S) = 0,70. б) Р( J) = 0,32, P(G) = 0,28, Р(Р) = 0,24, Р(5) = 0,30. в) P(J) = 0,26, P(G) = 0,14, Р(Р) = 0,30, P(S) = 0,30. 16. При использовании распределения вероятностей в задаче 15.в определите, чему равна вероятность того, что случайный потребитель не выберет модель S. 17. При использовании распределения вероятностей в задаче 15.в определите, чему равна вероятность того, что случайный потребитель выберет модель J или модель Р. 18. При использовании распределения вероятностей в задаче 15.в определите, чему равна вероятность того, что случайный потребитель не выберет модель J или модель Р. Б 19. В семье трое детей. Чему равна вероятность того, что родились сначала два мальчика, а затем девочка? Рождение близнецов исключается. Предположите, что рождение мальчика и рождение девочки равновероятны. 20. В семье трое детей. Чему равна вероятность того, что среди них два мальчика и одна девочка (родились в любом порядке)? Рождения близнецов исключаются. Предположите, что рождение девочки и рождение мальчика равновероятны. 21. Маленький секретный замок на дипломате имеет три колесика, каждое из которых снабжено 10 цифрами от 0 до 9. Открывающей комбинацией является определенная последовательность трех цифр без повторения. Чему равна вероятность угадать правильную комбинацию? 22. Секретный замок имеет пять колесиков, каждое снабженное 10 цифрами от 0 до 9. Открывающая комбинация представляет собой определенную последовательность пяти цифр без повторений. Чему равна вероятность указать правильную комбинацию? Обратитесь к описанию стандартной колоды, состоящей из 52 карт и рис. 6.5. Эксперимент состоит в сдаче пяти карт из стандартной колоды. В задачах 23-26 найдите вероятности следующих событий. 23. Извлечены пять карт черной масти. 24. Извлечены пять карт червонной масти. 25. Извлечены пять фигур.
582 Часть II. Конечная математика 26. Извлечены пять карт, которые не являются фигурами. *27. В списки государственного университета внесены двадцать тысяч студентов. Выберем случайным образом студента и запишем его или ее день рождения (месяц и день, но не год). Опишите пространство элементарных исходов для такого эксперимента и припишите им приемлемые вероятности. Какие предположения нужно сделать при этом? * 28. В напряженном соперничестве на выборах в Сенат США два первых кандидата набирают приблизительно одинаковое количество голосов, а за третьего кандидата готовы отдать голоса только 50% избирателей, от проголосовавших за одного из первых двух кандидатов. Случайным образом выбирается легитимный избиратель. У него интересуются, за кого из трех кандидатов он отдает свой голос. Опишите соответствующее пространство элементарных исходов для этого исследования и припишите им приемлемые вероятности. 29. Предположим, что написаны пять благодарностей и адреса на пяти конвертах. Благодарности случайным образом вкладываются в конверты и отправляются без сопоставления с адресатом. Чему равна вероятность того, что все благодарности вложены в конверты правильным образом? 30. Предположим, что шесть человек сдали свои пальто в гардероб. Допустим, что все номерки утеряны, и шесть пальто возвращаются случайным образом. Чему равна вероятность того, что пальто вернется к хозяину? Эксперимент состоит в подбрасывании двух идеальных игральных костей и сложении количества точек на двух выпавших гранях. Используя пространство элементарных событий, изображенное на рис. 6.8, и предполагая, что все элементарные события одинаково вероятны, найдите вероятности событий, определенных в задачах 31-46. 31. Сумма равна 2. 33. Сумма равна 6. 35. Сумма меньше 5. 37. Сумма не равна 7 или 11. 39. Сумма равна единице. 41. Сумма делится на 3. 43. Сумма равна 7 или 11. 45. Сумма делится на 2 или 3. Эксперимент состоит в подбрасывании трех идеальных монет. У одной из монет орел нанесен сразу на обе стороны. Вычислите вероятности получения определенных результатов в задачах 47-52. 47. Один орел. 49. Три орла. 51. Более одного орла. * 53. Ответьте на следующие вопросы. а) Можно ли получить 19 орлов при 20 подбрасываниях идеальной монеты? Поясните. 32. Сумма равна 10. 34. Сумма равна 8. 36. Сумма больше 8. 38. Сумма не равна 2, 4 или 6. 40. Сумма равна 13. 42. Сумма делится на 4. 44. Сумма равна 2, 3 или 12. 46. Сумма делится на 2 и 3. 48. Два орла. 50. Ни одного орла. 52. Более одной решки.
Глава 6. Теория вероятностей 583 б) Нужно ли подозревать, что монета неидеальная, если она подброшена 40 раз и получены 37 орлов? Почему да или почему нет? Какие эмпирические вероятности нужно приписать элементарным исходам, если есть подозрения, что монета неидеальная? * 54. Ответьте на следующие вопросы. а) Можно ли получить семь двойных шестерок при 10 подбрасываниях пары игральных костей? Поясните. б) Нужно ли подозревать, что две игральные кости неидеальные, если при 36 подбрасываниях 11 раз получены двойные шестерки? Почему да или почему нет? Какая эмпирическая вероятность должна быть приписана случайному событию, состоящему в выпадении двух пятерок, если возникают подозрения, что игральные кости шулерские? В Эксперимент состоит в подбрасывании двух идеальных игральных костей и сложении количества точек на выпавших гранях. Каждая игральная кость имеет число 1 на двух противоположных гранях, число 2 на двух других противоположных гранях и число 3 на оставшихся противоположных гранях. Вычислите вероятности получения сумм, определенных в задачах 55-62. 55. 2. 56. 3. 57. 4. 58. 5. 59. 6. 60. 7. 61. Сумма, равная нечетному числу. 62. Сумма, равная четному числу. Вернемся к стандартной колоде, состоящей из 52 карт. Эксперимент состоит в том, что сдаются пять карт из стандартной колоды. В задачах 63-70 требуется найти вероятности следующих событий. 63. Извлечены пять фигур или тузы. 64. Извлечены пять пронумерованных карт (от 2 до 10). 65. Извлечены четыре туза. 66. Извлечены четыре карты одного типа (четыре дамы, четыре короля и т.д.). 67. Извлечены десятка, валет, дама, король и туз (все одной масти). 68. Извлечены карты 2, 3, 4, 5 и 6 одной масти. 69. Извлечены два туза и три дамы. 70. Извлечены два короля и три туза. В задачах 71-74 смоделированы несколько экспериментов, использующих случайные числа, полученные с помощью графической утилиты. Например, подбрасывание идеальной игральной кости может быть смоделировано выбором случайного целого числа от 1 до 6, а 50 подбрасываний игральной кости — выбором 50 случайных целых чисел от 1 до 6 (см. рис. 6.11 и справочное руководство). 71. Из статистического графика исходов подбрасывания идеальной игральной кости 50 раз (см. рис. б) видно, например, что число 4 выпало ровно 11 раз. а) Чему равна эмпирическая вероятность того, что выпало число 6?
584 Часть II. Конечная математика randlnt(1> 6) 5 randlntC1,6,50)* Li <5 5 4 3 5 1 3... 0 6) Рис. 6.11. Иллюстрация к задачам 71-74 a) б) Чему равна вероятность того, что выпало число 6 при предположении равновероятности? в) Используйте графическую утилиту для моделирования 100 подбрасываний идеальной игральной кости и определите эмпирические вероятности шести исходов. 72. Используйте графическую утилиту для моделирования 200 подбрасываний монет номиналом 5 и 10 центов, представляя исходы ОО, ОР, РО и РР как 1, 2, 3 и 4 соответственно. а) Найдите эмпирические вероятности четырех исходов. б) Чему равна вероятность каждого исхода при предположении о равновероятности? * 73. Выполните следующие задания. а) Объясните, как использовать графическую утилиту для моделирования 500 подбрасываний монеты. б) Выполните моделирование и найдите эмпирические вероятности двух исходов. в) Чему равна вероятность каждого исхода при предположении о равновероятности? *74. Из урны, содержащей дюжину шаров, пронумерованных от 1 до 12, случайным образом извлекается шар. а) Объясните, как использовать графическую утилиту для моделирования 400 повторений этого эксперимента. б) Выполните моделирование и найдите эмпирическую вероятность вытащить шар с номером 8. в) Чему равна вероятность вытащить шар с номером 8 при предположении о равновероятности?
Глава 6. Теория вероятностей 585 Применение математики Экономика и бизнес 75. Потребительская дегустация. Двенадцать популярных марок пива дегустируются потребителями вслепую. а) Из 12 марок пива случайным образом выбираются четыре разные марки. Чему равна вероятность того, что четыре марки могут быть распознаны простым угадыванием, если потребителю не разрешается повторять ответы? б) Среди четырех марок пива, случайным образом выбранных из 12, допускаются повторения. Чему равна вероятность угадать все четыре марки, если потребителю разрешается повторять ответы? 76. Потребительская дегустация. Потребителей попросили вслепую распознать шесть популярных марок сладких газированных напитков. а) Три разные марки выбираются случайным образом из шести. Потребителю не разрешается повторять ответы. Чему равна вероятность того, что все три марки могут быть определены простым угадыванием? б) Среди трех марок, выбранных случайным образом из шести, допускаются повторения. Потребителю разрешается повторять ответы. Чему равна вероятность верного определения всех трех марок простым угадыванием? 77. Подбор персонала. Предположим, что шесть женщин и пять мужчин были выдвинуты к избранию на пять должностей. Пять должностей заполняются случайным образом из 11 претендентов. Чему равна вероятность выбрать: а) трех женщин и двух мужчин; б) четырех женщин и одного мужчину; в) пять женщин; г) по крайней мере четырех женщин. 78. Выбор комитета. Из состава служащих двух отделений, А и Б, в которых работают 15 и 20 человек соответственно, нужно образовать комитет по рассмотрению конфликтных ситуаций, состоящих из четырех человек. Эти четыре человека выбираются Случайным образом из 35 служащих. Чему равна вероятность следующих событий. а) Выбраны три сотрудника из отделения А и один — из отделения Б. б) Выбраны два сотрудника из отделения А и два — из отделения Б. в) Все выбранные сотрудники работают в отделении А. г) По крайней мере три выбранных сотрудника работают в отделении А. Биологические науки 79. WWW Медицина. Будущий лаборант должен отнести образец крови к одной крови из восьми стандартных групп. а) Из набора образцов всех восьми групп крови случайным образом выбираются три разных образца. Экзаменатор не позволяет повторять ответы. Чему равна вероятность того, что группы крови всех трех образов могут быть верно определены простым угадыванием?
586 Часть II. Конечная математика б) Среди трех случайным образом выбранных образцов крови из восьми возможных допускаются повторения. Экзаменатор позволяет повторять ответы. Чему равна вероятность верного определения групп крови всех трех образцов простым угадыванием? 80, Медицинское исследование. Вследствие ограниченности средств из восьми подходящих для исследования заболеваний сердца должны быть выбраны пять исследовательских центров. Чему равна вероятность того, что будут выбраны пять определенных исследовательских центров, если выбор делается случайным образом? Социальные науки 81. Состав правления. В городской совет входят 11 представителей: 6 демократов и 5 республиканцев. а) Чему равна вероятность того, что и президент, и вице-президент демократы, если они избираются случайным образом? б) Комитет из трех представителей выбирается случайным образом. Чему равна вероятность того, что республиканцы будут составлять большинство? 6.4. Объединение, пересечение и дополнение событий; шансы ■ Объединение и пересечение событий ■ Дополнение события ■ Шансы ■ Задачи на вычисление эмпирической вероятности В разделе 6.3 говорилось, что для данного пространства элементарных событий s = {ei,e2,...,en} любая функция Р, определенная на S таким образом, что и P(ei) -I- Р(б2) Н h Р(еп) = 1, называется функцией вероятности. Любое подмножество пространства S называется событием Е, причем вероятность события Е определяется как сумма вероятностей элементарных исходов, принадлежащих множеству Е.
Глава 6. Теория вероятностей 587 Объединение и пересечение событий Поскольку события являются подмножествами пространства элементарных исходов, объединение и пересечение событий являются просто объединением и пересечением множеств, как определено в приложении АЛ, т. 2. В этом разделе изучаются объединения событий и рассматриваются простейшие варианты пересечений. Объединение и пересечение событий5 Если А п В являются двумя событиями в пространстве элементарных исходов S, то объединение событий А и В, обозначаемое A U В, и пересечение событий АиВ, обозначаемое А П В, определены следующим образом: Ли В = {е € S | ее А или е € В}, ЛпВ = {ееВ|е€Лие€В} Событие А или В = A U В. Событие А и В = А А В. Пример 6.22 (Вероятность объединения и пересечения событий). Рассмотрим пространство элементарных исходов, возникающее в результате подбрасывания одной правильной игральной кости. 5 = {1,2,3,4,5,6}. 1. Чему равна вероятность того, что выпадет число, являющееся нечетным и точно делящимся на три? 2. Чему равна вероятность того, что выпадет число, являющееся нечетным или точно делящимся на три? Решение. 1. Пусть А — событие, состоящее в том, что выпало нечетное число, В — событие, состоящее в том, что выпало число, делящееся на три, и F — событие, состоящее в том, что выпало число, являющееся нечетным и делящимся на три. Эти множества выглядят так. А = {1,3,5}, В = {3,6}, F = А А В = {3}. 5 Теоретико-множественные обозначения рассмотрены в приложении А.1, т. 2.
588 Часть II. Конечная математика Таким образом, вероятность того, что выпадет число, являющееся нечетным и точно делящимся на три, равна Р(Г) = Р(АПВ)^П(^В)= п ) о 2. Пусть А и В — события, определенные в п. 1, а Е — событие, состоящее в том, что выпало число, являющееся нечетным или делящимся на три. Эти множества выглядят так. А = {1,3,5}, В = {3,6}, Е = A U В = {1,3,5,6}. Таким образом, вероятность того, что выпадет число, являющееся нечетным или точно делящимся на 3, равна _ z_4 _, . п(АиВ) 4 2 Р(Е) = F(AUB) = \ 71 (О у и о ц Упражнение 6.22. Используйте пространство элементарных событий в примере 6.22 и ответьте на следующие вопросы. 1. Чему равна вероятность того, что выпадет число, являющееся нечетным и простым? 2. Чему равна вероятность того, что выпадет число, являющееся нечетным или простым? ■ Предположим, что Е = A U В. Можно ли найти вероятность Р(Е) в терминах множеств А и В? Да, но нужно быть внимательными. Было бы хорошо, если бы выполнялось соотношение Р(А U В) = Р(А) + Р(В). (6.5) Это оказывается верным, если события А и В являются взаимно исключающими (непе- ресекающимися), т.е. если АПВ = 0 (рис. 6.12). В этом случае Р(АиВ) является суммой вероятностей всех элементарных исходов в множестве А, сложенной с суммой всех вероятностей элементарных исходов в множестве В. Что же происходит в том случае, когда события А и В не являются взаимно исключающими, т.е. если АПВ 0 (см. рис. 6.13)? Если просто сложить вероятности элементов множества А с вероятностями элементов множества В, некоторые вероятности включаются дважды, а именно, вероятности тех
Глава 6. Теория вероятностей 589 элементов, которые входят в оба множества А и В. Чтобы компенсировать этот двойной подсчет, вычтем вероятность Р(А П В) из равенства (6.5) и получим Р(А U В) = Р(А) + Р(В) - Р(А П В). (6-6) Отметим, что равенство (6.6) имеет место для обоих случаев, АпВ 0 иАпВ = 0, поскольку равенство (6.6) сводится к равенству (6.5) для последнего случая (Р(А А В) = = 0 = 0). Если существует какое-то сомнение относительно того, что события Ап В — взаимно исключающие, лучше использовать равенство (6.6). Рис. 6.12. Взаимно исклю¬ чающие множества: АГ\В = 0 Рис. 6.13. Множества, не являющиеся взаимно исключающими: А А В / 0 Вероятность объединения двух событий Для любых событий А и В Р(А U В) = Р(А) + Р(В) - Р(А А В). Если события А и В - взаимно исключающие, то Р(АиВ) = Р(А) + Р(В). Пример 6.23 (Вероятность объединения и пересечения событий). Предположим, что бросаются две идеальные игральные кости. 1. Чему равна вероятность того, что выпадет сумма, равная 7 или 11? 2. Чему равна вероятность того, что на обеих игральных костях выпадет одно и то же число или сумма меньше пяти? Решение. 1. Если А — событие, состоящее в том, что выпала сумма, равная семи, а В — событие, состоящее в том, что выпала сумма, равная 11, то событие, состоящее в том, что выпало 7 или 11 очков, представляет собой объединение A U В (рис. 6.14), где А = {(1,6), (2,5), (3,4), (4,3), (5,2), (6,1)}, В = {(5,6), (6,5)}.
590 Часть II. Конечная математика Поскольку события АиВ являются взаимно исключающими, для вычисления Р(А U В) можно использовать равенство (6.5). Р(АиВ) =Р(А) + Р(В) = А _ “ 36 + 36 ~ В этом пространстве равновероятных элементарных событий _ 8 _ 2 ~ 36 ~ 9 п(А) = 6, п(В) = 2 и n(S) = 36. 2. Пусть А — событие, состоящее в том, что на обеих игральных костях выпало одно и то же число, а В — событие, состоящее в том, что выпала сумма меньше 5. Тогда событие, состоящее в том, что на обеих костях выпало одно и то же число или сумма меньше 5, представляет собой объединение A U В (рис. 6.15), где А = {(1,1), (2,2), (3,3), (4,4), (5,5), (6,6)}, В = {(1,1), (1,2), (1,3), (2,1), (2,2), (3,1)}. Поскольку А А В = {(1,1), (2,2)}, множества А и В не являются взаимно исключающими. Для вычисления вероятности Р(Л U В) используем равенство (6.6). Р(А U В) = Р(А) + Р(В) -Р(АпР) = ^ + ^- ^ = ^ = -^. □О ОО ОО ОО 10 ■ Упражнение 6.23. Используя пространство элементарных событий из примера 6.23, дайте ответ на следующие вопросы. 1. Чему равна вероятность того, что выпадет сумма, равная двум или трем? 2. Чему равна вероятность того, что на обеих игральных костях выпадет одинаковое число или сумма, больше восьми? ■
Глава 6. Теория вероятностей 591 Вы, несомненно, отметите в примере 6.23, что нет необходимости использовать либо формулу (6.5), либо формулу (6.6). Можно рассуждать как в примере 6.22 и просто вычислить количество элементарных точек в множестве A U В. Следующий пример иллюстрирует использование формулы (6.6) в ситуации, когда визуальное представление элементарных точек непрактично. Пример 6.24 (Вероятность объединения и пересечения). Чему равна вероятность того, что число, случайным образом выбранное из первых 500 положительных целых чисел, делится (нацело) на три и четыре? Решение. Пусть А — событие, состоящее в том, что выбранное целое число делится на три, а В — событие, состоящее в том, что выбранное целое число делится на четыре. Отметим, что события Л и В не являются взаимно исключающими, поскольку числа, кратные 12, делятся как на три, так и на четыре. Поскольку вероятность вытащить любое из целых чисел от 1 до 500 постоянна, для определения Р(А U В) можно использовать числа п(А), п(В) и п(А А В). n(A) = наибольшее целое число, которое меньше или равно = 166, о п(В) = наибольшее целое число, которое меньше или равно = 125, п(А А В) = наибольшее целое число, которое меньше или равно = 41. Теперь можно вычислить вероятность Р(А U В). Р(А U В) = Р(А) + Р(В) - Р(А А В) = __ п(А) п(В) п(А А В) = n(5) + n(S) п(5) = 166 125 41 250 „ = 1 = - 0 500 500 500 500
592 Часть II. Конечная математика Упражнение 6.24. Чему равна вероятность того, что число, случайным образом выбранное из первых 140 положительных целых чисел, делится на четыре или шесть? ■ Дополнение события Предположим, что пространство элементарных событий S = {ei,..., еп} разделено на два подмножества Е и Е' так, что Е А Е' = 0, т.е. множества Е и Е' являются взаимно исключающими и Е U Е' = S. Тогда множество Е' называется дополнением множества Е относительно пространства S. Таким образом, множество Е' содержит все элементы пространства S, которые не входят в множество Е (рис. 6.16). P(S) = Р(Е U Е') = Р(Е) + Р(Е') = 1. Рис. 6.16. Дополнение множества Дополнения Р(Е) = 1 - Р(Е'), Р(Е') = 1 - Р(Р). (6.7) Если вероятность того, что идет дождь, равна 0,67, то вероятность того, что дождь не идет, равна 1 — 0,67 = 0,33; если вероятность того, что из скважины бьет нефть, равна 0,01, то вероятность того, что нефть из скважины не бьет, равна 0,99. Если вероятность того, что в семье, имеющей двух детей, есть по крайней мере один мальчик, равна 0,75, какова вероятность того, что в семье две девочки? (Ответ: 0,25.) Задание 6.7. 1. Предположим, что Е и F являются взаимно дополняющими событиями. Являются ли события Е и F взаимно исключающими? Поясните, почему да или почему нет. 2. Предположим, что Е и F являются взаимно исключающими событиями. Являются ли события Е и F взаимно дополняющими? Поясните, почему да или почему нет. и При вычислении величины Р(Е) существуют ситуации, в которых проще сначала найти вероятность Р(Е'), а затем использовать равенства (6.7) для вычисления вероятности Р(Е). Эти ситуации иллюстрируются следующими примерами. В Пример 6.25 (Контроль качества). Партия, состоящая из 45 деталей, среди которых девять бракованных, отправлена на сборочный завод. Отдел технического контроля случайным образом выбирает для тестирования 10 деталей и отказывается от всей партии, если в выборке обнаруживается более одной бракованной детали. Чему равна вероятность того, что партия будет возвращена поставщику?
Глава 6. Теория вероятностей 593 Решение. Если Е — событие, состоящее в том, что в случайной выборке из 10 деталей более одной детали оказываются бракованными, то Е' — дополнение события Е — представляет собой событие, состоящее в том, что ни одна из выбранных 10 деталей не является бракованной. Вычислить вероятность Р(Е') оказывается легче, чем вычислить величину Р(Е) непосредственно. Определив величину Р(Е'), используем равенство Р(Е) = 1 - Р(Е') для вычисления вероятности Р(Е). Пространство элементарных событий S для этого эксперимента — это множество всех подмножеств из 10 элементов, выбранных из множества, состоящего из 45 доставленных деталей. Таким образом, поскольку существует 45 — 9 = 36 исправных деталей, то Р(В') = ^ = ^«°,°8 ГЦО) С45ДО и Р(Е) = 1 - Р(Е') « 1 - 0,08 = 0,92. В Упражнение 6.25. Партия, состоящая из 40 деталей, среди которых восемь бракованных, отправляется на сборочный завод. Отдел технического контроля случайным образом выбирает для тестирования 10 деталей и возвращает всю партию, если в выборке обнаружено более одной бракованной детали. Чему равна вероятность того, что партия будет возвращена поставщику? Е Пример 6.26 (Задача о днях рождения). Чему равна вероятность того, что дни рождения по крайней мере двух людей из группы в п человек совпадают (совпадают месяц и день, исключая 29 февраля)? (Выберите группу, состоящую из 40 человек, и проверьте результаты решения этого примера.) Решение. Составим список дней рождения всех людей в группе. Пространство элементарных исходов определяется следующим образом. S = множество всех списков, содержащих п дней рождения. Предположим, что дни рождения всех людей в группе равновероятны, так что элементарные исходы в S равновероятны. Сколько элементарных исходов содержится в множестве S? Поскольку каждый человек может иметь один из возможных 365 дней рождения (исключая 29 февраля), из правила умножения следует, что количество элементарных событий в S равно 1-е 2-е лицо лицо n(S) = 365 • 365 3-е лицо 365 п-е лицо 365 = = 365п Теперь, пусть Е — событие, состоящее в том, что дни рождения по крайней мере двух людей в группе совпадают. Тогда Е/ — событие, состоящее в том, что в группе не существует двух человек, родившихся в один день. Для определения количества элементарных событий в Е' также можно использовать принцип умножения:
594 Часть II. Конечная математика 1-е 2-е 3-е п-е ЛИЦО лицо лицо лицо п(£') = 365 • 364 • 363 (366 — п) = [365 • 364 • 363 •... • (366 - п)] (365 - п)! (365 - п)! 365! (365-п) Г Умножим числитель и знаменатель на (365 — п)! Поскольку было сделано предположение, что элементарные события пространства элементарных событий S равновероятны, то 365! р(= п(£) = (365-п)! = 365! { ’ n(S) 365" 365" (365 — п)! ’ Таким образом, р(Е)^-т)- 1 - 365„(336655!_„)Г (м) Равенство (6.8) справедливо для любого п, удовлетворяющего условию 1 п < 365. (Чему равно Р(Е), если п > 365?) Например, в группе из шести человек выполняется следующее равенство. ( - 1 365! - 385 ’ 364 ’ 363 ’ 362 ’ 361 ‘ 360 ’ - 0 04 ~ (365)6 359! " 365 • 365 • 365 • 365 • 365 • 365 • ЗШ ” ’ ’ Интересно отметить, что с увеличением размера группы вероятность Р(Е) возрастает значительно быстрее, чем можно было бы ожидать. На рис. 6.17 изображен график вероятности Р(Е) для 1 < п < 39. В табл. 6.1 приведены значения Р(Е) ддя выбранных значений п. Отметим, что для группы, состоящей только из 23 человек, вероятность того, что дни рождения у двух или более человек совпадают, больше (Вид равенства (6.8), использованного для изображения графика на рис. 6.15, обсуждается в задаче 53 из практикума 6.4.) I □=1-365 М>ГП/365АХ> X »=гз.. ■■ х=гз_^_ _ ¥=.50725723 - О Рис. 6.17. Решение задачи о днях рождения с помощью графического калькулятора
Глава 6. Теория вероятностей 595 Таблица 6.1. Задача о днях рождения Количество людей в группе п Вероятность того, что дни рождения двух и более человек совпадают 5 0,027 10 0,117 15 0,253 20 0,411 23 0,507 30 0,706 40 0,891 50 0,970 60 0,994 70 0,999 И Упражнение 6.26. Вычислите вероятность Р(Е) для п = 4, используя равенство (6.8). ■ Задание 6.8. Определите наименьшее число п, при котором в группе, состоящей из п человек, вероятность того, что два или несколько человек родились в одном и том же месяце, больше 0,5. Обсудите предположения, лежащие в основе ваших вычислений. ■ Шансы Когда вероятность события Е известна, часто говорят о шансах “в пользу” или “против” события Е, а не о вероятности события Е. Взаимосвязь между этими двумя понятиями показана ниже. От вероятности к шансам Если Р(Е) — вероятность события Е, то 1. Шансы в пользу Е = Р(Е) / 1. } 1 - Р(Е) Р(Е') k ' 2. Шансы против Е = , Р(Е) / 0. P(£J (Замечание: как правило, шансы выражаются в виде отношения целых чисел.) Р(Е) Р(Е') Пример 6.27 (Вероятность и шансы). Если игральная кость подбрасывается один раз, то вероятность выпадения четверки равна j, в то время как шансы в пользу выпадения четверки равны 1 6 1 v- = - Читается как "1 к 5м и записывается как "1 : 5". о 5 6 Шансы против выпадения четверки равны |.
596 Часть II. Конечная математика В терминах справедливой игры это означает следующее. Если вы поставили один доллар на то, что выпадет четверка, то вы отдадите один доллар в банк, если выпадет любое другое число, и вам заплатят пять долларов из банка (и в дополнение к этому вам будет возвращена ваша ставка), если выпадет четверка. (Искушенный игрок скажет, что банк ставит 5 к 1 на то, что выпадет четверка при одном подбрасывании игральной кости.) ■ Предположим, что шансы в пользу события Е равны а : Ь. Игра является справедливой, если ставка а долл, теряется, если событие Е не происходит, и вы получаете b долл, (и в дополнение к этому вам возвращается ставка в размере а долл.), если событие Е происходит. (Игроки скажут, что банк ставит b к а на то, что событие Е произойдет.) Упражнение 6.27. 1. Чему равен шанс в пользу выпадения суммы, равной восьми, при одном подбрасывании двух идеальных игральных костей? 2. Какую сумму должен заплатить банк, чтобы игра была справедливой (и возвратить вашу ставку), если вы поставили пять долларов на то, что выпадет сумма, равная восьми, при условии, что эта сумма действительно выпала? ■ Теперь пойдем в другом направлении. Чему равна вероятность события, если известен шанс в пользу события? От шанса к вероятности Если шансы в пользу события Е равны а/Ь,то вероятность события Е равна а о 4- b Пример 6.28 (Шансы и вероятность). Если при повторяющихся подбрасываниях двух идеальных игральных костей шансы в пользу выпадения пятерки перед тем, как выпадет семерка, равен 2 к 3, то вероятность выпадения пятерки перед тем, как выпадет семерка, равна Р(Я) = = 2^3 = |. fl 4- о 2 -|- 3 5 ■ Упражнение 6.28. Чему равна вероятность выпадения шестерки перед тем, как выпадет семерка, при повторяющихся подбрасываниях пары идеальных игральных костей, если шансы против выпадения шестерки перед тем, как выпадет семерка, равны 6 к 5? (Будьте внимательны! Прочитайте задачу еще раз.) ■ Задачи на вычисление эмпирической вероятности ИВ следующих обсуждениях термин эмпирическая вероятность будет обозначать вероятность события, определенную выборкой, которая используется для аппроксимации вероятности соответствующего события во всей генеральной совокупности. Как приблизительная эмпирическая вероятность события, определенная из выборки, связана
Глава 6. Теория вероятностей 597 с действительной вероятностью события, связанного со всей генеральной совокупностью? В математической статистике доказывается важная теорема, называемая законом больших чисел (законом средних). Неформально она означает, что приблизительная эмпирическая вероятность может быть сделана настолько близкой к действительной вероятности, насколько Мы пожелаем, сделав выборку достаточно большой. В Пример 6.29 (Исследование рынка). В результате опроса 1000 жителей некоторого города выяснилось, что 500 человек употребляли определенную марку диетической сладкой газированной воды, 600 употребляли определенную марку обычной сладкой газированной воды и 200 употребляли оба вида “газировки”. Случайным образом выбирается житель города. Чему равна (эмпирическая) вероятность следующих событий? 1. Житель употреблял диетическую или обычную газированную воду? Чему равны (эмпирические) шансы в пользу этого события? 2. Житель употреблял один из видов газированной воды, но не оба? Чему равны (эмпирические) шансы против этого события? Решение. Пусть D — событие, состоящее в том, что человек употреблял диетическую, a R — событие, состоящее в том, что человек употреблял обычную газированную воду. События D и R могут быть использованы для деления жителей города на четыре взаимно исключающих подмножества (набор подмножеств является взаимоисключающим, если пересечение любых двух из них является пустым множеством). D П R — множество людей, которые употребляли оба вида газированной воды; D П R — множество людей, которые употребляли диетическую, но не употребляли обычную газированную воду; D' A R — множество людей, которые употребляли обычную, но не употребляли диетическую газированную воду; D' A R' — множество людей, которые не употребляли газированную воду вообще. Эти множества изображены на диаграмме Венна (рис. 6.18). Выборка из 1000 жителей также делится на четыре взаимоисключающих множества, где n(D) = 500, n(R) = 600 и n(D A R) = 200. Используя диаграмму Венна (рис. 6.19), можно определить количество элементарных точек во множествах DAT?', D'r\R и D'A R' (см. пример 6.2, раздел 6.1). Рис. 6.18. Генеральная совокупность Рис. 6.19. Выборочная совокупность
598 Часть II. Конечная математика Полученные значения лучше всего изучать, представив в табличном виде. Употребляют обычную газированную воду (Л), чел. Не употребляют обычную газированную воду (Я'), чел. Общее количество, чел. Употребляют диетическую газированную воду (£>), чел. 200 300 500 Не употребляют диетическую газированную воду (£>'), чел. 400 100 500 Общее количество, чел. 600 400 1000 Предполагая, что все элементарные события равновероятны, получим таблицу вероятностей, деля каждый элемент в приведенной выше таблице на 1000. Это эмпирические вероятности выборочной совокупности, которые можно использовать для аппроксимации соответствующих вероятностей всей генеральной совокупности. Употребляют обычную газированную воду (Я) Не употребляют обычную газированную воду (Я') Общая вероятность Употребляют диетическую газированную воду, (D) 0,2 0,3 0,5 Не употребляют диетическую газированную воду, (Z)') 0,4 0,1 0,5 Общая вероятность 0,6 0,4 1,0 Теперь мы готовы вычислить требуемые вероятности. 1. Событие, состоящее в том, что человек использует диетическую или обычную сладкую газированную воду, представляет собой множество Е = DUR. Вычислим вероятность Р(Е) двумя способами. Метод 1. Прямое вычисление. Р(Е) = P(D U R) = Рф) + Р(Я) - P(D П R) = 0,5 + 0,6 - 0,2 = 0,9. Метод 2. Используем дополнение множества Е. 2Р(Е) = 1 - Р(Е') = 1 - Р(Р' П Р') = Е' = (D U R)' = D' П R' = 1 — 0,1 = 0,9. (рис. 6.18) Р (Е} 0 9 9 В обоих случаях шанс в пользу события Е равен ? = - или 9:1. 2. Событие, состоящее в том, что человек употреблял один вид газированной воды, но не оба — это событие, состоящее в том, что человек употреблял диетическую, но
Глава 6. Теория вероятностей 599 не употреблял обычную, или в том, что употреблял обычную, но не диетическую газированную воду. В терминах множеств это событие имеет вид Е = (D П R') U U (£>' A R). Поскольку события D A R' и D' A R являются взаимно исключающими (см. диаграммы Венна на рис. 6.18), Р(В) = P[(D А 7?') U (D' А 7?)] = P(D А 7?') + Р(7У А 7?) = 0,3 + 0,4 = 0,7. Р (Е'} 0 3 3 Шансы против события Е равны ? = -, или 3:7. ■ D Упражнение 6.29. Житель города в примере 6.29 выбирается случайным образом. Чему равна (эмпирическая) вероятность следующих событий? 1. Житель не употреблял ни один из напитков. Чему равны (эмпирические) шансы в пользу этого события? 2. Житель не употреблял диетическую или не употреблял обычную газированную воду. Чему равны (эмпирические) шансы против этого события? ■ Ответы к упражнениям м- 6.25. 6.26. 6.27. 2) 31 долл. 6.22. 1) |. 6.23. 1) 1. 47 ^•24. — «0,336. 140 0,92. 0,016. 1) 5 : 31. 6.28. ^-«0,455. 6.29. 1) P(Df А 7?') — 0,1; шансы в пользу события D' A R' = |, или 1 : 9. 2) P(D U 7?') — 0,6; шансы против события D U R' = |, или 2 : 3. Практикум 6.4 А Задачи 1-6 отсылают к диаграмме Венна, изображенной ниже для событий АиВ в пространстве S равновероятных элементарных событий. Вычислите все указанные вероятности. 1. Р(АаВ). 2. Р(АиВ). 3. Р(Л'иВ).
600 Часть II. Конечная математика 4. Р(А А В'). 5. Р((А U В)'). 6. Р((А А В)'). Из стандартной колоды в 52 карты извлекается одна карта. Пусть Е — событие, состоящее в том, что извлеченная карта имеет червонную масть, a F — событие, состоящее в том, что извлеченная карта — нумерованная (от 2 до 10). Вычислите указанные в задачах 7-12 вероятности. 7. Р(В). 8. Р(Р). 9. P(F A F). 10. P(F U F). 11. P(F'). 12. Р(В'). Из контейнера, содержащего 25 одинаковых шаров, пронумерованных от 1 до 25, случайным образом извлекается один шар. В задачах 13-18, используя равенство (6.5) или (6.6), вычислите вероятность следующих событий. 13. Извлечено число, которое меньше 6 или больше 19. 14. Извлечено число, которое делится на 4 или делится на 7. 15. Извлечено число, которое делится на 3 или делится на 4. 16. Извлечено число, которое является нечетным или больше 15. 17. Извлечено число, которое является четным или делится на 5. 18. Извлечено число, которое меньше 12 или больше 13. 19. Если вероятность того, что кандидат выиграет на выборах, равна 0,51, чему равна вероятность того, что кандидат проиграет? 20. Вероятность того, что автомобиль выйдет из строя, пройдя меньше 50 000 миль, равна 0,03. Чему равна вероятность того, что автомобиль не выйдет из строя, пройдя 50 000 миль? В задачах 21-24, используя пространство с равновероятными элементарными событиями из примера 6.23 и равенство (6.5) или (6.6), вычислите вероятности следующих событий. 21. Выпала сумма, равная 5 или 6. 22. Выпала сумма, равная 9 или 10. 23. На первой или второй кости выпала единица. 24. На первой кости выпала единица или на второй кости выпало число, меньше трех. 25. Для данных вероятностей события Е найдите шансы в пользу и против события Е: а) |. б) 1 в) 0,4- г) 0,55. О 4 26. Для данных вероятностей события Е найдите шансы в пользу и против события Е. 3 1 а) -. б) -. в) 0,6. г) 0,35. 5 7 27. Вычислите вероятность события Е при следующих шансах в его пользу. \ 3 11 \ 4 \ 49 ” 8 б) У ” Г' 0 й'
Глава 6. Теория вероятностей 601 28. Вычислите вероятность события Е при следующих шансах в его пользу. . 5 4 3 23 •’ 9- б) 3- •’ т г) Тт В задачах 29 и 30 оцените справедливость каждого утверждения. Если утверждение всегда справедливо, объясните почему. Если нет, приведите контрпример. * 29. Справедливы ли следующие утверждения? а) Эмпирическая вероятность события меньше или равна его теоретической вероятности. б) Если события Е и F взаимно исключающие, то Р(Е) + P(F) = Р(Е U F) + F(F П F). * 30. Справедливы ли следующие утверждения. а) Если Р(Е) = |, шансы в пользу события Е равны шансам против события Е. б) Если шансы в пользу события Е равны а : Ь, то шансы против события Е' равны b : а. Б В задачах 31-34 вычислите шансы перечисленных событий. 31. При одном подбрасывании монеты выпал орел. 32. При одном подбрасывании игральной кости выпало число, делящееся на три. 33. При подбрасывании одной монеты три раза орел выпал по крайней мере один раз. 34. При двух подбрасываниях монеты орел выпал один раз. В задачах 35-38 вычислите шансы против событий. 35. При одном подбрасывании игральной кости выпало число, которое больше четырех. 36. При двух подбрасываниях монеты орел выпал дважды. 37. При одном подбрасывании игральной кости выпало число три или четное число. 38. При одном подбрасывании игральной кости выпало нечетное число или число, кратное трем. 39. Ответьте на следующие вопросы. а) Чему равны шансы в пользу выпадения суммы, равной пяти, при одном подбрасывании пары игральных костей? б) Если вы поставили один доллар на то, что выпадет сумма, равная пяти, то сколько должен заплатить банк (помимо возврата ставки) при условии, что это событие произошло, чтобы игра была справедливой? 40. Ответьте на следующие вопросы. а) Чему равны шансы в пользу выпадения суммы, равной 10, при одном подбрасывании пары игральных костей? б) Если вы поставили один доллар на то, что выпадет сумма, равная 10, то сколько должен заплатить банк (помимо возврата ставки) при условии, что это событие произошло, чтобы игра была справедливой?
602 Часть II. Конечная математика Пара игральных костей подбрасывается 1000 раз, в результате чего получены следующие частоты исходов. Сумма 2 3 4 5 6 7 8 9 10 11 12 Частота 10 30 50 70 110 150 170 140 120 80 70 Используйте эти частоты для вычисления приблизительных эмпирических вероятностей и шансов в пользу события в задачах 41 и 42. 41. Вычислите эмпирическую вероятность следующих событий. а) Выпала сумма, меньше четырех или больше девяти. б) Сумма является четной или нацело делящейся на пять. 42. Вычислите эмпирическую вероятность следующих событий. а) Сумма является простым числом или кратным четырем. б) Сумма является нечетным числом или кратным трем. В задачах 43-46 из стандартной колоды, состоящей из 52 карт, извлекается одна карта. Вычислите вероятность и шансы каждого из событий. 43. Извлечена фигура или карта трефовой масти. 44. Извлечен король или карта червовой масти. 45. Извлечена карта черной масти или туз. 46. Извлечена карта червонной масти или числом, которое меньше семи (считайте туз единицей). 47. Чему равна вероятность извлечь по крайней мере одну карту бубновой масти в пятикарточном раскладе из стандартной колоды, состоящей из 52 карт? 48. Чему равна вероятность извлечь по крайней мере одну карту черной масти в семикарточном раскладе из стандартной колоды, состоящей из 52 карт? 49. Чему равна вероятность того, что число, случайным образом выбранное из первых 1000 положительных целых чисел, кратно шести или восьми? 50. Чему равна вероятность того, что число, случайным образом выбранное из первых 600 положительных целых чисел, кратно шести или девяти? * 51. Объясните, как должны быть связаны события А, В и С из пространства эле¬ ментарных событий S, чтобы выполнялось следующее равенство. Р(АUBUC}= Р(А) + Р(В) + Р(С) - Р(А А В). * 52. Объясните, как три события А, В и С из пространства элементарных событий S должны быть связаны друг с другом, чтобы выполнялось следующее равенство. Р(А U В U С) = Р(А) + Р(В) + Р(С). *53. Покажите, что решение задачи о днях рождения в примере 6.26 может быть записано в виде Р (В) = 1 — Р^п • Объясните, почему для калькулятора, имеющего функцию Рп>г, эта форма может быть удобнее для непосредственного вычисления, чем другая форма, использованная в решении примера 6.26. Вычислите эту величину непосредственно с помощью обеих форм на калькуляторе для п = 25.
Глава 6. Теория вероятностей 603 * 54. Многие (но не все) калькуляторы сообщают о переполнении памяти при вычислении величины Рзб5,п Для п > 39 и 365п. Объясните, как вычислить вероятность Р(Е) Для любого п > 39 на таком калькуляторе. В 55. Чему равна вероятность того, что в группе из п человек (п < 12) по крайней мере два человека родились в одном и том же месяце? (Предположите, что события рождения в каждом месяце равновероятны.) 56. В группе из п человек (п 100) каждого человека просят выбрать число между 1 и 100, записать его на листе бумаги и положить этот лист в шапку. Чему равна вероятность того, что по крайней мере два человека вытащат те же листы бумаги, на которых они писали? 57. Пусть шансы в пользу события Е равны а : Ъ. Покажите, что Р(Е) = (Подсказка', решите уравнение Р(Е)/Р(Е') = а/b относительно Р(Е).) 58. Пусть Р(Е) с/d. Покажите, что шансы в пользу события Е равны с : (d — с). 59. На рис. 6.20, а продемонстрировано применение графической утилиты для моделирования 50 подбрасываний пары игральных костей. Результаты изображены на рис. 6.20, б. а) Используя рис. 6.20, найдите эмпирическую вероятность того, что выпадет сумма, равная 7 или 8. б) Чему равна теоретическая вероятность того, что выпадет сумма, равная 7 или 8? в) Примените графическую утилиту для моделирования 200 подбрасываний пары игральных костей и найдите эмпирическую вероятность того, что выпадет сумма, равная 7 или 8. randlnt<l>6>50)+ rand!nt<l?6?50>-> Li <4 8 5 9 6 7 9... а) б) Рис. 6.20. Иллюстрация к задаче 59 60. На рис. 6.21, а продемонстрировано применение графической утилиты для решения некоей задачи. Результаты моделирования изображены на рис. 6.21, б. а) Объясните, почему указанная команда не моделирует 50 подбрасываний пары игральных костей и не выводит их сумму. б) Опишите эксперимент, смоделированный этой командой. в) Смоделируйте ситуацию 200-кратного повторения эксперимента, описанного в п. б9 и найдите эмпирическую вероятность того, что будет получено число 7 или 8.
604 Часть II. Конечная математика randlnt<2,12,50) ->Li <5 9 9 2 12 5 1... 12 Рис. 6.21. Иллюстрация к задаче 60 г) Чему равна теоретическая вероятность того, что при моделировании будет получено значение 7 или 8? Применение математики Экономика и бизнес 61. Исследование рынка. Опрос 1000 студентов большого университета, в котором проводилась компания по исследованию потребительского рынка, показал, что у 750 студентов есть стереосистемы, у 450 — автомобили, а у 350 есть и автомобили, и стереосистемы. Случайным образом выбирается студент университета. Чему равна (эмпирическая) вероятность следующих событий? а) У студента есть и автомобиль, и стереосистема. б) У студента нет ни автомобиля, ни стереосистемы. 62. Исследование рынка. Если студент университета в задаче 61 выбирается случайным образом, чему равна (эмпирическая) вероятность следующих событий? а) У студента нет автомобиля. б) У студента есть автомобиль, но нет стереосистемы. 63. WWW Страхование. Изучая записи о транспортных происшествиях в границах некого города, страховая компания определила следующие (эмпирические) вероятности. Меньше 10000, Mi Пробег, миль/год 10 000-15 000 включительно, Мг Больше 15000, М3 Общая вероятность Вероятность попадания в аварию, А Вероятность избежать 0,05 0,1 0,15 0,3 аварии, А' 0,15 0,2 0,35 0,7 Общая вероятность 0,2 0,3 0,5 1,0
Глава 6. Теория вероятностей 605 Случайным образом выбирается водитель из этого города. Чему равна вероятность следующих событий? а) За год он проезжает меньше 10000 миль или попадает в аварийную ситуацию. б) За год он проезжает 10000 или более миль, не попадая в аварийную ситуацию. 64. Страхование. Используя (эмпирические) вероятности, найденные в задаче 63, вычислите вероятность следующих событий. а) Водитель, выбранный случайным образом, проезжает более 15 000 миль за год или попадает в аварийную ситуацию. б) Водитель, выбранный случайным образом, проезжает 15 000 или меньше миль за год и попадает в аварийную ситуацию. * 65. Производство. Производители ноутбуков дают 90-дневную ограниченную гарантию на свою продукцию, которая распространяется только на клавиатуру и дисковод. Их записи показывают, что в течение гарантийного периода 6% их изделий возвращаются из-за поломки клавиатуры, 5% возвращаются из-за поломки дисководов и 1% возвращается из-за поломки как клавиатуры, так и дисководов. Чему равна (эмпирическая) вероятность того, что компьютер будет работать без поломок в течение всего гарантийного периода? 66. Тестирование продукции. Для того чтобы проверить новый автомобиль на надежность, нужно выбрать четырех сотрудников компании, непрерывно эксплуатирующих его в течение одного года. В проверке вызвались участвовать 12 менеджеров и 8 служащих. Их выбор производится случайным образом. Чему равна вероятность того, что в тестировании примет участие по крайней мере один служащий? 67. Контроль качества. В магазин поступила партия 60 недорогих цифровых часов, включающая 9 поломанных. Отдел приемки случайным образом выбирает для тестирования 10 часов и возвращает всю партию, если в выборке обнаруживаются более одной пары неработающих часов. Чему равна вероятность того, что партия будет возвращена? 68. Контроль качества. Автоматическая линия выпускает 40 одноплатных компьютеров в смену, среди которых 7 неработающих. Отделение контроля качества случайным образом выбирает для тестирования 10 плат и забраковывает всю партию, если в выборке обнаруживается более одного неработающего компьютера. Чему равна вероятность того, что партия одноплатных компьютеров будет забракована? Биологические науки 69. WWW Медицина. Для того чтобы проверить, есть ли у нового лекарства побочные эффекты, фармацевтическая компания провела исследование, в котором приняли участие 1000 человек: 60 человек сообщили, что единственным побочным эффектом была потеря аппетита, 90 сообщили, что единственным побочным эффектом была потеря сна, и 800 человек сообщили, что побочных эффектов не было совсем. Лекарство допущено к продаже. Чему равна (эмпирическая) веро-
606 Часть II. Конечная математика ятность того, что человек, использующий лекарство, будет страдать как потерей аппетита, так и потерей сна? 70. Медицина. Для проведения медицинских исследований выделено тридцать подопытных животных: три самца и семь самок макак, шесть самцов и четыре самки шимпанзе, два кобеля и восемь сук. Случайным образом выбирается одно животное. Чему равна вероятность следующих событий? а) Выбирается шимпанзе или собака. б) Выбирается шимпанзе или самец любого животного. в) Выбирается животное, которое не является самкой обезьяны. Социальные науки В задачах 71 и 72 упоминаются данные из следующей таблицы, полученной в результате анкетирования 1000 жителей штата. В анкете содержались вопросы об партийной принадлежности и предпочтениях на будущих губернаторских выборах. (В таблице приняты такие обозначения: D — демократ, R — республиканец и U — беспартийный.) D R и Всего Кандидат А 200 100 85 385 Кандидат В 250 230 50 530 Не определились 50 20 15 85 Всего 500 350 150 1000 71. Политика. Случайным образом выбирается житель штата. Чему равна (эмпирическая) вероятность следующих событий? а) Выбранный житель не принадлежит ни одной партии или не имеет предпочтений. Каковы шансы такого события? б) Выбранный житель принадлежит политической партии и предпочитает кандидата А. Каковы шансы против этого события? 72. Политика. Случайным образом выбирается житель штата. Чему равна (эмпирическая) вероятность следующих событий? а) Выбранный житель демократ или отдает предпочтение кандидату В. Чему равны шансы в пользу такого события? б) Выбранный житель не является демократом и не имеет предпочтений. Каковы шансы против этого события? 73. Социология. Группа из пяти африканцев, пяти азиатов, пяти латиноамериканцев и пяти европейцев принимали участие в исследованиях по изучению расовых отличий в малых группах. Случайным образом выбирались три человека. Чему равна вероятность того, что по крайней мере один из них африканец?
Глава 6. Теория вероятностей 607 6.5. Условная вероятность, пересечение и независимость событий ■ Условная вероятность ■ Пересечение событий: правило умножения ■ Вероятностные деревья ■ Независимые события ■ Резюме В предыдущем разделе показано, что вероятность объединения двух событий можно выразить в терминах вероятностей отдельных событий. Теперь зададимся следующим вопросом: можно ли вычислить вероятность пересечения двух событий, зная вероятности отдельных событий? Как мы увидим, это вполне возможно. Однако сначала необходимо исследовать понятие, называемое условной вероятностью. Условная вероятность Вероятность одного события может зависеть от появления другого события. Например, если случайным образом выбрать человека из множества всех взрослых людей Соединенных Штатов (21 год и старше), то вероятность того, что этот человек болен раком легких, будет небольшой. Однако, если мы скажем, что этот человек заядлый курильщик, вероятность заболевания, конечно, увеличится. В общем случае вероятность появления события А при данном событии В называется условной (conditional probability) и обозначается как Р(А | В). В приведенном выше примере события Аи В формулируются так. А — у взрослого человека рак легких, В — человек старше 21 года и заядлый курильщик. Величина Р(А | В) представляет собой вероятность того, что у взрослого человека обнаружен рак легких при условии, что он заядлый курильщик. Сформулируем точное определение условной вероятности. Сначала рассмотрим относительно простую задачу, решим ее, руководствуясь интуитивными соображениями, а затем обобщим наши рассуждения. Чему равна вероятность выпадения простого числа (2, 3 или 5) при одном подбрасывании идеальной игральной кости? Пусть S = {1,2,3,4,5,6}. Тогда событие, состоящее в выпадении простого числа, представляет собой множество А = {2,3,5} (рис. 6.22). Рис. 6.22. Выпадение простого числа
608 Часть II. Конечная математика Таким образом, если предположить, что все элементарные события в пространстве элементарных событий равновероятны, получим следующий результат. Р(Л) = п (А) _ 3 ТГ(5) = 6 1 2 Зададим вопрос: “Чему равна вероятность того, что при одном подбрасывании правильной тральной кости выпало простое число, если известно, что выпало нечетное число?”. Дополнительная информация о другом событии (В — выпало нечетное число) проливает свет на задачу. Теперь интерес представляет только часть события А (выпадение простого числа), которая содержится в событии В (выпадение нечетного числа). Поскольку известно, что событие В произошло, оно становится новым пространством элементарных исходов. Диаграммы Венна, изображенные на рис. 6.23, иллюстрируют различные взаимосвязи между этими событиями. Итак, вероятность события А при данном событии В представляет собой количество элементов множества А, принадлежащих множеству В, деленное на общее количество элементов в множестве В. Р(А\В) = п(АпВ) _2 п(В) ~ 3‘ Рис. 6.23. Множество В является новым пространством элементарных исходов Деля числитель и знаменатель дроби п(АпВ)/п(В) на количество элементов n(S) в исходном пространстве элементарных исходов, можем выразить Р(А | В) в терминах Р(АП ПВ)иР(В)6. п (А П В) PMI ДХ-П(АПВ) _ n(S) _Р(АПВ) ( 1 п(В) ~ п(В) ~ Р(В) ‘ п(5) 6Отметим, что величина Р(А | В) — это вероятность, основанная на новом пространстве элементарных исходов В, в то время как величины Р(А А В) и Р(В) — вероятности, основанные на исходном пространстве элементарных исходов S.
Глава 6. Теория вероятностей 609 Вычислив правую часть доя определения вероятности Р(А | В) в приведенном выше примере, получим тот же результат. 2 рМ| т_Р(ЛПВ) _6_2 Р(Л|В)- Р(В) -з-з- 6 Используем приведенную выше формулу для обоснования следующего определения условной вероятности, которое применимо к любому пространству элементарных исходов, включая неравновероятные (см. пример 6.30). Условная вероятность Для событий А и В в произвольном пространстве элементарных исходов S определим условную вероятность события А при условии события В следующим образом р (Л | В) = Р(рЛ?.В), Р(В) / 0. (6.9) Пример 6.30 (Условная вероятность). Представьте себе круг с секторами, над которым вращается стрелка (рис. 6.24). Вероятность, того, что стрелка остановится над сектором с определенным числом (от 1 до 6), равна отношению площади соответствующего кругового сектора к площади всего круга. ei i z □ ч э о Р(е{) 0,1 0,2 0,1 0,1 0,3 0,2 Рис. 6.24. Рулетка 1. Чему равна вероятность того, что стрелка остановится на простом числе? 2. Чему равна вероятность того, что стрелка остановится на простом числе, при условии, что она остановилась на нечетном числе? Решение. Пусть события Е и F определены следующим образом. Е — стрелка остановилась на простом числе = {2,3,5}, F — стрелка остановилась на нечетном числе = {1,3,5}. 1. Р(Е) = Р(2) + Р(3) + Р(5) = 0,2 + 0,1 + 0,3 = 0,6.
610 Часть II. Конечная математика Р(Е | F) = 2. Учитывая, что Е A F = {3,5}, получаем следующий результат P(EQF) _ Р(3) + Р(5) Р(Р) - Р(1) + Р(3) + Р(5) - 0,14-0,3 ^0,4 0,1+ 0,1+ 0,3 0,5 ■ Упражнение 6.30. Проанализируйте еще раз условия примера 6.30 и ответьте на следующие вопросы. 1. Чему равна вероятность того, что стрелка остановится на числе, которое больше 4? 2. Чему равна вероятность того, что стрелка остановится на числе, которое больше 4, при условии, что выпавшее число является четным? ■ Задание 6.9. Сравните пример 6.30 с задачей об игральной кости, описанной в начале главы. Чем они похожи и чем отличаются? ■ D Пример 6.31 (Безопасность дорожного движения). Предположим, что статистические данные, собранные в большом городе, позволяют сделать следующие выводы о вероятности попасть в дорожно-транспортное происшествие (ДТП) в воскресенье. Вероятность Вероятность участия избежать в ДТП, А ДТП, А' Всего Дождь, Р 0,025 Сухая погода, Р' 0,015 0,335 0,625 0,360 0,640 Всего 0,040 0,960 1,000 S = {RA, RA', R’A, R’A'} 1. Вычислите вероятность ДТП. 2. Вычислите вероятность дождя. 3. Вычислите вероятность ДТП и дождя. 4. Вычислите вероятность ДТП в дождливую погоду. Решение, 1. А = {RA,R'A}, ДТП Р(А) = P(RA) + P(R'A) = 0,025 + 0,015 = 0,040. 2. R = {RA, RA'}, Дождь P(P) = P(RA) + P(PA') = 0,025 + 0,335 = 0,360. 3. А П R = {PA}, ДТП и дождь P(A П P) = P(PA) = 0,025.
Глава 6. Теория вероятностей 611 4. Р(А | R) = Р (АПЛ) _ 0,025 Р (Я) 0,360 = 0,069. ДТП в дождливую погоду Сравните этот результат с результатом решения задачи 1. Отметим, что Р(А | R) / Р(А), т.е. если нам известно, что в воскресенье будет дождь, то вероятность ДТП повышается.и П Упражнение 6.31. Используя таблицу, приведенную выше, вычислите следующие величины. 1. Вероятность того, что дождя не будет. 2. Вероятность того, что случится ДТП, но дождя не будет. 3. Вероятность того, что ДТП случится, если известно, что в воскресенье дождя не будет. (Используйте формулу (6.9) и результаты решения пп. 1 и 2.) ■ Пересечение событий: правило умножения Вернемся к исходной задаче этого раздела и представим вероятность пересечения двух событий в терминах вероятностей отдельных событий. Если Р(А) / 0 и Р(В) 0, то, используя формулу (6.9), можно записать следующие выражения. р,л | m _ РМПВ) „ . . ._Р(ВПЛ) W»- Р(В) » Р(В|Л>- Р(А) ' Решая первое равенство относительно величины Р(А Г1 В), а второе — относительно Р(В П А), получим Р(А П В) — Р(В)Р(А | В) и Р(В А А) = Р(А)Р(В | А). Поскольку А А В = В А А для любых множеств А и В, Р(А А В) = Р(В)Р(А | В) = Р(А)Р(В | А). Правило умножения Для событий А и В с ненулевыми вероятностями в пространстве элементарных исходов S выполняется следующая формула. Р(А А В) = Р(А)Р(В | А) = Р(В)Р(А | В). (6.10) В Пример 6.32 (Исследование потребительского рынка). Предположим, что 60% покупателей магазина женщины и 75% женщин имеют скидки. Чему равна вероятность того, что случайно выбранный покупатель является женщиной и имеет скидки? Решение. Введем следующие обозначения. S — все покупатели магазина, F — женщины-покупатели, С — покупатели со скидками.
612 Часть II. Конечная математика Если 60% покупателей женщины, то вероятность того, что случайным образом выбранный покупатель является женщиной, равна P(F) = 0,60. Поскольку 75% женщин-покупателей имеют скидки, вероятность того, что покупатель имеет скидки, при условии, что покупатель — женщина, равна Р(С | F) = 0,75. Используя равенство (6.10), получаем, что вероятность того, что покупатель — женщина и имеет скидки, равна P(FПС) = P(F)P(C | F) = 0,60 • 0,75 = 0,45. а И Упражнение 6.32. Если 80% покупателей-мужчин магазина в примере 6.32 имеют скидки, чему равна вероятность того, что случайным образом выбранный покупатель — мужчина и имеет скидки? ■ Вероятностные деревья В разделе 6.1 были использованы древовидные диаграммы, помогающие подсчитать количество составных исходов в последовательности экспериментов. Воспользуемся этим инструментом для решения следующей задачи. Пример 6.33 (Вероятностное дерево). Из урны, содержащей три синих и два белых шара, последовательно без возвращения извлекаются два шара (рис. 6.25). Чему равна вероятность вынуть белый шар при втором испытании? Решение. Начнем с древовидной диаграммы (рис. 6.26), изображающей составные исходы двух экспериментов (первое и второе испытание). Рис. 6.25. Урна с шарами Первое Второе извлечение извлечение Рис. 6.26. Древовидная диаграмма Припишем каждой ветви дерева соответствующую вероятность (рис. 6.27). Например, ветви Swi припишем вероятность, равную |, поскольку она представляет собой вероятность вытащить белый шар при первом испытании (в урне находятся два белых и три синих шара). Какую вероятность следует приписать ветви W1W2? Очевидно, условную вероятность P(w2 | wi), т.е. вероятность вытащить белый шар при втором испытании, при условии, что при первом испытании из урны был извлечен белый шар (без возвращения). Поскольку урна теперь содержит один белый шар и три синих, вероятность этого
Глава 6. Теория вероятностей 613 события равна Применяя тот же способ, припишем вероятности другим ветвям дерева и получим рис. 6.27. Чему равна вероятность составного исхода wi AW2, т.е. вероятность извлечь белый шар и при первом и при втором испытаниях?7 Используя правило умножения (6.6), получим следующий ответ. P(wi A W2) = P(W1)P(W2 I Wi) = G) (i) - Составное событие w\ A W2 соответствует единственному пути Sw±W2 на вероятностном дереве. Легко видеть, что вероятность достижения вершины W2 вдоль этого пути равна произведению вероятностей, приписанных ветвям, лежащим на этом пути. Итак, чтобы вычислить вероятность составного события, следует перемножить вероятности, приписанные ветвям, лежащим на пути к данной вершине. Эти вероятности часто записываются на концах путей, которым они соответствуют (рис. 6.28). Первое Второе Рис. 6.27. Вероятностное дерево Первое Второе извлечение извлечение Рис. 6.28. Пути на вероятностном дереве 1 10 2 . != 1 5 4 10 3_ 2 . 1- J. 10 5 4 10 J_ 3 . , 2_ 3 10 5 4 10 _3_ 3 2= A 10 5 4 10 Сумма= 1 Теперь легко завершить решение поставленной задачи. При извлечении белого шара во время второго испытания происходит либо событие Wi A W2, либо событие A W2. Поскольку эти составные исходы являются взаимно исключающими, получаем следующий результат. P(w2) = P(wi П w2) + Р(Ь1 n w2) = 1U 1U 1U о Эта величина представляет собой сумму вероятностей, записанных на концах двух путей, заканчивающихся в вершине W2. ■ Упражнение 6.33. Из урны, содержащей четыре красных и два белых шара, последовательно без возвращения вынуты два шара. Чему равна вероятность вытащить красный шар при втором испытании? ■ Последовательность двух экспериментов в примере 6.33 представляет собой пример случайного процесса. В общем, стохастический процесс представляет собой последова- 7Пространством элементарных исходов для составных событий является S = {wiW2, wifo, 61W2, bib2}. Если ПОЛОЖИТЬ W1 = {W1W2, W162} и W2 = {W1W2,61W2}, TO Wi A W2 = {W1W2}.
614 Часть II. Конечная математика тельность экспериментов, исходы которых непредсказуемы. Интерес представляет попытка предсказать поведение процесса в целом. Анализ примера 6.33 обобщается на стохастические процессы, включающие любую конечную последовательность экспериментов. Построение вероятностных деревьев Этап 1. Изобразим диаграмму в виде дерева, соответствующего всем сложным исходам последовательности экспериментов. Этап 2. Каждой ветви дерева припишем определенную вероятность. (Эта величина представляет собой вероятность появления события на правом конце ветви при условии, что произошли все события, указанные на пути к этой вершине. Вероятность составного исхода, соответствующего определенному пути на дереве, равна произведению вероятностей всех ветвей на этом пути.)8 Этап 3. Используем результаты этапов 1 и 2 для ответа на различные вопросы, связанные с последовательностью экспериментов в целом. Задание 6.10. Используя таблицу из примера 6.31 и формулу (6.9), завершите построение следующего вероятностного дерева. А 0,025 RA А' ? RA' А 9 RA А' ? RA' Рассмотрите разность между вероятностями P(R П А) и Р(А | R). D Пример 6.34 (Тестирование продукции). Крупная компания А по производству компьютеров заключает контракты на производство плат с двумя другими компаниями: 40% поставок должна обеспечивать компания В и 60% — компания С. Компания В в свою очередь заключает контракты на 70% заказов, которые она получила от компании А, с компанией D, а оставшиеся 30% — с компанией Е. Обе эти компании являются дочерними для компании В. Платы, произведенные компаниями D, Е и С, поставляются компании А, а затем используются при сборке компьютеров. Обнаружено, что на 1,5; 1 и 0,5% плат, созданных компаниями D, Е и С соответственно, в течение 90-дневного гарантийного обслуживания компьютера поступают жалобы. Чему равна вероятность того, что у конкретной платы в течение 90-дневного срока гарантийного обслуживания возникнут трудности с работоспособностью? 8Если пространство элементарных исходов S строится так, что каждый элементарный исход в S соответствует одному пути на дереве, и, если вероятность, приписанная каждому элементарному исходу в 5, является произведением вероятностей ветвей дерева на соответствующем пути, можно показать, что это распределение является не только приемлемым (все вероятности элементарных событий в S являются неотрицательными и их сумма равна 1), но и единственным.
Глава 6. Теория вероятностей 615 Решение. Построим вероятностное дерево (рис. 6.29). К бракованной плате ведут три пути. Умножим вероятности ветвей на каждом из путей и сложим эти три произведения. ^(Бракованная плата) = 0,4 • 0,7 • 0,015 + 0,4 • 0,3 • 0,01 + 0,6 • 0,005 = 0,0084. а Рис. 6.29. Вероятностное дерево при оценке качества продукции Рабочая плата Бракованная плата Рабочая плата Бракованная плата Рабочая плата Бракованная плата Щ Упражнение 6.34. Вернемся к примеру 6.34. Чему равна вероятность того, что плата в собранном компьютере произведена компанией Е или С? ■ Независимые события Вернемся к примеру 6.33, в котором рассматривалось последовательное извлечение без возвращения двух шаров из урны, содержащей три синих и два белых шара. Какая разница между извлечением “без возвращения” и “с возвращением”? На рис. 6.30 изображены вероятностные деревья, соответствующие каждому случаю. Убедимся в правильности полученных распределений вероятностей. Первое Второе Составные Второе Первое испытание испытание исходы испытание испытание а) без возвращения б) с возвращением Рис. 6.30. Древовидная диаграмма А — при втором испытании извлечен белый шар: {W1W2, biW2}, В — при первом испытании извлечен белый шар: {W1W2, wi^}.
616 Часть II. Конечная математика Теперь вычислим вероятности Р(А | В) и Р(А) для каждого варианта, изображенного на рис. 6.30. Вариант 1. Без возвращения. Р(А\В\ = Р(АпВ>) = P{wiw2} = 0,Ю = п k 1 ’ Р(В) Piw^w^} 0,10 + 0,30 ’ Р(А) = P{wiW2, byu^} = 0,10 4- 0,30 = 0,40. Отметим, что Р(А | В) Р(А). Следовательно, вероятность события А зависит от появления события В. Вариант 2. С возвращением. Р ( Л I ОЛ = Р(АГ\В) = P{W1W2} = 0,16 = „ 4„ V 1 ’ Р(В) P{wiw2,W1b2} 0,16 + 0,24 ’ ’ Р(А) = P{wiW2,6iW2} = 0,16 4- 0,24 = 0,40. Отметим, что Р(А | В) = Р(А). Следовательно, вероятность события А не зависит от появления события В. Интуитивно понятно, что если Р(А | В) = Р(А), то событие А “не зависит” от события В. Если события А и В удовлетворяют условию Р(А|В) = Р(А), то, заменяя левую часть ее эквивалентом из равенства (6.9), получим следующую формулу. Р(АПД) =РМ) Р (В) г 'А> ■ После умножения обеих частей на величину Р(В) последнее равенство превращается в следующее. Р(АПВ) = Р(А)Р(В). Итак, можно ввести такое определение независимых событий. Независимые события Пусть А и В — произвольные события в пространстве элементарных исходов S. События А и В являются независимыми тогда и только тогда, когда Р(АПВ) = Р(А)Р(В). (6.11) В противном случае говорят, что события Ан В являются зависимыми. Теорема 6.2 (О независимых событиях). Если А и В — независимые события с ненулевыми вероятностями в пространстве элементарных исходов S, то Р(А|В) = Р(А) и Р(В|А) = Р(В). (6.12) И наоборот, если выполняются оба равенства (6.12), то события А и В являются независимыми. ■
Глава 6. Теория вероятностей 617 На практике интуитивные предположения о независимости событий являются очевидными. Например, если монета подбрасывается дважды, второе подбрасывание не зависит от первого (монета не имеет памяти), если из колоды дважды извлекается карта с возвращением, второе испытание не зависит от первого (колода не имеет памяти) и т.д. Однако существуют пары событий, которые не являются очевидно независимыми (см. пример 6.36). Пример 6.35 (Проверка независимости). Покажите, что при двух подбрасываниях идеальной монеты события “при первом подбрасывании выпал орел” и “при втором подбрасывании выпал орел” являются независимыми. Решение. Рассмотрим пространство равновероятных элементарных событий, возникающих при двукратном подбрасывании идеальной монеты. S = {ОО,ОР,РО,РР}. Введем следующие обозначения. А — при первом подбрасывании выпал орел: {ОО, ОР}, В — при втором подбрасывании выпал орел: {ОО, ОР}. Вероятности этих событий вычисляются следующим образом. 2 1 2 1 1 v 7 4 2 4 2 4 Итак, Р(ДПВ) = ^ = |.^=Р(А)Р(В). Следовательно, события Ап В являются независимыми. ■ Упражнение 6.35. Используя данные из примера 6.35, вычислите вероятность Р(В | А) и сравните ее с величиной Р(В). ■ Пример 6.36 (Проверка независимости). Из стандартной колоды, содержащей 52 карты, извлекается одна карта. Проверьте независимость следующих событий (попробуйте заранее предугадать результат). 1. Е — вынута карта пиковой масти. F — вынута фигура. 2. G — вынута карта трефовой масти. Н — вынута карта червонной масти. Решение. 1. Для проверки независимости событий Е и F вычислим вероятности Р(Е П F) и P(E)P(F). Если они равны, то события Е и F независимы; если они не равны, то события Е и F зависимы. Q 14 19 Ч P(PnF) = ^, P(f;)p(F) = ^.^ = ±. oZ oZ oZ oZ Итак, события E и F независимы. (Вы это предполагали?)
618 Часть II. Конечная математика 2. Рассуждая аналогичным образом, вычислим следующие вероятности. 1Т 14 1 Р(СПН)=Р(0) = О, P(G)P(H) = OZ OZ 10 Итак, события G и Н являются зависимыми. (Вы это предполагали?) ■ Предупреждение. Студенты часто путают взаимно исключающие (непересекающиеся) события с независимыми событиями. Из одного факта не следует другой. В действительности нетрудно показать (см. задачу 47, практикум 6.5), что любые два взаимно исключающих события А и В с ненулевыми вероятностями всегда зависимы! Упражнение 6.36. Из стандартной колоды, содержащей 52 карты, извлекается одна карта. Проверьте, являются ли независимыми следующие события. 1. Е — вынута карта красной масти. F — ранг извлеченной карты делится на 5 (фигурам номер не присваивается). 2. G — вынут король. Н — вынута дама. ■ Задание 6.11. Являются ли независимыми следующие события? Обоснуйте свой ответ. А — команда Golden Eagles выиграла первый раунд чемпионата колледжей по баскетболу. В — команда Golden Eagles выиграла второй раунд чемпионата колледжей по баскетболу. ■ Понятие независимости можно распространить на несколько событий. Множество независимых событий9 События, образующие некоторое множество событий, называются независимыми, если для каждого конечного подмножества этого множества {Е^, Е2,.. ., Ек} выполняется равенство Р(ЕГ А Е2 А ... А Ек) = Р(Р1)Р(Р2) Р(ЕЛ). (6.13) В Пример 6.37 (Компьютерные системы управления). Космический корабль имеет четыре независимые компьютерные системы управления. Чему равна вероятность выхода из строя всех четырех систем, если вероятность выхода из строя (во время полета) каждой из систем равна 0,001? Решение. Введем следующие обозначения. Pi — выход из строя системы 1, Е2 — выход из строя системы 2, 9В русскоязычной литературе принято называть такие события независимыми в совокупности. — Примеч. ред.
Глава 6. Теория вероятностей 619 Ез — выход из строя системы 3, Е4 — выход из строя системы 4. Поскольку события Ei, Е2, Ез и Е^ независимы, получаем следующий результат. P(Ei Г) Е2 П Ез П Р4) = Р(Р1)Р(Р2)Р(^з)Р(Р4) = = (0,001)4 = 0,000000000001. Eg Упражнение 6.37. и Одна игральная кость бросается шесть раз. Чему равна вероятность получить последовательность 1, 2, 3, 4, 5, 6? ■ Резюме Ключевые понятия Условная вероятность PMI Р(ВПА) Р(Л 1 в) “ Р(В) ’ Р(В 1 л) - Р(А) • (Замечание: Р(А | В) — вероятность, определенная на новом пространстве элементарных исходов В, в то время как Р(А П В) и Р(В) — вероятности, определенные на первоначальном пространстве элементарных исходов S.) Правило умножения Р(А П В) = Р(А)Р(В | А) = Р(В)Р(А | В). Независимые события ■ События А и В независимы тогда и только тогда, когда Р(АПВ) = Р(А)Р(В). ■ Если А и В — независимые события с ненулевыми вероятностями, то Р(А | В) = Р(А) и Р(В | А) = Р(В). ■ Если А и В события с ненулевыми вероятностями и либо Р(А | В) = Р(А), либо Р(В | А) = Р(В), то А и В независимы. ■ Если Bi,В2} • ♦♦,Еп независимы, то Р(В! П Е2 П ... П Еп) = Р(В1)Р(В2) Р(ВП). Ответы к упражнениям 6.30. 1) 0,5. 6.31. 1) Р(Я') = 0,640; 2) 0,4. 2) Р(А П Я') = 0,015.
620 Часть II. Конечная математика 3) Р(Л |ЯЗ = =0,023. 6.32. Р(М Г\С) = Р(М)Р(С | М) = 0,32. 6.33. |. 6.34. 0,72. 6.35. Р(В | А) = = f = I = Р(В). ± I У1 у х 2 6.36. 1) События Е и F являются независимыми. 2) События G и F являются зависимыми. 6.37. Q) «0,0000214. Практикум 6.5 А Решите задачи 1-16, используя вероятности событий в пространстве элементарных исходов S, приведенные в таблице. АВС Всего Т 0,10 0,04 0Д)6 0^0“ Е 0,40 0,26 0,14 0,80 Всего 0,50 0,30 0,20 1,00 В задачах 1-4 найдите указанные вероятности непосредственно в таблице. 1. Р(В). 2. Р(Е). 3. Р(ВПР). 4. Р(СПР). В задачах 5-12 найдите указанные вероятности, используя равенство (6.9) и соответствующее табличное значение. 5. P(D | В). 6. Р(С | Е). 7. Р(В | D). 8. Р(Е | С). 9. Р(Р | С). 10. Р(Р | А). 11. Р(А | С). 12. Р(В | В). В задачах 13-16 проверьте независимость указанных событий. 13. А и D. 14. А и Е. 15. С и D. 16. С и Е. 17. Идеальная монета подбрасывается восемь раз. а) Чему равна вероятность того, что при восьмом подбрасывании выпадет орел, если в предшествующих семи подбрасываниях выпал орел? б) Чему равна вероятность выпадения восьми орлов или восьми решек?
Глава 6. Теория вероятностей 621 18. Идеальная кость бросается пять раз. а) Чему равна вероятность выпадения шестерки при пятом подбрасывании, если при предыдущих четырех подбрасываниях выпала шестерка? б) Чему равна вероятность того, что каждый раз будет выпадать одно и то же число? 19. Стрелка раскручивается над разделенным на секторы круговом диске, как показано на рисунке. Вероятность того, что стрелка остановится над определенным целым числом (от 1 до 5), равна отношению площади соответствующего кругового сектора к площади всего круга (см. таблицу). Введем следующие обозначения. Е — стрелка остановилась на четном числе. F — стрелка остановилась на числе, которое меньше четырех. а) Вычислите вероятность P(F | Е). б) Проверьте независимость событий Е и F, 20. Повторите решение задачи 19 для следующих событий. Е — стрелка остановилась на нечетном числе. F — стрелка остановилась на простом числе. Вычислите заданные вероятности в задачах 21 и 22, обратившись к следующему вероятностному дереву. 21. a) P(MAS). 22. а) Р^АЙ). б) Р(Я). б) Р(5).
622 Часть II. Конечная математика Б * 23. Идеальная монета подбрасывается дважды. Рассмотрим пространство равнове¬ роятных элементарных событий S = {ОО, ОР, РО,РР}. Интерес представляют следующие события. Ei — орел при первом подбрасывании, Е2 — решка при первом подбрасывании, Е3 — решка при втором подбрасывании, Е± — орел при втором подбрасывании. Определите, являются ли указанные пары событий независимыми или взаимно исключающими. a) Ei и Е4. б) Ei и Е2. * 24. Для указанных пар событий (см. задачу 23) определите, являются ли они неза¬ висимыми или взаимно исключающими. a) Ei и Е3. б) Е3 и Е4. 25. Чему равна вероятность того, что при двух подбрасываниях идеальной игральной кости выпадет четное число при каждом подбрасывании? При первом или втором подбрасывании? 26. Чему равны вероятности того, что при двух подбрасываниях идеальной игральной кости выпадет сумма, по крайней мере равная пяти при каждом подбрасывании и равная пяти при первом или втором подбрасывании? 27. Из стандартной колоды, содержащей 52 карты, последовательно извлекаются две карты. Чему равна вероятность того, что первая карта имеет трефовую масть, а вторая — червонную при следующих условиях? а) Карты извлекаются без возвращения. б) Карты извлекаются с возвращением. 28. Из стандартной колоды, содержащей 52 карты, последовательно извлекаются две карты. Чему равна вероятность того, что обе карты красной масти при следующих условиях? а) Карты извлекаются без возвращения. б) Карты извлекаются с возвращением. 29. Из стандартной колоды, содержащей 52 карты, случайным образом извлекается одна карта. События G и Н определены следующим образом. G — выбрана карта черной масти, Н — выбрана карта, номер которой делится на три (фигуры не учитываются). а) Вычислите вероятность Р(Н | G). б) Проверьте независимость событий Н и G. 30. Из стандартной колоды, содержащей 52 карты, случайным образом извлекается одна карта. События М и N определены следующим образом: М — извлечена карта бубновой масти, N — извлечена карта с четным номером (фигуры не учитываются). а) Вычислите вероятность P(N | М).
Глава 6. Теория вероятностей 623 б) Проверьте независимость событий М и N. 31. Пусть событие А означает, что в семье все дети — одного пола, а событие В означает, что в семье есть по крайней мере один мальчик. Вероятность того, что в семье есть девочка равна вероятности того, что в семье есть мальчик (обе равны 0,5). Проверьте независимость событий А и В при следующих условиях а) В семье двое детей. б) В семье трое детей. 32. Эксперимент состоит в подбрасывании п монет. Пусть событие А означает, что выпало по крайней мере два орла, а событие В означает, что все монеты легли одной стороной. Проверьте независимость событий Ап В при следующих условиях. а) Подбрасываются две монеты. б) Подбрасываются три монеты. В задачах 33-36 проводится следующий эксперимент: из урны, содержащей два красных и пять белых шаров последовательно извлекаются два шара. Пусть событие Ri означает, что шар, извлеченный при i-м испытании, окажется красным, aWi — белым. 33. Постройте вероятностное дерево для этого эксперимента и найдите вероятность каждого из событий Я1 А Яг, R\ A VT2, W\ П R^, Wi П VT2 при следующих условиях. а) Шар, извлеченный при первом испытании, был возвращен в урну перед вторым испытанием. б) Шар, извлеченный при первом испытании, не был возвращен в урну перед вторым испытанием. 34. Вычислите вероятность того, что шар, извлеченный при втором испытании, окажется красным при следующих условиях. а) Шар, извлеченный при первом испытании, был возвращен в урну перед вторым испытанием. б) Шар, извлеченный при первом испытании, не был возвращен в урну перед вторым испытанием. 35. Найдите вероятность того, что по крайней мере один из двух извлеченных шаров окажется красным при следующих условиях. а) Шар, извлеченный при первом испытании, был возвращен в урну перед вторым испытанием. б) Шар, извлеченный при первом испытании, не был возвращен в урну перед вторым испытанием. 36. Вычислите вероятность того, что оба извлеченных шара будут иметь один и тот же цвет при следующих условиях. а) Шар, извлеченный при первом испытании, был возвращен в урну перед вторым испытанием. б) Шар, извлеченный при первом испытании, не был возвращен в урну перед вторым испытанием.
624 Часть II. Конечная математика В задачах 37 и 38 рассмотрите справедливость следующих утверждений. Если утверждение справедливо всегда, объясните, почему. Если нет, приведите контрпример. * 37. Справедливо ли следующее утверждение? а) Если А и В — независимые события, то Р(А | В) = Р(В | А). б) Если Р(ААВ) = Р(А)Р(В | А), то события АиВ являются независимыми. * 38. Справедливо ли следующее утверждение? а) Если из урны, содержащей т красных и п белых шаров (m 1 и n 1), последовательно извлекаются с возвращением два шара, то P(Wi А Я2) = = Р(Р1ПРИ2). б) Если из урны, содержащей т красных и п белых шаров (m > 1 и n > 1), последовательно извлекаются без возвращения два шара, то P(W± А Я2) = = P(fl1AW2). В 39. Урна содержит два красных, три белых и четыре зеленых шара. Из урны последовательно без возвращения извлекаются два шара. Чему равна вероятность того, что оба шара будут иметь один и тот же цвет? 40. Чему равна вероятность того, что в эксперименте, описанном в задаче 39, не будет вынут ни один белый шар? 41. Урна содержит две однодолларовые банкноты, одну пятидолларовую и одну десятидолларовую. Игрок извлекает банкноты из урны без возвращения одну за другой до тех пор, пока не будет вынута десятидолларовая банкнота. На этом игра прекращается. Все банкноты достаются игроку. а) Чему равна вероятность выиграть 16 долл.? б) Чему равна вероятность вытащить все банкноты из урны? в) Чему равна вероятность того, что игра прекратится на втором испытании? 42. Анна и Барбара играют в теннис. Побеждает тот, кто выиграет два сета. Для любого сета вероятность того, что Анна выиграет, равна j. Найдите вероятность следующих событий. а) Анна выиграет матч. б) Будут сыграны три сета. в) Игрок, выигравший первый сет, выиграет матч. 43. Докажите, что если А и В — независимые события с ненулевыми вероятностями в пространстве элементарных исходов S, то Р(А | В) = Р(А) и Р(В | А) = Р(В). 44. Докажите, что если А и В — события с ненулевыми вероятностями в пространстве элементарных исходов S и либо Р(А | В) = Р(А), либо Р(В | А) = Р(В), то события АиВ являются независимыми. 45. Докажите, что Р(А | А) = 1, если Р(А) 0. 46. Докажите, что Р(А | В) -I- Р(А' | В) = 1. 47. Докажите, что события АиВ зависимы, если они являются взаимно исключающими и Р(А) 0 и Р(В) 0.
Глава 6. Теория вероятностей 625 48. Докажите, что Р(А | В) = 1, если множество В является подмножеством множества А и Р(В) 0. Применение математики Экономика и бизнес 49. WWW Трудовые отношения. Чтобы определить отношение служащих к последней забастовке, социологи случайным образом выбрали 1000 служащих, провели среди них опрос и составили следующую таблицу. Тарификация заработной платы Всего, чел. Почасовая (Н) Ставка (S) Ставка плюс премия (В) Поддерживают, чел. (У) 400 180 20 600 Против, чел. (N) 150 120 130 400 Всего, чел. 550 300 150 1000 а) Превратите эти числа в вероятности, разделив каждое из них на 1000. б) Чему равна вероятность того, что служащий поддержит забастовку, при условии, что он работает почасово? в) Чему равна вероятность того, что служащий поддержит забастовку, при условии, что он работает на ставку и получает премии? г) Чему равна вероятность того, что служащий работает на ставку, а также чему равна вероятность того, что он работает на ставку при условии, что он поддерживает забастовку? д) Чему равна вероятность того, что служащему платят почасово? Чему равна вероятность того, что ему платят почасово при условии, что он поддерживает забастовку? е) Чему равна вероятность того, что служащий работает на ставку, получает премии и выступает против забастовки? ж) Являются ли события ShY независимыми? з) Являются ли события Н и Y независимыми? и) Являются ли события В и N независимыми? 50. Контроль качества. Производитель автомобилей собирает 37% своих автомобилей на заводе А. У 5% автомобилей, собранных на заводе А, обнаруживаются сбои в работе устройства снижения токсичности отработавших газов. Чему равна вероятность того, что один из произведенных автомобилей был собран на заводе А и оснащен неправильно работающим устройством снижения токсичности отработавших газов? 51. Премиальные инициативы. Если менеджер по продажам достигает объема продаж 600 000 долл, в год, он допускается компанией к следующей премиальной игре: черный ящик содержит одну двадцатидолларовую купюру, две пятидолларовые купюры и одну однодолларовую купюру. Купюры последовательно вытаскиваются из ящика по одной без возвращения до тех пор, пока не будет
626 Часть II. Конечная математика извлечена двадцатидолларовая купюра. На этом игра прекращается. Менеджеру выплачивается премия в 1000 раз больше, чем сумма вынутых купюр. а) Чему равна вероятность выиграть премию, равную 26 000 долл.? б) Чему равна вероятность выиграть максимальную премию, равную 31 000 долл., вытащив все купюры из ящика? в) Чему равна вероятность того, что игра прекратится на третьем вытаскивании? 52. Подбор персонала. Для перевода в определенное техническое отделение компания требует, чтобы служащий прошел отборочный тест. Позволяется сделать максимум три попытки с шестимесячными интервалами между ними. Последние записи свидетельствуют о том, что 40% служащих проходят тест с первой попытки; из тех, кто потерпел неудачу при первой попытке и делал вторую попытку, тест прошли 60%; а из тех, кто потерпел неудачу при второй попытке и делал третью попытку, тест прошли 20%. а) Чему равна вероятность пройти тестирование при первой или второй попытках? б) Чему равна вероятность потерпеть неудачу при первых двух попытках и пройти тест с третьей попытки? в) Чему равна вероятность потерпеть неудачу при всех трех попытках? Биологические науки 53. WWW Контроль за использованием пищевых добавок. Компания, производящая мороженое, хочет использовать краситель для усиления цвета своего клубничного мороженого. Управление по контролю пищевых продуктов и лекарственных препаратов требует, чтобы краситель был проверен на лабораторных крысах. Результаты проверки на 1000 крыс подытожены в следующей таблице. Заболели раком (С) Не заболели раком (С') Всего Получали краситель с пищей, особей (Я) 60 440 500 Получали краситель с пищей, особей (Я') 20 480 500 Всего, особей 80 920 1000 а) Превратите эти числа в вероятности, разделив каждое из них на 1000. б) Являются ли события “заболели раком” и “получали краситель” независимыми событиями? *в) Управлению следует разрешить или запретить использование красителя? Объясните отрицательный или положительный ответ, используя вероятности Р(С | R) и Р(С). *г) Предположим, что 20 крыс получали с пищей красный краситель и заболели раком, всего раком заболели 80 крыс, а краситель получали в общей сложности 500 крыс. Как должно поступить управление, основываясь на этих результатах? Объясните, почему.
Глава 6. Теория вероятностей 627 54. WWW Генетика. Для изучения частоты проявления дальтонизма у мужчин и женщин случайным образом были опрошены 1000 человек. Были зафиксированы следующие результаты. Женщины, F Мужчины, F' Всего Дальтонизм, С 2 24 26 Нормальное зрения, С' 518 456 974 Всего 520 480 1000 а) Превратите эти числа в вероятности, разделив каждое из них на 1000. б) Чему равна вероятность того, что опрашиваемый — женщина, при условии, что она — дальтоник? в) Чему равна вероятность того, что опрашиваемый — дальтоник, при условии, что он — женщина? г) Являются ли события “быть дальтоником” и “быть мужчиной” независимыми? д) Являются ли события “быть дальтоником” и “быть женщиной” независимыми? Социальные науки 55. Психология. Для сравнения коэффициента IQ у мужчин и женщин случайным образом были отобраны 1000 человек. Были зафиксированы следующие результаты. Ниже 90, А IQ 90-120, В Выше 120, С Всего Женщины F 120 286 104 520 Мужчины F' 130 264 96 480 Всего 250 550 200 1000 а) Превратите эти числа в вероятности, разделив каждое из них на 1000. б) Чему равна вероятность того, что человек имеет коэффициент IQ ниже 90, при условии, что он — женщина? Ответьте на тот же вопрос, если он — мужчина. в) Чему равна вероятность того, что человек имеет коэффициент IQ выше 120, при условии, что этот человек — женщина? Ответьте на тот же вопрос, если он — мужчина. г) Чему равна вероятность того, что человек имеет коэффициент IQ ниже 90? д) Чему равна вероятность того, что человек имеет коэффициент IQ между 90 и 120? Чему равна вероятность того, что человек имеет коэффициент IQ между 90 и 120 при условии, что он — мужчина? е) Чему равна вероятность того, что человек — женщина и имеет коэффициент IQ выше 120? ж) Зависит ли каждое из событий А, В или С от событий F или F'?
628 Часть II. Конечная математика 56. Избирательные стратегии. Опрос жителей некоторого избирательного округа в большом городе выявил, что 55% жителей оказались членами демократической партии, и 60% членов демократической партии голосовали на последних выборах. Чему равна вероятность того, что случайно выбранный житель избирательного округа является членом демократической партии и голосовал на последних выборах? 6.6. Формула Байеса В предыдущем разделе рассматривалась вероятность появления события при условии, что перед ним произошло другое событие. Теперь перейдем к обратной задаче и попытаемся найти вероятность более раннего события при условии появления более позднего. Большое количество практических задач имеют именно такую формулировку. Сначала рассмотрим относительно простую задачу. Пример 6.38 (Вероятность раннего события при условии появления позднего). В первой урне содержатся три синих и два белых шара, а во второй урне — один синий и три белых (рис. 6.31). Подбрасывается игральный кубик. Если выпадет число 1 или 2, шар извлекается из первой урны, в противном случае шар извлекается из второй урны. Чему равна вероятность того, что шар вынут из первой урны, при условии, что он оказался синим? Чему равна эта вероятность для второй урны? Урна Рис. 6.31. Урны с шарами Урна Решение. Построим вероятностное дерево, полагая, что U\ обозначает первую урну, U2 — вторую, В — синий шар, a W — белый. Затем различным ветвям исходов припишем соответствующие вероятности. Например, Р (Ui) = |, Р (В | Ui) = | и т.д. Нас интересует вероятность P(f7i | В), т.е. вероятность того, что шар извлечен из первой урны, при условии, что он — синий. Используя равенство (6.9) из раздела 6.5,
Глава 6. Теория вероятностей можно записать следующую формулу. Р(^1|В) = Р (Ui Л В) Р(В) 629 (6.14) Изучив дерево, можно увидеть, что событие В находится на концах двух разных ветвей. Таким образом, Р(В) = Р(СЛ ПВ) + Р([/2 А В). (6.15) После подстановки равенства (6.15) в равенство (6.14) получим следующее выражение. P(U I В} = - Р(^АВ) = 11 1 ' Р(С71ПВ)+Р(С72ПВ) P(C/1)P(B|L/1) = Р([71)Р(В|С71) + Р(172)Р(В|С/2) Р(В|С71)Р(С/1) Р(В I i^1)P(i;1) + P(B I и2)р(и2у Р(А П В) = = Р(А)Р(В | А) (6.16) Формулу (6.16) в действительности намного проще использовать, чем кажется. Нет необходимости ее запоминать: просто необходимо понять ее вид относительно приведенного выше вероятностного дерева. Р(В | £71) = произведение вероятностей ветвей, ведущих к В через Ui = /3\ fl\ = \ 5 J \ 3 / Обычно стартуем из вершины В и движемся обратно через Ui. Р(В | U2) = произведение вероятностей ветвей, ведущих к В через U2 = /1\ /2\ = 1-1 I - I . Обычно стартуем из вершины В и движемся обратно через СГ2. Равенство (6.16) теперь можно интерпретировать в терминах вероятностного дерева следующим образом. Р(1/1 I в) = Р(и2 I В) = произведение вероятностей на ветви, ведущей к В через U\ _ сумма всех произведений вероятностей на ветвях, ведущих к В 3 1 л 5 * з _ _ ~ Л сс ~ 3 1,1 2 “ 11 5 3'43 произведение вероятностей на ветви, ведущей к В через U2 сумма всех произведений вероятностей на ветвях, ведущих к В 1.2 с - 3 Т,31-.2 =П^0’45- 5*3'4 3 (Замечание. Можно также вычислить вероятность P(U2 | В), вычитая величину P(U\ | В) из единицы. Почему?) ■ Упражнение 6.38. Повторите решение примера 6.38, но найдите P(Ui | VK) и P(U2 | IV). ■
630 Часть II. Конечная математика Задание 6.12. Проанализируйте следующее вероятностное дерево. c+J=l a+b=l a,b,c,d,e,f^O e+f=l 1. Рассмотрите различия между вероятностями Р(М | U) и P(U | М), а также между вероятностями P(N | V) и P(V | АГ) в терминах вероятностей a, b, с, d9 ей /. 2. Покажите, что ас + ad + be + bf = 1. В чем заключается значимость этого результата? I При обобщении результатов примера 6.38 оказывается полезным изучение структуры событий на диаграмме Венна, изображенной на рис. 6.32. Заметим, что события U\ и U? являются взаимно исключающими (непересекающимися), и их объединение равно S. Следующие два равенства можно интерпретировать в терминах этой диаграммы. Р(и I В} = PJP1S1B) = Р^ПВ) V11 ’ Р(В) Р (СА П В) + В (t/2 Л В) ’ Р(ТТ I R) = = Р(Г2ПВ) k 2 1 ’ Р(В) Р (Ur П В) + Р (и2 л ву Просмотрите внимательно равенства и диаграмму. Рис. 6.32. Диаграмма Венна для двух взаимоисключающих событий Конечно, нет причины останавливаться на этом. Предположим, что Ui, U2 и U3 — три взаимно исключающих события, объединение которых образует все пространство элементарных событий S. Тогда для произвольного события Е в пространстве 5, для которого Р(Е) 0, диаграмма Венна выглядит так, как показано на рис. 6.33, и Р(и \Е}= Р^ПД) = В(£71ПЕ) 1 1 1 7 Р(В) Р(г71ПВ) + Р(С/2ПВ) + Р(173ПВ)’ Аналогичные результаты имеют место для событий [72 и U$.
Глава 6. Теория вероятностей 631 Рис. 6.33. Диаграмма Венна для трех взаимоисключающих событий Рассуждая аналогичным образом, получим следующую известную теорему, которая впервые была установлена пресвитерианским священником Томасом Байесом (Thomas Bayes) (1702-1763). Теорема 6.3 (Формула Байеса). Пусть 171, [/2, • • •, Un — п взаимно исключающих событий, объединение которых образует все пространство элементарных исходов S. Пусть Е — произвольное событие в пространстве S такое, что Р(Е) / 0. Тогда P{U1 1 - ~Р(Ё) - Р^ПЕ) “ Р^хПЕ)+ Р(С/2 А£) + •••+ Р(Е7ПАЕ) “ Р(Е|С/1)Р(С/1) P(E\U1)P(U1)^P(E\U2)P(U2)^-^P(E\Un)P(Uny Аналогичные результаты имеют место для U2) , Un. ■ Нет необходимости запоминать формулу Байеса. На практике часто проще нарисовать вероятностное дерево и использовать следующее утверждение. Формула Байеса и вероятностные деревья произведение вероятностей ветвей, ведущих р (у I — к вершине Е через вершину Ui ' 1 ' ' сумма всех произведений вероятностей на ветвях, ведущих к вершине Е Аналогичные результаты имеют место для событий U2, , Un. В Пример 6.39 (Диагностирование туберкулеза). Для диагностирования туберкулеза разработан новый недорогой тест. Чтобы оценить эффективность нового текста, перед его использованием было случайным образом отобрано 1000 человек, диагноз которых устанавливался предельно точным, но дорогим методом. Точная методика диагностирования показала, что 8% из 1000 тестируемых людей больны туберкулезом. Впоследствии каждый из этих 1000 лиц подвергается новому исследованию. Получены следующие результаты: новый тест выявляет туберкулез у 96% больных и у 2% здоровых. Основываясь на этих результатах, вычислите вероятность того, что случайно выбранный человек действительно болен туберкулезом, при условии, что новый тест идентифицировал у него
632 Часть II. Конечная математика это заболевание. Чему равна вероятность того, что человек не болен туберкулезом, при условии, что новый тест идентифицирует у него это заболевание? (Иначе говоря, чему равна вероятность того, что новый тест дает ложноположительный результат?) Решение, Сначала построим соответствующее вероятностное дерево. Т — Туберкулез выявлен (точный метод) Г— Туберкулез не выявлен (точный метод) S — Туберкулез выявлен (новый метод) S' — Туберкулез не выявлен (новый метод) Нас интересует вероятность Р(Т | S), т.е. вероятность того, что человек болен туберкулезом, при условии, что новый тест идентифицировал у него это заболевание. Формула Байеса для данного случая имеет следующий вид. произведение вероятностей ветвей, ведущих р (р I _ к вершине S через вершину Т ' I ' сумма всех произведений вероятностей на ветвях, ведущих к вершине S Подставляя соответствующие значения из вероятностного дерева, получим следующий ответ. = 0,08-0,96 = 1 1 ’ 0,08-0,96 + 0,92-0,02 ’ ’ Вероятность того, что у человека нет туберкулеза, при условии, что новый тест выявляет у него это заболевание, обозначенная через Р(Т' | S), задается следующей формулой. Р(Г | S') = 1 - Р(Т | S) = Р(Т | S) + Р(Т' | S) = 1 = 1-0,81 = 0,19. а Другие важные вопросы, на которые следует ответить, сформулированы в упражнении 6.39. Q Упражнение 6.39. Чему равна вероятность того, что человек болен туберкулезом, при условии, что новый тест отрицает наличие у него туберкулеза? (Иначе говоря, чему равна вероятность того, что новый тест дает ложноотрицательный результат?) Чему равна вероятность того, что человек не болеет туберкулезом, при условии, что новый тест отрицает наличие у него туберкулеза? ■ Пример 6.40 (Качество продукции). Компания, владеющая тремя заводами, еженедельно производит 1000 холодильников. Завод А еженедельно выпускает 350 хо¬ лодильников, завод В — 250, а завод С — 400. Статистические данные свидетельствуют от том, что количество бракованных холодильников равно 5% на заводе А, 3% на заводе В и 7% на заводе С. Все холодильники доставляются на центральный склад. Чему равна вероятность того, что случайно выбранный со склада холодильник произведен на заводе А, при условии, что он бракованный?
Глава 6. Теория вероятностей 633 Решение. Начнем с построения вероятностного дерева. Начало А — выпущен заводом А В — выпущен заводом В С — выпущен заводом С D — бракованный Z)’—рабочий Вероятность того, что бракованный холодильник был выпущен заводом А, равна Р(А | D). Формула Байеса для данного случая имеет следующий вид. произведение вероятностей ветвей, ведущих Р (A I D) — к веРшине & через вершину А ' ' ' ~ сумма всех произведений вероятностей на ветвях, ведущих к вершине D Используя значения вероятностного дерева, получим такой ответ. 0,35 • 0,05 Р (A I « 0 33 v 1 ' 0,35 • 0,05 + 0,25 • 0,03 + 0,40 • 0,07 ’ ’ И Упражнение 6.40. Чему равна вероятность того, что бракованный холодильник в примере 6.40, случайно выбранный на складе, выпущен заводом В? Чему равна вероятность того, что этот холодильник выпущен заводом С? ■ Задание 6.13. Рассмотрите следующее вероятностное дерево. a, b, с, d, Предположим, что U и М — независимые вероятности a, b, с, d, ей /? события. Как их независимость влияет на Ответы к упражнениям 4 15 6.38. Р (t/i | W) = - « 0,21; Р (U2 | W) = - « 0,79. 6.39. Р(Т | S') = 0,004; Р(Т' | S') = 0,996. 6.40. Р(В | D) « 0,14; Р(С | D) « 0,53.
634 Часть II. Конечная математика Практикум 6.6 В задачах 1-6 необходимо найти вероятности, используя следующее вероятностное дерево. 2. P(N П В) = P(N)P(B | N). 4. | Р(В) = Р(М П В) + P(N П В). 1. Р(М П А) = Р(М)Р(А I М). 3. Р(А) = Р(М П А) + P(W П А). 5 Р (М I А) — — — 5 ( 1 } Р (М Г1 А) + Р (ЛГ П А)’ 6 P(N\B\ = РУ^ГВ} ’ I ) Р(ЛГПВ) + Р(Л/ПВ)' В задачах 7-10 необходимо найти вероятности, используя следующую диаграмму Венна и формулу Байеса (предполагается, что элементарные исходы в пространстве S равновероятны). 9. Р(1А | Я'). 7. Р(С/1 | Я). 10. Р(Р2 | R'). Б В задачах 11-16 необходимо найти вероятности, используя следующую древовидную диаграмму и формулу Байеса. 11. P(U | С). 14. Р(Р | С). 12. Р(у | С"). 15. P{V | С). 13. P(IV | С). 16. P(W | С").
Глава 6. Теория вероятностей 635 В задачах 17-22 необходимо найти вероятности событий, используя следующую диаграмму Венна и формулу Байеса (предполагается, что элементарные исходы в пространстве S равновероятны). В задачах 23 и 24 необходимо использовать вероятности из первого вероятного дерева для определения вероятностей всех ветвей второго вероятностного дерева. В задачах 25-28 случайным образом выбирается одна из двух урн. Выбор каждой из урн равновероятен. Затем из выбранной урны извлекается шар. Первая урна содержит один белый и четыре красных шара, а вторая урна содержит три белых и два красных шара. 25. Если вынут белый шар, чему равна вероятность того, что он вынут из первой урны? 26. Если вынут белый шар, чему равна вероятность того, что он вынут из второй урны? 27. Если вынут красный шар, чему равна вероятность того, что он вынут из второй урны?
636 Часть II. Конечная математика 28. Если вынут красный шар, чему равна вероятность того, что он вынут из первой урны? В задачах 29 и 30 урна содержит четыре красных и пять белых шаров. Два шара извлекаются последовательно без возвращения. 29. Если второй шар белый, чему равна вероятность того, что первый шар был белым? 30. Если второй шар красный, чему равна вероятность того, что первый шар красный? В задачах 31 и 32 первая урна содержит семь красных и три белых шара. Вторая урна содержит четыре красных и пять белых шаров. Из первой урны извлекается шар, который кладется во вторую урну. Затем из второй урны извлекается шар. 31. Если из второй урны извлечен красный шар, чему равна вероятность того, что из первой урны был вынут красный шар? 32. Если из второй урны вынут белый шар, чему равна вероятность того, что из первой урны был вынут белый шар? В задачах 33 и 34 необходимо использовать следующее вероятностное дерево. c + d=\ а + b=\ e+f=l a, b, * 33. Предположим, что с = е. Рассмотрите зависимость или независимость событий U и М. * 34. Предположим, что с = d = е = f. Рассмотрите зависимость или независимость событий М и N. В задачах 35 и 36 два шара последовательно извлекаются из урны, содержащей т голубых и п белых шаров (т 2, п 2). Рассмотрите справедливость каждого из утверждений. Если утверждение всегда справедливо, рассмотрите, почему. Если нет, приведите контрпример. * 35. Справедливы ли следующие высказывания? а) Если два шара вынуты с возвращением, то P(Bi | В2) = Р(В2 | Bi). б) Если два шара вынуты без возвращения, то P(Bi | В2) = Р(В2 | Bi). * 36. Справедливы ли следующие высказывания? а) Если два шара вынуты с возвращением, то Р(В\ | W2) = P(W2 | Bi). б) Если два шара вынуты без возвращения, то P(J3i | W2) = P(W2 | Bi). В 37. Из стандартной колоды, содержащей 52 карты, последовательно без возвращения вынуты две карты, и вторая карта оказалась червонной масти. Чему равна вероятность того, что первая карта червонной масти?
Глава 6. Теория вероятностей 637 38. Урна содержит 10 шаров, пронумерованных от 1 до 10. Последовательно без возвращения извлекаются два шара. Если второй шар имеет номер четыре, чему равна вероятность того, что первый шар имеет меньший номер? Чему равна вероятность того, что первый шар имеет четный номер? В задачах 39 и 40 трехкарточный расклад берется из стандартной колоды, содержащей 52 карты, а затем из него случайным образом извлекается одна из трех карт. 39. Если извлечена бубна, чему равна вероятность того, что все три карты бубновой масти? 40. Если извлечена бубна, чему равна вероятность того, что это единственная бубна среди этих трех карт? 41. Докажите, что P(U\ | R) + P(U[ | R) = 1. 42. Если Ui и U? — два взаимно исключающих события, объединение которых образует пространство равновероятных элементарных исходов S, а Е — произвольное случайное событие из пространства S, такое что Р(Е) ± 0. Докажите, что Р(и I п = п (U1 п 1 11 1 п(и1ПЕ) + п(и2ПЕУ Применение математики В следующих задачах слово “вероятность ” часто понимается в значении “приближенная эмпирическая вероятность ”. Экономика и бизнес 43. Проверка персонала. Руководство компании считает, что 30% нанятых секретарей работают плохо. Директору поручили случайным образом выбрать 100 секретарей и предложить им пройти тестирование. Тест успешно прошли 90% хорошо работающих секретарей и 20% работающих плохо. Чему равна вероятность того, что секретарь является хорошим работником, если он прошел тестирование? Если секретарь потерпел неудачу в тестировании, чему равна вероятность того, что он является хорошим секретарем? 44. Квалификация сотрудников. Руководство компании считает, что 75% ее сотрудников работают хорошо, а 25% — плохо. По данным отдела кадров, 80% сотрудников, работающих хорошо, имели опыт работы, в то время как среди плохо работающих служащих эта доля составляет только 40%. Предположим, что компания приняла на работу человека, имеющего опыт работы. Чему равна вероятность того, что этот человек будет удовлетворять требованиям компании? Если нанят человек, не имеющий опыта работы, чему равна вероятность того, что этот человек будет хорошо работать? 45. Качество продукции. Торговая компания получает радиоприемники с таймером в указанных количествах от трех субподрядчиков:: 20% — от компании А, 40% — от компании В и 40% — от компании С. Количество бракованной продукции, поставляемой субподрядчиками, составляет 1, 3 и 2% соответственно. Предположим, что покупатель возвращает бракованный радиоприемник. Чему равна вероятность того, что он выпущен компанией А, В или С соответственно?
638 Часть II. Конечная математика 46. Качество продукции. Компьютерный магазин закупает три модели компьютеров — А, В и С. Среди компьютеров, которые он покупает, 60% составляет модель А, 25% — модель В и 15% — модель С. Оказалось, что 20% компьютеров модели А, 15% компьютеров модели В и 5% компьютеров модели С в течение гарантийного периода подлежат ремонту. Если компьютер подлежат ремонту в течение гарантийного периода, чему равна вероятность того, что это компьютер модели А, В или С соответственно? Биологические науки 47. WWW Диагностирование онкологических заболеваний. Исследователи разработали новый тест для выявления определенного типа рака. Прежде чем использовать, необходимо оценить его эффективность. Для этого уровень заболевания сначала определяется другими, проверенными методами в случайной выборке из 1000 взрослых людей. Оказывается, что 2% из этой выборки болеют определенным типом рака. Впоследствии все 1000 человек подвергаются диагностике с помощью нового теста. Новый тест выявляет рак у 98% тех, кто им болен, и у 1% тех, кто им не болен. Основываясь на этих результатах, определите, чему равна вероятность того, что случайно выбранный человек болен раком, при условии, что тест диагностирует его? Определите вероятность того, что человек болен раком, при условии, что тест его не диагностирует? 48. Тест на беременность. В случайной выборке из 200 женщин, подозревающих свою беременность, 100 оказались действительно беременными. Новый тест на беременность выявил ее у 92 из 100 беременных женщин и у 12 женщин из 100, которые не являлись таковыми. Если этот тест выявляет беременность, чему равна вероятность того, что женщина действительно беременна? Если тест определяет, что она не беременна, чему равна вероятность, что она действительно не беременна? 49. WWW Медицинское обследование. В случайной выборке, состоящей из 1000 человек, у 7% больная печень. Среди тех, кто имеет больную печень, 40% являются сильно пьющими, 50% являются умеренно пьющими и 10% — трезвенниками. Из тех, кто имеет здоровую печень, 10% являются сильно пьющими, 70% являются умеренно пьющими и 20% — трезвенниками. Предположим, что из генеральной совокупности случайным образом выбирается человек, который оказывается сильно пьющим. Чему равна вероятность того, что у этого человека больная печень? Чему равна эта вероятность для непьющего? 50. WWW Диагностика туберкулеза. Тест на туберкулез прошли 1000 человек, 8% которых действительно болели туберкулезом. У людей, действительно больных туберкулезом, тест выявил туберкулез в 90% случаев, не дал определенного ответа в 7% случаев и выявил отсутствие туберкулеза в 3% случаев. У здоровых людей тест выявил туберкулез в 5% случаев, не дал определенного ответа в 10% случаев и установил отсутствие туберкулеза в оставшихся 85% случаев. Чему равна вероятность того, что случайно выбранный человек болен туберкулезом, при условии, что тест выявил у него туберкулез? Чему равна вероятность того, что человек не болен туберкулезом при условии, что тест не дал определенного ответа?
Глава 6. Теория вероятностей 639 Социальные науки 51. Детектор лжи. Разработан новый детектор лжи, который нужно тщательно протестировать. Для его проверки случайным образом было отобраны 100 человек. Каждый из них извлекает карточку из ящика, содержащего 100 других карточек. Половина карточек содержит указание солгать, а другая — говорить правду. Детектор определяет ложь у 80% тех, кто действительно лжет, и у 5% тех, кто говорит правду. Чему равна вероятность того, что случайно выбранный человек солгал при условии, что детектор это правильно определил? Чему равна вероятность, что он говорит правду при условии, что детектор определяет ложь? 52. Политика. Учет избирателей в неком округе показывают, что 45% из них демократы, 35% республиканцы и 20% беспартийные. На выборах 70% демократов, 40% республиканцев и 80% беспартийных голосовали в поддержку предложения об обустройстве парка. Если случайно выбранный избиратель голосовал в поддержку предложения, чему равна вероятность того, что он является республиканцем, беспартийным или демократом соответственно? 6.7. Случайная величина, распределение вероятностей и математическое ожидание ■ Случайная величина и распределение вероятностей ■ Математическое ожидание случайной величины ■ Приниятие решений и математическое ожидание Случайная величина и распределение вероятностей Когда производится случайный эксперимент, пространство элементарных исходов S выбирается таким образом, чтобы решить все представляющие интерес вероятностные задачи. Во многих экспериментах нас могут интересовать не элементарные исходы, а числовые значения, связанные с ними. Например, если подбрасываются три монеты, можно интересоваться количеством выпавших орлов, а не определенной комбинацией. При формировании случайной выборки студентов часто необходимо знать, какую долю составляют женщины, а не какие конкретно студенты — женщины. Аналогично игрок обычно интересуется суммой очков, выпавших на сторонах игральных костей, а не комбинацией очков, выпавших на каждом кубике. Во всех этих примерах существует правило, которое приписывает каждому элементарному исходу в пространстве S единственное действительное число. Говоря языком математики, мы имеем дело с функцией (см. раздел 1.1). Этот определенный вид функции был назван “случайной величиной”. Случайная величина Случайная величина — это функция, которая приписывает числовое значение каждому элементарному исходу в пространстве S. Термин случайная величина неудачен, поскольку сущность, которую он обозначает, не является ни случайной, ни величиной. Это — функция с числовыми значениями, определенная на пространстве элементарных исходов. Однако эта терминология устоялась
640 Часть II. Конечная математика и стала стандартной, поэтому придется с ней смириться. В дальнейшем случайные величины обозначаются прописными буквами, например, X. Вернемся к эксперименту, в котором подбрасываются три монеты. Пространство 5, состоящее из равновероятных элементарных событий, описано в табл. 6.2. Предположим, что нас интересует количество орлов (0, 1, 2 или 3), появившееся при каждом подбрасывании трех монет, и вероятность каждого из этих событий. Введем случайную величину (функцию), которая определяет количество орлов для каждого элементарного события в пространстве S (смотрите второй столбец в таблице 6.2). Например, X(ei) = 0, Xfa) = 1 и т. д. Случайная величина X приписывает определенное числовое значение каждому элементарному исходу в пространстве S. Таблица 6.2. Количество орлов при подбрасывании трех монет Пространство элементарных исходов Количество орлов S X(ei) ei: PPP 0 е2: РРО 1 е3: POP 1 е4: ОРР 1 65: РОО 2 ОРО 2 е7: OOP 2 е8: ООО 3 Нас интересует вероятность появления каждого значения, или интервал значений X, т.е. вероятность 0, 1, 2 и 3 орлов при одном подбрасывании трех монет. Обозначим эту вероятность следующим образом. р(я), где х е {0,1,2,3}. Функция р называется распределением вероятностей10 случайной величины X. Чему равна величина р(2), т.е. вероятность того, что при одном подбрасывании трех монет выпадут два орла? Событие “выпадает два орла” имеет следующий вид. Е = {РОО,ОРО,ООР}. Таким образом, /п\ п С®) 3 P(2) = ^(S) =8' Рассуждая аналогично, для величин р(0), р(1) ир(3) получим распределение вероятностей случайной величины X, представленное в табл. 6.3. График вероятностного распределения часто называется гистограммой (рис. 6.34). 10Распределение вероятностей случайной величины X определяется следующим образом: р(х) = Р({е< G S | Х(вг) = я}). Чтобы не загромождать формулы, это выражение, как правило, используется в упрощенном виде: р(х) = Р{Х = х}, или р(ж).
Глава 6. Теория вероятностей 641 Таблица 6.3. Распределение вероятностей Количество орлов, х 0 1 2 3 Вероятность, р(х) | | | | Рис. 6.34. Гистограмма распределения вероятностей Изучая табл. 6.3 и рис. 6.34, легко заметить следующие факты. 1. ООЙ < 1, х €{0,1,2,3}. 2. р(0) +р(1) +р(2) +р(3) = | + | + | + | = 1- Любое вероятностное распределение случайной величины X, связанной с конечным пространством элементарных исходов S, обладает следующими общими свойствами. Распределение вероятностей случайной величины X Распределение вероятностей Р(Х = ж) = р(х) случайной величины X, удовлетворяет следующим условиям. 1. 0 р(х) 1, X € {^1,^2, • • • ,#п}> 2. p(xi) +pGe2) + • • • +p(^n) = 1, где {xi,^2,...,хп} — множество значений случайной величины X (см. рис. 6.35). На рис. 6.35 изображен процесс формирования распределения вероятностей случайной величины. Математическое ожидание случайной величины Предположим, что эксперимент, состоящий в подбрасывании трех монет, повторяется много раз. Чему равно среднее количество орлов, выпавших при одном подбрасывании (общее количество орлов при всех подбрасываниях, деленное на общее количество подбрасываний)? Обратившись к распределению вероятностей, приведенному в табл. 6.3, видим, что частота событий, при которых не выпадает ни одного орла, равна |, выпадает один орел — |, два орла — | и три орла — |. Таким образом, ожидаемое среднее коли-
642 Часть II. Конечная математика Область значений случайной величины = область определения распределения вероятностей Пространство элементарных исходов = область определения случайной величины Случайная переменная Исходы эксперимента — не обязательно числовые величины Распределение вероятностей Область значений распределения вероятностей Вероятности, приписанные значениям случайной величины Числовые величины, связанные с исходами в пространстве S Рис. 6.35. Распределение вероятностей случайной величины с конечным пространством элементарных исходов чество орлов, выпадающих при одном подбрасывании трех монет, или математическое ожидание Е(Х) (expected value), выражается следующей формулой. ад = 0- | + 1-| + 2. | + 3. | = 1,5. Следует отметить, что математическое ожидание — это не значение, которое обязательно произойдет в одном эксперименте (при одном подбрасывании трех монет не может появиться полтора орла). Эта величина представляет собой среднее значение, которое появляется при большом количестве экспериментов. Иногда количество орлов больше математического ожидания, иногда — меньше, но, если эксперимент повторяется много раз, среднее количество орлов будет близко к 1,5. Введем следующее определение математического ожидания. Математическое ожидание случайной величины X Рассмотрим распределение вероятностей случайной величины X X Х^ Х% • • • Хп Pi Pl Р2 • • • Рп Здесь pi = p(xi). Математическое ожидание случайной величины X обозначается символом Е(Х) и выражается формулой Е(Х) = xip-i + Х2Р2 + • • • + хпрп. Еще раз подчеркнем, что математическое ожидание не является ожидаемой величиной исхода в конкретном испытании. Это — взвешенное среднее возможных исходов в повторяющихся экспериментах, в котором весами являются вероятности исходов.
Глава 6. Теория вероятностей 643 Вычисление математического ожидания случайной величины X Этап 1. Построим распределение вероятностей случайной величины X. Этап 2. Умножим каждое значение Xi величины X на соответствующую вероятность его появления рг, а затем сложим результаты. Пример 6.41 (Математическое ожидание). Чему равно математическое ожидание количества очков, выпавших при подбрасывании одной игральной кости? Решение. Выберем в качестве пространства элементарных исходов множество S = {1,2,3,4,5,6}. В этом случае каждый элементарный исход является равновероятным, а случайная величина представляет собой тождественную функцию. Итак, распределение вероятностей случайной величины X имеет следующий вид. Xj 1 2 3 4 5 6 D. I I 1 1 I 1 *^ 6 6 6 6 6 6 Следовательно, Е(Х) = 1Д + 2.1+3.±+4.1 + 5.1 = ^ = 3,5. 0 0 0 0 0 0 ■ Упражнение 6.41. Предположим, что игральная кость в примере 6.41 не является идеальной, а распределение вероятностей случайной величины X имеет следующий вид. Xi 1 2 3 4 5 6 Pi 0,14 0,13 0,18 0,20 0,11 0,24 Чему равно математическое ожидание XI [Замечание: сумма вероятностей равна единице] Задание 6.14. В группе, состоящей из 10 студентов, на первом экзамене один студент набрал 95 баллов, три студента — 85, один — 82, два — 75, два — 73 и один — 65. Распределение вероятностей экзаменационных баллов случайно выбранного студента является следующим. Xi 65 73 75 82 85 95 Pi о,1 0,2 0,2 0,1 0,3 0,1 Рассмотрите взаимоотношение между математическим ожиданием этого распределения вероятностей и средним баллом, полученным студентами этой группы. ■ Пример 6.42 (Математическое ожидание). Набор из 20 батареек для калькулятора содержит две отработанные. Из 20 батареек для проверки выбирается случайная выборка, содержащая три батарейки. Пусть X — случайная величина, представляющая собой количество отработанных батареек в выборке.
644 Часть II. Конечная математика 1. Найдите распределение вероятностей случайной величины X. 2. Найдите математическое ожидание количества отработанных батареек в выборке. Решение. 1. Количество вариантов формирования выборки, содержащей три из 20 батареек (порядок не важен), равно 620,3- Итак, пространство элементарных исходов состоит из 620,3 равновероятных событий. Выборка может содержать одну или две отработанные батарейки, а может не содержать их совсем. Распределение вероятностей интересующей нас случайной величины вычисляется следующим образом. р(0) - «0,716, 620,3 Р(1) = ^£112 «0,268, 620,3 р (2) = С’2^С'18’1 « 0,016. 620,3 Подытожим полученные результаты в следующей таблице. Xi 0 1 2 ~ Pi 0,716 0,268 0,016 [Замечание: 0,716 + 0,268 + 0,016 = 1,0] 2. Математическое ожидание количества отработанных батареек в выборке вычисляется следующим образом. Е(Х) = 0 • 0,716 + 1 • 0,268 + 2 • 0,16 = 0,3. Математическое ожидание не является одним из значений случайной величины. Оно представляет собой число, к которому будет приближаться среднее количество отработанных батареек в выборке, когда эксперимент будет повторяться неограниченное число раз. ■ Упражнение 6.42. Повторите решение примера 6.42, используя случайную выборку, состоящую из четырех элементов. ■ Пример 6.43 (Математическое ожидание выигрыша). Секторы рулетки раскрашены в разные цвета и пронумерованы от 0 до 5, а выпадение каждого из шести чисел равновероятно. Игрок, поставивший один доллар на любое конкретное число, выигрывает четыре доллара (и получает ставку обратно), если стрелка останавливается на выбранном числе, в противном случае ставка теряется. Чему равно математическое ожидание игры (средняя величина выигрыша)? Решение. Пространством равновероятных элементарных событий является следующее множество. 5 = {0,1,2,3,4,5}.
Глава 6. Теория вероятностей 645 Каждый элемент этого множества появляется с вероятностью |. Случайная величина X приписывает четыре доллара выигрышному числу и —1 — каждому из оставшихся. Таким образом, распределение вероятностей случайной величины X, называемое таблицей выигрышей, выглядит так, как показано ниже. Вероятность выиграть четыре доллара равна |, а вероятность потерять один доллар — |. Таблица выигрышей (распределение вероятностей случайной величины X) Xi , долл. 4 -1 1 5 Pi 6 6 Теперь можно вычислить математическое ожидание выигрыша. Е(Х) = 4.± + (-1)^ = О о — — - ~ —0,1667 ~ —17 центов за игру. 6 Таким образом, играя достаточно долго, игрок потеряет в среднем 17 центов за игру. Используя определение справедливой игры, сформулированное в разделе 6.4, можно показать, что игра является справедливой тогда и только тогда, когда Е(Х) = 0. Игра в примере 6.43 не является справедливой. Упражнение 6.43. Повторите решение примера 6.43 при условии, что в случае выигрыша игрок получает не четыре, а пять долларов. В случае проигрыша потеря также составляет один доллар. Стала ли эта игра справедливой? ■ Задание 6.15. В ходе игры монета подбрасывается дважды. Результатом считается количество выпавших орлов. Ставка равна одному доллару. Игрок сохраняет ставку, если не выпал ни один орел, и теряет ее, если выпал один орел. Какую сумму должен заплатить крупье, если выпадет два орла, чтобы игра была справедливой? Обоснуйте свои рассуждения. ■ Q Пример 6.44 (Математическое ожидание и страхование). Предположим, вас интересует страхование автомобильной стереосистемы от кражи. Сумма страховки равна 500 долл. Страховая компания назначает премию, равную 60 долл, в год, считая, что эмпирическая вероятность кражи стереосистемы в течение года равна 0,1. Чему равно математическое ожидание выплаты страховой компании, если вы принимаете эту страховку? Решение. В действительности страхование — это игра, в которой ваша ставка равна 60 долл. Шанс получить от страховой компании 440 долл. (500 — 60) равен 0,1, а шанс потерять ставку — 0,9. Чему равно математическое ожидание этой “игры”? Построим таблицу выигрышей (распределение вероятностей случайной величины X). Затем вычислим математическое ожидание следующим образом. Е(Х) = 440 • 0,1 + (-60) • 0,9 = -10 долл.
646 Часть II. Конечная математика Таблица страховых “выигрышей” Xi, долл. 440 —60 ОД 6У Это означает, что если вы будете заключать с этой страховой компанией страховые сделки в течение многих лет, и обстоятельства будут оставаться прежними, вы будете терять в среднем 10 долл, ежегодно. ■ П Упражнение 6.44. Вычислите математическое ожидание выигрыша страховой компании в примере 6.44. ■ Принятие решений и математическое ожидание Завершим этот раздел примером принятия решений. П Пример 6.45 (Анализ принятия решений). На воскресный день на большом открытом стадионе запланирован концерт популярной музыкальной группы. Организатор концерта обратился к прогнозу погоды, чтобы узнать вероятность дождя, которая в воскресенье равна 0,24. В отсутствие дождя на концерте организатор получит доход, равный 100 тыс. долл. Если дождь все же пойдет, то доход составит всего 10 тыс. долл. Страховая компания согласилась застраховать концерт от дождя на сумму 100 тыс. долл., потребовав взнос, равный 20 тыс. долл. Следует ли организатору покупать страховку? Решение, Организатор должен сделать выбор между двумя действиями: Ai — покупать страховку и А2 — не покупать ее. Чтобы принять правильное решение, следует вычислить математическое ожидание выигрыша для каждого из действий. Распределение вероятностей выигрыша приведено в следующей таблице. Таблица страховых “выигрышей” Pi Ai (со страховкой): долл. А2 (без страховки): долл. 0,24 (дождь) 90000 10000 0,76 (нет дождя) 80 000 100 000 Отметим, что сумма, равная 90000, получена по формуле: “страховая премия” (100000 долл.) — “страховой взнос” (20000 долл.) + “издержки” (10000 долл.). Математическое ожидание выигрыша для каждого действия вычисляется следующим образом. Ai (со страховкой): Аг (без страховки): Е(Х) = xrP! + х2р2 = Е(х) = 10 000 • 0,24 + 100 000 • 0,76 = = 90 000 • 0,24 + 80 000 • 0,76 = = 78 400 долл. = 82 400 долл. Оказывается, организатору лучше купить страховой полис за 20 000 долл. Оценка среднего выигрыша в долгосрочной перспективе при выборе решения — наиболее распространенная практика принятия решений в условиях неопределенности. ■
Глава 6. Теория вероятностей 647 дня Упражнение 6.45. Чему равно математическое ожидание выигрыша страховой компании в примере 6.45, если она продаст полис? ■ Ответы к упражнениям 6.41. Е(Х) = 3,73. Pi 0,632 0,337 0,032й 2) 0,4 6.43. Е(Х) = 0 долл.; игра является справедливой. 6.44. Е(Х) = —440 • 0,1 4- 60 • 0,9 = 10 долл. (Это количество, конечно, необходимо для того, чтобы покрыть расходы и получить прибыль.) 6.45. Е(Х) = —80000 • 0,24 + 20000 • 0,76 = —4000. (Это означает, что у страховой компании есть иная информация относительно погоды, чем у организатора; другими словами, компания не должна подписывать страховой полис.) Практикум 6.7 Где это возможно, постройте распределение вероятностей соответствующей случайной величины X или таблицу выигрышей. А 1. Чему равно математическое ожидание случайной величины X, если распределение вероятностей случайной величины X задается следующей таблицей? —3 0 4 Pi 0,3 0,5 0,2 2. Чему равно математическое ожидание случайной величины X, если распределение вероятностей случайной величины X задано следующей таблицей? ~ —2 ~ б 1 ~ Pi 0,1 0,2 0,4 0,2 0,1 3. Чему равно математическое ожидание количества орлов при подбрасывании двух идеальных монет? 4. Чему равно математическое ожидание количества мальчиков для семьи с двумя детьми, если исключить рождение близнецов и предположить, что рождение мальчика и девочки равновероятно? 5. Подбрасывается идеальная монета. Если выпадает орел, выигрыш составляет один доллар. Если выпадает решка, потеря составляет один доллар. Чему равно математическое ожидание выигрыша? Является ли игра справедливой? 11 Замечание: вследствие ошибки округления сумма = 1,001 ~ 1.
648 Часть II. Конечная математика 6. Повторите задачу 5, предполагая, что подбрасывается неидеальная монета, для которой вероятность выпадения орла равна 0,55, а решки 0,45. Б 7. Поставив на кон четыре доллара, игрок бросает одну идеальную игральную кость, получает количество долларов, соответствующее выпавшему числу. Например, если выпадают пять очков, игроку выплачивают пять долларов, т.е. его чистый доход составляет один доллар. Если выпадает единица, убыток составляет три доллара и т.д. Чему равно математическое ожидание выигрыша? Является ли игра справедливой? 8. Повторите задачу 7, если ставка равна 3,50 долл. 9. Подбрасываются две монеты. Если выпадают два орла или две решки, выигрыш составляет два доллара; если выпадает орел и решка, потеря составляет три доллара. Чему равно математическое ожидание игры? 10. Сколько должна составлять потеря в задаче 9, если выпадет орел и решка, чтобы игра была справедливой? 11. Представьте себе, что ваш друг предлагает вам такую игру: если при четырех подбрасываниях игральной кости шестерка выпадет по крайней мере один раз, он получает от вас один доллар, в противном случае он сам выплачивает вам один доллар. Чему равно математическое ожидание выигрышей в этой игре? 12. При трех подбрасываниях игральной кости вы потеряете 10 долл., если пятерка выпадет по крайней мере один раз, а в противном случае выиграете семь долларов. Чему равно математическое ожидание выигрыша? * 13. Игральная кость бросается один раз. Выигрыш составляет пять долларов, если выпадает единица или двойка, и 10 долл., если выпадает тройка, четверка или пятерка. Сколько должен составлять проигрыш, если выпадет шестерка, чтобы игра была справедливой? Опишите шаги, которые вы предпринимаете, чтобы получить ответ. * 14. Игральная кость бросается один раз. Проигрыш равен 12 долл., если выпадает число, кратное трем. Сколько должен составлять выигрыш, если не выпадает число, кратное трем, чтобы игра была справедливой? Опишите процесс и обоснование ваших рассуждений. 15. Один раз бросается пара игральных костей. Предположим, что вы теряете 10 долл., если выпадает сумма очков, равная семи, и выигрываете 11 долл., если выпавшая сумма очков равна 11 или 12. Сколько вы должны выиграть или проиграть, если выпадает любая другая сумма, чтобы игра была справедливой? 16. Монета бросается трижды. Предположим, что вы теряете три доллара, если выпадают три орла, теряете два доллара, если выпадают два орла, и выигрываете три доллара, если ни один орел не выпадает. Сколько вы должны выиграть или проиграть, если выпадает один орел, чтобы игра была справедливой? 17. Из стандартной колоды, содержащей 52 карты, извлекается карта. Если карта оказывается королем, вы выигрываете 10 долл.; в противном случае вы теряете один доллар. Чему равно математическое ожидание выигрыша?
Глава 6. Теория вероятностей 649 18. Из стандартной колоды, содержащей 52 карты, извлекается карта. Если карта имеет бубновую масть, вы выигрываете 10 долл.; в противном случае вы теряете четыре долл. Чему равно математическое ожидание выигрыша? 19. Из стандартной колоды, содержащей 52 карты, извлекается пятикарточный расклад. Если расклад содержит по крайней мере одного короля, вы выигрываете 10 долл.; в противном случае вы теряете один доллар. Чему равно математическое ожидание выигрыша? 20. Из стандартной колоды, содержащей 52 карты, извлекается пятикарточный расклад. Если расклад содержит по крайней мере одну бубну, вы выигрываете 10 долл.; в противном случае вы теряете четыре доллара. Чему равно математическое ожидание выигрыша? 21. Ниже приведена таблица выигрышей для двух курсов акций, Ai или А2. Какая из двух акций даст наибольшее математическое ожидание прибыли? Чему оно равно? Pi Xi (Al), ДОЛЛ. Xi (Аг), долл. 0,1 -200 -100 0,2 100 200 0,4 400 300 0,3 100 200 22. Ниже приведена таблица выигрышей для трех возможных курсов акций. Какая из трех акций даст наибольшее математическое ожидание прибыли? Чему оно равно? Pi (Al), ДОЛЛ. Xi (A2), долл. xi (A3), долл. 0,2 500 400 300 0,4 1200 1100 1000 0,3 1200 1800 1700 0,1 1200 1800 2400 23. Колеса рулеток в игровых заведениях Невады обычно имеют 38 равномерно расположенных впадин, пронумерованных числами 00, 0, 1, 2, ..., 36. Игрок, поставивший один доллар на любое из указанных чисел, выигрывает 35 долл, (и получает ставку обратно), если шарик останавливается на выбранном числе; в противном случае ставка теряется. Чему равно математическое ожидание выигрыша? 24. Сектора рулетки, пронумерованные числами от 1 до 36, (см. задачу 23) поровну разделены на красные и черные. Игрок, поставивший один доллар на черное, выигрывает один доллар (и возвращает ставку), если шарик останавливается на черном; в противном случае (если шарик останавливается на красном, 0 или 00), ставка теряется. Чему равно математическое ожидание выигрыша? * 25. Математическое ожидание вашего выигрыша составляет 100 долл. Ставка равна 100 долл., но если вы выигрываете, то получаете 100000 долл, (включая ставку 100 долл.), чистый доход составляет 99900 долл. Чему равна вероятность вы-
650 Часть II. Конечная математика играть? Стоит ли вам принимать участие в такой игре? Рассмотрите факторы, влияющие на ваше решение. *26. Математическое ожидание вашего выигрыша равно —0,50 долл. Ставка равна два доллара, но в случае выигрыша вы получаете 20 долл, (включая ставку). Следовательно, чистый доход составляет 18 долл. Чему равна вероятность выигрыша? Стоит ли вам принимать участие в такой игре? Обсудите факторы, которые будут влиять на ваше решение. В 27. Пять тысяч билетов благотворительной лотереи продаются по одному доллару за каждый. Билеты извлекаются из урны случайно, а денежные выигрыши назначаются следующим образом: один выигрыш на сумму 500 долл., три выигрыша по 100 долл., пять выигрышей по 20 долл, и 20 выигрышей по 5 долл. Чему равно математическое ожидание выигрыша в этой лотерее, если вы купите один билет? 28. Десять тысяч лотерейных билетов продаются по 2 долл, за каждый. Выигрыши назначаются следующим образом: два выигрыша по 1000 долл., четыре выигрыша по 500 долл, и 10 выигрышей по 100 долл. Чему равно математическое ожидание выигрыша в этой лотерее, если вы купите один билет? 29. В ящике, содержащем 10 электрических ламп, есть три бракованные лампы. Для проверки из ящика извлекается случайная выборка, состоящая из двух ламп. Пусть X — случайная величина, представляющая собой количество бракованных ламп в выборке. а) Найдите распределение вероятностей случайной величины X. б) Найдите математическое ожидание количества бракованных ламп в выборке. 30. В ящике, содержащем восемь электрических ламп, есть три бракованных. Для проверки из ящика извлекается случайная выборка, состоящая из двух ламп. Пусть X — случайная величина, представляющая собой количество. а) Найдите распределение вероятностей случайной величины X, б) Найдите математическое ожидание количества бракованных ламп в выборке. 31. Одна тысяча лотерейных билетов продается по одному доллару за каждый. Из урны случайным образом извлекаются (без возвращения) три билета, причем за каждый назначена плата, равная 200 долл. Предположим, что вы покупаете 5 билетов. а) Составьте таблицу платежей для 0, 1, 2 и 3 выигрышных билетов из пяти купленных. (Если вы не имеете ни одного выигрышного билета, вы теряете пять долларов; если у вас есть один выигрышный билет, ваш чистый доход составляет 195 долл., поскольку ваши начальные пять долларов не будут вам возвращены, и т.д.) б) Чему равно математическое ожидание вашего выигрыша? 32. Повторите решение задачи 31, если покупаются 10 билетов. 33. Для моделирования игры в рулетку с помощью графической утилиты выбирается случайное число из диапазона от —1 до 36 (—1 представляет 00; см. задачу 23). Команда, представленная на рис. 6.36, а, моделирует 200 игр.
Глава 6. Теория вероятностей randInt(-1,36,20 0)->Li <22 33 20 2 35 ... 651 а) Рис. 6.36. Решение задачи с помощью графической утилиты а) Используя гистограмму, изображенную на рис. 6.36, б, определите чистый выигрыш или проигрыш при ставке, равной одному доллару, на число 13 в каждой из 200 игр. б) Сравните результаты решения п. а с математическим ожиданием выигрыша. в) Примените графическую утилиту для моделирования игры в рулетку при ставке, равной одному доллару, на число 7 в каждой из 500 игр и сравните смоделированные выигрыши или потери с математическим ожиданием выигрышей и потерь. 34. Примените графическую утилиту для моделирования игры в рулетку при ставке, равной одному доллару, на черное в каждой из 400 игр (см. задачи 24 и 33). Сравните результаты моделирования и математическое ожидание выигрышей или потерь. Применение математики Экономика и бизнес 35. WWW Страхование. Ежегодный взнос по страховке картин от кражи составляет 150 долл. Стоимость страховки равна 5000 долл. Если эмпирическая вероятность того, что картина будет украдена в течение года равна 0,01, чему равно математическое ожидание страхового возмещения, если вы купите страховой полис? 36. Страхование. Повторите решение задачи 35 и определите математическое ожидание прибыли страховой компании. 37. Анализ принятия решений. После тщательной проверки и анализа компания рассматривает возможность бурения скважин в двух различных местах. По оценкам специалистов, в случае удачи (вероятность этого события равна 0,2) скважина А принесет 30 млн. долл, дохода, а в случае неудачи (вероятность этого события равна 0,8) потери составят 3 млн. долл. Аналогично в случае удачи (вероятность этого события равна 0,1) скважина В принесет 70 млн. долл, дохода, а в случае неудачи (вероятность этого события равна 0,9) потери составят четыре мил-
652 Часть II. Конечная математика лиона долларов. Какой из участков должна выбрать компания в соответствии с математическим ожиданием прибыли от каждого из них? 38. Анализ принятия решения. Повторите решение задачи 37, предполагая, что дополнительный анализ изменил вероятность успеха на участке В с 0,1 до 0,11. Биологические науки 39. Генетика. Предположим, что рождение девочки и рождение мальчика в каждом случае не являются равновероятными. Распределение вероятностей количества мальчиков в семье, имеющей трех детей, приведено в таблице. Чему равно математическое ожидание количества мальчиков в семье, имеющей трех детей? Количество мальчиков, xi pi б бдТ" 1 0,36 2 0,38 3 0,14 40. WWW Генетика. Растение с розовыми цветами имеет генотип RW. Если скрестить два таких растения, то с вероятностью, равной 0,25, получим растение с красными цветами (RR), с вероятностью, равной 0,50, — растение с розовыми цветами (RW или WR) и с вероятностью, равной 0,25, — растение с белыми цветами (WW). Чему равна вероятность того, что в потомстве растений этого типа присутствует ген W? Количество генов, W Xi pi б б^5~ 1 0,50 2 0,25 Социальные науки 41. Политика. Штаб кандидата, баллотирующегося в органы государственной власти, организует сбор денежных пожертвований. Рассматриваются два способа агитации.
Глава 6. Теория вероятностей 653 -41: отправка почтового уведомления с последующей рассылкой, : опрос на дому с последующими телефонными беседами. В таблице приведены средний размер пожертвований и вероятность их получения в ходе агитации, полученные в ходе предыдущей предвыборной кампании. At а2 Х{, долл./чел. Pi Xi, долл./чел. Pi 10 0,3 15 0,3 5 0,2 3 0,1 0 0,5 0 0,6 1,0 1,0 Чему равны математические ожидания пожертвований? Какому из подходов следует отдать предпочтение, руководствуясь математическими ожиданиями пожертвований? Ключевые слова, основные обозначения и формулы 6.1. Основные принципы счета. Техника счета; количество элементов во множестве; правило сложения множеств; древовидная диаграмма; правило умножения множеств. п(А); п(Л U В) = п(А) + п(В) — п(Л А В); п(А U В) = п(А) + п(В), если п(А Г\ В) = 0. 6.2. Перестановки и сочетания, n-факториал; нуль-факториал; перестановка; перестановка п предметов; размещение из п предметов по г; сочетания; сочетания из п предметов по г. п! = п(п - 1)(п — 2) • ... • 2 • 1; 0! = 1; Tl\ Рп,п = п!; Рп г — ~; 0 г п. (п — г)! 6.3. Пространство элементарных событий, события и вероятность. Случайный эксперимент; эксперимент; пространство элементарных исходов; событие; элементарный исход; элементарное событие; составное событие; фундаментальное пространство элементарных исходов; вероятность события; приемлемое распределение вероятностей; обоснованное распределение вероятностей; функция вероятности; частота; относительная частота; эмпирическая вероятность; приближенная относительная вероятность; предположение о равновозможности. Р(Е); Р(Е) = п(Е). п
654 Часть II. Конечная математика 6.4. Объединение, пересечение и дополнение событий; шансы. Объединение; пересечение; событие А или событие В; событие А и событие В; взаимно исключающие; непересекающиеся; дополнение события; шансы; справедливая игра; закон больших чисел. A U В; АП В; Р(А U В) = Р(А) + В(В) - Р(А П В); Р(А U В) = Р(А) + Р(В), если А П В = 0; А'; Р(А') = 1 - Р(А); (шанс В). 6.5. Условная вероятность, пересечение и независимость. Условная вероятность события А при условии события В; правило умножения; случайный процесс; независимые события; зависимые события. Р (А | В) = Р(р 2 В), когда Р(В) / 0; В (В) Р(А А В) = Р(А)Р(В | А); Р(А А В) = Р(А)Р(В) тогда и только тогда, когда Аи В независимы; Р(А | В) = Р{А) и Р(В | А) = Р(В), если АиВ независимы и Р(А) / 0, Р(В) / 0; P(Ei А Е2 А ... А Еп) = РСВ^Р^)..... Р(ВП), если Ei,E2, ... ,Еп независимы. 6.6. Формула Байеса. р (и , р\ = Р(Д | l/1)P(t/1) = V11 ’ Р(В|В1)Р(В1) + Р(В|В2)Р(172) + ----ЬР(В|С/п)Р(1/п) произведение вероятностей ветвей, _ ведущих к вершине Е через вершину U\ сумма всех произведений вероятностей ветвей, ведущих к вершине Е 6.7. Случайная величина, распределение вероятностей и математическое ожидание. Случайная величина; распределение вероятностей случайной величины X; гистограмма; математическое ожидание случайной величины; таблица выигрышей; справедливая игра. Р(Х) = Х1Р1 -I- Х2Р2 + • • • + Хпрп.
Глава 6. Теория вероятностей 655 Упражнения для повторения Выполните все упражнения этого обзорного раздела и сравните результаты с ответами, помещенными в конце книги. Ответы ко многим упражнениям на повторение приводятся вместе с номером соответствующего раздела (курсивом). Если у вас возникают затруднения при решении какой-либо задачи, повторите материал соответствующего раздела. А 1. Подбрасывается одна игральная кость и монета. Сколько существует всех возможных составных исходов? Решите эту задачу при следующих условиях. а) Используется древовидная диаграмма. б) Используется правило умножения. 2. Используйте диаграмму Венна для определения количества элементов в каждом из следующих множеств: а) А. в) А П В. д) U. ж) (А А В)'. е) А'. з) (A U В)'. 3. Вычислите величины Сб,2 и Р6?2. 4. Сколько существует способов рассадить шесть человек на шести стульях в ряд? Решите задачу, используя правило умножения. 5. Решите задачу 4, используя перестановки и сочетания. 6. Какова вероятность того, что в одном раскладе из пяти карт, полученном из стандартной колоды, состоящей из 52 карт, все карты — трефовой масти? 7. Бетти и Билл являются членами лыжного клуба, который состоит из 15 человек. Для того чтобы выбрать президента и бухгалтера клуба, проводится жеребьевка. Чему равна вероятность того, что Бетти будет президентом клуба, а Билл бухгалтером? (Человек не может занимать более одной должности.) 8. На отдельных карточках напечатаны десять первых букв английского алфавита. Чему равна вероятность вытащить три карточки и получить кодовое слово dig, вытащив букву d при первой попытке, г — при второй и g — при третьей? Чему равна вероятность вытащить трехкарточный расклад, содержащий буквы d, i, g в любом порядке?
656 Часть II. Конечная математика 9. Лекарство имеет побочные эффекты для 50 людей из 1000 принимающих его. Чему равна приблизительная эмпирическая вероятность того, что у человека, использующего лекарство, возникнут побочные эффекты? 10. Секторы рулетки пронумерованы пятью цифрами 1, 2, 3, 4 и 5, выпадание каждой из которых равновероятно. Ставка в игре составляет три доллара, а вероятный выигрыш равен долларовому эквиваленту числа, на котором останавливается стрелка рулетки. Чему равно математическое ожидание выигрыша? Является ли игра справедливой? 11. Пусть А и В — случайные события в пространстве элементарных исходов S, Р(А) = 0,3; Р(В) = 0,4 и Р(А П В) = 0,1. Вычислите следующие величины. а) Р(А'). б) P(AUB). 12. Стрелка рулетки останавливается на секторе R с вероятностью, равной 0,3, на секторе G — с вероятностью, равной 0,5, и на секторе В — с вероятностью, равной 0,2. Найдите вероятность и шанс того, что стрелка остановится на секторе R либо G. 13. Если при повторяющихся подбрасываниях пары игральных костей шанс на выпадение суммы, равной восьми, перед выпадением суммы, равной семи, равен пять к шести, то чему равна вероятность выпадения суммы, равной восьми, перед выпадением суммы, равной семи? Решите задачи 14-22, используя следующую таблицу вероятностей. X Y Z Всего S 0,10 0,25 0,15 0,50 Т 0,05 0,20 0,02 0,27 R 0,05 0,15 0,03 0,23 Всего 0,20 0,60 0,20 1,00 14. Вычислите вероятность Р(Т). 15. Вычислите вероятность P(Z). 16. Вычислите вероятность Р(Т П Z). 17. Вычислите вероятность P(R П Z). 18. Вычислите вероятность P(R | Z). 19. Вычислите вероятность P(Z | R). 20. Вычислите вероятность Р(Т | Z). 21. Являются ли события Т и Z независимыми? 22. Являются ли события S и X независимыми? При решении задач 23-30 необходимо вычислить указанные величины, используя следующее вероятностное дерево. Начало
Глава 6. Теория вероятностей 657 23. Р(Л). 24. Р(В | А). 25. Р(В | А'). 26. Р(ЛпВ). 27. Р(А'ПВ). 28. Р(В). 29. Р(А | В). 30. Р(А | В'). 31. Ответьте на следующие вопросы. а) Если 10 из 32 студентов в классе родились в июне, июле или августе, чему равна приблизительная эмпирическая вероятность того, что любой студент из класса родился в июне, июле или августе? б) Если рождение каждого студента равновероятно в любом из 12 месяцев, чему равна теоретическая вероятность рождения в июне, июле или августе? *в) Обсудите различие между ответами к пп. а и б. В задачах 32 иЗЗ обсудите справедливость каждого из утверждений. Если утверждение всегда справедливо, объясните, почему. Если нет, приведите контрпример. * 32. Справедливы ли следующие утверждения? а) Если А и В являются пустыми множествами, то А и В — взаимно исключающие события. б) Если Аи В являются взаимно исключающими событиями, то Р(АПВ) = Р(Л)Р(В). * 33. Справедливы ли следующие утверждения? а) Если А и В — независимые события, то Р(АиВ) = Р(А) + Р(В). б) События А и А' независимы. Б 34. Игрок подбрасывает две монеты и получает пять долларов, если выпадают два орла, теряет четыре доллара, если выпадает одна решка, и выигрывает два доллара, если орел не выпадает ни разу. (Стоит ли играть в эту игру?) Вычислите математическое ожидание выигрыша. Является ли игра справедливой? 35. Секторы рулетки пронумерованы цифрами 1, 2 и 3, появление каждой из которых равновероятно. Если рулетка вращается дважды, чему равна вероятность следующих событий? а) Дважды выпадает одно и то же число. б) Выпадают числа, сумма которых равна 5. 36. Из стандартной колоды, содержащей 52 карты, извлекается одна карта. Чему равны вероятности и шансы следующих событий? а) Выбран валет или дама. б) Выбран валет или карта пиковой масти. в) Выбранная карта не является тузом.
658 Часть II. Конечная математика 37. Ответьте на следующие вопросы. а) Каков шанс выпадения суммы, равной пяти, при одном подбрасывании пары игральных костей? б) Если ваша ставка на то, что выпадет сумма, равная пяти, составляет один доллар, сколько должен заплатить банк (с учетом возврата ставки), чтобы игра была справедливой? 38. Две монеты подбрасываются 1000 раз со следующими частотами. Два орла 210 Один орел 480 Ни одного орла 310 а) Вычислите эмпирическую вероятность каждого исхода. б) Вычислите теоретическую вероятность каждого исхода. в) Используя теоретические вероятности, полученные при решении п. б, вычислите математическое ожидание частоты каждого исхода, предполагая, что монеты являются идеальными. * 39. У человека пять детей. Каждый из этих детей имеет троих детей, которые в свою очередь имеют по двое детей. Определите, сколько потомков имеет человек. *40. Идеальная монета подбрасывается 10 раз. При каждом из первых девяти подбрасываний исходами являются орлы. Чему равна вероятность выпадения орла при десятом подбрасывании? 41. Эксперимент состоит в подбрасывании пары идеальных игральных костей. Пусть X — случайная величина, связанная с суммой выпавших значений. а) Найдите распределение вероятностей случайной величины X, б) Вычислите математическое ожидание случайной величины X. 42. Эксперимент состоит в подбрасывании пары игральных костей. В качестве пространства элементарных исходов выбрано множество упорядоченных пар целых чисел из множества {1,2,3,4,5,6}. Что собой представляет множество А, соответствующее выпавшей сумме, кратной четырем? Что собой представляет множество, соответствующее выпавшей сумме, кратной шести? Чему равны Р(Л), Р(В), Р(А А В) и Р(А U В)? 43. Вам сказали, что элементарные исходы {61,62,63,64} имеют следующие эмпирические вероятности: P(ei) « 0,1, Р(в2) « —0,2, Р(ез) « 0,6, Р(вд) « 2. Существует три причины, по которым Р не может быть распределением вероятностей. Назовите их. 44. Используйте следующую информацию для составления таблицы частот, приведенной ниже: п(А) = 50, п(В) = 45, n(AUB) = 80, п(Р) = 100.
Глава 6. Теория вероятностей 659 А А1 Всего “В В' Всего 45. Стрелка вращается над диском с пронумерованными секторами. Вероятности того, что стрелка остановится в секторах от 1 до 5, приведены в следующей таблице. е< 1 2 3 4 5 Pi 0,2 0,1 0,3 0,3 0,1 а) Чему равна вероятность того, что стрелка остановится на нечетном числе? б) Чему равна вероятность того, что стрелка остановится на числе, которое меньше четырех, при условии, что она остановится на нечетном числе? 46. Из стандартной колоды, содержащей 52 карты, случайным образом извлекается одна карта. Обозначим через Е событие “извлечена красная карта” а через F — событие “извлечен туз”. а) Вычислите вероятность P(F | Е). б) Проверьте независимость событий Е и F. 47. Сколько трехбуквенных кодовых слов можно составить из первых восьми букв алфавита, если буквы не могут повторяться? Если буквы могут повторяться? Если соседние буквы не могут быть одинаковыми? 48. Решите следующие задачи, используя определения величин РП)Г или Сп>г. а) Сколько трехразрядных открывающих комбинаций можно составить для секретного замка с шестью разрядами, если цифры не могут повторяться? б) Пять теннисистов вышли в финал. Сколько должно быть объявлено игр, если все игроки должны сыграть друг с другом один раз? 49. Используя графическую утилиту, найдите наибольшее значение сочетаний Сп,г> если п = 25. В задачах 50-54 урна U\ содержит два белых шара и три красных шара, а урна U? содержит два белых шара и один красный шар. 50. Из урны U\ последовательно извлекаются два шара. Чему равна вероятность вытянуть белый шар, за которым следует красный, при следующих условиях? а) Первый шар извлекается с возвращением. б) Первый шар извлекается без возвращения. 51. В каком из двух пунктов задачи 50 речь идет о зависимых событиях? 52. Чему равно математическое ожидание количества красных шаров в задаче 50 при следующих условиях? а) Первый шар извлекается с возвращением. б) Первый шар извлекается без возвращения. 53. Урна выбирается случайным образом с помощью подбрасывания идеальной монеты, затем из нее извлекается шар. Вычислите следующие величины. а) Р(Я|С/1). 6) P{R\Ui).
660 Часть II. Конечная математика в) Р(Я). г) P(U, I R). Д) P(U2 | W). е) Р^АЯ). 54. Являются ли события “выбрана урна и “извлечен красный шар” в задаче 53 независимыми? 55. Чему равна вероятность следующих событий в пятикарточном раскладе, извлеченном из стандартной колоды в 52 карты? а) Все карты в раскладе имеют бубновую масть. б) В раскладе три карты имеют бубновую масть и две — пиковую. Запишите ответы в терминах сочетаний Сп,г или размещений РП)Г, не вычисляя результат. 56. В группу из 10 человек входит одна супружеская пара. Случайным образом выбираются четыре человека. Чему равна вероятность того, что будет выбрана супружеская пара? 57. Если три операции О2, Оз производятся последовательно с возможным количеством исходов Ni, N2, N3 соответственно, определите количество ветвей в соответствующей древовидной диаграмме. * 58. Из стандартной колоды извлекается пятикарточный расклад. Являются ли зависимыми два следующих события? S — расклад состоит полностью из карт пиковой масти, Н — расклад состоит полностью из карт червонной масти. 59. С помощью команды графической утилиты, показанной на рис. 6.37, а, смоделировано 50 повторяющихся подбрасываний пары игральных костей, после каждого из которых записалось минимальное из двух выпавших чисел. Гистограмма результатов изображена на рис. 6.37, б. min<randlnt(Ь6? 50>>randlnt<l>6> 50))4Ы <13 113 12... б) а) Рис. 6.37. Решение задачи с помощью графической утилиты а) Используйте рис. 6.37, б ддя определения эмпирической вероятности того, что минимальное число равно двум. б) Чему равна теоретическая вероятность того, что минимальное число равно двум? в) Примените графическую утилиту для моделирования 200 подбрасываний пары игральных костей, определите эмпирическую вероятность того, что минимальное число равно четырем и сравните ее с теоретической вероятностью.
Глава 6. Теория вероятностей 661 60. Из стандартной колоды, содержащей 52 карты, случайным образом извлекается карта. Используя графическую утилиту, смоделируйте 800 таких испытаний и определите вероятность того, что карта является черным валетом и сравните ее с теоретической вероятностью. В 61. Три идеальные монеты подбрасываются 1000 раз. В результате регистрируются следующие частоты исходов. Количество орлов 0 12 3 Частота 120 360 350 17(Г~ а) Чему равна приблизительная эмпирическая вероятность выпадения двух орлов? б) Чему равна теоретическая вероятность выпадения двух орлов? в) Чему равно математическое ожидание частоты выпадения двух орлов? 62. Вы заключили пари с другом на один доллар, что получите одну или несколько пар шестерок при 24 подбрасываниях двух идеальных игральных костей. Чему равно математическое ожидание вашего выигрыша? Чему равно математическое ожидание выигрыша вашего друга? Является ли игра справедливой? 63. Отделение, занимающееся разработкой программного обеспечения, состоит из шести женщин и четырех мужчин. а) Сколькими способами они могут выбрать главного программиста, программиста систем резервного копирования и программиста библиотек? б) Чему равна вероятность того, что на все три должности будут избраны женщины, если должности в предыдущей задаче выбираются с помощью жеребьевки? в) Сколькими способами они могут выбрать команду из трех программистов для работы над определенным проектом? г) Предположим, что выбор осуществляется жеребьевкой. Чему равна вероятность того, что большинство в команде будут составлять женщины? 64. В группу из 150 человек входят 52 играющих в шахматы, 93 играющих в шашки и 28 играющих и в шахматы, и в шашки. Сколько человек в группе не играют ни в одну из игр? 65. Если три человека выбраны из группы, состоящей из семи мужчин и трех женщин, чему равна вероятность того, что среди выбранных людей выбрана по крайней мере одна женщина? Из стандартной колоды, содержащей 52 карты, последовательно без возвращения извлекаются две карты. В задачах 66 и 67 определите указанные вероятности. 66. Вторая карта имеет червонную масть при условии, что первая карта имеет ту же масть. 67. Первая карта имеет червонную масть при условии, что вторая карта имеет ту же масть.
662 Часть II. Конечная математика 68. Две идеальные игральные кости пронумерованы следующим образом: на одной стороне — три очка, на двух — два, а на остальных трех — по одному. После подбрасывания костей цифры на выпавших сторонах суммируются. Пусть X — случайная величина, связанная с пространством элементарных исходов S — = {2,3,4,5,6}. а) Вычислите распределение вероятностей случайной величины X. б) Вычислите математическое ожидание случайной величины X. 69. Предположим, что в задаче 68 ставка равна 3,50 долл, (кость бросается один раз) и вам возвращается долларовый эквивалент, соответствующий выпавшей сумме. Чему равно математическое ожидание выигрыша? Является ли игра справедливой? Если она не является справедливой, сколько следует платить, чтобы сделать ее справедливой? 70. Сколько существует различных семей, имеющих пять детей, пол которых учитывается в порядке рождения (т.е. последовательностей (М, Д, Д, М, М), (Д, М, Д, М, М) и т. д.)? Сколько будет таких семей, если порядок рождения не принимается во внимание? 71. Предположим, что в урну положены три белых шара и один — черный. Шары вынимаются последовательно без возвращения до тех пор, пока не будет вынут черный шар. Затем игра прекращается. Вы выигрываете, если черный шар вытаскивается на четвертом шаге. а) Чему равны вероятность и шанс на выигрыш? б) Если вы ставите один доллар, сколько крупье должен выплатить за выигрыш (с учетом возврата ставки), чтобы игра была справедливой? 72. Если каждого из пяти человек просят указать любимую книгу из списка 10 бестселлеров, чему равна вероятность того, что по крайней мере двое из них назовут одну и ту же книгу? * 73. Может ли набор, состоящий из г предметов, выбранных из множества, включа¬ ющего п предметов, где п — положительное целое число, быть одновременно сочетанием и размещением? Объясните. * 74. Пусть А и В — события с ненулевыми вероятностями в пространстве элемен¬ тарных событий S. При каких условиях Р(А | В) совпадает с Р(В | Л)? Применение математики Экономика и бизнес 75. Транспортировка, Продукция, хранящаяся на оптовой базе А, поставляется в пять розничных магазинов: В, С, D, Е и F. Сколько различных планов перевозки можно составить так, чтобы, один автомобиль, выехавший из А, посетил каждый магазин ровно один раз и затем вернулся на базу? 76. Исследование рынка. В результате опроса 1000 человек выяснилось, что средства 340 из них вложены в акции, 480 — в облигации и 210 — и в акции, и в облигации, а) Сколько респондентов владеют акциями и облигациями?
Глава 6. Теория вероятностей 663 б) Сколько респондентов не владеют ни акциями, ни облигациями? в) Сколько респондентов владеют облигациями, но не акциями? 77. Исследование рынка. В результате опроса 100 жителей города было обнаружено, что 40 из них читают ежедневную утреннюю газету, 70 — ежедневную вечернюю газету и 30 — читают обе газеты. Чему равна (эмпирическая) вероятность следующих событий? а) Случайно выбранный житель города читает ежедневную газету. б) Случайно выбранный житель города не читает ежедневные газеты. в) Случайно выбранный житель города читает одну ежедневную газету. 78. Исследование рынка. Исследование рынка рекламы показало, выяснила, что 40% жителей некого региона знакомы с рекламой новой продукции и 85% из них приобрели ее. Чему равна вероятность того, что отдельно рассматриваемый житель этого региона рекламу видел и приобрел новую продукцию? 79. Исследование рынка. Компания по пошиву одежды провела опрос среди 1000 случайно выбранных респондентов, чтобы определить взаимосвязь между возрастом покупателя и ежегодно приобретаемых купленных джинсов. Результаты приведены в следующей таблице. Возрастная группа Количество ежегодно приобретаемых джинсов Всего, чел. 0 1 2 Больше 2 Младше 12 лет, чел 60 70 30 10 170 12-18 лет, чел 30 100 100 60 290 19-25 лет, чел 70 110 120 30 330 Старше 25 лет, чел 100 50 40 20 210 Всего, чел 260 330 290 120 1000 Введем следующие обозначения. А — респондент покупает 2 пары джинсов. В — возраст респондента от 12 до 18 лет. С — респондент не покупает более трех пар джинсов. D — респондент покупает более двух пар джинсов. а) Вычислите вероятности Р(А), Р(В), Р(А П В), Р(А | В), Р(В | Л). *б) Являются ли события АиВ независимыми? Объясните. в) Вычислите вероятности Р(С), Р(В), Р(С А В), Р(С | В), Р(Р | С). *г) Являются ли события С и D взаимно исключающими или независимыми? Аргументируйте свой ответ. 80. Анализ принятия решения. Менеджер торговой компании в результате проведенного анализа представил два плана повышения объемов продаж. По его оценкам, в случае успешной реализации (с вероятностью, равной 0,8) план А принесет 10 млн. долл, чистого дохода, а в случае неудачи (с вероятностью, равной 0,2) — 2 млн. долл, убытка. Кроме того, менеджер считает, что план Б в случае успешной реализации (с вероятностью, равной 0,7) принесет 12 млн. долл, чистого дохода, а в случае неудачи (с вероятностью, равной 0,3) — 2 млн. долл, убытка.
664 Часть II. Конечная математика Чему равно математическое ожидание прибыли для каждого плана? Какой из планов следует выбрать, исходя из величины математического ожидания? 81. Страхование. Мотоцикл застрахован от кражи на 300 долл. Ежегодный страховой взнос равен 30 долл. Предположим, вероятность того, что мотоцикл будет украден в течение года (установленная эмпирическим путем), равна 0,08. Чему равно математическое ожидание страхового возмещения? 82. Контроль качества. На сборочный завод поступают двенадцать обработанных с высокой точностью деталей, включая две не отвечающих промышленным стандартам. Отдел контроля качества случайным образом выбирает четыре детали и бракует всю партию, если среди них окажется хотя бы одна нестандартная. Чему равна вероятность того, что партия будет забракована? 83. Контроль качества. В сервисный центр поступила дюжина компьютерных плат, включая две бракованные. Для проверки всей партии случайным образом выбираются три платы. Пусть X — случайная величина, указывающая количество бракованных плат в выборке. а) Найдите распределение вероятностей случайной величины X. б) Вычислите математическое ожидание количества бракованных плат в выборке. Биологические науки 84. WWW Медицина: кардиологические исследования. В результате медицинского обследования большого количества людей было выяснено, что у 82% населения нет никаких проблем с сердцем, у 11% есть небольшие отклонения и у 7% — серьезные сердечные заболевания. Кардиограмму делают 95% людей, имеющих нормально работающее сердце, 30% людей с незначительными отклонениями и 5% — с серьезными сердечными заболеваниями. Чему равна вероятность того, что у человека, сделавшего кардиограмму, нормально работает сердце? 85. Генетика. Шесть мужчин из 100 и одна женщина из 100 — дальтоники. Случайным образом выбирается человек, у которого диагностирован дальтонизм. Чему равна вероятность того, что это — мужчина? (Считается, что в исследуемой группе одинаковое количество мужчин и женщин.) Социальные науки 86. WWW Предпочтения избирателей. В предварительном голосовании 30 студентов математического факультета определяли президента студенческого общества. Предварительное голосование позволило вычислить следующие эмпирические вероятности: кандидат А должен получить 53% голосов, кандидат В — 37%, а кандидат С — 10%. Спустя неделю кандидат В выиграл выборы. Обсудите факторы, объясняющие разницу между результатами предварительного голосования и реальных выборов. Домашнее задание 6.1. Победа в телешоу Ведущий телевизионного игрового шоу предоставляет вам (игроку) возможность выбрать одну из трех закрытых дверей, обозначенных А, В и С. За одной из дверей находится автомобиль, а за двумя другими — тыквы. После того как вы выбрали одну из
Глава 6. Теория вероятностей 665 дверей (остается закрытой), ведущий открывает одну из двух дверей, не выбранных вами. Ведущий знает, за какой дверью находится автомобиль, и всегда выбирает дверь, за которой находится тыква. Затем он дает вам шанс придерживаться вашего первоначального выбора или переключиться на другую неоткрытую дверь. Какую из следующих двух стратегий вы бы выбрали и почему? Автомобиль Стратегия 1. Придерживаться первоначального выбора. Стратегия 2. Переключиться на другую неоткрытую дверь. 1. “Здравый смысл”. Попросите студентов группы сделать выбор в пользу одной из стратегий и обосновать его. Предложите задачу друзьям и спросите, какую стратегию они бы предпочли и почему. Опишите результаты проведенных обсуждений. 2. Моделирование. Для моделирования этой задачи используйте подобие рулетки с тремя одинаковыми по площади секторами, раскрашенными в разные цвета, и предположите, что автомобиль спрятан за дверью С. Для первоначального выбора двери нужно раскрутить стрелку и дождаться, пока она не остановиться в одном из секторов. {Замечание. Вместо рулетки можно использовать игральную кость: выбирается дверь А, если выпадают одно или два очка, дверь В, если выпадает три или четыре очка, и дверь С, если выпадают пять или шесть очков.) Далее описана стратегия следования первоначальному выбору. Смоделировать стратегию переключения на другую неоткрытую дверь вам предстоит самостоятельно. Стратегия 1. Придерживаться первоначального выбора 1) Предположим, что стрелка указывает на сектор (дверь) А. Ведущий открывает дверь В, зная, что автомобиль находится за дверью С, и обнаруживает тыкву. Придерживаясь первоначального выбора, вы проигрываете, получая тыкву. 2) Предположим, что стрелка указывает на сектор (дверь) В. Ведущий открывает дверь А, зная, что автомобиль находится за дверью С, и обнаруживает тыкву. Вы настаиваете на выборе двери В и проигрываете, получая тыкву. 3) Предположим, что стрелка указывает на сектор (дверь) С. Ведущий открывает либо дверь А, либо дверь В, и обнаруживает тыкву. Вы придерживаетесь выбора двери С и выигрываете, получая автомобиль. Стратегия 2. Переключиться на другую неоткрытую дверь Модифицируйте стратегию первоначального выбора так, чтобы она обеспечивала выигрыш при переключении на другую неоткрытую дверь.
666 Часть II. Конечная математика Каждый студент в вашей группе должен проделать по крайней мере 100 подходов к моделированию каждой стратегии, записывая выигрыши и проигрыши. Соберите результаты работы всех студентов в вашей группе. Определите относительную частоту выигрышей для каждой стратегии и приблизительные эмпирические вероятности выигрыша. Теперь, как вы думаете, какой стратегии следовало бы придерживаться и почему? 3. Вероятностные деревья. Предполагается, что автомобиль находится за дверью С. Как и ранее, нарисуем вероятностное дерево только для первой стратегии. Вероятностное дерево, описывающее вторую стратегию, вам предстоит нарисовать самостоятельно. Стратегия 1. Придерживаться первоначального выбора Рассмотрим следующее вероятностное дерево и вычислим вероятность выигрыша в первой стратегии. Начало Стрелка останавливается в секторе А Стрелка останавливается в секторе В Стрелка останавливается в секторе С << Ведущий открывает 1 Игрок ставит на дверь В сектор Л и проигрывает „ Игрок ставит на Ведущий открывает сектор В и проигрывает Игрок ставит на Ведущий открывает сектор С и дверь А выигрывает Ведущий открывает Игрок ставит на дверь В сектор С и выигрывает 1 2 Стратегия 2. Переключиться на другую неоткрытую дверь Постройте вероятностное дерево для второй из предложенных выше стратегий и вычислите вероятность выигрыша. Сравните результаты, полученные в пп. 1, 2 и 3. Какую из стратегий вы бы выбрали сейчас и почему? Рассмотрите эту задачу с другом и убедите его в том, что выбранная вами стратегия является наилучшей. Теперь рассмотрим задачу о трех картах. В ящике находятся три карты. Обе стороны первой карты белые, обе стороны второй карты — черные, а у третьей карты одна сторона белая, а другая сторона — черная. Карта извлекается случайным образом и кладется на стол. Сверху оказывается белая сторона карты. Чему равна вероятность того, что другая сторона карты будет черной? Опишите, как бы вы решали эту задачу, как методом моделирования, так и теоретических вычислений. Примените оба подхода для решения задачи. Домашнее задание 6.2. Жеребьевка в профессиональных видах спорта Чему равна вероятность того, что ураган Ипатия (Hypatia) поразит земли севернее Кейп-Хатгераса (Cape Hatteras)? Или того, что новая тюрьма графства будет заполнена преступниками к концу года? Или что посетитель супермаркета выстаивает в очереди более семи минут?
Глава 6. Теория вероятностей 667 Фактически невозможным ответить на эти вопросы, приписывая вероятности элементарным исходам в едином пространстве элементарных исходов. Пространство элементарных исходов может быть настолько большим, что работать с ним будет очень сложно, а определять вероятности элементарных исходов — невозможно. Моделирование предлагает альтернативный подход. В нескольких упражнениях этой главы моделирование событий подбрасывания монеты, подбрасывания пары игральных костей, игры в рулетку и т.д. применялась графическая утилита. Мы также прибегали к ее использованию при определении приблизительных эмпирических вероятностей более сложных событий. Примером могут служить профессиональные спортивные лиги, проводящие ежегодные жеребьевки. Жеребьевка позволяет определить порядок проведения соревнований или выступления команд. Жеребьевки проводятся так, чтобы благоприятствовать выступлению слабых команд, но привносят в турнир элемент случайности. Предположим, что в турнире принимает участие 10 команд, которые не вышли в финал, назовем их А, В, С, D, Е, F, G, Н, I и J. В этом списке команды указаны в порядке возрастания квалификации. Жеребьевка команд сводится к извлечению шаров из барабана, который первоначально содержит 1000 шаров разных цветов: количество шаров, представляющих команды А-J разное — 600, 200, 100, 50, 30, 10, 4, 3, 2 и 1 соответственно. Если первый извлеченный шар представляет команду D, то именно она будет участвовать в первом матче. Прежде чем будет вынут второй шар, все остальные шары, соответствующие команде D, удаляются из барабана. Если следующий вынутый шар представляет команду F, то она будет записана второй в турнирной сетке и т.д. Извлечение шаров продолжается до тех пор, пока не будет установлен порядок проведения соревнований для всех команд, скажем, DFABCHGJEI. 1. Чему равна теоретическая вероятность того, что в жеребьевке команда А будет выбрана первой? 2. Чему равна теоретическая вероятность того, что в жеребьевке команда А будет выбрана второй? Событие извлечения первого шара из барабана можно смоделировать в графической утилите. Для этого генерируются случайные числа от 1 до 1000, и: целое число из интервала 1-600 приписывается команде А, число из интервала 601-800 — команде В, а число из интервала 801-900 — команде С и т.д. (рис. 6.38). randlnt(l?1000) 812 Рис. 6.38. Команда генерирования случайных чисел На рис. 6.38 показано, что первый вынутый шар представляет команду С. Теперь, вместо того, чтобы удалять все шары, представляющие команду С, из барабана, мы их будем попросту игнорировать в последующих испытаниях, заполнив барабан теми же 1000 шарами, что и в предыдущем испытании. Предположим, что с помощью команды
668 Часть II. Конечная математика randlnt (1, 1000, 5) было сгенерировано множество, состоящее из следующих пяти элементов: [966, 432, 656, 843, 729], которые сопоставляется с командами Е, А, В, С и В соответственно. Следовательно, указанные команды будут выступать в порядке С, Е, А, В (числа 843 и 729 не рассматриваются, поскольку представляют команды, жеребьевка которых уже проведена). Теперь представим себе, что шары, представляющие команды А, В и С, удалены из барабана. Предположим,что с помощью команды randlnt (901, 1000, 5) получено следующее множество [943, 969, 990, 928, 948], элементы которого указывают на команды D, Е, F, D и D соответственно. В итоге мы можем определить порядок участия в турнире первых шести команд: CEABDF. Вообразим, что шары, представляющие первые шесть команд турнирной сетки, удалены из барабана. Выполнив команду randlnt(991, 1000, 5), можно получить множество [993, 995, 1000, 992, 997], элементы которого сопоставляются с командами G, Я, J, G и Я. Проанализировав полученный результат можно определить порядок размещения в турнирной сетке всех 10 команд: CEABDFGHJI. 1. Проведите по крайней мере 20 жеребьевок. 2. Основываясь на результатах моделирования, определите, чему равна эмпирическая вероятность того, что команда А окажется первой или второй в списке? Сравните ее с соответствующей теоретической вероятностью. 3. Чему равна вероятность того, что команда А окажется на пятом или более дальнем месте списка? Или что в первых трех испытаниях будет извлечен шар, представляющий команду F? Либо что в одном из первых пяти испытаний будет извлечен шар, представляющий команду J? 4. Обсудите сложности вычисления теоретических вероятностей, рассмотренных в п. 3.
Цепи Маркова ■ 7.1. Свойства цепей Маркова ■ 7.2. Регулярные цепи Маркова ■ 7.3. Поглощающие цепи Маркова ■ Ключевые слова, основные обозначения и формулы ■ Упражнения для повторения ■ Домашнее задание 7.1. Социальная мобильность ■ Домашнее задание 7.2. Разорение игрока Введение В этой главе мы рассмотрим вероятностные и матричные методы анализа стохастических процессов, представляющих собой последовательность испытаний, удовлетворяющих некоторым условиям. Эти последовательности испытаний называются цепями Маркова в честь русского математика Андрея Маркова (1856-1922) — основоположника теории стохастических процессов. Ранее теория цепей Маркова применялась в основном в физике. Впоследствии область ее применения распространилась на финансовое дело, маркетинговые исследования, генетику, медицину, демографию, психологию и политические науки. В первом разделе рассматриваются общие свойства цепей Маркова, а в остальных — свойства двух разновидностей марковских цепей.
670 Часть II. Конечная математика 7.1. Свойства цепей Маркова ■ Введение ■ Матрицы переходов и матрицы состояния ■ Степени матриц переходов ■ Решение практических задач Введение В этом разделе мы сосредоточимся на анализе свойств физических систем и их возможных состояний. Для иллюстрации рассмотрим следующие примеры. 1. Во время работы Нью-Йоркской фондовой биржи акции могут расти, падать или не меняться в цене. Этот процесс можно представить как физическую систему с тремя возможными состояниями: рост, падение и постоянство цен. 2. Пассажира общественного транспорта можно рассматривать как физическую систему: он может использовать или не использовать месячный проездной билет. 3. Один из трех кандидатов (от республиканской, демократической или другой партии), победивший на выборах в избирательном округе, определяется простым большинством в результате тайного голосования. Исход голосования можно описать как физическую систему, которая может пребывать в одном и только одном из трех возможных состояний: победа кандидата республиканской, демократической партии или другой партии. Если система переходит из одного состояния в другое случайным образом, то последовательность таких состояний называется стохастическим процессом (от греческого слова stochos, что значит “догадка”). Рассмотрим простейший стохастический процесс и на его примере введем общие определения. Продукция компании А, производящей зубную пасту, в настоящее время занимает 10% рынка. Руководство компании попросило фирму, проводящую маркетинговые исследования, предсказать долю рынка, занимаемую зубной пастой А после массированной рекламной кампании. Для оценки будущей доли рынка был проведен всеобъемлющий опрос потребителей. Оказалось, что если потребитель использует зубную пасту А, то вероятность того, что, когда она закончится, он купит ее снова, равна 0,8. С другой стороны, если потребитель пользуется другой зубной пастой, то вероятность того, что, когда она закончится, он перейдет на пасту А, равна 0,6. Это значит, что каждый потребитель зубной пасты представляет собой систему, пребывающую в одном из двух следующих состояний. А — потребитель пользуется зубной пастой А; А! — потребитель пользуется другой зубной пастой. Вероятности, полученные при исследовании рынка, можно изобразить на диаграмме переходов (рис. 7.1).
Глава 7. Цепи Маркова 671 Рис. 7.1. Диаграмма переходов С другой стороны, эту же информацию можно представить в виде матрицы вероятностей переходов. Следующее состояние Предыдущее состояние ' 0,8 0,2 ' 0,6 0,4 Задание 7.1. 1. Рассмотрим диаграмму переходов 7.1. Какова вероятность того, что потребитель сменит зубную пасту А на другую? 2. Рассмотрим матрицу вероятностей переходов Р. Какова вероятность того, что потребитель, который не пользуется зубной пастой А, не станет ее покупать и в следующий раз? 3. На рис. 7.1 сумма вероятностей, соответствующих стрелкам, выходящим из каждого состояния, равна единице. Будет ли это верно для любой диаграммы? Аргументируйте ответ. 4. Рассмотрим матрицу вероятностей переходов Р. Сумма вероятностей в каждой строке равна единице. Будет ли это верно для любой матрицы вероятностей переходов? Аргументируйте ответ. ■ Продукция компании, производящей зубную пасту, охватывает 10% всего рынка перед проведением рекламной компании, представим это в виде матрицы начального распределения. . .А 50 = [ 0,1 0,9 ] Для случайно выбранного потребителя вероятность того, что он пользуется пастой А (состояние А), равна 0,1, а вероятность того, что он не пользуется пастой А (состояние А'), равна 0,9. Поэтому матрицу So можно также рассматривать как матрицу начальных вероятностей. Каковы будут вероятности пребывания потребителя в состояниях А и А' после проведения рекламной кампании? Рассмотрим приведенное ниже вероятностное дерево. (Замечание. Символ Ао обозначает состояние А перед проведением рекламной компании, А'г — состояние А' завершения первого тура рекламной кампании и т.д.)
672 Часть II. Конечная математика Состояние 0 Состояние 1 (исходное Как и в главе 6, найдем требуемые вероятности непосредственно с помощью вероятностного дерева. = р'(а» n Ai) + р('а^ а а) ; = = 0,1 0,8+ 0,9 0,6 = 0,62, p(Ai)L=Z(^ = = 0,1 0,2+ 0,9 0,4 = 0,38. (Замечание. P(A^) + Р(А'1) = 1, как и должно быть.) Это значит, что матрица исходного состояния имеет следующий вид. А А' 51 = [ 0,62 0,38 ] Эта матрица описывает вероятности пребывания случайно выбранного покупателя в состояниях А и А' после первого тура рекламной компании. Как видим, доля участия зубной пасты А в рынке выросла от 10 до 62%. Если требуется найти вероятности пребывания покупателя в состояниях А и А' после десятого тура рекламной кампании, то к вероятностному дереву нужно добавить дополнительные ветки. По мере их добавления читатель, скорее всего, придет в замешательство, и понятно от чего — количество веток на каждом этапе удваивается. На десятом этапе их количество составляет уже 211 = 2028! К счастью, суммирование по веткам можно заменить матричным умножением. В частности, произведение матрицы исходного состояния So и матрицы переходов дает матрицу первого состояния Si. 50Р = [ 0,1 0,9 ] Исходное состояние 0,8 0,2 0,6 0,4 Матрица переходов А А' = [ 0,1 • 0,8 + 0,9 • 0,6 0,1 • 0,2 + 0,9 • 0,4 ] = [ 0,62 0,38 ] = Sx. Первое состояние Сравните с вероятностным деревом
Глава 7. Цепи Маркова 673 Легко понять, что матрицу второго состояния S2 (после второго тура рекламной компании) можно получить, умножив матрицу первого состояния на матрицу переходов. А А' 8гР = [ 0,62 0,38 ] Первое состояние 0,8 0,2 0,6 0,4 А А* = [ 0,724 0,276 ] = S2. Второе состояние Матрица третьего состояния S3 получается аналогичным способом S2p = [ 0,724 0,276 ] Второе состояние ' П Я О 9 1 0 6 04 = [ °’7448 0,2552 ]=S3. L ’ ’ J Третье состояние Анализируя матрицы первых трех состояний, можно заключить, что после каждого тура рекламной компании доля рынка, занимаемая зубной пастой А, растет. Может ли оказаться, что доля зубной пасты А на рынке будет расти вплоть до 100%, или объем продаж рано или поздно остановится на более низкой отметке? Ответы на эти вопросы будут даны в следующей главе, где рассматриваются методы исследования матрицы состояния в долговременной перспективе. Матрицы переходов и матрицы состояния Рассмотренная выше последовательность событий (изменение объемов продаж зубной пасты), описываемых постоянной матрицей переходов Р, представляет собой особый случай стохастических процессов, который называется цепью Маркова. В общем случае, цепью Маркова (или процессом Маркова) называется последовательность событий, испытаний или наблюдений, для которой матрица вероятностей перехода из одного состояния в другое постоянна. У марковского процесса нет истории. Цепи Маркова Для заданной цепи Маркова с п состояниями матрица fc-ro состояния имеет вид &k = [«fci ? Зк2, • • • 1 Здесь каждый элемент — это доля элементов генеральной совокупности, пребывающих в г-м состоянии после fc-ro испытания или, что эквивалентно, вероятность пребывания случайно выбранного элемента в г-м состоянии после к-го испытания. Сумма всех элементов матрицы fc-ro состояния Sk равна единице. Матрицей переходов называется постоянная квадратная матрица Р порядка п, у которой элемент на пересечении г-й строки и j-ro столбца равен вероятности перехода из г-ro состояния в j-e при следующем наблюдении или испытании. Сумма элементов каждой строки матрицы Р равна единице. Замечания. 1. Поскольку элементы fc-й матрицы состояния — вероятности, то их значения представляются действительными числами от 0 до 1. 2. В результате перестановки состояний и соответствующих им вероятностей в матрице переходов получается другая, но эквивалентная матрица переходов. Например,
674 Часть II. Конечная математика обе приведенные ниже матрицы — это обсуждавшиеся выше матрицы переходов, применяемые при вычисления доли рынка зубной пасты. А А* А Г 0,8 0,2 Г ~ А' 0,6 0,4 А' 0,4 0,2 А 0,6 ‘ 0,8 Такого рода перестановки влияют на форму матриц, которые используются для решения задачи, однако окончательный результат не изменяется. В разделе 7.3 мы рассмотрим ситуацию выбора специальной формы матрицы переходов. Впрочем, сейчас порядок состояний в матрице переходов не имеет значения. Как упоминалось выше, для поиска матриц состояния в цепях Маркова используется операция произведения матриц. Вычисление матриц состояния в цепях Маркова Если So — матрица исходного состояния, а Р — матрица переходов для цепи Маркова, то последовательность матриц состояния задается следующими формулами. 51 = SqP матрица первого состояния 52 = SiP матрица второго состояния 53 = S2P матрица третьего состояния Sk = Sk-iP матрица fc-ro состояния И Пример 7.1 (Страхование). Страховая компания выяснила, что в среднем за 10 лет 23% водителей, попадавших в аварию в течение года, становятся участниками ДТП и в следующем году. Кроме того, оказалось, что только 11% водителей, не попадавших в ДТП в течение года, становятся его участником в этом году. Выполните следующие задания, рассматривая указанные процентные значения как приближенные эмпирические вероятности. 1. Постройте диаграмму переходов. 2. Запишите матрицу переходов Р. 3. Предположим, что в текущем году в аварию попали 5% водителей. Какова вероятность того, что случайно выбранный водитель попадет в аварию в следующем году? Через год? Решение. 1. 0,23 А — Участвовали в ДТП А' — Не участвовали в ДТП
Глава 7. Цепи Маркова 675 2. Текущий год Следующий год А А' А Г 0,23 0,77 ' А' 0,11 0,89 Матрица переходов 3. Матрица исходного состояния So имеет вид: А А' So = [ 0,05 0,95 ] Матрица исходного состояния Таким образом, А А' S0P = [ 0,05 0,95 ] Текущий год 0,23 0,11 0,77 0,89 (начальное состояние) A Af = [ 0,116 0,884 ] = Si Следующий год (первое состояние) А А1 SXP = [ 0,116 0,884 ] Следующий год (первое состояние) 0,23 0,77 0,11 0,89 А А' = [ 0,12392 0,87608 ] = S2 Через год (второе состояние) Вероятность того, что случайно выбранный водитель попадет в аварию в следующем году, равна 0,116, при этом поучаствовать в ДТП через год он может с вероятностью, равной 0,12392. Таким образом, ожидается, что 11,6% водителей попадут в аварии в следующем году, а 12,392% — через год. ■ W Упражнение 7.1. Нш В страховой компании водителя относят к категории низкого риска (эта категория обозначается буквой L), если он не попадал в аварию на протяжении одного года. Оказалось, что 98% водителей этой категории остаются в ней и в следующем году, а 78% водителей, которые не относились к другим категориям (множество Z/), включаются в нее в следующем году. 1. Постройте диаграмму переходов. 2. Запишите матрицу переходов Р, 3. Предположим, что в текущем году к категории низкого риска отнесены 90% водите¬ лей. Какова вероятность того, что случайно выбранный водитель останется в этой категории и в следующем году? Через год? ■ Степени матриц переходов Рассмотрим операцию возведения матриц переходов в степень. Задание 7.2. Задана матрица перехода Р и матрица исходного состояния So- А А1 А Г 0,9 0,1 А9 0,7 0,3 А А' So = [ 0,5 0,5 ] . 1. Вычислите матрицы S2 и S4.
Часть II. Конечная математика 676 2. Вычислите матрицы Р2 и Р4 (напоминаем, что Р2 = РР, а Р4 = Р2Р2). 3. Вычислите матрицы SqP2 и SqP4. 4. Сравните результаты решения задач 1 и 2. Какую интерпретацию можно дать элементам матриц Р2 и Р4? ■ Матрицы состояний для цепей Маркова определяются рекурсивно. Это значит, что каждая матрица состояния выражается через предыдущую. Например, для поиска матрицы четвертого состояния нужно вычислить матрицы трех предыдущих состояний. 51 = 50Р S2 = SiP S3 = S2P S4 = S3P Есть ли способ найти заданную матрицу состояния без предварительного вычисления матриц предыдущих состояний? Если подставить уравнение для Si в уравнение для S2, затем выполнить подстановку получившегося выражения в уравнение для S3 и так далее, то получается следующая цепочка уравнений: 51 = S0P; 52 = SXP = S0P2; s3 = S2P = SXP2 = 50P3; S4 = S3P = S2P2 = 5iP3 = S0P4. В общем, матрицу fc-ro состояния можно вычислить с помощью формулы Sk = SoPk. Резюмируем этот важный результат в теореме 7.1. Теорема 7.1 (Степени матрицы перехода). Если Р — матрица переходов и So — матрица исходного состояния, то матрицу /с-го состояния можно найти с помощью следующей формулы. Sk = SQPk. Элемент, стоящий на пересечении г-й строки и j-ro столбца матрицы Рк, представляет собой вероятность перехода системы из г-го состояния в j-e после к наблюдений (или испытаний). Сумма элементов в каждой строке матрицы Рк равна единице. ■ Пример 7.2 (Использование матрицы Рк для поиска состояния Sk). Найдите матрицу Р4 и вычислите с ее помощью матрицу S4 при следующих условиях. 0,1 0,6 0,9 0,4 So = [ 0,2 0,8 ]. Решение, Р2 = РР = р4 = р2 р2 = 0,55 0,3 ’ 0,1 0,9 ‘ 0,6 0,4 ’ 0,1 0,6 0,45 ' 0,7 0,9 ' 0,4 ’ 0,55 0,45 ' ’ 0,55 0,45 ‘ ’ 0,4375 0,5625 ' 0,3 0,7 0,3 0,7 0,375 0,625 = [ 0,3875 0,4375 0,375 0,5625 0,625 S4 = S0P4 = [ 0,2 0,8 ] 0,6125 ] .
Глава 7. Цепи Маркова 677 Упражнение 7.2. Найдите матрицу Р4 и вычислите с ее помощью матрицу S4 при следующих условиях. А А' 0,8 0,2 0,3 0,7 А А' So = [ 0,8 0,2 ] Если для вычисления произведений степеней матриц можно применить графический калькулятор или компьютер, то поиск матрицы состояния для любого числа испытаний становится совершенно рутинной процедурой. Пример 7.3 (Использование графической утилиты для вычисления Sk с помощью Рк). Вычислите матрицу S& с помощью матрицы Р8, используя графическую утилиту и матрицы So и Р, заданные в примере 7.2. Значения элементов матрицы Ss округлите до шести десятичных знаков. Решение, Запишем матрицы So и Р в память компьютера, а затем используем уравнение 58 - S0P8. Результат вычисления показан на рис. 7.2. Итак, с точностью до шести десятичных знаков получаем следующий ответ. S8 = [ 0,399219 0,600781 ] . в р [[.1 .9] [.6 .411 so [[.2 .811 50*Рл8 [[.399219 .6007811J Рис. 7.2. Решение задачи в примере 7.3 с помощью графической утилиты Упражнение 7.3. Вычислите матрицу Ss с помощью матрицы Р8, используя графическую утилиту и матрицы So и Р, заданные в упражнении 7.2. Элементы матрицы Ss округлите до шести десятичных знаков. ■ Решение практических задач Проиллюстрируем применение теоремы 7.1 для решения практической задачи. И Пример 7.4 (Зачисление в университет). Студенты, которые зачислены в университет по программе MBA, считаются первокурсниками до тех пор, пока они успешно не сдадут 15 зачетов. Далее их переводят на второй курс, где они приступают к изучению более сложных дисциплин и начинают писать дипломную работу. Опыт работы показал, что ежегодно 10% студентов первого курса (обозначим это состояние буквой
678 Часть II. Конечная математика F) отчисляют из университета (обозначим это состояние буквой £>), а 30% первокурсников переводят на второй курс (обозначим это состояние буквой S). Со второго курса ежегодно исключают 10% студентов, а 30% второкурсников заканчивают университет (обозначим это множество буквой G). Исключенные студенты и окончившие университет назад не возвращаются. 1. Постройте диаграмму переходов. 2. Вычислите матрицу переходов Р. 3. Какова вероятность того, что студент первого курса окончит университет за 4 года? Какова вероятность того, что он будет исключен на протяжении 4 лет? Решение. 1. Если 10% первокурсников исключаются, а 30% переходят на второй курс, то 60% студентов остаются на первом курсе на второй год (см. рисунок). Аналогично на второй год остаются 50% студентов-второкурсников. Поскольку в случае исключения студенты не возвращаются в университет, то все отчисленные студенты (состояние D) пребывают в нем и в следующем учебном году. Это условие отображается единицей, приписанной у стрелки, замкнутой на состоянии D. То же касается и состояния G. °’6С 1 F 0,1 D ) О1 0,3 0,17 Диаграмма переходов °’5С ( s 0,4 G ) О1 F D S G F 0,6 од 0,3 0 ~ п D 0 1 0 0 2- р ~ S 0 0,1 0,5 0,4 Матрица переходов G 0 0 0 1 3. Вероятность того, что студент первого курса перейдет из состояния F в состояние G на протяжении четырех лет, равна элементу первой строки четвертого столбца матрицы Р4 (согласно теореме 7.1). Для вычисления матрицы Р4 вручную требуется два умножения. 0,6 0,1 0,3 0 ■ 0,6 0,1 0,3 0 0 1 0 0 0 1 0 0 0 0,1 0,5 0,4 0 0,1 0,5 0,4 0 0 0 1 0 0 0 1 0,36 0,19 0,33 0,12 0 10 0 0 0,15 0,25 0,6 0 0 0 1
Глава 7. Цепи Маркова 679 _ р2р2 __ ■ 0,36 0,19 0,33 0,12 ‘ ’ 0,36 0,19 0,33 0,12 ‘ 0 1 0 0 0 1 0 0 0 0,15 0,25 0,6 0 0,15 0,25 0,6 0 0 0 1 0 0 0 1 ■ 0,1296 0,3079 0,2013 0,3612 ' 0 1 0 0 0 0,1875 0,625 0,75 0 0 0 1 Таким образом, вероятность того, что студент окончит университет за четыре года, равна 0,3612. Аналогично вероятность исключения студента за четыре года равна 0,3079 (элемент в первой строке и втором столбце матрицы Р4). ■ ага Упражнение 7.4. Еш Вернемся к примеру 7.4. В конце учебного года производится проверка степени готовности выпускных работ у студентов второго курса. Из прошлого опыта известно, что у 30% студентов выпускные работы успешно защищаются (обозначим это состояние буквой А), а 10% студентов исключаются за неуспеваемость (обозначим это состояние буквой D), и к учебе они больше не возвращаются. Остальные студенты продолжают писать дипломную работу. 1. Постройте диаграмму переходов. 2. Вычислите матрицу переходов Р. 3. Какова вероятность того, что студент второго курса окончит университет за четыре года? Какова вероятность, что он будет исключен на протяжении четырех лет? ■ Задание 7.3. Вернемся к примеру 7.4. Состояния D и G называются поглощающими (absorbing states), потому что, попав в это состояние, студент уже его не покидает. Подробнее поглощающие состояния обсуждаются в разделе 7.3. 1. Как распознать поглощающие состояния на диаграмме переходов? Постройте диаграмму переходов для двух состояний, одно из которых является поглощающим, а другое — нет. 2. Как распознать поглощающие состояния в матрице переходов? Постройте матрицу переходов, которая соответствует диаграмме, построенной при решении задачи !.■ Ответы к упражнениям 7.1. 1) 0,98 А — низкий уровень риска А'—другие уровни риска Следующий год 2) Текущий год L L' L Г 0,98 0,02 L' 0,78 0,22
680 Часть II. Конечная математика 3) Следующий год: 0,96; через год; 0,972. ’■2' = [ 0^625 S ] • S< = [°'6125 °'38751 ■ Практикум 7.1 А В задачах 1-8 идет речь о следующей матрице переходов А В А [ 0,8 0,2 ‘ Г ~ В [ 0,4 0,6 ’ В задачах 1-4 необходимо найти состояние Sy при заданном состоянии So и объяс- нить результат с помощью древовидной диаграммы. *1. So=[10], *2. So =[0 1]. *3. So = [0,5 0,5]. *4. So = [0,3 0,7]. В задачах 5-8 необходимо найти состояние S2 при заданном состоянии So и объяснить его смысл. *5. So =[10]. *6. So =[0 1]. *7. So = [0,5 0,5]. *8. So = [0,3 0,7]. В задачах 9-14 приведены матрицы перехода, в которых пропущены некоторые вероятности. Существует ли однозначный способ их восстановить? Если да, то дополните диаграмму отсутствующими числами и постройте соответствующие матрицы переходов. Если нет, объясните, почему. *9. х *Ю. 0)9 Z~X 0,4 Cl А ) ( В ¥С? ?Cl А ) ( В j)0,2 0,7 ?
Глава 7. Цепи Маркова 681 Можно ли однозначно подобрать числа а, Ь и св задачах 15-20 так, чтобы матрица Р стала матрицей переходов? Если да, то заполните матрицу отсутствующими элементами и постройте соответствующую диаграмму переходов. Если нет, объясните, почему это невозможно. А В С А В С А ' 0 0,5 а А а 0 0,9 ' *15. Р = В ь 0 0,4 *16. Р = В 0,2 0,3 Ь С 0,2 с ОД _ С 0,6 с 0 АВС АВС А ' 0 а 0,3 ' А ' 0 1 а *17. Р = В 0 b 0 *18. Р = В 0 0 b С с 0,8 0 С с 0,5 0 А В С А В С А ‘ 0,2 0,1 0,7 А а 0,8 0,1 *19. Р = В а 0,4 с *20. Р = В 0,3 Ь 0,4 С 0,5 Ь 0,4 С 0,6 0,5 С Б В задачах 21-24 используйте приведенную информацию для построения матрицы переходов. 21. Марковский процесс состоит из двух состояний А и В. Вероятность перехода из состояния А в состояние В за одно испытание равна 0,7, а из состояния В в состояние А — 0,9. 22. Марковский процесс состоит из двух состояний А и В. Вероятность перехода из состояния А в состояние А за одно испытание равна 0,6, а из состояния В в состояние В — 0,2. 23. Марковский процесс состоит из трех состояний А, В пС. Вероятность перехода из состояния А в состояние В за одно испытание равна 0,1, из состояния А в состояние С — 0,3, из состояния В в состояние А — 0,2, из состояния В в состояние С — 0,5, а из состояния С в состояние С — 1. 24. Марковский процесс состоит из трех состояний А, В и С. Вероятность перехода из состояния А в состояние В за одно испытание равна единице, из состояния
682 Часть II. Конечная математика В в состояние А — 0,5, из состояния В в состояние С — 0,5, а из состояния С в состояние А — 1. В задачах 25-34 идет речь о следующей матрице перехода и ее степенях. АВС А Р = В 0,6 0,3 0,1 0,2 0,5 0,3 , С А 0,1 0,2 0,7 _ АВС ‘ 0,43 0,35 0,22 ‘ Р2 = В 0,25 0,37 0,38 , С 0,17 0,27 0,56 А АВС ' 0,35 0,348 0,302 ’ Р3 = В 0,262 0,336 0,402 С 0,212 0,298 0,49 25. Вычислите вероятность перехода из состояния А в состояние В за два испытания. 26. Вычислите вероятность перехода из состояния В в состояние С за два испытания. 27. Вычислите вероятность перехода из состояния С в состояние А за три испытания. 28. Вычислите вероятность перехода из состояния В в состояние В за три испытания. *29. Вычислите состояние S2, если So = [1 0 0], и объясните его смысл. *30. Вычислите состояние S2, если Sq = [0 1 0], и объясните его смысл. *31. Вычислите состояние S3, если So = [0 0 1], и объясните его смысл. *32. Вычислите состояние S3, если So = [1 0 0], и объясните его смысл. 33. Используя графическую утилиту для вычисления степеней Р, найдите наименьшее число п такое, что матрицы Рп и Р71^1 равны с точностью до двух десятичных знаков. 34. Используя графическую утилиту для вычисления степеней Р, найдите наименьшее число п такое, что матрицы Рп и Pn+1 равны с точностью до трех десятичных знаков. В задачах 35-38 заданы матрица переходов Р и исходное состояние So. Вычислите матрицу, рассматривая указанные процентные значения, и используйте ее для поиска S4. А В А Г 0,8 0,2 ' F ~ В 0,3 0,7 ] ’ So = [0,4 0,6]. А В 0,1 0,9 0,6 0,4 So= [0,8 0,2]. 35. р=*
Глава 7. Цепи Маркова 683 В С С В А ■ 0 0,4 0,6 ‘ А ' 0 1 0 37. Р = В 0 0 1 ; 38. Р = В 0,8 0 0,2 С 1 0 0 С 1 0 0 So = [0,4 0,4]. 0,2 So = [0,2 0,3 0,5]. *39. Марковский процесс из двух состояний описывается матрицей переходов Р. Начальное состояние описывается матрицей So = [0 1]. Объясните связь между элементами матрицы к-го состояния и к-й степенью матрицы переходов Р. * 40. Повторите решение задачи 39, если матрица исходного состояния равна So = В 41. Задана матрица переходов. А В С D А ’ 0,2 0,2 0,3 0,3 ‘ В 0 1 0 0 С 0,2 0,2 0,1 0,5 D 0 0 0 1 а) Вычислите матрицу Р±. б) Вычислите вероятность перехода из в) Вычислите вероятность перехода из г) Вычислите вероятность перехода из состояния А в D за четыре испытания, состояния С в В за четыре испытания, состояния В в Л за четыре испытания. 42. Повторите решение задачи 41 для матрицы А В С D А ’ 0,5 0,3 0,1 0,1 ' В 0 1 0 0 С 0 0 1 0 D 0,1 0,2 0,3 0,4 Матрица называется матрицей вероятностей, если все ее элементы являются действительными числами со значениями от нуля до единицы и сумма элементов в каждой строке равна единице. Так, матрицы переходов являются квадратными матрицами вероятностей, а матрицы состояний — матрицами вероятностей с одной строкой. * 43. Пусть матрица р _ а 1 —а 1-6 b является матрицей вероятностей. Покажите, что Р2 также является матрицей вероятностей. * 44. Пусть матрицы Р = а 1 — а 1-6 6 и S = [с 1 — с]
684 Часть II. Конечная математика являются матрицами вероятностей. Покажите, что матрица SP также является матрицей вероятностей. В задачах 45-48 для вычисления указанной матрицы состояния используйте формулу Sk = SQPk и графическую утилиту. * 45. Задана матрица переходов для цепи Маркова. 0,4 0,6 Г ” |_ 0,2 0,8 _ ’ а) Пусть 5о = [О 1]. Найдите S2, S4, Sg,... Можно ли определить предельное значение S, к которому стремятся матрицы 5^? б) Повторите решение п. а для So = [1 0]. в) Повторите решение п. а для Sq = [0,5 0,5]. г) Вычислите произведение SP для каждой из матриц S, найденных в пп. а-в. д) Дайте короткое словесное описание поведения матриц состояния этой марковской цепи на основе наблюдений, сделанных при решении пп. а-г. * 46. Повторите решение задачи 45 для матрицы Г 0,9 0,1 ‘ Г 0,4 0,6 ‘ *47. Вернемся к задаче 45. Вычислите матрицу Рк для к = 2,4,8,... Можно ли определить предельную матрицу Q, к которой стремятся матрицы Рк1 Какое отношение имеет матрица Q к результатам задачи 45? * 48. Вернемся к задаче 46. Найдите матрицу Рк для к = 2,4,8,... Можно ли опре¬ делить предельную матрицу Q, к которой стремятся матрицы Рк? Какое отношение имеет матрица Q к результатам задачи 46? Применение математики Экономика и бизнес 49. Планирование. Открытый ресторан в летние месяцы не работает только в дождливую погоду. Из прошлого опыта известно, что если днем идет дождь, то вероятность дождя на следующий день составляет 0,4. Если же днем нет дождя, то на следующий день он может пойти с вероятностью 0,06. а) Составьте диаграмму переходов. б) Вычислите матрицу переходов. в) Пусть в среду идет дождь. Какова вероятность того, что в субботу ресторан будет закрыт? А в воскресенье? 50. Планирование. Повторите решение задачи 49, если вероятность дождя на следующий день составляет 0,6, если в этот день шел дождь и 0,1 — если дождя не было.
Глава 7. Цепи Маркова 685 51. Реклама, На протяжении футбольного первенства проводилась массированная рекламная компания крема для бритья известного бренда X. Исследование показало, что через неделю его продолжают использовать 80% исходного количества потребителей, а 20% потребителей отдают предпочтение кремам других производителей. Было также установлено, что каждую неделю 20% потребителей кремов других производителей переходят на крем для бритья X, а 80% — нет. а) Составьте диаграмму переходов. б) Вычислите матрицу переходов. в) Пусть в начале рекламной компании кремом X пользуются 20% потребителей. Сколько процентов потребителей будут использовать этот крем через неделю после начала рекламной компании? А через две недели? 52. Аренда автомобилей. Агентство сдает автомобили напрокат (и принимает обратно) в двух основных аэропортах Нью-Йорка — имени Кеннеди и Ла Гвардиа (LaGuardia). Допустим, что машину, арендованную в одном из аэропортов, нужно вернуть в один из них. Если машина арендована в аэропорту Ла Гвардиа, то вероятность ее возвращения в него же равна 0,8, если же в аэропорту имени Кеннеди, то вероятность возврата ее в исходный пункт равна 0,7. Будем считать, что фирма располагает 100 автомобилями, и каждый из них арендуется и возвращается один раз в день. Предположим, что в начале дня в каждом аэропорту было по 50 машин. а) Сколько автомобилей может оказаться в каждом аэропорту на следующий день? б) Сколько автомобилей может оказаться в каждом аэропорту через два дня? 53. Страхование жилья. Рынок страховых услуг, предоставляемых домовладельцам в некотором городе, разделен между двумя компаниями: National Property и National Family. Компания National Property обеспечивает страховку 50% домов в городе, a National Family — 30%. Оставшиеся дома застрахованы другими, более мелкими компаниями. Для увеличения количества клиентов компания National Family намерена предложить скидки. В ближайшие несколько лет на рынке страхования прогнозируются следующие ежегодные изменения. Каждый год 25% клиентов компании National Property будут переходить к компании National Family, а 10% — в другие компании. Далее, 10% клиентов компании National Family перейдут к компании National Property, и 5% — в другие компании. И, наконец, 15% клиентов других компаний перейдут к компании National Property, и 35% — в компанию National Family. а) Постройте диаграмму переходов. б) Вычислите матрицу переходов. в) Какой процент домов будет застрахован компанией National Property в следующем году? Через год? г) Какой процент домов будет застрахован компанией National Family в следующем году? Через год? 54. Служебные контракты. Годичные контракты на обслуживание систем отопления частных домов города заключили две компании: Alpine Heating и Badger Furnaces. В настоящее время 25% домовладельцев пользуются услугами ком-
686 Часть II. Конечная математика пании Alpine Heating, и 30% — Badger Furnaces. Остальные домовладельцы не заключали контрактов на предоставление указанных услуг. Для привлечения новых клиентов обе фирмы развернули активную рекламную компанию. На следующие несколько лет получен такой прогноз. Ежегодно 35% домовладельцев, у которых ранее не было контрактов, будут заключать их с компанией Alpine Heating, а 40% — Badger Furnaces. Впоследствии 10% клиентов каждой из компаний перейдут из одной в другую и 5% откажутся от заключения контрактов. а) Постройте диаграмму переходов. б) Вычислите матрицу переходов. в) Какой процент от общего числа домов будет пользоваться услугами компании Alpine Heating в следующем году? А через год? г) Какой процент от общего числа домов будет пользоваться услугами компании Badger Furnaces в следующем году? А через год? 55. Обучение сотрудников. Национальная сеть туристических агентств разработала программу по обучению новых туристических агентов. Изначально все новые агенты классифицируются как начинающие. Каждые шесть месяцев у каждого агента проверяют уровень его квалификации. Из прошлого опыта известно, что после каждой полугодовой проверки 40% начинающих агентов получают промежуточную квалификацию (они нуждаются лишь в минимальном наставничестве), 10% агентов оказываются непригодными к этой работе, а остальные продолжают работу как начинающие. Впоследствии 30% агентов промежуточной квалификации становятся квалифицированными агентами (не требуется наставник), 10% агентов оказываются непригодными к работе, а остальные продолжают работать в качестве агентов промежуточной квалификации. а) Постройте диаграмму переходов. б) Вычислите матрицу переходов. в) Какова вероятность того, что начинающий агент станет квалифицированным агентом через год? Через два года? 56. Обучение сотрудников. Все сварщики на фабрике начинают работать в должности ученика. Уровень квалификации каждого ученика проверятся ежегодно. Из прошлого опыта известно, что 10% учеников становятся профессиональными сварщиками, 20% признаются непригодными к работе, а остальные продолжают обучение. а) Постройте диаграмму переходов. б) Вычислите матрицу переходов. в) Какова вероятность того, что ученик станет профессиональным сварщиком через два года? А через четыре года? Биологические науки 57. Медицинское страхование. Университет, расположенный на Среднем Западе, предлагает своим сотрудникам медицинское страхование трех видов: полное (clinic-based health medicine organization — HMO), ограниченное (preferred provider organization — PPO) и традиционное (fee-for service program — FFS). Ежегодно есть период времени, на протяжении которого сотрудник может изменить форму
Глава 7. Цепи Маркова 687 своего страхования. Из прошлогодних записей известно, что 20% сотрудников были застрахованы по программе НМО, 25% — по программе РРО, а остальные — по программе FFS. На протяжении периода, когда можно было менять страховую программу, из программы страхования НМО 15% сотрудников перешли в программу РРО и 5% — в программу FFS, из программы РРО 20% сотрудников перешли в программу НМО и 10% — в программу FFS, а из программы FFS 25% сотрудников перешли в программу НМО и 30% — в программу РРО. а) Постройте диаграмму переходов. б) Какой процент сотрудников будет застрахован по программе каждого типа в конце периода, на протяжении которого происходят изменения? в) Если такая тенденция сохранится и в будущем, какой процент сотрудников будет застрахован по программе каждого типа через год? 58. Стоматологическое страхование. Вернемся к задаче 57. На протяжении периода, когда возможны изменения, сотрудники университета могут выбирать между двумя формами стоматологической страховки: неполной (low-opinion plan — LOP) и полной (high-opinion plan — НОР). В прошлом году 40% сотрудников были застрахованы по программе LOP и 60% — по программе НОР. На протяжении переходного периода 30% сотрудников, застрахованных по программе LOP, перешли на программу НОР, а 10% застрахованных по программе НОР перешли на программу LOP. а) Постройте матрицу переходов. б) Каким будет процентное содержание сотрудников, застрахованных по каждой из программ по истечении переходного периода в текущем году? в) Если такая тенденция сохранится и в будущем, какой процент сотрудников будет застрахован по программе каждого типа через год? 59. Рынок недвижимости. Согласно переписи 1990 года, 36,4% семей в округе Колумбия (District of Columbia) владеют недвижимостью, тогда как остальные ее арендуют. На протяжении следующего десятилетия 7,6% домовладельцев стали арендаторами (остальная часть домовладельцев сохранила свой статус). Аналогично 10,8% арендаторов стали домовладельцами (остальная часть арендаторов также сохранила свой статус). а) Постройте соответствующую матрицу переходов. б) Используя матрицу переходов, построенную при решении п. а, определите, какой процент домовладельцев сохранит свой статус в 2000 году. в) Пусть матрица переходов не меняется и в будущем. Какой процент домовладельцев сохранит свой статус в 2010 году? 60. Рынок недвижимости. Согласно переписи 1990 года, 58,4% семей в Аляске владеют недвижимостью, тогда как остальные ее арендуют. На протяжении следующего десятилетия 2,1% домовладельцев стали арендаторами (остальная часть домовладельцев сохранила свой статус). Аналогично 23,4% арендаторов стали домовладельцами (остальная часть арендаторов также сохранила свой статус), а) Постройте соответствующую матрицу переходов.
688 Часть II. Конечная математика б) Используя матрицу переходов, построенную при решении п. а, определите, какой процент домовладельцев сохранит свой статус в 2000 году. в) Пусть матрица переходов не меняется и в будущем. Какой процент домовладельцев сохранит свой статус в 2010 году? 7.2. Регулярные цепи Маркова ■ Стационарные матрицы ■ Регулярные цепи Маркова ■ Решение практических задач ■ Приближенные вычисления с помощью графической утилиты Если задан марковский процесс с матрицей переходов Р и матрицей начального состояния So, то элементы матрицы Sk равны вероятностям пребывания системы в соответствующих состояниях после к испытаний. А что будет происходить с этими вероятностями, если увеличивать число к? В этом разделе мы рассмотрим ограничения на матрицу переходов Р, которые позволяют предсказывать поведение при неограниченном увеличении параметра к как матриц состояния Sk, так и степеней матрицы перехода Pk. Стационарные матрицы Начнем с анализа конкретного примера — задачи о компании, производящей зубную пасту из предыдущего раздела. Напомним, что матрица переходов имеет следующий вид. А А' 0,8 0,2 А — используют зубную пасту, 0,6 0,4 А' — не используют зубную пасту. Изначально компания занимает 10% рынка зубной пасты. Как изменится распределение рынка зубной пасты, если вероятности в матрице перехода Р будут верны и для более поздних моментов времени? Для прояснения этого вопроса рассмотрим несколько первых матриц состояния (подробности вычисления матричных произведений пропущены). So = [0,1 0,9] , 51 = S0P = [0,62 0,38] , 52 = S3P = [0,724 0,276] , 53 = S2P = [0,7448 0,2552] , 54 = S3P = [0,74896 0,25104] , 55 = SiP = [0,749792 0,250208] , 56 = S3P = [0,7499584 0,2500416] . Оказывается, что матрицы состояний по мере увеличения номера становятся все ближе и ближе к матрице S = [0,75 0,25]. Вычислим произведение матрицы S (матрица, предельная для последовательности состояний) и матрицы переходов. SP = [0,75 0,25] 0,8 0,6 0,2' 0,4 = [0,75 0,25] = S.
Глава 7. Цепи Маркова 689 Никакого изменения! Матрица [0,75 0,25] называется стационарной. Если система пребывает в этом состоянии или находится близко от него, то говорят, что система находится в устойчивом состоянии. Это значит, что в дальнейшем состояние системы вообще не меняется или меняется незначительно. В контексте рассматриваемого примера это значит, что по истечении большого интервала времени потребитель будет пользоваться зубной пастой А с вероятностью 0,75, т.е. компания удерживает 75% рынка (если предполагать, что матрица переходов не меняется со временем). Ниже приведено общее определение стационарной матрицы. Стационарная матрица для цепи Маркова Матрица состояния S = [si s% ... sn] называется стационарной матрицей для цепи Маркова с матрицей переходов Р, если SP = S, где Si 0, i = 1,..., п и $i + S2 Ч 1- sn = 1. Задание 7.4. 1. Допустим, что изначально компания, производящая зубную пасту, владела 5% рынка (вместо 10%). Запишите матрицу начального состояния, вычислите следующие шесть матриц состояния и предскажите поведение этих матриц состояния, если процесс вычисления степеней продолжить и далее. 2. Повторите вычисления, проделанные при решении задачи 1, если компания начала рекламную компанию, уже обладая 90% общего рынка зубной пасты. ■ Регулярные цепи Маркова Обязательно ли для марковской цепи существование единственной стационарной матрицы? Если она существует, то обязательно ли матрицы состояний стремятся к ней как к пределу? К сожалению, ответы на эти вопросы — отрицательные (см. задачи 31-34 практикума 7.2). Однако существует важный класс цепей Маркова, для которого ответы на оба вопроса являются утвердительными. Такие цепи Маркова называются регулярными. Регулярные цепи Маркова Матрица переходов Р называется регулярной, если все матричные элементы некоторой степени Р являются положительными числами. Цепь Маркова называется регулярной, если ее матрица переходов является регулярной. Пример 7.5 (Распознавание регулярной цепи Маркова). Какие из приведенных ниже матриц — регулярны? 1. р = 0,8 0,2 0,6 0,4 • 2. Р = 0 1‘ 1 0 3. Р = ’0,5 0,5 0 0,5 1 0 0 0,5 0
690 Часть II. Конечная математика Решение. 1, Эта матрица перехода взята из примера о компании, производящей зубную пасту. Поскольку все элементы матрицы Р — положительны, можно сделать вывод, что эта матрица регулярна. 2. Среди элементов матрицы Р имеется два нулевых элемента, поэтому рассмотрим несколько ее высших степеней: Р2 = ’1 О' 0 1 ’ Очевидно, степени матрицы Р будут и далее осциллировать между Р и Z, единичной матрицей 2x2. Поэтому у всех степеней Р будут существовать нулевые матричные элементы, так что матрица Р не является регулярной. 3. Рассмотрим высшие степени Р. 0,25 0,5 0,25' "0,375 0,375 0,25 ‘ р2 = 0,5 0,25 0,25 , Р3 = 0,5 0,375 0,125 0,5 0,5 0 0,25 0,5 0,25 Поскольку все элементы Р3 являются положительными целыми числами, то матрица Р регулярна. ■ Упражнение 7.5. Какие из приведенных ниже матриц — регулярны? 0' 0 3. Р = 0 0,5 0,5 1 0 0 0,5 0 0,5 Задание 7.5. Опишем деятельность компании, производящей зубную пасту (она рассмотрена в начале раздела), с помощью обычной транзитивной матрицы Р и стационарной матрицы S, где р=[вд !м] и 5=[°’75 “■25i- Сравните Р2, Р4, и Р8. Обсудите любые видимые отношения между Рк и S. ■ Соотношения между последовательными матрицами состояний, степенями матрицы переходов и стационарным состоянием для цепи Маркова перечислены в теореме 7.2. Доказательство этой теоремы дается в более специализированных учебниках. Теорема 7.2 (Свойства регулярных цепей Маркова). Пусть Р — матрица переходов для регулярной цепи Маркова.
Глава 7. Цепи Маркова 691 1. Существует единственная стационарная матрица 5, которая является решением уравнения SP = P. 2. При заданной матрице исходного состояния So матрицы Sk стремятся к пределу S. 3. Матрицы Рк стремятся к предельной матрице Р, каждая строка матрицы Р совпадает со стационарной матрицей S. ■ Пример 7.6 (Поиск стационарной матрицы). Задана матрица переходов для стационарной цепи Маркова: = ГО,7 0,3 Г ~ [0,2 0,8 ’ 1. Вычислите стационарную матрицу S. 2. Определите взаимосвязь между матрицами Sk и Рк при больших значениях к. Решение. 1. Стационарная матрица S существует в силу регулярности матрицы Р. Для ее поиска необходимо решить уравнение SP = S. Пусть S = [в! 32] , тогда г -I 0,7 0,3 г -| [S1 521 |_0,2 0,8] = S2i ’ Выполняя умножение в левой части, получаем равенство [(0,7si + 0,2s2)(0,3si -I- 0,8s2)] = [51 ^2] , которое эквивалентно следующей системе. 0,7si + 0,2s2 = si или —0,3si 4- 0,2з2 = 0; 0,3si 4- 0,8з2 = з2 или 0,3si — 0,2з2 = 0. Система (7.1) является зависимой и имеет бесконечно много решений. Однако среди них нас интересуют только те решения, которые являются матрицами состояний. Поэтому систему уравнений (7.1) нужно дополнить еще одним уравнением, чтобы получить систему с единственным решением. —0,331 4“ 0,2з2 = 0; 0,3si - 0,2з2 = 0; (7.2) si 4- s2 = 1. Решение системы (7.2) получается матричными методами. Оно имеет следующий вид. si = 0,4 и з2 = 0,6. Итак, матрица S = [0,4 0,6] является стационарной.
692 Часть II. Конечная математика Проверка. SP = [0,4 0,6] 0,7 0,3 0,2 0,8 = [0,4 0,6] = S. 2. При заданной матрице начального состояния So теорема 7.2 гарантирует, что последовательность матриц состояния Sk стремится к стационарной матрице S. рк = 0,7 0,2 0,3] k 0,8 стремится к предельной матрице Р = 0,4 0,4 0,6 0,6 Упражнение 7.6. Задана матрица перехода для цепи Маркова 0,6 0,4 0,1 0,9 Вычислите стационарную матрицу S и предельную матрицу Р. Решение практических задач П Пример 7.7 (Страхование). Вернемся к примеру 7.1, где была вычислена следующая матрица переходов из одной категории в другую. А А' А [0,23 0,77 Г А' |_0,11 0,89 А — участвовали в ДТП, А' — не участвовали в ДТП. Пусть эти вероятности сохраняют значение и в будущем. Каков процент водителей, попадающих в ДТП на протяжении произвольно выбранного года? Решение. Для определения того, что случится в отдаленном будущем, найдем стационарную матрицу из следующей системы г п Го,23 0,771 г 52J [0,11 0,89_| “ t51 s2] И S1 + 32 — 1, которая эквивалентна следующей системе. 0,23$i -I- 0,11«2 = si; или — 0,77$i -I- 0,1152 = 0; 0,7751 -|- 0,89«2 = «25 0,77«i — 0,1152 = О’, 51 + 52 = 1. 51 4- 52 = 1. Решая эту систему, получим ответ. 51 = 0,125 и 52 = 0,875. Таким образом, стационарная матрица равна [0,125 0,875]. Если матрица переходов не изменяется в течение длительного периода времени, то в следующем году в аварию попадут 12,5% водителей. I
Глава 7. Цепи Маркова 693 Упражнение 7.7. Вернемся к упражнению 7.1, где была найдена следующая матрица переходов водителей из одной категории в другую. L L' L [0,98 0,02' Г L' [о,78 0,22 L — низкий риск, I/ — высокий риск. Пусть эти вероятности сохраняют значение и в будущем. Какой процент водителей можно отнести к категории низкого риска в произвольно выбранном году? ■ И Пример 7.8 (Оценка уровня квалификации служащих). Компания оценивает уровень квалификации каждого служащего как “ниже среднего”, “средний” или “выше среднего”. Последняя проверка показала, что ежегодно 10% служащих квалификации ниже средней повышают свой уровень и получают среднюю квалификацию, а 25% служащих средней квалификации переходят на уровень выше среднего. С другой стороны, 15% служащих средней квалификации понижают свой уровень и переходят на уровень ниже среднего, и 15% служащих квалификации выше средней переходят на уровень средней квалификации. Правилами компании запрещена смена квалификации с уровня “ниже средней” на уровень “выше средней” (и наоборот) на протяжении одного года. Какой процент служащих компании будут иметь уровень квалификации ниже среднего через большой промежуток времени? Среднюю квалификацию? Квалификацию выше средней? Решение. Для начала построим матрицу переходов: Текущий год Следующий год А- А А+ А- ’0,9 0,1 0 А 0,15 0,6 0,25 А+ 0 0,15 0,85 А — ниже среднего, А — средний, А+ — выше среднего. Для определения поведения этой системы в отдаленном будущем найдем стационарную матрицу, решая следующую систему. [»1 «2 S3] 0,9 0,15 0 ОД 0,6 0,15 0 0,25 0,85_ = [si 32 s3] И «1 + 32 + S3 = 1 Она эквивалентна системе 0,9si + 0,15з2 = si; или —0,lsi + 0,15з2 =0; 0,l$i -I- 0,6«2 + 0,15зз = «2’, 0,lsi — 0,4$2 4- 0,15з3 = 0; 0,25з2 4- 0,85з3 = зз; 0,25«2 - 0,15з3 = 0; <$1 4“ 82 4“ <$3 = 1. 31 + 32 4- S3 = 1. Используя для решения этой системы четырех уравнений с тремя неизвестными метод исключения Гаусса-Жордана, получим следующий ответ. si = 0,36; si = 0,24; si = 0,4.
694 Часть II. Конечная математика Это значит, что через достаточно большой промежуток времени 36% служащих будут иметь уровень квалификации ниже среднего, 24% — среднюю квалификацию и 40% — квалификацию выше средней. ■ Упражнение 7.8. Компания, продающая товары по почте, в зависимости от числа заказов классифицирует своих клиентов как “постоянных”, “обычных” или “случайных”. Из прошлого опыта известно, что ежегодно 5% постоянных клиентов становятся обычными и 12% — случайными клиентами; 5% обычных клиентов переходят в категорию “постоянных” и 5% — становятся случайными клиентами; 9% случайных клиентов становятся постоянными и 10% — переходят в категорию обычных. Считая, что указанные процентные отношения сохраняются и в будущем, найдите процентный состав каждой категории клиентов через большой промежуток времени. В Приближенные вычисления с помощью графической утилиты Ю- Если Р — матрица переходов для регулярной марковской цепи, степени матрицы Р стремятся к предельной матрице Р, причем каждая строка матрицы Р равна стационарной матрице S (см. теорему 7.2.3). Теперь мы воспользуемся этим свойством для приближенного поиска S с помощью вычисления матриц Рк для достаточно больших значений к. Продемонстрируем это на следующем примере. Пример 7.9 Приближенное вычисление стационарной матрицы). Вычислите приближенные значения матриц Р и S с точностью до четырех десятичных знаков, рассчитывая степени матрицы переходов Р. Результат вычислений проверьте подстановкой в уравнение SP = S. 0,5 0,7 0,4 0,2 0,1 0,1 0,3 0,2 0,5 Решение. Для вычисления матрицы Р с точностью до четырех десятичных знаков введем матричные элементы Р в память компьютера (рис. 7.3, а) и установим в окне вывода требуемую точность. Далее будем вычислять степени матрицы Р до тех пор, пока строки матрицы Рк не станут идентичными. Анализируя результат на рис. 7.3, б, приходим к следующим результатам. 0,4943 0,4943 0,4943 0,1494 0,3563 0,1494 0,3563 0,1494 0,3563 и S = [0,49430,14940,3563] . Вводя матрицу S в память компьютера и вычисляя произведение SP, видим, что матрицы SP и S равны с точностью до четырех десятичных знаков (рис. 7.3, в). ■
Глава 7. Цепи Маркова 695 [[.5 .2 .3] [.7 .1 .21 [.4 .1 .5]] [[.4943 .1494 .3563] [.4943 .1494 .3563] [.4943 .1494 .3563]] S [[.4943 .1494 .3563]] S*P [[.4943 .1494 .3563]] а)Р в) Проверка: SP = S Рис. 7.3. Решение задачи в примере 7.9 с помощью графической утилиты let Упражнение 7.9. Повторите решение примера 7.9 для матрицы 0,3 0,2 0,1 0,6 0,Г 0,3 0,5 0,2 0,7 Замечания. <> _ 1. Для вычисления Р в примере 7.9 мы использовали довольно маленькое значение числа к. Во многих графических утилитах матрицы Рк вычисляются для больших значений к столь же быстро, как и для малых. Однако в таких вычислениях возникают ошибки округления. Для их устранения нужно начинать вычислять матрицы Рк для малых значений к, например для к = 8, и затем удваивать значение к до тех пор, пока строки матрицы Рк не совпадут с требуемой точностью. 2. Если некоторые элементы матрицы Рк стремятся к нулю, то в утилитах могут использоваться научные обозначения для отображения этих значений как очень малых чисел. На рис. 7.4 показан результат вычисления 100-й степени матрицы переходов Р. Элемент на пересечении второй строки и второго столбца матрицы Р100 стремится к нулю, однако в результате вычисления утилиты получается 5,1538 х х IO’53. Это значит, что такие значения в матрице Р следует просто заменить нулями. Из результатов, приведенных на рис. 7.4, можно заключить, что Г1 о lfc 0,7 0,3 стремится к Р = [[1 Рл100 [[1.0000 (1.0000 [.7 0 ] .311 0.0000 5.1538е-531] ] 0 0 Рис. 7.4. Решение задачи в примере 7.9 при больших значениях к
696 Часть II. Конечная математика Ответы к упражнениям 7.5. 1) Регулярна. 2) Нерегулярна. 7.6. 5= [0,2 0,8];Р=[°^ . 7.7. 97,5%. 3) Регулярна. 7.8. 28% постоянных, 43% обычных и 29% случайных. 7.9. Р = 0,1618 0,1618 0,1618 0,2941 0,2941 0,2941 0,5441 0,5441 0,5441 ; S = [0,1618 0,2941 0,5441]. Практикум 7.2 А В задачах 1-14 определите, какая из матриц переходов является регулярной. 1. р = 0,1 0,7 '1 0' 0,9' 0,3 ' 3. Р = 0 1 5. Р = 0,2 0,8' 1. Р = 1 Г1 0,6 0,3 0 0 0,4 0,4 0,3‘ 9. Р = 0 0 1 0,4 0,2 0,4 ’0 1 0 11. Р = 0,4 0 0,6 . О 1 0 ’0 0 1 13. Р = 0,8 0,1 0,1 0 1 0 0,2 0,5 0,5 0,5 0,1' 1 8. Р = 0 0,3 0 1 0,7 0 1 10. Р = 0,1 0,1 0,8 0,5 0,3 0,2 '0,1 0,3 0,6 12. Р = 0 0 1 1 0 0 0 0,5 0,5 14. Р = 1 0 0 . 1 0 0 15. Р = 17. Р = 19. Р = Б В задачах 15-22 для каждой из матриц переходов Р решите уравнение SP = S для поиска стационарной матрицы и найдите предельную матрицу Р. 0,1 0,9' 16 р = 0,8 0,2 0,6 0,4 0,3 0,7 '0,5 0,5' 18. Р = '0,9 0,1" 0,3 0,7 0,7 0,3 0,5 0,1 0,4' 0,4 0,1 0,5' 0,3 0,7 0 20. Р = 0,2 0,8 0 0 0,6 0,4 0 0,5 0,5
Глава 7. Цепи Маркова 697 0,8 0,2 0 ’0,2 0,8 0 21. Р = 0,5 0,1 0,4 22. Р = 0,6 0,1 0,3 0 0,6 0,4 0 0,9 0,1 Верны ли утверждения, приведенные в задачах 23 и 24? Аргументируйте ответ. * 23. Верны ли следующие утверждения? а) Если два элемента матрицы переходов Р размером 2x2 равны нулю, то матрица Р — не регулярна. б) Если три элемента матрицы переходов Р размером 3x3 равны нулю, то матрица Р — не регулярна. * 24. Верны ли следующие утверждения? а) Если Р — матрица переходов для цепи Маркова размером п х п и S — матрица размером 1 х п, удовлетворяющая уравнению SP — 5, то S является стационарной матрицей. б) Если для матрицы переходов Р марковской цепи существует стационарная матрица S, то матрица Р — регулярна. Е\ В задачах 25-28 найдите приближенную стационарную матрицу S для каждой из матриц переходов Р, вычисляя степени матриц Р. Результаты округлите до четырех десятичных знаков. 25. Р = 0,51 0,27 0,5 0,49 0,73 0,5 1 0 26. Р = 0,68 0,19 0,2 0,32 0,81 0,2 1 0,6' 27. Р = 0 0,5 1 0,5 28. Р = 0,5 0 1 0,5 0,8 0,1 1 0,1 0,5 0 1 0,5 В 29. В красной урне находится два красных шара и три синих, а в синей урне — один красный шар и четыре синих. Из урны вынимают шар, определяют его цвет и возвращают обратно в ту же урну. Следующий шар вынимается из урны с тем же цветом, что и у ранее вытянутого шара. Таким образом, цепь Маркова состоит из двух состояний: шар вынимается из красной урны или шар вынимается из синей урны. а) Постройте диаграмму переходов для этого процесса. б) Вычислите матрицу переходов. в) Вычислите стационарную матрицу и опишите поведение системы через большой промежуток времени. 30. Повторите решение задачи 29, если в красной урне находится 5 красных и 3 синих шара, а в синей урне — 1 красный и 3 синих шара. *31. Задана матрица переходов 1 о 0 1 а) Опишите поведение матриц состояния Si, S2, 5з> •, если начальная матрица состояний имеет вид: So = [0,2 0,8].
698 Часть II. Конечная математика б) Повторите решение п. а, если начальная матрица состояний имеет вид: So = = [0,5 0,5]. в) Опишите поведение последовательности матриц Рк, к = 2,3,4,... г) В какой части нарушается теорема 7.2 для данной матрицы переходов? Почему нет противоречия? * 32. Задана матрица переходов Р= 0 1 О' 0 1 0 0 0 а) Опишите поведение матриц состояния Si, S2, S3, • • •, если начальная матрица состояний имеет вид: So = [0,2 0,3 0,5]. б) Повторите решение п. а для начальной матрицы состояний 1 1 1 3 3 3 в) Опишите поведение последовательности матриц Рк, к = 2,3,4,... г) В какой части нарушается теорема 7.2 для данной матрицы переходов? Почему нет противоречия? * 33. Задана матрица переходов для марковской цепи. 1 0,2 0 0 0 0,2 0,6 0 1 а) Покажите, что обе матрицы R = [1 0 0] и S = [0 0 1] являются стационарными для матрицы Р. Почему это не противоречит теореме 7.2.1? б) Вычислите еще одну стационарную матрицу для Р. (Подсказка', рассмотрите матрицу Т = aR + (1 — a)S, где 0 < а < 1.) в) Сколько можно построить различных стационарных матриц? * 34. Задана матрица переходов для марковской цепи. 0,7 0 0,2 0 0,3' 1 0 0 0,8 а) Покажите, что обе матрицы R = [0,4 0 0,6] и S = [0 1 0] являются стационарными для матрицы Р. Почему это не противоречит теореме 7.2.1? б) Вычислите еще одну стационарную матрицу для Р. (Подсказка', рассмотрите матрицу Т = aR + (а — 1)S, где 0 < а < 1.) в) Сколько можно построить различных стационарных матриц? В задачах 35 и 36 требуется применение графической утилиты. о 35. Вернемся к матрице переходов Р задачи 33. К какой матрице Р стремятся степени матрицы Р? Будут ли строки матрицы Р стационарными матрицами для Р?
Глава 7. Цепи Маркова 699 36. Вернемся к матрице переходов Р задачи 34. К какой матрице Р стремятся степени матрицы Р? Будут ли строки матрицы Р стационарными матрицами для Р? * 37. Задана матрица переходов для цепи Маркова. ‘0,1 0,5 0,4 0,3 0,2 0,5 0,7 0,1 0,2 Пусть Мк обозначает максимальный элемент второй строки матрицы Рк. Заметим, что Mi = 0,5. а) Найдите элементы М2,Мз,М4 и М5 с точностью до трех десятичных знаков. б) Объясните, почему Мк Mk+i для всех положительных значений к. * 38. Задана матрица переходов для цепи Маркова. 0 0,2 0,8 Р = 0,3 0,3 0,4 0,6 0,1 0,3 Пусть rrik обозначает максимальный элемент третьей строки матрицы Рк. Заметим, что mi = 0,3. а) Вычислите элементы m2, тз, и с точностью до трех десятичных знаков. б) Объясните, почему гпк mfc+i для всех положительных значений к. Применение математики Экономика и бизнес 39. Транспортировка. Большинством железнодорожных вагонов владеют отдельные компании. Когда вагон покидает свое депо, он становится частью национального парка вагонов и может быть использован другими железными дорогами. Согласно установленным правилам вагон в конечном счете возвращается владельцу. Оказалось, что на типичной железной дороге ежемесячно 11% вагонов покидают депо и присоединяются к национальному парку вагонов и 29% вагонов железной дороги возвращаются назад. Пусть эти процентные отношения сохраняются и в будущем. Какая часть вагонов железной дороги будет находиться в пределах собственной транспортной сети через длительный промежуток времени? 40. Транспортировка. Железная дорога (см. задачу 39) владеет также цистернами. Оказалось, что 14% цистерн покидают депо и присоединяются к национальному парку цистерн и 26% цистерн железной дороги возвращаются назад. Какая часть цистерн железной дороги будет находиться в пределах собственной транспортной сети через длительный промежуток времени? 41. Трудовые ресурсы. В табл. 7.1 показано, какой процент женщин был задействован на общественных работах в США в разные годы.
700 Часть II. Конечная математика WWW Таблица 7.1. Количество женщин, занятых общественной работой Год Доля, % 1970 43,3 1980 51,5 1990 53,5 Ниже приведена матрица переходов Р, которая была предложена для моделирования этих данных. Обозначение L применяется для доли женщин, занятых общественной работой, a L' — всех остальных. Следующее десятилетие L L' Текущее L 0,93 0,07 _ р десятилетие L' 0,2 0,8 — а) Пусть So = [0,433 0,567]. Вычислите матрицы Si и S2 (обе матрицы вычислите точно, а затем округлите результаты до трех десятичных знаков). б) Постройте новую таблицу, сравнивая результаты решения п. а с данными из табл. 7.1. в) Какой процент женщин будет занят общественной работой через большой промежуток времени согласно приведенной матрице переходов? 42. Исследование рынка. В табл. 7.2 приведены данные о доле домов, оснащенных телевизионной антенной, за указанные годы. WWW Таблица 7.2. Оснащение домой телевизионной антенной Год Доля, % 1990 56,4 1995 63,4 2000 68,0 Ниже приведена матрица переходов Р, которая была предложена для моделирования этих данных. Здесь символ С обозначает процент домов, в которых есть телеантенна. Следующий пятилетний период С С1 Текущий пяти- С 0,92 0,08 _ р летний период С' 0,26 0,74 ” а) Пусть So = [0,564 0,436]. Вычислите матрицы Si и S2 (обе матрицы вычислите точно, а затем округлите результаты до трех десятичных знаков). б) Постройте новую таблицу, сравнивая результаты решения п. а с данными из табл. 7.1. в) Какой процент домов будет оборудован телеантенной через большой промежуток времени согласно приведенной матрице переходов?
Глава 7. Цепи Маркова 701 43. Доля рынка. Потребители в некотором штате могут выбирать между тремя поставщиками телекоммуникационных услуг: GTT, NCJ и Dash. В результате активной рекламной компании рынок потребителей находится в постоянном движении. Ежегодно компания GTT теряет 5% клиентов, которые переходят к компании NCJ, и 20%, которые переходят к компании Dash. Далее, 15% клиентов компании NCJ переходят к компании GTT и 10% — к компании Dash. И, наконец, 5% клиентов компании Dash переходят к компании GTT и 10% — к компании NCJ. Как будет распределен рынок телефонной связи в отдаленном будущем, если считать, что эти процентные соотношения сохранятся? 44. Доля рынка. Потребители в некотором районе могут выбирать между тремя поставщиками услуг: APS, GX и WWP. Еженедельно компания APS теряет 10% клиентов, которые переходят к компании GX, и 20%, которые переходят к компании WWP. Далее, 15% клиентов компании GX переходят к компании APS и 10% — к компании WWP. И, наконец, 5% клиентов компании WWP переходят к компании АРХ и 5% — к компании GX. Как будет распределен рынок между компаниями в отдаленном будущем, если считать, что эти процентные соотношения сохранятся? 45. Страхование. Компания, страхующая автомобили, разделяет своих клиентов на три категории: нежелательные, обычные и привилегированные. Ежегодно 40% нежелательных клиентов переходят в категорию обычных, 20% обычных клиентов становятся привилегированными. Кроме того, 20% привилегированных клиентов переходят в категорию обычных, а 20% обычных клиентов становятся нежелательными. На протяжении одного года переходы клиентов из категории нежелательных в привилегированную группу (и наоборот) запрещены. Как будут распределены клиенты по категориям в отдаленном будущем, если считать, что эти процентные соотношения сохранятся? 46. Страхование. Решите задачу 45, предположим, что 40% клиентов из привилегированной категории становятся обычными, а остальные данные остаются без изменений. Для решения задач 47 и 48 требуется применение графической утилиты. 47. Доля рынка. Компания Acme Soap производит один сорт мыла, который называется Standart Acme (для краткости — £4), компания Best Soap производит два сорта мыла, которые называются Standart Best (для краткости — SB) и Deluxe Best (для краткости — DB). В настоящее время компания Acme Soap владеет 40% рынка, а его оставшаяся часть принадлежит компании Deluxe Best. Для большего охвата рынка в компании Acme Soap принимается решение выпустить второй сорт мыла. Новый сорт, обозначенный X, проходил тестирование в двух крупных городах. Ниже приведена матрица переходов, которая построена в результате недельных наблюдений за привычками и вкусами покупателей.
702 Часть II. Конечная математика Будем считать, что эта матрица переходов в будущем не изменится. Найдите последовательность состояний для приближенного поиска стационарной матрицы с точностью до двух десятичных знаков, если исходное состояние имеет вид So = [0,3 0,3 0,4 0]. Какая доля рынка будет принадлежать сорту X, если компания Acme Soap решит выпустить его в продажу? 48. Доля рынка. Вернемся к задаче 47. Химики компании Acme Soap разработали другой сорт мыла, обозначенный Y. Результаты тестовых продаж этого сорта вместе с тремя другими сортами приведены в следующей матрице переходов. SB SB DB SA 0,3' 0,3 0,2 0,2 DB 0,2 0,2 0,2 0,4 SA 0,2 0,2 0,4 0,2 Y 0,1 0,2 0,3 0,4 Повторите решение задачи 47 для поиска стационарной матрицы с точностью до двух десятичных знаков. Какая доля рынка будет принадлежать сорту У, если компания Acme Soap решит выпустить его в продажу? Какой сорт мыла выгоднее выпускать — X или У? Биологические науки 49. Генетика. Некоторое растение может иметь красные, розовые или белые цветки в соответствии с генотипами RR, RW и WW. Если каждый из этих генотипов скрещивается с генотипом розовых цветков, то матрица переходов будет иметь такой вид. Следующее поколение эасные Розовые Белые Красные 0,5 0,5 0 Текущее Розовые 0,25 0,5 0,25 поколение Белые 0 0,5 0,5 Пусть каждое поколение растений скрещивается только с растениями, имеющими только розовые цветки. Покажите, что, независимо от набора генов исходного поколения, генотип каждого растения со временем стабилизируется на следующих значениях: 25% красных, 50% розовых и 25% белых цветков (найдите стационарную матрицу). 50. Мутация генов. Допустим, что ген в хромосоме может иметь либо тип А, либо тип В. Пусть вероятность мутации гена типа А в ген типа В за одно поколение составляет 10“4, а вероятность мутации гена типа В в ген типа А — 10“6. а) Чему равна матрица переходов? б) Какова вероятность, что через много поколений ген будет иметь тип А? Какова вероятность, что через много поколений ген будет иметь тип В? (Вычислите стационарную матрицу.) Социальные науки 51. Общественный транспорт. Новая система метро только что вступила в действие. Оказалось, что за первый месяц услугами метро воспользовались 25% пас-
Глава 7. Цепи Маркова 703 сажиров, а остальные 75% продолжают пользоваться автомобилями. В следующей матрице переходов приведена информация о транспортных предпочтениях горожан. Следующий месяц Метро Метро 0,8 Автомобиль 0,3 Автомобиль Текущий месяц 0,2 0,7 а) Чему равна матрица исходного состояния? б) Какой процент пассажиров будет пользоваться новой системой метро через один месяц? Через два месяца? в) Вычислите процент пассажиров, которые будут пользоваться каждым видом транспорта через большой промежуток времени. 52. Политика: принятие законов. Во время обсуждения в сенате возникла опасность принятия закона с недопустимо большой задержкой. Причиной тому послужил сенатор Хэнкс, который мог с вероятностью 0,1 изменить свое мнение в течение следующих пяти минут. Предположим, что такое положение верно для каждого пятиминутного промежутка в течение всего времени дебатов. Известно, что закон был принят на протяжении 24 часов. Какова вероятность того, что сенатор Хэнкс проголосовал “за”? Какова вероятность, что он проголосовал “против”? а) Сначала заполните матрицу переходов. Следующие 5 минут Текущие 5 минут За Против За [0,9 0,1' Против б) Найдите стационарную матрицу и ответьте на поставленные выше вопросы. в) Какой будет стационарная матрица, если вероятность изменения мнения сенатора Хенкса равна р? Центром населенности 48 штатов в Соединенных Штатах называется точка равновесия на карте. В 1790 году центр населенности находился в 23 милях восточнее Балтимора, штат Мэриленд. В 1990 году центр населенности сдвинулся приблизительно на 900 миль на восток и на 100 миль на юг и переместился в точку в южной
704 Часть II. Конечная математика части штата Миссури. Для изучения миграции населения американское бюро переписи населения разделило штаты на четыре группы, как показано на рисунке. В задачах 53 и 54 идет речь о миграции населения между этими группами. 53. Миграция населения. В табл. 7.3 приведены данные о количестве жителей южного региона (в процентах к общему населению США) за указанные годы. Соедний Рис. 7.5. Регионы в Соединенных Штатах и центр населенности WWW Таблица 7.3. Год Доля, % 1970 30/Г“ 1980 33,3 1990 34,4 Для моделирования этих данных была предложена следующая матрица переходов (символ S обозначает южный регион). Следующее Текущее десятилетие десятилетие S Sf S Го,61 0,39 S' 0,21 0,79 а) Пусть So = [0,309 0,691]. Вычислите матрицы Si и S2 (вычислите точно, а затем округлите ответ до трех десятичных знаков). б) Постройте новую таблицу согласно результатам решения п. а и сравните ее с табл. 7.3. в) Какой процент населения будет жить в южном регионе в отдаленном будущем согласно указанной матрице переходов? 54. Миграция населения. В табл. 7.4 приведены данные о количестве жителей северного региона (в процентах к общему населению США) за указанные годы. WWW Таблица 7.4. Год Доля, % Л970 24Д 1980 21,7 1990 20,4
Глава 7. Цепи Маркова 705 55. Для моделирования этих данных была предложена следующая матрица переходов (символ N обозначает северный регион). Следующее десятилетие N N' Текущее N 0,61 0,39 десятилетие N' 0,09 0,91 а) Пусть Sq = [0,241 0,759]. Вычислите матрицу Si и S2 (вычислите точно, а затем округлите ответ до трех десятичных знаков). б) Постройте новую таблицу согласно результатам решения п. а и сравните ее с табл. 7.4. в) Какой процент населения будет жить в северном регионе в отдаленном будущем согласно указанной матрице переходов? 7.3. Поглощающие цепи Маркова ■ Поглощающие состояния и поглощающие цепи ■ Стандартная форма ■ Предельная матрица ■ Приближенные вычисления с помощью графической утилиты Из раздела 7.2 известно, что степени регулярных матриц переходов стремятся к предельной матрице. У остальных матриц переходов такого свойства нет. В этом разделе мы рассмотрим другой тип цепей Маркова — так называемые поглощающие цепи Маркова. Несмотря на то что регулярные и поглощающие цепи Маркова отличаются друг от друга, у них есть одно очень важное общее свойство — степени матриц переходов для поглощающих цепей также стремятся к предельной матрице. После обсуждения основных понятий мы построим методы поиска предельных матриц и обсудим отношения между матрицами состояния марковской цепи и элементами матрицы перехода. Поглощающие состояния и поглощающие цепи Состояние марковской цепи называется поглощающим, если система, попав в это состояние, больше не может его покинуть. Пример 7.10 (Распознавание поглощающих состояний). Определите поглощающие состояния для следующих матриц переходов. АВС АВС А ’1 0 0 А 0 0 1' 1. Р — В 0,5 0,5 0 2. Р — В 0 1 0 С 0 0,5 0,5 С 1 0 0
706 Часть II. Конечная математика Решение, 1. Вероятность перехода из состояния А в состояние А равна единице, а из состояния А в состояния В и С — нуль. Это значит, что, попадая в состояние А, система больше его не покидает. По этой причине состояние А является поглощающим. Вероятность перехода из состояния В в состояние А отлична от нуля, и система может покинуть состояние В, т.е. оно не является поглощающим. Аналогично состояние С тоже не поглощающее, поскольку возможен переход из этого состояния в другое с отличной от нуля вероятностью. 2, Как и в предыдущем случае, единица на пересечении второй строки и второго столбца свидетельствует о том, что состояние В является поглощающим. Переходы из состояния А в состояние С и обратно происходят с отличными от нуля вероятностями, поэтому эти состояния не являются поглощающими. ■ Упражнение 7.10. Определите поглощающие состояния для следующих матриц переходов. А А 0,5 В 0 С 0,5' А АВС "0 1 О’ 1. Р = В 0 1 0 2. Р = В 1 0 0 С 0 0,5 0,5 С 0 0 1 Обобщим свойства поглощающих состояний в следующей теореме. Теорема 7.3 (Поглощающие состояния и матрицы переходов). Состояние марковской цепи является поглощающим тогда и только тогда, когда в строке, которая соответствует этому состоянию в матрице переходов, элемент на главной диагонали равен единице, а остальные — нулю. ■ Наличие поглощающих состояний в матрице переходов еще не гарантирует ни существования предела для степеней матрицы переходов, ни существования стационарного состояния. Например, рассмотрим степени матрицы переходов из примера 7.10.2. Поскольку Р2 = I — единичная матрица 3x3, получаем следующие результаты. рЗ = рр2 = р1 = р поскольку р2 = д р4 = ррЗ = рр = т поскольку р3 - р И РР = Р2 = I. И вообще, степени этой матрицы переходов Р будут осциллировать между значениями Р и I, не стремясь ни к какому предельному значению. Задание 7.6. 1. Задана матрица начального состояния So = [а b с]. Вычислите первые четыре матрицы состояний Si, S2, S3 и S4 для марковской цепи с матрицей переходов Р = 0 0 1 0 1 0 1 0 0
Глава 7. Цепи Маркова 707 2. Будут ли стремиться матрицы состояний к стационарной матрице? Аргументируйте ответ. ■ Цепь Маркова с матрицей перехода с одним или несколькими поглощающими состояниями допускает существование предельной матрицы, если на нее наложены дополнительные ограничения, которые приведены в следующем определении. Поглощающие цепи Маркова Цепь Маркова называется поглощающей, если выполняются следующие условия. 1. В ней имеется только одно поглощающее состояние. 2. Переход из непоглощающего состояния в поглощающее может произойти за ко- нечное количество шагов. Как упоминалось выше, поглощающие состояния можно определить, анализируя строки матрицы переходов. Матрицу переходов можно также использовать для определения типа марковской цепи (поглощающая цепь или нет), однако это может оказаться довольно сложной процедурой, особенно для большой матрицы переходов. Для этой цели часто удобнее использовать диаграмму переходов. В следующем примере второй подход используется для определения типов марковских цепей, перечисленных в примере 7.10. Пример 7.11 (Распознавание поглощающих цепей Маркова). С помощью диаграммы переходов определите, является ли матрица Р матрицей переходов поглощающей цепи Маркова. АВС АВС А ’1 0 0' А 0 0 1' 1. Р = В 0,5 0,5 0 г. р = в 0 1 0 С 0 0,5 0,5 с 1 0 0 Решение. 1. Из примера 7.10.1 известно, что состояние А является единственным поглощающим состоянием такой цепи Маркова. Второе требование в определении поглощающей цепи также удовлетворяется, поскольку переход из непоглощающих состояний В и С в поглощающее состояние А может быть произведен за конечное количество шагов. Это проще всего показать, построив диаграмму переходов. Анализируя диаграмму, можно сделать вывод, что переход из состояния В в поглощающее состояние А может быть произведен за один шаг, а из состояния С в состояние А — за два шага. Таким образом, цепь Маркова с матрицей переходов Р является поглощающей.
708 Часть II. Конечная математика 2. Аналогично предыдущей задаче построим диаграмму переходов для матрицы Р, как показано ниже. Из диаграммы видно, что переход из состояний АиС в поглощающее состояние В вообще невозможен. Таким образом, цепь Маркова с матрицей переходов Р не является поглощающей. Упражнение 7.11. С помощью диаграммы переходов определите, является ли матрица Р матрицей переходов поглощающей цепи Маркова. А В С АВС А 0,5 0 0,5' А 0 1 О' 1. Р = В 0 1 0 2. Р = В 1 0 0 С 0 0,5 0,5 С 0 0 1 ■ Задание 7.7. Определите, какие из приведенных ниже высказываний — истинны, а какие — ложные. Ответы проиллюстрируйте с помощью примеров и словесных аргументов. 1. Марковская цепь с двумя состояниями, одно из которых — поглощающее, а другое — нет, всегда является поглощающей марковской цепью. 2. Марковская цепь с двумя состояниями, где оба — поглощающие, всегда является поглощающей марковской цепью. 3. Марковская цепь с тремя состояниями, два из которых — поглощающие, а одно — нет, всегда является поглощающей марковской цепью. ■ Стандартная форма Матрица переходов в цепи Маркова не обязательно определена однозначно. Рассмотрим диаграмму переходов, приведенную на рис. 7.6. Поскольку четыре состояния можно расставить 4! = 24 различными способами, то этой диаграмме можно сопоставить 24 различные матрицы переходов. (Элементы в некоторых матрицах совпадают, однако сами матрицы — отличаются друг от друга, поскольку их строки и столбцы пронумерованы по-разному.) Для примера ниже приведены три матрицы переходов М, N и Р, которые соответствуют этой диаграмме.
Глава 7. Цепи Маркова А 709 М = А ГОД С 0,5 D 0 В 0,6 1 о о с 0,3 о 0,2 О D О О 0,3 1 В 0 D N = B С А В С D В 1 О О 1 0,3 о О 0,6 В D '1 О О 1 0,6 о О 0,3 С А О о о о 0,2 0,5 0,3 0,1 А С О О о о 0,1 0,3 0,5 0,2 (7-3) Рис. 7.6. Диаграмма цепи Маркова Обратите внимание на то, что в матрицах N и Р все поглощающие состояния записаны перед непоглощающими. В этом случае говорят, что матрица переходов записана в стандартной форме. Запись матриц в стандартной форме, как мы убедимся позже, полезна при вычислении предельных матриц для поглощающих цепей Маркова. Общее определение стандартной формы приведено ниже. Стандартная форма для поглощающих цепей Маркова Матрица переходов для поглощающей цепи Маркова представлена в стандартной форме, если ее строки и столбцы пронумерованы так, что все поглощающие состояния предшествуют непоглощающим. (Стандартных форм может быть несколько.) Каждую стандартную форму можно разбить на четыре подматрицы следующего вида. А N А I ; 0 А — все поглощающие состояния, N R \ Q N — все непоглощающие состояния. Здесь I — единичная матрица, 0 — нулевая.
710 Часть II. Конечная математика Возвращаясь к матрице Р в формуле (7.3), мы можем заключить, что подматрицами стандартной формы будут следующие. 1 0 0 1] ’ 0,6 0 0 0,3 0,1 0,3 0,5 0,2 В с В D А С 1 о о о о 1:0 о б;ё 6 : од о;з О 0,3 : 0,5 0,2 Задание 7.8. Диаграмма, изображенная на рис. 7.6, была использована для поиска матриц М, N и Р в уравнении (7.3). Придумайте процедуру преобразования матрицы М к стандартным формам вида N и Р методом перестановки строк и столбцов (не прибегая к диаграмме переходов). Обсудите достоинства и недостатки применения диаграмм переходов по сравнению с операциями перестановки строк и столбцов при поиске стандартных форм.и Предельная матрица Теперь мы можем перейти к рассмотрению поведения марковских цепей для больших промежутков времени. Начнем с примера. И Пример 7.12 (Торговля недвижимостью). Компании, занимающиеся торговлей недвижимостью, намерены со временем скупить все фермы на определенной территории и застроить ее жилыми домами. Ежегодно 20% фермеров решают продать свои владения компании Л, а 30% — компании В. Остальные продолжают жить на своей земле. Купив фермы, ни одна из компаний их уже не продает. 0 0 0 R = 1. Постройте диаграмму переходов для марковского процесса и определите, является ли соответствующая цепь Маркова поглощаюшей. 2. Постройте матрицу переходов в стандартной форме. 3. Пусть в начале процесса скупки ферм обе компании не имели в распоряжении ни одной фермы. Найдите процент ферм, купленных каждой из компаний по истечении большого промежутка времени. 4. Пусть компания А владела 50% ферм, прежде чем к скупке недвижимости приступила компания В. Найдите процент ферм, купленных каждой из компаний по истечении большого промежутка времени.
Глава 7. Цепи Маркова 711 Решение. А — ферма покупается компанией А В — ферма покупается компанией В С — ферма не продается 2. Используем диаграмму переходов для построения матрицы переходов в стандартной форме. А В с А 1 0 0 Р = В 0 1 0 Стандартная форма С 0,2 0,3 0,5 3. Перед началом скупки все фермеры находились в состоянии С (владели своими фермами). Таким образом, So = [0 0 1]. Последовательные матрицы переходов имеют такой вид (детали умножения пропущены). 51 = S0P = [0,2 0,3 0,5] , 52 = SiP = [0,3 0,45 0,25] , 53 = S2P = [0,35 0,525 0,125] , 54 = 53P = [0,375 0,5625 0,0625] , 55 = S4P = [0,3875 0,58125 0,03125] , 56 = S5P = [0,39375 0,590625 0,015625] , 57 = S6P = [0,396875 0,5953125 0,0078125] , Ss = SyP = [0,3984375 0,59765625 0,00390625] , S9 = S8P = [0,39921875 0,598828125 0,001953125] . Легко видеть, что последовательность матриц состояния стремится к следующему пределу. АВА S = [0,4 0,6 0] . Это значит, что компания А со временем скупит 40% ферм, а компания В — оставшиеся 60% ферм. 4. В этом случае 50% фермеров находились в состоянии А и остальные — в состоянии С. Поэтому исходное состояние имеет вид So = [0,5 0 0,5]. Последовательные
712 Часть II. Конечная математика матрицы переходов имеют такой вид (детали вычислений пропущены). 51 = S0P = [0,6 0,15 0,25] , 52 = SiP = [0,65 0,225 0,125] , 53 = S2P = [0,675 0,2625 0,0625] , 54 = S3P = [0,6875 0,28125 0,03125] , 55 = S4P = [0,69375 0,290625 0,015625] , 56 = S5P = [0,696875 0,2953125 0,0078125] , 57 = S6P = [0,6984375 0,29765625 0,00390625] , 58 = S7P = [0,69921875 0,298828125 0,001953125] , Легко видеть, что последовательность матриц состояния стремится к пределу, отличному от матрицы, полученной при решении задачи 3. АВС S' = [0,7 0,3 0] . Исходя из начальных условий, делаем вывод, что компания А со временем скупит 70% ферм, а компания — оставшиеся 30%. ■ Упражнение 7.12. Повторите решение примера 7.12 для ситуации, когда 10% фермеров продают свое имущество компании А, 40% — компании В, а остальные продолжают владеть фермами. ■ Напомним, что согласно теореме 7.2 из раздела 7.2 последовательность матриц состояния регулярной марковской цепи всегда стремится к стационарной матрице. Более того, стационарная матрица всегда является единственной. Это значит, что изменения исходного состояния не влияют на ее вид. Для поглощающей цепи Маркова последовательность матриц состояний также стремится к стационарной матрице, однако она может быть неединственной. Чтобы в этом убедиться, рассмотрим матрицу переходов Р и матрицы состояний S и Sf из примера 7.12. АВС А Г1 0 0‘ Р=В 0 1 0 С 0,2 0,3 0,5 АВС АВС S = [0,4 0,6 0] S' = [0,7 0,3 0] В том, что матрицы S и S' являются стационарными, можно убедиться с помощью следующих операций умножения. SP = [0,4 0,6 0] S'P = [0,7 0,3 0] 1 0 0,2 "1 0 0,2 0 1 0,3 0 1 0,3 о о 0,5 0 0 0,5 = [0,4 0,6 = [0,7 0,3 0] =s. 0] = s'.
Глава 7. Цепи Маркова 713 Более того, у поглощающих цепей Маркова количество стационарных состояний бесконечно (см. задачи 41 и 42, практикум 7.3). Это значит, что при изменении исходного состояния различные последовательности матриц состояния могут стремиться к различным стационарным матрицам. В разделе 7.2 мы использовали стационарную матрицу для поиска предельной матрицы Р. Поскольку для поглощающих цепей существует много стационарных матриц, то этот подход для таких цепей применять нельзя. Однако оказывается, что в поглощающих цепях предельные матрицы существуют, и их достаточно просто найти. В теореме 7.4 указаны основные подходы к вычислениям. Доказательство этой теоремы — предмет более специализированных учебников. Теорема 7.4 (Предельные матрицы для поглощающих цепей Маркова). Если стандартная форма матрицы переходов Р для марковской цепи имеет вид то последовательность матриц Рк при увеличении к стремится к матрице Р, которая имеет вид Матрица F выражается формулой F = (7 — Q)-1 и называется фундаментальной матрицей для матрицы переходов Р. В формуле для фундаментальной матрицы размерность единичной матрицы — такая же, как и у матрицы Q, ■ Пример 7.13 (Поиск предельной матрицы). 1. Вычислите предельную матрицу Р для матрицы переходов Р в стандартной форме, приведенной в примере 7.12. 2. Используйте матрицу Р для поиска предела последовательности состояний, если задано исходное состояние Sq = [0 0 1]. 3. Используйте матрицу Р для поиска предела последовательности состояний, если задано исходное состояние So = [0,5 0 0,5]. Решение, 1. Используя пример 7.12, получаем следующие результаты. I : о R : Q где 1 = 1 0 0 1 R = [0,2 0,3] ; Q = [0,5] .
714 Часть II. Конечная математика Если I = [1] — единичная матрица 1x1, то/ — Q — также будет матрицей 1x1, поэтому F = (/ — Q)-1 будет попросту обратным числом к единственному матричному элементу матрицы I — Q. FR = [2] [0,2 0,3] = [0,4 0,6] . Предельная матрица будет иметь следующий вид. 2. Согласно теореме 7.1 раздела 7.1 предельные матрицы выражаются формулой Sk = = ЗцРк, а матрица Рк стремится к матрице Р. Следовательно, пределом последовательности Sk будет матрица S0P = [о 0 1] '1 о 0,4 0,6 = [0,4 0,6 0] . 0 0 0 Это согласуется с результатом решения задачи 3 из примера 7.12. 3. В этом случае пределом последовательности матриц будет матрица Это согласуется с результатом решения задачи 4 из примера 7.12. Упражнение 7.13. Повторите вычисления, проделанные в примере 7.13 для матрицы Р (в стандартной форме), приведенной в упражнении 7.12. ■ Подчеркнем еще раз, что предельная матрица для регулярной цепи Маркова состоит из вероятностей переходов между состояниями для удаленных моментов времени. То же относится и к поглощающим цепям Маркова. Сравним матрицу переходов и предельную матрицу из примера 7.13. А В с А В с А "1 0 0 А "1 0 О' Р = В 0 1 0 стремится к Р = В 0 1 0 С 0,2 0,3 0,5 С 0,4 0,6 0 Строки матриц Р и Р, соответствующие поглощающим состояниям АиВ, идентичны. Это значит, что вероятность перехода из состояния А в состояние А равна единице как для
Глава 7. Цепи Маркова 715 исходного состояния, так и для всех испытаний, а также в предельной матрице. Третья строка матрицы Р состоит из вероятностей переходов из непоглощающего состояния С в состояния А, В или С для большого промежутка времени. В фундаментальной матрице F содержится некоторая дополнительная информация о цепи Маркова. В примере 7.13 эта матрица равна F = [2]. Можно показать, что элементы матрицы F определяют среднее количество испытаний, необходимых для перехода из непоглощающего состояния в поглощающее. В примере 7.13 единственный элемент 2 матрицы F указывает на то, что в среднем на протяжении двух лет фермер переходит из состояния С (владения фермой) в одно из поглощающих состояний (продажи фермы). Некоторые фермеры переходят в поглощающее состояние за год, другим требуется более двух лет, однако в среднем переход происходит за два года. Часть этих замечаний сформулирована в теореме 7.5, которая здесь приводится без доказательства. Теорема 7.5 (Свойства предельной матрицы Р). Пусть Р — матрица переходов (в стандартной форме) для поглощающей цепи Маркова, F — фундаментальная матрица, а Р — предельная. Тогда справедливы следующие утверждения. 1. Элемент, стоящий на пересечении i-й строки и J-ro столбца матрицы Р, задает вероятность перехода из г-го в j-e состояние по истечении большого промежутка времени. Для непоглощающих состояний эти же вероятности образуют матрицу FR, на основе которой формируется матрица Р, 2. Сумма элементов каждой строки фундаментальной матрицы F равна среднему числу испытаний, необходимых для перехода из непоглощающего состояния в некоторое поглощающее. (Заметим, что строки матриц F и FR соответствуют непоглощающим состояниям в том же порядке, в котором они приведены в стандартной форме матрицы переходов Р.) ■ Замечания. 1. Наличие нулевой матрицы в нижнем левом углу предельной матрицы Р в теореме 7.5 свидетельствует о том, что по истечении большого промежутка времени вероятности переходов между непоглощающими состояниями всегда стремятся к нулю. Это значит, что все элементы марковской цепи со временем оказываются в поглощающих состояниях. 2. Если матрица переходов не представлена в стандартной форме, то предельную матрицу все равно можно найти (см. задачи 37 и 38, практикум 7.3). Однако для исследования поведения марковской цепи на больших интервалах времени целесообразно использовать стандартную форму. Мы рассмотрели все методы, необходимые для анализа поведения марковской цепи на больших интервалах времени. Применим их теперь для решения рассмотренной ранее задачи (пример 7.4 раздела 7.1). ■V Пример 7.14 (Зачисление в университет). Ниже приведена диаграмма переходов ШН для студентов, которые зачислены в университет по программе MBA.
716 Часть II. Конечная математика F— первокурсники, S — второкурсники, D — отчисленные, G — выпускники. 1. Какой процент студентов первого курса окончит университет по истечении большого интервала времени? Какой процент студентов второго курса не окончит университет? 2. За сколько лет (в среднем) студент первого курса заканчивает учебу? А студент второго курса? Решение. 1. Для начала заметим, что у данной цепи Маркова имеется два поглощающих состояния — D и G. Стандартная форма марковской цепи имеет следующий вид. D G F S 1 0 0 0 ' 0 1 0 0 [/'О' 0,1 О 0,6 0,3 R Q 0,1 0,4 0 0,5j Перечислим подматрицы. 1 0 1 0 о о 0,1 0 1 Го,6 0,3' 0,1 0,4 ’ О 0,5 Итак, f = (/-q)-i = QJ j 0,6 0,3 0,4 -0,31 1 _ 2,5 1,5 0 0,5 0 0,5 ] 0 2 и FR = 2,5 1,5 0,1 0 0,4 0,6 0 2 0,1 0,4 0,2 0,8 Таким образом, предельная матрица будет иметь следующий вид. D G F S D ’1 0 0 0 G 0 1 0 0 F 0,4 0,6 0 0 S 0,2 0,8 0 0 Т О FR:: о Из этой предельной матрицы следует, что со временем университет заканчивают 60% студентов первого курса, и 20% студентов второго курса исключаются за неуспеваемость.
Глава 7. Цепи Маркова 717 2. Сумма элементов в первой строке фундаментальной матрицы F равна 2,5 -I-1,5 = = 4. Согласно теореме 7.5 студент первого курса проведет в среднем четыре года в состояниях F и S, прежде чем перейти в состояния D и G. Сумма элементов во второй строке фундаментальной матрицы F равна 0-1-2 = 2. Следовательно, студент второго курса будет учиться в среднем два года, прежде чем окончит университет или будет исключен. ■ Упражнение 7.14. Повторите решение примера 7.14 для следующей диаграммы переходов. F — первокурсники, S — второкурсники, D — отчисленные, G — выпускники. Приближенные вычисления с помощью графической утилиты Z2L Как и для регулярных цепей Маркова, предельную матрицу Р для поглощающих цепей с матрицей переходов Р можно приближенно найти, вычисляя матрицы Рк для достаточно больших значений к с помощью графической утилиты. Например, для стандартной формы матрицы переходов Р из примера 7.14 вычисление Р50 приводит к следующему результату. ’1 0 0 0 50 ’1 0 0 О' 0 1 0 0 0 1 0 0 0,1 0 0,6 0,3 — 0,4 0,6 0 0 0,1 0,4 0 0,5 0,2 0,8 0 0 Здесь, как и раньше, научные обозначения заменены нулями (см. замечание 2 раздела 7.2). Предупреждение. Прежде чем искать матрицу Рк для приближенного вычисления матрицы Р, целесообразно проверить, существует ли матрица Р вообще. Попытавшись приближенно вычислить предельную матрицу, которая на самом деле не существует, вы получите обманчивый результат. Для примера рассмотрим следующую матрицу переходов. '1 0 0 0 0 0,2 0,2 0 0,3 0,3 Р = 0 0 0 0,5 0,5 0 0 1 0 0 0 0 1 0 0
718 Часть II. Конечная математика Вычисляя матрицу Р50 с помощью графической утилиты, получаем следующее. "1 0 0 0 0 0,25 0 0,625 0,0625 0,0625 р50 = 0 0 1 0 0 (7.4) 0 0 0 0,5 0,5 0 0 0 0,5 0,5 Возникает желание на этом остановиться и считать матрицу (7.4) достаточно хорошим приближенным значением для матрицы Р. Но это неверно! Если считать матрицу Р50 приближенным значением матрицы Р, то матрица Р51 также должна являться приближенным значением этой матрицы. Однако вычисление матрицы Р51 приводит к совершенно другому результату. "1 0 0 0 0 0,25 0 0,125 0,3125 0,3125 р50 = 0 0 0 0,5 0,5 (7-5) 0 0 1 0 0 0 0 1 0 0 Вычисляя другие степени матрицы Р, получаем, что четные степени матрицы стремятся к матрице (7.4), а нечетные — к матрице (7.5). Это значит, что матрица переходов Р не имеет предельной матрицы. Графические утилиты также можно использовать и для точного вычисления Р, как показано на рис. 7.7 для матрицы переходов из примера 7.14. Преимуществом этого подхода является промежуточное вычисление фундаментальной матрицы F, суммы элементов по строкам которой дают дополнительную информацию о поведении цепи на больших интервалах времени. I [[1 01 [0 U1 R [[.1 0 1 [.1 .411 а) запись в память матриц / и R ((.6 .31 [0 .5]] F*R [[2.5 1.51 [0 2 ] 1 [[.4 .61 [-2 .811 б) запись в память матрицы Q в) вычисление F и FR Рис. 7.7. Матричные вычисления Ответы к упражнениям 7.10. 1) Состояние В — поглощающее. 2) Состояние С — поглощающее. 7.11. 1) Поглощающая цепь Маркова. 2) Непоглощающая цепь Маркова.
Глава 7. Цепи Маркова 719 3) Компания А скупит 20% ферм, а компания В — 80%. 4) Компания А скупит 60% ферм, а компания В — 40%. А _ А Г1 7.13. 1) Р = В 0 с [о,2 3) [0,4 0,6]. В с 0 О' 1 О 0,8 О 2) [0,2 0,8]. 7.14. 1) 30% студентов первого курса заканчивают университет, 50% студентов второго курса не заканчивают университет. 2) Студенты первого курса оканчивают университет в среднем за 3,5 года, студенты второго курса исключаются из университета в среднем за 2,5 года. Практикум 7.3 А В задачах 1-6 укажите поглощающие состояния в приведенных матрицах переходов. А в с А В С А '0,6 0,3 0,1 А 0 1 0 1. Р = В 0 1 0 2. Р — В 0,3 0,2 0,5 С о 0 1 С L0 0 1 А в с А В С А 0 0 1' А '1 0 0 3. Р = В 1 0 0 . 4. Р = В 0,3 0,4 0,3 С о 1 °. С о 0 1 А в с D А В с D А '1 0 0 0 А 0 1 0 0 _ п В 0 0 0 0 . п В 1 0 0 0 5" р ~ с 0,1 0,1 0,5 0,3 6‘ Р ~ С 0,1 0,2 0,3 0,4 D 0 0 0 1 D 0,7 0,1 0,1 0,1
720 Часть II. Конечная математика В задачах 7-10 укажите поглощающие состояния в приведенных диаграммах переходов. Б В задачах 11—14 найдите стандартную форму цепи Маркова, исходя из приведенной диаграммы переходов. В задачах 15-18 найдите стандартную форму цепи Маркова, исходя из приведенной матрицы переходов. А В с А В С А '0,2 0,3 0,5' А 0 0 1 15. Р = В 1 0 0 16. Р^ В 0 1 0 С 0 0 1 С 0,7 0,2 0,1
Глава 7. Цепи Маркова 721 А В С D А В с D А 0,1 0,2 0,3 0,4' А 0 0,3 0,3 0,4‘ п В 0 1 0 0 п в 0 1 0 0 п.р^с 0,5 0,2 0,2 0,1 18. Р - с 0 0 1 0 D 0 0 0 1 D 0,8 0,1 0,1 0 В задачах 19-24 найдите предельную матрицу для приведенной матрицы переходов в стандартной форме. Вычислите вероятность перехода из непоглощающего состояния в поглощающее и необходимое для этого среднее количество испытаний по истечении большого промежутка времени. А В С А в с А ’1 0 0 А ’1 0 0 19. Р = В 0 1 0 20. Р = В 0 1 0 С L0,l 0,4 0,5 С 0,3 0,2 0,5 А В С А В А А ‘1 0 0 А ’1 0 0 21. Р = В 0,2 0,6 0,2 22. Р = В 0,1 0,6 0,3 С 0,4 0,2 0,4 С 0,2 0,2 0,6 А В С D А В С D А ’1 0 0 0 А ’1 0 0 0 0 1 0 0 24 Р-В 0 1 0 0 0,1 0,2 0,6 0,1 24-р~ С 0,1 0,1 0,7 0,1 D 0,2 0,2 0,3 0,3 D 0,3 0,1 0,4 0,2 В задачах 25-30 речь идет о матрицах из задач 19-24. Для каждой матрицы переходов найдите предельную матрицу Р и определите поведение последовательности состояний на большом интервале времени для приведенных матриц исходного состояния. 25. Для матрицы Р из задачи 19 при заданной матрице исходного состояния. a) So = [0 0 1]. б) So =[0,2 0,5 0,3]. 26. Для матрицы Р из задачи 20 при заданной матрице исходного состояния. a) So = [0 0 1]. б) So =[0,2 0,5 0,3]. 27. Для матрицы Р из задачи 21 при заданной матрице исходного состояния. a) So = [0 0 1]. б) So =[0,2 0,5 0,3]. 28. Для матрицы Р из задачи 22 при заданной матрице исходного состояния. a) So = [0 0 1]. б) So =[0,2 0,5 0,3]. 29. Для матрицы Р из задачи 23 при заданной матрице исходного состояния. a) So = [0 0 0 1]. б) So = [0 0 1 0]. в) So = [0 0 0,4 0,6]. г) So =[0,1 0,2 0,3 0,4]. 30. Для матрицы Р из задачи 24 при заданной матрице исходного состояния. a) So = [0 0 0 1]. б) So = [О 0 1 0]. в) So = [0 0 0,4 0,6]. г) So =[0,1 0,2 0,3 0,4].
722 Часть II. Конечная математика В задачах 31 и 32 обсудите истинность каждого из утверждений. Если утверждение всегда верно, объясните, почему. Если ложно — приведите контрпример. *31. Верно ли следующее утверждение? а) Если цепь Маркова является поглощающей, то каждое состояние в ней — поглощающее. б) Если в цепи Маркова имеются поглощающие состояния, то это — поглощающая цепь. * 32. Верно ли следующее утверждение? а) Если система покидает непоглощающее состояние в поглощающей цепи Маркова, то она больше никогда в него не возвращается. б) Поглощающая цепь Маркова может иметь бесконечное количество стационарных матриц. В задачах 33-36 используйте графическую утилиту для приближенного вычисления предельной матрицы для каждой из указанных стандартных форм. А В С D А В С D А '1 0 0 0 А "1 0 0 0 33. р = в эл. г с 0 1 0 0 ЛЛ Р-В 0 1 0 0 0,5 0,3 0,1 о,1 34- р~ с 0,1 0,1 0,5 0,3 D 0,6 0,2 0,1 о,1 D 0 0,2 0,3 0,5_ А В с D Е А В С D Е А 1 0 0 0 0 А ’1 0 0 0 0 В 0 1 0 0 0 В 0 1 0 0 0 35. Р = С 0 0,4 0,5 0 0,1 . 36. Р=С 0,5 0 0 0 0,5 D 0 0,4 0 0,3 0,3 D 0 0,4 0 0,2 0,4 Е 0,4 0,4 0 0,2 0 Е 0 0 0,1 0,7 0,2 В 37. Ниже приведена матрица переходов для поглощающей цепи Маркова, записанная в нестандартной форме. А В с D А 0,2 0,2 0,6 0 В 0 1 0 0 С 0,5 од 0 0,4 D 0 0 0 1 Найдите предельную форму матрицы Р, следуя описанной ниже процедуре. Этап 1. Используя диаграмму переходов как подсказку, переставьте строки и столбцы матрицы Р так, чтобы получилась стандартная форма цепи Маркова. Этап 2. Вычислите предельную матрицу для этой стандартной формы. Этап 3. Опять, используя диаграмму переходов как подсказку, проведите вычисления п. 1 в обратном порядке для поиска предельной матрицы для исходной матрицы Р.
Глава 7. Цепи Маркова 723 38. Повторите решение задачи 37 для матрицы А В с D А ’1 0 0 0 В 0,3 0,6 0 0,1 С 0,2 0,3 0,5 0 D 0 0 0 1 39. Проверьте ответ задачи 37 с помощью графической утилиты, вычисляя матрицу Рк для достаточно больших значений числа к. 40. Проверьте ответ задачи 38 с помощью графической утилиты, вычисляя матрицу Рк для достаточно больших значений числа к. * 41. Покажите, что матрица S = [ш 1 — х 0], где 0 < х < 1, является стационарной матрицей для следующей матрицы переходов. А в С А ’1 0 0 Р = В 0 1 0 С 0,1 0,5 0,4 Обсудите возможность обобщения этого результата на случай произвольной поглощающей цепи Маркова с двумя поглощающими состояниями и одним непоглощающим. * 42. Покажите, что матрица S = [х 1 — х 0 0], где 0 х С 1, является стационарной матрицей для следующей матрицы переходов. А в С D А -1 0 0 0 В 0 1 0 0 С 0,1 0,2 0,3 0,4 D 0,6 0,2 0,1 0,1 Обсудите возможность обобщения этого результата на случай произвольной поглощающей цепи Маркова с двумя поглощающими состояниями и двумя непоглощающими. 43.* Задана поглощающая цепь Маркова со следующей матрицей переходов в стандартной форме. А В С D А "1 0 0 0 В 0,2 0,3 0,1 0,4 С 0 0,5 0,3 0,2 D 0 0,1 0,6 0,3 I : о R : Q Обозначим максимальный элемент матрицы Qk с помощью w&. Заметим, что wi = 0,6. а) Вычислите W2, W4, wie и W32 с точностью до трех десятичных знаков. б) Опишите вид матрицы Qk для больших значений числа к.
724 Часть II. Конечная математика 44.* Вернемся к матрицам Р и Q из задачи 43. Для положительного целого числа к положим 7fc = l + Q + Q2H h Qk. а) Объясните, почему 7^+1 = TkQ +1. б) Используя графическую утилиту и формулу из п. а, найдите матрицы 7^. Вычислите и объясните связь между матрицами Тк и (7 — Q)-1 для больших значений числа к. Применение математики Экономика и бизнес 45. Кредитование. Кредитный кредитные разделяет займы на приобретение автомобилей на следующие четыре категории: полностью выплаченные (это состояние F); выплачиваемые без задержек (состояние G); выплачиваемые с незначительными задержками (состояние А); выплачиваемые с серьезными задержками или безнадежно просроченные (состояние В). Статистические данные свидетельствуют, что 10% счетов из состояния G переходят в состояние F, 80% остаются в состоянии G и 10% переходят в состояние А. Далее, 10% счетов из состояния А переходят в состояние F, 40% остаются в состоянии А и 10% переходят в состояние В. а) Какой процент кредитных займов будет выплачен полностью через большой промежуток времени? б) Какой процент займов в состоянии А перейдет в состояние В в течение большого промежутка времени? в) Через сколько месяцев (в среднем) кредитный заем перейдет из состояния А в состояние F или В? 46. Обучение сотрудников. Национальная цепь мастерских по ремонту систем торможения и выхлопа у легковых автомобилей разработала программу обучения для автомехаников. Все автомеханики начинают обучение с изучения выхлопной системы автомобиля. Квалификация каждого автомеханика проверяется каждые три месяца. Собранные данные показывают, что в результате каждой проверки 30% автомехаников, обучавшихся ремонту выхлопной системы, получают квалификацию мастера и допускаются к изучению тормозной системы автомобиля. 20% рабочих признаются непригодными к такой работе, а остальные возвращаются к изучению выхлопной системы. В дальнейшем 30% рабочих, изучающих тормозную систему, становятся полностью квалифицированными механиками (не нуждаются в дальнейшем обучении), 10% признаются непригодными к этой работе, а остальные возвращаются к изучению тормозной системы. а) Какой процент рабочих, изучающих выхлопную систему автомобиля, становятся полностью квалифицированными автомеханиками через большой промежуток времени? б) Какой процент автомехаников, изучающих тормозную систему автомобиля, признаются непригодными к выполнению своих обязанностей через большой промежуток времени?
Глава 7. Цепи Маркова 725 в) Через какое среднее время (в кварталах) рабочий, изучающий выхлопную систему автомобиля, становится полностью квалифицированным автомехаником или выбывает из программы обучения по непригодности? 47. Продвижение на рынке. Три компании по производству электронного оборудования, выпускающие графические калькуляторы для колледжей и технических университетов, проводят агрессивную маркетинговую политику, предлагая существенные скидки, широко рекламируя возможности своей продукции и щедро финансируя обучающие программы. Исходя из высокой стоимости и большой потребности в вычислительном оборудовании, учебные заведения, единожды остановив свой выбора на одном из производителей, не обращаются к закупкам продукции конкурирующих компаний. Ежегодно 6% учебных заведений принимают решение использовать калькуляторы компании А, 3% учебных заведений — калькуляторы компании В и 11% учебных заведений — калькуляторы компании С. Остальные учебные заведения обходятся в учебном процессе без использования калькуляторов. а) Какой объем рынка будет охватывать каждая компания по истечению большого промежутка времени? б) Сколько (в среднем) нужно лет, чтобы руководство учебного заведения приняло решение применять калькуляторы в учебном процессе? 48. Пенсионные вклады. Ежегодно сотрудникам компании предоставляется возможность принять участие в одной из пенсионных программ — А, В или С. Единожды выбрав программу, от нее нельзя отказаться в пользу другой программы. Данные прошлых лет показывают, что ежегодно 4% сотрудников выбирают программу А, 14% сотрудников — программу В и 7% сотрудников — программу С. Остальные сотрудники отказываются принимать участие в предлагаемых пенсионных программах. а) Какой процент сотрудников выберет программу А через большой промежуток времени? Ответьте на этот же вопрос для программ В и С. б) За сколько (в среднем) лет сотрудник решается на участие в пенсионной программе? Биологические науки 49. Медицина. Пациенты, отказавшиеся от операции, подвергаются интенсивной терапии (сокращенно — ИТ) до тех пор, пока их состояние не стабилизируется. Далее их переводят в кардиологическое отделение (сокращенно — КО), где они остаются до окончания пребывания в больнице. Согласно записям в типичной больнице оказалось, что ежедневно умирает 2% пациентов, проходивших курс ИТ, 52% таких больных переводятся в КО, а оставшиеся пациенты продолжают лечение в отделении ИТ. Впоследствии из-за осложнений 4% пациентов КО переходят в отделение ИТ, 1% пациентов умирает, 22% выписываются из больницы, а остальные и далее продолжают оставаться в кардиологическом отделении. а) Какой процент пациентов отделения ИТ выписывается из больницы по истечении большого промежутка времени?
726 Часть II. Конечная математика б) Какой процент пациентов КО умирает по истечении большого промежутка времени? в) Сколько дней (в среднем) пациент отделения ИТ проводит в больнице? 50. Медицина. Рассмотренная в задаче 49 модель применима и к категории пациентов, которым заменяют клапан сердца. Оказывается, что ежедневно умирают 2% таких пациентов, лежавших в отделении ИТ, 60% больных этого отделения переводятся в КО, оставшиеся пациенты продолжают лечение в отделении ИТ. Затем из-за осложнений 5% пациентов КО переходят в отделение ИТ, 1% пациентов умирает, 19% выписываются из больницы, остальные и далее продолжают оставаться в КО. а) Какой процент пациентов КО выписывается из больницы по истечении большого промежутка времени? б) Какой процент пациентов отделения ИТ умирает по истечении большого промежутка времени? в) Сколько (в среднем) дней пациент КО проводит в больнице? Социальные науки 51. Поведение животных. Крысу помещают в лабиринт, изображенный на рис. 7.8. В исходном состоянии она находится в комнате F или В. Перемещаясь по помещению лабиринта, крыса со временем попадает в комнату L или R, где находит пищу. Считается, что животное выбирает выход из исходной комнаты случайным образом и, попадая в комнату с пищей, уже не покидает ее. а) Какова вероятность того, что в течение большого промежутка времени крыса попадет из комнаты В в комнату R1 б) Сколько (в среднем) раз крыса покинет комнату В, прежде чем найдет пищу? Рис. 7.8. Иллюстрация к задачам 50 и 51 52. Поведение животных. Повторите решение задачи 51, если выход из комнаты В в комнату R блокирован. Ключевые слова, основные обозначения и формулы 7.1. Свойства цепей Маркова. Стохастические процессы; диаграмма переходов; матрица вероятностей переходов; матрица начального распределения; матрица вероятностей исходных состояний; матрица исходного состояния; марковская цепь или процесс; матрица fc-ro состояния; рекурсивное определение матриц состояния; степени матриц переходов. Sk = Sk_-J> = S0Pk.
Глава 7. Цепи Маркова 727 7.2. Регулярные цепи Маркова, Стационарная матрица; регулярная матрица переходов; регулярная цепь Маркова; предельная матрица. SP = S; SP = S; Матрица Рк стремится к матрице Р 7.3. Поглощающие цепи Маркова. Поглощающее состояние; поглощающая цепь Маркова; стандартная форма; предельная матрица; фундаментальная матрица. Упражнения для повторения Выполните все упражнения этого обзорного раздела и сравните свои ответы с ответами, помещенными в конце книги. Ответы ко многим упражнениям на повторение приводятся вместе с номером раздела (курсивом), чтобы указать, где обсуждался каждый тип задач. Если вы не можете решить какую-либо задачу, повторите материал соответствующего раздела. 1. Ниже приведены матрица переходов Р и матрица исходного состояния So- Вычислите матрицы Si и S2 и объясните их смысл. А В А [0,6 0,4 В 0,2 0,8 So = [0,3 0,7] . В задачах 2-6 задана матрица переходов для цепи Маркова. Определите, какие из состояний будут поглощающими, и укажите тип цепи (регулярная, поглощающая или ни та, ни другая). 4. Р = А [0 1 В 1 о В о 0,3 А А [0 В 0,7 5. Р = В С В [0,7 0,3 ’ АВС А [0,8 0 0,2 А В О 1 А В о о 1 о 0 1 D 0 о 6. р = А В С D А Г1 0 0 0 В 0 1 о о С 0 0 0,3 0,7 D 0 0 0,6 0,4
728 Часть II. Конечная математика В задачах 7-10 постройте матрицы переходов по указанным диаграммам переходов. Определите, какие из состояний будут поглощающими, и укажите тип цепи (регулярная, поглощающая или ни та, и ни другая). Б 11. Цепь Маркова состоит из трех состояний: А, В и С. Вероятность перехода из состояния А в состояние В за одно испытание равна 0,2, из состояния Ав С — 0,5, из состояния В в А — 0,8, из состояния В в С — 0,2, из состояния С в Л — 0,1, из состояния С в В — 0,3. Постройте диаграмму переходов и найдите соответствующую матрицу переходов для этой цепи. 12. Задана матрица переходов А В 0,4 0,6 0,9 0,1 ’ Вычислите вероятности следующих событий. а) Переход из состояния А в состояние В за два испытания. б) Переход из состояния В в состояние А за три испытания. В задачах 13 и 14 решите уравнение SP = S для поиска стационарной матрицы S и предельной матрицы Р. 13. Р = А В А [0,4 0,6 В [о,2 0,8 * А ГО,4 14. Р = В 0,5 0 В 0,6 0,3 0,8 С о 0,2 0,2 С В задачах 15 и 16 найдите предельную матрицу для указанной стандартной формы матрицы переходов. Вычислите вероятности переходов из каждого непоглощающего состояния в поглощающее для больших промежутков времени и требуемое для этого перехода среднее количество испытаний.
Глава 7. Цепи Маркова 729 15. Р = В о С [0,3 В О 1 о,1 с о о 0,6 16. Р = А в С D А '1 0 0 0 В 0 1 0 0 С 0,1 0,5 0,2 0,2 D 0,1 0,1 0,4 0,4 ki В задачах 17-20, используя графическую утилиту, приближенно вычислите предельные матрицы для указанных матриц переходов. 17. Матрица Р из задачи 13. 18. Матрица Р из задачи 14. 19. Матрица Р из задачи 15. 20. Матрица Р из задачи 16. 21. Вычислите стандартную форму для поглощающей цепи Маркова со следующей матрицей переходов. А в С D А 0,6 0,1 0,2 0,1‘ В 0 1 0 0 С 0,3 0,2 0,3 0,2 D 0 0 0 1 В задачах 22 и 23 исследуйте поведение последовательности состояний на большом интервале времени при заданных матрицах переходов и исходного состояния. А В с А 0 1 0 Р = В 0 0 1 С 0,2 0,6 0,2 а) 50 = [0 0 1]. б) So = [0,5 0,3 0,2]. А В с А ’1 0 0 23. Р = В 0 1 0 С 0,2 0,6 0,2 a) So = [0 0 1]. б) So = [0,5 0,3 0,2]. * 24. Пусть Р — матрица переходов размером 2x2 для цепи Маркова. Может ли матрица Р быть регулярной, если два ее элемента равны 0? Аргументируйте ответ. *25. Пусть Р — матрица переходов размером 3x3 для цепи Маркова. Может ли матрица Р быть регулярной, если три ее элемента равны нулю? Четыре элемента равны 0? Аргументируйте ответ. В 26. В красной урне лежат два красных шара, один синий и один зеленый. В синей урне лежит один красный шар, три синих и один зеленый. В зеленой урне лежат шесть красных шаров, три синих и один зеленый. Из урны вынимают шар, определяют его цвет и возвращают обратно в ту же урну. Следующий шар вынимается из урны с тем же цветом, что и у ранее вытянутого шара. Таким
730 Часть II. Конечная математика образом, цепь Маркова состоит из трех состояний: шар вынимается из красной урны, из синей или зеленой. а) Постройте диаграмму переходов для этого процесса. б) Вычислите матрицу переходов Р. в) Укажите тип цепи (регулярная, поглощающая или ни та, и ни другая). г) Вычислите предельную матрицу Р, если она существует, и опишите поведение этого процесса на больших интервалах времени. 27. Повторите решение задачи 26, если из красной урны удалены зеленые шары. *28. Покажите, что матрица S = [ж у z 0], где 0 х 1, 0 з/ С 1, nx + y + z = l, является стационарной матрицей для следующей матрицы переходов. А в с D А ’1 0 0 0 Р-В 0 1 0 0 р~с 0 0 1 0 D о,1 0,3 0,4 0,2 Обсудите возможность обобщения этого результата на случай произвольной поглощающей цепи Маркова с тремя поглощающими состояниями и двумя непоглощающими. В задачах 29-35 приведите пример цепи Маркова, которой соответствуют указанные свойства, или объясните, почему такая цепь не существует. * 29. Регулярная цепь Маркова с поглощающим состоянием. * 30. Регулярная поглощающая цепь Маркова. *31. Регулярная цепь Маркова с двумя различными стационарными матрицами. * 32. Поглощающая цепь Маркова с двумя различными стационарными матрицами. * 33. Цепь Маркова, у которой нет предельной матрицы. * 34. Регулярная цепь Маркова, у которой нет предельной матрицы. * 35. Поглощающая цепь Маркова, у которой нет предельной матрицы. zx В задачах 36 и 37 используйте графическую утилиту для вычисления предельной матрицы (если она существует) для приведенной матрицы переходов с точностью до трех десятичных знаков. А В С D А В с D А 0,2 0,3 0,1 0,4‘ А 0,1 0 0,3 0,6' „ п В 0 0 1 0 л В 0,2 0,4 0,1 0,3 36 Р = С 0 0,8 0 0,2 37.Р = С 0,3 0,5 0 0,2 D 0 0 1 0 D 0,9 0,1 0 0
Глава 7. Цепи Маркова 731 Применение математики Экономика и бизнес 38. Изменение предпочтений. Продукция компании X занимает 20% рынка. Маркетинговая отдел определил, что потребитель, пользующийся продукцией X, с вероятностью 0,7 будет продолжать его покупать. С другой стороны, потребитель, который не пользуется продукцией X (обозначим его символом X'), с вероятностью 0,5 в следующий раз остановит свой выбор именно на ней. а) Постройте диаграмму переходов. б) Вычислите матрицу переходов. в) Запишите матрицу исходного состояния. г) Вычислите матрицу первого состояния и определите ее смысл. д) Вычислите стационарную матрицу. е) Какую долю рынка будет охватывать продукция X в отдаленном будущем, если считать, что матрица переходов со временем не меняется? 39. Продажа оборудования. С использованием недавних технологических разработок были спроектированы три модели новых фрезеровальных станков — А, В и С. Учитывая необходимость полного переоборудования и начальные цены на товар, компании-потребители выбирают только одну модель станка и в будущем не переходят на какую-либо другую. Ежегодно 6% компаний покупают станки модели А, 8% компаний — станки модели В и 11% компаний — станки модели С. Остальные продолжают работать со старым оборудованием. а) Какой процент рынка будет занимать каждая из моделей станков через большой промежуток времени? б) За сколько (в среднем) лет компания, покупающая станки, обновляет парк оборудования? 40. Исследование рынка. В табл. 7.5 приведен процент домовладельцев, у которых есть видеомагнитофон (в разные годы). WWW Таблица 7.5. Количество домовладельцев, у которых есть видеомагнитофон Год Доля, % 1990 68^6 1995 81,0 2000 85,1 Ниже приведена матрица переходов, которая моделирует эти данные. Символ V обозначает владельцев видеомагнитофонов. Через 5 лет V V' V Го,91 0,091 D Текущим год , =Р
732 Часть II. Конечная математика а) Пусть So = [0,686 0,314]. Вычислите матрицы Si и S2 (вычислите матрицы точно, а затем округлите результат до трех десятичных знаков). б) Постройте новую таблицу согласно полученным в п. а результатам и сравните выводы с табл. 7.5. в) Какой процент домовладельцев будет располагать видеомагнитофонами в отдаленном будущем (согласно приведенной матрице переходов)? 41. WWW Обучение сотрудников. Чтобы стать членом общества страхования претендент должен сдать десять экзаменов членам общества. Успешная сдача первых двух экзаменов является предпосылкой для получения работы в статистическом отделе крупной страховой компании. Ежегодно 15% учеников сдают следующие три экзамена и становятся ассоциированными членами общества, 5% покидают компанию (больше не возвращаясь), а остальные продолжают обучение. Ежегодно 17% ассоциированных членов сдают оставшиеся пять экзаменов и становятся членами общества страхования, 3% покидают компанию (больше не возвращаясь), а остальные продолжают оставаться ассоциированными членами. а) Какая часть учеников (в процентах) в отдаленном будущем станут членами общества? б) Какой процент ассоциированных членов в отдаленном будущем покинет компанию? в) За сколько лет (в среднем) ученик становится членом общества или исключается из программы? Биологические науки 42. Генетика. Некоторое растение может иметь красный, розовый или белый цветки в соответствии с генотипами RR, RW и WW. Если каждый из этих генотипов скрещивается с генотипом красных цветков, то матрица переходов будет иметь такой вид. Текущее поколение Следующее поколение Красный Розовый Белый Красный 10 0 Розовый 0,5 0,5 0 Белый 0 10 Пусть каждое поколение скрещивается только с растениями, имеющими красные цветки. Покажите, что все цветки, в конечном счете, будут красными (найдите предельную матрицу). Социальные науки 43. Курение. В табл. 7.6 приведены данные о количестве курящих совершеннолетних граждан Соединенных Штатов для ряда лет.
Глава 7. Цепи Маркова 733 WWW Таблица 7.6. Количество курящих совершеннолетних граждан Соединенных Штатов Год Доля, % 1985 30,1 1990 25,4 1995 24,7 Ниже приведена матрица переходов, моделирующая эти данные. Символ S обозначает курящих граждан. Следующий пятилетний период S S' S Го,38 0,62 Текущий пятилетнии период 5/ g 20 0 80 а) Пусть So = [0,301 0,699]. Вычислите матрицы Si и S2 (сначала вычислите матрицы точно, а затем округлите результат до трех десятичных знаков). б) Постройте новую таблицу согласно полученным в п. а результатам и сравните выводы с табл. 7.6. в) Какой процент совершеннолетнего населения Соединенных Штатов будет курящим в отдаленном будущем (согласно приведенной матрице переходов)? Домашнее задание 7.1. Социальная мобильность Правительство развивающейся страны разделяет жителей на основе годового дохода на три категории: низшую (обозначим L), среднюю (обозначим М) и высшую (обозначим U). Моделью перемещения граждан между этими тремя категориями является цепь Маркова со следующей матрицей переходов. L М и L 0,9 о,1 0 М 0,15 0,8 0,05 и 0 0,1 0,9 Так, например, каждый год 10% жителей переходят из низшей в среднюю категорию, а 5% жителей — из средней в высшую и т.д. 1. Пусть на начало исследований 75% жителей находятся в низшей категории, 20% — в средней и 5% — в высшей. Дайте прогноз, сколько жителей будет в каждой из категорий через 1, 2, 3 года. 2. Объясните, почему матрица Р — регулярна и найдите стационарную и предельную матрицы. Одной из целей правительства является устранение среднего класса. По этой причине перед министрами экономики, образования и социального обеспечения ставится задача проводить политику, в результате которой уменьшится количество
734 Часть II. Конечная математика переходов из низшего в средний класс. Иначе говоря, следует уменьшить число а в следующей матрице переходов. L м и L 1 — а а 0 Р' = М 0,15 0,8 0,05 и О 0,1 0,9 3. Вычислите предельную матрицу для матрицы переходов Р', если а = 0,2 или а = 0,3. 4. Найдите такое значение числа для а, при котором процент жителей среднего класса через большой промежуток времени будет в три раза меньше процента жителей низшего класса. 5. Можно ли полностью устранить низший класс путем увеличения параметра а? Аргументируйте ответ. Домашнее задание 7.2. Разорение игрока zl Задача об игре со случайным исходом между двумя игроками, которая продолжается до тех пор, пока один из них не разорится, часто называется задачей о разорении игрока (gambler’s ruin problem). Допустим, например, что двое людей, которые вместе располагают четырьмя долларами, играют в простую игру с подбрасыванием монеты. Подбрасывается монета. Если выпадает орел, то игрок А платит игроку В один доллар. Если выпадает решка, то игрок В платит игроку А один доллар. Игра продолжается до тех пор, пока один из игроков не завладеет всеми деньгами, а второй, соответственно, не разорится, откуда и произошло название “разорение игрока”. Эту игру можно рассматривать как поглощающую цепь Маркова, где состояниями могут служить суммы денег, которыми располагает игрок А. Так, возможными состояниями будут 0, 1, 2, 3 и 4 доллара. Игра заканчивается, когда у игрока А не будет ни одного доллара или будет четыре доллара, т.е. у этой цепи есть два поглощающих состояния. Ниже приведена диаграмма и матрица переходов. 1. Вычислите фундаментальную матрицу F и предельную матрицу Р. Какова вероятность победы игрока А, если в начале игры он располагает суммой в один, два или три доллара? Сколько (в среднем) раз подбрасывают монету до тех пор, пока игрок А, в самом начале располагавший одним долларом, не победит? Ответьте на этот вопрос, если у игрока А было два доллара или три доллара.
Глава 7. Цепи Маркова 735 2. Повторите решение задачи 1, если игроки в сумме обладают пятью долларами. Постройте диаграмму переходов и найдите стандартную форму матрицы переходов. Рассмотрите все возможные суммы, которые могут быть у игрока А в начале игры. 3. Колесо рулетки в казино Невады разделено на 38 одинаковых секций, которые пронумерованы числами 00, 0, 1,2,..., 36. Сектора от 1 до 36 поровну разделены на красные и черные. Игрок, поставивший один доллар на черное, выигрывает один доллар (и возвращает ставку), если шарик остановится на черном. В противном случае (шарик останавливается на красном, на 0 или 00) игрок теряет один доллар. У игрока есть три доллара, и он намерен делать ставки по одному доллару на черное до тех пор, пока не удвоит свою сумму или не проиграет. Определите вероятность проигрыша игрока. Вычислите среднее количество ставок, которые делаются до проигрыша игрока или до удвоения его средств. Решите задачу с помощью цепей Маркова.
Ответы к упражнениям Глава 1 Практикум 1.1 1. Функция. 3. Не является функцией. 5. Функция. 7. Функция. 9. Не является функцией. 11. Функция. 13. 1. 15. -5. 17. 15. 19. -3. 21. 3. 23. 7. 25. 7. 27. -45. 29. 0. 31. у = 0. 33. у = 4. 35. х = -5, 0, 4. 37. х = -6. 39. Все действительные числа. 41. Все действительные числа, за исключением —4. 43. Все действительные числа, за исключением —4 и 1. 45. х 7. 47. х < 7. 49. /(2) = 0, причем 0 — это число, следовательно, значение /(2) существует. С другой стороны, значение /(3) не определено, поскольку знаменатель равен нулю, следовательно, значение /(3) не существует. 51. д(х) = 2х3 — 5. 53. G(z) = 2у/х — х2.
Ответы к упражнениям 737 55. Функция f умножает элемент из области определения на число 2, а затем вычитает из результата число 3. 57. Функция F умножает куб элемента из области определения на число 3, а затем вычитает из результата два квадратных корня этого элемента. 59. Функция с областью определения R. 61. Функция с областью определения R. 63. Не является функцией. Например, при х = 1, у = ±3. 65. Функция, областью определения которой является вся числовая ось, за исключением числа х = 4. 67. Не является функцией. Например, при х = 4, у = ±3. 69. 4. 73. 4. 77. Зо,2 3ah 4~ h2. 71. h — 1. 75. 8а + 4h - 7. 81. P(w) = 2w 4 , w > 0. 83. S(Z) = Z(50 - Z), 0 I 50. 85. 54 долл., 42 долл. 87. a) R(x) = (75 - 3x)x, 1 x 20. p(x), долл. 100 - 1 4 8 12 16 20 72 252 408 468 432 300 в) R(x), млн. долл. 500 400 300 200 100 0 -j- 1—I н♦ X, млн. шт. $ 10 15 20
738 Ответы к упражнениям 89. а) Р(х) = 59х - Зх2 - 125, 1 < х 20. б) X, МЛН. ШТ. R(x), млн. долл. 1 4 8 12 16 20 -69 63 155 151 51 -145 в) Р(х), млн. долл. 200-- 100- / 100 X, млн. шт. 100 -100 -200 91. a) V(z) = х(8 - 2х)(12 - 2ж). б) 0 х 4. 7 X, дюймы V(х), куб. дюймы 1 2 3 60 64 36 г) И(х) 75М 50 и/ 50 0 01 1 2 3 V 93. а) Из графика следует, что в окрестности числа 2 есть число х, в котором объем равен 65. Из таблицы следует, что с точностью до одного десятичного знака это число х равно 1,9. 1,8 1,9 2 У(ж) 66,5 65,4 64 б) Из результатов вычислений следует, что с точностью до двух десятичных знаков это число х равно 1,93. X Yi 1.9 1.91 № 1.94 1.95 1.96 65.436 65.307 65.176 65.04 64.902 64.76 64.614 x=i.9; 3 *7 Р) 'll) 95. v = — ; 1,9032 см/с. 15 + w
Ответы к упражнениям 739 Практикум 1.2 1. Область определения: все действительные числа; область значений: все действительные числа. 3. Область определения: [0, оо); область значений: (—оо,0]. 5. Область определения: все действительные числа; область значений: [0, оо). 7. Область определения: все действительные числа; область значений: все действительные числа. 21. График функции д(х) = — |х + 3| получается из графика функции у = |я| с помощью отражения относительно оси х и сдвига на три единицы влево.
740 Ответы к упражнениям 23. График функции /(я) = (х — 4)2 — 3 получается из графика функции у = х2 с помощью сдвига на четыре единицы вправо и три единицы вниз. Лх) 25. График функции / (ж) = 1—у/х получается из графика функции у — у/х, отраженного относительно оси х и сдвинутого на семь единиц влево. Лх) 27. График функции h(x) = -зи получается из графика функции у = |х|, отраженного относительно оси х и растянутого с коэффициентом 3. й(х) 10 29. График базисной функции у — х2 сдвинут на две единицы влево и на три единицы вниз. Уравнение: у = (х + 2)2 — 3. 31. График базисной функции у = х2 отражен относительно оси х, сдвинут на три единицы вправо и на две единицы вверх. Уравнение: у = 2 — (ж — З)2. 33. График базисной функции у = у/х отражен относительно оси х и сдвинут на четыре единицы вверх. Уравнение: у = 4 — у/х. 35. График базисной функции у = х3 сдвинут на две единицы влево и на одну единицу вниз. Уравнение: у = (х + 2)3 — 1. 37. д(х) = у/х — 2 — 3. 39. д(х) = — + 3|. 41. д(х) = — (х — 2)3 — 1.
Ответы к упражнениям 741 49. График базисной функции у = |х| отражен относительно оси х и сжат с коэффициентом 0,5. Уравнение: у = —0,5 |х|. 51. График базисной функции у = х2 отражен относительно оси х и растянут с коэффициентом 2. Уравнение: у = — 2х2. 53. График базисной функции у = отражен относительно оси х и растянут с коэффициентом 3. Уравнение: у = —З^х. 55. Порядок действий на результат не влияет. 57. Порядок действий может повлиять на результат. 59. Порядок действий может повлиять на результат. 61. а) График базисной функции у = у/х отражен относительно оси х, растянут с коэффициентом 4 и сдвинут на 115 единиц вверх. 63. а) График базисной функции у = х3 сжат с коэффициентом 0,00048 и сдвинут на 500 единиц вправо и 60 000 вверх.
742 Ответы к упражнениям 65. а) 8,5 + 0,065а:, если 0 х 700, —9 + 0,09а:, если х > 700. б) '0,035, 67. а) Т(х) = 625а: - 825, б) если 0 х < 30 000, если 30 000 < х 60 000, если х > 60000. в) 1657 долл., 3570 долл. 69. а) График базисной функции у = х растянут с коэффициентом 5,5 и сдвинут на
Ответы к упражнениям 743 71. а) График базисной функции у = у/х растянут с коэффициентом 7,08. Практикум 1.3 1. а. 3. в, наклон равен нулю. 9. Наклон равен 3, координата пересечения графика с осью у равна 1. 11. Наклон равен —координата пересечения графика с осью у равна —6.
744 27. 29. 33. 37. 41. 45. 49. 51. 55. 57. 59. Ответы к упражнениям б) Координата пересечения графика с осью х: 3,5; координата пересечения графика с осью у: —4,2. в) 10 -10 10 -10 г) Координата пересечения графика с осью х: 3,5; координата пересечения графика с осью у: —4,2. д) х > 3,5; или (3,5; оо). х — 4, у = —3. 31. х = —1,5; у = —3,5. 3 35. у = -х + 1. з94 у = —4х + 5. У = 4,6. _5 4* 0. 5я + 4т/ = —14. Линейная функция. х = 5. Ни то, ни другое. 53. у = 5. Постоянная функция. Графики имеют одинаковые координаты пересечения с осью у, (0; 2). 43. Не определен. 47. 2х — Зу = —11. Линейная функция. б) Варьируя величину С, можно получить семейство параллельных линий, поскольку ее изменение влияет только на координаты точек пересечения графика с осью у. График функции д совпадает с графиком функции f при всех значениях переменной х, удовлетворяющих неравенству тх + b 0, и является отражением графика f относительно оси х при всех значениях переменной х, удовлетворяющих условию тх + b < 0. Функция д не является линейной ни на одном интервале.
Ответы к упражнениям 745 61. а) 130 долл., 220 долл. в) Наклон равен 6. Сумма денег, лежащих на счету, растет со скоростью 6 долл, в год. 63. а) ОД = 180х + 200. б) 2360 долл. 65. а) р(х), долл. 200< 01 5000 10005Х,ШТ' б) р(х) = -^ж + 200. в) р(3000) = 150 долл. г) Наклон равен — « —0,02. При увеличении спроса на одну единицу цена уменьшается на 0,02 долл. 67. а) Предложение: р = 0,4я — 0,9. Спрос: р = — 0,4а? + 6,42. б) 9,15 млн. бушелей при цене 2,76 долл, за бушель. 69. а) X 0 1 2 3 4 Объем продаж 16,7 19,8 23,6 26,9 32,7 /(®) 16,1 20,0 23,9 27,9 31,8
746 Ответы к упражнениям 6) у, млрд. долл. в) 55,22 млрд, долл., 74,77 млрд. долл. 71. а) /(а;) = - 1,25а;+ 97. б) 63,25%, 30,75%. в) В середине 1997 г. 73. 0,2а: + 0,1 у = 20. 75. а) 64 г., 35 г. Практикум 1.4 1. (х - 2)2 - 1. 3. (а: - З)2 + 5.
Ответы к упражнениям 747 5. График функции /(ж) получается из графика функции у = х2 путем сдвига на две единицы вправо и одну единицу вниз. 7. График функции т(х) получается из графика функции у = х2 путем отражения относительно оси х, сдвига на три единицы вправо и пять единиц вверх. 9. а) т. б) 9- в) /• г) п. 11. а) Координаты пересечения графика с осью х: 1,3. Координаты пересечения графика с осью у\ —3. б) Вершина: (2,1). в) Максимум: 1. г) Область значений: у 1, или (-00,1]- д) Интервал возрастания: х 2, или (—оо, 2]. е) Интервал убывания: ж > 2, или [2, оо). 13. а) Координаты пересечения графика с осью х: —3, —1. Координаты пересечения графика с осью у: 3. б) Вершина: (-2,-1). в) Минимум: —1. г) Область значений: у —1, или (—1, оо]. д) Интервал возрастания: х —2, или [—2, оо). е) Интервал убывания: х —2, или (—оо, —2]. 15. а) Координаты пересечения графика с осью х: 3 ± \/2. Координата пересечения графика с осью у\ —7. б) Вершина: (3,2). в) Максимум: 2. г) Область значений: у ^2, или (-оо,2]. 17. а) Координаты пересечения графика с осью х: — 1 ± у/2. Координата пересечения графика с осью у: — 1. б) Вершина: (-1,-2). в) Минимум: —2. г) Область значений: у —2, или (—2, оо]. 19. у = — [ж — (—2)2] -I- 5, или у = —(х + 2)2 + 5. 21. у = (ж - I)2 - 3. 23. Стандартная форма: (ж — 4)2 — 4. а) Координаты пересечения графика с осью х: 2 и 6, координата пересечения с осью у: 12. б) Вершина: (4,-4). в) Минимум: —4. г) Область значений: у —4, или [—4, оо). 25. Стандартная форма: —4(х — 2)2 -h 1. а) Координаты пересечения графика с осью х: 1,5 и 2,5; координата пересечения с осью у: —15.
748 Ответы к упражнениям б) Вершина: (2,1). в) Максимум: 1. г) Область значений: у < 1, или (—оо, 1]. 27. Стандартная форма: 0,5(ж — 2)2 + 3. а) Координаты пересечения графика с осью х: нет, координата пересечения с осью у\ 5. б) Вершина: (2,3). в) Минимум: 3. г) Область значений: у 3, или [3, оо). 29. а) -4,87; 8,21. б) -3,44; 6,78. в) Нет решения. 31. Вершина параболы лежит на оси х. 33. д(х) = 0,25(я - З)2 - 9,25. а) Координаты пересечения графика с осью х: —3,08; 9,08. Координата пересечения графика с осью у: —7. б) Вершина: (3;—9,25). в) Минимум: —9,25. г) Область значений: у —9,25, или [—9,25; оо). 35. /(ж) = —0,12(ж - 4)2 + 3,12. а) Координаты пересечения графика с осью х: —1,1; 9,1. Координата пересечения графика с осью у: 1,2. б) Вершина: (4; 3,12). в) Максимум: 3,12. г) Область значений: у 3,12, или (—оо; 3,12]. 37. х = -5,37; 0,37. 39. -1,37 < х < 2,16. 41. х —0,74 или х 4,19. 43. Оси: х = 2, вершина: (2; 4), область значений: у 4, или [4, оо). С осью х пересечений нет. б) 1,64; 7,61. в) 1,64 < х < 7,61. г) 0 < х < 1,64 или 7,61 < х 10.
Ответы к упражнениям 749 47. а) у б) 1,10; 5,57. в) 1,10 < х < 5,57. г) 0 < х < 1,10 или 5,57 < х 8. 49. Например, /(ж) = х2 + 1 и д(х) = — (х — 4)2 - 1. Оба графика не пересекают ось х. 51* а) ж, фунт/кв. дюйм 28 30 32 34 36 Пробег, тыс. миль 45 52 55 51 47 /(ж) 45,3 51,8 54,2 52,4 46,5 б) Дх), Лж. шеа бо| 50 40 30 20 10 х, шшДЬи. ггДПп Зо в) /(31) = 53,50 тыс. миль; /(35) = 49,95 тыс. миль. 53. a) R(x), jeu. llce. 500 400 300 200 100 б) 12 500 000 микросхем. 468 750 000 долл. в) 37,50 долл.
750 Ответы к упражнениям б) 2 415 000 микросхем и 17 251 000 микросхем. в) Убытки: 1 х < 2,415 или 17,251 < х 20. Прибыль: 2,415 < х < 17,251. 57. а) Р(х) = 59х - Зя2 - 125. в) Точка пересечения с осями и точка безубыточности: 2 415 000 микросхем и 17 251000 микросхем. д) Максимальный доход равен 165 083 000 при производстве 9 833000 микросхем. Эта величина намного меньше максимального дохода, равного 468 750 000. 59. х = 0,14 см. Упражнения для повторения 2. а) Не является функцией. б) Функция. в) Функция. г) Не является функцией. (7. У) 3. а) —2. б) -8. в) 0. г) Не определена. 4. а) у = 4. б) х — 0.
Ответы к упражнениям 751 в) У = 1. г) X = — 1 или 1. Д) У = -2. г) у (1.2) 5 -5 6. а) п. б) д. в) т; наклон равен нулю. г) /; наклон не определен. У = ~\х + 6. (1.3) 8. Вертикальная линия: х = —6, горизонтальная линия: у = 5. (7.3)
752 Ответы к упражнениям 9. Координата пересечения графика с осью х = 9. Координата пересечения графика с осью у = —6. Наклон = |. (1.3) 10. f(x) = — (х — 2)2 + 4. График функции f(x) получается из графика функции у = х2 с помощью отражения относительно оси х, сдвига на две единицы вправо и четыре единицы вверх. И. а) д. б) т. в) п. г) f (12, 1.4) 12. а) Координаты пересечения графика с осью х: —4; 0. Координата пересечения графика с осью у: 0. б) Вершина: (—2; —4). в) Минимум: —4. г) Область значений: у —4, или [—4, оо). д) Возрастает на интервале [—2, оо). е) Убывает на интервале (—оо, — 2] (1.4) 13. Линейные функции: 1, 3, 5 и 6, постоянная функция: 4. (1.3) 14. а) Все действительные числа, за исключением чисел х = — 2 и 3. б) х < 5. (1.1) 15. Функция д умножает элемент области определения на 2, а затем извлекает из результата три квадратных корня этого элемента. (1.1) 16. Стандартный вид: 4 (х + |)2 — 4. а) Координаты пересечения графика с осью х: — |. Координата пересечения гра¬ фика с осью у\ —3. б) Вершина: (—4). в) Минимум: —4. г) Область значений: у —4, или [—4, оо).
Ответы к упражнениям 753 17. График функции х = —3 является вертикальной линией, пересекающей ось х в точке —3, а график функции у — 2 является горизонтальной линией, пересекающей ось у в точке 2. (7.3) 18. а) у б) Уравнение f(x) = 3 имеет одно решение. Уравнение f(x) = 2 имеет два решения. Уравнение f(x) =s 1 имеет три решения. в) f(x) = 3: х = —4,28. /(ж) = 2: х = -4,19; 1,19. /(х) = 1: х = -4,10; 0,35; 1,75. (7.7) 19. —2. (7.7) 20. 2а + h - 3. (7.7) 21. График функции т получается из графика функции у = |х| с помощью отражения относительно оси х и сдвига на четыре единицы вправо. (7.2) 22. График функции д получается из графика функции у = х3 с помощью сжатия с коэффициентом 0,3 и сдвига на три единицы вправо. (7.2) 23. График функции у = х2 растягивается с коэффициентом 2, отражается относительно оси х и сдвигается на три единицы влево. Уравнение: у = —2(х + З)2. (7.2) 24. f (х) = 2 V^+3. (7.2)
754 Ответы к упражнениям б) у = 3. (7.3) 27. а) Зх + 2у = 1. б) у = 5. в) х = —2. (73) 28. у = —(х —4)2 + 3. (1.2, 1.4) 29. f(x) = —0,4(х - 4)2 + 7,6. а) Координаты пересечения графика с осью х: —0,4; 8,4. Координата пересечения графика с осью у: 1,2. б) Вершина: (4,0; 7,6). в) Максимум: 7,6. г) Область значений: х < 7,6, или (—сю; 7,6]. (7.4) а) Координаты пересечения графика с осью х: —0,4; 8,4. Координата пересечения графика с осью у: 1,2. б) Вершина: (4,0; 7,6). в) Максимум: 7,6. г) Область значений: х < 7,6; или (—сю; 7,6] (7.4) 31. График функции у = у/х растягивается с коэффициентом 2, отражается относительно оси х и сдвигается на одну единицу влево и одну единицу вниз. Уравнение: у = = -2^ТТ-1. (7.2) 32. Графики перпендикулярны друг другу. (Можно показать, что если наклоны двух линий являются взаимно обратными величинами со знаком минус, то эти линии взаимно перпендикулярны.) (7.3) 33. а) . 1 -. \/х -h h + у/х б) 7-^—ГГ (/ 2) х (х + h) 34. G(x) = 0,3(х + 2)2 - 8,1.
Ответы к упражнениям 755 а) Координаты пересечения графика с осью х: —7,2; 3,2. Координата пересечения графика с осью у: —6,9. б) Вершина: (-2;—8,1). в) Минимум: —8,1. г) Область значений: х —8,1, или [—8,1; сю). д) Убывает: (—оо, —2], возрастает: [—2, оо). (1.4) а) Координаты пересечения графика с осью х: —7,2; 3,2. Координата пересечения графика с осью у: —6,9. б) Вершина: (-2;—8,1). в) Минимум: —8,1. г) Область значений: х —8,1, или [—8,1; оо). д) Убывает: (—оо, —2], возрастает: [—2,оо). (1.4) 36. a) V(t) = — 1250* + 12 000,0 t 8. К(0, долл. О 4 б) У(5) = 5750 долл. (7.3) б) г = 0,1447, или 14,47 сложного годового процента. (7.7, 7.2) 38. а) 7? = 1,6С. б) 192 долл. в) 110 долл. г) Наклон равен 1,6. Он представляет изменение розничной цены при изменении себестоимости на единицу. (7.3)
756 Ответы к упражнениям 39. а) X 0 5 10 15 20 Потребление куриных яиц, шт. 271 255 233 236 256 /(®) 274 248,5 237 239,5 256 '3; 40. a) S (а:) = < 0,057а: + 1,86; 0,0346а: + 6,34; (0,0217а: +19,24; если 0 < х < 20; если 20 < х < 200; если 200 < х < 1000; если х > 1000. б) 5(х),долл. 41. а) Предложение: р = 0,04а: — 14,5. Спрос: р = —0,02а: + 11. б) 425 млн. бушелей по цене 2,50 долл, за бушель. (7J) 42. а) С = 84000 + 15а:; R = 50а:. R(x) С(х), долл. б) R = С при х = 2400 единиц; R<C при 1 < х < 2400; R> С при х > 2400. в) См. п. б. (1.3)
Ответы к упражнениям 757 43. а) 7?(х), С(х), тыс. долл. б) R = С при х = 4,686 тыс. единиц и х = 27,314 тыс. единиц; R < С при Ю < 4,686 или 27,214 < х 40; R > С при 4,686 < х 27,314. в) Максимальный доход равен 500 тыс. долл. (500 000 долл.) достигается при выпуске 20 тыс. единиц. В этом случае оптовая цена равна р(20) = 25 долл. (73, 1.4) 44. а) Р(х) = 7?(х) - С(х) = х(50 - 1,25а;) - (160 + 10х). б) Р = 0 при х — 4,686 тыс. единиц и х = 27,314 тыс. единиц; R < 0 при 1 < х < < 4,686 или 27,314 < х 40; Р > 0 при 4,686 < х < 27,314. в) Максимальная прибыль, равная 160000 долл. (160000 долл.), достигается при выпуске 16 тыс. единиц. В этом случае оптовая цена равна р(16) = 30 долл. (1.4) 3 45. a) S(x) = ~2х2 +420я. б) Область определения: 0 х 280. в) 5(х),кв. футы х, футы г) Уравнение S(x) = 25000 имеет два решения: одно — в окрестности числа 90, а другое — в окрестности числа 190. д) 86 футов, 194 фута. е) Максимальная область, равная 29 400 кв. футов, достигается при х = 140 футов и у = 105 футов. (1.4) 46. а) Р(х) = 15я + 20. б) 95.
758 Ответы к упражнениям г) 15. (1.4) 47. а) 1 фунт, 3 фунта. Глава 2 Практикум 2.1 1. а) 2. б) 1. в) 2. г) 0. д) 1. е) 1. 3. а) 5. б) 4. в) 5. г) 1. Д) 1. е) 1. 5. а) 6. б) 5. в) 6. г) 0. д) 1. е) 1. 7. а) 3. б) 4.
Ответы к упражнениям 759 в) Отрицательный. 9. а) 2. б) 5. в) Отрицательный. 11. а) 0. б) 1. в) Отрицательный. 13. а) 5. б) 6. в) Положительный. 15. а) Координата пересечения графика с осью х: —2, координата пересечения графика с осью у: —1. б) Область определения: все действительные числа, кроме числа 2. в) Вертикальная асимптота: х = 2, горизонтальная асимптота: у = 1. Г) Лх) 17. а) Координата пересечения графика с осью х: 0, координата пересечения графика с осью у: 0. б) Область определения: все действительные числа, кроме числа —2. в) Вертикальная асимптота: х = — 2, горизонтальная асимптота: у = 3.
760 Ответы к упражнениям 19. а) Координата пересечения графика с осью х: 2, координата пересечения графика с осью у: —1. б) Область определения: все действительные числа, кроме числа 4. в) Вертикальная асимптота: х = 4, горизонтальная асимптота: у = —2. -10 21. График станет более похожим на график функции у = 2я4. 23. График станет более похожим на график функции у = —х5. -500 у = 2/ -500 у = 2х4-5х2 + х + 2
Ответы к упражнениям 761 У = _/ у = —х5 + 4л3 — 4х + 1 29. [-9/2;9/2]. - 1,84;0,42; 1,92. 31. [-5; 5], -2,50; -1,22; 0,22; 1,50. 33. [-16; 16],-0,92; 1,30; 11,38. 37. а) Координата пересечения графика с осью х: 0, координата пересечения графика с осью у: 0. б) Вертикальные асимптоты: х = —2, х = 3; горизонтальная асимптота: у = 2. в) Лх) 39. а) Координаты пересечения графика с осью х: ±\/3, координата пересечения графика О с осью у: б) Вертикальные асимптоты: х = —3, х = 3; горизонтальная асимптота: у = —2.
762 Ответы к упражнениям 41. а) Координата пересечения графика с осью х: 0, координата пересечения графика с осью у: 0. б) Вертикальные асимптоты: х = —3, х = 2; горизонтальная асимптота: у = 0. В) Л*) 43. f{x) = х2 — х — 2. 45. f(x) = 4х — х3. 47. а) С(х) = 180х 4- 200. . 180x4-200 б) С(х) = .
Ответы к упражнениям 763 г) 180 долл, за сноуборд. ла ' X 2500 + 175п + 25п2 49. а) С (х) = в) 10 лет; 675,00 долл, в год. г) 1 500 ,, ч Л/ х 0,00048(х - 500)3 + 60000 51. а) С (х) = о в) 750 услуг в месяц, 90 долл, за услугу. LinReg ч=ах+Ь а=". 0506666667 Ь=19.96666667 QuadReg y=ax2+bx+c 3=2.4444444е"4 Ь=".0065555556 0=2.086111111 б) х = 195; р = 10,09 долл. 55. а) CubicReg б) 2215,4 млрд. долл. 57. а) 0,06 см/с.
764 Ответы к упражнениям б) у(х),см/с CubicReg 59. а) b=~.0078831169 с=.2399206349 d=8.48030303 б) 7,0; 5,8. Практикум 2.2 1. а) к. б) д. в) h. г) /•
Ответы к упражнениям 765 15. 46ly. 17. е. 19. 8e3,6t. 21. График функции д является отражением графика функции f относительно оси х. 23. График функции д является графиком функции /, сдвинутым на одну единицу влево. 25. График функции д является графиком функции /, сдвинутым на одну единицу вверх. 27. График функции д является графиком функции /, растянутым с коэффициентом 2 и сдвинутым на две единицы влево. в) у
766 Ответы к упражнениям 41. а = 1; —1. 45. х = —1;6. 49. х = 3. 43. х = 1. 47. х = 3. 51. х = —3; 0.
Ответы к упражнениям 767 55. х 100 Г' 57. х = 1,40. 59. х = -0,73. 61. а) 2633,56 долл. б) 7079,54 долл. 63. а) 11871,65 долл. б) 20427,93 долл. 65. 10 706 долл. 67. а) 10 715,44 долл. б) 10 720,72 долл. в) 10 703,65 долл. 69. 32 201,82 долл. 71. При неограниченном возрастании величины t число N стремится к числу 2. N, чел. 0 73. а) 1998: 1492000 долл.; 2010: 26781000. lExpfteg у=а*ЬЛ: 75. а) 10%. б) 1%. 77. а) N = 47,Зе0,116< млн. случаев заболевания. б) 2005: 106500000; 2010: 190300000.
768 Ответы к упражнениям в) С(х), млн. чел. 79. а) Р = 5,7e0,0114t. б) 6,8 млрд.; 8,5 млрд. 81. а) 22033000,000. ExpReg у=а*ЬЛх а=2.776782415 Ь=1.752754444 Практикум 2.3 1. 27 = З3. 5. 8 = 43/2. 9. log, 8= |. 13. 0. 17. 1. 21. -3. 25. logb р ~ logb Q- 29. logb Р ~ 1оёь 9 ~ 1оёь г ~ 1оёьs- 33. у = 2. 37. х = 2. 41. b = 100. 45. |logbN. 49. logb 50 - 0,2t logb 2. 3. 10° = 1. 7. log749 = 2. 11. logb A = u. 15. 1. 19. 3. 23. 3. 27. 5 logb L. 31. x = 9. 35. b = 10. 39. у - -2. 43. 5 logb x -3 logb У- 47. 2 logb x + | logb У- 51. logbF + tlogb(l + r).
Ответы к упражнениям 769 53. loge 100 - 0,01*. 57. 1 = 8. 61. Нет решений. 63. у 55. х = 2. 59. х = 7. 65. График функции у = log2(a; — 2) представляет собой график функции у — log2rr, сдвинутый на две единицы вправо. 67. Область определения: (—1, — оо). Область значений: все действительные числа. 69. а) 3,54743. б) -2,16032. в) 5,62629. г) -3,19704. 71. а) 13,4431. б) 0,0089. в) 16,0595. г) 0,1514. 73. 1,0792. 75. 1,4595. 77. 30,6589. 79. 18,3559. 81. Возрастание: (0, оо). у 83. Убывание: (0,1]. Возрастание: [1, оо). у
770 Ответы к упражнениям 85. Возрастание: (—2, оо). 87. Возрастание: (0,оо). у Ю; 89. Поскольку b° = 1 при всех допустимых значениях основания b (Ь > 0, b / 1). 91. у = С' 10°’8х. 95. 4 года. 93. х > у/х > 1пж при 1 < х 16. 97. 9,87 года; 9,80 года. 99. 6,758%. 101. а) 5373. LnReg y=a+blnx а=256.4659159 Ь= -24.03812068 б) 7220. LnReg y=a+blnx а= -127.8085281 b=20-01315349 105. 147,5 бушеля/акр. .nReg y=a+blnx а=-535.1958095 Ь=145.237116 107. 901 год. Упражнения для повторения 1. v = Iniz. (2.3) 2. у = logs. (2.3)
Ответы к упражнениям 3. М = eN. (2.3) 5. 52*. (2.2) 7. а; = 9. (2.3) 9. х = 4. (2.3) 11. х = 13,128. (2.3) 13. х = 0,318. (2.3) 14. 1) 3. 2) 2. 3) 3. 4) 1. 5) 1. 6) 1. (2.1) 15. 1) 4. 2) 3. 3) 4. 4) 0. 5) 1. 6) 1. (2.1) 16. 1) 2. 2) 3. 3) Положительный. (2.1) 17. 1) 3. 2) 4. 3) Отрицательный. (2.1) 771 4. и = 10". (2.3) 6. е2< (2.2) 8. х = 6. (2.3) 10. х = 2,157. (2.3) 12. х = 1273,503. (2.3) 18. 1) Координата пересечения графика с осью х: —4, координата пересечения графика с осью у: —2. 2) Все действительные числа, за исключением числа х = 2. 3) Вертикальная асимптота: х = 2, горизонтальная асимптота: у = 1. 4) Лх)
772 Ответы к упражнениям 19. 1) Координата пересечения графика с осью х: координата пересечения графика с осью у. —2. 2) Все действительные числа, за исключением числа х = — 2. 3) Вертикальная асимптота: х = — 2, горизонтальная асимптота: у = 3. -10 20. х = 8. (2.3) 22. х = 3. (2.2) 24. х = 0; |. (2.2) 26. х = (2.3) 28. х = 13,3113 (2.3) 30. х = 0,0097. (2.3) 32. х = 1,4650. (2.3) 34. х = 9,0065. (2.3) 36. 2,8074. (2.3) 38. Они очень похожи. (2.1) 21. х = 3. (2.2) 23. х = -1; 3. (2.2) 25. х = -2. (2.3) 27. х = 27. (2.3) 29. х = 158,7552. (2.3) 31. 1,4359. (2.3) 33. х = 92,1034. (2.3) 35. х = 2,1081. (2.3) 37. х = -1,0387. (2.3)
Ответы к упражнениям 773 40. х = -1,14; 6,78. (2.1) 42. 1 + 2ех - е~х. (2.2) 44. Возрастание: [—2; 4]. (2.2) 41. (-1,54; -0,79); (0,69; 0,99). (2.2, 2.3) 43. 2е~2х - 2. (2.2) 45. Убывание: [0, оо). (2.2) 46. Возрастание: (-1; 10] (2.3) 47. loglO’ и = ^2; Ine^ = 7Г и е1п^2 = л/2. (2.3) 48. х = 2. (2.3) 50. х = 1. (2.3) 49. х = 2. (2.3) 51. х = 300. (2.3)
774 Ответы к упражнениям 52. у = ce~5t. (2.3) 53. Если logj х = у, то Р — х, т.е. 1 = х при всех положительных действительных числах х, что невозможно. (2.3) 54. 6951,36 долл. (2.2) 55. 6947,68 долл. (2.2) 56. 16,7 года. (2.3) 57. 10,6 года. (2.3) ДО'г -к ЧОО 58. а) С(х) = 40я + 300; С (х) = . в) у = 40. г) 40 долл, за пару. _ х / ч 20ж3 - 360я2 + 2300гг - 1000 59. а) С (х) = в) 8,667 тыс. пациентов. г) 567 долл, на пациента. 60. а) 2833 набора. QuadReg y=ax2+bx+c а=5.9477212е"6 b=".1924018814 с=422.3467853 б) 4836. LinReg ч=ах+Ь а=.0387421907 Ь=-7.364689544 в) Равновесная цена: 131,59 долл, равновесное количество: 3587 наборов. (2.1). 61. а) 2000: 127 млн.; 2010: 3482 млн. (2.2)
Ответы к упражнениям 775 б) lExpReg у=а*ЬЛх а=4.597015793 Ь=1.393049401 N = 22t или N = 4‘. 15 дней. (2.2, 2.3) 62. а) б) 63. к = 0,00942; 489 футов. (2.2, 2.3) 64. а) 1996: 1724 млн. бушелей. 2010: 2426 млн. бушелей. (2.3) LnReg у=а+Ыпх а= -21796.9294 Ь=5153.244133 65. 23,4 года. (2.2, 2.3) 67. а) 2010: 886 млрд. долл. 66. 23,1 года. (2.2, 2.3) ExpReg y=a*b^x а=39.23474084 Ь=1.109502846 б) Средняя точка: 2004 г. Глава 3 5 3. 18%; — года. 7. г = 0,08, или 8%. 11. Р = 888,89 долл. 15. A/(l + rt). Практикум 3.1 1. 0,095; года. 6 5. I = 20 долл. 9. А = 112 долл. 13. г = I/Pt, 17. Графики являются линейными и пересекают ось у в точке 1000. Их наклоны равны 40, 80 и 120 соответственно. 19. 80 долл. 21. 9,23 долл. 23. 7685,00. 25. 10,125%. 27. 18%. 29. 1680 долл.; 36%. 31. 6,351%. 33. 991,01%. 35. 13,538%. 37. 21,833%. 39. 11,318%. 41. 112,275%. 43. 69,411%.
776 Ответы к упражнениям Практикум 3.2 1. А = 112,68 долл. 3. А = 3433,50 долл. 5. Р = 2419,99 долл. 7. Р = 7351,04 долл. 9. 0,75% в месяц. 11. 1,75% за квартал. 13. 9,6 сложного процента в месяц. 15. 9 сложных процентов за полгода. 17. а) 126,25 долл., 26,25 долл. б) 126,90 долл., 26,90 долл. в) 127,05 долл., 27,05 долл. 19. а) 5982,07 долл. б) 7157,03 долл. 21. Все три графика являются возрастающими, выпуклыми вверх и пересекают ось у в одной и той же точке. Чем больше рост стоимости, тем сильнее возрастает функция. Суммы, начисленные в конце восьмилетнего периода, равны 1376,40, 1892,46 и 2599,27 долл, соответственно. Период Проценты, долл. Сумма, долл. 0 1000,00 1 97,50 1097,50 2 107,01 1204,51 3 117,44 1321,95 4 128,89 1450,84 5 141,46 1592,29 6 155,25 1747,54 25. а) 6755,64 долл. б) 4563,87 долл. 27. а) 10,38%. б) 12,68%. 29. 5,5 года. 31. п « 12. 33. а) 7,25 года. б) 6 лет. 35. 16 267,11 долл. 37. 147830,80 долл. 39. 15,82 долл./кв. футов/мес. 41. 32 года. 43. 9 сложных процентов в месяц, поскольку ее эффективность равна 9,38%. Эффективность инвестиции под 9,3 сложного годового процента равна 9,3%. 45. а) В 1998 г. спустя 222 года после подписания инвестиция стоила бы 76139,26 долл, б) Если доход начисляется ежемесячно, ежедневно или непрерывно, она стоила бы 77409,05, 78033,73 или 78055,09 долл, соответственно.
Ответы к упражнениям 777 47. Два года и десять месяцев. 51. 3615 дней, 9 лет. 49. 328 792,05 долл. Годы Точная скорость роста Правило 72 6 12,2 12,0 7 10,4 10,3 8 9,1 9,0 9 8,0 8,0 10 7,2 7,2 11 6,5 6,5 12 5,9 6,0 55. 23 квартала. 57. Если срок инвестиции не превышает 11 лет, то, чтобы получить максимальный доход, следует выбрать инвестицию под 10 простых процентов, в противном случае необходимо предпочесть 7 сложных процентов. 59. 6,58%. 61. 6,96%. 63. 11917,40 года. 65. 5,99%. 67. а) 6,50%. б) 6,56%. в) 6,55%. 69. 13,44%. 71. 17,62%. Практикум 3.3 1. FV = 13435,19 долл. 3. FV = 60401,98 долл. 5. РМТ = 123,47 долл. 7. РМТ = 310,62 долл. 9. п = 17. 11. i = 0,09. 13. Сумма: 84895,40 долл., проценты: 24895,40 долл. 15. 20931,01 долл. 17. 667,43 долл. 19. 763,39 долл.
778 Ответы к упражнениям 21. „ Период Сумма, долл. Доход, долл. Баланс, долл. 1 1000,00 0,00 1000,00 2 1000,00 83,20 2083,20 3 1000,00 173,32 3256,52 4 1000,00 270,94 4527,46 5 1000,00 376,69 5904,15 23. Первый год: 50,76 долл., второй год: 168,09 долл., третий год: 296,42 долл. 25. 111050,77 долл. 27. 1308,75 долл. 29. а) 4,76%. б) 189,56 долл. 31. 33 месяца. 33. 7,77%. 35. 7,13% 37. После И ежеквартальных выплат. Практикум 3.4 1. PV = 3458,41 долл. 5. РМТ = 199,29 долл. 9. п = 29. 13. 35693,18 долл. 17. 69,58 долл., 839,84 долл. 3. PV = 4606,09 долл. 7. РМТ = 586,01 долл. 11. i = 0,029. 15. 11 241,81 долл., 1358,19 долл. 19. 314,72 долл., 17 319,68 долл. Номер выплаты Сумма выплаты, долл. Проценты, долл. Сокращение непогашенного остатка, долл. Непогашенный остаток, долл. 0 5000,00 1 706,29 140,00 566,29 4433,71 2 706,29 124,14 582,15 3851,56 3 706,29 107,84 598,45 3252,11 4 706,29 91,09 615,20 2637,91 5 706,29 73,86 632,43 2005,48 6 706,29 56,15 650,14 1355,34 7 706,29 37,95 668,34 687,00 8 706,24 19,24 687,00 0,00 Всего 5650,27 650,27 5000,00 23. Первый год: 466,05 долл., второй год: 294,93 долл., третий год: 107,82 долл. 25. 97929,78 долл., 116070,22 долл. 27. 143,85 долл./мес., 904,80 долл./мес. 29. Ежемесячная выплата: 555,56 долл. а) 65 928,20 долл. б) 45 591,28 долл. в) 27 334,78 долл.
Ответы к упражнениям 779 31. а) Ежемесячная выплата: 669,15 долл., доход = 80596 долл. б) 178 месяцев, сбереженная сумма: 23 687 долл. 33. а) 157. б) 243. в) Количество изъятий не ограничено. 35. а) Ежемесячные изъятия: 1229,66 долл., общий доход: 185 338,80 долл. б) Ежемесячные взносы: 162,65 долл. 37. 29 799 долл. 39. 36 297,60 долл. 41. Все три графика являются возрастающими, выпуклыми вниз и пересекают ось х в одной и той же точке. Неуплаченные остатки всегда соответствуют отношению 2:3:4. Ежемесячные выплаты равны 402,31, 603,47 и 804,62 долл., общий доход равен 94831,60, 142 249,20 и 189663,20 соответственно. 43. 14,45%. 45. 10,21%. Упражнения для повторения 1. А = 104,50 долл. (3.1) 2. Р = 800 долл. (3.1) 3. t = 0,75 года, или 9 месяцев. (3.7) 4. г = 6%. (3.1) 5. А = 1393,68. (3.2) 6. Р = 3193,50. (3.2) 7. FV = 69 770,03. (3.3) 8. РМТ = 115,00 долл. (3.3) 9. PV = 33 944,27 долл. (3.4) 10. РМТ = 166,07 долл. (3.4) 11. п » 16. (3.2) 12. п « 41. (3.3) 13. п « 41. (3.3) 13. 3350,00 долл.; 350,00 долл. (3.7) 14. 19654,42 долл. (3.2) 15. 12 944,67 долл. (3.2) АО. ai „ „ Период, годы Проценты, долл. Сумма, долл. 0 400,00 1 21,60 421,60 2 22,77 444,37 3 24,00 468,36 4 25,29 493,65 01 „ „ Период, лет Проценты, долл. Выплаты, долл. Баланс, долл. 1 100,00 100,00 2 5,40 100,00 205,40 3 11,09 100,00 316,49 4 17,09 100,00 433,58 17. Если срок инвестиции не превышает 9 лет, то, чтобы получить максимальный доход, следует выбрать инвестицию под 13 простых процентов, в противном случае необходимо предпочесть 9 сложных процентов. 18. 164402 долл. (3.2)
780 Ответы к упражнениям 19. Девять сложных процентов, начисляемых ежеквартально, поскольку эффективность этой инвестиции равна 9,31%, в то время как эффективность инвестиции под 9,25 сложного процента, начисляемого ежегодно, равна 9,25%. (3.2) 20. 25 861 долл., 6661 долл. (3.3) 22. 10 210,25 долл. (3.2) 24. 15%. (3.1) 26. 2 года, 3 месяца. (3.2) 28. а) 571,499 долл. б) 1973,277 долл. (3.3) 29. 10,45%. (3.2) 30. а) 174%. б) 65,71%. (3.7) 31. 725,89 долл. (3.3) 32. а) 140 945,57 долл. б) 789,65 долл. в) 136,828 долл. (3.3, 3.4) 33. 102,99 долл.; 943,52 долл. (3.4) 35. 3374 дней; 10 лет. (3.2) 37. 5 лет и 10 месяцев. (3.3) 39. 28,8%. (3.7) 21. 11,64 долл. (3.7) 23. 6268,21. (3.2) 25. 10 318,91 долл.; 2281,09 долл. (3.4) 27. 139 месяцев, 93 месяцев. (3.2). 34. 576,48 долл. (3.3) 36. 175,28 долл.; 2516,80 долл. (3.4) 38. 18 лет. (3.4) 40. Номер выплаты Сумма выплаты, долл. Проценты, долл. Сокращение непогашенного остатка, долл. (3-4) Непогашенный остаток, долл. 0 1000,00 1 265,82 25,00 240,82 759,18 2 265,82 18,98 246,84 512,34 3 265,82 12,81 253,01 259,33 4 265,81 6,48 259,33 0,00 5 1063,27 63,27 1000,00 42. 55 347 долл.; 185 830,24 долл. (3.3). 44. 6,33%. (3.7) 41. Один год и один месяц. (3.2). 43. 6,30%. (3.2) 45. 44 депозита. (3.3) 46. а) 1189,52 долл. б) 72 963,07. в) 7237,31. (3.4) 47. После 360-й выплаты сертификат стоил бы 53 394,30 долл. Однократный взнос 10 000 долл, позволил бы выплатить заем за 252 месяца. Если бы ежемесячные взносы вкладывались под семь сложных процентов, выплачиваемых ежемесячно, то после 360 взносов сумма достигла бы 67 234,20 долл. (3.2, 3.3, 3.4) 48. Более низкая процентная ставка позволила бы сэкономить 12 246,20 долл.(3.4)
Ответы к упражнениям 781 49. 3807,59 долл. (5.2) 51. 4844,96 долл. (3.1) 53. 7,24%. (5.2) 54. а) 398 807 долл. б) 374204 долл. (5.5) 50. 5,79%. (5.2) 52. 6697,11 долл. (5.4) 55. а) 30 лет: 569,26 долл.; 15 лет: 749,82 долл. б) 30 лет: 69 707,99 долл.; 15 долл.: 37 260,74 долл. (5.4) 56. 20516 долл. (5.4) 57. 33,52%. (5.4) 58. а) 10,74%. б) 15 лет; 40 лет. (5.5) Глава 4 Практикум 4.1 1. График 2; решения нет. 5. х = 2, у = 4. 9. х = 4, у = 5. 13. и = 2, v = —3. 17. х = 1, у = —5. 3. График 1; х = —3, у = 1. 7. Решения нет (параллельные линии). 11. х = 1, у = 4. 15. т = 8, п = 6. 19. Решения нет (система является несовместной). 4 21. х = —у = 1. 3 У 23. Бесконечно много решений (зависимые уравнения). 25. х = у = 27. х = 1,1; у = 0,3. 5 5 29. х = у = -. 31. (2,41; 1,12). 33. (-3,31;-2,24).
782 Ответы к упражнениям 39. 41. а) (20; -24). б) (-4; 6). в) Решения нет. 43. а) Предложение: 143 футболки; спрос: 647 футболок. б) Предложение: 857 футболок; спрос: 353 футболок. в) Равновесная цена: 6,50 долл., равновесное количество: 500 футболок. г) Р 15 Точка равновесия (5; 6,5) Кривая предложения 10-= 5 О Кривая спроса * 1 Гог g Количество, сотен шт. 45. а) р = 0,001? + 0,15. б) р= —0,002? +1,74. в) Равновесная цена: 0,68 долл., равновесное количество: 530. г) Точка ; г равновесия \ (530; 0,68) 1 ■ ■ I Кривая предложения .. Кривая I i!i iW.j 0 500 1000 Количество, сотен шт. 47. а) При х = 120 единиц затраты равны доходу, т.е. 216000 долл, б) у, долл. 400000 0 300 000+ 200 000: Уб 100 000:: Уравнение доходов Прибыль Уравнение затрат (120; 216 000) Точка безубыточности НЖНШННН »х, шт. 50 100 150 200 49. а) При х = 1920 видеокассет затраты равны доходу, т.е. 38 304 долл.
Ответы к упражнениям 783 б) у, долл. 60 000 40 000 20 000 0 Уравнение доходов Прибыль^/ ,, 2*^ Уравнение Убыгю^^^ затрат (1920; 38 304) / Точка безубыточности Ч I I I I I И > 14 I I I I » X, ШТ. 1000 2000 3000 51. Базовая цена: 17,95 долл.; доплата: 2,45 долл./фунт. 53. 5720 фунтов смеси робусты; 6160 фунтов смеси некрепкого кофе. 55. Смесь А: 80 г.; смесь Б: 60 г. 57. а) р Расстояние, см б) d = 141 см (приближенно). в) Крыса будет пребывать в нерешительности. Практикум 4.2 1. Матрица Л имеет размер 2x3. Матрица С имеет размер 1x3. 3. С. 7. aw ~ ~4; &2з = 11. а) 2 х 4. б) 1. в) б2з = 7; /12 = —6. 5. В. 9. — 1; 8;0. ' 4 -6 -8 ' 1 -3 2 17. ’ 1 -3 2 _ 8 -12 -16 21. ’ 1 -3 2 2 0 -12 25. |й2 Я2- О —4 12 -8 4 -6 -8 0 6 -16 1 -3 3 -3 -10 27. 67?i + ► ^2- 29. —Т?2 + Я1 —> /?1. о
784 Ответы к упражнениям 31. х\ = 3, Х2 = 2. Все пары линий пересекаются в одной и той же точке. 43. Бесконечно много решений: Х2 = s; х± = 2s — 3 при любых действительных числах з. 45. Бесконечно много решений: Х2 = s; х^ = | при любых действительных числах s. 47. xi = — 1, Х2 = 3. 49. Решения нет. 51. Бесконечно много решений: Х2 = t; х^ = + 2 при любых действительных числах t 53. xi = 2, Х2 = —1. 55. xi = 2, ^2 = — 1- 57. хг = 1,1; х2 = 0,3. 59. хг = -23,125; х2 = 7,8125. 61. X! = 3,225; х2 = -6,9375. Практикум 4.3 1. Привеленна форма. 3. Не является приведенной: Т?2 Яз. 5. Не является приведенной: |/?2 —► #2- 7. Приведенная форма. 9. Не является приведенной: 2R2 + Ri —* R±. 11. xi = —2, Х2 = 3, хз — 0.
Ответы к упражнениям 785 13. Xi = 2t + 3, Х2 = — t — 5, Хз = t при всех действительных t. 15. Решения нет. 17. xi = 2s + — 5, Х2 = s, хз = —3t + 2, х± = t при всех действительных s и t. г 1 0 —7 ' 19. L 0 1 3 " 1 0 2 _5 -1 3 23. 0 1 —2 1 з . 0 0 1 0 . 27. xi = О, Х2 = —2, Хз = 2. 25. xi = -2, х2 = 3, хз = 1. 29. xi = 2t + 3, Х2 = t — 2, хз = t при всех действительных t. 31. х\ — 1, Х2 = 2. 33. Решения нет. 35. #1 = t — 1, Х2 = 2t + 2, хз = t. 37. xi = —2s +1 + 1, Х2 = s, хз = t при всех действительных s и t. 39. Решения нет. 41. xi = 2,52 — 4; Х2 = t; хз = —5 при всех действительных t. 43. х\ = 1, Х2 = —2, Хз = 1. 45. а) Зависимая система с двумя параметрами и бесконечным количеством решений. б) Зависимая система с одним параметром и бесконечным количеством решений. в) Независимая система с единственным решением. г) Невозможно. 47. При к = — 3 система имеет бесконечное количество решений, а при всех остальных значениях к — только одно. 49. При к = 3 система имеет бесконечное количество решений, а при всех остальных значениях к — только одно. 51. xi — 2s — 3t + 3, х2 = з + 2t + 2, хз = s, х± = t при всех действительных s nt. 53. xi = —0,5; Х2 = 0,2; хз = 0,3; Х4 = —0,4. 55. xi = 2s — l,5t + 1; Х2 = s; хз ~ — t + 1,5; Х4 = 0,5t — 0,5; х$ = t при всех действительных s nt. 57. a) xi — количество одноместных лодок, х2 — количество двухместных лодок, хз — количество четырехместных лодок. 0,5xi + Х2 + 1,5хз = 380; 0,6xi-F 0,9х2+ 1,2х3 = 380; 0,2xi + 0,3x2 + 0,5хз = 120. Двадцать одноместных, 220 двухместных и 100 четырехместных лодок. б) 0,5xi + Х2 + 1,5х3 = 380; 0,6xi +0,9х2 + 1,2х3 = 330.
786 Ответы к упражнениям t — 80 одноместных, 420 — 2t двухместных и t четырехместных лодок. Здесь t — целое число, удовлетворяющее неравенствам 80 < t С 210. в) 0,5xi + Х2 = 380; 0,6rci + 0,9x2 = 330; 0,2^1 + 0,Зх2 = 120. Единого расписания не существует. 59. xi — количество цистерн емкостью 8000 галлонов, хг — количество цистерн емкостью 16000 галлонов, хз — количество цистерн емкостью 24000 галлонов. Я1 + х2 + х3 = 24, 8000x1 4- 16000x2 + 24000хз = 520000, t — 17 цистерн емкостью 8000 галлонов, 41 — 2t цистерн емкостью 16 000 галлонов и t цистерн емкостью 24000 галлонов. Здесь t — целое число, равное 17, 18, 19 или 20. 61. Минимальная величина ежемесячных затрат равна 24100 долл, при аренде 7 цистерн емкостью 16000 галлонов и 17 цистерн емкостью 24000 галлонов. 63. xi — федеральный подоходный налог, х% — федеральный подоходный налог штата, хз — местный подоходный налог. xi -I- 0,5x2 + 0,5хз = 3 825 000; 0,2xi + Х2 + 0,2хз = 1 530 000; 0,lxi -|- 0,1x2 4“ Хз = 765 000. Налоговые обязательства: 57,65%. 65. xi — доход компании А, облагаемый налогом, Х2 — доход компании Б, облагаемый налогом, хз — доход компании В, облагаемый налогом, х± — доход компании Г, облагаемый налогом. Xi — 0,08x2 — 0,03хз — 0,07x4 = 2,272; —0,12x1 + Х2 — 0,11хз — 0,13x4 = 2,106; —0,11X1 — 0,09x2 + хз — 0,08x4 = 2,736; —0,06x1 — 0,02x2 — 0,14хз + Х4 = 3,168. Сумма доходов, облагаемых налогами, для компании А равна 2 927000 долл., для компании Б — 3 372 000, для компании В — 3,675 000 долл., а для компании Г — 3 926 000 долл. 67. а) xi — блюдо Л, унций; Х2 — блюдо Б, унций; хз — блюдо В, унций. 30xi + 10x2 + 20хз = 340, lOxi + 10x2 + 20хз = 180, lOxi 4- 30x2 Я- 20хз = 220. Восемь унций блюда Л, две унции блюда Б и четыре унции блюда В.
Ответы к упражнениям 787 б) 30xi + 10х2 = 340, lOxi + 10я?2 - 180, lOxi -I- 30x2 = 220. Не существует комбинации, удовлетворяющей всем ограничениям. в) 30xi + 10х2 + 20х3 = 340, lOxi + 10x2 -И 20хз = 180, Восемь унций блюда А, 10 — t унций блюда Б и t унций блюда В. Здесь 0 t < 5. 69. xi — количество баррелей смеси А, Х2 — количество баррелей смеси Б, х3 — количество баррелей смеси В, Х4 — количество баррелей смеси Г. 30xi -|- 30x2 ЗОх3 + 60x4 = ООО, 50xi -I- 75x2 + 25х3 + 25x4 = 750, 30xi -J- 20x2 “И 20х3 -И 50x4 = 700. 10 -1 баррелей смеси А, t — 5 баррелей смеси Б, 25 — 2t баррелей смеси В и t баррелей смеси Г. Здесь 5 < t 10. 71. xi — количество часов в группе Я, Х2 — количество часов в группе Б. 30xi + 20x2 = 600, lOxi + 20х2 = 400. Группа А: 10 ч, группа Б: 15 ч. 73. а) 6-я улица и Вашингтон-авеню: xi -I- Х2 = 1200, 6-я улица и Линкольн-авеню: #2 + хз = 1000, 5-я улица и Линкольн-авеню: х3 + Х4 = 1300. б) xi = 1500 — t, Х2 = t - 300, х3 = 1300 — Z и Х4 = t, где 300 < t < 1300. в) 1300; 300. г) Вашинггон-авеню: 500; 6-я улица: 700; Линкольн-авеню: 300. Практикум 4.4 1. -1 О' 5 -3 5 —7 5. -5 2 0 4 5' 9. -3 1 -5' 13. —2 —4 17 —15 6 1 /. —20 8 3 -2 -4 21. 6 —4 -8 -9 6 12 3. Невозможно. 5 -10 0 20 [—15 10 -5 30 и. f2 41 1 -5 15. [-7]. 19. [11]. Г-12 12 18 20 -18 -6 ’
788 25. 29. 33. 37. 41. 43. 47. 51. 53. 55. 57. 59. Ответы к упражнениям Невозможно. -66 69 39 92 -18 -36 -18 48 54 -34 11 2 4 27 35. Невозможно. —26 -15 —25" 39. —4 -18 4 2 43 -19 Последовательность матриц Вп сходится к матрице 0,25 0,75 0,25 0,75 Последовательность матриц АВп сходится к матрице [0,25 0,75]. а = —1, b = 1, с - 3, d = —5. 45. х = 2, у = —3. х — 3, у = 2. 49. а = 3, Ь = 4, с = 1, d = 2. Все утверждения верны. Г итара Б 33 57 Модель А Модель Б анджо 26 77 Базов комп. Сырье, долл. Оплата труда, долл. ая AM/FM "Круиз- пектация Кондиц. радио контроль" 180 82 54 30 1075 98 81 30 Модель В а) 17,20 до; б) 27,40 до; 1Л. 1Л. 1715 106 108 36 в) Затраты г) MN = на оплату тру; МА VA 17.20 12,20“ 27.20 19,30 38,60 27,40 ха в каждом цехе выражается матрицей MN. 1- местная лодка 2- местная лодка. 4-местная лодка "0 1 0 1 1‘ 0 0 1 1 2 а) А2 = 0 0 0 1 0 1 0 1 0 0 0 1 0 0 0 Существует единственный способ долететь из Нью-Йорка в Форт-Лодердейл, используя два рейса. Существуют два способа долететь из Нью-Йорка в Вашингтон, используя только два рейса. Элементы матрицы А2 представляют собой количество разных рейсов, с помощью которых можно долететь из пункта i в пункт j, используя только два рейса.
Ответы к упражнениям 789 1 О б) А3 = О О 1 1 1 1 1 0 2 1 О О О 1 1 О 1 О О 1 О 2 О Существует единственный способ долететь из Миртл-Бич в Форт-Лодердейл, используя только три рейса. Существуют два способа долететь из Нью-Йорка в Миртл-Бич, используя только три рейса. Элементы матрицы А3 представляют собой количество разных рейсов, с помощью которых можно долететь из пункта i в пункт J, используя только три рейса. 2 3 4 4 4 2 3 3 4 3 в) А + А2 + А3 + А4 = 1 1 1 1 1 1 2 2 3 3 1 1 2 2 2 Используя четыре рейса, можно добраться из любого пункта отправления в любой пункт прибытия. 61. а) 70 г. б) 30 г. в) Матрица МN выражает количество (в граммах) белков, углеводов и жиров в 20 ун¬ циях муки каждого вида. X У Z ■ 70 60 50 Белки г) MN = 63. a) 2650 дол б) 4400 дол 380 50 [Л. [Л. 360 40 340 30 Углеводы Жиры в) ] Ст г) Иатрм оимос 2650' 4400 [ца МN выражает общие затраты на проведение опроса в каждом городе, ть Беркли Окленд ‘ Телефонный Посещение Почтовая звонок на дому рассылка д) [1 11 N = [ 3000 1300 13000 Количество контактов 6500 Беркли 10800 Окленд’ 0 0 1 1 1 0" 1 0 0 1 1 0 65. а) 0 1 0 1 0 0 0 0 0 0 0 1 0 0 1 1 0 1 1 1 1 0 0 0
790 Ответы к упражнениям г) Фрэнк, Барт, Аарон и Элвис (поровну), Чарльз, Дэн. Практикум 4.5 15. Нет. 13. Да. 17. Да. 19. 21 25. 29. 33. 3 -1 —2 1 -5 -12 —2 4 2 —2 3 1 5 ’1,5 -о, 5' —2 1 3" 1 -1 23. 27. 37. Не существует. -1 -3 7 —2 6 -5 -3 0‘ 1 * -3 1 ’ —2 2 1 -Г 1 1 31. Не существует. 35. 1 —2 -1 2 -3 -2 39 -3 4 3 —2 3 2 2 —4 . -1 1,5’ —2 -1,25 -1,75 -5,5 0,5 0,5' 1 0 -0,375 -1,25 0,25 45. Матрица М~г существует тогда и только тогда, диагонали не равны нулю. 41 когда все элементы на ее главной
791 Ответы к упражнениям 47. А'1 - Л; А2 = I. 49. Л"1 = А; А2 = I. 0,5 - -0,3 0,85 -0,25' 0 0,1 0,05 -0,25 ЭХ* -1 0,9 -1,55 0,75 • —0,5 0,4 -1,3 0,5 _ 1,75 5,25 8,75 -1 -18,75 ' 1,25 3,75 6,25 -1 -13,25 53. -4,75 -13,25 - -22,75 3 48,75 -1,375 -4,625 -7,875 1 16,375 3,25 8,75 15,25 -2 -32,25 55. 36 44 5 5 61 82 14 14 25 37 49 64 36 54 47 66 43 62. 57. GONE WITH THE WIND 59. 27 19 55 9 35 50 114 74 70 69 23 19 48 18 62 36 69 72 42 72 52 78 79 56 70 5 19 29 0 24 61. THE GREATEST SHOW ON EARTH. Практикум 4.6 1. 3xi + X2 = 5, 2xi — ^2 = —4. 3. —3xi + X2 = 3, 2xi + = -4, ’3 -4 Х1‘ т 2 1 52. 5 —xi + 3x2 — 2хз = 2. 1 -3 2 х/ -3 7. —2 3 0 = 1 1 1 4 _хз_ —2 11. xi = 0, X2 = 4. 15. xi = 11, X2 = 4. 9. xi = -8, x2 = 2. 13. xi = 3, X2 = —2. 17. a) xi = —3, X2 = 2. б) Xi = —1, X2 = 2. в) xi = —8, X2 = 3. 19. a) xi = 17, X2 = —5. б) xi = 7, x2 = -2. в) xi = 24, X2 = —7. 21. a) xi = 1, X2 = 0, хз = 0. б) xi = -7, X2 = -2, x3 = 3. в) xi = 17, X2 = 5, хз = —10. 23. a) xi = 8, X2 = -6, хз = —2. б) Xi = —6, X2 = 6, Хз = 2. в) xi = 20, x2 = -16, Хз = —10. 25. xi = 2t + 2,5; X2 = t', t — любое действительное число. 27. Решения нет. 29. xi = 13t 4- 3, Х2 = 8t + 1, хз = t, t — любое действительное число. 31. Х = (А — В)~1С.
792 Ответы к упражнениям 33. X = (А + 1)~1С. 35. X = (4 + В)~\С + D). 37. а) х\ = 1, х2 = 0. б) xi = -2000, х2 = 1000. в) xi = 2001, х2 — —1000. 39. X! = 18,2; х2 = 27,9; х3 = -15,2. 41. xi = 24, х2 = 5, х3 = —2, х4 = 15. 43. а) х\ — количество билетов стоимостью четыре доллара, х2 — количество билетов стоимостью восемь долларов, хх + %2 = юооо, 4xi + 8x2 = ki Вырученная сумма Концерт 1: 6000 билетов стоимостью четыре доллара и 4000 билетов стоимостью восемь долларов. Концерт 2: 5000 билетов стоимостью четыре доллара и 5000 билетов стоимостью восемь долларов. Концерт 3: 3000 билетов стоимостью четыре доллара и 7000 билетов стоимостью восемь долларов. б) Нет. в) 40 000 -I- 4t долл., где t — целое число, удовлетворяющее условию 0 < t 10 000. 45. xi — время выполнения заказа на заводе А, х2 — время выполнения заказа на заводе Б. 10a?i + 8x2 = fci, Количество рам для легковых автомобилей 5xi + 8x2 = Количество рам для прицепов Заказ 1: 280 ч на заводе А и 25 ч на заводе Б. Заказ 2: 160 ч на заводе А и 150 ч на заводе Б, Заказ 3: 80 ч на заводе А и 225 ч на заводе Б. 41. xi — премия президента, х2 — премия исполнительного вице-президента, хз — премия ассоциированного вице-президента, Х4 — премия помощника вице-президента. xi + 0,03x2 + 0,03х3 + 0,03x4 = 60000; 0,025xi -F Х2 + 0,025хз + 0,025x4 = 50 000; 0,02xi + 0,02x2 + хз + 0,02x4 = 40000; 0,015xi + 0,015x2 -I- 0,015хз + Х4 = 30000. Президент: 56 600 долл., исполнительный вице-президент: 47 000 долл., ассоциированный вице-президент: 37 400 долл., помощник вице-президента: 27 900 долл. 49. a) xi — количество унций в смеси Л, х2 — количество унций в смеси Б. 0,20xi + 0,14x2 = ki Белки 0,04xi 4- 0,03x2 = к2 Жиры Блюдо 1: 50 унций смеси А и 500 унций смеси Б.
Ответы к упражнениям 793 Блюдо 2: 450 унций смеси Л и 0 унций смеси Б. Блюдо 3: 150 унций смеси А и 500 унций смеси Б. б) Нет. Практикум 4.7 1. 40 центов в отрасли Л и 20 центов в отрасли Е. 3. 0,6 -0,2 -0,2 0,9 1,8 0,4 0,4 1,2 Х1 ^2 5. Х = 16,4 9,2 7. 20 центов в отрасли Л, 10 центов в отрасли В и 10 центов в отрасли Е. И. Сельское хозяйство: 18 млрд, долл., строительство: 15,6 млрд, долл., энергетика: 22,4 млрд. долл. 24 46 13. 1,4 0,6 0,4 1,6 ’1,58 0,24 0,58 15. 0,4 1,2 0,4 0,22 0,16 1,22 “38,6 18 17,4 17. а) Сельское хозяйство: 80 млн. долл., промышленное производство: 64 млн. долл. б) Спрос на продукцию сельского хозяйства вырастет до 54 млн. долл., а спрос на продукцию промышленного производства уменьшится до 38 млн. долл. 19. Общий объем производства энегретической отрасли должен составлять 75% общего объема производства горной промышленности. 21. Каждый элемент матрицы должен быть больше нуля и меньше единицы включительно. 23. Угольная промышленность: 28 млрд, долл., Металлургическая промышленность: 26 млрд. долл. 25. Сельское хозяйство: 148 млн. долл., туризм: 146 млн. долл. 27. Сельское хозяйство: 40,1 млрд, долл., промышленность: 29,4 млрд, долл., энергетика: 34,4 млрд. долл. 29. Год 1: сельское хозяйство: 65 млрд, долл., энергетика: 83 млрд, долл., сфера услуг: 71 млрд, долл., промышленность: 88 млрд. долл. Год 2: сельское хозяйство: 81 млрд, долл., энергетика: 97 млрд, долл., сфера услуг: 83 млрд, долл., промышленность: 99 млрд. долл. Год 3: сельское хозяйство: 117 млрд, долл., энергетика: 124 млрд, долл., сфера услуг: 106 млрд, долл., промышленность: 120 млрд. долл. Упражнения для повторения 1. х = 4, у = 4. (4.1) 2. х = 4, у = 4. (4.1)
794 3. 4. 5. 7. 9. И. 13. 15. 17. 18. 19. 21. 23. 25. 26. 27. 28. 29. 30. а) Не является приведенной. Ri R^. б) Не является приведенной. —* Ri- в) Приведенная форма. г) Не является приведенной. — 17?г + Bi —»Вь (4.3) Ответы к упражнениям а) 2 х 5; 3 х 2. б) «24 = 3, «15 = 2, &31 = — 1, ^22 = 4. в) Матрица АВ не определена. Матрица В А определена. (4.2, 4.4) '3 3' 4 2 -3 О' 1 -1 ‘ xi = 8, Х2 = 2. (4.6) Результат не определен. 3 4 * (4.4) (44) 6. 8. (4-4) (4-4) 10. Результат не определен. (4.4) 2 3 4 6 ’ 14. Результат не определен. (4.4) 12. (44) 4 7 5 5 ‘ [8]. (44) —2 3 3 “4 ‘ = 9, х2 = -П. (4.2) = 9, х2 = -11; xi = 16, х2 = —19; х\ = —2, х2 = 4. (4.6) (44) 16. an = 9, х2 = -И. (4.1) Результат не определен. (4.4) 2 (4 4) 8 6 v } [9]. (44) 2 -1 0 2 1 .(4.5) _ 1 2_ 20. 22. 24. '10 4 —2 4 6 ’10 -1 1 -8 6 ’ -1 2 3 -5 —4 —7 (44) -3" 6 .(4.4) 9 f -5 .(4.4) -2 _5 2 1 1 2 Х1 = 2, Х2 = 1, Хз = —1. xi = —5t — 12, х2 = 3t + 7, хз = t, где t — любое действительное число. (4.3) = 2, х2 = 1, хз = —1; xi = 1, х2 = —2, хз = 1; х^ = —1, х2 = 2, хз = —2. (4.6) При к = 3 система имеет бесконечное количество решений, а при остальных значе¬ ниях к — только одно. (4.3) 1,4 0,3' 0,8 1,6 х = 3,46; у = 1,69. (4.1) а) б) xi (i-му1 48" 56 ’ (4 7)
Ответы к упражнениям 795 31. '-0,9 0,8 0,1 51 -од 0,2 —4 .(4.5) -0,1 О 32. 33. xi = 1400, х2 = 3200, х3 = 2400. (4.6) xi = 1400, х2 = 3200, х3 = 2400. (4.3) 34. (I - М)-1 '1,3 0,4 0,7' 81' 0,2 1,6 0,3 ; X = 49 0,1 0,8 1,4 62 .(4.7) 35. а) Единственное решение. б) Либо нет решения, либо есть бесконечно много решений. (4.6). 36. а) Единственное решение. б) Нет решения. в) Бесконечно много решений. (4.3). 37. б) Существует только одно решение. (4.6). 38. а) С = 243000 + 22,45а?. R = 59,95ж. б) х = 6480 машин. R = С = 388 476 долл. в) Если х > 6480, то компания получает прибыль, если же х < 6480, то компания несет убытки. у, долл. 500 000 - 400 000 300 000 200000 100 0004 R С (6480,388476) Точка безубыточности о WI I >Х, ШТ. 10000 39. xi — количество тонн руды А, добытой на шахте Войсе-Бэй; хг — количество тонн руды В, добытой на шахте Хоук-Ридж. 0,02a;i + 0,03x2 = 6; 0,04x1 + 0,02x2 = 8. 40. а) xi = 150 тонн руды А, добытой на шахте Войсе-Бэй; хг = 100 тонн руды В, добытой на шахте Хоук-Ридж. (4.3) 37,5] [6 8 ~ 100 ’ ’150' xi 52. = 150 тонн руды А, добытой на шахте Войсе-Бэй; Х2 = 100 тонн руды В, добытой на шахте Хоук-Ридж. б) -25 50 -25 -25 50 -25 37,5] [7,5] = [ 75 200 ’ Si 52. xi = 75 тонн руды А, добытой на шахте Войсе-Бэй; Х2 = 200 тонн руды В, добытой на шахте Хаук Ридж. 41. a) xi — количество вагонов вместимостью 3000 куб. футов, х% — количество вагонов вместимостью 4500 куб. футов, х$ — количество вагонов вместимостью 6000 куб. 7
796 Ответы к упражнениям футов. Х1 + Х2 + Хз = 20, 3000^1 4- 4500^2 + бОООхз = 108 000, xi = t — 12 вагонов вместимостью 3000 куб. футов, х2 = 32 — 2t вагонов вместимостью 4500 куб. футов, хз = t вагонов вместимостью 6000 куб. футов, где t = 12, 13, 14, 15 или 16. б) Минимальная стоимость месячной аренды равна 5700 долл, достигается при аренде восьми вагонов вместимостью 4500 куб. футов и 12 вагонов вместимостью 6000 куб. футов. (4.3) 42. а) Элементы матрицы MN представляют собой стоимость материалов, произведен¬ ных из сплавов разных поставщиков. Поставщик А б) MN = 7620 13 880 Поставщик Б 7530 13 930 Сплав 1 Сплав 2 Поставщик А Поставщик Б в) [1 1] 7WW = [21 500 21460] Общая стоимость сырья. 43. а) 6 ,35 долл. б) Элементы матрицы MN представляют собой затраты на оплату труда, затраченного на производство калькуляторов на каждом заводе. СА ТХ \ мы _ 3,65 3,00 Модель А в) МЛ - ^б 35 5 2()j МодельБ 44. xi — сумма, инвестированная под 5%, х2 — сумма, инвестированная под 10%. #1 + #2 = 5000; 0,05xi -I- 0,1х2 = 400. 2000 долл, под 5%; 3000 долл, под 10%. (4.3) 45. 2000 долл, под 5% и 3000 долл, под 10%. (4.6) 46. Нет. Годовой доход будет колебаться между 250 и 500 долл. (4.6) 47. xi — количество билетов стоимостью восемь долларов, х2 — количество билетов стоимостью 12 долл., хз — количество билетов стоимостью 20 долл. xi + х2 -К х3 = 25 000, 8xi + 12х2 + 20хз = fci, Сумма выручки — 0- Концерт 1: 5000 билетов стоимостью 8 долл., 15 000 билетов стоимостью 12 долл, и 5000 билетов стоимостью 20 долл. Концерт 2: 7500 билетов стоимостью 8 долл., 10000 билетов стоимостью 12 долл, и 10000 билетов стоимостью 20 долл. (4.6)
Ответы к упражнениям 797 48. #i + #2 + = 25 000, 8x1 + 12х2 + 20хз = fci. Предполагаемая выручка Концерт 1: 2t — 5000 билетов стоимостью 8 долл., 30000 — 3t билетов стоимостью 12 долл, и t билетов стоимостью 20 долл. Концерт 2: 2t — 7500 билетов стоимостью 8 долл., 32 500 — 3t билетов стоимостью 12 долл, и t билетов стоимостью 20 долл. Концерт 3: 2t — 10 000 билетов стоимостью 8 долл., 35 000 — 3t билетов стоимостью 12 долл, и t билетов стоимостью 20 долл. Параметр t является целым числом, удовлетворяющим неравенствам 5000 t < 11666. (4.3) 49. а) Сельское хозяйство: 80 млрд, долл., промышленность: 60 млрд. долл. б) Сельское хозяйство: 135 млрд, долл., промышленность: 145 млрд. долл. (4.7) 50. GRAPHING UTILITY. (4.5). 51. а) 1-я улица и Эльм-стрит: х\ + х4 = 1300. 2-я улица и Эльм-стрит: xi — Х2 = 400. 2-я улица и Оак-стрит: хг + хз = 700. 1-я улица и Оак-стрит: х3 — х4 = —200. б) xi = 1300 - t, Х2 = 900 - t, хз = t — 200. х4 = t, где 200 < t < 900. в) 900; 200. г) Эльм-стрит: 800; 2-я улица: 400; Оак-стрит: 300. (4.3) Ь [1 В С Ь 1 0 0“ 0 1 1 0 0 1 0 0 0 А В А ГО 1 б) м + м2 = в 2 ° G £ D [1 1 С 1 1 0 0 D 1‘ . Ранг: В, С, A, D. (4.4) 0 Глава 5 Практикум 5.1
798 Ответы к упражнениям 19. а) Область решения выделена двойной штриховкой. -10
Ответы к упражнениям 799 21. а) Область решения выделена двойной штриховкой. б) Область решения не заштрихована. 25. I; (0; 16), (6; 4), (18; 0). 29. Ограниченная. у 2х + у=10 31. Неограниченная. 33. Ограниченная.
800 Ответы к упражнениям 45. Ограниченная. 16х+13у=119 47. а) Прямые Зх 4- = 36 и Зх + 2у = 30 пересекаются в точке (8; 3). Прямые Зх + + 4?/ = 36 и х = 0 пересекаются в точке (0; 9). Прямые Зх + 4у — 36 и у — 0 пересекаются в точке (12; 0). Прямые Зх + 2у = 30 и х = 0 пересекаются в точке (0; 15). Прямые Зх + 2у = 30 и у = 0 пересекаются в точке (10; 0). Прямые х = 0 и у = 0 пересекаются в точке (0; 0). б) (8;3), (0; 9), (0;0). 49. бх + 4 г/ 108, х + у 24, х 0, 1/^0.
Ответы к упражнениям 801 51. а) Все производственные планы лежат на графике 50х + 601/ = 1000, обеспечивая прибыль, равную 1100 долл. б) Существует несколько возможных решений. Все производственные планы лежат на графике 50# -I- 601/ = 1150, обеспечивая прибыль, равную 1150 долл. 53. 20x4-101/^ 460, ЗОх 4- 301/ 960, 5х 4-101/ 220, 55. 10x4-201/^ 800, 20х 4-101/ < 640, х 0, У > 0. Практикум 5.2 1. 16 3. 84. 5. 32. 7. 36. 9. МахР = 30 при Xi = 4 и Х2 = 2. 11. Min z = 14 при xi = 4 и Х2 = 2. Максимума нет. 13. МахР = 260 при xi = 2 и Х2 = 5. 15. Min z = 140 при Xi = 14 и Х2 = 0. Максимума нет. 17. MinP = 20 при Xi = 0 и Х2 = 2. МахР = 150 при xi = 5 и Х2 = 0. 19. Область допустимых решений пуста. Оптимальных решений нет. 21. MinP = 140 при xi = 3 и Х2 = 8. МахР = 260 в точках с координатами xi = 8 и Х2 = 10, xi = 12 и Х2 = 2 и в любой точке отрезка, соединяющей точки (8; 10) и (12; 2). 23. МахР = 26000 при xi = 400 и Х2 = 600. 25. МахР = 5507 при xi = 6,62 и Х2 = 4,25. 27. Мах 2 = 2 при xi = 4 и хг = 2. Minz не существует. 29. а) 2а < Ь. б) -а < b < 2а. О 1 В) ь < -а. г) Ь = 2а.
802 Ответы к упражнениям Д) Ъ = -а. О 31. а) Пусть xi — количество пар обычных лыж, а Х2 — количество пар лыж для слалома, выпускаемых за день. Минимизировать Р = 40xi + 30x2 при условиях 6х 1 4- 4^2 С 108, xi + #2 24, xi > 0, х2 0. Максимальная прибыль равна 780 долл, при выпуске 6 пар обычных лыж и 18 пар для слалома. б) Если выпускать 18 пар обычных лыж и отказаться от производства для слалома, максимальная прибыль упадет до 720 долл. в) Если отказаться от производства обычных лыж и выпускать 24 пары для слалома, максимальная прибыль поднимется до 1080 долл. 33. а) Пусть xi — количество рабочих дней на заводе А, а Х2 — количество рабочих дней на заводе Б, Минимизировать С = 1000x1 + 900x2 при условиях 20xi + 25x2 200, 60x1 -|- 50x2 > 500, xi 0, х2 0. Завод А: 5 дней. Завод Б: 4 дня. Минимальная величина затрат равна 8600 долл. б) Завод А: 10 дней. Завод Б; 0 дней. Минимальная величина затрат равна 6000 долл. в) Завод Л: 0 дней. Завод Б: 10 дней. Минимальная величина затрат равна 8000 долл. 35. Пусть xi — количество автобусов, а Х2 — количество микроавтобусов. Минимизировать С = 1200xi + 100x2 при условиях 40xi + 8x2 400, 3xi 4- Х2 36, xi 0, х2 0. 7 автобусов, 15 микроавтобусов. Минимальная величина затрат равна 9900 долл. 37. Пусть xi — количество вложений в фонд депозитных сертификатов, а Х2 — количество вложений во взаимный фонд. Минимизировать Р = 0,05xi 4- 0,09x2 при условиях xi + Х2 60 000, х2 10 000, Х1 2х2, Xi, Х2 0,
Ответы к упражнениям 803 40 000 долл. — в фонд депозитных сертификатов и 20 000 долл. — во взаимный фонд. Максимальный доход равен 3800 долл. 39. а) Пусть xi — количество галлонов продукции, произведенной согласно старому процессу, а х2 — количество галлонов продукции, произведенной по новому процессу. Максимизировать Р = 60xi + 20х2 при условиях 20xi + 5х2 < 16 000, 40xi + 20х2 30000, xi 0, х2 0. Мах Р = 450 долл, при выпуске 750 галлонов продукции только по старому процессу. б) МахР = 380 долл, при выпуске 400 галлонов продукции по старому процессу и 700 галлонов продукции только по новому процессу. в) Мах Р = 288 долл, при выпуске 1440 галлонов продукции по только по новому методу. 41. а) Пусть xi — количество упаковок удобрения марки А, а х2 — количество упаковок удобрения марки Б. Максимизировать N = 8xi 4- Зх2 при условиях 4xi + 4х2 > 1000, 2xi + Х2 < 400, xi > 0, х2 > 0, 150 упаковок марки А, 100 упаковок марки Б. Максимальное количество азота равно 1500 фунтов. б) Ни одной упаковки марки Я, 250 упаковок марки Б, максимальное количество азота равно 750 фунтов. 43. Пусть xi — количество кубических ярдов смеси А, а х2 — количество кубических ярдов смеси Б. Минимизировать С = 30xi + 35х2 при условиях 20xi + Юх2 > 460, 30xi + 30х2 960, 5xi + Юх2 > 220, xi > 0, х2 0, 20 куб. ярдов смеси А, 12 куб. ярдов смеси Б. 1020 долл. 45. Пусть xi — количество мышей, а х2 — количество крыс.
804 Ответы к упражнениям Максимизировать Р = xi + х% при условиях lOxi -I- 20x2 < 800, 20xi + 10x2 640, 0, 0. Х1 > х2 48; 16 мышей, 32 крысы. Практикум 5.3 1. а) 2. б) Две базисные и три небазисные переменные. в) Два линейных уравнения с двумя переменными. 3. а) 5. б) 4. в) Пять базисных и четыре небазисных переменных. г) Пять линейных уравнений с пятью переменными. 5. Небазисные Базисны :е Допустимое решение? 1 ^1, #2 51, 52 Да 2 Xl, S1 %2, 52 Да 3 Xi, 32 Х2, 51 Нет 4 Х2, Зх *1, 52 Нет 5 Т2, S2 Xi, S1 Да 6 81, «2 Xi, Х2 Да /. Х2 32 Допустимое решение? 1 0 0 50 40 Да 2 0 50 0 -60 Нет 3 0 20 30 0 Да 4 25 0 0 15 Да 5 40 0 -30 0 Нет 6 20 10 0 0 Да #1 + Х2 + <$1 = 16, 2xi + %2 + s2 = 20.
Ответы к упражнениям 805 Х1 Х2 «2 Точка пересечения Допустимое решение? 0 0 16 20 о Да 0 16 0 4 А Да 0 20 —4 0 В Нет 16 0 0 -12 Е Нет 10 0 6 0 D Да 4 12 0 0 С Да И. 2д?1 + Х2 + 51 = 22, Х1 Н" Н" 52 = 12, XI + 2х2 + 5з = 20. я?1 51 52 53 Точка пересечения Допустимое решение? 0 0 22 12 20 о Да 0 22 0 -10 -24 с Нет 0 12 10 0 —4 в Нет 0 10 12 2 0 А Да 11 0 0 1 9 G Да 12 0 —2 0 8 Н Нет 20 0 -18 -8 0 I Нет 10 2 0 0 6 F Да 8 6 0 -2 0 Е Нет 4 8 6 0 0 D Да Практикум 5.4 1. 1) Базисные: хг, 5i, Р; небазисные: xi, «2- 2) xi = 0, Х2 = 12, si = 15, 52 = О, Р = 20. 3) Требуется дополнительная итерация. 3. 1) Базисные: #2, #з, ^з, Р; небазисные: xi, 5i, 52. 2) xi = О, х2 = 15, хз = 5, 51 = 0, 52 = О, S2 = 12, Р = 45. 3) Оптимального решения нет.
806 Ответы к упражнениям Включаемая переменная Х2 51 «2 р Исключаемая переменная —* 51 Г Ф 4 1 0 0 4 «2 3 5 0 1 0 24 Р . ~8 -5 0 0 1 0 Х1 1 4 1 0 0 4 -5'2 0 —7 -3 1 0 12 Р 0 27 8 0 1 32 Включаемая переменная Х2 Х2 1 51 1 52 0 53 0 о 4 ‘ Исключаемая переменная 3 0 1 1 0 0 8 53 0 0 2 0 1 0 2 Р . -4 0 -3 0 0 1 5 Х1 ■ 1 1 2 1 2 0 0 0 2 ‘ 52 0 3 2 _ 1 2 1 0 0 2 53 0 0 2 0 1 0 2 Р L о 2 -1 0 0 1 13 . 2xi + х2 + «1 = ю, Xi + 3X2 + S2 = ю, — 15371 — 10X2 + Р=0. Включаемая переменная Х1 Х2 51 52 р Исключаемая переменная —> 51 1 1 0 0 10 2) 52 1 3 0 1 0 10 Р -15 -10 0 0 1 0 3) МахР = 80 при xi = 4 и х2 = 2. 11. 1) 2x1+ X2 + S1 =Ю, Xi + 3X2 + «2 = Ю, -30x1- Х2 + Р=0. Включаемая переменная Ж1 Х2 51 52 р Исключаемая переменная —> 51 1 1 0 0 10 ■ 2) 52 1 3 0 1 0 10 Р -30 -1 0 0 1 0 3) МахР = 150 при xi = 5 и х2 = 0. 13. МахР = 260 при xi = 2 и Х2 = 5. 15. Оптимального решения нет. 17. МахР = 7 при xi = 3 и Х2 = 5.
Ответы к упражнениям 807 19. МахР = 58 при Xi = 12, х2 = 0 и хз = 2. 21. МахР = 17 при xi = 4, х2 = 3 и хз = 0. 23. МахР = 22 при Xi = 1, х2 = 6 и хз = 0. 25. МахР = 26000 при х\ = 400 и х2 = 600. 27. МахР = 450 при х± = 0, х2 = 180 и хз = 30. 29. МахР = 88 при х^ = 24 и х2 = 8. 31. Выбор любого столбца приводит к одному и тому же оптимальному решению: тахР = 13 при х± = 3 и х2 = 10. 33. Выбор Р = 60 при х\ = 12, х2 = 8 и хз = 0. Выбор столбца 2: тахР = 60 при xi = 0, х2 = 20 и хз = 0. 35. Пусть — количество компонента^, х2 — количество компонентаР, хз — количество компонента В. Максимизировать Р = 7a?i + 8ж2 + Ю^з при условиях 2^1 -I- Зж2 4- 2жз < 1000, xi + х2 + 2хз < 800, xi, х2, хз 0, 200 единиц компонента А, 0 единиц компонента Б и 300 единиц компонента В. Максимальная прибыль равна 4400 долл. 37. Пусть д?1 — объем вложений в государственные облигации, х2 — объем вложений во взаимные фонды, хз — объем вложений в фонды денежного рынка. Максимизировать Р = 0,08xi + 0,13ж2 -I- 0,15х3 при условиях xi 4- х2 + хз < 100 000, -х\ 4- х2 4- хз 0, Xi, ^2> #з > 0, 50 000 долл. — в правительственные облигации, 0 — во взаимные фонды и 50 000 — в фонды денежного рынка. Максимальный доход равен 11500 долл. 39. Пусть х\ — количество рекламных роликов, размещенных в дневное время, х2 — количество рекламных роликов, размещенных в вечернее время, хз — количество рекламных роликов, размещенных в ночное время.
808 Ответы к упражнениям Максимизировать Р = 14000xi + 24000x2 + 18000х3 при условиях xi + х2 + хз < 15, lOOOxi + 2000x2 + 1500хз 20000, xi, ^2, #з 0, 10 роликов — в дневное, 5 — в вечернее и 0 — в ночное время. Максимальное количество потенциальных покупателей равно 260 000. 41. а) Пусть xi — количество домов в колониальном стиле, х2 — количество многоуровневых домов, хз — количество усадеб. Максимизировать Р = 20000xi + 18000х2 + 24000х3 при условиях jxi + |х2 + хз < 30, 60000X1 + 60000x2 + 80000хз 3 200 000, 4000x1 + 3000x2 + 4000хз 180000, Х1, ^2, #3 > 0, 20 домов в колониальном стиле, 20 многоуровневых домов и 10 усадеб. Максимальная прибыль равна 1000 000 долл. б) Ни одного дома в колониальном стиле, 40 многоуровневых домов и 10 усадеб. Максимальная прибыль равна 960000 долл. 20000 рабочих часов остаются неиспользованными. в) 45 домов в колониальном стиле, ни одного многоуровневого дома и ни одной усадьбы. Максимальная прибыль равна 1125 000 долл. 7,5 акра земли и 500 000 долл, капитала остаются неиспользованными. 43. Пусть xi — количество коробок ассортимента I, х2 — количество коробок ассортимента II, хз — количество коробок ассортимента III. Максимизировать Р = 4xi + Зх2 -I- 5хз при условиях 4xi -I- 12x2 + 8х3 < 4800, 4xi + 4x2 + 8х3 4000, 12xi + 4х2 + 8x3 5600, ХХ, ^2, ^3 0, а) 200 коробок конфет ассортимента I, 100 коробок конфет ассортимента II и 350 коробок конфет ассортимента III. Максимальная прибыль равна 2850 долл. б) 100 коробок конфет ассортимента I, ни одной коробки конфет ассортимента II и 550 коробок конфет ассортимента III. Максимальная прибыль равна 3150 долл. 200 конфет с фруктовой начинкой остаются неиспользованными. в) Ни одной коробки конфет ассортимента I, 50 коробок конфет ассортимента II и 675 коробок конфет ассортимента III. Максимальная прибыль равна 3525 долл. 400 конфет с фруктовой начинкой остаются неиспользованными. 45. Пусть xi — количество граммов пищевого продукта Л, х2 — количество граммов пищевого продукта Б, х$ — количество граммов пищевого продукта В.
Ответы к упражнениям 809 Максимизировать Р = 3xi + 4^2 И- 5хз при условиях xi -I- Зх2 + 2хз < 30, 2xi + ^2 4- 2хз 24, Х1, ^2, #з 0, Ни одного грамма пищевого продукта Л, 3 г пищевого продукта Б, 10,5 г пищевого продукта В. Максимальное количество белков равно 64,5 единицы. 47. Пусть xi — количество студентов, х2 — количество аспирантов, х3 — количество сотрудников. Максимизировать Р = 18xi 4- 25х2 + ЗОхз при условиях xi + х2 + хз 20, lOOxi + 150х2 + 200х3 3200, xi, х2, хз > 0, Ни одного студента, 16 аспирантов и 4 сотрудника. Максимальное количество опросов равно 520. Практикум 5.5 —5 0 1. 3 -1 8 3. [1 —2 0 4] 2 5 1 2 -6 0 -1 0 1 3 9. 1) Максимизировать Р = 4^/i -I- 5j/2 при условиях yi 4- 2у2 8, 32/1 4- 2/2 9, 2/1, 2/2 0. 2) 2/1 +22/2 4-xi =8, 32/1+ 2/2 + х2 =9, -42/1-51/2 + Р=0. У1 У2 Х2 Р 1 2 10 0 8 ' 3) 3 10 10 9 —4 —5 б б 1 0 11. 1) МахР = 121 При yi = 3 и у2 = 5. 2) Min С = 121 при xi = 1 и х2 = 2. -1 0 8 4 2 0-1 —7 1 3
810 Ответы к упражнениям 13. 1) Максимизировать Р = 131/1 + 121/2 при условиях 4?/i + 3?/2 9, У1+У2^ 2, У1, У2 > 0. 2) Min С = 26 при xi = 0 и х2 = 13. 15. 1) Максимизировать Р = 151/1 + 81/2 При УСЛОВИЯХ 21/1 + Т/2 7, 3?/1 + 2у2 12, 2/1, У2 0. 2) Min С = 54 при xi = 6 и т2 = 1. 17. 1) Максимизировать Р = 8i/i + 4г/2 при условиях 21/1 - Ъу2 11, У1 + 31/2 4, Уъ У2 > 0. 2) Min С = 32 при xi = 0 и х2 = 8. 19. 1) Максимизировать Р = 6yi + 4ту2 при условиях - 31/1 + у2 7, У1 ~ %У2 9, У1, У2 0. 2) Оптимального решения нет. 21. Min С = 24 при #1 = 8 и я2 = 0. 23. Min С = 20 при xi = 0 и х2 = 4. 25. Min С = 140 при #1 = 14 и х2 = 0. 27. Min С = 44 при xi = 6 и х2 = 2. 29. Min С = 43 при xi = 0, ж2 = 1 и ж3 = 3. 31. Оптимального решения нет. 33. Две переменные и четыре ограничения. 35. Два ограничения и любое количество переменных. 37. Нет. Двойственная задача не является стандартной задачей максимизации. 39. Да. Умножьте обе части неравенства на —1. 41. Min С — 44 при xi = 0, х2 = 3 и хз = 5. 43. Min С = 166 при х\ = 0, Х2 = 12, х3 = 20 и х± = 3.
Ответы к упражнениям 811 45. а) Пусть х\ — количество рабочих часов на заводе в Седарбурге, — количество рабочих часов на заводе в Графтоне, х$ — количество рабочих часов на заводе в Вест- Бенде. Максимизировать С = 70xi -И 75х2 + 90хз при условиях 20^1 + 10х2 4- 20хз > 300, lOxi 4- 20х2 4- 20хз > 200, xi, я2, > 0, 10 ч работы в день на заводе в Седарбурге, 5 ч работы в день на заводе в Вест-Бенде, простой на заводе в Графтоне. Минимальная величина затрат равна 1150 долл. б) 15 ч работы в день на заводе в Вест-Бенде, простой на заводах в Седарбурге и Графтоне. Минимальная величина затрат равна 1350 долл. в) 10 ч работы в день на заводе в Графтоне, 10 ч работы в день на простой на заводе в Вест-Бенде и простой на заводе в Седарбурге. Минимальная величина затрат равна 1650 долл. 47. а) Пусть xi — количество дисководов для носителей стандартной емкости, заказанных в компании Associated Electronics, х2 — количество дисководов для носителей повышенной емкости, заказанных в компании Associated Electronics, хз — количество дисководов для носителей стандартной емкости, заказанных в компании Digital Drivers, Х4 — количество дисководов для носителей стандартной емкости, заказанных в компании Digital Drivers. Минимизировать С = 250xi 4- 350х2 4- 290хз 4- 320x4 при условиях xi 4- х2 < 1000, х3 + х4 2000, xi 4-хз > 1200, х2 4- Х4 > 1600, Х1, Х2, Хз, Х4 > 0, 1000 дисководов для носителей стандартной емкости, заказанных в компании Associated Electronics, 200 дисководов для носителей стандартной емкости и 1600 дисководов для носителей повышенной емкости, заказанных в компании Digital Drivers. Минимальная стоимость закупки равна 820 000 долл. 49. Пусть xi — количество унций блюда А, х2 — количество унций блюда Б, хз — количество унций пищи В. Минимизировать С = 20xi 4- 24х2 4- 18хз при условиях 20xi + 10х2 4- Юхз > 300, lOxi 4* Юх2 4- Юхз > 200, lOxi 4- 15х2 4- Юхз > 240, Xi, х2, хз > 0, 10 унций блюда А, 8 унций блюда Б и 2 унции блюда В. Минимальное потребление холестерина составляет 428 единиц.
812 Ответы к упражнениям 51. Пусть xi — количество учеников, переведенных из Северного округа в Центральный, Х2 — количество учеников, переведенных из Северного округа в Вашингтон, хз — количество учеников, переведенных из Южного округа в Центральный, х± — количество учеников, переведенных из Северного округа в Вашингтон. Минимизировать С = 5xi + 2x2 + Зхз + 4x4 при условиях Х1+Х2 300, хз 4- Х4 500, 4-х3 ^ 400, Х2 4- £4 С 500, X1, Х2, Хз, Х4 0, 300 учеников следует перевести из Северного округа в Центральный, 400 — из Южного округа в Центральный и 100 — из Южного округа в Вашингтон. Минимальная величина затрат на перевозки равна 2200 долл. Практикум 5.6 1. 1) Максимизировать Р = 5xi + 2x2 — Mai при условиях + 2x2 + 51 — 12, #1+ Х2 — s2 + ai = 4, xi, а?2, Si, 32, cti 0. 2) Xi Х2 31 32 О1 Р 1 2 10 0 0 1 10-110 12 ‘ 4 -М-5 -М — 2 0 М 0 1 -Ш 3) х\ — 12, Х2 = 0, si = 0, з2 = 8, ai = 0, Р = 60. 4) МахР = 60 при xi = 12 и #2 = 0. 3. 1) Максимизировать Р = 3xi 4- 5^2 — Mai при условиях 2xi Х2 4- si —8, Х14"Х2 4-ai = 6, Х11 £2? 31, О1 > 0. Xi Х2 31 О1 Р 2) 2 110 0 1 10 10 8 6 -М - 3 -М - 5 0 б 1 -6М 3) xi = 0, Х2 = б, si = 2, ai = 0, Р = 30. 4) Мах Р = 30 при xi = 0 и Х2 = 6. 5. 1) Максимизировать Р = 4xi 4- 3x2 — Mai при условиях - Xi -I- 2х2 4- 31 =2, £14- £2 -32-Fai = 4, Xi, Х2, 31, 32, О1 0.
Ответы к упражнениям 813 2) Xi Х2 31 32 О'! Р -1 21000 1 10-110 2 ' 4 -М-4 -М-3 0 М 0 1 —4М 3) Оптимального решения нет. 4) Оптимального решения нет. 7. 1) Максимизировать Р = 5xi + 10x2 — Ма\ при условиях Xi 4- Х2 + si = 3, 2xi 3^2 — 32 4~ di = 12, Xi, Х2, 31, 32, (11 0. 2) Xi Х2 31 32 CLi Р 1 110 0 0 2 30-110 3 ' 12 -2М-5 -3M-W б М б 1" -12М 3) xi = 0, х2 = 3, si = 0, 32 = 0, ai = 3, Р = — ЗМ 4- 30. 4) Оптимального решения нет. 9. Min Р = 1 при xi = 3 и Х2 = 5. Мах Р = 16 при xi = 8 и х2 = 0. 11. МахР = 44 при Xi = 2 и Х2 = 8. 13. Оптимального решения нет. 7 3 15. Min С* = —9 при Xi = 0; Х2 = -г и Хз = т- 4 4 17. МахР = 32 при xi = 0, Х2 = 4 и хз = -2. 35 15 19. МахР = 65 при Xi = —; Х2 = 0 и х3 = —. 21. МахР = 120 при xi = 20, Х2 = 0 и х3 = 20. 23. Задача 5: неограниченная область допустимых решений. Задача 7: пустая область допустимых решений.
814 Ответы к упражнениям 25. Min С = —30 при = 0; х2 = - и х$ = 0. 49 22 27. МахР = 17 при х\ = —; и х3 = —. ... 135 15 3 29. МшС = —- при xi = —; х2 = - и х% = 0. 2 4 4 31. МахР = 372 при х± = 28, х2 = 4 и х$ = 0. 33. Пусть xi — количество модулей емкостью 16 Кбайт, х2 — количество модулей емкостью 64 Кбайт. Максимизировать Р = 18^1 + ЗОхз при условиях lOxi 4- 15x2 < 2200, 2xi + 4x2 С 500, Xi 50, Xi, Х2 0, 130 модулей емкостью 16 Кбайт и 60 модулей емкостью 64 Кбайт. Максимальная прибыль равна 4140 долл. 35. Пусть xi — количество рекламных объявлений, размещенных в газете Sentinel, х2 — количество рекламных объявлений, размещенных в газете Journal, хз — количество рекламных объявлений, размещенных в газете Tribune. Минимизировать С = 200xi 4- 200x2 4- ЮОхз при условиях xi 4- х2 4- Хз < 10, 2000x1 4- 500x2 4- 1500х3 16 000, > 0. Следует разместить два объявления в газете Sentinel и восемь объявлений в газете Tribune. В газете Journal ни одного объявления размещать не нужно. Минимальная величина затрат равна 1200 долл. 37. Пусть xi — количество порций блюда А, х2 — количество порций блюда Б, х$ — количество порций блюда В. Минимизировать С = 0,6xi 4- 0,4х2 4- 0,9хз при условиях lOxi 4- Юх2 4- 20хз > 10, 2xi 4- 3x2 4- 4хз $ 24, ^1,X2,X3 > 0. Ни одной порции блюда А, четыре порции блюда Б и три порции блюда В. Минимальная величина затрат равна 4,30 долл. 39. Пусть xi — количество куб. ярдов смеси А, х2 — количество куб. ярдов смеси Б, х$ — количество куб. ярдов смеси В. Минимизировать Р = 12xi 4- 16x2 4- 8x3 при условиях 12xi 4- 8x2 4- 16хз < 700, 16xi 4- 8x2 4- 16х3 800, xi, х2, х3 0,
Ответы к упражнениям 815 25 куб. ярдов смеси Л, 50 куб. ярдов смеси Б и 0 куб. ярдов смеси В, Максимальное количество азота равно 1100 фунтов. 41. Пусть xi — количество рам для легковых автомобилей, выпущенных на заводе в Милуоки, #2 — количество рам для грузовых автомобилей, выпущенных на заводе в Милуоки, хз — количество рам для легковых автомобилей, выпущенных на заводе в Рейсине, х* — количество рам для грузовых аввтомобилей, выпущенных на заводе в Рейсине. Минимизировать С = 50xi + 70x2 + 50х3 + 70x4 при условиях -I- хз < 250, Х2 4" Хд 350, #1+^2 С 300, хз + #4 200, 150^1 + 200x2 50 000, 135х3 + 180х4 35 000, Xi, Х2, Хз, Х4 0. 43. Пусть xi — количество баррелей компонента А, использованного при производстве обычного бензина, хг — количество баррелей компонента А, использованного при производстве бензина повышенного качества, х3 — количество баррелей компонента Б, использованного при производстве обычного бензина, Х4 — количество баррелей компонента Б, использованного при производстве бензина повышенного качества, Х5 — количество баррелей компонента В, использованного при производстве обычного бензина, Хб — количество баррелей компонента В, использованного при производстве бензина повышенного качества. Максимизировать Р = lOxi + 18x2 4- 8х3 + 4x4 + 4х3 -И 12xg при УСЛОВИЯХ Х1 4" х2 s ; 4оооо, Хз 4“ Х4 ; 25000, Хз 4“ xq < ; 15000, Х1 4- хз 4- хз J i 30000, Х2 4- Х4 4“ Xg s 25000, -5x1 4- 5х3 + 15х5 J ? 0, —15x2 —5х4 + 5х6; г 0, Xi, Х2, Х3, Х4, Х5, Xg г 0. 45. Пусть xi — доля инвестиций в высокотехнологичные фонды, Х2 — доля инвестиций в глобальные фонды, х3 — доля инвестиций в корпоративные облигации, Х4 — доля инвестиций в муниципальные облигации, х3 — доля инвестиций в депозитные сертификаты. Минимизировать Р = 0,1 Ixi -I- 0,1X2 + 0,09х3 + 0,08x4 -I- 0,05х3 при УСЛОВИЯХ Х1 + Х2 + Хз 4- Х4 + Х5 = 1; 2,7xi 4- 1,8x2 -И 1,2хз 4- 0,5x4 1,8; Х5 > 0,2; Xi, Х2, Хз, Х4, Хз 0.
816 Ответы к упражнениям 47. Пусть х\ — количество унций блюда Л, — количество унций блюда Б, х^ — количество унций блюда В. Минимизировать С = 0,4xi + 0,6^2 -I- 0,8хз при условиях 30xi -|- 10x2 4“ ЗОхз 400, lOxi 4~ 10x2 4“ Юхз 200, lOxi + 30х2 + 20х3 300, 8xi 4- 4x2 + бхз 150, 60xi 4- 40x2 4- 50хз 900, Xi, х2, хз 0. 49. Пусть xi — количество учеников из города А, принятых в школу I, Х2 — количество учеников из города А, принятых в школу II, хз — количество учеников из города Б, принятых в школу I, Х4 — количество учеников из города Б, принятых в школу II, х5 — количество учеников из города В, принятых в школу I, xq — количество учеников из города В, принятых в школу II. Минимизировать С = 4xi 4- 8x2 4- 6x3 4- 4x4 4- ЗХ5 4- Эхе при условиях Xi 4-Х2 Хз + Х4 = 500, = 1200, Х5 +хд = 1800, Х1 + х3 + х5 ^ 2000, Х2 + Х4 “1“ Xq 5^ 2000, Х\ + Хз + Xq 1400, Х2 + х4 + Xq 1400, Х1 < 300, Х2 < 300, Хз 720, Х4 < 720, х5 < 1080, Xq < 1080, Х1, Х2, Хз, Э ?4, Xq, Xq 0. Упражнения для повторения 1. Ограниченная область. 2. Неограниченная область.
Ответы к упражнениям 817 (5.2) 3. МахР = 24 при Xi = 4 и Х2 = 0. (5.2) 4. 2xi 4~ Х2 И- 51 = 8, Х1 + 2х2 + 52 = 10. (5.5) 5. Две базисные и две небазисные переменные. (5.5) 6. Х1 31 «2 Допустимое решение? 0 0 8 10 Да 0 8 0 -6 Нет 0 5 3 0 Да 4 0 0 6 Да 10 0 -12 0 Нет 2 4 0 0 Да Включаемая переменная I Xi Х2 51 S2 Р Исключаемая переменная — S1 1 1 0 0 8 ‘ 7. 32 1 2 0 1 0 10 Р -6 —2 0 0 1 0 (5.4) 8. МахР = 24 при Xi = 4 и х2 = 0. (5.4) 9. Базисные переменные: хг, 52, 53, Р; небазисные переменные: xi, х3, si. Включаемая переменная Исключаемая переменная Х1 Х2 1 хз 3 31 -1 $2 0 Зз 0 р 0 20 ' 31 2 32 0 4 1 0 0 0 30 зз 2 0 5 2 1 1 0 10 Р . -8 0 -5 3 0 0 1 50 Х2 ■ 0 1 —2 -3 0 -1 0 10 “ 32 0 0 7 2 —2 1 _з 2 0 15 Xi 1 0 5 2 1 0 1 2 0 5 Р L б 0 15 11 0 4 1 90 J (5.4) 10. a) xi = 0, Х2 = 2, $i = 0, 52 = 5, Р = 12; требуется дополнительная итерация. б) xi = 0, Х2 = 0, si = 0, 52 = 7, Р = 22; дополнительная итерация не нужна. в) xi = 6, х2 = 0, 51 = 15, 52 = О, Р = 10; оптимальное решение. (5.4) 11. Min С = 40 при xi = 0 и Х2 = 20. (5.5)
818 12. 13. 14. 15. 16. 17. 18. 19. 20. 24. Ответы к упражнениям Максимизировать Р = 15j/i + 20уг- при условиях 1/1 + 21/2 5, З2/1 + 2/2^2, 2/1, 2/2 > 0. (5.5) 2/1 + 2у2 +xi 31/1 + 2/2 —151/1 — 201/2 + х2 = 5, = 2, + Р=0. (5.5) 2/1 2/2 Xi X2 p 1 2 1 0 0 5 3 1 0 1 0 2 -15 -20 0 0 1 0 (5.5) МахР = 40 при yi = 0 и у2 = 2. (5.4) Min С = 40 при xi = 0 и х2 = 20. (5.5) МахР = 26 при Xi = 2 и х2 = 5. (5.2) МахР = 26 при Xi = 2 и х2 = 5. (5.4) Min С = 51 при Xi — 9 и х2 = 3. (5.2) Максимизировать Р = 10j/i -I- 151/2 + З3/3 при условиях 2/1 + 2/2 3, 2/1+22/2+2/3^ 8, 2/1, 2/2, 2/з > 0- (5.5) 1) Модифицированная задача. Максимизировать Р = Xi + 3^2 — Mai при условиях Xi + х2 — si + ai = 6, Xi + 2х2 + s2 = 8, xi, #2, si) 52, ai > 0. (5.5) 2) Предварительная симплекс-таблица. Xi 1 1 _ -1 -3 0 M 0 1 Исходная симплекс-таблица. x2 «1 1 -1 2 0 ai s2 P 1 0 0 0 1 0 Xi X2 51 ai s2 P 1 1 -1 1 0 0 6 1 2 0 0 1 0 8 -M - 1 -M -3 M 0 0 i -6M _ 3) Xi = 4, x2 = 2, si - 0, ai = = 0, 82 = 0, , P = 10. 4) Поскольку ai = 0, оптимальным решением исходной задачи является max Р = 10 при Xi = 4 и х2 = 2. (5.6) 6 ' 8 0
Ответы к упражнениям 819 25. 1) Модифицированная задача. Максимизировать Р = х\ + х2 — Ма\ при условиях xi + Х2 — si + ai = 5, xi + 2x2 + 82 = 4, Х1, Т2, «1, 82, 0. 2) Предварительная симплекс-таблица. Х1 Х2 81 Qi 82 Р 1 1 -1 1 0 0 5 ■ 1 2 0 0 1 0 4 /-1' "1 ■" б'~ М ' о"Т]’о ' Исходная симплекс-таблица. х2 81 ai 82 р 1 1 -1 1 0 0 5 1 2 0 0 1 0 4 -М-1 -М-1 м 0 0 1 -5М 3) д?1 = 4, х2 = 0, si = 0, s2 = 0, ai = 1, Р = — М + 4. 4) Поскольку ai 0, оптимального решения не существует. (5.6) 26. Максимизировать Р = 2a?i + Зх2 + #з — Ма\ — Ма2 при условиях х\ — Зя2 + Яз + 81 = 7, Х\ + Х2 — 2д?з — s2 -f- о-i = 2, Зд?1 + 2х2 - хз + а2 = 4, Х2, хз, 8i, 82, ai, а2 0. (5.6) 27. Геометрический метод решает задачи минимизации и максимизации с помощью двух дополнительных переменных. Если область допустимых решений ограничена, никаких ограничений на коэффициенты и константы не существует. Если область допустимых решений не ограничена, а коэффициенты целевой функции положительны, задача минимизации имеет решение, но задача максимизации — нет. (5.2) 28. Базовый симплекс-метод с фиктивными переменными решает стандартную задачу максимизации с учетом ограничений вида “меньше или равно” с неотрицательными константами в правой части. (5.4) 29. Двойственный метод решает задачу минимизации с положительными коэффициентами целевой функции. (5.5) 30. М-метод решает любую задачу линейного программирования. (5.6) 31. МахР = 36 при хг = 6 и ж2 = 8. (5.2)
820 Ответы к упражнениям 32. Min С = 15 при xi = 3 и Х2 = 3. (5.5) 33. Min С = 15 при xi = 3 и х2 = 3. (5.5) 34. Min С = 9960 при хг = 0, х2 = 240, хз = 400 и х± = 60. (5.5) 35. Пусть xi — количество обычных парусов, х2 — количество спортивных парусов. Максимизировать Р = 100xi + 200x2 при условиях 2xi 4- 3x2 С 150, 4xi + 10x2 380, xi, х2 0. а) Мах Р = 8500 долл, при выпуске 45 обычных и 20 спортивных парусов. б) Максимальная прибыль возрастает до 9880 долл, при выпуске 38 спортивных и отказе от производства обычных парусов. в) Максимальная прибыль падает до 7500 долл, при отказе от выпуска спортивных парусов и производстве 75 обычных парусов. (5.2) 36. Пусть xi — объем инвестиций в акции нефтяных компаний, х2 — объем инвестиций в акции металлургических комбинатов, х$ — объем инвестиций в правительственные облигации. Максимизировать Р = 0,12xi + 0,09^2 + 0,05хз при условиях xi + х2 + х3 < 150 000, xi ^ 50000, £1 +^2 -^з 25 000, xi, х2, х3 0. а) Максимальный доход равен 12 500 долл., если 50 000 долл, вложено в акции нефтяных компаний, 37 500 долл. — в акции металлургических комбинатов, и 62 500 долл. — в правительственные облигации. в) Максимальный доход равен 13 625 долл., если 87 500 долл, вложено в акции металлургических комбинатов и 62 500 долл. — в правительственные облигации. (5.4) 37. Пусть xi — количество электродвигателей, выпущенных на заводе А и проданных заводу X, х2 — количество электродвигателей, выпущенных на заводе А и проданных заводу У, х3 — количество электродвигателей, выпущенных на заводе Б и проданных заводу X, Х4 — количество электродвигателей, выпущенных на заводе Б и проданных заводу Y. Минимизировать С = 5xi + 8x2 + 9хз + 7x4 при условиях xi + х2 < 1500, хз + 1000, xi +х3 ^ 900, х2 -Нх4^ 1200, xi, х2, Х3, Х4 0. Min С = 13100 долл., если с завода А на завод X поставляются 900 электродвигателей, на завод Y — 200 электродвигателей, а 1000 электродвигателей, выпущенных заводом Б поставляются заводу У. (5.5)
Ответы к упражнениям 821 38. Пусть xi — количество фунтов удлиненного риса в смеси А, Х2 — количество фунтов удлиненного риса в смеси Б, х$ — количество фунтов канадского риса в смеси Л, Х4 — количество фунтов канадского риса в смеси Б. Максимизировать Р = 0,8xi + 0,5x2 — 1,9хз — 2,2x4 при условиях 0,1^1 — 0,9хз < 0, 0,05x2 — 0,95x4 < 0, xi + Х2 8000, хз + Х4 < 500, Xi, Х2, Хз, Х4 0. Максимальная прибыль, равная 3350 долл., достигается, если для производства 1500 фунтов смеси Я используются 1350 г удлиненного риса и 150 фунтов канадского риска, а для производства 7000 фунтов смеси Б используются 6650 г удлиненного риса и 350 фунтов канадского риска. (5.4) 39. а) Пусть xi — количество граммов смеси Л, Х2 — количество граммов смеси Б. Минимизировать С = 0,04xi + 0,09x2 при условиях 2xi + 5x2 > 850, 2xi + 4x2 > 800, Xi, Х2 > 0. Min С = 16,50 долл, при использовании 300 г смеси Л и 50 г смеси Б. б) Минимальная стоимость уменьшается до 13,00 долл, при использовании 100 г смеси Л и 150 г смеси Б. в) Минимальная стоимость возрастает до 17 долл, при использовании 425 г смеси Л и отказе от смеси Б. (5.2) Глава 6 Практикум 6.1 1. 115. 5. 165. 9. 95. 13. Четыре способа. 3. 300. 7. 210. 11. 260. Первая монета Вторая монета Совместные (исходы) (исходы) (исходы) (исходы) Начало О О Р О Р (О, О) (О, Р) (Р,О) (Р.Р) 2-2 = 4 способа.
822 Ответы к упражнениям 15. 12 совместных исходов. Начало 2 • 6 = 12 способа. 17. а) 9. б) 18. 19. а) 8. б) 144. 21. п(А П В') = 60, п(А П В) = 20, п(А' П В) = 30, п(А' А В') = 90. 23. п(А Г) В') = 5, п(А А В) = 20, п(А' А В) = 35, п(А' А В') = 40. 25. п(А А В') = 15, п(А П В) = 70, п(А' А В) = 40, п(А' А В') = 25. 27. А А' Всего В 30 60 90 В' 40 70 110 Всего 70 130 200 29. А А' Всего в 20 35 55 В' 25 20 45 Всего 45 55 100 31. А А' Всего в 58 8 66 В' 17 7 24 Всего 75 15 90 33. а) Истина. б) Ложь. 35. 5 • 3 • 4 • 2 = 120. 37. 6 • 5 • 4 • 3 = 360. 6 • 6 • 6 ■ 6 = 1296. 6 ■ 5 • 5 • 5 = 750. 39. 10 • 9 • 8 • 7 • 6 = 30240. 10 -10 -10 • 10 • 10 = 100000. 10 • 9 • 9 • 9 • 9 = 65610. 41. 26 • 26 • 26 • 10 • 10 • 10 = 17576000. 26 • 25 • 24 • 10 ■ 9 • 8 = 11232000.
Ответы к упражнениям 823 43. Не имеет. В любом случае возможны одни и те же восемь вариантов выбора. 4S. 14. 47. 14. 49. а) Шесть совместных исходов. б) 3 • 2 = 6. 51. 12. 53. а) 1010. б) 190. в) 270. 55. 1570. 57. а) 102. б) 289. в) 1470. г) 1372. 59. а) 12 классификаций. б) 2 • 2 • 3 = 12. 61. 2905. Практикум 6.2 1. 5040. 3. 90. 5. 970200. 7. 20. 9. 126. И. 1260. 13. 210. 15. 1. 17. 22 100. 19. 132600.
824 Ответы к упражнениям 21. 0,000495. 23. 0,390156. 25. Перестановки. 27. Сочетания. 29. Ни то, ни другое. 31. Рю,з = 10 • 9 • 8 = 720. 33. С7,з = 35; Р10,з = 210. 35. Факториал а:! растет намного быстрее показательной функции 3®, которая, в свою очередь, растет намного быстрее кубической функции х3. 37. С1з,5 = 1287. 39. Си,5(713,2 = 100 386. 41. С8,3(710,4(77,2 = 246960. 43. Симметрия является следствием равенства Сп<г = Сп, п-г- 45. а) С8>2 = 28. б) <78,з = 56. в) С8,4 = 70. 47. Р5,2 = 20; Р5,3 = 60; Р5,4 = 120; Р5,5 = 120. 49. а) Р8,5 = 6720. б) С8,5 = 56. в) 2 • Сб,4 = 30. 51. Во многих калькуляторах к — 69, однако ваш калькулятор может отличаться от других. 53. (724,12 = 2 704156. 55. а) <724,з = 2024. б) (719,3 = 969. 57. а) Сзодо = 30045015. б) (78,2(712,5^10,3 = 2 661 120. 59. а) (76,з<75,2 = 200. б) С6,4<75,1 = 75. в) (76)5 = 6. г) Сц>5 = 462. д) (7б,4^5,1 + Сб,5 = 81. 61. 336; 512. 63. Р4,2 = 12. 5. 1. ч- ■ч Практикум 6.3 1. Событие Е является достоверным. ч ч ”■ Гв- 15. а) Отбросить. Вероятность не может быть отрицательной. б) Отбросить. Р( J) + P(G) + Р(Р) + P(S) / 1. в) Принять.
Ответы к упражнениям 825 «4 23. С26,5/С52,5 « 0,025. 17. Р( J) + Р(Р) = 0,56. 21. 1/Рю.з = 0,0014. 25. С12,5/С52,5 ~ 0,000305. 27. S = Все 365 дней, за исключением високосных лет. при условии, что все дни года с одинаковой вероятностью могут быть днем рождения студента. 29. 1/Р5 5 = 1/5! = 0,00833. - ' 4L 5- «• I «■ j- 53. а) Да. б) Да, поскольку в среднем в 40 испытаниях должно выпасть 20 орлов. Р(Н) = |§ = 314 39. 0. «■I 474 574 ‘Ч 65. 48/С52,5 « 0,000018. 69. С4,2С52,5/С52,5 « 0,000009. = 0,925. Р(Т) = ® = 0,075. 554 *4 63. С16,5/С52,5 « 0,00168. 67. 4/0*52,5 « 0,0000015. 7 71.а)- = 0,14. б) | «0,167. в) Ответ зависит от результатов моделирования. 73. а) Обозначить исходы ОпР цифрами 1 и 2 соответственно и выбрать 500 случайных целых чисел, равных 1 или 2. б) Ответ зависит от результатов моделирования. в) 0,5. 75. а) 1/Р12,4« 0,000084. б) 1/124 = 0,000048. 77. а) С6,3С5,2/Сц,5« 0,433. б) C6)4C5)i/C11)5 « 0,162. в) С6,5/Сц,5« 0,013. г) (Сб,4^5,1 + « 0,175. 79. а) 1/Р8,3« 0,0030. б) 1/83 «0,0020.
826 Ответы к упражнениям 81. а) Р6,2/Рц,2« 0,273. 6) (6*5,3 + Сб, 1^5,2)76*11,3 « 0,424. Практикум 6.4 1. 0,15. 5. 0,1. ’4 13. 0,44. (6.5) 17. 0,6. (6.6) ч 3. 0,55. "• й 15. 0,48. (6.6) 19. 0,49. 23. И. 36 25. а) б) в) г) 27. а) б) 3 5 5’ 3’ 1 : з’ ' 2 ; з;: п ¥ ’ 3_ 1Г 11 18 4 5’ 0,49. Ложь. Истина. 3 Г 3 2 9 ’ 1Г 29. а) б) 31. 1 : 1. 35. 2 : 1. 39. .) 1. б) 8 долл. 4L а) 0,31; g. 43. 11. 26 15 33. 7 : 1. 37. 1 : 2. 45. —; Z. 13’ 6 49. =0,25. 1000 47. 0,78. 51. Либо события А, В и С являются взаимоисключающими, либо невзаимоисключающими являются только события АиВ.
Ответы к упражнениям 827 53. Во-первых, эта формула предусматривает только несколько этапов вычислений, а во- вторых, во многих калькуляторах вычисление числа 365! приводит к переполнению, в то время как функция Рзбб.п вызывает переполнение только при очень больших значениях п. 191 55. Р (Е) = 1 - — . v 7 (12- п)!12п 10 10 20 „ , 59-а) 50 + 50 = 50 = °Л б) — + — = — « 0,306. ' 36 36 36 в) Ответ зависит от результата моделирования. 61. а) Р(С U S) = Р(С) + P(S) - Р(С П S) = 0,45 + 0,75 - 0,35 = 0,85. б) Р(С" П 5') = 0,15. 63. a) P(Afi U А) = P(Mi) + Р(А) - Р(ЛЛ П А) = 0,2 + 0,3 - 0,05 = 0,45. б) Р([(М2 П Л') U (М3 П А') = Р(М2 П А') + Р(М3 П А') = 0,2 + 0,35 = 0,55. 65. Р(А" П D') = 0,9. 67. 0.83. 69. Р(А П 5) = = 0,05. v 7 1000 71. a) P(U U N) = 0,22; 7 б) Р ([(£> П A) U (Я П А)]) = 0,3; -. 73. 1 - С15,з/С20,3 « 0,6. Практикум 6.5 1. 0,30. 3. 0,04. 0,04 5- «0,133. 7.0,2. 0,30 9. 0,3. 11. 0. 13. Независимые. 15. Зависимые. 17, а) i б) 2 • (i)8 « 0,00781. 19. а) 1. б) Зависимые. 21. а) 0,18. б) 0,26. 23. а) Независимые и невзаимоисключающие. б) Зависимые и взаимоисключающие. 1 1 _ 1 1 1 1 - 3 5’ 2 ' 2 “ 4’ 2 + 2 4 “ 4’
828 Ответы к упражнениям 27. а) 7 4? « 0,0637. 4 51 б) 0,0625. 4 4 2’- а> п б) Независимые. 31. а) Зависимые. б) Независимые. Р(Л. п л2) Р(Л1 n w2) =Ц РОГхП Л2) = $ W Ж2) = Л P(RlCiR1)=ji Р(№ ^2) = я P(^n Л2) = ji POFjfl W2) = $ 35. a) 6) 37. a) 6) 24 49 11 21* Ложь. Ложь. 39. £ 18’ 41. a) 0,167. б) 0,25. в) 0,25. 45. P(A | A) = P(A П A)/P(A) = P(A)/P(A) = 1. 47. P(A)P(B) / 0 = P(A П P).
Ответы к упражнениям 829 49. а) н S В Всего У 0,400 0,180 0,020 0,600 N 0,150 0,120 0,130 0,400 Всего 0,550 0,300 0,150 1,000 б) Р(У |Я) = ^«0,727. U,ooU Р1У В> =“л50“‘ИМ г) Р(5) = 0,300; P(S I У) = 0,300. д) Р(Я) = 0,550; Р(Я | У) и 0,667. е) Р(В П N) = 0,130. ж) Да. з) Нет. и) Нет. 51. а) 0,167. б) 0,25. в) 0,25. ’ С С Всего R 0,06 0,44 0,5 R' 0,02 0,48 0,5 Всего 0,08 0,92 1,00 б) Зависимые. в) Р(С | R) = 0,12 и Р(С) = 0,08. Поскольку Р(С | Я) > Р(С), краситель следует запретить. г) Р(С | Я) = 0,04 и Р(С) = 0,08. Поскольку Р(С | Я) < Р(С), краситель не следует запрещать, так как, по-видимому, он предотвращает рак. 55. а) А В С Всего F 0,130 0,286 0,104 0,520 F' 0,120 0,264 0,096 0,480 Всего 0,250 0,550 0,200 1,000 б) Р(А | F) = = 0,250; Р(А | F') = = 0,250. в) Р(С | F) = = 0,2°0; Р(С | F') = = 0,200. ' v 1 ' 0,520 ’ v 1 ’ 0,480 г) Р(Д) = 0,250. д) Р(В) = 0,550. Р(Я | F') = 0,550. е) P(F Г) С) = 0,104. ж) Нет. События А, В и С не зависят от событий F и F'. Практикум 6.6 1. 0,6 • 0,6 = 0,48. 3. 0,6 0,8+ 0,4 0,3 = 0,60.
830 5. 0,80. 9. 0,375. 13. 0,50. 17. 0,125. 21. 0,375. 25. 0,25. 29. 0,50. Ответы к упражнениям 7. 0,417. 11. 0,222. 15. 0,278. 19. 0,50. 27. 0,333. 31. 0,745. 33. События MnU являются независимыми. Это следует из приведенных ниже результатов. Р(М \U) = c = Р(М); P(U \М)=а = P(U); P(MdU) = са = P(M)P(U). 35. а) Ложь. б) Истина. 37. 0,235. 39. 0,052. Р (Ui ПЯ) Р П R) _ Р (Ui Г\ R) + Р (U[r\ R) _ P(R) _ ’ Р (R) + Р (Я) - Р (Я) “ Р(Я) “ ’ 43. 0,913; 0,226. 45. 0,091; 0,545; 0,364. 47. 0,667; 0,000412. 49. 0,231; 0,036. 51. 0,941; 0,0588. Практикум 6.7 1. Е(Х) = -0,1. 3. Распределение вероятностей. Е(Х) = 1- 5. Таблица выигрышей. Xi 0 1 2 1 1 i Pi 4 2 4 Xi, долл. 1 — 1 Pi 1 1 2 2 Е(Х) = 0. Игра является справедливой.
Ответы к упражнениям 831 7. Таблица выигрышей. Xi, долл. -3 -2 -1 0 1 2 Pi 1 1 1 1 1 1 6 6 6 6 6 6 Е(Х) = — 50 центов. Игра является несправедливой. 9. -0,50 долл. 11. —0,036 долл.; 0,036 долл. 13. 40 долл. Обозначим символом х величину проигрыша при выпадении шестерки. Заполним таблицу выигрышей. Затем устанавливаем, что математическое ожидание выигрыша равно нулю, и определяем величину х. 15. Выигрыш должен быть равен одному доллару. 17. -0,154 долл. 19. 2,75 долл. 21. Аг; 210 долл. 23. Таблица выигрышей. х^ долл. 35 —1 гг 38 38 Е(Х) = -5,26 цента. 25. 0,002. 27. Таблица выигрышей. Xi, долл. 499 99 19 4 —1 Pi 0,0002 0,0006 0,001 0,004 0,9942 Е(Х) = -80 центов. 29. а) 0 1 2 v. J- X- X Рг 15 15 15 б) 0,60. 31. а) Xi, долл. —5 195 395 595 Pi 0,985 0,0149 0,0000599 0,0000бббб~ б) Е(Х) « —2 долл. 33. а) -92 долл. б) Средний выигрыш за игру равен = —0,46 долл. Математическое ожидание выигрыша равно —0,0526. в) Величина выигрыша или проигрыша зависит от результатов моделирования. Ожидаемый проигрыш равен 26,32 долл.
832 Ответы к упражнениям 35. Таблица выигрышей. 4850 -150 Pi 0,01 0,99 Е{Х) = —100 долл. 37. Скважина А с математическим ожиданием, равным Е(Х) = 3,6 млн. долл. 39. 1,54. 41. Для вида агитации Ai математическое ожидание пожертвований равно Е(Х) = = 4 долл., а для вида агитации А2 — 4,80 долл. Таким образом, поквартирный обход с последующими телефонными звонками предпочтительнее. Упражнения для повторения 1. а) 12 составных исходов. б) 6 • 2 = 12. (6.1) 2. а) 65. б) 75. в) 35. г) 105. д) 150. е) 85. ж) 115. з) 45. (6.1) 3. 15; 30. (6.2) 4. 6-5 4-3-21 = 720. {6.1) 5. Р6,6 = 6! = 720. (6.2) 6. « 0,0005. (6.3) 1/Fi5,2« 0,0048. (6.3) 8. l/Pio.3 « 0,0014. 1/Сю.з « 0,0083. (6.3) 9. 0,05. (6.3) 10. Таблица выигрышей, долл. —2—1012 7 1 1 1 1 Г” г1 5 5 5 5 5 Е(Х) = 0. Игра является справедливой. (6.7)
Ответы к упражнениям 833 24. 0,2. (6.5) 26. 0,08. (6.5) 28. 0,26. (6.5) 30. 0,43. (6.6) И. а) 7. б) 0,6. (6.4) 12. P(R U G) = 0,8. Шансы события R U G равны 8 : 2. (6.4) 5 13. — « 0,455. (6.4) 14. 27. (6.5) 15. 0,20. (6.5) 16. 0,02. (6.5) 17. 0,03. (6.5) 18. 0,15. (6.5) 19. 0,1304. (6.5) 20. 0,1. (6.5) 21. Нет, поскольку Р(Т | Z) Р(Т). (6.5) 22. Да, поскольку P(S Г) X) = P(S)P(X). (6.5) 23. 0,4. (6.5) 25. 0,3. (6.5) 27. 0,18. (6.5) 29. 0,31. (6.6) в) По мере увеличения объема выборки эмпирическая функция распределения стремится к теоретической. (6.3) 32. а) Истина. б) Ложь. (6.4) 33. а) Ложь. б) Ложь. (6.5) 34. Таблица выигрышей. Xi, долл. 5 -4 2 Pi 0,25 0,5 0,25 Е(Х) = -25 центов. Игра не является справедливой. (6.7) 35. а) б) (6.5) 36. а) Ь 2 : 11. 4 6>i3;4:9- в) 12 : 1. (6.4) 37. а) 1:8. б) 8 долл. (6.4)
834 Ответы к упражнениям 38. а) Р(Два орла) = 0,21; Р(Один орел) = 0,48; Р(Ни одного орла) = 0,31. б) Р(Два орла) = 0,25; Р(Один орел) = 0,50; Р(Ни одного орла) = 0,25. в) Два орла, 250; один орел, 500; ни одного орла, 250. (6.3, 6.7) 39. Пятеро детей, 15 внуков и 30 правнуков, т.е. 50 наследников. (6.1, 6.2) 40. |. Поскольку монета не имеет памяти, десятое подбрасывание монеты не зависит от предыдущих девяти. (6.5) 41. а) Xi, долл. 2 3 4 5 6 7 8 9 10 11 12 Pi 1 2 3 4 5 6 5 4 3 2 36 36 36 36 36 36 36 36 36 36 36 б) Е(Х) = 7. (6.7) 42. А = {(1; 3), (2; 2), (3; 1), (2; 6), (3; 5), (4; 4), (5;3), (6; 2), (6; 6)}, В = {(1; 5), (2; 4), (3; 3), (4; 2), (5; 1), (6; 6)}; Р(Л) = Р(В) = Р(А ПВ) = Р(А U В) = £• (6.4) 43. а) Вероятность события не может быть отрицательной. б) Сумма вероятностей простых событий должна быть равной единице. в) Вероятность события не может превышать единицу. (6.3) А А1 Всего в 15 30 45 В' 35 20 55 Всего 50 50 100 45. а) 0,6. б) (6.5) О 46. а) ± б) Независимые. (6.5) 47. 336; 512; 392. (6.1) 48. а) Р6,3 = 120. 6) С5’2 = Ю. (6.2) 49. С25,12 = С25дз = 5200300. (6.2) 5°. а) А. б> i (М 51. Пункт 2. (6.5) 52. а) 1,2. 6) 1,2. (6.7) ,3. .) 6)|
Ответы к упражнениям 835 г>л д) о е) ^.(6.5, 6.6) 54. Нет. (6.5) 55. а) С1з,5/С52,5- б) 613,3 • 613,2/6*52,5- (6.3) 56. С6,2/С10А = (6.3) 57. Nr-N2-N3. (6.1) 58. События S и Н являются взаимоисключающими. Следовательно, P(S П Н) = О, поскольку P(S) / 0 и Р(Н) / 0. Таким образом, P(S А Н) P(S)P(H). Отсюда следует, что события S и Н зависят друг от друга. (6.5) 9 59. а) —- = 0,18. □и 9 б) — = 0,25. 7 36 в) Эмпирическая вероятность зависит от результатов моделирования. Теоретическая вероятность равна « 0,139. (6.3) 60. Эмпирическая вероятность зависит от результатов моделирования. Теоретическая вероятность равна » 0,038. (6.3) 61. а) 0,350. б) | = 0,375. в) 375. 62. -0,0172; 0,0172; нет. (6.7) 63. а) Рю.з = 720. б) Рб,з/Р10,3 = g- в) (710,3 = 120. г) (<76,3 + С’6,2С4,1)/С1о,з = |. (6.1, 6.2, 6.3) 64. 33. (6.1) 17 65. 1 - (7713/С10,з = (6.3) АО. 66. » 0,235. (6.5) 51 67. и 0,235. (6.5) 51
836 Ответы к упражнениям 68. а) Xi, долл. 2 3 4 5 6 9 12 10 4 1 36 36 36 36 36 б) ад = у. (6.7) 69. Е(Х) и —0,167 долл.; нет. 10/3 долл. « 3,33 долл. (6.7) 70. 25 = 32; 6. (6.1) 71. а) 1 : 3. б) 3 долл. (6.4, 6.6) 72. 1 - 10!/ (5! 105) « 0,70. (6.4) 73. Да, если г = 0 или г = 1. (6.2) 74. Р(А | В) = Р(В | А), если и только если Р(А) = Р(В) или Р(А П В) = 0. (6.5) 75. Р5,5 = 120. (6.7) 76. а) 610. б) 390. в) 270. (6.1) 77. а) 0,8. б) 0,2. в) 0,5. (6.1, 6.4) 78. Р(А А Р) = Р(А)Р(Р | 4) = 0,34. (6.5) 79. а) Р(А) = 0,290; Р(В) = 0,290; Р(А А В) = 0,100; Р(А | В) = 0,345; Р(В | 4) = = 0,345. б) Нет, так как Р(4 ПВ) / Р(А)Р(В). в) Р(С) = 0,880; P(D) = 0,120; Р(С A D) = 0; Р(С | D) = 0; P(D | С) = 0. г) Да, поскольку С A D = 0. Зависимые, поскольку Р(С Q D) = 0 и P(C)P(D) 0. (6.5) 80. План А: Е(Х) = 7,6 млн. долл.; план Б: Е(Х) = 7,8 млн. долл. (6.7) 81. Таблица выигрышей. Xi, долл. 270 —30 Pi 0,08 0,92 Е(Х) = -6 долл. (6.7) 82. 1 - Cio,4/Ci2,4 « 0,576. (6.2, 6.4) 83. а) Xi, долл. 0 12 v 12 A 22 22 22 б) ад = ±.(6.7) 84. 0,955. (6.6) 85. | « 0,857. (6.6)
Ответы к упражнениям 837 Глава 7 Практикум 7.1 А В 1. Si = [0,8 0,2]. Вероятность оказаться в состоянии А после одного испытания равна 0,8, а вероятность оказаться в состоянии В после одного испытания — 0,8. Исходное Первое состояние состояние А; 1-0,8 + 0-0,4 = 0,8 В: 1-0,2 + 0-0,6 = 0,82 А В 3. Si = [0,6 0,4]. Вероятность оказаться в состоянии А равна 0,6, а вероятность оказаться в состоянии В — 0,4. Исходное Первое состояние состояние А: 0,5-0,8+ 0,5-0,4 = 0,6 В: 0,5-0,2+ 0,5-0,6 = 0,4 5. S2 = [0,72 0,28]. Вероятность оказаться в оказаться в состоянии В — 0,28. А В 7. S2 = [0,64 0,36]. Вероятность оказаться в оказаться в состоянии В — 0,36. состоянии А равна 0,72, а вероятность состоянии А равна 0,64, а вероятность 9. А В А [0,4 0,6' В 0,7 0,3 11. Нет.
838 13. 15. 17. 19. 21. 23. 25. 29. 31. 33. 35. Ответы к упражнениям а = 0,5; b — 0,6; с = 0,7. а = 0,7; b = 1; с = 0,2. Нет. 0,35. 27. 0,212. АВС S2 = [0,43 0,35 0,22]. Вероятности перейти из состояния А в состояния А, В или С за два испытания. АВС S2 = [0,212 0,298 0,49]. Вероятности перейти из состояния С в состояния А, В или С за три испытания. п — 9. А р4 _ А 0,4375 Г “ В [о,375 В 0,5625' 0,625 А ; 54 = [0,425 В 0,575].
Ответы к упражнениям 839 А 4L .) Вс D А 0,36 0,6 0,4 А 0,0154 О 0,0102 О В С 0,16 0,48 АВС О 0,4 ; S4 = [0,452 0,152 0,396]. 0,24 0,Зб] BCD 0,3534 0,0153 0,6159 1 О О 0,2962 0,0103 0,6833 ' О 0 1 А 31. Р4 = В С б) 0,6159. в) 0,2962. г) 0. 45. а) [0,25 0,75]. б) [0,25 0,75]. в) [0,25 0,75]. г) [0,25 0,75]. д) Все матрицы перехода стремятся к одной и той же матрице S = [0,25 0,75], независимо от матрицы исходного состояния. 0,25 0,25 чи 45. 47. Q = 0,75 0,75 Строки матрицы Q совпадают со строками матрицы S из зада- 49. a) R — дождь, R' — нет дождя. б) R 0,4 R’ 0,06 0,6 0,94 в) Суббота: 0,196; воскресенье: 0,12664. X X’ X [0,8 0,2’ 6) X' [О,2 0,8 в) 32%; 39,2%. 53. а) N — National Property, U — United Family, О — другие компании.
840 N б) U О Ответы к упражнениям N U "0,65 0,25 0,1 0,85 0,15 0,35 О 0,1 ' 0,05 0,5 в) 38,5%; 32%. г) 45%; 53,65%. 55. а) В — начинающий агент, I — агент промежуточной квалификации, Т — уволенные агенты, Q — квалифицированные агенты. в) 0,12; 0,3612. НМО РРО FFS НМО ГО,8 0,15 0,05' 57. а) РРО 0,2 0,7 0,1 FFS 0,25 0,3 0,45 б) НМО: 34,75%; РРО: 37%; FFS: 28,25%. в) НМО: 42,2625%; РРО: 39,5875%; FFS: 18,15%. 59. а) Н Н Го,924 R 0,108 R 0,076' 0,892 б) 40,5%. в) 43,8%. Практикум 7.2 1. Регулярная. 5. Регулярная. 9. Регулярная. 13. Регулярная. 15. S= [0,4 0,6]; Р= 17. S =[0,375 0,625]; Р = 3. Нерегулярная. 7. Нерегулярная. 11. Нерегулярная. 0,6 0,6 ’ 0,375 0,625 0,375 0,625
Ответы к упражнениям 841 19. S = [0,3 0,5 0,2], Р = 0,6 0,24 0,6 0,24 0,6 0,24 0,20 0,20 0,20 23. а) Истина. б) Ложь. 25. S = [0,3553 0,6447]. 27. S = [0,3636 0,4091 0,2]. б) 29. а) Красный шар Красная урна 0,98 Синяя урна 0,78 Синий шар 0,02 0,22 в) [0,25 0,75]. В достаточно долгой серии испытаний красная урна будет выбираться в 25% случаев, а синяя — в 75%. 31. а) Матрица состояний изменяется от [0,2 0,8] до [0,8 0,2]. Следовательно, она не стремится ни к одной определенной матрице. б) Все матрицы состояний равны Sq. Следовательно, матрица Sq является стационарной. в) Степени матрицы Р изменяются от матрицы Р до единичной матрицы I. Следовательно, она не стремится ни к одной определенной матрице. г) Пункты 2 и 3 теоремы 7.2 для этой матрицы не выполняются. Поскольку матрица Р не является регулярной, это условие противоречием не является. 33. а) Поскольку матрица Р не является регулярной, может существовать несколько стационарных матриц. б) [0,5 0 0,5] представляет собой другую стационарную матрицу. в) Матрица Р имеет бесконечно много стационарных матриц. 35. Р= 0,25 0 0 0 0,75 0 1 Каждая строка матрицы Р является стационарной матрицей для матрицы Р. 37. а) 0,39; 0,3; 0,284; 0,277. б) Каждый элемент второго столбца матрицы Pfc+1 является произведением строки матрицы Р и второго столбца матрицы Рк. Все элементы следующих столбцов не превосходят Следовательно, их произведение не превосходит 39. 72,5%. 41. a) Si = [0,516 0,484]; S2 = [0,577 0,423]. } Год Доля, % Вычисленная доля, % 1970 43,3 43,3 1980 51,5 51,6 1990 57,5 57,7 в) 74,1%. 43. GTT: 25%, NCJ: 25%, Dash'. 50%. 1 0
842 Ответы к упражнениям 45. Нежелательные: 20%; обычные: 40%; привилегированные: 40%. 47. 51%. 49. Стационарная матрица = [0,25 0,50 0,]. 51. а) [0,25 0,75]. б) 42,5%; 51,25%. в) 60% — на метро, 40% — на автомобилях. Практикум 7.3 1. В, С. 3. Ни одного поглощающего состояния. 5. A, D. 7. В — поглощающее состояние; поглощающая цепочка. 9. С — поглощающее состояние; ни одной поглощающей цепочки. В b А С в ’1 0 0 0 13 D 0 1 0 0 13- А 0,4 0,1 0,3 0,2 С 0,4 0,3 0 0,3 В D А С в ’1 0 0 0 17 D 0 1 0 0 17‘ А 0,2 0,4 0,1 0,3 С 0,2 0,1 0,5 0,2 В А С В ’1 0 0 11. А 0,5 0,2 0,3 С 0,1 0,5 0,4 С А В С ’1 0 0 15. А 0,5 0,2 0,3 В 0,1 1 0 А в С А ’1 0 О’ 19. Р = В 0 1 0 С 0,2 0,8 0 Р(С -► А) = 0,2; Р(С -> В) = 0,8. Чтобы перейти из состояния С в состояние А или В необходимо, в среднем два испытания. А А Г1 21. Р = В 1 С L1 В С 0 О' 0 0 0 0 Р(В —► А) = 1; Р(С —► А) = 1. Чтобы перейти из состояния В в состояние А, необходимо в среднем четыре испытания, а из состояния С в состояние А — три. А 23. Р = ® С ь А ’1 о 0,36 0,44 BCD 0 0 0’ 1 0 0 0,64 0 0 0,56 0 0 Р(С —► А) = 0,36; Р(С —► В) = 0,64; P(D —► А) = 0,44; P(D —► В) = 0,56. Чтобы перейти из состояния С в состояние А или В, необходимо в среднем 3,2 испытания, а из состояния D в состояние А или В — 2,8.
Ответы к упражнениям 843 25. а) [0,2 0,8 0]. б) [0,26 0,74 0]. 27. а) [1 0 0]. б) [1 0 0]. 29. а) [0,44 0,56 0 0]. б) [0,36 0,64 О 0]. в) [0,408 0,597 О 0]. г) [0,384 0,616 О 0]. 31. а) Истина. б) Ложь. А в С А ’1 0 0 „ в 0 1 0 33. с 0,6375 0,3625 0 D 0,7375 0,2625 0 А 9 С ь А 0 0,52 0 0,48' 37 В 0 1 0 0 371 С 0 0,36 0 0,64 Ь 0 0 0 0 А В 35. С Ь Е А '1 О 0,0875 0,1875 0,4375 В С D Е О О О О' 1 000 0,9125 000 0,8125 000 0,5625 000 43. а) 0,370; 0,297; 0,227; 0,132; 0,045. б) При больших значениях к все элементы матрицы Qk стремятся к нулю. 45. а) 75%. б) 12,5%. в) 7,5 месяцев. 47. а) Компания Л: 30%, компания В: 15%, компания С: 55%. б) Пять лет. 49. а) 91,52%. б) 4,96%. в) 6,32 дня. 51. а) 0,375. б) 1,75 раза. Упражнения для повторения АВ АВ 1. S\ = [0,32 0,68]; S2 = [0,325 0,672]. Вероятность оказаться в состоянии А после одного испытания равна 0,328, а вероятность оказаться в состоянии В после двух испытаний — 0,672. (7.1) 2. Состояние А является поглощающим; цепь является поглощающей. (7.2, 7.3) 3. Ни одного поглощающего состояния; цепь является регулярной. (7.2, 7.3)
844 4. 5. 6. 7. 8. 9. 10. И. 12. 13. 14. Ответы к упражнениям Ни одного поглощающего состояния; цепь не является ни регулярной, ни поглощающей. (7.2, 7.3) Состояния В и С являются поглощающими; цепь является поглощающей. (7.2, 7.3) Состояния АиВ являются поглощающими; цепь не является ни регулярной, ни поглощающей. (7.2, 7.3) А А Го В 0,1 с |_0 в с 1 о 0 0,9 1 0 Ни одного поглощающего состояния. Цепь не является ни регулярной, ни поглощающей. (7.1, 7.2, 7.3) А А ГО В 0,1 С [о В С 1 0 0,2 0,7 0 1 Состояние С является поглощающим. Цепь является поглощающей. (7.1, 7.2, 7.3) А А ГО В 0,1 С 0 В с 0 1 0,2 0,7 1 0 Ни одного поглощающего состояния. Цепь является регулярной. (7.1, 7.2, 7.3) А В А Г0,3 0,2 В 0 1 СО 0 D [О 0 С D 0 0,5' 0 0 0,2 0,8 0,3 0,7 Состояние С является поглощающим. Цепь не является ни регулярной, ни поглощающей. (7.1, 7.2, 7.3) А В А Г0,3 0,2 В 0,8 0 С 0,1 0,3 С 0,5 0,2 0,6 а) 3. б) 0,675. (7.1) S=[0,25 0,75]; Р = В 0,75 0,75 А S = [0,4 В 0,48 С А 0,12], Р= В С А В 0,4 0,48 0,4 0,48 0,4 0,48 С 0,12" 0,12 0,12 • (7.2)
Ответы к упражнениям 845 15. А А Г1 В О В О 1 с О’ о 16. С [0,75 0,25 0- Р(С —► А) = 0,75; Р(С —► В) = 0,25. Чтобы перейти из состояния С в поглощающее состояние, необходимо в среднем 2,5 испытания. (7.3) А "1 0 0,2 0,3 В о 1 0,8 0,7 С о о о о О О' О О О 21. А В С D Р(С -> А) = 0,2; Р(С — В) = 0,8; Р(Р> —» А) = 0,3; Р(£> -> В) = 0,7. Чтобы перейти из состояния С в поглощающее состояние, необходимо в среднем 2 испытания. Чтобы перейти из состояния D в поглощающее состояние, необходимо в среднем 3 испытания. (7.3) А 1 0 0,1 0,2 А А В С Ь 22. а) А б) 23. а) А [0,25 А [0,55 В о 1 0,1 0,2 В 0,4 В 0,4 В 0,75 0]. В с 0,45 0].(7.J) С О I О I 0,6 I 0,3 | С 0,5]. С 0,5]. С D О О 0,2 0,3 . (7.3). б) 24. Нет. Каждая строка матрицы Р должна содержать нули и единицу, но ни одна из четырех матриц, обладающих этим свойством, не является регулярной. (7.2) 25. Да. Например, матрица Р = О О 0,2 1 1 R б) В G 0 0 0,3 0,5 является регулярной. (7.2) 26. а) 0,2 0,5 0,6 Синий 0,2 0,3 0,2 0,1 Зеленый R "0,5 0,2 0,6 В 0,25 0,6 0,3 0,25' 0,2 0,1 в) Регулярная.
846 27. 29. 30. 31. 32. 33. 34. 35. 36. 37. Ответы к упражнениям R R Го,4 г) В 0,4 G 0,4 6 В 0,4 0,2’ 0,4 0,2 0,4 0,2 В долговременной перспективе красная урна будет выбрана в 40% случаев, синяя — в 40%, а зеленая — в 20%. (7.2). R 0,1 0,0 0 б) В 0,2 0,6 0,2 6 0,6 0,3 0,1 в) Поглощающая. R & В R Г1 0 О' г) В 1 0 0 . 6 10 0 Если выбрана красная урна, то в дальнейшем ни синяя, ни зеленая урны выбраны не будут. Для того чтобы извлечь красный шар из синей урны в среднем необходимы 3,67 испытания, а из зеленой урны — 2,33 испытания. (7.2) Такой цепи не существует. (7.2, 7.3) Такой цепи не существует. (7.2, 7.3) Такой цепи не существует. (7.2, 7.3) Матрицы S = [1 к матрице 0 0] и S' = [0 1 А '1 0 0,6 А Р=& С 0] являются стационарными по отношению В 0 1 0,3 С 0 0 0,1 • (7-3) А В р = аГ° 1 В 1 0 Такой цепи не существует. (7.2, 7.3) Такой цепи не существует. (7.2, 7.3) Предельной матрицы не существует. (7.2, 7.3) А В С b А 0,392 0,392 0,392 0,392 В 0,163 0,163 0,163 0,163 С 0,134 0,134 0,134 0,134 D 0,311 0,311 0,311 0,311 • (7.2)
Ответы к упражнениям 847 38. а) °’3 0,7 (Д X Y f X' У) 0,5 Y-Х о,5^—' X X' X Го,7 0,3 X' [о,5 0,5 • X X' в) [0,2 0,8]. X X' г) [0,54 0,46]. 54% потребителей купят в следующий раз товар компании X. д) [0,625 0,375]. е) 62,5%. (7.2) 39. а) Модель А: 24%, модель Б: 32%, модель В: 44%. б) Четыре года. (7.3) V V 40. a) = [0,806 0,194]; 51 = V [0,846 V 0,154]. ' Год Доля, % Вычисленная доля, % 1990 68,6 68,6 1995 81,0 80,6 2000 85,1 84,6 в) 86,6%. (7.2) 41. а) 64,75%. б) 15%. в) 8,75 года. Красный Розовый Белый Красный 0,5 0,25 0,25 42. Р = Розовый 0,2 0,6 0,2 (7.3) Белый 0,6 0,3 0,1 S S' S S' 43. a) Si = [0,254 0,746]; S2 = [0,246 0,754]. Год Доля, % Вычисленная доля, % 1985 30,1 30,1 1990 25,5 25,4 1995 24,7 24,6 в) 24,4%. (7.2)
Предметный указатель А Гистограмма, 640 Абсцисса, 19 Анализ безубыточности, 30 межотраслевых связей, 380 прибылей и убытков, 30 регрессионный, 124 Аннуитет, 244 будущая стоимость, 244 простой, 244 График вертикальное растяжение, 49 вертикальное сжатие, 49 вертикальный перенос, 47 горизонтальный перенос, 47 отражение относительно оси х, 49 параллельный перенос, 47 уравнения, 20 д Б Будущая стоимость, 168 Декартова система координат, 18 Диаграмма В Величина равновесная, 78; 293 случайная, 639 Вероятностное дерево, 614 Вероятность, 572 условная, 607 эмпирическая, 576 Вершина параболы, 95 Время роста, 232 Вычитание матриц, 335 Венна, 539 переходов, 670 Дополнение, 592 Доход, 30 Доходность годовая, 234 итоговая, 279 Е Единица, 353 3 Зависимость цены от спроса, 30; 77 Г Генеральная совокупность, 539 Задача двойственная, 477
Предметный указатель 849 линейного программирования, 425 максимизации, 423 стандартная, 443 минимизации, 482 модифицированная, 501 транспортная, 486 Закон больших чисел, 597 Значение оптимальное, 421 И Индикатор, 455 К Капитал, 214 Квадрант, 18 Координата, 19 Корень уравнения, 134 Л Линия постоянной прибыли, 423 Логарифм Бриггса, 189 десятичный, 189 Непера, 189 натуральный, 189 М М-метод, 499; 506 Математическое ожидание, 642 Матрица, 302 вероятностей, 683 верхняя треугольная, 346 вырожденная, 361 диагональная, 346 единичная, 354 инцидентности, 349 исходного состояния, 672 квадратная, 302 начального распределения, 671 начальных вероятностей, 671 нулевая, 335 обратная, 355 отрицательная, 335 переходов, 673 предельная, 710 приведенная, 315 размер, 302 размерность, 302 расшифровки, 362 сингулярная, 361 стационарная, 689 столбец, 302 строка, 302 технологическая, 382 шифрования, 361 элемент, 302 Метод Гаусса-Жордана, 311 исключения Гаусса-Жордана, 314; 317 наименьших квадратов, 211 обратной матрицы, 371 решения системы уравнений Гаусса, 289 графический, 284 подстановка, 287 Множество решений, 19 универсальное, 539 Модель двухотраслевая, 380 математическая, 422 Н Наклон прямой, 70 Начало координат, 19 Норма прибыли внутренняя, 281 О Область допустимых решений, 407 значений, 22 определения, 22 решений неограниченная, 411 ограниченная, 411 Опорный план, 457 Ордината, 19 Основная сумма, 168 Остаток, 210 Ось абсцисс, 18
Предметный указатель 850 вертикальная, 18 горизонтальная, 18 координатная, 18 ординат, 18 параболы, 96 Расходы, 30 фиксированные, 30; 77 Регрессия линейная, 125 Решение П Парабола, 91 Переменная базисная, 454 включаемая, 455 зависимая, 23 избыточная, 500 исключаемая, 455 искомая, 421 искусственная, 500 небазисная, 454 независимая, 23 Переменные издержки, 30; 77 Перестановка, 552 Погашение долга, 260 Подгонка кривой, 124 Полином, 131 регрессионный, 136 степень, 131 Полуплоскость, 402 верхняя, 402 левая, 402 нижняя, 402 правая, 402 Правило сложения множеств, 537 умножения, 541 Преобразование графика, 46 Прибыль, 30 Приращение, 69 Произведение матриц, 337 Процентная ставка, 168 Проценты, 214 простые, 171; 214 сложные, 168; 224 базисное допустимое, 453 оптимальное, 423; 425 С Симплекс-таблица исходная, 454 предварительная, 501 Система исходная, 452 линейных уравнений, 283 зависимая, 285 независимая, 285 совместная, 285 Событие, 568 составное, 568 элементарное, 568 События взаимно исключающие, 588 зависимые, 616 независимые, 616 Состояние поглощающее, 705 Сочетание, 555 Ставка процентная истинная, 234 эффективная, 234 Стоимость будущая, 215 номинальная, 215 текущая, 256 Столбец ведущий, 455 Стохастический процесс, 613; 670 Сумма матриц, 334 Р Размещение, 553 Разностное отношение, 29 Распределение вероятностей, 641 Т Таблица выигрышей, 645
851 Предметный указатель Точка однозначная, 182 равновесия, 78; 293 показательная, 157 угловая, 409 постоянная, 64 У Убытки, 30 Угловой коэффициент, 72 рациональная, 138 целевая, 425 Ц Умножение матрицы на число, 335 Цель, 421 Условие Цена неотрицательности, 422 равновесная, 78\ 293 Ф Цепь Маркова, 673 Факториал, 551 поглощающая, 707 Фонд регулярная, 689 амортизационный, 248 Ч Формула Частота события, 575 Байеса, 628 погашения долга, 260 Ш Функция, 22 Шанс, 595 квадратичная, 91 кусочно-определенная, 51 э линейная, 64 Эксперимент, 567 логарифмическая, 183 Элемент обратная, 181 ведущий, 457
КОНКРЕТНАЯ МАТЕМАТИКА 2-е издание Рональд Л. Грэхем, Дональд Э. Кнут, Орен Паташник КОНКРЕТНАЯ МАТЕМАТИКА МАТЕMAT И Ч ЕС К И Е О СIК)ВЫ ИНФОРМАГИКИ www.williamspublishing.com ISBN 978-5-8459-1923-6 В основу данной книги положен одноименный курс лекций Станфордского университета. Название “конкретная математика” происходит от слов “КОНтинуальная” и “дисКРЕТНАЯ” математика. Назначение данной книги — обеспечить читателя техникой оперирования с дискретными объектами, что совершенно необходимо для математиков, работающих в области информатики. Книга ориентирована в первую очередь на практиков (хотя и теоретики найдут в ней много полезного), и изобилует массой конкретных примеров и упражнений. Конкретность изложения абстрактного материала — еще одно пояснение названия книги. Широта охвата столь различных тем в одной книге могла бы вызвать подозрения в некоторой легковесности, если бы не имена ее авторов — известных американских математиков. Тем не менее слово “легкий” к книге вполне применимо, так как стиль изложения достаточно далек от сухого академизма. Как признаются сами авторы, они считают математику развлечением, и они сделали все, чтобы читатели книги получили от ее прочтения не только знания, но и удовольствие. в продаже
ГЛУБОКОЕ ОБУЧЕНИЕ И TENSORFLOW ДЛЯ ПРОФЕССИОНАЛОВ Сантану Паттанаяк Глубокое обучение и TensorFlow для профессионалов Математический подход к построению систем искусе iвенного ите/шекта на Python www.williamspublishing.com Данная книга представляет собой углубленное практическое руководство, которое позволит читателям освоить методы глубокого обучения на уровне, достаточном для развертывания готовых решений. Прочитав книгу, вы сможете быстро приступить к работе с библиотекой TensorFlow и заняться оптимизацией архитектур глубокого обучения. Весь программный код доступен в виде блокнотов iPytnon и сценариев, позволяющих с легкостью воспроизводить примеры и экспериментировать с ними. Благодаря этой книге вы: • овладеете полным стеком технологий глубокого обучения с использованием TensorFlow и получите необходимую для этого математическую подготовку; • научитесь развертывать сложные приложения глубокого обучения в производственной среде с помощью TensorFlow; • сможете проводить исследования в области глубокого обучения и выполнять самостоятельные эксперименты в TensorFlow. ISBN: 978-5-907144-25-5 в продаже
Дискретная математика и комбинаторика Джеймс Андерсон , Дискретная | МАТЕМАТИКА И КОМБИНАТОРИКА * . * а ■ г М_ ДЖЕЙ МС АНДЕРСОН www.dialektika.com Данная книга содержит доступное для начинающего читателя и достаточно полное изложение основных разделов дискретной математики. Особое внимание в ней уделено математической логике. Автор считает это важным как для развития техники доказательств, так и в более широком аспекте развития логического мышления. Кроме оснований математической логики, в книге изложены основы теории множеств, теории графов, теории алгоритмов, комбинаторики, элементы теории вероятностей. Она содержит обширные сведения по алгебре и теории чисел. Книга планировалась автором как основа односеместрового или двусеместрового курса по дискретной математике. Первые восемь глав закладывают основы знаний по предмету и образуют относительно самостоятельную часть, поэтому преподаватель имеет определенную свободу в выборе материала остальных глав. Чтение книги требует некоторой математической культуры, хотя для изучения основных глав достаточно будет знаний по математике в объеме средней школы. ISBN 978-5-907144-07-1 в продаже
ИССЛЕДОВАНИЕ ОПЕРАЦИИ ДЕСЯТОЕ ИЗДАНИЕ ХзмдиА. Таха www.dialektika.com Исследование операций ориентировано на решение практических задач, которые можно описать с помощью математической модели. В книге представлены основные разделы теории исследования операций: математическое программирование (линейное и нелинейное, детерминированное, эвристическое и вероятностное динамическое), теория принятия решений и теория игр, теория управления запасами, теория массового обслуживания, имитационное моделирование. Книга может служить учебным пособием по теории и практическому применению методов исследования операций. Каждая тема начинается с вводного материала, доступного студентам первых курсов, далее уровень изложения постепенно повышается и рассчитан уже на студентов старших курсов и аспирантов. Написанная без излишнего академизма (но достаточно строго) книга будет полезна широкому кругу читателей: студентам, аспирантам и преподавателям высших учебных заведений, экономистам, инженерам, разработчикам программного обеспечения и т.д. ISBN 978-5-6040043-2-6 в продаже
МАТЕМАТИКА ДЛЯ ЧАЙНИКОВ 2-Е ИЗДАНИЕ Марк Зегарелли ВЕДЬ ЭТО ТАК ПРОСТО! Математика чайников Освойте обыкновенные дроби, десятичные дроби и проценты Изучите основные арифметические операции Марк Зегарелли текстовые мдачи www.dialektika.com Многие школьники на этапе перехода от арифметики к алгебре почему-то начинают испытывать страх перед математикой. Эта книга поможет вам разобраться с любыми математическими трудностями. Здесь вы найдете все необходимое для освоения школьного курса математики и перехода к изучению алгебры и геометрии. Начав со знакомства с числами, дробями и процентами, вы затем освоите более сложные темы, такие как единицы измерения, множества и алгебраические уравнения. Основные темы книги: • множители, делители и кратные; • обыкновенные дроби, десятичные дроби и проценты; • основные геометрические фигуры и тела; • диаграммы и графики; • экспоненциальная запись числа, единицы измерения, статистика и множества; • введение в алгебру. ISBN:978-5-907144-62-0 в продаже
МАТЕМАТИКА СБОРНИК ЗАДАЧ ДЛЯ ЧАЙНИКОВ З-Е ИЗДАНИЕ Марк Зегарелли www.dialektika.com Этот сборник задач окажет помощь в усвоении основных математических понятий, проработке примеров практических задач и поэтапном анализе их решений. Пользуясь этим пособием, читатель может на практике освоить простые и десятичные дроби, научиться выполнять основные арифметические операции над числами, решать алгебраические уравнения, вычислять площади и объемы геометрических форм и тел, строить графики в двумерной координатной плоскости и ознакомиться с различными системами счисления. Книга адресована всем желающим изучить математику и поможет им преодолеть невольный страх перед этой дисциплиной. ISBN 978-5-907144-11-8 в продаже
ВВЕДЕНИЕ В ГЛУБОКОЕ ОБУЧЕНИЕ Евгений Черняк ВВЕДЕНИЕ в ГЛУБОКОЕ ОБУЧЕНИЕ ЕВГЕНИЙ ЧЕРНЯК www.dialektika.com Автор книги — давний исследователь искусственного интеллекта, специализирующийся на обработке естественного языка, революцию в котором сделало глубокое обучение. К сожалению, ему потребовалось много времени, чтобы это понять. Можно сказать, что нейронные сети угрожают революцией уже третий раз, а отнюдь не первый. Тем не менее автор внезапно оказался далеко позади и изо всех сил пытался наверстать упущенное. Именно поэтому он сделал то, что сделал бы на его месте любой уважающий себя профессор: запланировал преподавание материала и начал ускоренный курс, просматривая веб-страницы. Этим объясняется несколько выдающихся особенностей этой книги. Во-первых, краткость. Во-вторых, она сильно зависит от проекта. Автор считает, что материал по информатике лучше изучать при написании программ, поэтому книга во многом отражает его привычки в преподавании. Эта книга, в первую очередь, задумана как учебник для курса по глубокому обучению. Курс, который автор преподает в Брауне, предназначен как для выпускников, так и для остальных студентов, и охватывает весь материал. ISBN 978-5-907203-10-5 в продаже
нейронные сети и глубокое ОБУЧЕНИЕ УЧЕБНЫЙ КУРС Чару Аггарвал Чару Ai гарвал Нейронные Сети и глубокое обучение Учебный курс L адемми ‘И Springer www.dialektika.com В книге рассматриваются как классические, так и современные модели глубокого обучения. В первых двух главах основной упор сделан на понимании взаимосвязи традиционного машинного обучения и нейронных сетей. Главы 3 и 4 посвящены подробному обсуждению процессов тренировки и регуляризации нейронных сетей. В главах 5 и 6 рассмотрены сети радиально-базисных функций (RBF) и ограниченные машины Больцмана. В главах 7 и 8 обсуждаются рекуррентные и сверточные нейронные сети. Главы 9 и 10 посвящены более сложным темам, таким как глубокое обучение с подкреплением, нейронные машины Тьюринга, самоорганизующиеся карты Кохонена и генеративно-состязательные сети. Книга предназначена для студентов старших курсов, исследователей и специалистов-практиков. ISBN:978-5-907203-01-3 в продаже
НАУКА О ДАННЫХ УЧЕБНЫЙ КУРС Стивен С. Скиена |СЕРИЯ КНИГ О КОМПЬЮТЕРНЫХ НАУКАХ Наука о данных Стивен С. Скиена £1 Springer www.williamspublishing.com Для того чтобы понять мир, необходимо собрать и проанализировать данные о нем. Объединение последних технологических тенденций предоставляет новые возможности для применения анализа данных к более сложным задачам, чем когда-либо прежде. Результатом всего этого стало рождение науки о данных — новой области, посвященной максимизации значения обширных коллекций информации. Как дисциплина, наука о данных находится где-то на пересечении статистики, информатики и машинного обучения, но стоит она отдельно, как самостоятельный персонаж. Эта книга служит введением в науку о данных, сосредоточиваясь на навыках и принципах, необходимых для построения систем, предназначенных для анализа и интерпретации данных. Профессиональная практика автора как исследователя и преподавателя убедила его в том, что одной из главных сложностей науки о данных является то, что она значительно сложней, чем выглядит. ISBN 978-5-907144-74-3 в продаже
ГЛУБОКОЕ ОБУЧЕНИЕ ГОТОВЫЕ РЕШЕНИЯ Давид Осинга www.williamspublishing.com Благодаря готовым примерам, приведенным в книге, вы научитесь решать задачи, связанные с классификацией и генерированием текста, изображений и музыки. В каждой главе описывается несколько решений, объединяемых в единый проект, например приложение, реализующее тренировку музыкальной рекомендательной системы. Также имеется глава с описанием методик, которые в случае необходимости помогут выполнить отладку нейронной сети. Основные темы книги: ■ использование векторных представлений слов для вычисления схожести текстов; ■ построение рекомендательной системы фильмов на основе ссылок в Википедии; ■ визуализация внутренних состояний нейронной сети; ■ создание модели, рекомендующей эмодзи для фрагментов текста; ■ повторное использование предварительно обученных сетей для создания службы обратного поиска изображений; ■ генерирование пиктограмм с помощью генеративносостязательных сетей (GAN), автокодировщиков и рекуррентных сетей (RNN); ■ распознавание музыкальных жанров и индексирование коллекций песен. ISBN: 978-5-907144-50-7 в продаже
ПРИКЛАДНОЕ МАШИННОЕ ОБУЧЕНИЕ С ПОМОЩЬЮ SCIKIT-LEARN И TENSORFLOW Орельен Жерон O’REILLY’ Прикладное машинное обучение с помощью Scikit-Learn и TensorFlow КОНЦЕПЦИИ, ИНСТРУМЕНТЫ И ТЕХНИКИ Л (l ДЛЯ СОЗДАНИЯ ИНТЕЛЛЕКТУАЛЬНЫХ i fl Л систем ^7 www.dialektika.com Благодаря серии недавних достижений глубокое обучение значительно усилило всю область машинного обучения. В наше время даже программисты, почти ничего не знающие об этой технологии, могут использовать простые и эффективные инструменты для реализации программ, которые способны обучаться на основе данных. В настоящем практическом руководстве показано, что и как следует делать. За счет применения конкретных примеров, минимума теории и двух фреймворков Python производственного уровня — Scikit-Learn и TensorFlow — автор книги Орельен Жерон поможет вам получить интуитивное представление о концепциях и инструментах, предназначенных для построения интеллектуальных систем. Вы узнаете о ряде приемов, начав с простой линейной регрессии и постепенно добравшись до глубоких нейронных сетей. Учитывая наличие в каждой главе упражнений, призванных закрепить то, чему вы научились, для начала работы нужен лишь опыт программирования. ISBN 978-5-9500296-2-2 в продаже
PYTHON И МАШИННОЕ ОБУЧЕНИЕ МАШИННОЕ И ГЛУБОКОЕ ОБУЧЕНИЕ С ИСПОЛЬЗОВАНИЕМ PYTHON, SCIKIT-LEARN И TENSORFLOW, 2-Е ИЗДАНИЕ Себастьян Рашка и Вахид Мирджалили MHI ни. )К! HI I'loi; ПОЛНОЦВЕТНОЕ ИЗДАНИЕ Себастьян Рашка Вахид Мирджалили Python и машинное обучение M.iuihi и к>е и I IV . ' ■ ■ • | " с /споли-ion.! I ■■ I . ■ ■ I • ■ . . . , I . , ОЛ Packt> www.dialektika.com Машинное обучение поглощает мир программного обеспечения, и теперь глубокое обучение расширяет машинное обучение. Освойте и работайте с передовыми технологиями машинного обучения, нейронных сетей и глубокого обучения с помощью 2-го издания бестселлера Себастьяна Рашки. Будучи основательно обновленной с учетом самых последних библиотек Python с открытым кодом, эта книга предлагает практические знания и приемы, которые необходимы для создания и содействия машинному обучению, глубокому обучению и современному анализу данных. Если вы читали 1-е издание книги, то вам доставит удовольствие найти новый баланс классических идей и современных знаний в машинном обучении. Каждая глава была серьезно обновлена, и появились новые главы по ключевым технологиям. У вас будет возможность изучить и поработать с TensorFlow более вдумчиво, нежели ранее, а также получить важнейший охват библиотеки для нейронных сетей Keras наряду с самыми свежими обновлениями библиотеки scikit-leam. ISBN 978-5-907114-52-4 в продаже
АНАЛИТИЧЕСКОЕ ПРОГНОЗИРОВАНИЕ ДЛЯ ЧАЙНИКОВ 2-Е ИЗДАНИЕ Анассе Бари Мохамед Чаучи Томми Юнг Аналитическое прогнозирование www.dialektika.com Современный бизнес опирается на эффективное использование данных для прогнозирования тенденций и продаж. Основным инструментом решения этих задач является аналитическое прогнозирование. В книге просто и понятно показано, как его применять. Авторы учат подготавливать и обрабатывать свои данные, ставить цели, создавать прогностические модели, привлекать к работе заинтересованные стороны и т.д. В книге описываются этапы создания проектов по аналитическому прогнозированию, показано, как определять типы данных, даются советы по моделированию, описывается работа с алгоритмами, излагаются основы классификации, кластеризации и машинного обучения. Книга будет полезной широкому кругу читателей, интересующихся машинным обучение и аналитическим прогнозированием. ISBN 978-5-907203-07-5 в продаже